accounting for lease mdash lessee and lessor astle manufacturing company manufacture 692687

Accounting for Lease—Lessee and Lessor

Astle Manufacturing Company manufactures and leases a variety of items. On January 2, 2008, Astle leased a piece of equipment to Haws Industries Co. The lease is for six years for an annual amount of $33,500, payable in advance. The lease payment includes executory costs of $1,500 per year. The equipment has an estimated useful life of nine years, and it was manufactured by Astle at a cost of $120,000. It is estimated that the equipment will have a residual value of $60,000 at the end of the 6 year lease term. There is no provision for purchase or renewal by Haws at the end of the lease term. However, a third party has guaranteed the residual value of $60,000. The equipment has a fair market value at the lease inception of $187,176. The implicit rate of interest in the contract is 10%, the same rate at which Haws can borrow money at its bank. All lease payments after the first one are made on December 31 of each year. Both companies use the straight line method of depreciation.

Instructions:

1. Give all the entries relating to the lease on the books of the lessor and lessee for 2008.

2. Show how the lease would appear on the balance sheet of Astle Manufacturing Company and Haws Industries Co. (if applicable) as of December 31, 2008.

3. Assume that Astle sold the equipment at the end of the 6 year lease for $85,000. Give the entry to record the sale, assuming that all lease entries have been properly made.

cash flow treatment of capital lease mdash lessor the following information relates 692688

Cash Flow Treatment of Capital Lease—Lessor

The following information relates to a capital lease between Bradford Electric Co. (lessee) and Widstoe Manufacturing Inc. (lessor). The lease term began on January 1, 2008.Widstoe recorded the lease as a sale and made the following entries related to the lease during 2008. Assume this was the only lease Widstoe had during the year.

Jan 1

Deferred Initial Direct Costs                                      

6,000

 

 

Cash                                                     

 

6,000

1

Lease Payments Receivable                                       

88,000

 

 

Sales                                                     

 

88,000

1

Cost of Goods Sold                                            

70,000

 

 

Inventory                                                  

 

64,000

 

Deferred Initial Direct Costs                                   

 

6,000

1

Cash                                                       

11,132

 

 

Lease Payments Receivable                                     

 

11,132

Dec 31

Cash                                                       

11,132

 

 

Lease Payments Receivable                                     

 

11,132

31

Lease Payments Receivable                                       

6,242

 

 

Interest Revenue                                            

 

6,242

Instructions:

1. Prepare the partial Operating Activities section of the statement of cash flows for 2008 for Widstoe Manufacturing Inc. under the indirect method. Widstoe reported net income of $148,504 inclusive of the lease revenue in the preceding entries.

2. Prepare the partial Operating Activities section of the statement of cash flows for 2008 for Widstoe Manufacturing Inc. under the direct method. Assume that cash provided by operating activities exclusive of the lease transactions is $124,262.

disclosure requirements mdash operating leases jaquar mining and manufacturing compa 692689

Disclosure Requirements—Operating Leases

Jaquar Mining and Manufacturing Company leases from Emory Leasing Company three machines under the following terms.

• Machine 1: Lease period—10 years, beginning April 1, 2002; lease payment—$18,000 per year, payable in advance.

• Machine 2: Lease period—10 years, beginning July 1, 2006; lease payment—$30,000 per year, payable in advance.

• Machine 3:Lease period—15 years, beginning January 1,2007;lease payment—$12,500 per year, payable in advance. All of the leases are classified as operating leases.

Instructions: Prepare the note to the 2008 financial statements that would be required to disclose the lease commitments of Jaquar Mining and Manufacturing Company. Jaquar uses the calendar year as its accounting period.

capitalizing the value of operating leases the following information comes from the 692690

Capitalizing the Value of Operating Leases

The following information comes from the financial statements of Travis Campbell Company.

 

Total liabilities                                                               

$100,000

Total stockholders’ equity                                                       

80,000

Property, plant, and equipment                                                    

110,000

Sales                                                                      

500,000

In addition, Travis Campbell has a large number of operating leases. The payments on these operating leases total $30,000 per year for the next 10 years. All of these lease payments occur at the end of the year. The incremental borrowing rate of Travis Campbell Company is 10%. This is also the rate implicit in all of the leases that Travis Campbell signs.

Instructions:

1. Compute the following ratio values:

(a) Debt ratio (total liabilities/total assets).

(b) Debt ratio, assuming that Travis Campbell’s operating leases are accounted for as capital leases.

(c) Asset turnover (sales/total assets).

(d) Asset turnover, assuming that Travis Campbell’s operating leases are accounted for as capital leases.

2. Briefly describe how the accounting for assets used under operating leases distorts the values of financial ratios.

sample cpa exam questions 1 in a sale leaseback transaction a gain resulting from th 692691

Sample CPA Exam Questions

1. In a sale leaseback transaction, a gain resulting from the sale should be deferred at the time of the sale leaseback and subsequently amortized when:

I. The seller lessee has transferred substantially all the risks of ownership.

II. The seller lessee retains the right to substantially all of the remaining use of the property.

(a) I only.

(b) II only.

(c) Both I and II.

(d) Neither I nor II.

2. At the inception of a capital lease, the guaranteed residual value should be:

(a) Included as part of minimum lease payments at present value.

(b) Included as part of minimum lease payments at future value.

(c) Included as part of minimum lease payments only to the extent that guaranteed residual value is expected to exceed estimated residual value.

(d) Excluded from minimum lease payments.

sale leaseback of a building on january 3 2008 aspen inc sold a building with a book 692692

Sale Leaseback of a Building

On January 3, 2008,Aspen Inc. sold a building with a book value of $2,100,000 to Spruce Industries for $2,025,040. Aspen immediately entered into a leasing agreement wherein Aspen would lease the building back for an annual payment of $320,000. The term of the lease is 10 years, the expected remaining useful life of the building. The first annual lease payment is to be made immediately, and future payments will be made on January 1 of each succeeding year. Spruce’s implicit interest rate is 12%.

Instructions:

1. Prepare the journal entries that both Aspen and Spruce should make on January 3, 2008, relating to this sale leaseback transaction.

2. Prepare the journal entries that both parties should make at the end of 2008 to accrue interest and to amortize the leased building. (Assume a salvage value of $0 and use of the straight line method.)

how should the lease be recorded louise corporation entered into a leasing arrangeme 692693

How Should the Lease Be Recorded?

Louise Corporation entered into a leasing arrangement with Wilder Leasing Corporation for a certain machine. Wilder’s primary business is leasing; it is not a manufacturer or dealer.

Louise will lease the machine for a period of three years, which is 50% of the machine’s economic life. Wilder will take possession of the machine at the end of the initial 3 year lease. Louise does not guarantee any residual value for the machine. Louise’s incremental borrowing rate is 10%, and the implicit rate in the lease is 81⁄2%. Louise has no way of knowing the implicit rate used by Wilder. Using either rate, the present value of the minimum lease payments is between 90% and 100% of the fair value of the machine at the date of the lease agreement. Louise has agreed to pay all executory costs directly, and no allowance for these costs is included in the lease payments. Wilder is reasonably certain that Louise will pay all lease payments, and because Louise has agreed to pay all executory costs, there are no important uncertainties regarding costs to be incurred by Wilder.

1. With respect to Louise (the lessee), answer the following.

(a) What type of lease has been entered into? Explain the reason for your answer.

(b) How should Louise compute the appropriate amount to be recorded for the lease or asset acquired?

(c) What accounts will be created or affected by this transaction, and how will the lease or asset and other costs related to the transaction be matched with earnings?

(d) What disclosures must Louise make regarding this lease or asset?

2. With respect to Wilder (the lessor), answer the following:

(a) What type of leasing arrangement has been entered into? Explain the reason for your answer.

(b) How should this lease be recorded by Wilder, and how are the appropriate amounts determined?

(c) How should Wilder determine the appropriate amount of earnings to be recognized from each lease payment?

(d) What disclosures must Wilder make regarding this lease?

should we buy or lease meeker machine and die company has learned that a sophisticat 692694

Should We Buy or Lease?

Meeker Machine and Die Company has learned that a sophisticated piece of computer operated machinery is available to either buy or rent. The machinery will result in three employees being replaced, and quality of the output has been tested to be superior in every demonstration. There is no doubt that this machinery represents the latest in technology; however, new inventions and research make it difficult to estimate when the machinery will be made obsolete by new technology. The physical life expectancy of the machine is 10 years; however, the estimated economic life is between two and five years. Meeker has a debt to equity ratio of 0.75. If the machine is purchased and the minimum down payment is made, the outstanding loan balance on the machine will cause the debtto equity ratio to increase to 1.1. The monthly payments if the machine is purchased are 20% lower than the lease payments if it is leased. The incremental borrowing rate for Meeker is 11%. The rate implicit in the lease is 12%. What factors should Meeker consider in deciding how to finance the acquisition of the machine?

how should the leases be classified and accounted for on january 1 toronto company a 692695

How Should the Leases Be Classified and Accounted for?

On January 1,Toronto Company, a lessee, entered into three no cancelable leases for new equipment, lease J, lease K, and lease L. None of the three leases transfers ownership of the equipment to Toronto at the end of the lease term. For each of the three leases, the present value at the beginning of the lease term of the minimum lease payments is 75% of the fair value of the equipment to the lessor at the inception of the lease. This excludes that portion of the payments representing executory costs, such as insurance, maintenance, and taxes to be paid by the lessor, including any profit thereon. The following information is peculiar to each lease:

(a) Lease J does not contain a bargain purchase option; the lease term is equal to 80% of the estimated economic life of the equipment.

(b) Lease K contains a bargain purchase option; the lease term is equal to 50% of the estimated economic life of the equipment.

(c) Lease L does not contain a bargain purchase option; the lease term is equal to 50% of the estimated economic life of the equipment.

1. How should Toronto Company classify each of the three leases and why? Discuss the rationale for your answer.

2. What amount, if any, should Toronto record as a liability at the inception of the lease for each of the three leases?

3. Assuming that the minimum lease payments are made on a straight line basis, how should Toronto record each minimum lease payment for each of the three leases?

more leases mean lower profits digital x ray inc has introduced a new line of equipm 692696

More Leases Mean Lower Profits

Digital X Ray, Inc., has introduced a new line of equipment that may revolutionize the medical profession. Because of the new technology involved, potential users of the equipment are reluctant to purchase the equipment, but they are willing to enter into a lease arrangement as long as they can classify the lease as an operating lease. The new equipment will replace equipment that Digital X Ray, Inc., has been selling in the past. It is estimated that a 25% loss of actual equipment sales will occur as a result of the leasing policy for the new equipment. Management must decide how to structure the leases so that they can treat them as operating leases. Some members of management want to structure the leases so that Digital X Ray, Inc., as lessor, can classify the lease as a sales type lease and thus avoid a further reduction of income. Others believe that they should treat the leases as operating leases and minimize the income tax liability in the short term. They are uncertain, however, as to how the financial statements would be affected under these two different approaches. They also are uncertain as to how leases could be structured to permit the lessee to treat the lease as an operating lease and the lessor to treat it as a sales type lease. You are asked to respond to their questions.

structuring a lease to avoid liability recognition johnson pharmaceuticals needs cas 692697

Structuring a Lease to Avoid Liability Recognition

Johnson Pharmaceuticals needs cash. One option being considered by the board of directors is to sell the plant facilities to a group of venture capitalists and then lease the facilities back for a long term period with the option of repurchasing the plant facilities at the end of the lease. The chairman has commented that this option will provide Johnson Pharmaceuticals the needed cash but will result in a large lease liability on the balance sheet. As the chief financial officer, you comment that if the company carefully structures the terms of the lease agreement, it may be able to avoid recognizing the lease liability. The chairman has asked you to prepare a memo discussing the specific ways in which a lease agreement can be structured to avoid recognizing the liability on the balance sheet.

recognizing a profit from leasing in june 1988 british amp commonwealth plc b amp c 692698

Recognizing a Profit from Leasing

In June 1988, British & Commonwealth PLC (B&C) acquired Atlantic Computers, the world’s third largest computer leasing company. In April 1990, B&C placed Atlantic Computers into administrative receivership and wrote off its $900 million investment in the company. The reason for the write off? Atlantic’s method of accounting for leases. Atlantic had developed what was called a “flexlease,” which allowed customers to upgrade their computers at specified points during the lease period. The flexlease involved two separate contracts, one with a financing institution and the other with Atlantic. When customers elected to exercise their flex options, Atlantic would take back the equipment, pay off the remainder of the contract to the lender, and sell the equipment in the used computer market. Even though the original lease arrangement did not meet the criteria for a sales type lease, Atlantic was estimating the profits to be made from the sale of those computers that would be returned, assuming customers exercised their flex options, and was recognizing these sales profits when the original lease contract was signed. 1. Is there anything wrong with Atlantic’s method of accounting for the profits to be made on the “flexleases”?

2. When would be the most appropriate time for Atlantic to recognize profits from the sale of a computer that was returned under a flex option?

3. Why would British & Commonwealth PLC get rid of Atlantic rather than simply change the accounting practice?

 

sales type lease with an unguaranteed residual value refer to practice 15 16 assume 692648

Sales Type Lease with an Unguaranteed Residual Value

Refer to Practice 15 16. Assume the same facts except that the $500 bargain purchase option is instead a $500 unguaranteed residual value. Make the journal entries necessary on the books of the lessor company to record (1) the lease signing (including receipt of the first lease payment) and (2) the recognition of interest revenue at the end of the first year.

Practice 15 16

Sales Type Lease with a Bargain Purchase Option

On January 1, the lessor company purchased a piece of equipment for $6,000 as inventory. The lessor company immediately leased the equipment under a sales type lease agreement. The lease calls for the lessor company to receive five annual lease payments of $2,500 per year, to be received at the beginning of the year. In addition to the five annual payments of $2,500 at the beginning of each year, the lessor is to receive a bargain purchase option amount of $500 at the end of five years. The interest rate implicit in the lease is 12%. Make the journal entries necessary on the books of the lessor company to record (1) the lease signing (including receipt of the first lease payment) and (2) the recognition of interest revenue at the end of the first year.

third party guarantees of residual value on january 1 the lessor company purchased a 692649

Third Party Guarantees of Residual Value

On January 1, the lessor company purchased a piece of equipment for $6,000 as inventory. The lessor company immediately leased the equipment under a sales type lease agreement. The lease calls for the lessor company to receive five annual lease payments of $2,500 per year, to be received at the beginning of the year. In addition to the five annual payments of $2,500 at the beginning of each year, the lessor is to receive a guaranteed residual value of $3,000 at the end of five years. The fair value on the date of the lease signing is equal to the present value of the lessor’s minimum payments; the interest rate implicit in the lease is 10%. The equipment has a useful life of eight years, there is no bargain purchase option, and the title does not transfer at the end of the lease term. Also, the residual value is guaranteed by a third party insurance company, not by the lessee company. Make the journal entries necessary to record the lease signing, including the first lease payment, (1) on the books of the lessor company and (2) on the books of the lessee company.

selling a leased asset during the lease term mdash lessor on december 31 of year 1 t 692650

Selling a Leased Asset During the Lease Term—Lessor

On December 31 of Year 1, the company, a lessor, sold some machinery that it had been leasing under a direct financing lease arrangement. On January 1 of Year 1 (after receipt of the lease payment for the year), the following account balances were associated with the lease:

Gross Lease Payments Receivable

$117,000

Unearned Interest Revenue

20000

Present Value of Lease Payments Receivable

$97,000

The interest rate implicit in the lease is 10%. The leased machinery is sold for $65,000 cash. Make the journal entry or entries necessary on the books of the lessor to record this sale.

leases on a statement of cash flows mdash lessor on january 1 the lessor company pur 692651

Leases on a Statement of Cash Flows—Lessor

On January 1, the lessor company purchased some equipment (for cash) that the company then immediately leased. The lease contract calls for the receipt of $3,000 payments at the end of each year for 10 years. The residual value of the equipment at the end of the 10 year lease term is expected to be $4,000. The rate implicit in the lease is 12%. Except for lease related items, there were no changes in current operating assets or liabilities during the year; no purchases or sales of property, plant, or equipment; and no dividends paid, stock issued, or loans obtained or repaid. The equipment has a total useful life of 15 years with no salvage value. Prepare a complete statement of cash flows for the lessor using the indirect method of reporting operating cash flow assuming that the lease is accounted for as (1) an operating lease (net income was $20,000) and (2) a direct financing lease (net income was $20,405).

criteria for capitalizing leases atwater manufacturing co leases its equipment from 692654

Criteria for Capitalizing Leases

Atwater Manufacturing Co. leases its equipment from Westside Leasing Company. In each of the following cases, assuming none of the other criteria for capitalizing leases are met, determine whether the lease would be a capital lease or an operating lease under FASB

Statement No. 13.Your decision is to be based only on the terms presented, considering each case independently of the others.

(a) At the end of the lease term, the market value of the equipment is expected to be $20,000. Atwater has the option of purchasing it for $5,000.

(b) The fair market value of the equipment is $75,000. The present value of the lease payments is $67,000 (excluding any executory costs).

(c) Ownership of the property automatically passes to Atwater at the end of the lease term.

(d) The economic life of the equipment is 12 years. The lease term is eight years.

(e) The lease requires payments of $9,000 per year in advance plus executory costs of $500 per year. The lease period is three years, and Atwater’s incremental borrowing rate is 12%. The fair market value of the equipment is $28,000.

(f) The lease requires payments of $6,000 per year in advance, which includes executor costs of $500 per year. The lease period is three years, and Atwater’s incremental borrowing rate is 10%. The fair market value of the equipment is $16,650.

entries for operating lease mdash lessee alma inc leases some of the equipment it us 692656

Entries for Operating Lease—Lessee

Alma Inc. leases some of the equipment it uses. The lease term is five years, and the lease payments are to be made in advance as shown in the following schedule.

January 1, 2008                                                              

$ 50,000

January 1, 2009                                                              

50,000

January 1, 2010                                                              

70,000

January 1, 2011                                                              

90,000

January 1, 2012                                                              

120,000 

Total                                                                   

$380,000  

The equipment is to be used evenly over the 5 year period. For each of the five years, give the entry that should be made at the time the lease payment is made to allocate the proper share of rent expense to each period. The lease is classified as an operating lease by Alma Inc.

entries for lease mdash lessee bingham smelting company entered into a 15 year no ca 692657

Entries for Lease—Lessee

Bingham Smelting Company entered into a 15 year no cancelable lease beginning January 1, 2008, for equipment to use in its smelting operations. The term of the lease is the same as the expected economic life of the equipment. Bingham uses straight line depreciation for all plant assets. The provisions of the lease call for annual payments of $290,000 in advance plus $20,000 per year to cover executory costs, such as taxes and insurance, for the 15 year period of the lease. At the end of the 15 years, the equipment is expected to be scrapped. The incremental borrowing rate of Bingham is 10%. The lessor’s computed implicit interest rate is unknown to Bingham. Record the lease on the books of Bingham and give all the entries necessary to record the lease for its first year plus the entry to record the second lease payment on December 31, 2008. (Round to the nearest dollar.)

entries for lease mdash lessee on january 2 2008 jacques company entered into a no c 692658

Entries for Lease—Lessee

On January 2, 2008, Jacques Company entered into a no cancelable lease for new equipment. The equipment was built to Jacques Company’s specifications and is in an area in which rental to another lessee would be difficult. Rental payments are $300,000 a year for 10 years, payable in advance. The equipment has an estimated economic life of 20 years. The taxes, maintenance, and insurance are to be paid directly by Jacques Company, and the title to the equipment is to be transferred to Jacques at the end of the lease term. Assume the cost of borrowing funds for this type of an asset by Jacques Company is 12%.

1. Give the entry on Jacques’ books that should be made at the inception of the lease.

2. Give the entries for 2008 and 2009, assuming the second payment and subsequent payments are made on December 31 and assuming double declining balance amortization.

schedule of lease payments wallin construction co is leasing equipment from astro in 692659

Schedule of Lease Payments

Wallin Construction Co. is leasing equipment from Astro Inc. The lease calls for payments of $75,000 a year plus $5,000 a year executory costs for five years. The first payment is due on January 1, 2008, when the lease is signed, with the other four payments coming due on December 31 of each year. Wallin has also been given the option of purchasing the equipment at the end of the lease at a bargain price of $110,000.Wallin has an incremental borrowing rate of 8%, the same as the implicit interest rate of Astro. Wallin has hired you as an accountant and asks you to prepare a schedule showing how the lease payments will be split between principal and interest and the outstanding lease liability balance over the life of the lease.

entry for purchase by lessee cordon enterprise company leases many of its assets and 692660

Entry for Purchase by Lessee

Cordon Enterprise Company leases many of its assets and capitalizes most of the leased assets. At December 31, the company had the following balances on its books in relation to a piece of specialized equipment:

Leased Equipment                                                            

$80,000

Accumulated Amortization—Leased Equipment                                       

49,300

Obligations under Capital Leases                                                 

26,000

Amortization has been recorded up to the end of the year, and no accrued interest is involved. At December 31, Cordon decided to purchase the equipment for $32,000 and paid cash to complete the purchase. Give the entry required on Cordon’s books to record the purchase.

 

direct financing lease with residual value massachusetts casualty insurance company 692663

Direct Financing Lease with Residual Value

Massachusetts Casualty Insurance Company decides to enter the leasing business. It acquires a specialized packaging machine for $300,000 cash and leases it for a period of six years, after which the machine is to be returned to the insurance company for disposition. The expected unguaranteed residual value of the machine is $20,000. The lease terms are arranged so that a return of 12% is earned by the insurance company.

1. Calculate the annual lease payment, payable in advance, required to yield the desired return.

2. Prepare entries for the lessor for the first year of the lease, assuming the machine is acquired and the lease is recorded on January 1, 2008. The first lease payment is made on January 1, 2008, and subsequent payments are made each December 31.

3. Assuming that the packaging machine is sold by Massachusetts to the lessee at the end of the six years for $29,000, give the required entry to record the sale.

capital lease with guaranteed residual value mdash lessee mario automobile company l 692665

Capital Lease with Guaranteed Residual Value—Lessee

Mario Automobile Company leases automobiles under the following terms. A 3 year lease agreement is signed in which the lessor receives annual rental of $4,000 (in advance). At the end of the three years, the lessee agrees to make up any deficiency in residual value below $3,500. The cash price of the automobile is $13,251. The implicit interest rate is 12%,which is known to the lessee, and the lessee’s incremental borrowing rate is 14%. The lessee estimates the residual value at the end of three years to be $4,200 and depreciates its automobiles on a straight line basis.

1. Give the entries on the lessee’s books required in the first year of the lease, including the second payment on April 30, 2009. Assume the lease begins May 1, 2008, the beginning of the lessee’s fiscal year.

2. What balances relative to the lease would appear on the lessee’s balance sheet at the end of Year 3?

3. Assume that at the end of the three years, the automobile is sold by the lessee (with the permission of the lessor) for $3,800. Prepare the entries to record the sale and settlement with the lessor.

sales type lease mdash lessor salcedo co leased equipment to erickson inc on april 1 692666

Sales Type Lease—Lessor

Salcedo Co. leased equipment to Erickson Inc. on April 1, 2008. The lease, appropriately recorded as a sale by Salcedo, is for an 8 year period ending March 31, 2013. The first of eight equal annual payments of $175,000 (excluding executory costs) was made on

April 1, 2008. The cost of the equipment to Salcedo is $940,000. The equipment has an estimated useful life of eight years with no residual value expected. Salcedo uses straight line depreciation and takes a full year’s depreciation in the year of purchase. The cash selling price of the equipment is $1,026,900.

1. Give the entry required to record the lease on Salcedo’s books.

2. How much interest revenue will Salcedo recognize in 2008?

 

effect of lease on reported income mdash lessee and lessor on february 20 2008 topha 692668

Effect of Lease on Reported Income—Lessee and Lessor

On February 20, 2008,Topham Inc. purchased a machine for $1,200,000 for the purpose of leasing it. The machine is expected to have a 10 year life, no residual value, and is depreciated on the straight line basis to the nearest month. The machine was leased to Lutts Company on March 1, 2008, for a 4 year period at a monthly rental of $22,000. Assume that the lease payments are made at the end of the month and that the appropriate interest rate is 12% compounded monthly. There is no provision for the renewal of the lease or purchase of the machine by the lessee at the expiration of the lease term. Topham paid $60,000 of commissions associated with negotiating the lease in February 2008.

1. What expense should Lutts record as a result of the lease transaction for the year ended December 31, 2008?

2. What income or loss before income taxes should Topham record as a result of the lease transaction for the year ended December 31, 2008?

cash flow treatment of capital leases mdash lessor on january 1 2008 delhi club comp 692670

Cash Flow Treatment of Capital Leases—Lessor

On January 1, 2008, Delhi Club Company purchased some equipment for $45,372 in cash. Delhi Club immediately leased the equipment; Delhi Club is the lessor. The lease contract calls for the receipt of $10,000 payments at the end of each year for five years. The residual value of the equipment at the end of the 5 year lease term is expected to be $8,000. The rate implicit in the lease is 8%. Except for lease related items, there were no changes in current operating assets or liabilities during the year; no purchases or sales of property, plant, or equipment; and no dividends paid, stock issued, or loans obtained or repaid. The equipment has a total useful life of 10 years with no salvage value. Prepare a complete statement of cash flows for Delhi Club for 2008 using the indirect method of reporting operating cash flow assuming that the lease is accounted for as (1) an operating lease (net income was $50,000), (2) a direct financing lease (net income was $48,167), and (3) a sales type lease (net income was $48,167; for comparability, make the unreasonable assumption that sales and cost of good sold are the same amount).

lease disclosures mdash lessee the following lease information was obtained by a sta 692671

Lease Disclosures—Lessee

The following lease information was obtained by a staff auditor for a client, Kroller Inc., at December 31, 2008. Indicate how this information should be presented in Kroller’s 2 year comparative financial statements. Include any notes to the statements required to meet generally accepted accounting principles. Lease payments are made on December 31 of each year.

Leased building; minimum lease payments per year; 10 years remaining life                    

$ 45,000

Executory costs per year                                                       

2,000

Capitalized lease value, 12% interest                                               

343,269

Accumulated amortization of leased building at December 31, 2008                        

114,423

Amortization expense for 2008                                                   

22,885

Obligations under capital leases; balance at December 31, 2008                           

239,770

Obligations under capital leases; balance at December 31, 2007                           

254,259

lease disclosure on the financial statements acme enterprises leased equipment from 692672

Lease Disclosure on the Financial Statements

Acme Enterprises leased equipment from Monument Equipment Co. on January 1, 2008. The terms of the lease agreement require five annual payments of $20,000 with the first payment being made on January 1, 2008, and each subsequent payment being made on December 31 of each year. Because the equipment has an expected useful life of five years, the lease qualifies as a capital lease for Acme. Acme does not know Monument’s implicit interest rate and therefore uses its own incremental borrowing rate of 12% to calculate the present value of the lease payments. Acme uses the sum of the years’ digits method for amortizing leased assets. The expected salvage value of the leased asset is $0.

1. Prepare a schedule that shows the lease obligation balance in each year of the lease.

2. Prepare an asset amortization schedule for the leased asset.

3. Compare the amount shown on the year end balance sheet for the leased asset with that of the lease obligation for the years 2008 through 2012 and explain why the amounts differ.

 

sale leaseback accounting on july 1 2008 baker corporation sold equipment it had rec 692674

Sale Leaseback Accounting

On July 1, 2008, Baker Corporation sold equipment it had recently purchased to an unaffiliated company for $570,000. The equipment had a book value on Baker’s books of $450,000 and a remaining life of five years. On that same day, Baker leased back the equipment at $135,000 per year, payable in advance, for a 5 year period. Baker’s incremental borrowing rate is 10%, and it does not know the lessor’s implicit interest rate. What entries are required for Baker to record the transactions involving the equipment during the first full year, assuming the second lease payment is made on June 30, 2009? Ignore consideration of the lessee’s fiscal year. The lessee uses the double declining balance method of depreciation for similar assets it owns outright.

entries for capital lease mdash lessee lease criteria extractor company leased a mac 692676

Entries for Capital Lease—Lessee; Lease Criteria

Extractor Company leased a machine on July 1, 2008, under a 10 year lease. The economic life of the machine is estimated to be 15 years. Title to the machine passes to Extractor Company at the expiration of the lease, and thus, the lease is a capital lease. The lease payments are $97,000 per year, including executory costs of $3,000 per year, all payable in advance annually. The incremental borrowing rate of the company is 9%, and the lessor’s implicit interest rate is unknown.Extractor Company uses the straight line method of amortization and the calendar year for reporting purposes.

Instructions:

1. Give all entries on the books of the lessee relating to the lease for 2008.

2. Assume that the lessor retains title to the machine at the expiration of the lease, that there is no bargain renewal or purchase option, and that the fair market value of the equipment is $710,000 as of the lease date. Using the criteria for distinguishing between operating and capital leases according to FASB Statement No.13, what would be the amortization expense for 2008?

operating lease mdash lessee and lessor calderwood industries leases a large special 692677

Operating Lease—Lessee and Lessor

Calderwood Industries leases a large specialized machine to Youngstown Company at a total rental of $1,800,000, payable in five annual installments in the following declining pattern: 25% for each of the first two years, 22% in the third year, and 14% in each of the last two years. The lease begins January 1, 2008. In addition to the rent, Youngstown is required to pay annual executory costs of $15,000 to cover unusual repairs and insurance. The lease does not qualify as a capital lease for reporting purposes. Calderwood incurred initial direct costs of $15,000 in obtaining the lease. The machine cost Calderwood $2,100,000 to construct and has an estimated life of 10 years with an estimated residual value of $100,000. Calderwood uses the straight line depreciation method on its equipment. Both companies report on a calendar year basis.

Instructions:

1. Prepare the journal entries on Calderwood’s books for 2008 and 2012 related to the lease.

2. Prepare the journal entries on Youngstown’s books for 2008 and 2012 related to the lease.

investment in common stock mdash fair market value less than book value jjj inc purc 692611

Investment in Common Stock—Fair Market Value Less than Book Value

JJJ Inc. purchased 35% of ABC Co. on January 4, 2008, for $280,000 when ABC’s book value was $810,000. On that day, the market value of the net assets of ABC equaled their book values with the following exceptions:

 

Book

Market

Equipment

$175,000

$140,000

Buildings

40,000

65,000

The equipment has a remaining useful life of 10 years, and the building has a remaining useful life of 20 years. ABC reported the following related to operations for 2008 and 2009:

 

Net Income (Loss)

Dividends

2008

$ 80,000

$15,000

2009

(10,000)

8,000

Instructions: Provide the entries made by JJJ Inc. relating to its investment in ABC for the years 2008 and 2009.

accounting for marketable equity securities trans america trust co owns both trading 692613

Accounting for Marketable Equity Securities

Trans America Trust Co. owns both trading and available for sale securities. The following securities were owned on December 31, 2007:

Trading Securities:

 

 

Total

Market Value,

Market

Security

Shares

Cost

Dec 31, 2007

Adjustment

Albert Groceries, Inc.                    

600

$ 9,000

$11,500

$2,500 Dr.

West Data, Inc.                         

1,000

27,000

18,000

9,000 Cr

Steel Co                              

450

9,900

10,215

315 Dr.

Total                               

 

 $45,900

 $39,715

 $6,185 Cr

Available for Sale Securities:

 

 

Total

Market Value,

Market

Security

Shares

Cost

Dec 31, 2007

Adjustment

Dairy Products                       

2,000

$ 86,000

$ 90,000

$ 4,000 Dr.

Vern Movies, Inc.                      

15,000

390,000

365,000

25,000 Cr

Disks, Inc.                           

5,000

60,000

80,000

20,000 Dr.

Total                             

 

 $536,000

 $535,000

 $ 1,000 Cr

The following transactions occurred during 2008:

(a) Sold 500 shares of West Data, Inc., for $9,500.

(b) Sold 200 shares of Disks, Inc., for $3,000.

(c) Transferred all shares of Albert Groceries, Inc., to the available for sale portfolio when the total market value was $12,900.

(d) Transferred the remaining shares of Disks, Inc., to the trading securities portfolio when the market price was $20 per share. These shares were subsequently sold for $18 per share. At December 31, 2008, market prices for the remaining securities were as follows:

Security

Market Price per Share

Albert Groceries, Inc.                                           

$22

West Data, Inc.                                                 

15

Steel Co                                                     

21

Dairy Products                                                 

42

Vern Movies, Inc.                                                

28

Instructions: Prepare all journal entries necessary to record Trans America Trust Co.’s marketable equity securities transactions and year end adjustments for 2008. Assume that all declines in market value are temporary.

accounting for long term investments on january 2 2006 bradley company acquired 20 o 692614

Accounting for Long Term Investments

On January 2, 2006, Bradley Company acquired 20% of the 100,000 shares of outstanding common stock of Caldecott Corp. for $20 per share. The purchase price was equal to Caldecott’s underlying book value. Bradley plans to hold this stock to influence the activities of Caldecott.

The following data are applicable for 2006 and 2007:

 

2006

2007

Caldecott dividends (paid Oct. 31)

$30,000

$32,000

Caldecott earnings

75,000

90,000

Caldecott stock market price at year end

25

24

On January 2, 2008, Bradley Company sold 5,000 shares of Caldecott stock for $24 per share. During 2008, Caldecott reported net income of $64,000, and on October 31, 2008, Caldecott paid dividends of $20,000. At December 31, 2008, after a significant stock market decline, which is expected to be temporary, Caldecott’s stock was selling for $15 per share. After selling the 5,000 shares, Bradley does not expect to exercise significant influence over Caldecott, and the shares are classified as available for sale.

Instructions:

1. Make all journal entries for Bradley Company for 2006, 2007, and 2008, assuming the 20% original ownership interest allowed significant influence over Caldecott.

2. Make the year end valuation adjusting entries for Caldecott Company for 2006, 2007, and 2008, assuming the 20% original ownership interest did not allow significant influence over Caldecott.

investment securities and the statement of cash flows julie company came into existe 692615

Investment Securities and the Statement of Cash Flows

Julie Company came into existence with a $2,000 cash investment by owners on January 1, 2008, and entered into the following transactions during 2008:

 

Sales                                                                       

$ 3,200

Cash expenses                                                                

(2,700)

Purchase of building on January 1                                                  

550

Purchase of trading securities                                                     

500

Sale of trading securities (cost, $200)                                                 

340

Purchase of available for sale securities                                               

300

Sale of available for sale securities (cost, $100)                                         

60

The following additional information is available:

Balance in accounts receivable on December 31                                        

$190

Depreciation expense recognized for the year                                         

50

Market value of remaining trading securities on December 31                              

210

Market value of remaining available for sale securities on December 31                       

270

Instructions:

1. Prepare an income statement for 2008.

2. Prepare a complete statement of cash flows for 2008. Use the indirect method of reporting cash from operating activities.

3. Prepare a balance sheet as of December 31, 2008.

accounting for the impairment of a loan jayleen associates loaned norris company 750 692617

Accounting for the Impairment of a Loan

Jayleen Associates loaned Norris Company $750,000 on January 1, 2006. The terms of the loan were payment in full on January 1, 2011, plus annual interest payments at 11%. The interest payment was made as scheduled on January 1, 2007; however, due to financial setbacks, Norris was unable to make its 2008 interest payment. Jayleen considers the loan impaired and projects the following cash flows from the loan as of December 31, 2008, and 2009. Assume that Jayleen accrued the interest at December 31, 2007, but did not continue to accrue interest due to the impairment of the loan.

Projected Cash Flows:

 

Amount Projected

Amount Projected

Date of Flow

as of Dec 31, 2008

as of Dec 31, 2009

Dec 31, 2009                                 

$ 50,000

$ 50,000

Dec 31, 2010                                

100,000

150,000

Dec 31, 2011                                

200,000

300,000

Dec 31, 2012                                

300,000

250,000

Dec 31, 2013                                

100,000

 

Instructions:

1. Prepare the valuation adjusting entry at December 31, 2008.

2. Prepare the journal entry to record the $50,000 receipt on December 31, 2009.

3. Prepare the valuation adjusting entry at December 31, 2009.

4. Prepare the 2010 journal entries, assuming the receipt of $150,000 as scheduled; also assume that estimates for future cash flows remain the same as they were at the end of 2009.

but do we really have ldquo mark to market rdquo accounting now the movement toward 692618

But Do We Really Have “Mark to Market” Accounting Now?

The movement toward the use of market value for investments has been given the label “mark to market.” Previously, marketable equity securities were valued at the lower of cost or market. The shift to market, whether higher or lower than cost, is a significant departure from the past. Even though all investments classified as trading or available for sale are now valued at market values, only market changes for trading securities affect the income statement. To many accounting theorists, this is indeed a cop out on the part of the FASB. These accountants reason that if market changes are going to be recognized on the balance sheet, they should be recognized on the income statement as well. This position was held by the two FASB members who voted against the issuance of Statement No. 115. Evaluate the rationale for this compromise position. What arguments for the two different approaches (income and equity) do you think are most persuasive and why? What future events could cause standard setters to revise this approach?

i rsquo m not a bank so why must i worry about fasb statement no 115 accounting meth 692620

I’m Not a Bank, So Why Must I Worry about FASB Statement No. 115?

Accounting methods of financial institutions, such as savings and loan companies and banks, were the major reasons the FASB studied the valuation issues relating to investments. FASB Statement No. 115, however, affects all companies that invest in marketable debt and equity securities. As controller of a retailing company, you are concerned with the classification of “trading security.” How can you decide whether the investments you have are trading or available for sale securities? In discussing this issue with other controllers, you are surprised to hear some of them indicate that Statement No. 115 really doesn’t affect the reported income of nonfinancial institutions and that all securities for these companies are considered available for sale securities. Other controllers were concerned by this statement because this reasoning would make accounting for investments less conservative than it was before FASB Statement No.115. Do you agree with either of these points of view and why? In what way has FASB Statement No. 115 made accounting for investments less conservative?

why is 49 ownership enough in 1986 the coca cola company borrowed 2 4 billion to pur 692621

Why Is 49% Ownership Enough?

In 1986, The Coca Cola Company borrowed $2.4 billion to purchase several large soft drink bottling operations. Then a separate company, Coca Cola Enterprises, was formed to bottle and distribute Coke throughout the country. The Coca Cola Company sold 51% of Coca Cola Enterprises to the public and retained a 49% ownership. The $2.4 billion debt incurred to finance the purchase was transferred to the balance sheet of Coca Cola Enterprises. While 49% ownership does not guarantee control, it does give The Coca Cola Company significant influence over the bottling company. For example, The Coca Cola Company determines the price at which it will sell concentrate to Coca Cola Enterprises and reviews Coca Cola Enterprises’ marketing plan. In addition, The Coca Cola Company’s chief operating officer is chairman of Coca Cola Enterprises, and six other current or former Coca Cola Company officials are serving on Coca Cola Enterprises’ board of directors.             

1. From an accounting standpoint, what is the significance of owning more than 50% of a company’s stock?

2. Why would The Coca Cola Company elect to own less than 50% of its distribution network?

3. In the consolidation process, the parent’s and the subsidiary’s individual asset and liability account balances are added together and reported on the consolidated financial statements, whereas with the equity method, the net investment is reported as an asset on the investor company’s balance sheet. Why would The Coca Cola Company want to avoid consolidation?

which method of accounting for investments is appropriate international inc owns com 692622

Which Method of Accounting for Investments Is Appropriate?

International Inc. owns companies or the stock of companies in countries all over the world. International is reviewing its methods of accounting for those companies and has asked you to provide input as to whether the cost method, the equity method, or consolidation is appropriate for each of the following subsidiaries. Provide justification for your suggestions.

1. Subsidiary 1: This subsidiary, ME Oil, is an oil company located in the Middle East. A growing anti American sentiment in the country in which the company is located has led International to remove all non native employees. There is a growing fear that the government may nationalize ME Oil. International Inc. owns 75% of the oil company.

2. Subsidiary 2: Ecological Inc., a company that produces environmentally safe products, has production facilities in more than 10 states. The ownership of the company is widely held, with International Inc. holding the largest block of stock. International has succeeded in placing its president and vice president in two of the five board of directors’ seats of Ecological Inc. International owns 15% of Ecological Inc.’s outstanding stock.

3. Subsidiary 3: International Inc. recently purchased 100% of the outstanding stock of Harmon National Bank. This subsidiary represents International’s first purchase of a nonmanufacturing facility, and management has expressed concern about the comparability of the different accounting methods used by financial institutions.

4. Subsidiary 4: International has been involved in a takeover battle with Beatrix Inc. involving Campton Soups. Beatrix recently purchased 50% of the stock of Campton. International has owned 30% of Campton’s stock for five years.

what is the difference in accounting between the cost and equity methods logical cor 692623

What Is the Difference in Accounting between the Cost and Equity Methods?

Logical Corporation, a producer of medical products, disclosed the following investments in affiliates in the notes to its July 31, 2008, financial statements:

 

2008

2007

Investments (if classified as available for sale)

$ 822,188

$ 50,000

Investments (if using the equity method)

1,677,181

2,009,647

Discuss the factors that determine whether Logical uses the cost or the equity method in accounting for its investment in affiliates. What events are recorded when the security is accounted for as an available for sale security? What events are recorded when the equity method is used? What does the investment account represent when the security is classified as available for sale? What does it represent using the equity method?

deciphering financial statements the walt disney company locate disney rsquo s finan 692625

Deciphering Financial Statements (The Walt Disney Company)

Locate Disney’s financial statements and related notes on the Internet and answer the following questions:

1. Locate Disney’s note that discusses investments. What amount of the investment portfolio is classified as available for sale? Now look at Disney’s balance sheet. What percentage of the total investment account is available for sale securities? What types of securities constitute the balance in that account?

2. Locate Disney’s note that discusses what types of securities are included in “Investments” on the balance sheet. Also examine the note that defines cash and cash equivalents. Are all of Disney’s investment securities listed under Investments in the balance sheet?

3. Review the note on financial instruments to determine how the carrying value of investments compared to the fair value on September 30, 2004. Why is this number so much less than that reported as Investments on the balance sheet?

deciphering financial statements archer daniels midland company the investing activi 692626

Deciphering Financial Statements (Archer Daniels Midland Company)

The Investing Activities section of the statement of cash flows of Archer Daniels Midland

Company (ADM), seller of agricultural commodities and products, follows.

Archer Daniels Midland Company

Consolidated Statements of Cash Flows

 

Year ended June 30

(In thousands)

2004

2003

2002

Investing Activities

 

 

 

Purchases of property, plant and equipment                   

$(509,237)

$ (435,952)

$(362,974)

Proceeds from sales of property, plant and equipment           

57,226

40,061

16,553

Net assets of businesses acquired                           

(93,022)

(526,970)

(40,012)

Investments in and advances to affiliates, net                   

(112,984)

(130,096)

(65,928)

Distributions from affiliates, excluding dividends                

122,778

40,113

68,891

Purchases of marketable securities                          

(857,786)

(328,852)

(384,149)

Proceeds from sales of marketable securities                  

786,492

271,340

345,004

Other, net                                             

32,098

11,258

(11,108)

Total Investing Activities                           

 $(574,435)

 $(1,059,098)

 $(433,723)

Based on the information given, answer the following questions.

Instructions:

1. Based on all the buying and selling activity associated with ADM’s marketable securities, how do you think the company classifies the bulk of its $3.0 billion portfolio of securities—as trading, available for sale, or held to maturity? Now, take a look at ADM’s note relating to its classification of all of its marketable securities.

Marketable Securities The Company classifies its marketable securities as available for sale, except for certain designated securities which are classified as trading securities. Available for sale securities are carried at fair value, with the unrealized gains and losses, net of income taxes, reported as a component of other comprehensive income (loss). Unrealized gains and losses related to trading securities are included in income on a current basis. The Company uses the specific identification method when securities are sold or classified out of accumulated other comprehensive income (loss) into earnings.

2. Was your answer to (1) the same as ADM’s classification policy? With the company selling one third of its investment portfolio in 2004, are the company’s actions consistent with its classification policy? Finally, take a look at a portion of ADM’s consolidated statements of shareholders’ equity from its 2004 annual report.

Archer Daniels Midland Company

Consolidated Statements of Shareholders’ Equity

 

 

 

 

Accumulated

 

 

Common Stock

Reinvested

Other Comprehensive

Total Shareholders’

(In thousands)

Shares

Amount

Earnings

Income (Loss)

Equity

Balance June 30, 2003

644,855

$5,373,005

$1,863,150

$(166,958)

$7,069,197

Comprehensive income

 

 

 

 

 

Net earnings

 

 

494,710

 

 

Other comprehensive income

 

 

 

249,913

 

Total comprehensive income

 

 

 

 

744,623

Cash dividends paid—

 

 

 

 

 

$024 per share

 

 

(174,109)

 

(174,109)

Treasury stock purchases

(309)

(4,113)

 

 

(4,113)

Other

6,202

62,618

 

 

62,618

Balance June 30, 2004

650,748

$5,431,510

$2,183,751

$ 82,955

$7,698,216

3. Assume that, other than available for sale securities, ADM had no other items that impacted other comprehensive income. Did the company’s portfolio of marketable securities experience an unrealized net gain or an unrealized net loss for the year? If these securities had been classified as trading, where would this amount have been reported?

deciphering financial statements ford motor company the following note is taken from 692627

Deciphering Financial Statements (Ford Motor Company)

The following note is taken from Ford Motor Company’s 2004 annual report:

 

Amortized

Unrealized

Book/Fair

 

Cost

Gains Losses

Value

Available for sale securities

 

 

 

 

US government and agency                         

$1,179

$ 3

$10

$1,172

Other debt securities                              

1,100

15

10

1,105

Equity                                          

50

37

3

84

Total                                         

 $2,329

 $55

 $23

 $2,361

Investments in available for sale securities at December 31, 2003, were as follows (in millions):

 

Amortized

Unrealized

Book/Fair

 

Cost

Gains Losses

Value

Available for sale securities

 

 

 

 

US government and agency                         

$1,402

$ 6

$ 5

$1,403

Other debt securities                              

972

21

8

985

Equity                                          

47

31

3

75

Total                                         

 $2,421

 $58

 $16

 $2,463

The proceeds and net gains/(losses) from available for sale securities sales were as follows (in millions):

Proceeds

Gains/(Losses)

2004

2003

2004

2003

$8,402

$9,376

$(6)

$23

1. What amount of gains and losses on available for sale securities is reported in the 2004 income statement? How much is realized? How much is unrealized?

2. What is the amount of the net adjustment for unrealized holding gains and losses on available for sale securities as of the end of 2004?

3. Using the amounts of realized and unrealized gains and losses, estimate the total economic return on Ford’s available for sale portfolio during 2004 (ignoring interest and dividends).

writing assignment going around the income statement fasb statement no 115 outlines 692628

Writing Assignment (Going around the income statement)

FASB Statement No. 115 outlines two different treatments for unrealized gains and losses depending on whether the security is classified as trading or available for sale. Unrealized gains and losses for trading securities are reported on the income statement while unrealized gains and losses for available for sale securities are reported as part of accumulated other comprehensive income in stockholders’ equity. Your assignment is to develop an argument, in writing, for including unrealized gains and losses for available for sale securities on the income statement. Include in your 1 page paper reasons as to why the FASB might have chosen the reporting rules that it did and be able to refute the Board’s reasoning.

researching accounting standards to help you become familiar with the accounting sta 692629

Researching Accounting Standards

To help you become familiar with the accounting standards, this case is designed to take you to the FASB’s Web site and have you access various publications. In this chapter, we discussed issues relating to investing in debt and equity securities.

For this case, we will use Statement of Financial Accounting Standards No.115, “Accounting for Certain Investments in Debt and Equity Securities.” Open FASB Statement No. 115.

1. In paragraph 13, the reporting of unrealized gains and losses is discussed. How are unrealized gains and losses accounted for with trading securities? With available for sale securities?

2. Paragraph 16 discusses the accounting for securities that may be impaired. If available for sale and held to maturity securities are determined to be permanently impaired, how is that impairment to be accounted for? Why aren’t trading securities included in the discussion in paragraph 16?

cumulative spreadsheet analysis this assignment is based on the spreadsheet prepared 692631

Cumulative Spreadsheet Analysis

This assignment is based on the spreadsheet prepared in (1) of the cumulative spreadsheet assignment for Chapter 13. Review that assignment for a summary of the assumptions made in preparing a forecasted balance sheet, income statement, and statement of cash flows for 2009 for Skywalker Company. This assignment involves changing assumption (h) in the Chapter 13 assignment. Assume that Skywalker’s investment securities portfolio contains the following available for sale securities as of December 31, 2008:

 

Original Cost

Market Value,
12/31/05

Security A

$10

$22

Security B

25

18

Security C

5

8

Security D

40

15

Security E

1

7

Total 

$81

$70

 

As mentioned in the Chapter 13 assignment, Skywalker intends to invest another $28 in available for sale securities (security F) in 2009 in order to increase the total value of the portfolio by 40% to $98 ($70 + $28). Because Skywalker cannot predict future stock prices, the best forecast is that the market values of Securities A through E will remain the same during 2009 and that the market value of security F, to be acquired in 2009 for $28, will remain at $28. Revise the spreadsheet made in (1) of the Chapter 13 assignment in accordance with the above and following assumptions. In each case, any gains or losses expected to be realized in 2009 should be reported in a separate income statement line, “Investment income, net.”

1. Skywalker intends to sell security A in 2009 at an anticipated price of $22. That $22 will be used to buy security G. Skywalker’s best forecast is that the market value of security G will remain at $22 through the end of 2009.

2. How does the sale of security A in (1) impact expected cash from operating activities in 2009? Explain.

3. Repeat (1) and (2) assuming that, instead of selling security A, Skywalker intends to sell security D in 2009 at an anticipated price of $15, which will be used to buy security G. Skywalker’s best forecast is that the market value of security G will remain at $15 through the end of 2009.

journal entries for a capital lease mdash lessee refer to practice 15 6 assume that 692639

Journal Entries for a Capital Lease—Lessee

Refer to Practice 15 6. Assume that the lease is to be accounted for as a capital lease. Also assume that the leased asset is to be amortized over the 12 year asset life rather than the 10 year lease term. Make the journal entries necessary on the books of the lessee company (1) on the lease signing date and (2) at the end of the first year, including the recording of the first lease payment.

Practice 15 6

Journal Entries for an Operating Lease—Lessee

On January 1, the lessee company signed an operating lease contract. The lease contract calls for $3,000 payments at the end of each year for 10 years. The rate implicit in the lease is 10%. Make the journal entries necessary on the books of the lessee company (1) on the lease signing date and (2) to record the first lease payment.

leases on a statement of cash flows mdash lessee refer to practice 15 6 net income f 692642

Leases on a Statement of Cash Flows—Lessee

Refer to Practice 15 6. Net income for the year was $10,000. Except for lease related items, there were no changes in current operating assets or liabilities during the year, no purchases or sales of property, plant, or equipment, and no dividends paid, stock issued, or loans obtained or repaid. Prepare a complete statement of cash flows using the indirect method of reporting operating cash flow assuming that the lease is accounted for as (1) an operating lease (net income was $10,000) and (2) a capital lease (net income was $9,621).

Practice 15 6

Journal Entries for an Operating Lease—Lessee

On January 1, the lessee company signed an operating lease contract. The lease contract calls for $3,000 payments at the end of each year for 10 years. The rate implicit in the lease is 10%. Make the journal entries necessary on the books of the lessee company (1) on the lease signing date and (2) to record the first lease payment.

journal entries for a direct financing lease mdash lessor refer to practice 15 12 as 692644

Journal Entries for a Direct Financing Lease—Lessor

Refer to Practice 15 12. Assume that the lease is accounted for as a direct financing lease instead of as an operating lease. The interest rate implicit in the lease is 15%. Make the journal entries necessary on the lessor’s books to record (1) the signing of the lease, (2) the receipt of the initial $2,600 lease payment on the lease signing date, and (3) the recognition of interest revenue at the end of the first year.

Practice 15 12

Journal Entries for an Operating Lease—Lessor

On January 1, the lessor company purchased a piece of equipment for $10,000. The equipment has an expected life of five years with zero salvage value. The lessor company immediately leased the equipment under an operating lease agreement. The lease calls for the lessor company to receive lease payments of $2,600 per year to be received at the beginning of the year. Make the journal entries necessary on the books of the lessor company to record (1) the purchase of the equipment for cash, (2) the lease signing (including receipt of the first lease payment), and (3) depreciation of the leased equipment.

sales type lease with a bargain purchase option on january 1 the lessor company purc 692647

Sales Type Lease with a Bargain Purchase Option

On January 1, the lessor company purchased a piece of equipment for $6,000 as inventory. The lessor company immediately leased the equipment under a sales type lease agreement. The lease calls for the lessor company to receive five annual lease payments of $2,500 per year, to be received at the beginning of the year. In addition to the five annual payments of $2,500 at the beginning of each year, the lessor is to receive a bargain purchase option amount of $500 at the end of five years. The interest rate implicit in the lease is 12%. Make the journal entries necessary on the books of the lessor company to record (1) the lease signing (including receipt of the first lease payment) and (2) the recognition of interest revenue at the end of the first year.

accounting methods for equity securities for each of the following independent situa 692584

Accounting Methods for Equity Securities

For each of the following independent situations, determine the appropriate accounting method to be used: cost or equity. For cost method situations, determine whether the security should be classified as trading or available for sale. For equity method situations, determine whether consolidated financial statements would be required. Explain the rationale for your decision.

(a) ATV Company manufactures and sells four wheel recreational vehicles. It also provides insurance on its products through its wholly owned subsidiary, RV Insurance Company.

(b) Buy Right Inc. purchased 20,000 shares of Big Supply Company common stock to be held as a long term investment. Big Supply has 200,000 shares of common stock outstanding.

(c) Super Tire Manufacturing Co. holds 5,000 shares of the 10,000 outstanding shares of nonvoting preferred stock of Valley Corporation. Super Tire considers the investment as being long term in nature.

(d) Takeover Company owns 15,000 of the 50,000 shares of common stock of Western Supply Company. Takeover has tried and failed to obtain representation on Western’s board of directors. Takeover intends to sell the securities if it cannot obtain board representation at the next stockholders’ meeting, scheduled in three weeks.

(e) Espino Inc. purchased 50,000 shares of Independent Mining Company common stock. Independent has a total of 125,000 common shares outstanding. Espino has no intention to sell the securities in the foreseeable future.

investment in equity securities on january 10 2008 washington corporation acquired 2 692585

Investment in Equity Securities

On January 10, 2008, Washington Corporation acquired 20,000 shares of the outstanding common stock of United Company for $900,000. At the time of purchase, United Company had outstanding 80,000 shares with a book value of $3.6 million. On December 31, 2008, the following events took place:

(a) United reported net income of $180,000 for the calendar year 2008.

(b) Washington received from United a dividend of $0.75 per share of common stock.

(c) The market value of United Company stock had temporarily declined to $40 per share.

Give the entries that would be required to reflect the purchase and subsequent events on the books of Washington Corporation, assuming that (1) the security is classified as available for sale and (2) the equity method is appropriate.

trading securities during 2008 litten company purchased trading securities as a shor 692591

Trading Securities

During 2008, Litten Company purchased trading securities as a short term investment. The costs of the securities and their market values on December 31, 2008, follow:

 

 

Market Value,

Security

Cost

Dec 31, 2008

A                                                      

$ 65,000

$ 75,000

B                                                      

100,000

54,000

C                                                      

220,000

226,000

At the beginning of 2008, Litten had a zero balance in the market adjustment—trading securities account. Before any adjustments related to these trading securities, Litten had net income of $300,000.

1. What is net income after making any necessary trading security adjustments? (Ignore income taxes.)

2. What would net income be if the market value of security B were $95,000?

accounting for trading securities during 2007 sunbeam inc purchased the following tr 692592

Accounting for Trading Securities

During 2007, Sunbeam Inc. purchased the following trading securities:

 

 

Market Value,

Security

Cost

Dec 31, 2007

Luthor Corp common                                       

$22,000

$25,000

10% US Treasury notes                                      

17,000

10,000

ChevCo bonds                                             

16,000

19,000

At the beginning of 2007, Sunbeam had a zero balance in Market Adjustment—Trading Securities.

1. What entry would be made at year end, assuming the preceding values?

2. What entry would be made during 2008, assuming one half of the Luthor Corp. common stock is sold for $13,000?

3. Give the entry that would be made at the end of 2008, assuming the following situations:

(a) The market value of remaining securities is $41,000.

(b) The market value of remaining securities is $43,500.

(c) The market value of remaining securities is $48,000.

debt and equity securities american steel corp acquired the following securities in 692593

Debt and Equity Securities

American Steel Corp. acquired the following securities in 2008:

 

 

 

Market Value,

Security

Classification

Cost

Dec 31, 2008

A                                         

Trading

$10,000

$12,000

B                                         

Trading

16,000

10,000

C                                         

Available for sale

12,000

15,000

D                                         

Available for sale

20,000

15,000

E                                          

Held to maturity

20,000

22,000

At the beginning of 2008,American Steel had a zero balance in each of its market adjustment accounts.

1. What entry or entries would be made at the end of 2008, assuming the preceding market values?

2. If net income before any adjustments related to marketable securities was $100,000, what would reported income be after adjustments? (Ignore income taxes.)

temporary and ldquo other than temporary rdquo changes in value the securities portf 692594

Temporary and “Other than Temporary” Changes in Value

The securities portfolio for Hill Top Industries contained the following trading securities:

Securities

Initial

Market Value,

Market Value,

(common stock)

Cost

Dec 31, 2007

Dec 31, 2008

Randall Co                                   

$10,000

$12,000

$15,000

Streuling Co                                  

7,000

4,000

2,000

Santana Co                                   

21,000

18,000

22,000

1. Assuming that all changes in fair value are considered temporary, what is the effect of the changes in value on the 2007 and 2008 financial statements? Give the valuation entries for these years, assuming that the market adjustment account has a $0 balance at the beginning of 2007.

2. Assume that at December 31, 2008, management believed that the market value of the Streuling Co. common stock reflected an “other than temporary” decline in the value of that stock. Give the entries to be made on December 31, 2008, under this assumption.

reclassification of securities kyoto inc had the following portfolio of securities a 692595

Reclassification of Securities

Kyoto Inc. had the following portfolio of securities at the end of its first year of operations:

 

 

 

Year End

Security

Classification

Cost

Market Value

A                                         

Trading

$ 8,000

$13,000

B                                         

Trading

15,000

18,000

1. Provide the entry necessary to adjust the portfolio of securities to its market value.

2. In the following year, Kyoto elects to reclassify security B as an available for sale security. On the date of the transfer, security B’s market value is $16,500. Provide the journal entry to reclassify security B.

 

reclassification of securities bicknel technologies inc purchased the following secu 692596

Reclassification of Securities

Bicknel Technologies Inc. purchased the following securities during 2007:

 

 

 

Market Value,

Security

Classification

Cost

Dec 31, 2007

A                                       

Trading

$ 2,000

$ 4,000

B                                       

Trading

7,000

6,000

C                                       

Available for sale

18,000

16,000

D                                       

Available for sale

5,000

4,000

E                                       

Held to maturity

14,000

15,000

At the beginning of 2007, Bicknel Technologies had a zero balance in each of its market adjustment accounts. During 2008, after the 2007 financial statements had been issued, Bicknel determined that security B should be reclassified as an available for sale security and security C should be reclassified as a trading security. The market values on the date of the transfer are $5,500 for security B and $17,000 for security C.

Prepare the journal entries to do the following:

1. Adjust the portfolio of securities to its market value at December 31, 2007.

2. Reclassify security B as an available for sale security in 2008.

3. Reclassify security C as a trading security in 2008.

valuation of securities bridgeman paper co reported the following selected balances 692597

Valuation of Securities

Bridgeman Paper Co. reported the following selected balances on its financial statements for each of the four years 2006–2009:

 

2006

2007

2008

2009

Market adjustment—Trading securities

0

5500

3750

(1200)

Market adjustment—Available for sale securities

0

(1300)

900

1350

Based on these balances, reconstruct the valuation entries that must have been made each year.

accounting for securities during 2007 the first year of its operations profit indust 692598

Accounting for Securities

During 2007, the first year of its operations, Profit Industries purchased the following securities:

 

 

 

Market Value,

Market Value,

Security

Classification

Cost

Dec 31, 2007

Dec 31, 2008

A                          

Trading

$18,000

$13,000

$ 9,000

B                           

Trading

8,000

9,000

10,000

C                          

Available for sale

17,000

15,000

17,000

D                          

Available for sale

24,000

28,000

13,000

During 2008, Profit sold one half of security A for $8,000 and one half of security D for $15,000.

Provide the journal entries required to do the following:

1. Adjust the portfolio of securities to its market value at the end of 2007.

2. Record the sale of security A and security D.

3. Adjust the portfolio of securities to its market value at the end of 2008.

investment securities and the statement of cash flows indicate how each of the follo 692599

Investment Securities and the Statement of Cash Flows

Indicate how each of the following transactions or events would be reflected in a statement of cash flows prepared using the indirect method. Each transaction or event is independent of the others. For items (a) and (d), assume that the balance in the market adjustment account was zero at the beginning of the year.

(a) At year end, the trading securities portfolio has an aggregate cost of $185,000 and an aggregate fair value of $150,000.

(b) During the year, trading securities and available for sale securities were purchased for $50,000 and $70,000, respectively. The securities were paid for in cash.

(c) Trading securities on hand at the beginning of the period (cost $40,000) were sold for $62,000 cash.

(d) At year end, the trading securities portfolio has an aggregate cost of $170,000 and an aggregate fair value of $190,000.

 

accounting for the impairment of a loan galaxy enterprises loaned 200 000 to vader i 692601

Accounting for the Impairment of a Loan

Galaxy Enterprises loaned $200,000 to Vader Inc. on January 1, 2007. The terms of the loan require principal payments of $40,000 each year for five years plus interest at the market rate of interest of 8%. The first principal and interest payment is due on January 1, 2008. Vader made the required payments during 2008 and 2009. However, during 2009 Vader began to experience financial difficulties, requiring Galaxy to reassess the collectability of the loan. On December 31, 2009, Galaxy determines that the remaining principal payments will be collected, but the collection of interest is unlikely.

1. Compute the present value of the expected future cash flows as of December 31, 2009.

2. Provide the journal entry to record the loan impairment as of December 31, 2009.

3. Provide the journal entries for 2010 to record the receipt of the principal payment on January 1 and the recognition of interest revenue as of December 31, assuming that Galaxy’s assessment of the collectability of the loan has not changed.

accounting for trading securities fox company made the following transactions in the 692602

Accounting for Trading Securities

Fox Company made the following transactions in the common stock of NOP Company:

July 10, 2006

Purchased 10,000 shares at $45 per share.

Sept. 29, 2007

Sold 2,000 shares for $51 per share.

Aug. 17, 2008

Sold 2,500 shares for $33 per share.

The end of year market prices for the shares were as follows:

December 31, 2006

$47 per share

December 31, 2007

39 per share

December 31, 2008

31 per share

Instructions: Prepare the necessary entries for 2006, 2007, and 2008, assuming the NOP stock is classified as a trading security.

recording and valuing trading securities myers amp associates reports the following 692603

Recording and Valuing Trading Securities

Myers & Associates reports the following information on its December 31, 2006, balance sheet:

Trading securities (at cost)          

$225,850

 

Less: Market adjustment—trading securities

2260

$223,590

Supporting records of Myers’ trading securities portfolio show the following debt and equity securities:

Security

Cost

Market Value

200 shares Conway Co common                                

$ 25,450

$ 24,300

$80,000 US Treasury 7% bonds                                 

79,650

77,400

$120,000 US Treasury 7 1/2% bonds                             

120,750

121,890

Total                                                    

 $225,850

 $223,590

Interest dates on the treasury bonds are January 1 and July 1.Myers & Associates uses the revenue approach to record the purchase of bonds with accrued interest. During 2007 and 2008,Myers & Associates completed the following transactions related to trading securities:

2007

Jan. 1 Received semiannual interest on U.S. Treasury bonds. Assume that the appropriate adjusting entry was made on December 31, 2006.

Apr. 1 Sold $60,000 of the 7 1/2% U.S. Treasury bonds at 102 plus accrued interest. Brokerage fees were $200.

May 21 Received dividend of $0.25 per share on the Conway Co. common stock. The dividend had not been recorded on the declaration date.

July 1 Received semiannual interest on U.S. Treasury bonds and then sold the 7% bonds at 97 1/2. Brokerage fees were $250.

Aug. 15 Purchased 100 shares of Nieman Inc. common stock at $116 per share plus brokerage fees of $50.

Nov. 1 Purchased $50,000 of 8% U.S. Treasury bonds at 101 plus accrued interest. Brokerage fees were $125.

Interest dates are January 1 and July 1.

Dec. 31 Market prices of securities were Conway Co. common, $110; 7 1/2% U.S. Treasury bonds, 101 3/4; 8% U.S. Treasury bonds, 101; Nieman Inc. common, $116.75.

2008

Jan. 2 Recorded the receipt of semiannual interest on the U.S. Treasury bonds.

Feb. 1 Sold the remaining 7 1/2% U.S. Treasury bonds at 101 plus accrued interest. Brokerage fees were $300.

Instructions:

1. Prepare journal entries for the preceding transactions and to accrue interest on December 31, 2007. Ignore any amortization of premium or discount on U.S. Treasury bonds. Give computations in support of your entries.

2. Show how trading securities would be presented on the December 31,2007,balance sheet.

accounting for debt and equity securities during 2008 buzz company purchased 4 000 s 692604

Accounting for Debt and Equity Securities

During 2008, Buzz Company purchased 4,000 shares of Honey Company common stock for $12 per share and 2,500 shares of Pollen Company common stock for $27 per share. These investments are intended to be held as ready sources of cash and are classified as trading securities.

Also in 2008, Buzz purchased 4,500 shares of Flower Company common stock for $25 per share and $50,000 of treasury notes at 102. These securities are classified as available for sale.

During 2008, Buzz received the following interest and dividend payments on its investments:

Honey Company

$2 per share dividend

Pollen Company

$1 per share dividend

Flower Company

$3 per share dividend

Treasury notes

5% annual interest earned for 6 months

Market values of the securities at December 31, 2008, were as follows:

Honey Company

$16 per share

Pollen Company

$18 per share

Flower Company

$23 per share

Treasury notes

103

On March 23, 2009, the 2,500 shares of Pollen common stock were sold for $18 per share. On June 30, 2009, the treasury notes were sold at 101 plus accrued interest.

Market values of remaining securities at December 31, 2009, were as follows:

Honey Company

$15 per share

Flower Company

$29 per share

Instructions:

1. Prepare all 2008 and 2009 journal entries related to these securities.

2. Describe how the following items would be treated on Buzz Company’s statement of cash flows for the year ended December 31, 2009. Buzz uses the indirect method of reporting cash flows from operating activities.

(a) Proceeds from the sale of Pollen shares and any realized gain or loss from the sale.

(b) Proceeds from the sale of the treasury securities and any realized gain or loss from the sale.

(c) Any unrealized gain or loss on the remaining securities.

journal entries and balance sheet presentation for investments in securities on dece 692605

Journal Entries and Balance Sheet Presentation for Investments in Securities

On December 31, 2006, Durst Company’s balance sheet showed the following balances related to its securities accounts:

Trading securities                                                  

$155,000

 

Less: Market adjustment—trading securities                               

(7,250) 

$147,750

Available for sale securities                                           

$108,000

 

Add: Market adjustment—available for sale securities                         

10,000 

118,000

Interest receivable—NYC water bonds                                  

 

1,250

Durst’s securities portfolio on December 31,2006,was made up of the following securities:

Security

Classification

Cost

Market

1,000 shares Herzog Corp. stock

Trading

$75,000

$76,250

800 shares Taylor Inc. stock

Trading

55,000

52,825

10% New York City water bonds (interest payable

 

 

 

semiannually on January 1 and July 1)

Trading

25,000

18,675

1,000 shares Martin Inc. stock

Available for sale

59,000

65,000

2,000 shares Outdoors Unlimited Inc. stock

Available for sale

49,000

53,000

During 2007, the following transactions took place:

Jan. 3 Received interest on the New York City water bonds.

Mar. 1 Purchased 300 additional shares of Herzog Corp. stock for $22,950, classified as a trading security.

Apr. 15 Sold 400 shares of the Taylor Inc. stock for $69 per share.

May 4 Sold 400 shares of the Martin Inc. stock for $62 per share.

July 1 Received interest on the New York City water bonds.

Oct. 30 Purchased 1,500 shares of Cook Co. stock for $83,250, classified as a trading security.

The market values of the stocks and bonds on December 31, 2007, are as follows:

Herzog Corp stock                                                      

$7660 per share

Taylor Inc. stock                                                        

$6850 per share

Cook Co stock                                                         

$5525 per share

New York City water bonds                                                 

$20,555

Martin Inc. stock                                                        

$6100 per share

Outdoors Unlimited Inc. stock                                              

$2700 per share

Instructions:

1. Make all necessary journal entries for 2007, including any year end accrual or adjusting entries.

2. Show how the marketable securities would be presented on the balance sheet at December 31, 2007. Assume that the available for sale securities are classified as current assets.

 

journal entries for trading securities during 2008 and 2009 kopson co made the follo 692606

Journal Entries for Trading Securities

During 2008 and 2009, Kopson Co. made the following journal entries to account for transactions involving trading securities:

2008

 

 

 

 

 

(a)

Nov

1

Investment in Trading Securities—10% US Treasury Bonds               

106,883

 

 

 

 

Cash                                                    

 

106,883

 

 

 

To record the purchase of $100,000 of US Treasury

 

 

 

 

 

bonds at 10325 Brokerage fees were $300 Interest is

 

 

 

 

 

payable semiannually on January 1 and July 1

 

 

(b)

Dec 31

 

Unrealized Increase/Decrease in Value of

 

 

 

 

 

Available for Sale Securities                                      

4,283

 

 

 

 

Market Adjustment—Trading Securities                           

 

4,283

 

 

 

To record the decrease in market value of the current

 

 

 

 

 

marketable securities based on the following data

 

 

 

Cost

 

Market

Market Adjustment

Fleming Co stock                             

$ 25,250

$ 23,350

$1,900 Cr

Dobson Co stock                            

32,450

33,950

1,500 Dr.

10% US Treasury bonds                       

106,883

103,000

3,883 Cr

Total                                    

 $164,583

 $160,300

 $4,283 Cr

The beginning balance in Market Adjustment—Trading Securities was a $500 credit. There were no other entries in 2008.

2009

 

 

 

 

 

(c)

Jan

1

Cash                                                      

5,000

 

 

 

 

Interest Revenue                                           

 

5,000

 

 

 

To record interest revenue for 6 months

 

 

(d) July

 

1

Cash                                                      

5,000

 

 

 

 

Interest Revenue                                           

 

5,000

 

 

 

To record interest revenue for 6 months

 

 

(e)

Dec

6

Investment in Available for Sale Securities—Fleming Co                  

25,250

 

 

 

 

Investment in Trading Securities—Fleming Co                      

 

25,250

 

 

 

To reclassify Fleming Co stock from trading securities

 

 

 

 

 

to available for sale securities Market price was $24,500

 

 

 

 

 

at the date of reclassification

 

 

(f)

Dec 31

 

Unrealized Increase/Decrease in Value of Available for

 

 

 

 

 

Sale Securities                                                

3,483

 

 

 

 

Market Adjustment—Available for Sale Securities                    

 

300

 

 

 

Market Adjustment—Trading Securities                           

 

3,183

 

 

 

To record the decrease in market value of available for

 

 

 

 

 

sale securities based on the following data

 

 

             

 

 

Cost

Market

Market Adjustment

Dobson Co stock

$ 32,450

$ 32,650

$ 200 Dr.

10% US Treasury bonds

106,883

103,500

3,383 Cr

Fleming Co stock

25,250

24,950

300 Cr

Total

$164,583

$161,100

$3,483 Cr

There were no other entries in 2009.

Instructions: For each incorrect entry, give the entry that should have been made. Assume the revenue approach is used. Ignore any premium or discount amortization on U.S. Treasury bonds.

valuation of equity securities the investment portfolio of morris inc on december 31 692607

Valuation of Equity Securities

The investment portfolio of Morris Inc. on December 31, 2007, contains the following securities:

• Opus Co. common, 3% ownership, 5,000 shares; cost, $100,000; market value, $95,000; classified as a trading security.

• Garrod Inc. preferred, 2,000 shares; cost, $40,000; market value, $43,000; classified as a trading security.

• Sherrill Inc. common, 30% ownership, 20,000 shares; cost, $1,140,000; market value,

$1,130,000; classified as an influencing investment.

• Jennings Co. common, 15% ownership, 25,000 shares; cost, $67,500; market value,

$50,000; classified as an available for sale security.

Instructions:

1. Give the valuation adjustment required at December 31, 2007, assuming that all investments were purchased in 2007 and none of the indicated declines in market value are considered other than temporary.

2. Assume that the Jennings Co.’s common stock market decline is considered other than temporary. Give the valuation entry required at December 31, 2007, under this change in assumption.

3. Assume that the market values for the long term investment portfolio at December 31, 2008, were as follows:

Opus Co common                                                           

$ 102,000

Garrod Inc. preferred                                                           

43,000

Sherrill Inc. common                                                          

1,115,000

Jennings Co common                                                          

45,000

Give the valuation entries at December 31, 2008, assuming that the investment categories remain the same and that all declines in 2007 and 2008 are temporary except for the 2007 decline in Jennings Co. stock.

 

investments in common stock both seco inc and hillsborough corp have 100 000 shares 692608

Investments in Common Stock

Both Seco Inc. and Hillsborough Corp. have 100,000 shares of no par common stock outstanding. World Inc. acquired 10,000 shares of Seco stock for $6 per share and 30,000 shares of Hillsborough stock for $12 per share in 2005. Both securities are being held as long term investments. Changes in retained earnings for Seco and Hillsborough for 2007 and 2008 are as follows:

 

 

Hillsborough

 

Seco Inc.

Corp

Retained earnings (deficit), January 1, 2007

$175,000

$ (45,000)

Cash dividends, 2007

(37,500)

 

$137,500

$(45,000)

Net income, 2007

40,000

70,000

Retained earnings, December 31, 2007

$177,500

$ 25,000

Cash dividends, 2008                                               

(45,000)

(20,000)

Net income, 2008                                                 

70,000

40,000

Retained earnings, December 31, 2008                                 

 $202,500

 $ 45,000

Market value of stock: December 31, 2007                             

$ 750

$ 1300

December 31, 2008                             

650

1500

Instructions: Give the entries required on the books of World Inc. for 2007 and 2008 to account for its investments.

long term investments in stock mdash equity method on january 1 2008 compustat co bo 692609

Long Term Investments in Stock—Equity Method

On January 1, 2008, Compustat Co. bought 30% of the outstanding common stock of Freelance Corp. for $258,000 cash. Compustat Co. accounts for this investment by the equity method. At the date of acquisition of the stock, Freelance Corp.’s net assets had a carrying value of $590,000. Assets with an average remaining life of five years have a current market value that is $130,000 in excess of their carrying values. The remaining difference between the purchase price and the value of the underlying stockholders’ equity cannot be attributed to any identifiable tangible or intangible asset. Accordingly, the remaining difference is allocated to goodwill. At the end of 2008, Freelance Corp. reports net income of $180,000. During 2008, Freelance Corp. declared and paid cash dividends of $20,000.

Instructions: Give the entries necessary to reflect Compustat Co.’s investment in Freelance Corp. for 2008.

who gets the cash dividend on march 23 2008 the board of directors of mycroft compan 692554

Who Gets the Cash Dividend?

On March 23, 2008, the board of directors of Mycroft Company declared a quarterly cash dividend on its $1 par common stock of $0.50 per share, payable on May 10, 2008, to the shareholders of record on April 14, 2008. Before April 9, Mycroft’s shares traded in the stock market “with dividend,” meaning that the quoted stock price included the right to receive the dividend. After April 9, the shares traded “ex dividend,” meaning that the quoted price did not include the right to receive the dividend. Before April 9, Mycroft’s shares were selling for $30 per share. What should happen to Mycroft’s stock price on April 9, the dividend date? What should happen to Mycroft’s stock price on March 23, the dividend declaration date?

deciphering financial statements the walt disney company the 2004 financial statemen 692556

Deciphering Financial Statements (The Walt Disney Company)

The 2004 financial statements for The Walt Disney Company can be found on the Internet.

Locate those financial statements and consider the following questions.

1. What is the par value of Disney’s common stock? What was the average issuance price of Disney’s common stock?

2. Does Disney use the cost method or the par value method of accounting for treasury stock? As of September 30, 2004, what was the average cost of the repurchased shares held in treasury?

3. Combining information from questions (1) and (2), estimate the decrease in Disney’s retained earnings if all of the treasury shares were retired.

4. From Disney’s foreign currency translation adjustment, deduce whether the foreign currencies got stronger or weaker in 2004 (relative to the U.S. dollar) in the countries where Disney has subsidiaries.

5. In the notes to Disney’s financial statements, the accounting for stock options is summarized.

Does Disney use the fair value or the intrinsic value method?

deciphering financial statements general motors in 1993 general motors paid cash div 692557

Deciphering Financial Statements (General Motors)

In 1993, General Motors paid cash dividends on 11 different classes of capital stock. Those classes of stock were as follows:

 

Dividends

Total Dividends

 

per Share

(in millions)

Preferred stock, $500 series                                    

$168

$ 26

Preferred stock, $375 series                                   

126

10

Preferred stock, E I series                                     

142

46

Preferred stock, Series A Conversion                            

331

590

Depositary Shares, Series B                                    

228

1011

Depositary Shares, Series C                                   

325

1036

Depositary Shares, Series D                                   

198

311

Depositary Shares, Series G                                    

234

538

$1 2/3 par value common stock                                

080

5658

Class E common stock                                       

040

972

Class H common stock                                       

072

641

Total                                                   

 

$1,0839

1. The January 1, 1993, balance in General Motors’ retained earnings was a negative $3.354 billion. The December 31, 1993, balance was a negative $2.003 billion. How is it possible that General Motors was able to pay cash dividends during 1993?

2. This question will require a little research. What is the difference in stockholder rights between General Motors’ $12/3 common stock and the Class E and Class H common shares? How did the Class E and Class H shares come into existence?

3. In May 1993, General Motors redeemed all of the $5.00 series and $3.75 series preferred stock. The board of directors stated that the redemption of these preferred shares would give the company more financial flexibility by eliminating certain covenants associated with the shares. Get a copy of GM’s most recent annual report and find out how many of the 11 issues of General Motors’ capital stock outstanding in 1993 are still outstanding.

deciphering financial statements swire pacific limited the equity categories for swi 692558

Deciphering Financial Statements (Swire Pacific Limited)

The equity categories for Swire Pacific Limited are illustrated in Exhibit 13 9, on page 790. Using the information in the exhibit, answer the following questions:

1. Recall that the primary purpose of defining different reserve categories is to distinguish between distributable and no distributable equity. As of December 31, 2004, how much of Swire Pacific’s equity is distributable?

2. What is the U.S. equivalent of Swire Pacific’s revenue reserve?

3. What do you think is the purpose of the capital redemption reserve?

4. As of December 31, 2004, Swire Pacific has a property valuation reserve of HK$34,680 million. Assume that this reserve is recognized as part of one big revaluation—what journal entry would be made? The property valuation reserve is not distributable—why not?

5. What happened to property values related to Swire’s holdings during 2004?

writing assignment strategic accounting par value or cost method j d michael company 692559

Writing Assignment (Strategic accounting: par value or cost method?)

J.D. Michael Company has been very successful in recent years. Cash flow from operations is more than sufficient to cover the cost of all capital expenditures as well as regular cash dividends. J. D. Michael has decided to use some of its extra cash to begin a program of repurchasing its own shares in the open market. The shares will not be retired but will be held for potential reissue. Because J. D. Michael has never repurchased its own shares before, it has not had to make a choice between the par value and cost methods of accounting for treasury stock. As the resident expert on accounting in the company, you have been asked to Dr. aft a 1 page memo to the board of directors recommending either the cost method or the par value method of accounting for treasury stock. Your memo should adDr.ess issues like the prevailing practice, the likely effect on the financial statements (particularly the equity section), and the potential impact of the treasury stock accounting treatment on the ability to maintain steady cash dividend payments in the future.

researching accounting standards to help you become familiar with the accounting sta 692560

Researching Accounting Standards

To help you become familiar with the accounting standards, this case is designed to take you to the FASB’s Web site and have you access various publications.

In the chapter, we discussed share based payments and related disclosure. For this case, we will use Statement No. 123(R), Share Based Payments. Open FASB Statement No. 123(R).

1. Beginning with paragraph (a) of the summary, the standard details four reasons cited by the FASB for issuing SFAS 123(R). Briefly summarize those four reasons.

2. paragraph 1 of the standard specifies the measurement objective to be satisfied when valuing share based payments. What is the measurement objective?

3. Paragraph 64 of the standard details the disclosure requirements associated with share based payments. Briefly summarize the disclosure requirements.

cumulative spreadsheet analysis this spreadsheet assignment is a continuation of the 692562

Cumulative Spreadsheet Analysis

This spreadsheet assignment is a continuation of the spreadsheet assignments given in earlier chapters. If you completed those assignments, you have a head start on this one. Refer back to the instructions for preparing the revised financial statements for 2008 as given in (1) of the Cumulative Spreadsheet Analysis assignment in Chapter 3.

1. Skywalker wishes to prepare a forecasted balance sheet, a forecasted income statement, and a forecasted statement of cash flows for 2009. Use the financial statement numbers for 2008 as the basis for the forecast, along with the following additional information.

(a) Sales in 2009 are expected to increase by 40% over 2008 sales of $2,100.

(b) In 2009, new property, plant, and equipment acquisitions will be in accordance with the information in (q).

(c) The $480 in operating expenses reported in 2008 breaks down as follows: $15 in depreciation expense and $465 in other operating expenses.

(d) New long term debt will be acquired in 2009 in accordance with (u).

(e) Cash dividends will be paid in 2009 in accordance with (w).

(f) New short term loans payable will be acquired in an amount sufficient to make Skywalker’s current ratio in 2009 exactly equal to 2.0.

(g) Skywalker anticipates repurchasing additional shares of stock during 2009 in accordance with (x).

(h) Because changes in future prices and exchange rates are impossible to predict, Skywalker’s best estimate is that the balance in accumulated other comprehensive income will remain unchanged in 2009.

(i) In the absence of more detailed information, assume that the balances in Investment Securities, Long Term Investments, and Other Long Term Assets will all increase at the same rate as sales (40%) in 2009.The balance in Intangible Assets will change in accordance with item (r).

(j) In the absence of more detailed information, assume that the balance in the other long term liabilities account will increase at the same rate as sales (40%) in 2009.

(k) The investment securities are classified as available for sale securities. Accordingly, cash from the purchase and sale of these securities is classified as an investing activity.

(l) Assume that transactions impacting other long term assets and other long term liabilities accounts are operating activities.

(m) Cash and investment securities accounts will increase at the same rate as sales.

(n) The forecasted amount of accounts receivable in 2009 is determined using the forecasted value for the average collection period. The average collection period for 2009 is expected to be 14.08 days. To make the calculations less complex, this value of 14.08 days is based on forecasted end of year accounts receivable rather than on average accounts receivable.

(o) The forecasted amount of inventory in 2009 is determined using the forecasted value for the number of days’ sales in inventory. The number of days’ sales in inventory for 2009 is expected to be 107.6 days. To make the calculations simpler, this value of 107.6 days is based on forecasted end of year inventory rather than on average inventory.

(p) The forecasted amount of accounts payable in 2009 is determined using the forecasted value for the number of days’ purchases in accounts payable. The number of days’ purchases in accounts payable for 2009 is expected to be 48.34 days. To make the calculations simpler, this value of 48.34 days is based on forecasted end of year accounts payable rather than on average accounts payable.

(q) The forecasted amount of property, plant, and equipment (PP&E) in 2009 is determined using the forecasted value for the fixed asset turnover ratio. The fixed asset turnover ratio for 2009 is expected to be 3.518 times. To make the calculations simpler, this ratio of 3.518 is based on forecasted end of year gross property, plant, and equipment balance rather than on the average balance. (Note: For simplicity, ignore accumulated depreciation in making this calculation.)

(r) Skywalker has determined that no new intangible assets will be acquired in 2009.

Intangible assets are amortized according to the information in (t).

(s) In computing depreciation expense for 2009, use straight line depreciation and assume a 30 year useful life with no residual value. Gross PP&E acquired during the year is only depreciated for half the year. In other words, depreciation expense for 2009 is the sum of two parts: (1) a full year of depreciation on the beginning balance in PP&E, assuming a 30 year life and no residual value, and (2) a half year of depreciation on any new PP&E acquired during the year, based on the change in the gross PP&E balance.

(t) Skywalker assumes a 20 year useful life for its intangible assets. Assume that the $100 in intangible assets reported in 2008 is the original cost of the intangibles. Include the amortization expense with the depreciation expense in the income statement.

(u) New long term debt will be acquired (or repaid) in an amount sufficient to make

Skywalker’s debt ratio (total liabilities divided by total assets) in 2009 exactly equal to 0.80.

(v) Assume an interest rate on short term loans payable of 6.0% and on long term debt of 8.0%. Only a half year’s interest is charged on loans taken out during the year. For example, if short term loans payable at the end of 2009 is $15 and given that short term loans payable at the end of 2008 were $10, total short term interest expense for 2009 would be $0.75 [($10 x 0.06) + ($5 x 0.06 x 1/2)].

(w) Skywalker has decided to begin paying cash dividends in 2009. Skywalker intends to maintain a dividend payout ratio (cash dividends divided by net income) of 40%.

(Note: Make sure you adjust your spreadsheet formula so that if net income happens to be negative, cash dividends are no lower than $0.)

(x) Skywalker has decided to continue its stock repurchase program in 2009. Skywalker intends to spend $50 repurchasing shares during the year. Skywalker accounts for treasury stock purchases using the cost method. Clearly state any additional assumptions that you make.

2. According to the forecast for 2009, state whether Skywalker is expected to issue new shares of stock. Would your answer change if Skywalker were not to repurchase the shares as described in (x)?

3. Repeat (2), with the following changes in assumptions:

(a) The debt ratio in 2009 is exactly equal to 0.70.

(b) The debt ratio in 2009 is exactly equal to 0.95.

4. Comment on how it is possible for a company to have negative net paid in capital when net paid in capital is equal to paid in capital minus treasury stock.

equity method cost greater than book value on january 1 of year 1 dr idge company pu 692569

Equity Method: Cost Greater than Book Value

On January 1 of Year 1, Dr. idge Company purchased 2,500 shares of the 10,000 outstanding shares of Company C for a total of $100,000. At the time of the purchase, the book value of Company C’s equity was $300,000.Company C assets having a market value greater than book value at the time of the acquisition were as follows:

 

Book

Market

Remaining

Asset

Value

Value

Life

Inventory                                          

$ 40,000

$ 50,000

less than 1 year

Building                                            

200,000

250,000

10 years

Goodwill                                          

0

40,000

indefinite

Company C’s net income in Year 1 was $70,000. Dividends per share paid by Company C were $2.00 in Year 1. (1) Make all journal entries necessary on Dr. idge’s books to record its investment in Company C in Year 1. Assume that the goodwill is not impaired. (2) Compute the Year 1 ending balance in Dr. idge Company’s Investment in Company C account.

changes in value available for sale securities refer to practice 14 ndash 10 make th 692571

Changes in Value: Available for Sale Securities

Refer to Practice 14–10. Make the adjusting journal entries for (a) and (b) and the computations for (c) and (d), assuming that the securities are classified as available for sale.

Practice 14–10

Changes in Value: Trading Securities

On December 1, the company purchased securities for $1,000. On December 31, the company still held the securities. Make the necessary adjusting journal entry to record a change in value of the securities assuming that their December 31 market value was (a) $1,200 and (b) $850. In addition, before considering the impact of the change in value of the securities, the net income for the company was $1,500. Compute net income assuming that the December 31 market value of the securities was (c) $1,200 and (d) $850. Ignore income taxes. Assume that the securities are classified as trading.

changes in value held to maturity securities refer to practice 14 ndash 10 make the 692572

Changes in Value: Held to Maturity Securities

Refer to Practice 14–10. Make the adjusting journal entries for (a) and (b) and the computations for (c) and (d), assuming that the securities are classified as held to maturity. The changes in value are not deemed to be “other than temporary.”

Practice 14–10

Changes in Value: Trading Securities

On December 1, the company purchased securities for $1,000. On December 31, the company still held the securities. Make the necessary adjusting journal entry to record a change in value of the securities assuming that their December 31 market value was (a) $1,200 and (b) $850. In addition, before considering the impact of the change in value of the securities, the net income for the company was $1,500. Compute net income assuming that the December 31 market value of the securities was (c) $1,200 and (d) $850. Ignore income taxes. Assume that the securities are classified as trading.

changes in value equity method refer to practice 14 ndash 10 make the adjusting jour 692573

Changes in Value: Equity Method

Refer to Practice 14–10. Make the adjusting journal entries for (a) and (b) and the computations for (c) and (d), assuming that the securities are accounted for using the equity method. Ignore the impact of the investee company income and dividends. The changes in value are not deemed to be “other than temporary.”

Practice 14–10

Changes in Value: Trading Securities

On December 1, the company purchased securities for $1,000. On December 31, the company still held the securities. Make the necessary adjusting journal entry to record a change in value of the securities assuming that their December 31 market value was (a) $1,200 and (b) $850. In addition, before considering the impact of the change in value of the securities, the net income for the company was $1,500. Compute net income assuming that the December 31 market value of the securities was (c) $1,200 and (d) $850. Ignore income taxes. Assume that the securities are classified as trading.

sale of securities and the market adjustment account the company purchased the follo 692574

Sale of Securities and the Market Adjustment Account

The company purchased the following securities during Year 1:

 

 

 

Market Value

Security

Classification

Cost

(Dec 31, Year 1)

A                                         

Trading

$ 9,000

$10,000

B                                         

Trading

10,000

16,000

On July 23, Year 2, the company sold all of the shares of security B for a total of $9,500. As of December 31, Year 2, the shares of security A had a market value of $5,800. No other activity occurred during Year 2 in relation to the trading security portfolio. (1) What amount should the company report as realized gain or loss in the Year 2 income statement? Clearly indicate whether the amount is a gain or a loss. (2) What amount should the company report as unrealized gain or loss in the Year 2 income statement? Clearly indicate whether the amount is a gain or a loss.

transfer between categories to and from trading the company purchased the following 692575

Transfer between Categories: To and From Trading

The company purchased the following securities during Year 1:

 

 

 

Market Value

Security

Classification

Cost

(Dec 31, Year 1)

A                                         

Trading

$5,000

$4,000

B                                         

Available for sale

6,000

8,000

In Year 2, the company reclassified both of these securities. Security A was reclassified as available for sale; the market value of security A at the time of the reclassification was $5,500. Security B was reclassified as trading; the market value of security B at the time of the reclassification was $4,100. Make the journal entries necessary to record both of these reclassifications.

cash flow and available for sale securities the company entered into the following t 692577

Cash Flow and Available for Sale Securities

The company entered into the following transactions during the year:

Purchase of investment securities                                                      

$400

Sale of investment securities                                                         

470

The company had no investment securities at the beginning of the year. The cost of the investment securities sold was $350. The market value of the remaining securities was $65 on December 31.The net income for the year was $880. Assume that net income does not include any noncash items and does not reflect gains or losses related to investment securities. Assume that the securities are classified as available for sale. Compute (1) cash flow from operating activities and (2) cash flow from investing activities.

cash flow and trading securities refer to practice 14 ndash 18 assume that the secur 692578

Cash Flow and Trading Securities

Refer to Practice 14–18. Assume that the securities are classified as trading. Compute (1) cash flow from operating activities and (2) cash flow from investing activities.

Practice 14–18

Cash Flow and Available for Sale Securities

The company entered into the following transactions during the year:

Purchase of investment securities                                                       

$400

Sale of investment securities                                                         

470

The company had no investment securities at the beginning of the year. The cost of the investment securities sold was $350. The market value of the remaining securities was $65 on December 31.The net income for the year was $880. Assume that net income does not include any noncash items and does not reflect gains or losses related to investment securities. Assume that the securities are classified as available for sale. Compute (1) cash flow from operating activities and (2) cash flow from investing activities.

loan impairment initial measurement on january 1 of year 1 the lending company made 692580

Loan Impairment: Initial Measurement

On January 1 of Year 1, the lending company made a $10,000, 8% loan. The $800 interest is receivable at the end of each year, with the principal amount to be received at the end of five years. As of the end of Year 1, the first year’s interest of $800 has not yet been received because the borrower is experiencing financial difficulties. The lending company negotiated a restructuring of the loan. The payment of all of the interest ($4,000 = $800 x 5 years) will be delayed until the end of the 5 year loan term. In addition, the amount of principal repayment will be Dr. opped from $10,000 to $5,000. Make the journal entry necessary on the lending company’s books to record this loan impairment on December 31 of Year 1.

loan impairment subsequent interest revenue refer to practice 14 ndash 21 make all j 692581

Loan Impairment: Subsequent Interest Revenue

Refer to Practice 14–21. Make all journal entries necessary on the lending company’s books in connection with the loan during Year 2, Year 3,Year 4, and Year 5. Assume that all cash payments are received according to the renegotiated schedule.

Practice 14–21

Loan Impairment: Initial Measurement

On January 1 of Year 1, the lending company made a $10,000, 8% loan. The $800 interest is receivable at the end of each year, with the principal amount to be received at the end of five years. As of the end of Year 1, the first year’s interest of $800 has not yet been received because the borrower is experiencing financial difficulties. The lending company negotiated a restructuring of the loan. The payment of all of the interest ($4,000 = $800 x 5 years) will be delayed until the end of the 5 year loan term. In addition, the amount of principal repayment will be Dr. opped from $10,000 to $5,000. Make the journal entry necessary on the lending company’s books to record this loan impairment on December 31 of Year 1.

recording securities transactions the following transactions of knight inc occurred 692582

Recording Securities Transactions

The following transactions of Knight, Inc., occurred within the same accounting period:

(a) Purchased $105,000 U.S. Treasury 7% bonds, paying 103 plus accrued interest of $1,200. In addition, Knight paid brokerage fees of $470. Knight uses the revenue approach to record accrued interest on purchased bonds. Knight classified this security as a trading security.

(b) Purchased 1,700 shares of Sand Co. common stock at $85 per share plus brokerage fees of $1,750. Knight classifies this stock as an available for sale security.

(c) Received semiannual interest on the U.S. Treasury bonds.

(d) Sold 250 shares of Sand at $97 per share.

(e) Sold $30,000 of U.S. Treasury 7% bonds at 102 plus accrued interest of $350.

(f) Purchased a $20,000, 6 month certificate of deposit. The certificate is classified as a trading security.

Prepare the entries necessary to record these transactions.

accounting for the purchase and sale of securities during january 2008 aragorn inc p 692583

Accounting for the Purchase and Sale of Securities

During January 2008, Aragorn Inc. purchased the following securities:

Security

Classification

No of Shares

Total Cost

Gimli Corporation stock                        

Trading

500

$ 9,000

Legolas International Inc. stock                   

Available for sale

1,000

22,000

Glorfindel Enterprises stock                     

Available for sale

2,500

42,500

Mirkwood Co bonds                          

Held to maturity

24,000

US Treasury bonds                         

Trading

11,000

During 2008, Aragorn received interest from Mirkwood and the U.S. Treasury totaling $3,630. Dividends received on the stock held amounted to $1,760. During November 2008, Aragorn sold 200 shares of the Gimli stock at $17 per share and 250 shares of the Glorfindel stock at $19 per share. Give the journal entries required by Aragorn to record the (1) purchase of the debt and equity securities; (2) receipt of interest and dividends during 2008; and (3) sale of the equity securities during November.

reconstruction of equity transactions manti company had the following account balanc 692530

Reconstruction of Equity Transactions

Manti Company had the following account balances on its balance sheet at December 31, 2008, the end of its first year of operations. All stock was issued on a subscription basis.

Common stock subscriptions receivable                                              

$150,000

Common stock, $1 par                                                           

3,000

Common stock subscribed                                                       

9,000

Paid in capital in excess of par—common                                            

348,000

8% preferred stock, $100 par                                                      

120,000

Paid in capital in excess of par—8% preferred                                         

60,000

10% preferred stock, $50 par                                                     

25,000

Retained earnings                                                              

10,000

The reported net income for 2008 was $55,000. Assume that revenues and expenses were closed to a temporary account, Income Summary. Use this account to complete the closing process.

Instructions: From the data given, reconstruct in summary form the journal entries to record all transactions involving the company’s stockholders. Indicate the amount of dividends distributed on each class of stock.

comprehensive analysis and reporting of stockholders rsquo equity egbert company has 692531

Comprehensive Analysis and Reporting of Stockholders’ Equity

Egbert Company has two classes of capital stock outstanding: 10%,$20 par preferred and $1 par common. During the fiscal year ended November 30, 2008, the company was active in transactions affecting the stockholders’ equity. The following summarizes these transactions:

 

Number of

Price per

Type of Transaction

Shares

Share

(a) Issue of preferred stock                                       

8,000

$26

(b) Issue of common stock                                       

25,000

65

(c) Reacquisition and retirement of preferred stock                      

4,000

29

(d) Purchase of treasury stock—common (reported at cost)               

10,000

70

(e) Stock split—common (par value reduced to $050)                   

2 for 1

 

(f) Reissuance of treasury stock—common (after stock split)               

10,000

55

Balances of the accounts in the Stockholders’ Equity section of the November 30, 2007, balance sheet were

Preferred stock, 60,000 shares                                                  

$ 1,200,000

Common stock, 200,000 shares                                                  

200,000

Paid in capital in excess of par—preferred                                           

300,000

Paid in capital in excess of par—common                                          

12,600,000

Retained earnings                                                            

780,000

Dividends were paid at the end of the fiscal year on the common stock at $1.10 per share and on the preferred stock at the preferred rate. Net income for the year was $700,000.

Instructions: Based on the preceding data, prepare the Stockholders’ Equity section of the balance sheet as of November 30, 2008. (Note: A work sheet beginning with November 30, 2007, balances showing transactions for the current year will facilitate the preparation of this section of the balance sheet.)

accounting for various capital stock transactions the stockholders rsquo equity sect 692532

Accounting for Various Capital Stock Transactions

The stockholders’ equity section of Webster Inc. showed the following data on December 31, 2007: common stock, $3 par, 300,000 shares authorized, 250,000 shares issued and outstanding, $750,000; paid in capital in excess of par, $7,050,000; additional paid in capital from stock options, $150,000; retained earnings, $480,000. The stock options were granted to key executives and provided them the right to acquire 30,000 shares of common stock at $35 per share. The options had a value of $5 each on the grant date. The following transactions occurred during 2008.

Mar. 31 Key executives exercised 4,500 options outstanding at December 31, 2007. The market price per share was $44 at this time.

Apr. 1 The company issued bonds of $2,000,000 at par, giving each $1,000 bond a detachable warrant enabling the holder to purchase two shares of stock at $40 each for a 1 year period. Market values immediately following issuance of the bonds were $4 per warrant and $998 per $1,000 bond without the warrant.

June 30 The company issued rights to stockholders (one right on each share, exercisable within a 30 day period) permitting holders to acquire one share at $40 with every 10 rights submitted. Shares were selling for $43 at this time. All but 6,000 rights were exercised on July 31, and the additional stock was issued.

Sept. 30 All warrants issued with the bonds on April 1 were exercised.

Nov. 30 The market price per share Dr. opped to $33, and options came due. Because the market price was below the option price, no remaining options were exercised.

Instructions:

1. Provide entries to record these transactions.

2. Prepare the stockholders’ equity section of the balance sheet as of December 31, 2008 (assume net income of $210,000 for 2008).

 

accounting for various capital stock transactions pine view co organized on june 1 2 692533

Accounting for Various Capital Stock Transactions

Pine view Co., organized on June 1, 2007, was authorized to issue stock as follows:

• 80,000 shares of preferred 9% stock, convertible, $100 par

• 250,000 shares of common stock, $2.50 stated value

During the remainder of Pine view Co.’s fiscal year ended May 31, 2008, the following transactions were completed in the order given:

(a) 30,000 shares of preferred stock were subscribed for at $105, and 90,000 shares of common stock were subscribed for at $26. Both subscriptions were payable 30% upon subscription, the balance in one payment.

(b) The second subscription payment was received, except one subscriber for 6,000 shares of common stock defaulted on payment. The full amount paid by this subscriber was returned, and all of the fully paid stock was issued.

(c) 15,000 shares of common stock were reacquired by purchase at $28. (Treasury stock is recorded at cost.)

(d) Each share of preferred stock was converted into four shares of common stock.

(e) The treasury stock was exchanged for machinery with a fair market value of $430,000.

(f) There was a 2 for 1 stock split, and the stated value of the new common stock is $1.25.

(g) Net income was $83,000. Assume that revenues and expenses have been closed to a temporary account, Income Summary.

Instructions:

1. Give the journal entries to record these transactions. (For net income, give the entry to close the income summary account to Retained Earnings.)

2. Prepare the Stockholders’ Equity section as of May 31, 2008.

issuance repurchase and resale of capital stock papa tom rsquo s company had the fol 692534

Issuance, Repurchase, and Resale of Capital Stock

Papa Tom’s Company had the following transactions occur during 2008:

(a) Issued 10,000 shares of common stock to the founders for land valued at $350,000.Par value of the common stock is $1 per share.

(b) Issued 2,000 shares of $100 par preferred stock for cash at $115.

(c) Sold 3,000 shares of common stock to the company president for $50 per share.

(d) Purchased 500 shares of outstanding preferred stock issued in (b) for cash at par.

(e) Purchased 1,000 shares of the outstanding common stock issued in (a) for $42 per share.

(f) Reissued 200 shares of repurchased preferred stock at $104.

(g) Reissued 400 shares of reacquired common stock for $50 per share.

(h) Repurchased 100 shares of the common stock sold in (g) for $47 per share. These same

100 shares were later reissued for $45 per share.

Instructions:

1. Prepare the necessary entries to record the preceding transactions involving PapaTom’s preferred stock. Assume that the par value method is used for recording treasury stock.

2. Prepare the necessary entries for the common stock transactions assuming that the cost method is used for recording treasury stock.

treasury stock transactions transactions that affected barter company rsquo s stockh 692535

Treasury Stock Transactions

Transactions that affected Barter Company’s stockholders’ equity during 2008, the first year of operations, follow.

(a) Issued 30,000 shares of 9% preferred stock, $20 par, at $26.

(b) Issued 50,000 shares of $3 par common stock at $33.

(c) Purchased and immediately retired 4,000 shares of preferred stock at $28.

(d) Purchased 6,000 shares of its own common stock at $35.

(e) Reissued 1,000 shares of treasury stock at $37.

No dividends were declared in 2008, and net income for 2008 was $185,000.

Instructions:

1. Record each of the transactions. Assume that treasury stock acquisitions are recorded at cost.

2. Prepare the stockholders’ equity section of the balance sheet at December 31, 2008.

accounting for stock options the board of directors of muir company adopted a fixed 692536

Accounting for Stock Options

The board of directors of Muir Company adopted a fixed stock option plan to supplement the salaries of certain executives of the company. Options to buy common stock were granted as follows:

 

 

Number

Exercise

Price of Shares

Option Value at

Date

Employee

of Shares

Price

at Date of Grant

Date of Grant

Jan. 1, 2005

D. R. Call

80,000

$30

$32

$ 9

Jan. 1, 2006

J. K. Neilson

45,000

38

41

10

Jan. 1, 2007

B. D. Gwynn

25,000

43

47

11

Options are nontransferable and can be exercised beginning three years after date of grant, provided the executive is still employed by the company. Stock options were exercised as follows:

 

 

Number of

Price of Shares at

Date

Employee

Shares

Date of Exercise

Dec 31, 2008                              

D R Call

80,000

$48

Dec 31, 2009                              

J K Neilson

45,000

43

Dec 31, 2010                              

B D Gwynn

25,000

49

Stock of the company has a $1 par value. The accounting period for the company is the calendar year.

Instructions:

1. Provide all entries that would be made on the books of Muir relative to the stock option plan for the period 2005 to 2010 inclusive.

2. Prepare the required note disclosure relative to the stock option plan for the year 2007 and for the year 2009.

performance based stock options bauil corporation a new environmental control compan 692537

Performance Based Stock Options

Bauil Corporation, a new environmental control company, initiated a performance based stock option plan for its management on January 1, 2007. The plan provided for the granting of a variable number of stock options to management personnel who worked for the entire 4 year period ending December 31, 2010, depending on the net income earned by the company in 2010. No options were granted for the first $50,000 of net income. Thereafter, the following options were available based on the level of net income in 2010.

$50,000–$99,999                                                      

5,000 stock options

$100,000–$124,999                                                     

10,000 stock options

$125,000–$149,999                                                     

15,000 stock options

$150,000 or more                                                     

25,000 stock options

The exercise price for the $5 par common stock was $25 per share. The fair value of the options on the grant date was $7. Assume the market price for the Bauil stock and Bauil’s forecasted 2010 net income were as follows at each of the following dates:

 

Stock Price

Forecasted 2010 Income

January 1, 2007                                  

$27

$110,000

December 31, 2007                               

30

130,000

December 31, 2008                               

29

160,000

December 31, 2009                               

35

140,000

December 31, 2010                               

36

130,000 (actual)

Instructions:

Prepare journal entries related to the stock options of Bauil for the period 2007–2010 assuming that all available options are exercised on December 31, 2010.

analysis of stock transactions you have been asked to audit greystone company during 692538

Analysis of Stock Transactions

You have been asked to audit Greystone Company. During the course of your audit, you are asked to prepare comparative data from the company’s inception to the present. You have determined the following:

(a) Greystone Company’s charter became effective on January 2, 2004, when 2,000 shares of no par common and 1,000 shares of 7% cumulative, nonparticipating, preferred stock were issued. The no par common stock had no stated value and was sold at $120 per share, and the preferred stock was sold at its par value of $100 per share.

(b) Greystone was unable to pay preferred dividends at the end of its first year. The owners of the preferred stock agreed to accept 2 shares of common stock for every 50 shares of preferred stock owned in discharge of the preferred dividends due on December 31, 2004. The shares were issued on January 2, 2005. The fair market value was $100 per share for common on the date of issue.

(c) Greystone Company acquired all of the outstanding stock of Booth Corporation on May 1, 2006, in exchange for 1,000 shares of Greystone common stock.

(d) Greystone split its common stock 3 for 2 on January 1, 2007, and 2 for 1 on January 1, 2008.

(e) Greystone offered to convert 20% of the preferred stock to common stock on the basis of two shares of common for one share of preferred. The offer was accepted, and the conversion was made on July 1, 2008.

(f) No cash dividends were declared on common stock until December 31, 2006. Cash dividends per share of common stock were declared as follows:

 

June 30

Dec 31

2006

$319

2007

$175

275

2008

125

125

Instructions: Compute the following:

1. The number of shares of each class of stock outstanding on the last day of each year from 2004 through 2008.

2. Total cash dividends applicable to common stock for each year from 2006 through 2008.

 

accounting for stock transactions morris corporation is publicly owned and its share 692539

Accounting for Stock Transactions

Morris Corporation is publicly owned, and its shares are traded on a national stock exchange. Morris has 16,000 shares of $2 stated value common stock authorized. Only 75% of these shares have been issued, and of the shares issued, only 11,000 are outstanding. On December 31, 2007, the Stockholders’ Equity section revealed that the balance in Paid In Capital in Excess of Stated Value was $416,000, and the Retained Earnings balance was $110,000. Treasury stock was purchased at an average cost of $37.50 per share. During 2008, Morris had the following transactions:

Jan. 15 Morris issued, at $55 per share, 800 shares of $50 par, 5% cumulative preferred stock; 2,000 shares are authorized.

Feb. 1 Morris sold 1,500 shares of newly issued $2 stated value common stock at $42 per share.

Mar. 15 Morris declared a cash dividend on common stock of $0.15 per share, payable on April 30 to all stockholders of record on April 1.

Apr. 15 Morris reacquired 200 shares of its common stock for $43 per share. Morris uses the cost method to account for treasury stock.

30 Morris paid dividends.

30 Employees exercised 1,000 options granted in 2003 under a fixed stock option plan. When the options were granted, each option entitled the employee to purchase one share of common stock for $50 per share. The share price on the grant date was $51 per share. On April 30, when the market price was $55 per share, Morris issued new shares to the employees. The fair value of the options at the grant date was $6.

May 1 Morris declared a 10% stock dividend to be distributed on June 1 to stockholders of record on May 7.

The market price of the common stock was $55 per share on May 1 (before the stock dividend). (Assume that treasury shares do not participate in stock dividends.)

31 Morris sold 150 treasury shares reacquired on April 15 and an additional 200 shares costing $7,500 that had been on hand since the beginning of the year. The selling price was $57 per share.

June 1 Morris distributed the stock dividend.

Sept. 15 The semiannual cash dividend on common stock was declared, amounting to $0.15 per share. Morris also declared the yearly dividend on preferred stock. Both are payable on October 15 to stockholders of record on October 1.

Oct. 15 Morris paid dividends.

Net income for 2008 was $50,000. Assume that revenues and expenses were closed to a temporary account, Income Summary. Use this account to complete the closing process.

Instructions:

1. Compute the number of shares and dollar amount of treasury stock at the beginning of 2008.

2. Make the necessary journal entries to record the transactions in 2008 relating to stockholders’ equity.

3. Prepare the stockholders’ equity section of Morris Corporation’s December 31, 2008, balance sheet.

 

accounting for stock transactions ellis corporation was organized on june 30 2005 af 692540

Accounting for Stock Transactions

Ellis Corporation was organized on June 30,2005. After 2 1/2 years of profitable operations, the equity section of Ellis’s balance sheet was as follows:

Contributed capital:

 

Common stock, $3 par, 600,000 shares authorized,

 

200,000 shares issued and outstanding   

$600,000

Paid in capital in excess of par           

6,000,000

Retained earnings                       

2,800,000

Total stockholders’ equity                 

$9,400,000

During 2008, the following transactions affected stockholders’ equity:

Jan. 31 Reacquired 10,000 shares of common stock at $32; treasury stock is recorded at cost.

Apr. 1 Declared a 30% stock dividend. (Applies to all issued stock.)

30 Declared a $0.75 cash dividend. (Applies only to outstanding stock.)

June 1 Issued the stock dividend and paid the cash dividend.

Aug. 31 Sold all treasury stock at $35.

Instructions: Provide the journal entries to record the stock transactions.

stock dividend and cash dividend on january 1 2008 cozumel company had 100 000 share 692541

Stock Dividend and Cash Dividend

On January 1, 2008, Cozumel Company had 100,000 shares of $0.50 par value common stock outstanding. The market value of Cozumel’s common stock was $18 per share. Cozumel’s Retained Earnings balance on January 1 was $460,000. During 2007, Cozumel had declared and paid cash dividends of $0.75 per share. Net income for 2008 is expected to be $130,000. Cozumel has a large loan from McGraw Bank; part of the loan agreement stipulates that Cozumel must maintain a minimum Retained Earnings balance of $350,000. Cozumel’s board of directors is debating whether to declare a stock dividend in addition to its $0.75 per share annual cash dividend. Three proposals have been presented:

(1) no stock dividend, (2) a 10% stock dividend, and (3) a 25% stock dividend.

Instructions: As a shareholder in Cozumel Company, which of the three proposals do you favor? Support your answer.

stockholders rsquo equity transactions seneca inc was organized on january 2 2007 wi 692542

Stockholders’ Equity Transactions

Seneca Inc. was organized on January 2, 2007, with authorized capital stock consisting of 50,000 shares of 10%, $200 par value preferred, and 200,000 shares of no par, no statedvalue common. During the first two years of the company’s existence, the following selected transactions took place:

2007

Jan. 2 Sold 10,000 shares of common stock at $16.

2 Sold 3,000 shares of preferred stock at $216.

Mar. 2 Sold common stock as follows: 10,800 shares at $22; 2,700 shares at $25.

July 10 Acquired a nearby piece of land, appraised at $400,000, for 600 shares of preferred stock and 27,000 shares of common. (Preferred stock was recorded at $216, the balance being assigned to common.)

Dec. 16 Declared the regular preferred dividend and a $1.50 common dividend.

28 Paid dividends declared on December 16.

31 Assume that revenues and expenses were closed to a temporary account, Income Summary. The Income

Summary account showed a credit balance of $450,000, which was transferred to Retained Earnings.

2008

Feb. 27 Reacquired 12,000 shares of common stock at $19. The treasury stock is carried at cost. (State law requires that an appropriation of Retained Earnings be made for the purchase price of treasury stock. Appropriations are to be returned to Retained Earnings upon resale of the stock.)

June 17 Resold 10,000 shares of the treasury stock at $23.

July 31 Resold all of the remaining treasury stock at $18.

Sept. 30 Sold 11,000 additional shares of common stock at $21.

Dec. 16 Declared the regular preferred dividend and an $0.80 common dividend.

28 Dividends declared on December 16 were paid.

31 The income summary account showed a credit balance of $425,000, which was transferred to Retained Earnings.

Instructions:

1. Give the journal entries to record these transactions.

2. Prepare the Stockholders’ Equity section of the balance sheet as of December 31, 2008.

retained earnings and the statement of cash flows the following items relate to the 692544

Retained Earnings and the Statement of Cash Flows

The following items relate to the activities of Schmidt Company for 2008:

(a) Cash dividends declared and paid on common stock during the year totaled $90,000.

In addition, on January 15, 2008, dividends of $25,000 that were declared in 2007 were paid.

(b) Retained earnings of $145,000 were appropriated during the year in anticipation of a major capital expansion in future years.

(c) Depreciation expense was $59,000.

(d) Equipment was purchased for $215,000 in cash.

(e) Early in the year, a 10% stock dividend was declared and distributed. This stock dividend resulted in the distribution of 40,000 new shares of $1 par common stock. The market value per share immediately after the stock dividend was $55.

(f) Cash revenues for the year totaled $582,000.

(g) Cash expenses for the year totaled $305,000.

(h) Old machinery was sold for its book value of $20,000.

(i) Near the end of the year, a 2 for 1 stock split was declared. The 440,000 shares of $1 par common stock outstanding at the time were exchanged for 880,000 shares with a par value of $0.50.

(j) Cash dividends totaling $27,000 were declared and paid on preferred stock.

(k) Land was acquired in exchange for 5,000 shares of $0.50 par value common stock. The land had a fair market value of $170,000.

(l) Assume no changes in current operating receivable and payable balances during the year.

Instructions: Prepare a statement of cash flows for Schmidt Company for the year ended

December 31, 2008. Use the indirect method for reporting cash flows from operating activities.

reporting stockholders rsquo equity the stockholders rsquo equity section of nilsson 692545

Reporting Stockholders’ Equity

The Stockholders’ Equity section of Nilsson Corporation’s balance sheet as of December 31, 2007, is as follows:

Common stock ($5 par, 500,000 shares authorized,

275,000 issued and outstanding)

$1,375,000

 

Paid in capital pital in excess of par

550,000

 

Total paid in capital

 

$1,925,000

Unappropriated retained earnings

$1,335,000

 

Appropriated retained earnings

500,000

 

Total retained earnings

 

1,835,000

Total stockholders’ equity

 

$3,760,000

Nilsson Corporation had the following stockholders’ equity transactions during 2008:

Jan. 15 Completed the building renovation, for which $500,000 of retained earnings had been restricted. Paid the contractor $485,000, all of which is capitalized.

Mar. 3 Issued 100,000 additional shares of the common stock for $8 per share.

May 18 Declared a dividend of $1.50 per share to be paid on July 31, 2008, to stockholders of record on June 30, 2008.

June 19 Approved additional building renovation to be funded internally. The estimated cost of the project is $400,000, and retained earnings are to be restricted for that amount.

July 31 Paid the dividend.

Nov. 12 Declared a property dividend to be paid on December 31, 2008, to stockholders of record on

November 30, 2008. The dividend is to consist of 35,000 shares of Hampton Inc. stock that are currently recorded in Nilsson’s books at $9 per share. The fair market value of the stock on

November 12 is $13 per share.

Dec. 31 Reported $885,000 of net income on the December 31, 2008, income statement. (Assume that revenues and expenses were closed to a temporary account, Income Summary. Use this account to complete the closing process.) In addition, the stock was distributed in satisfaction of the property dividend.

The Hampton stock closed at $14 per share at the end of the day’s trading.

Instructions:

1. Make all necessary journal entries for Nilsson to account for the transactions affecting stockholders’ equity.

2. Prepare the December 31, 2008, Stockholders’ Equity section of the balance sheet for Nilsson.

auditing stockholders rsquo equity you have been assigned to the audit of belcore in 692546

Auditing Stockholders’ Equity

You have been assigned to the audit of Belcore Inc., a manufacturing company. You have been asked to summarize the transactions for the year ended December 31, 2008, affecting stockholders’ equity and other related accounts. The Stockholders’ Equity section of Belcore’s December 31, 2007, balance sheet follows:

Stockholders’ Equity

Contributed capital:

 

Common stock, $2 par value, 500,000 shares authorized,

 

90,000 shares issued, 88,790 shares outstanding                                   

$ 180,000

Paid in capital in excess of par                                                

1,820,000

Paid in capital from treasury stock                                             

22,500

Total contributed capital                                                    

 $2,022,500

Retained earnings                                                            

324,689

Total contributed capital and retained earnings                                    

 $2,347,189

Less: Cost of 1,210 shares of treasury stock                                        

72,600

Total stockholders’ equity                                                   

 $2,274,589

You have extracted the following information from the accounting records and audit working papers.

2008

Jan. 15 Belcore reissued 650 shares of treasury stock for $40 per share. The 1,210 shares of treasury stock on hand at December 31, 2007, were purchased in one block in 2007. Belcore used the cost method for recording the treasury shares purchased.

Feb. 2 Sold 90, $1,000, 9% bonds due February 1, 2011, at 103 with one detachable stock warrant attached to each bond. Interest is payable annually on February 1. The fair market value of the bonds without the stock warrants is 97. The detachable warrants have a fair value of $60 each and expire on February 1,

2009. Each warrant entitles the holder to purchase 10 shares of common stock at $40 per share.

Mar. 6 Subscriptions for 1,400 shares of common stock were issued at $44 per share, payable 40% down and the balance by March 20.

20 The balance due on 1,200 shares was received and those shares were issued. The subscriber who defaulted on the 200 remaining shares forfeited the down payment in accordance with the subscription agreement.

Nov. 1 There were 55 stock warrants detached from the bonds and exercised.

Instructions: Provide journal entries required to summarize the preceding transactions.

sample cpa exam questions 1 on january 2 2009 kine co granted morgan its president c 692547

Sample CPA Exam Questions

1. On January 2, 2009, Kine Co. granted Morgan, its president, compensatory stock options to buy 1,000 shares of Kine’s $10 par common stock. The options call for a price of $20 per share and are exercisable beginning on December 31, 2009. The options can be exercised any time during the three years beginning with this date. Morgan exercised the options on December 31, 2009. The market price of the stock was $40 on January 2, 2009, and $70 on December 31, 2009. The fair value of the options was $25. By what net amount should stockholders’ equity increase as a result of the grant and exercise of the options?

a. $20,000

b. $25,000

c. $30,000

d. $50,000

2. A company issued rights to its existing shareholders without consideration. The rights allowed the recipients to purchase unissued common stock for an amount in excess of par value. When the rights are issued, which of the following accounts will be increased?

 

 

Additional

 

Common Stock

Paid In Capital

a.

Yes

Yes

b.

Yes

No

c.

No

No

d.

No

Yes

3. If a corporation sells some of its treasury stock at a price that exceeds its cost, this excess should be

a. reported as a gain in the income statement.

b. treated as a reduction in the carrying amount of remaining treasury stock.

c. credited to Additional Paid In Capital.

d. credited to Retained Earnings

4. Which of the following should be reported as a stockholders’ equity contra account?

a. Discount on convertible bonds

b. Premium on convertible bonds

c. Cumulative foreign exchange translation loss

d. Organization costs

strategic conversion of preferred stock colter corporation suspended dividend paymen 692549

Strategic Conversion of Preferred Stock

Colter Corporation suspended dividend payments on all four classes of capital stock outstanding because of a downturn in the economy. The four classes of stock include 7% preferred stock, cumulative, $50 par; 5% preferred stock, noncumulative, convertible, $35 par; 9% preferred stock, noncumulative, $80 par; and common stock. Fifteen thousand shares of each class of stock were outstanding. Dividends had been paid through 2005.Colter did not pay dividends in 2006 or 2007. In 2008, the economy improved, and a proposal to pay a dividend of $1.50 per share of common stock was made. You own 100 shares of the 5%, noncumulative, convertible preferred stock and have been considering converting those 100 shares to common stock at the existing conversion rate of 3 to 1 (3 shares of common for 1 share of preferred). The rate is scheduled to Dr. op to 2 to 1 at the end of 2008. Because the price of common stock has been rising rapidly, you are trying to decide between retaining your preferred stock or converting to common stock before the price goes higher and the ratio is lowered. Assuming there is no conversion of preferred stock, how much cash does Colter need to pay the proposed dividend? What are the merits of converting your stock at this time as opposed to waiting until after the dividend is paid and the conversion ratio decreases? Explain the issues involved.

treasury stock transactions mdash you can rsquo t lose the following is adapted from 692552

Treasury Stock Transactions—You Can’t Lose!

The following is adapted from an article appearing in Forbes:

The board of HOSPITAL CORP. OF AMERICA authorized the buyback of 12 million of the firm’s own shares at a total cost of $564 million. However, after the stock market crash of 1987, HCA’s shares were trading at only 31 1/8. So, HCA was now out $190.5 million on its investment—right? Common sense would answer yes, but beyond common sense lurks the logic of accounting. According to generally accepted accounting principles, HCA didn’t lose a penny on the buyback. Call it a no risk investment. In this era of stock market volatility, stock buybacks offer firms the opportunity to tell shareholders that they have a terrific investment—without ever having to own up to the bad news if it turns sour. Consider the criticism in the preceding paragraph and evaluate the reasonableness of the accounting for treasury stock transactions.

how much should our dividend be largo corp has paid quarterly dividends of 0 70 per 692553

How Much Should Our Dividend Be?

Largo Corp. has paid quarterly dividends of $0.70 per share for the last three years and is trying to continue this tradition. Largo’s balance sheet is as follows:

Largo Corp Balance Sheet December 31, 2008

Assets

 

Liabilities

 

Current assets:

 

Current liabilities:

 

Cash

$ 50,000

Accounts payable

$ 520,000

Accounts receivable

450,000

Taxes payable

100,000

Inventory

1,200,000

Accrued liabilities

90,000

Total current assets

$1,700,000

Total current liabilities

$ 710,000

Investments

500,000

Bonds payable

1,500,000

Property, plant, and equipment (net)

1,600,000

Total liabilities

$2,210,000

 

 

Stockholders’ Equity

 

 

 

Common stock ($1 par, 69,000 shares

 

 

 

outstanding)

$ 69,000

 

 

Additional paid in capital

621,000

 

 

Retained earnings

900,000

 

 

Total stockholders’ equity

$1,590,000

Total assets

$3,800,000

Total liabilities and stockholders’ equity

$3,800,000

Largo’s net income in 2008 was $400,000. Should Largo continue its $0.70 per share quarterly dividend in the first quarter of 2009? Should Largo increase the cash dividend?

accumulated other comprehensive income refer to practice 13 20 compute the balance i 692502

Accumulated Other Comprehensive Income

Refer to Practice 13 20. Compute the balance in (1) Retained Earnings and (2) Accumulated

Other Comprehensive Income as of the end of each year: 2006, 2007, 2008.

Practice 13 20

Comprehensive Income

The company started business on January 1, 2006. Net income and dividends for the first three years of the company’s existence are as follows:

 

Net Income (Loss)

Dividends

2006

($1,000)

$0

2007

400

100

2008

1,700

300

The company has some foreign subsidiaries and also maintains a portfolio of available for sale securities. During 2006, 2007, and 2008, the U.S. dollar value of the equity of the foreign subsidiaries and the market value of the securities in the available for sale portfolio fluctuated as follows:

 

Change in U.S. Dollar Value

Change in Value of Portfolio

2006

Increase of $350

Decrease of $1,100

2007

Decrease of $800

Decrease of $600

2008

Decrease of $170

Increase of $420

Compute comprehensive income for each of the three years: 2006, 2007, and 2008.

international equity reserves the company based in the united kingdom has the follow 692503

International Equity Reserves

The company, based in the United Kingdom, has the following equity accounts:

Retained earnings                                                               

$1,000

Asset revaluation reserve                                                         

3,200

Par value of shares                                                               

100

Special reserve                                                                 

400

Share premium                                                                 

1,700 

Total equity                                                                  

$6,400  

Compute the amount of (1) no distributable and (2) distributable equity.

statement of changes in stockholders rsquo equity beginning balances in the equity a 692504

Statement of Changes in Stockholders’ Equity

Beginning balances in the equity accounts were as follows:

Common stock, at par            

$1,500

Paid in capital in excess of par      

10,000

Accumulated other comprehensive income

(2,200)

Retained earnings                

15,000

Treasury stock                  

(5,000)

Total stockholders’ equity       

$19,300

The following is true for the year:

(a) Net income was $4,500.

(b) Equity increased $300 from an increase in value of available for sale securities.

(c) Dividends were $1,000.

(d) Treasury stock of $1,200 was purchased. Assume the cost method.

(e) Shares of stock for $500 were issued. Par value was $40.

Prepare a statement of changes in stockholders’ equity for the year.

issuance of common stock verdero company is authorized to issue 100 000 shares of 2 692505

Issuance of Common Stock

Verdero Company is authorized to issue 100,000 shares of $2 par value common stock. Verdero has the following transactions:

(a) Issued 20,000 shares at $30 per share; received cash.

(b) Issued 250 shares to attorneys for services in securing the corporate charter and for preliminary legal costs of organizing the corporation. The value of the services was $9,000.

(c) Issued 300 shares, valued objectively at $10,000, to the employees instead of paying them cash wages.

(d) Issued 12,500 shares of stock in exchange for a building valued at $295,000 and land valued at $80,000. (The building was originally acquired by the investor for $250,000 and has $100,000 of accumulated depreciation; the land was originally acquired for $30,000.)

(e) Received cash for 6,500 shares of stock sold at $38 per share.

(f) Issued 4,000 shares at $45 per share; received cash.

Make the journal entries necessary for Verdero Company to record each transaction.

issuance of capital stock with subscriptions timpview company was incorporated on ja 692508

Issuance of Capital Stock with Subscriptions

Timpview Company was incorporated on January 1, 2008, with the following authorized capitalization:

• 20,000 shares of common stock, stated value $5 per share

• 5,000 shares of 7% cumulative preferred stock, par value $15 per share

Make the entries required for each of the following transactions:

(a) Issued 12,000 shares of common stock for a total of $672,000 and 3,000 shares of preferred stock at $20 per share.

(b) Subscriptions were received for 2,500 shares of common stock at a price of $52. A 30% down payment is received.

(c) Collected the remaining amount owed on the stock subscriptions and issued the stock.

(d) Sold the remaining authorized shares of common stock at $61 per share.

acquisition and retirement of stock marci company reported the following balances re 692509

Acquisition and Retirement of Stock

Marci Company reported the following balances related to common stock as of December 31, 2007:

Common stock, $1 par, 100,000 shares issued and outstanding

$ 100,000

Paid in capital in excess of par

1,800,000

The company purchased and immediately retired 8,000 shares at $24 on August 1, 2008, and 15,000 shares at $17 on December 31, 2008. Make the entries to record the acquisition and retirement of the common stock. (Assume all shares were originally sold at the same price.)

 

treasury stock par value and cost methods the stockholders rsquo equity of thomas co 692510

Treasury Stock: Par Value and Cost Methods

The stockholders’ equity of Thomas Company as of December 31, 2007, was as follows:

Common stock, $1 par, authorized 275,000 shares;

 

240,000 shares issued and outstanding 

$ 240,000

Paid in capital in excess of par

3,840,000

Retained earnings

900,000

On June 1, 2008, Thomas reacquired 15,000 shares of its common stock at $16. The following transactions occurred in 2008 with regard to these shares.

July 1 Sold 5,000 shares at $20.

Aug. 1 Sold 7,000 shares at $14.

Sept. 1 Retired 1,000 shares.

1. Using the cost method to account for treasury stock:

(a) Prepare the journal entries to record all treasury stock transactions in 2008.

(b) Prepare the stockholders’ equity section of the balance sheet at December 31, 2008, assuming Retained Earnings of $1,005,000 (before the effects of treasury stock transactions).

2. Using the par value method to account for treasury stock:

(a) Prepare the journal entries to record all treasury stock transactions in 2008.

(b) Prepare the Stockholders’ Equity section of the balance sheet at December 31, 2008, assuming Retained Earnings of $1,005,000 (before the effects of treasury stock transactions).

accounting for stock warrants western company wants to raise additional equity capit 692512

Accounting for Stock Warrants

Western Company wants to raise additional equity capital. After analysis of the available options, the company decides to issue 1,000 shares of $20 par preferred stock with detachable warrants. The package of the stock and warrants sells for $90. The warrants enable the holder to purchase 1,000 shares of $2 par common stock at $30 per share. Immediately following the issuance of the stock, the stock warrants are selling at $9 per share. The market value of the preferred stock without the warrants is $85.

1. Prepare a journal entry for Western Company to record the issuance of the preferred stock and the attached warrants.

2. Assuming that all the warrants are exercised, prepare a journal entry for Western to record the exercise of the warrants.

3. Assuming that only 70% of the warrants are exercised (and the remaining 30% lapse), prepare the journal entries for Western to record the exercise and expiration of the warrants.

accounting for a performance based stock option plan rhiener corporation initiated a 692514

Accounting for a Performance Based Stock Option Plan

Rhiener Corporation initiated a performance based employee stock option plan on January 1, 2007. The performance base for the plan is net sales in the year 2009. The plan provides for stock options to be awarded to the employees as a group on the following basis:

Level

Net Sales Range

Options Granted

1

<$250,000

10,000

2

$250,000–$499,999

20,000

3

$500,000–$1,000,000

30,000

4

>$1,000,000

40,000

The options become exercisable on January 1, 2010. The option exercise price is $20 per share. On January 1, 2007, each option had a fair value of $9. The market prices of Rhiener stock on selected dates in 2007–2009 were as follows:

January 1, 2007                                                                   

$25

December 31, 2007                                                                

30

December 31, 2008                                                                

35

December 31, 2009                                                                

32

Year 2009 sales estimates as of selected dates were as follows:

January 1, 2007                                                               

$400,000

December 31, 2007                                                            

450,000

December 31, 2008                                                            

550,000

Actual sales for 2009 were $700,000. Calculate the compensation expense Rhiener should report for the years 2007,2008, and 2009 related to this performance based stock option plan.

stock appreciation rights san juan corporation established a stock option plan that 692515

Stock Appreciation Rights

San Juan Corporation established a stock option plan that provides for cash payments to employees based on the appreciation of stock prices from an established option price. The plan was instituted on January 1, 2008, and provides benefits to employees who work for the succeeding three years. Cash payments to employees will be made on January 1, 2011, and will equal the excess of the stock price over the option price on that date. In total, 10,000 of these cash stock appreciation rights (SARs) were granted to employees. The option price established for the stock is $10 per share. The market price of San Juan stock on selected dates in 2008–2010 was as follows:

January 1, 2008                                                                   

$15

December 31, 2008                                                                

16

December 31, 2009                                                                 

20

December 31, 2010                                                                

18

Prepare the journal entries on San Juan’s books for the years 2008, 2009, 2010, and 2011 related to this plan.

convertible preferred stock stockholders rsquo equity for yuri co on december 31 was 692516

Convertible Preferred Stock

Stockholders’ equity for Yuri Co. on December 31 was as follows:

Preferred stock, $15 par, 30,000 shares issued and outstanding                            

$ 450,000

Paid in capital in excess of par—preferred stock                                      

90,000

Common stock, $10 par, 150,000 shares issued and outstanding                           

1,500,000

Paid in capital in excess of par—common stock                                       

750,000

Retained earnings                                                            

1,450,000

Preferred stock is convertible into common stock.

Provide the entry made on Yuri Co.’s books assuming that 4,000 shares of preferred are converted under each assumption listed:

1. Preferred shares are convertible into common on a share for share basis.

2. Each share of preferred stock is convertible into 4.0 shares of common.

3. Each share of preferred stock is convertible into 1.5 shares of common.

reporting errors from previous periods endicott company rsquo s december 31 2007 bal 692517

Reporting Errors from Previous Periods

Endicott Company’s December 31, 2007, balance sheet reported retained earnings of $86,500, and net income of $124,000 was reported in the 2007 income statement. While preparing financial statements for the year ended December 31, 2008, Tom Dr. yden, accountant for Endicott Company, discovered that net income for 2007 had been overstated by $36,000 due to an error in recording depreciation expense for 2007. Net income for 2008 was $106,000, and dividends of $30,000 were declared and paid in 2008.

1. What effect, if any, would the $36,000 error made in 2007 have on the company’s 2008 financial statements?

2. Compute the amount of retained earnings to be reported in Endicott Company’s December 31, 2008, balance sheet.

stock dividends the balance sheet of carmen corporation shows the following common s 692520

Stock Dividends

The balance sheet of Carmen Corporation shows the following:

Common stock, $1 stated value, 80,000 shares issued and outstanding

$ 80,000

Paid in capital in excess of stated value

1,120,000

Retained earnings

350,000

A 25% stock dividend is declared, with the board of directors authorizing a transfer from Retained Earnings to Common Stock at the stated value of the shares.

1. Provide entries to record the declaration and issuance of the stock dividend.

2. What was the effect of the issuance of the stock dividend on the ownership equity of each stockholder in the corporation?

3. Provide entries to record the declaration and issuance of the dividend if the board of directors had elected to declare a 15% stock dividend instead of 25%. The market value of the stock is $10 per share after the 15% stock dividend is issued.

stock dividends and stock splits the capital accounts for shop right market on june 692521

Stock Dividends and Stock Splits

The capital accounts for Shop Right Market on June 30, 2008, are as follows:

Common stock, $5 par, 40,000 shares issued and outstanding                             

$ 200,000

Paid in capital in excess of par                                                   

835,000

Retained earnings                                                            

2,160,000

Shares of the company’s stock are selling at this time at $22. What entries would you make in each of the following cases?

(a) A 10% stock dividend is declared and issued.

(b) A 50% stock dividend is declared and issued.

(c) A 2 for 1 stock split is declared and issued.

correcting the retained earnings account the retained earnings account for gotfried 692524

Correcting the Retained Earnings Account

The retained earnings account for Gotfried Corp. shows the following debits and credits. Indicate all entries required to correct the account. What is the corrected amount of retained earnings?

Account: Retained Earnings

Date

Item

Debit

Credit

Balance

Debit

Credit

Jan.

1

Balance

 

 

 

263,200

(a)

 

Loss from fire

2,625

 

 

260,575

(b)

 

Goodwill impairment

26,250

 

 

234,325

(c)

 

Stock dividend

70,000

 

 

164,325

(d)

 

Loss on sale of equipment

24,150

 

 

140,175

(e)

 

Officers’ compensation related to income of prior periods—

 

 

 

 

 

 

accrual overlooked

162,750

 

22,575

 

(f)

 

Loss on retirement of preferred shares at more than issuance price

35,000

 

57,575

 

(g)

 

Paid in capital in excess of par

 

64,750

 

7,175

(h)

 

Stock subscription defaults

 

4,235

 

11,410

(i)

 

Gain on retirement of preferred stock at less than issuance price

 

12,950

 

24,360

(j)

 

Gain on early retirement of bonds at less than book value

 

7,525

 

31,885

(k)

 

Gain on life insurance policy settlement

 

9,500

 

41,385

(l)

 

Correction of prior period error

 

25,025

 

66,410

equity adjustments the following data are for radial company contributed capital and 692525

Equity Adjustments

The following data are for Radial Company:

Contributed capital and retained earnings                                           

$875,000

Foreign currency translation adjustment                                            

72,000

Minimum pension liability adjustment                                              

86,000

Unrealized gain on available for sale securities                                        

95,000

Compute total stockholders’ equity for Radial Company.

analysis of owners rsquo equity from the following information reconstruct the journ 692526

Analysis of Owners’ Equity

From the following information, reconstruct the journal entries that were made by Rivers Corporation during 2008.

 

Dec 31, 2008

Dec 31, 2007

 

Amount

Shares

Amount

Shares

Common stock                                 

$175,000

7,000

$150,000

6,000

Paid in capital in excess of par                     

54,250

36,000

Paid in capital from treasury stock                  

1,000

200

Retained earnings                               

76,500*

49,000

Treasury stock                                 

15,000

300

reporting stockholders rsquo equity kenny co began operations on january 1 2007 by i 692527

Reporting Stockholders’ Equity

Kenny Co. began operations on January 1, 2007, by issuing at $15 per share one half of the 950,000 shares of $1 par value common stock that had been authorized for sale. In addition, Kenny has 500,000 shares of $5 par value, 6% preferred shares authorized. During 2007,Kenny had $1,025,000 of net income and declared $237,500 of dividends.

During 2008,Kenny had the following transactions:

Jan. 10 Issued an additional 100,000 shares of common stock for $17 per share.

Apr. 1 Issued 150,000 shares of the preferred stock for $8 per share.

July 19 Authorized the purchase of a custom made machine to be delivered in January 2009. Kenny restricted $295,000 of retained earnings for the purchase of the machine.

Oct. 23 Sold an additional 50,000 shares of the preferred stock for $9 per share.

Dec. 31 Reported $1,215,000 of net income and declared a dividend of $635,000 to stockholders of record on January 15, 2009, to be paid on February 1, 2009.

1. Prepare the stockholders’ equity section of Kenny’s balance sheet for December 31, 2007.

2. Prepare a statement of changes in stockholders’ equity for 2008.

3. Prepare the stockholders’ equity section of Kenny’s balance sheet for December 31, 2008.

journalizing stock transactions vicars company began operations on january 1 authori 692528

Journalizing Stock Transactions

Vicars Company began operations on January 1. Authorized were 20,000 shares of $1 par value common stock and 4,000 shares of 10%, $100 par value convertible preferred stock. The following transactions involving stockholders’ equity occurred during the first year of operations:

Jan. 1 Issued 500 shares of common stock to the corporation promoters in exchange for property valued at $17,000 and services valued at $7,000. The property had cost the promoters $9,000 three years before and was carried on the promoters’ books at $5,000.

Feb. 23 Issued 1,000 shares of convertible preferred stock with a par value of $100 per share. Each share can be converted to five shares of common stock. The stock was issued at a price of $150 per share, and the company paid $7,500 to an agent for selling the shares.

Mar. 10 Sold 3,000 shares of the common stock for $39 per share. Issue costs were $2,500.

Apr. 10 Sold 4,000 shares of common stock under stock subscriptions at $45 per share. No shares are issued until a subscription contract is paid in full. No cash was received.

July 14 Exchanged 700 shares of common stock and 140 shares of preferred stock for a building with a fair market value of $51,000. The building was originally purchased for $38,000 by the investors and has a book value of $22,000. In addition, 600 shares of common stock were sold for $24,000 in cash.

Aug. 3 Received payments in full for half of the stock subscriptions and payments on account on the rest of the subscriptions. Total cash received was $140,000. Shares of stock were issued for the subscriptions paid in full.

Dec. 1 Declared a cash dividend of $10 per share on preferred stock, payable on December 31 to stockholders of record on December 15, and a $2 per share cash dividend on common stock, payable on January 5 of the following year to stockholders of record on December 15. (No dividends are paid on unissued subscribed stock.)

31 Paid the preferred stock dividend.

31 Received notice from holders of stock subscriptions for 800 shares that they would not pay further on the subscriptions because the price of the stock had fallen to $25 per share. The amount still due on those contracts was $30,000. Amounts previously paid on the contracts are forfeited according to the agreements.

Net income for the first year of operations was $60,000. Assume that revenues and expenses were closed to a temporary account, Income Summary. Use this account to complete the closing process.

Instructions:

1. Prepare journal entries to record the preceding transactions on Vicars’ books.

2. Prepare the Stockholders’ Equity section of the balance sheet at December 31 for Vicars.

stockholders rsquo equity transactions and balance sheet presentation atlantic pacif 692529

Stockholders’ Equity Transactions and Balance Sheet Presentation

Atlantic Pacific Corporation was organized on September 1, 2008, with authorized capital stock of 150,000 shares of 7% cumulative preferred stock with a $40 par value and 1,200,000 shares of no par common stock with a $2 stated value. During the balance of the year, the following transactions relating to capital stock were completed:

Oct. 1 Received subscriptions for 200,000 shares of common stock at $39, payable $20 down and the balance in two equal installments due November 1 and December 1. On the same date, 17,800 shares of common stock were issued to Alan Williams in exchange for his business. Assets transferred to the corporation were valued as follows: land, $195,000; buildings, $216,000; equipment, $62,000; merchandise, $105,000. Liabilities of the business assumed by the corporation were mortgage payable, $46,000; accounts payable, $14,000; accrued interest on mortgage, $900. The fair value of the net assets is considered to be a reliable reflection of the value of the business; no goodwill is recognized. 3 Received subscriptions for 110,000 shares of preferred stock at $51, payable $21 down and the balance in two equal installments due November 1 and December 1.

Nov. 1 Collected amounts due on this date from all common and preferred stock subscribers.

12 Received subscriptions for 390,000 shares of common stock at $42, payable $20 down and the balance in two equal installments due December 1 and January 1.

Dec. 1 Collected amounts due on this date from all common stock and preferred stock subscribers and issued stock fully paid for.

Instructions:

1. Prepare journal entries to record these transactions.

2. Prepare the Contributed Capital section of stockholders’ equity for the corporation as of December 31, including any equity offsets.

sample cpa exam questions 1 on december 31 2009 moss co issued 1 000 000 of 11 bonds 692463

Sample CPA Exam Questions

1. On December 31, 2009, Moss Co. issued $1,000,000 of 11% bonds at 109. Each $1,000 bond was issued with 50 detachable stock warrants, each of which entitled the bondholder to purchase one share of $5 par common stock for $25. Immediately after issuance, the market value of each warrant was $4. On December 31, 2009, what amount should Moss record as discount or premium on issuance of bonds?

a. $40,000 premium

b. $90,000 premium

c. $110,000 discount

d. $200,000 discount

2. On July 31, 2009, Dome Co. issued $1,000,000 of 10%, 15 year bonds at par and used a portion of the proceeds to call its 600 outstanding 11%, $1,000 face value bonds, due on July 31, 2019, at 102. On that date, unamortized bond premium relating to the 11% bonds was $65,000. In its 2009 income statement, what amount should Dome report as gain or loss, before income taxes, from retirement of bonds?

a. $53,000 gain

b. $0

c. ($65,000) loss

d. ($75,000) loss

troubled debt restructuring mdash modification of terms volatile company after havin 692464

Troubled Debt Restructuring—Modification of Terms

Volatile Company, after having experienced financial difficulties in 2006, negotiated with two major creditors and arrived at an agreement to restructure its debts on December 31, 2006. The two creditors were M. Voisin and G. Stock. Voisin was owed principal of $325,000 and interest of $40,000 but agreed to accept equipment worth $70,000 and notes receivable from Volatile Company’s customers worth $275,000. The equipment had an original cost of $95,000 and accumulated depreciation of $35,000. Stock was owed $650,000 and agreed to extend the terms and to accept immediate payment of $200,000 and the remaining agreed upon balance of $477,403 to be paid on December 31, 2008. All payments were made according to schedule.

Instructions: Prepare Volatile’s journal entries to record the restructuring on December 31, 2006, and the entries necessary to make the adjustments and record payments on December 31, 2007, and 2008.

troubled debt restructuring mdash modification of terms in the latter part of 2007 o 692465

Troubled Debt Restructuring—Modification of Terms

In the latter part of 2007, Odessa Company experienced severe financial pressure and was in default of meeting interest payments on long term notes of $6,000,000 due on December 31, 2012. The interest rate on the debt was 11%, payable semiannually on June 30 and December 31. In an agreement with Modern Investment Corporation, Odessa obtained acceptance of a change in principal and interest terms for the remaining 5 year life of the notes. The changes in terms are as follows:

(a) A reduction of principal of $475,000.

(b) A reduction in the interest rate to 8%.

(c) Odessa agreed to pay on December 31, 2007, both the $660,000 of interest in arrears and the normal interest payment under the old terms.

Instructions:

1. Compute the total dollar difference in cash payments by Odessa over the 5 year period as a result of the restatement of terms.

2. Prepare the journal entries for the restructuring of the debt, payment of interest under the old terms, and the first two interest payments under the new terms that Odessa would make.

leave my current ratio alone soto inc a closely held corporation has never been audi 692468

Leave My Current Ratio Alone!

Soto Inc., a closely held corporation, has never been audited and is seeking a large bank loan for plant expansion. The bank has requested audited financial statements. In conference with the president and majority stockholder of Soto, the auditor is informed that the bank looks very closely at the current ratio. The auditor’s proposed reclassifications and adjustments include the following:

(a) A note payable issued 41⁄2 years ago matures in six months from the balance sheet date.

The auditor wants to reclassify it as a current liability. The controller says no because “we are probably going to refinance this note with other long term debt.”

(b) An accrual for compensated absences. Again the controller objects because the amount of the pay for these absences cannot be estimated. “Some employees quit in the first year and don’t get vacation, and it is impossible to predict which employees will be absent for illness or other causes. Without being able to identify the employees, we can’t determine the rate of compensation.”

If you were the auditor, how would you respond to the controller?

accounting for bonds startup company decided to issue 100 000 worth of 10 5 year bon 692469

Accounting for Bonds

Startup Company decided to issue $100,000 worth of 10%, 5 year bonds dated January 1, 2007, with interest payable semiannually on January 1 and July 1 of each year. Due to printing and other delays, Startup was not able to sell the bonds until July 1, 2007. The bonds were sold to yield 12% interest, and they are callable at 102 after January 1, 2009. The company expects interest rates to fall during the next few years and is planning to retire this bond issue and to replace it with a less costly one if the expected decline occurs. Assume that you have just been hired as the accountant for Startup Company. The financial vice president would like you to identify the accounting issues involved with the bond transaction. You are also asked to explain why the company received less than $100,000 on the sale of the bonds and to compute the anticipated gain or loss on retirement of the bonds, assuming retirement on July 1, 2009, and use of straight line amortization.

disaster bonds natural disasters occur all too often californians worry about earthq 692470

Disaster Bonds

Natural disasters occur all too often. Californians worry about earthquakes. Residents of Florida worry about hurricanes. Folks along the Mississippi River worry about flooding. The Midwest has its twisters, and the Rocky Mountain states have wildfires. Insurance companies worry about them all. In simple terms, insurance companies make money by charging customers premiums that exceed the amount expected to be paid out in claims. What are insurance companies doing? They are spreading the risks and costs across many people. If your home is lost in a fire and you are not insured, you are responsible for paying to have your home rebuilt. If you are insured, all the policyholders of your insurance company chip in, in effect, to rebuild your house. In the case of a mega disaster, there is a risk that insurance companies will not have the resources to cover all losses of policyholders. The insurance industry estimates that a worst case disaster would result in $50 billion in losses—enough to force many insurance companies out of business. If a disaster of this magnitude were to occur, many insurance companies wouldn’t have enough policyholders over whom to spread the losses. So how do insurance companies deal with the enormous risks associated with “acts of God”? Disaster bonds! Disaster bonds are a relatively new invention. These bonds allow insurance companies to share the risks of mega disasters with bondholders. In August 1996, Merrill Lynch & Co. began marketing the first major “act of God” bond issue. The bonds are issued by USAA, a car and home insurer based in San Antonio. These are the terms of the bonds: If USAA incurs over $1 billion in hurricane claims from a single storm over a 1 year period, investors in the disaster bonds will lose both interest and principal payments. Thus, if a huge hurricane hits the East Coast and claims from policyholders of USAA exceed $1 billion, USAA can use the money it would have paid to bondholders to pay policyholders. USAA is trying to do what insurance companies do best—spread the risk. While yields for traditional bonds were around 8% in August 1996, the expected yield on disaster bonds was around 15%. Why do you think there is such a high yield on disaster bonds?

is there a loss on conversion holton co recently issued 1 000 000 face value 8 30 ye 692471

Is There a Loss on Conversion?

Holton Co. recently issued $1,000,000 face value, 8%, 30 year debentures at 97. The debentures are callable at 103 upon 30 days’ notice by the issuer at any time beginning five years after the date of issue. The debentures are convertible into $1 par value common stock of the company at the conversion price of $12.50 per share for each $500 or multiple thereof of the principal amount of the debentures ($500/$12.50 = 40 shares for each $500 of face value). Assume that no value is assigned to the conversion feature at the date of issue of the debentures. Assume further that five years after issue, debentures with a face value of $100,000 and book value of $97,500 are tendered for conversion on an interest payment date when the market price of the debentures is 104 and the common stock is selling at $14 per share. J. K. Biggs, the company accountant, records the conversion as follows:

Bonds Payable                                                       

100,000

 

Discount on Bonds Payable                                           

 

2,500

Common Stock                                                   

 

8,000

Paid In Capital in Excess of Par                                        

 

89,500

Julie Robinson, staff auditor for the company’s CPA firm, reviews the transaction and feels the conversion entry should reflect the market value of the stock. According to Robinson’s analysis, a loss on the bond conversion of $14,500 should be recognized. Biggs objects to recognizing a loss, so Robinson discusses the problem with the audit manager, K. Ashworth. Ashworth has a different view and recommends using the market value of the debentures as a basis for recording the conversion and recognizing a loss of only $6,500. Evaluate the various positions. Include in your evaluation the substitute entries that would be made under both Robinson’s and Ashworth’s proposals.

deferred interest and interest rate resets corporations commonly incur debt in finan 692472

Deferred Interest and Interest Rate Resets

Corporations commonly incur debt in financing the acquisition of other companies or in fighting takeover attacks by competitors. Two strategies often employed involve deferring interest payments and incorporating interest rate resets. For example, Interco Inc. incurred large amounts of debt in 1989 to make itself unattractive as a takeover target. The debt postponed interest payments until 1991 at which time interest was to be paid at 14%. Interco’s strategy was to sell a portion of its business, Ethan Allen Inc., to redeem the debt.

However, the sale netted $120 million less than expected. Western Union incurred $500 million in debt that carried with it a reset provision. The provision called for increased interest rates if the bonds were not trading at a specified price. Western Union’s reset provision increased interest rates from 16.5% to 19.25% in 1990. While interest expense rose, revenues Dr.opped 28% from 1988 to 1989 as a result of fax machines making Western Union’s telex service obsolete.

1. What is the significance of debt with respect to company acquisitions?

2. Why would corporations use deferred interest features and interest rate resets?

3. In the case of Interco, how would incurring large amounts of debt be an effective method for fighting a takeover?

circle k corporation and its debt covenants when companies raise money through the i 692473

Circle K Corporation and Its Debt Covenants

When companies raise money through the issuance of bonds or other long term debt instruments, debt holders typically require the company to comply with certain conditions, or covenants. The notes to Circle K’s 1989 financial statements provide an example of debt covenants: The notes (Senior Secured Notes) required the Company to observe certain financial covenants, including covenants relating to maintenance of a minimum consolidated net worth, a fixed charge coverage ratio, limitations on dividends, purchases of capital stock and a requirement that any successor by merger or similar transaction to the Company have a comparable net worth and assume all the obligations under the notes. In addition to using debt to finance expansion, Circle K financed many of its store acquisitions through sales and leaseback transactions. These types of transactions represent a form of long term debt financing and often involve covenants as well. The notes to the 1989 financial statements detail the results of a violation of covenants: As of April 30, 1989, the Company was not in compliance with the fixed charge ratio of one of its sale and leaseback transactions involving 250 stores. Because of its noncompliance with such ratio, the Company is required to place $5 million per year into escrow.

1. What is the purpose of debt covenants?

2. What is the purpose of requiring an annual $5 million payment into escrow?

3. If Circle K’s financial condition is such that it violates its financing covenants, will requiring the company to place $5 million in escrow help to ease the financial strains?

in substance defeasance another form of early extinguishment of debt is referred to 692476

In Substance Defeasance

Another form of early extinguishment of debt is referred to as in substance defeasance, or economic defeasance. In substance defeasance is a process of transferring assets, generally cash and securities, to an irrevocable trust, and using the assets and earnings therefrom to satisfy the long term obligations as they come due. In some instances, the debt holders are not aware of these transactions and continue to rely on the issuer of the debt for settlement of the obligation. In other words, there has been no “legal defeasance” or release of the debtor from the legal liability. Before FASB Statement No. 125 was issued in 1996, an in substance defeasance was treated as an extinguishment of debt even though the debt is not actually repaid. The provisions of Statement No. 125 (and its successor, Statement No. 140) no longer allow debt to be removed from the balance sheet through in substance defeasance. Under Statement No. 140, what conditions must be satisfied for debt to be removed from the balance sheet? In what way do these conditions stop the use of in substance defeasance as a way to remove debt from the balance sheet?

do we really have income jefferson corporation has 20 000 000 of 10 bonds outstandin 692477

Do We Really Have Income?

Jefferson Corporation has $20,000,000 of 10% bonds outstanding. Because of cash flow problems, the company is behind in interest payments and in contributions to its bonds retirement fund. The market value of the bonds has declined until it is currently only 50% of the face value of the bonds. After lengthy negotiations, the principal bondholders have agreed to exchange their bonds for preferred stock that has a current market value of $10,000,000. The accountant for Jefferson Corporation recorded the transaction by charging Bond Liability for the entire $20,000,000 and crediting Preferred Stock for the same amount. This entry thus transfers the amount received by the company from debt to equity. The CPA firm performing the annual audit, however, does not agree with this treatment. The auditors argue that this transfer represents a troubled debt restructuring due to the significant concessions made by the bondholders, and under these conditions, the FASB requires Jefferson to use the market value of the preferred stock as its recorded value. The difference between the $20,000,000 face value of the bonds and the $10,000,000 market value of the preferred stock is a reportable gain. The controller of Jefferson, L. Rogers, is flabbergasted. “Here we are, almost bankrupt, and you tell us we must report the $10,000,000 as a gain. I don’t care what the FASB says; that’s a ridiculous situation. You can’t be serious.”

The auditor in charge of the engagement is adamant. “We really have no choice. You have had a forgiveness of debt for $10,000,000.You had use of the money, and based on current conditions, you won’t have to pay it back. That situation looks like a gain to me.” What position do you think should be taken? Consider the external users of the financial statements and their needs in your discussion.

deciphering financial statements boston celtics in december of 2002 the boston celti 692479

Deciphering Financial Statements (Boston Celtics)

In December of 2002, the Boston Celtics were purchased by a private investment group. Now that the Celtics are owned by a private group, their financial statements are not publicly available. However, prior to their going private, their financial statements were publicly available. A portion of those financial statements (the Liabilities and Equity section of the balance sheet) is shown below. Celtics Basketball Holdings Limited Partnership was the name of the entity under which the results of the Boston Celtics were reported prior to their going private. Review the Liabilities and Equity section of the balance sheet and answer the following questions.

1. What is Deferred Game Revenues? How would that liability have arisen?

2. What does the account Deferred Compensation represent? Note that this account has both a current and noncurrent portion.

3. As of June 30, 2001, what did the Celtics report as total assets?

4. Consider your answer to part (3) in light of the $50,000,000 amount of outstanding notes payable. If you were a creditor of the Celtics, would you be concerned? How is it possible for the organization to continue to function with such a large partners’ deficit?

Celtics Basketball Holdings, LP Liabilities and Partners’ Capital (Deficit)

 

June 30, 2001

June 30, 2000

CURRENT LIABILITIES

 

 

Accounts payable and accrued expenses                       

$ 23,506,664

$ 24,478,303

Deferred game revenues                                   

6,498,726

9,204,607

Deferred compensation—current portion                      

1,226,316

1,278,410

TOTAL CURRENT LIABILITIES                                

 $ 31,231,706

 $ 34,961,320

NOTES PAYABLE TO BANK                                  

50,000,000

50,000,000

DEFERRED COMPENSATION—noncurrent portion               

5,182,821

6,369,646

OTHER NON CURRENT LIABILITIES                          

 

708,000

PARTNERS’ CAPITAL (DEFICIT)

 

 

Celtics Basketball Holdings, LP:

 

 

General Partner                                        

1,015

1,008

Celtics Pride GP—Limited Partner                          

(29,111,174)

(29,437,209)

Castle Creek Partners, LP—Limited Partner                  

(31,144,430)

(31,493,235)

 

 (60,254,589)

 (60,929,436)

Celtics Basketball, LP—General Partner                     

1,081

1,074

TOTAL PARTNERS’ CAPITAL (DEFICIT)                         

 $(60,253,508)

 $(60,928,362)

deciphering financial statements hewlett packard amp dell review the 2004 balance sh 692480

Deciphering Financial Statements (Hewlett Packard & Dell)

Review the 2004 balance sheet data for Hewlett Packard (HP) and Dell shown below.

(in millions)

Hewlett Packard

Dell

Current assets                                                   

$42,901

$16,897

Current liabilities                                                 

28,588

14,136

Total liabilities                                                   

38,574

16,730

Total stockholders’ equity                                          

37,564

6,485

Retained earnings                                                 

15,649

9,174

1. Compute each company’s current ratio for 2004. Based on the result, which company appears to be more liquid?

2. Compute each company’s debt to equity ratio for 2004. Which company appears to have the most debt in relation to stockholders’ equity?

3. Which company has a larger amount of long term debt in its financing mix?

4. Why would HP have such a large amount in retained earnings at the end of 2004 relative to Dell?

writing assignment i like these ldquo no interest rdquo bonds j r chump president of 692483

Writing Assignment (I like these “no interest” bonds.)

J. R. Chump, president of ProKeeper Industries, is contemplating the issuance of long term debt to finance plant expansion and renovation. J. R. Chump, president of ProKeeper Industries, is contemplating the issuance of long term debt to finance plant expansion and renovation. In the past, his company has issued traditional debt instruments that require regular interest payments and a retirement of the principal on the maturity date. However, he has noticed that several competitors have recently issued bonds that either do not require interest payments or defer interest payments for several years. He has asked you, his chief financial officer, to prepare a short memo adDr.essing the following questions.

1. Why would a company issue bonds that require interest payments if bonds that do not require interest payments are being sold in the open market?

2. If the company were to issue 10 year bonds with a face value of $100,000 and the market rate of interest is 10%, what would be the proceeds from the sale if the bonds were zero interest bonds? What would be the proceeds if the annual interest payments did not begin for 5 years and the stated rate of interest were 10%? What would be the proceeds if the bonds paid interest annually for 10 years at 10%?

3. What factors must a business consider when determining the interest terms associated with long term debt?

researching accounting standards to help you become familiar with the accounting sta 692484

Researching Accounting Standards

To help you become familiar with the accounting standards, this case is designed to take you to the FASB’s Web site and have you access various publications.

In this chapter, we have discussed off balance sheet financing and recent changes in the accounting for one of the more common methods of off balance sheet financing, variable interest entities. For this case, we will use Interpretation 46(R), “Consolidation of

Variable Interest Entities—An Interpretation of ARB No. 51. Open FIN 46(R).

1. Read paragraph 2. What does the term variable interests mean?

2. Read paragraph 23. What additional disclosure is required by the primary beneficiary of a variable interest entity?

ethical dilemma keeping our debt covenants you are the chief financial officer of a 692485

Ethical Dilemma (Keeping our debt covenants)

You are the chief financial officer of a local manufacturing company, Larsen Enterprises. This company is run by two brothers, Steve and John Larsen. The Larsen brothers have built this company up from a small 5 man shop to a company now employing over 200 people. The national economy has recently taken a turn for the worse, which has affected the Larsen’s business. In fact, the company’s performance of late has been such that it is in jeopardy of violating several of its debt covenants (promises made to the lending institution). If the company violates these covenants, the bank has the option of calling the debt due immediately. If the debt is called, Larsen is not sure what will happen, but it will certainly not be good. The covenant that is in jeopardy relates to the current ratio. If the current ratio Dr.ops below 2, Larsen Enterprises is considered in technical default on its debt. Steve and John have come to you and asked you to suggest ways in which the current ratio, which currently stands at 1.9, could be increased. Take a moment and think of ways in which the current ratio might be manipulated. Identify specific actions that the Larsen brothers might take to increase the current ratio. Is it in the best interests of shareholders and lending institutions for Steve and John to make business decisions that have cosmetic effects on the financial statements?

cumulative spreadsheet analysis this spreadsheet assignment is a continuation of the 692486

Cumulative Spreadsheet Analysis

This spreadsheet assignment is a continuation of the spreadsheet assignments given in earlier chapters. If you completed those assignments, you have a head start on this one. Refer back to the instructions for preparing the revised financial statements for 2008 as given in (1) of the Cumulative Spreadsheet Analysis assignment in Chapter 3.

1. Skywalker wishes to prepare a forecasted balance sheet, a forecasted income statement, and a forecasted statement of cash flows for 2009. Use the financial statement numbers for 2008 as the basis for the forecast, along with the following additional information.

(a) Sales in 2009 are expected to increase by 40% over 2008 sales of $2,100.

(b) In 2009, new property, plant, and equipment acquisitions will be in accordance with the information in (q).

(c) The $480 in operating expenses reported in 2008 breaks down as follows: $15 in depreciation expense and $465 in other operating expenses.

(d) New long term debt will be acquired in 2009 in accordance with (u).

(e) No cash dividends will be paid in 2009.

(f) New short term loans payable will be acquired in an amount sufficient to make Skywalker’s current ratio in 2009 exactly equal to 2.0.

(g) Skywalker does not anticipate repurchasing any additional shares of stock during 2009.

(h) Because changes in future prices and exchange rates are impossible to predict, Skywalker’s best estimate is that the balance in accumulated other comprehensive income will remain unchanged in 2009.

(i) In the absence of more detailed information, assume that the balances in Investment Securities, Long Term Investments, and Other Long Term Assets will all increase at the same rate as sales (40%) in 2009.The balance in Intangible Assets will change in accordance with item (r).

(j) In the absence of more detailed information, assume that the balance in the other long term liabilities account will increase at the same rate as sales (40%) in 2009.

(k) The investment securities are classified as available for sale securities. Accordingly, cash from the purchase and sale of these securities is classified as an investing activity.

(l) Assume that transactions impacting other long term assets and other long term liabilities accounts are operating activities.

(m) Cash and investment securities accounts will increase at the same rate as sales.

(n) The forecasted amount of accounts receivable in 2009 is determined using the forecasted value for the average collection period. The average collection period for 2009 is expected to be 14.08 days. To make the calculations less complex, this value of 14.08 days is based on forecasted end of year accounts receivable rather than on average accounts receivable.

(o) The forecasted amount of inventory in 2009 is determined using the forecasted value for the number of days’ sales in inventory. The number of days’ sales in inventory for 2009 is expected to be 107.6 days. To make the calculations simpler, this value of 107.6 days is based on forecasted end of year inventory rather than on average inventory.

(p) The forecasted amount of accounts payable in 2009 is determined using the forecasted value for the number of days’ purchases in accounts payable. The number of days’ purchases in accounts payable for 2009 is expected to be 48.34 days. To make the calculations simpler, this value of 48.34 days is based on forecasted end of year accounts payable rather than on average accounts payable.

(q) The forecasted amount of property, plant, and equipment (PP&E) in 2009 is determined using the forecasted value for the fixed asset turnover ratio. The fixed asset turnover ratio for 2009 is expected to be 3.518 times. To make the calculations simpler, this ratio of 3.518 is based on forecasted end of year gross property, plant, and equipment balance rather than on the average balance.

(r) Skywalker has determined that no new intangible assets will be acquired in 2009. Intangible assets are amortized according to the information in (t).

(s) In computing depreciation expense for 2009, use straight line depreciation and assume a 30 year useful life with no residual value. Gross PP&E acquired during the year is only depreciated for half the year. In other words, depreciation expense for 2009 is the sum of two parts: (1) a full year of depreciation on the beginning balance in PP&E, assuming a 30 year life and no residual value, and (2) a half year of depreciation on any new PP&E acquired during the year, based on the change in the gross PP&E balance.

(t) Skywalker assumes a 20 year useful life for its intangible assets. Assume that the $100 in intangible assets reported in 2008 is the original cost of the intangibles. Include the amortization expense with the depreciation expense in the income statement.

For this exercise, make the following additional assumptions:

(u) New long term debt will be acquired (or repaid) in an amount sufficient to make

Skywalker’s debt ratio (total liabilities divided by total assets) in 2009 exactly equal to 0.80.

(v) Assume an interest rate on short term loans payable of 6.0% and on long term debt of 8.0%. Only a half year’s interest is charged on loans taken out during the year.

For example, if short term loans payable at the end of 2009 is $15 and given that short term loans payable at the end of 2008 were $10, total short term interest expense for 2009 would be $0.75 [($10 x 0.06) + ($5 x 0.06 x 1/2)]. Clearly state any additional assumptions that you make.

2. Repeat (1) with the following changes in assumptions.

(a) The debt ratio in 2009 is exactly equal to 0.70.

(b) The debt ratio in 2009 is exactly equal to 0.90.

3. Comment on the differences in the forecasted values of cash from operating activities in 2009 under each of the following assumptions about the debt ratio: 0.70, 0.80, and 0.90. Explain exactly why a change in debt ratio has an impact on cash from operating activities.

accounting for a basic stock based compensation plan on january 1 the company grante 692491

Accounting for a Basic Stock Based Compensation Plan

On January 1, the company granted 100,000 stock options to key employees. Each option allows an employee to buy one share of $1 par common stock for $30, which was the market price of the shares on the grant date of January 1. In order to be able to exercise the options, the employees must remain with the company for three entire years. It is estimated that the fair value of each option on the date of grant was $3. At the end of three years, all of the options were exercised when the market price of the shares was $42 per share. Make all of the journal entries necessary with respect to these options in the first year. Also make the journal entry that would be made at the end of three years to record the exercise of the options.

accounting for a written put option on january 1 year 1 the company wrote a put opti 692495

Accounting for a Written Put Option

On January 1,Year 1, the company wrote a put option agreeing to purchase 100 shares of its own stock for $50 per share on December 31,Year 2, at the option of the purchaser of the put option. The market price of the company’s shares on January 1, Year 1, was $50 per share. As of January 1, Year 1, this put option has a fair value of $1,200. Because the company’s shares increased in value during Year 1, the put option has a fair value of just $350 on December 1, Year 1. On December 31, Year 2, the company’s shares have a market price of $46 per share, so the purchaser of the put option exercised the option on that date. Make the journal entries necessary on January 1, Year 1, on December 31, Year 1, and on December 31, Year 2 on the books of the company that wrote the put option.

comprehensive income the company started business on january 1 2006 net income and d 692501

Comprehensive Income

The company started business on January 1, 2006. Net income and dividends for the first three years of the company’s existence are as follows:

 

Net Income (Loss)

Dividends

2006

($1,000)

$0

2007

400

100

2008

1,700

300

The company has some foreign subsidiaries and also maintains a portfolio of available forsale securities. During 2006, 2007, and 2008, the U.S. dollar value of the equity of the foreign subsidiaries and the market value of the securities in the available for sale portfolio fluctuated as follows:

 

Change in U.S. Dollar Value

Change in Value of Portfolio

2006

Increase of $350

Decrease of $1,100

2007

Decrease of $800

Decrease of $600

2008

Decrease of $170

Increase of $420

Compute comprehensive income for each of the three years: 2006, 2007, and 2008.

working capital and current ratio using the following information compute 1 working 692414

Working Capital and Current Ratio

Using the following information, compute (1) working capital and (2) current ratio.

Deferred sales revenue                                                           

$ 900

Accounts payable                                                               

1,100

Accounts receivable                                                             

1,750

Cash                                                                        

400

Sales                                                                        

10,000

Accrued wages payable                                                           

250

Sales returns and allowances                                                       

700

Bonds payable (to be repaid in 6 months)                                             

1,000

Bonds payable (to be repaid in 5 years)                                                

4,000

debt to equity ratio consider the following information 692423

Debt to Equity Ratio

Consider the following information:

Short term debt                                                              

$ 10,000

Interest expense                                                              

7,500

Total current liabilities                                                           

25,000

Long term debt                                                               

70,000

Cash                                                                       

2,700

Total liabilities                                                                

120,000

Total stockholders’ equity                                                        

90,000

Income before income taxes                                                       

12,000

Compute the debt to equity ratio assuming that (1) debt is defined to include all liabilities, (2) debt is defined to include just interest bearing debt, and (3) debt is defined to include just long term, interest bearing debt.

times interest earned ratio refer to practice 12 17 compute the times interest earne 692424

Times Interest Earned Ratio

Refer to Practice 12 17. Compute the times interest earned ratio.

Practice 12 17

Debt to Equity Ratio

Consider the following information:

Short term debt                                                              

$ 10,000

Interest expense                                                              

7,500

Total current liabilities                                                          

25,000

Long term debt                                                               

70,000

Cash                                                                       

2,700

Total liabilities                                                                

120,000

Total stockholders’ equity                                                        

90,000

Income before income taxes                                                      

12,000

Compute the debt to equity ratio assuming that (1) debt is defined to include all liabilities, (2) debt is defined to include just interest bearing debt, and (3) debt is defined to include just long term, interest bearing debt.

bond interest and premium or discount amortization assume that 200 000 of baker scho 692435

Bond Interest and Premium or Discount Amortization

Assume that $200,000 of Baker School District 6% bonds are sold on the bond issue date for $185,788. Interest is payable semiannually, and the bonds mature in 10 years. The purchase price provides a return of 7% on the investment.

1. What entries would be made on the investor’s books for the receipt of the first two interest payments, assuming premium or discount amortization on each interest date by (a) the straight line method and (b) the effective interest method? (Round to the nearest dollar.)

2. What entries would be made on Baker School District’s books to record the first two interest payments, assuming premium or discount amortization on each interest date by (a) the straight line method and (b) the effective interest method? (Round to the nearest dollar.)

discount and premium amortization tanzanite corporation issued 500 000 of 7 debentur 692436

Discount and Premium Amortization

Tanzanite Corporation issued $500,000 of 7% debentures to yield 11%, receiving $424,624.

Interest is payable semiannually, and the bonds mature in five years.

1. What entries would be made by Tanzanite for the first two interest payments, assuming premium or discount amortization on interest dates by (a) the straight line method and

(b) the effective interest method? (Round to the nearest dollar.)

2. What entries would be made on the books of the investor for the first two interest receipts, assuming premium or discount amortization on interest dates and that one party obtained all the bonds and used the straight line method of amortization? (Round to the nearest dollar.)

3. If the sale is made to yield 5%, $543,760 being received, what entries would be made by Tanzanite for the first two interest payments, assuming premium or discount amortization on interest dates by (a) the straight line method and (b) the effective interest method? (Round to the nearest dollar.)

retirement of bonds the december 31 2007 balance sheet of spring company includes th 692439

Retirement of Bonds

The December 31, 2007, balance sheet of Spring Company includes the following items:

8% bonds payable due December 31, 2014

$200,000

Premium on bonds payable

8,750

The bonds were issued on December 31, 2006, at 105, with interest payable on June 30 and December 31 of each year. The straight line method is used for premium amortization. On April 1, 2008, Spring retired $100,000 of these bonds at 99 plus accrued interest. Prepare the journal entries to record retirement of the bonds, including accrual of interest since the last payment and amortization of the premium.

convertible bonds clarkston inc issued 1 000 000 of convertible 10 year 11 bonds on 692442

Convertible Bonds

Clarkston Inc. issued $1,000,000 of convertible 10 year, 11% bonds on July 1, 2007. The interest is payable semiannually on January 1 and July 1. The discount in connection with the issue was $9,500, which is amortized monthly using the straight line basis. The debentures are convertible after one year into five shares of the company’s $1 par common stock for each $1,000 of bonds. On August 1, 2008, $100,000 of the bonds were converted. Interest has been accrued monthly and paid as due. Any interest accrued at the time of conversion of the bonds is paid in cash. Prepare the journal entries on Clarkston’s books to record the conversion, amortization, and interest on the bonds as of August 1 and August 31, 2008. (Round to the nearest dollar.)

modification of debt terms moriarty co is experiencing financial difficulties income 692445

Modification of Debt Terms

Moriarty Co. is experiencing financial difficulties. Income has exhibited a downward trend, and the company reported its first loss in company history this past year. The firm has been unable to service its debt and, as a result, has missed two semiannual interest payments. In an attempt to turn the company around, management has negotiated a modification of its debt terms with bondholders. These modified terms are effective January 1, 2008. The bonds are $10,000,000, 10 year, 10% bonds that were issued on January 2, 2003, and currently have an unamortized premium of $210,000. Prepare the necessary journal entries on Moriarty’s books for each of the following independent situations.

(a) Bondholders agree to forgive past due interest and reduce the interest rate on the debt from 10% to 5%.

(b) Bondholders agree to forgive past due interest and forgive $3,000,000 of the face amount of the debt.

(c) Bondholders agree to forgive past due interest, reduce the interest rate on the debt from 10% to 6%, and forgive $2,000,000 of the face value of the debt.

amortizing a mortgage and the effect on the financial statements on january 1 2008 p 692447

Amortizing a Mortgage and the Effect on the Financial Statements

On January 1, 2008, Picard Inc. purchased a new piece of equipment from La Forge Engineering to expand its production facilities. The equipment was purchased at a cost of $800,000. Picard financed the purchase with an $800,000 mortgage to be repaid in annual payments over five years at a rate of 10%. The mortgage was arranged through Pulaski Bank. The annual payments of $211,038 are to be made on December 31 of each year.

Instructions:

1. Prepare a mortgage amortization schedule for the 5 year life of the mortgage.

2. Assuming the equipment is expected to last for five years (with zero salvage value), determine the net amount at which the equipment will be reported on the balance sheet at the end of each year for its 5 year life using straight line depreciation.

3. Compare the liability amount to be disclosed on the balance sheet at the end of each year for the 5 year mortgage term with the asset amount to be disclosed at the end of the same years. Identify the primary reasons for the differences each year.

bond issuance and adjusting entries on january 1 2008 encino company issued bonds wi 692448

Bond Issuance and Adjusting Entries

On January 1, 2008, Encino Company issued bonds with a face value of $1,000,000 and a maturity date of December 31, 2017. The bonds have a stated interest rate of 8%, payable on January 1 and July 1. They were sold to Sea Ray Company for $820,744, a yield of 11%. It cost Encino $40,000 to issue the bonds. This amount was deferred and amortized over the life of the issue using the straight line method. Assume that both companies have December 31 year ends and that Encino uses the effective interest method to amortize any premium or discount and Sea Ray uses the straight line method.

Instructions:

1. Make all entries necessary to record the sale and purchase of the bonds on each company’s books.

2. Prepare the adjusting entries as of December 31, 2008, for both companies. Assume Sea Ray is carrying the bonds as a long term held to maturity security.

computation of bond market price and amortization of premium or discount signal ente 692449

Computation of Bond Market Price and Amortization of Premium or Discount

Signal Enterprises decided to issue $900,000 of 10 year bonds. The interest rate on the bonds is stated at 7%, payable semiannually. At the time the bonds were sold, the market rate had increased to 8%.

Instructions:

1. Determine the maximum amount an investor should pay for these bonds. (Round to the nearest dollar.)

2. Assuming that the amount in (1) is paid, compute the amount at which the bonds would be reported by the investor after being held for one year. Use two recognized methods of handling amortization of the difference in cost and maturity value of the bonds and give support to the method you prefer. (Round to the nearest dollar.)

cash flow effects of a bond premium on january 1 2008 data link inc issued 100 000 1 692452

Cash Flow Effects of a Bond Premium

On January 1, 2008, Data link Inc. issued $100,000,10%,10 year bonds when the market rate of interest was 8%. Interest is payable on June 30 and December 31. The following financial information is available:

Sales                                                                      

$300,000

Cost of sales                                                                

180,000

Gross profit                                                                  

120,000

Interest expense                                                              

?

Depreciation expense                                                          

(14,500)

Other expenses                                                              

(82,000)

Net income                                                                 

?

 

 

Dec 31, 2008

Jan 1, 2008

Accounts receivable

$55,000

$48,000

Inventory

87,000

93,000

Accounts payable

60,000

58,000

All purchases of inventory are on account. Other expenses are paid for in cash.

Instructions:

1. Prepare the journal entry to record the issuance of the bonds on January 1, 2008.

2. Compute (a) the amount of cash paid to bondholders for interest during 2008, (b) the amount of premium amortized during 2008, assuming Datalink uses the straight line method for amortizing bond premiums and discounts, and (c) the amount of interest expense for 2008.

3. Prepare the Cash Flows from Operating Activities section of Datalink’s statement of cash flows using (a) the direct method and (b) the indirect method.

bond entries mdash issuer greenwood company sold 4 000 000 of 7 first mortgage bonds 692454

Bond Entries—Issuer

Greenwood Company sold $4,000,000 of 7% first mortgage bonds on October 1, 2000, at $3,479,683 plus accrued interest. The bonds were dated July 1, 2000; interest payable semiannually on January 1 and July 1; redeemable after June 30, 2005, to June 30, 2008, at 101, and thereafter until maturity at 100; and convertible into $1 par value common stock as follows:

• Until June 30, 2005, at the rate of five shares for each $1,000 bond.

• from July 1, 2005, to June 30, 2008, at the rate of four shares for each $1,000 bond.

• after June 30, 2008, at the rate of three shares for each $1,000 bond.

The bonds mature 10 years from their issue date. The company adjusts its books monthly and closes its books as of December 31 each year.

The following transactions occur in connection with the bonds.

2006

July 1 Converted $1,500,000 of bonds into stock with no gain or loss recognized.

2007

Dec. 31 Reacquired $1,000,000 face value of bonds at 99.75 plus accrued interest. These were immediately retired.

2008

July 1 Called the remaining bonds for redemptions and paid accrued interest. For purposes of obtaining funds for redemption and business expansion, a $3,000,000 issue of 9% bonds was sold at 97. These bonds are dated July 1, 2008, and are due in 20 years.

Instructions: Prepare journal entries necessary for Greenwood Company in connection with the preceding transactions, including monthly adjustments, where appropriate, as of the following dates. Assume bond discount amortization is made using the straight line method. (Round to the nearest dollar.)

1. October 1, 2000

4. December 31, 2007

2. December 31, 2000

5. July 1, 2008

3. July 1, 2006

bond entries mdash investor on june 1 2007 sunderland inc purchased as a long term i 692455

Bond Entries—Investor

On June 1, 2007, Sunderland Inc. purchased as a long term investment 400 of the $1,000 face value, 8% bonds of Stateline Corporation for $364,547. The bonds were purchased to yield 10% interest. Interest is payable semiannually on December 1 and June 1. The bonds mature on June 1, 2013. Sunderland uses the effective interest method of amortization. On November 1, 2008, Sunderland sold the bonds for $392,500. This amount includes the appropriate accrued interest. Sunderland intended to hold these bonds until they matured, so year to year market value fluctuations were ignored in accounting for the bonds.

Instructions: Prepare a schedule showing the income or loss before income taxes from the bond investment that Sunderland should record for the years ended December 31, 2007, and 2008.

bond entries mdash investor on may 1 2005 glacier bay co acquired 30 000 of horizon 692456

Bond Entries—Investor

On May 1, 2005, Glacier Bay Co. acquired $30,000 of Horizon Corp. 8% bonds at 97 plus accrued interest. Interest on bonds is payable semiannually on March 1 and September 1, and bonds mature on September 1, 2008. On May 1, 2006, Glacier Bay Co. sold bonds of $10,000 for 103 plus accrued interest. On July 1, 2007, bonds of $15,000 were exchanged for 2,000 shares of Horizon Corp. common, no par value, quoted on the market on this date at $9. Interest was received on bonds to date of exchange. On September 1, 2008, remaining bonds were redeemed and accrued interest was received.

Instructions: Give journal entries for 2005–2008 to record the preceding transactions on the books for Glacier Bay Co., including any adjustments that are required at the end of each fiscal year ending on December 31. Assume bond premium or discount amortization is by the straight line method. Ignore any potential impact of year to year market value changes on the accounting for the bonds.

note payable entries mdash investor and issuer fitzgerald inc issued 750 000 of 8 ye 692457

Note Payable Entries—Investor and Issuer

Fitzgerald Inc. issued $750,000 of 8 year, 11% notes payable dated April 1, 2004. Interest on the notes is payable semiannually on April 1 and October 1. The notes were sold on April 1, 2004, to an underwriter for $720,000 net of issuance costs. The notes were then offered for sale by the underwriter, and on July 1, 2004, L. Baum purchased the entire issue as a long term investment. Baum paid 101 plus accrued interest for the notes. On June 1, 2007, Baum sold the investment in Fitzgerald notes to J. Gott as a short term investment. Gott paid 96 plus accrued interest for the notes as well as $1,500 for brokerage fees. Baum paid $1,000 brokerage fees to sell the notes. Gott held the investment until April 1, 2008, when the notes were called at 104 by Fitzgerald.

Instructions: Prepare all journal entries required on the books of Fitzgerald Inc. for 2004 and 2008; on the books of Baum for 2004 and 2007; and on the books of Gott for 2007 and 2008. Assume that each entity uses the calendar year for reporting purposes and that issue costs are netted against the note proceeds by Fitzgerald. Any required amortization is made using the straight line method. Ignore any potential impact of year to year market value changes on the accounting for the notes by the investors.

reacquisition of bonds gerona company authorized the sale of 300 000 of 10 10 year d 692459

Reacquisition of Bonds

Gerona Company authorized the sale of $300,000 of 10%, 10 year debentures on January 1, 2003. Interest is payable on January 1 and July 1. The entire issue was sold on April 1, 2003, at 103 plus accrued interest. On April 1,2008,$100,000 of the bond issue was reacquired and retired at 99 plus accrued interest. On June 30, 2008, the remaining bonds were reacquired at 98 plus accrued interest and refunded with an issue of $200,000 of 9% bonds which were sold at 100.

Instructions: Give the journal entries for 2003 and 2008 (through June 30) on Gerona Company’s books. The company’s books are kept on a calendar year basis. (Round to the nearest dollar. Assume straight line amortization of the premium or discount. Ignore any potential impact of year to year market value changes on the accounting for the bonds.)

deferred interest bonds and the selling of assets at the beginning of 2006 wheel r d 692460

Deferred Interest Bonds and the Selling of Assets

At the beginning of 2006, Wheel R. Dealer purchased the net assets of Consolidated Corp. by issuing 10 year, 10% bonds with a face value of $100,000,000, with semiannual interest payments made on June 30 and December 31 and no interest payments made until 2011. Dealer hopes to sell off assets of Consolidated and realize enough cash to buy back the bonds on the open market prior to interest payments becoming due in 2011. At the end of 2008, Dealer sold net assets with a carrying value of $85,000,000 for $70,000,000 and used the proceeds to retire the bond issue.

Instructions:

1. Prepare the journal entry to record the issuance of the bonds on January 2, 2006, assuming a market rate of 8%.

2. Prepare the journal entry to record the sale of the net assets.

3. Compute the market value of the bonds on January 3, 2009, the day of retirement, assuming a market rate of 14%.

4. Prepare the journal entry to record the retirement of the bond issue on January 3, 2009, assuming a carrying value of $96,000,000 and the market value as computed in (3).

5. Explain how Mr. Dealer can buy his bonds back three years after their initial sale for less than he originally sold them for and without ever having made an interest payment.

6. Should Mr. Dealer be able to reduce the liability to market value even if he does not retire the bonds?

convertible bonds robison co issued 1 000 000 of convertible 10 year debentures on j 692461

Convertible Bonds

Robison Co. issued $1,000,000 of convertible 10 year debentures on July 1, 2007. The debentures provide for 9% interest payable semiannually on January 1 and July 1. The discount in connection with the issue was $12,000, which is being amortized monthly on a straight line basis. The debentures are convertible after 1 year into seven shares of the Robison Co.’s $1 par value common stock for each $1,000 of debentures. On August 1, 2008, $100,000 of debentures were turned in for conversion into common stock. Interest has been accrued monthly and paid as due. Accrued interest on debentures is paid in cash upon conversion.

Instructions: Prepare the journal entries to record the conversion, amortization, and interest in connection with the debentures as of August 1, 2008, August 31, 2008,and December 31, 2008, including closing entries for year end. No gain or loss is to be recognized on the conversion. (Round to the nearest dollar.)

early extinguishment and conversion of bonds on january 1 2007 brewster company issu 692462

Early Extinguishment and Conversion of Bonds

On January 1, 2007, Brewster Company issued 2,000 of its 5 year, $1,000 face value, 11% bonds dated January 1 at an effective annual interest rate (yield) of 9%. Interest is payable each December 31. Brewster uses the effective interest method of amortization. On December 31, 2008, the 2,000 bonds were extinguished early through acquisition in the open market by Brewster for $1,980,000 plus accrued interest. On July 1, 2007, Brewster issued 5,000 of its 6 year, $1,000 face value, 10% convertible bonds dated July 1 at an effective annual interest rate (yield) of 12%. Interest is payable every June 30 and December 31. The bonds are convertible at the investor’s option into Brewster’s common stock at a ratio of 10 shares of common stock for each bond. On July 1, 2008, an investor in Brewster’s convertible bonds tendered 1,500 bonds for conversion into 15,000 shares of Brewster’s common stock, which had a fair market value of $105 and a par value of $1 at the date of conversion.

Instructions:

1. Make all necessary journal entries for the issuer and the investor to record the issuance of both the 11% and the 10% bonds. Ignore any potential impact of year to year market value changes on the investor accounting for the bonds.

2. Make all necessary journal entries to record the early extinguishment of both debt instruments assuming:

(a) Brewster considered the conversion to be a significant culminating event, and the investors considered their investment in convertible bonds to be debt rather than equity.

(b) Brewster considered the conversion to be a no culminating event, and the investors considered their investment in convertible bonds to be equity rather than debt.

accounting for patents on january 3 2000 merris company spent 89 000 to apply for an 692389

Accounting for Patents

On January 3, 2000, Merris Company spent $89,000 to apply for and obtain a patent on a newly developed product. The patent had an estimated useful life of 10 years. At the beginning of 2004, the company spent $16,000 in successfully prosecuting an attempted infringement of the patent. At the beginning of 2005, the company purchased for $37,000 a patent that was expected to prolong the life of its original patent by five years. On July 1,

2008, a competitor obtained rights to a patent that made the company’s patent obsolete.

Instructions: Give all the entries that would be made relative to the patent for the period 2000–2008, including entries to record the purchase of the patent, annual patent amortization, and ultimate patent obsolescence. (Assume the company’s accounting period is the calendar year.)

intangible impairment on december 31 2007 magily company acquired the following thre 692390

Intangible Impairment

On December 31, 2007, Magily Company acquired the following three intangible assets:

(a) A trademark for $30,000. The trademark has seven years remaining in its legal life. It is anticipated that the trademark will be renewed in the future, indefinitely, without problem.

(b) Goodwill for $150,000.The goodwill is associated with Magily’s Abacus Manufacturing reporting unit.

(c) A customer list for $22,000. By contract, Magily has exclusive use of the list for five years. Because of market conditions, it is expected that the list will have economic value for just three years. On December 31, 2008, before any adjusting entries for the year were made, the following information was assembled about each of the intangible assets:

(a) Because of a decline in the economy, the trademark is now expected to generate cash flows of just $1,000 per year. The useful life of the trademark still extends beyond the foreseeable horizon.

(b) The cash flow expected to be generated by the Abacus Manufacturing reporting unit is $25,000 per year for the next 22 years. Book values and fair values of the assets and liabilities of the Abacus Manufacturing reporting unit are as follows:

 

Book Values

Fair Values

Identifiable Assets                                         

$270,000

$300,000

Goodwill                                               

150,000

?

Liabilities                                               

180,000

180,000

(c) The cash flows expected to be generated by the customer list are $12,000 in 2009 and $8,000 in 2010.

Instructions: The appropriate discount rate for all items is 6%. Make all journal entries necessary on December 31, 2008, in connection with these three intangible assets.

exchange of assets youth development co acquired the following assets in exchange fo 692392

Exchange of Assets

Youth Development Co. acquired the following assets in exchange for various nonmonetary assets.

2008

Mar. 15 Acquired from another company a large lathe in exchange for three small lathes. The small lathes had a total cost of $36,000 and a remaining book value of $13,000. The new lathe had a market value of $19,000, approximately the same value as the three small lathes. This transaction is deemed NOT to have commercial substance.

June 1 Acquired 250 acres of land by issuing 2,500 shares of common stock with par value of $1 and market value of $85. Market analysis reveals that the market value of the stock was a reasonable value for the land.

July 15 Acquired a used piece of heavy, earth moving equipment, market value, $105,000, by exchanging a used molding machine with a market value of $30,000 (book value, $6,000; cost, $42,000) and land with a market value of $95,000 (cost, $50,000). Cash of $20,000 was received by Youth Development Co. as part of the transaction.

Aug. 15 Acquired a patent, franchise, and copyright for two used milling machines. The book value of each milling machine was $3,500, and each originally cost $12,000. The market value of each machine is $15,000. It is estimated that the patent and franchise have about the same market values, and the market value of the copyright is 50% of the market value of the patent.

Nov. 1 Acquired a new packaging machine for four old packaging machines. The old machines had a total cost of $60,000 and a total remaining book value of $15,000. The new packaging machine has an indicated market value of $40,000, approximately the same value as the four machines. This transaction is deemed to have commercial substance.

Instructions: Prepare the journal entries required on Youth Development Co.’s books to record the exchanges.

computation of depreciation and amortization information pertaining to hedlund corpo 692393

Computation of Depreciation and Amortization

Information pertaining to Hedlund Corporation’s property, plant, and equipment for 2008 follows.

Account balances at January 1, 2008:

Debit

Credit

Land                                                            

$ 150,000

 

Buildings                                                         

1,200,000

 

Accumulated Depreciation—Buildings                                    

 

$263,100

Machinery and Equipment                                            

900,000

 

Accumulated Depreciation—Machinery and Equipment                       

 

250,000

Automotive Equipment                                              

115,000

 

Accumulated Depreciation—Automotive Equipment                         

 

84,600

 

Depreciation data:

Depreciation Method

Useful Life

Buildings                                        

150% declining balance

25 years

Machinery and Equipment                            

Straight line

10 years

Automotive Equipment                              

Sum of the years’ digits

4 years

Leasehold Improvements                            

Straight line

The salvage values of the depreciable assets are immaterial. Depreciation is computed to the nearest month.

Transactions during 2008 and other information are as follows:

(a) On January 2, 2008, Hedlund purchased a new car for $20,000 cash and trade in of a 2 year old car with a cost of $18,000 and a book value of $5,400. The new car has a cash price of $24,000; the market value of the trade in is not known.

(b) On April 1, 2008, a machine purchased for $23,000 on April 1, 2003,was destroyed by fire. Hedlund recovered $15,500 from its insurance company.

(c) On May 1, 2008, costs of $168,000 were incurred to improve leased office premises. The leasehold improvements have a useful life of eight years. The related lease terminates on December 31, 2014.

(d) On July 1, 2008, machinery and equipment were purchased at a total invoice cost of $280,000; additional costs of $5,000 for freight and $25,000 for installation were incurred.

(e) Hedlund determined that the automotive equipment comprising the $115,000 balance at January 1, 2008, would have been depreciated at a total amount of $18,000 for the year ended December 31, 2008.

Instructions:

1. Compute the total depreciation and amortization expense that would appear on Hedlund’s income statement for the year ended December 31, 2008. Also compute the accumulated depreciation and amortization that would appear on the balance sheet at December 31, 2008.

2. Compute the total gain or loss from disposal of assets that would appear in Hedlund’s income statement for the year ended December 31, 2008.

3. Prepare the noncurrent operating assets section of Hedlund’s December 31, 2008, balance sheet.

comprehensive depreciation and amortization at december 31 2007 martin company rsquo 692394

Comprehensive Depreciation and Amortization

At December 31, 2007, Martin Company’s noncurrent operating asset and accumulated depreciation and amortization accounts had balances as follows:

 

 

Accumulated Depreciation

Category

Cost of Asset

and Amortization

Land                                         

$ 130,000

 

Buildings                                      

1,200,000

$265,400

Machinery and equipment                         

775,000

196,200

Automobiles and trucks                           

132,000

86,200

Leasehold improvements                          

221,000

110,500

 

Category

Depreciation Method

Useful Life

Land improvements                              

Straight line

12 years

Buildings                                      

150% declining balance

25 years

Machinery and equipment                          

Straight line

10 years

Automobiles and trucks                           

150% declining balance

5 years

Leasehold improvements                           

Straight line

8 years

Depreciation is computed to the nearest month. The salvage values of the depreciable assets are immaterial.

Transactions during 2008 and other information are as follows:

(a) On January 6, 2008, a plant facility consisting of land and a building was acquired from Atlas Corp. for $600,000. Of this amount, 20% was allocated to land.

(b) On April 6, 2008, new parking lots, streets, and sidewalks at the acquired plant facility were completed at a total cost of $192,000. These expenditures had an estimated useful life of 12 years.

(c) The leasehold improvements were completed on December 31, 2004, and had an estimated useful life of eight years. The related lease, which would have terminated on December 31, 2010, was renewable for an additional 4 year term. On April 29, 2008, Martin exercised the renewal option.

(d) On July 1, 2008, machinery and equipment were purchased at a total invoice cost of $250,000. Additional costs of $10,000 for delivery and $30,000 for installation were incurred.

(e) On August 30, 2008, Martin purchased a new automobile for $15,000.

(f) On September 30, 2008, a truck with a cost of $24,000 and a carrying amount of $8,100 on the date of sale was sold for $11,500. Depreciation for the nine months ended September 30, 2008, was $2,352.

(g) On December 20, 2008, a machine with a cost of $17,000 and a carrying amount of $2,975 at date of disposition was scrapped without cash recovery.

Instructions: Compute total depreciation and amortization expense for the year ended December 31, 2008.

sample cpa exam questions 1 in january 2008 vorst co purchased a mineral mine for 2 692395

Sample CPA Exam Questions

1. In January 2008,Vorst Co. purchased a mineral mine for $2,820,000 with removable ore estimated at 1,200,000 tons. After it has extracted all the ore, Vorst believes it will be able to sell the property for $300,000. During 2008, Vorst incurred $360,000 of development costs preparing the mine for production and removed and sold 60,000 tons of ore. In its 2008 income statement, what amount should Vorst report as depletion?

(a) $135,000

(b) $144,000

(c) $150,000

(d) $159,000

2. Turtle Co. purchased equipment on January 2, 2006, for $50,000. The equipment had an estimated 5 year service life with an expected salvage value of $0. Turtle’s policy for 5 year assets is to use the double declining balance depreciation method for the first two years of the asset’s life and then switch to the straight line depreciation method. In its December 31, 2008, balance sheet, what amount should Turtle report as accumulated depreciation for equipment?

(a) $30,000

(b) $38,000

(c) $39,200

(d) $42,000

tax depreciation methods and the time value of money the following two depreciation 692396

Tax Depreciation Methods and the Time Value of Money

The following two depreciation methods are acceptable for tax purposes:

(a) Straight line with a half year convention. The half year convention is the assumption that all assets are acquired in the middle of the year. Therefore, a half year’s depreciation is allowed in the first year.

(b) 200% declining balance with a half year convention. There is a switch to straight line depreciation on the remaining cost when straight line yields a larger amount than does 200% declining balance. On January 1, 2008, Marci Company purchased a piece of equipment for $350,000. The equipment has an estimated useful life of five years and no estimated residual value.

Instructions:

1. For tax purposes, depreciation reduces taxes payable by reducing taxable income. If the tax rate is 40%, for example, a $100 depreciation deduction will reduce taxes by $40. Ignoring the time value of money, calculate the total reduction in taxes Marci will realize through the recovery of the asset cost over the life of the equipment. Assume that the tax rate is 40%. Is the answer the same for each of the two acceptable depreciation methods?

2. For each of the acceptable methods, compute the present value (as of January 1, 2008) of the depreciation tax savings. Assume that the appropriate interest rate is 10% and that the tax savings occur at the end of the year.

3. Should a company be required to use the same depreciation method in its financial statements as it uses for tax purposes?

we don rsquo t need no depreciation the managements of two different companies argue 692397

We Don’t Need No Depreciation!

The managements of two different companies argue that because of specific conditions in their companies, recording depreciation expense should be suspended for 2008. Evaluate carefully their arguments.

(a) The president of Guzman Co. recommends that no depreciation be recorded for 2008 because the depreciation rate is 5% per year, and price indexes show that prices during the year have risen by more than this figure.

(b) The policy of Liebnitz Co. is to recondition its building and equipment each year so that they are maintained in perfect repair. In view of the extensive periodic costs incurred in 2008, officials of the company believe that the need for recognizing depreciation is eliminated.

goodwill must be amortized nevada corporation purchased stardust club for 2 000 000 692398

Goodwill Must Be Amortized!

Nevada Corporation purchased Stardust Club for $2,000,000, which included $500,000 for goodwill. Nevada Corporation incurs large promotional and advertising expenses to maintain Stardust Club’s popularity. As the annual financial statements are being prepared, the CPA of Nevada Corporation, N. Ander Thal, insists that some of the goodwill be amortized against revenue. Thal received his accounting degree in 1971 and cites APB Opinion No. 17, which requires goodwill to be written off over a maximum life of 40 years. Marie Stevenson, Nevada Corporation’s controller, feels that amortization of the purchased goodwill in the same periods as heavy expenses are incurred to maintain the goodwill in effect creates a double charge against income of the period. Stevenson argues that no write off of goodwill is necessary and that goodwill has actually increased in value. In addition, Stevenson claims that current GAAP does not require goodwill to be amortized. Evaluate these two positions.

is it really worth that much ferris bueller inc owns a building in des moines iowa t 692399

Is It Really Worth that Much?

Ferris Bueller, Inc., owns a building in Des Moines, Iowa, that was built at a cost of $5,000,000 in 1997. The building was used as a manufacturing facility from 1998 to 2007. However, economic conditions have made it necessary to consolidate Ferris Bueller’s operations, and the building has been leased as of January 1, 2008, as a warehouse for 10 years at an annual rental of $240,000. Taxes, insurance, and normal maintenance costs are to be paid by the lessee. At the end of the 10 year period, Ferris Bueller may offer the lessee a renewal of the lease or again use the building in its operations. The building is being depreciated on a straight line basis over a 40 year life. In early 2008, Julie Ramos, a new staff accountant for Ferris Bueller, was assigned to review the building accounts and raised a question to Alison Crowther, her supervisor, concerning the carrying value of the Des Moines building. As of January 1, 2008, Julie feels the Des Moines building was impaired and should be written down in value. Alison is unsure about the current position of the FASB on this issue and invites Julie to prepare a memorandum recommending a specific write down amount, with supporting justification. Prepare the memorandum, assuming current interest rates are 10%.

is nothing sacred fasb statement no 93 requires all not for profit organizations to 692403

Is Nothing Sacred?

FASB Statement No. 93 requires all not for profit organizations to compute and report depreciation expense in their external financial statements. Previously, many not for profits, including many religious institutions, did not report depreciation expense. Many users and preparers of financial statements for religious institutions were upset about the idea of depreciating churches. Robert Anthony, a well known professor of accounting at Harvard University, was quoted as saying, “Depreciating catheDr.als and churches is stupid.” Monsignor Austin Bennett of Brooklyn claimed that the rule would cause “more trouble for American churches than all the sinners in their congregations.” Robert K. Mautz in “Monuments, Mistakes and Opportunities,” Accounting Horizons, June 1988, argued that buildings and monuments owned by governments and not for profit institutions may be more liabilities than assets because no revenue is generated from them, but they must be maintained. Consider the following questions.

1. Why do churches prepare external financial statements?

2. It is claimed that requiring churches to record depreciation expense will increase the cost of a church’s annual audit. How?

3. One person was quoted in The Wall Street Journal as saying, “As some . . . communities change in character, so does the value of the churches. Our depreciation values would have to change every year.” Evaluate this comment.

let rsquo s take a bath professor linda de angelo found evidence suggesting that whe 692404

Let’s Take a Bath!

Professor Linda De Angelo found evidence suggesting that when the management of a company is ousted under fire, the new management tends to take an earnings “bath” after gaining control. A “bath” is a large reduction in earnings due to asset write downs, reorganization charges, discontinuance of segments, and other extraordinary charges. As an example, Circle K Corporation declared Chapter 11 bankruptcy and changed management during fiscal 1990. For the year, Circle K reported a reorganization and restructuring charge of $639 million, consisting primarily of write downs of long term assets. This contributed to a net loss for the year of $773 million, compared to average net income for the previous four years of about $40 million per year. Why might the new management of a company want to “take a bath” in its first year?

should financial reporting follow tax legislation during the 1960s and 1970s the u s 692405

Should Financial Reporting Follow Tax Legislation?

During the 1960s and 1970s, the U.S. Congress used a tax measure known as the investment tax credit to encourage companies to expand their investment base. Under these provisions,companies received reductions of their tax liabilities based on a percentage of newinvestments in noncurrent operating assets. This approach was used in lieu of reducing taxrates as a stimulus to expansion. In 1981, the adoption of the ACRS method of cost allocationfor noncurrent operating assets added further stimulation to the economy by permittingcompanies to write off the cost of their property over a shorter than normal period.In 1986, Congress passed a massive Tax Reform Act that significantly reduced tax ratesfor all taxpaying entities. At the same time, the investment tax credit was eliminated andthe ACRS legislation was replaced by a modified ACRS approach that lengthened the timeperiod for the allocation. These latter provisions reduced the net impact of the reduced taxrates. Because elected government officials do not like to be identified with increased taxrates, there remains the possibility that further modifications to tax accounting for noncurrentoperating assets will be made.Should financial reporting for noncurrent operating assets be affected by tax legislation?Support your answer.

deciphering financial statements the walt disney company locate the 2004 financial s 692406

Deciphering Financial Statements (The Walt Disney Company)

Locate the 2004 financial statements for The Walt Disney Company on the Internet and consider the following questions:

1. What depreciation method does Disney use for its parks, resorts, and other property? For its film and television costs?

2. Where do you have to look to find out that Disney’s 2004 total depreciation and amortization expense was $1,210 million?

3. As of September 30, 2004, what percentage of film and television production costs was expected to be amortized within the next three years? 4. In 1996, Disney acquired ABC. The following information concerning the acquisition was provided in Disney’s 1997 annual report:

On February 9, 1996, the Company completed its acquisition of ABC. The aggregate consideration paid to ABC shareholders consisted of $10.1 billion in cash and 155 million shares of Company common stock valued at $8.8 billion based on the stock price as of the date the transaction was announced. The acquisition has been accounted for as a purchase and the acquisition cost of $18.9 billion was allocated to the assets acquired and liabilities assumed based on estimates of their respective fair values. Assets acquired totaled $4.0 billion (of which $1.5 billion was cash) and liabilities assumed were $4.3 billion. A total of $19.0 billion, representing the excess of acquisition cost over the fair value of ABC’s net tangible assets, was allocated to intangible assets and is being amortized over forty years. As seen in the consolidated balance sheet of Disney’s 2004 financial statements, the original cost associated with the total goodwill was only $16.966 billion as of September 30, 2004. What do you think is the explanation for this difference between the $19.0 billion originally recorded for goodwill and the $16.966 billion listed in 2004?

deciphering financial statements delta air lines the following information is from t 692407

Deciphering Financial Statements (Delta Air Lines)

The following information is from the June 30, 1998, balance sheet for Delta Air Lines (all dollar amounts are in millions):

 

1998

1997

Flight equipment       

$11,180

$9,619

Less: Accumulated depreciation

3,895

3,510

Delta also included this note to its financial statements:

Depreciation and Amortization—Effective July 1, 1998, the Company increased the depreciable life of certain new generation aircraft types from 20 to 25 years. Owned flight equipment is depreciated on a straight line basis to a residual value equal to 5% of cost.

1. Assume that all flight equipment will be affected by this change in policy. The new policy will not be reflected in the 1998 financial statements because the policy was changed on July 1, 1998. Estimate the total depreciation expense recognized by Delta on flight equipment for the year ended June 30, 1998, using the old 20 year life and the new 25 year life. Assume that there were no flight equipment retirements during the year and that new acquisitions are depreciated for half the year.

2. How reasonable is the assumption that there were no flight equipment retirements in 1998?

deciphering financial statements ford motor company the following information comes 692408

Deciphering Financial Statements (Ford Motor Company)

The following information comes from the 2004 financial statements of Ford Motor Company (all dollar amounts are in millions):

 

2004

2003

Land

$ 727

$ 675

Buildings and land improvements

12,598

12,204

Machinery, equipment and other

46,387

44,449

Construction in progress

2,089

2,647

Total land, plant and equipment

$ 61,801

$ 59,975

Accumulated depreciation

(31,013)

(30,048)

Net land, plant and equipment

$ 30,788

$ 29,927

Statement of Cash Flows for 2004

 

 

Operating activities:

 

 

Depreciation

$ 3,242

 

Investing activities:

 

 

Capital expenditures

(6,287)

 

1. Estimate the book value of property and equipment disposed of during 2004.

2. Assume that a half year’s depreciation is taken on all assets acquired and disposed of during the year. Estimate the average depreciation life of Ford’s property and equipment. Assume that none of the disposals was land, and eliminate the land balance when estimating the average depreciation life.

3. Estimate the average age of property and equipment (excluding land) owned by Ford as of December 31, 2004.

deciphering financial statements at amp t corporation the following information was 692409

Deciphering Financial Statements (AT&T Corporation)

The following information was extracted from the 1998 annual report of AT&T Corporation (all dollar amounts are in millions):

 

1998

1997

1996

1995

1994

1993

1992

1991

Total revenues

$53,223

$51,577

$50,688

$79,609

$75,094

$69,351

$66,647

$64,455

Operating income

7,487

6,836

8,709

1,215

7,949

6,498

6,529

1,428

Net income

6,398

4,415

5,793

139

4,710

(5,906)

3,442

171

Common share

 

 

 

 

 

 

 

 

owners’ equity

16,949

18,910

17,320

17,274

17,921

13,374

20,313

17,973

• 1998 data reflect $2.5 billion of pretax business restructuring charges.

• 1995 data reflect $7.8 billion of pretax business restructuring and other charges.

• 1993 data reflect a $9.6 billion net charge for three accounting changes.

• 1991 data reflect $4.5 billion of pretax business restructuring and other charges.

Instructions:

1. For each year 1991–1998,calculate operating income as a percentage of total revenues, net income (loss) as a percentage of total revenues, and return on common equity (use end of year equity).

2. Repeat (1) after adding back the effects of the special charges in 1991, 1993, 1995, and 1998. For calculating net income, assume that the incremental income tax rate is 40%.

3. A large portion of the special charges in 1991 and 1995 were related to asset write downs. These write downs were recorded before FASB Statement No. 144 was issued; thus, more flexibility occurred in determining when an asset was impaired. Comment on the impact of special charges on the usefulness of financial accounting data.

writing assignment one depreciation method please the fasb frequently receives recom 692410

Writing Assignment (One depreciation method, please!)

The FASB frequently receives recommendations about areas it should consider for study. Depreciation accounting has not been ad Dr. essed as a separate topic by the FASB, and several alternative methods are used for recording this expense on the books. Assume that a group of financial analysts recommends to the FASB that a study be made of depreciation accounting with the objective of selecting one method as the only acceptable one. The analysts reason that only then will comparability in financial statements be achieved. You have recently been hired as a member of the FASB’s research staff. Write a summary memo presenting the arguments for and against the FASB following the recommendation by the analysts.

researching accounting standards to help you become familiar with the accounting sta 692411

Researching Accounting Standards

To help you become familiar with the accounting standards, this case is designed to take you to the FASB’s Web site and have you access various publications. In this chapter, we discussed issues relating to fixed assets. For this case, we will use

Statement of Financial Accounting Standards No.153,“Exchanges of Nonmonetary Assets.”

Open FASB Statement No. 153.

1. What previous accounting standard does FASB Statement No. 153 amend?

2. In paragraph 2 of the summary, two International Accounting Standards are identified that relate to this Statement. What are those two International Accounting Standards?

3. Read paragraph 21. When is a nonmonetary exchange considered to have commercial substance?

ethical dilemma profit manipulation during labor negotiations you and your partner o 692412

Ethical Dilemma (Profit manipulation during labor negotiations)

You and your partner own a small data entry company. You contract with businesses to manually enter data, such as library card catalogs and medical records, into a computer database. Your most significant physical assets are a large office building you own, along with the computer hardware and software necessary for operations. Your business has been running for five years, and you now have 100 employees. Operating cash flow has always been healthy, and you and your partner have been able to withDr.aw significant amounts of cash from the business. Recently, you have seen growing discontent among your employees because of their low wages and lack of fringe benefits. You and your partner are preparing for the first meeting with an employee grievance committee. Your partner has taken responsibility for preparing the company’s financial statements. You are embarrassed to admit that this is the first set of financial statements you have ever examined—you have never sought bank financing and all equity funding has come from you and your partner. You are surprised when you first review the statements because they reveal that the company has experienced significant losses in each of its five years of operation. A closer look at the statements reveals that your partner has used the double declining balance method of depreciation for your office building and computer equipment. He has also assumed very short useful lives and zero residual values. Your calculations indicate that using the straight line method with more realistic useful life and residual value assumptions would increase profits Dr. amatically, even to the extent that substantial profits would be reported in each of the first five years of operation. The meeting with the employee grievance committee is tomorrow. Your partner has been your friend since first grade. What, if anything, should you do?

group depreciation holdaway inc a small furniture manufacturer purchased the followi 692356

Group Depreciation

Holdaway, Inc., a small furniture manufacturer, purchased the following assets at the end of 2007.

Description

Cost

Salvage

Life

Delivery truck                                            

$24,000

$5,000

5 years

Circular saws                                             

900

130

7 years

Workbench                                              

320

8 years

Forklift                                                 

9,000

500

5 years

Compute the following amounts for 2008 using group depreciation on a straight line basis:

1. Depreciation expense

2. Group depreciation rate

3. Average life of the assets

group depreciation entries lundquist inc uses the group depreciation method for its 692357

Group Depreciation Entries

Lundquist, Inc., uses the group depreciation method for its furniture account. The depreciation rate used for furniture is 21%. The balance in the furniture account on December 31, 2007, was $125,000, and the balance in Accumulated Depreciation—Furniture was $61,000. The following purchases and dispositions of furniture occurred in the years 2008–2010 (assume that all purchases and disposals occurred at the beginning of each year).

Year

Assets Purchased—Cost (Cash)

 

Assets Sold

 

 

Cost

Selling Price (Cash)

2008                     

$35,000

$27,000

$8,000

2009                    

27,600

15,000

6,000

2010                    

24,500

32,000

8,000

1. Prepare the summary journal entries Lundquist should make each year (2008–2010) for the purchase, disposition, and depreciation of the furniture.

2. Prepare a summary of the furniture and accumulated depreciation accounts for the years 2008 2010.

depreciation of special components jackson manufacturing acquired a new milling mach 692358

Depreciation of Special Components

Jackson Manufacturing acquired a new milling machine on April 1, 2003. The machine has a special component that requires replacement before the end of the useful life. The asset was originally recorded in two accounts, one representing the main unit and the other for the special component. Depreciation is recorded by the straight line method to the nearest month, residual values being disregarded. On April 1, 2009, the special component is scrapped and is replaced with a similar component. This component is expected to have a residual value of approximately 25% of cost at the end of the useful life of the main unit, and because of its materiality, the residual value will be considered in calculating depreciation. Specific asset information is as follows:

Main milling machine:

 

Purchase price in 2003                                                       

$74,800

Residual value                                                             

$6,200

Estimated useful life                                                         

10 years

First special component:

 

Purchase price                                                             

$12,000

Residual value                                                             

$500

Estimated useful life                                                          

6 years

Second special component:

 

Purchase price                                                             

$16,500

What are the depreciation charges to be recognized for the years (1) 2003, (2) 2009, and (3) 2010?

depletion expense on january 2 2007 cynthia foster purchased land with valuable natu 692360

Depletion Expense

On January 2, 2007, Cynthia Foster purchased land with valuable natural ore deposits for $10 million. The estimated residual value of the land was $2 million. At the time of purchase, a geological survey estimated 2 million tons of removable ore were under the ground. Early in 2007, roads were constructed on the land to aid in the extraction and transportation of the mined ore at a cost of $750,000. In 2007, 50,000 tons were mined. In 2008, Cynthia fired her mining engineer and hired a new expert. A new survey made at the end of 2008 estimated 3 million tons of ore were available for mining. In 2008,150,000 tons were mined. Assuming that all the ore mined was sold, how much was the depletion expense for 2007 and 2008?

recording an impairment loss della bee company purchased a manufacturing plant build 692363

Recording an Impairment Loss

Della Bee Company purchased a manufacturing plant building 10 years ago for $1,300,000. The building has been depreciated using the straight line method with a 30 year useful life and 10% residual value. Della Bee’s manufacturing operations have experienced significant losses for the past two years, so Della Bee has decided that the manufacturing building should be evaluated for possible impairment. Della Bee estimates that the building has a remaining useful life of 15 years, that net cash inflow from the building will be $50,000 per year, and that the fair value of the building is $380,000.

1. Determine whether an impairment loss should be recognized.

2. If an impairment loss should be recognized, make the appropriate journal entry.

3. How would your answer to (1) change if the fair value of the building was $560,000?

impairment of intangibles an intangible asset cost 300 000 on january 1 2008 on janu 692366

Impairment of Intangibles

An intangible asset cost $300,000 on January 1, 2008. On January 1, 2009, the asset was evaluated to determine whether it was impaired. As of January 1, 2009, the asset was expected to generate future cash flows of $25,000 per year (at the end of the year). The appropriate discount rate is 5%.

1. Give the entries to record amortization in 2008 and any impairment loss in 2009 assuming that as of January 1, 2008, the asset was assumed to have a total useful life of 10 years and that as of January 1, 2009, there were nine years remaining.

2. Give the entries to record amortization in 2008 and any impairment loss in 2009 assuming that as of January 1, 2008, the asset was assumed to have an indefinite useful life and that as of January 1, 2009, the remaining life was still indefinite.

impairment of goodwill largest company acquired large company on january 1 as part o 692367

Impairment of Goodwill

Largest Company acquired Large Company on January 1. As part of the acquisition,$10,000 in goodwill was recognized; this goodwill was assigned to Largest’s Production reporting unit. During the year, the Production reporting unit reported revenues of $13,000. Publicly traded companies with operations similar to those of the Production unit had price to revenue ratios averaging 1.60. The fair values and book values of the assets and liabilities of the Production reporting unit are as follows:

 

Book Values

Fair Values

Identifiable assets

$21,300

$20,500

Goodwill

10,000

?

Liabilities

7,600

7,600

Make the journal entry necessary to recognize any goodwill impairment loss.

long term operating asset held for sale on april 1 2008 brandoni company has a piece 692369

Long Term Operating Asset Held for Sale

On April 1, 2008, Brandoni Company has a piece of machinery with a cost of $100,000 and accumulated depreciation of $75,000. On April 1, Brandoni decided to sell the machine within 1 year. As of April 1, 2008, the machine had an estimated selling price of $10,000 and a remaining useful life of 2 years. It is estimated that selling costs associated with the disposal of the machine will be $1,000. On December 31, 2008, the estimated selling price of the machine had increased to $15,000, with estimated selling costs increasing to $1,600.

1. Make the entry to record the initial classification of the machine as held for sale on April 1, 2008.

2. Make the entry to record depreciation expense on the machine for the period April 1 through December 31, 2008.

3. Make the entry, if any, needed on December 31, 2008, to reflect the change in the expected selling price.

exchange of machinery assume that coaltown corporation has a machine that cost 52 00 692370

Exchange of Machinery

Assume that Coaltown Corporation has a machine that cost $52,000, has a book value of $35,000, and has a market value of $40,000. The machine is used in Coaltown’s manufacturing process. For each of the following situations, indicate the value at which the company should record the new asset and why it should be recorded at that value.

(a) Coaltown exchanged the machine for a truck with a list price of $43,000.

(b) Coaltown exchanged the machine with another manufacturing company for a similar machine with a list price of $41,000.

(c) Coaltown exchanged the machine for a newer model machine from another manufacturing company. The new machine had a list price of $62,000, and Coaltown paid a “large” amount of cash of $15,000.

(d) Coaltown exchanged the machine plus a “small” amount of cash of $3,000 for a similar machine from Newton Inc., a manufacturing company. The newly acquired machine is carried on Newton’s books at its cost of $55,000 with accumulated depreciation of $42,000; its fair market value is $43,000. In addition to determining the value, give the journal entries for both companies to record the exchange.

exchange of truck on january 2 2008 butler delivery company traded with a dealer an 692371

Exchange of Truck

On January 2, 2008, Butler Delivery Company traded with a dealer an old delivery truck for a newer model. Data relative to the old and new trucks follow:

Old truck:

 

Original cost                                                                    

$18,000

Accumulated depreciation as of January 2, 2008                                         

15,000

New truck:

 

List price                                                                      

$20,000

Cash price without trade in                                                        

19,400

Cash paid with trade in                                                            

17,800

1. Give the journal entries on Butler’s books to record the purchase of the new truck.

2. Give the journal entries on Butler’s books if the cash paid was $15,600 and that amount is considered “large.”

computation of book and tax depreciation midwest states manufacturing purchased fact 692373

Computation of Book and Tax Depreciation

Midwest States Manufacturing purchased factory equipment on March 15,2007. The equipment will be depreciated for financial purposes over its estimated useful life, counting the year of acquisition as a half year. The company accountant revealed the following information regarding this machine:

Purchase price                                                                

$75,000

Residual value                                                                

$9,000

Estimated useful life                                                            

10 years

1. What amount should Midwest States Manufacturing record for depreciation expense for 2008 using the (a) double declining balance method and (b) sum of the years’ digits method?

2. Assuming the equipment is classified as 7 year property under the modified accelerated cost recovery system (MACRS), what amount should Midwest States Manufacturing deduct for depreciation on its tax return in 2008?

depreciation under different methods on january 1 2005 ron shelley purchased a new t 692376

Depreciation Under Different Methods

On January 1, 2005, Ron Shelley purchased a new tractor to use on his farm. The tractor cost $100,000. Ron also had the dealer install a front end loader on the tractor. The cost of the front end loader was $7,000. The shipping charges were $600, and the cost to install the loader was $800. The estimated life of the tractor was eight years, and the estimated service hour life of the tractor was 12,500 hours. Ron estimated that he could sell the tractor for $15,000 at the end of eight years or 12,500 hours. The tractor was used for 1,725 hours in 2008. A full year’s depreciation was taken in 2005, the year of acquisition.

Instructions: Compute depreciation expense for 2008 under each of the following methods:

1. Straight line

2. Double declining balance

3. Sum of the years’ digits

4. Service hours

depreciation and the steady state lyell company started a newspaper delivery busines 692378

Depreciation and the Steady State

Lyell Company started a newspaper delivery business on January 1, 2005. On that date, the company purchased a small pickup truck for $14,000.Lyell planned to depreciate the truck over three years and assumed an $800 residual value. During 2005 and 2006, Lyell’s business expanded. On January 1, 2006, Lyell purchased a second truck, identical to the first. On January 1, 2007, Lyell purchased a third truck, again identical to the first two. During 2007, Lyell’s growth leveled off. However, on January 1, 2008, Lyell bought another truck to replace the one (purchased in 2005) that had just worn out. All trucks purchased cost the same amount as the first truck.

Instructions:

1. Compute depreciation expense for 2005, 2006, 2007, and 2008 using the following:

(a) Straight line method

(b) Sum of the years’ digits method

2. What general conclusions can be Dr. awn from your calculations in (1)?

changes in estimates the following independent cases describe facts concerning the o 692381

Changes in Estimates

The following independent cases describe facts concerning the ownership of racing bicycles.

(a) Maurizio Fon Dr .iest, winner of the 2006 Milan–San Remo cycling classic, purchased a new Colnago bicycle for $8,000 at the beginning of 2006. The bicycle was being depreciated using the straight line method over an estimated useful life of seven years, with a $1,000 salvage value. At the beginning of 2008, the Italian superstar paid $1,600 to upgrade the bicycle. As aresult, the useful life of the bicycle was extended by one year. The salvage value remained $1,000.

(b) John Museeuw, winner of his country’s own Tour of Flanders cycling classic in 2006, purchased a new Bianchi bicycle for $6,000 at the beginning of 2005. The bicycle was being depreciated using the double declining balance method over an estimated useful life of five years, with a $1,000 salvage value. At the beginning of 2006, when the Belgian superstar won at Flanders, the salvage value of his Bianchi (eventual selling price) jumped to $2,000.

(c) Gilbert Duclose Lasalle, winner of the 2005 and 2006 Paris Roubaix cycling classics, purchased a new Lemond Armstrong bicycle for $7,000 in 2004. The French superstar did not use his new bicycle during the 2004 season. However, in 2005 and 2006, Lasalle used his bicycle to win Paris Roubaix and logged 6,000 and 8,000 kilometers, respectively, each year. Lasalle estimated that the bicycle had a productive life of 20,000 kilometers. He did not use the bike in 2007, but in 2008 he decided to upgrade the bike with $2,000 of new components, giving the bicycle an additional 10,000 kilometers of productive use. During the 2008 season, he logged 12,000 kilometers on the bike. The estimated salvage value of the bicycle is $1,000.

Instructions: In each case, compute the depreciation for 2008.

financial statements for mining company roscoe corp was organized on january 2 2008 692382

Financial Statements for Mining Company

Roscoe Corp. was organized on January 2, 2008. It was authorized to issue 74,000 shares of common stock. On the date of organization, it sold 20,000 shares at $50 per share and gave the remaining shares in exchange for certain land bearing recoverable ore deposits estimated by geologists at 900,000 tons. The property is deemed to have a value of $2,700,000 with no residual value. During 2008, purchases of mine buildings and equipment totaled $250,000.During the year, 75,000 tons were mined; 8,000 tons of this amount were unsold on December 31, the balance of the tonnage being sold for cash at $17 per ton. Expenses incurred and paid for during the year, exclusive of depletion and depreciation, were as follows:

Mining                                                                      

$173,500

Delivery                                                                    

20,000

General and administrative                                                       

19,500

Cash dividends of $2 per share were declared on December 31, payable January 15, 2009. It is believed that buildings and sheds will be useful only over the life of the mine; hence, depreciation is to be recognized in terms of mine output.

Instructions: Prepare an income statement and a balance sheet for 2008.Ignore income taxes.

depletion expense in 2004 heslop mining company purchased property with natural reso 692383

Depletion Expense

In 2004,Heslop Mining Company purchased property with natural resources for $5,400,000. The property was relatively close to a large city and had an expected residual value of $700,000.

The following information relates to the use of the property:

(a) In 2004,Heslop spent $300,000 in development costs and $500,000 in buildings on the property. Heslop does not anticipate that the buildings will have any utility after the natural resources are depleted.

(b) In 2005 and 2007,$200,000 and $700,000,respectively,were spent for additional developments on the mine.

(c) The tonnage mined and estimated remaining tons for years 2004–2008 are as follows:

Year

Tons Extracted

Estimated Tons Remaining

2004                   

                0

4,000,000

2005                   

                1,200,000

2,800,000

2006                   

                1,100,000

1,800,000

2007                   

                800,000

900,000

2008                   

                900,000

0

Instructions: Compute the depletion and depreciation expense for the years 2004–

2008.

depletion and depreciation in 2003 sunbeam corporation acquired a silver mine in eas 692384

Depletion and Depreciation

In 2003, Sunbeam Corporation acquired a silver mine in eastern Alaska. Because the mine is located deep in the Alaskan frontier, Sunbeam was able to acquire the mine for the low price of $50,000. In 2004, Sunbeam constructed a road to the silver mine costing $5,000,000. Improvements to the mine made in 2004 cost $750,000.Because of the improvements to the mine and to the surrounding land, it is estimated that the mine can be sold for $600,000 when mining activities are complete. During 2005, five buildings were constructed near the mine site to house the mine workers and their families. The total cost of the five buildings was $1,500,000. Estimated residual value is $250,000. In 2003, geologists estimated 4 million tons of silver ore could be removed from the mine for refining. During 2006, the first year of operations, only 5,000 tons of silver ore were removed from the mine. However, in 2007, workers mined 1 million tons of silver. During that same year, geologists discovered that the mine contained 3 million tons of silver ore in addition to the original 4 million tons. Improvements of $275,000 were made to the mine early in 2007 to facilitate the removal of the additional silver. Early in 2007, an additional building was constructed at a cost of $225,000 to house the additional workers needed to excavate the added silver. This building is not expected to have any residual value. In 2008, 2.5 million tons of silver were mined and costs of $1,100,000 were incurred at the beginning of the year for improvements to the mine.

Instructions:

1. Compute the depreciation and depletion charges for 2006, 2007, and 2008.

2. Give the journal entries to record the depreciation and depletion charges for 2008.

computation of depreciation and depletion the following independent situations descr 692385

Computation of Depreciation and Depletion

The following independent situations describe facts concerning the ownership of various assets.

(a) Dewey Company purchased a tooling machine in 1998 for $60,000. The machine was being depreciated on the straight line method over an estimated useful life of 20 years with no salvage value. At the beginning of 2008, when the machine had been in use for 10 years, Dewey paid $12,000 to overhaul the machine. As a result of this improvement, Dewey estimated that the useful life of the machine would be extended an additional five years.

(b) Emerson Manufacturing Co., a calendar year company, purchased a machine for $65,000 on January 1, 2006. At the date of purchase, Emerson incurred the following additional costs:

Loss on sale of old machinery                                                 

$1,500

Freight cost                                                              

500

Installation cost                                                            

2,000

Testing costs prior to regular operation                                          

400

The estimated salvage value of the machine was $5,000, and Emerson estimated that the machine would have a useful life of 20 years, with depreciation being computed using the straight line method. In January 2008, accessories costing $4,860 were added to the machine to reduce its operating costs. These accessories neither prolonged the machine’s life nor did they provide any additional salvage value.

(c) On July 1, 2008, Lund Corporation purchased equipment at a cost of $34,000. The equipment has an estimated salvage value of $3,000 and is being depreciated over an estimated life of eight years under the double declining balance method of depreciation. For the six months ended December 31, 2008, Lund recorded a half year’s depreciation.

(d) Aiken Company acquired a tract of land containing an extractable natural resource. Geological surveys estimate that the recoverable reserves will be 3,800,000 tons and that the land will have a value of $500,000 after restoration. Relevant cost information follows:

Land

$10,000,000

Tons mined and sold in 2008

700,000

(e) In January 2008, Marcus Corporation entered into a contract to acquire a new machine for its factory. The machine, which had a cash price of $200,000, was paid for as follows:

 

$30,000

Down payment                                                   

185,000

500 shares of Marcus common stock with an agreed upon value of $370 per share 

$215,000

Prior to the machine’s use, installation costs of $7,000 were incurred. The machine has an estimated useful life of 10 years and an estimated salvage value of $10,000. The straight line method of depreciation is used.

Instructions: In each case, compute the amount of depreciation or depletion for 2008.

depreciation and the cash flow statement oakeson company is a manufacturing firm wor 692386

Depreciation and the Cash Flow Statement

Oakeson Company is a manufacturing firm. Work in process and finished goods inventories for December 31, 2008, and December 31, 2007, follow:

 

Dec. 31, 2008

Dec. 31, 2007

Work in process inventory (including depreciation)

$70,000

$75,000

Finished goods inventory (including depreciation)

123,000

110,000

Depreciation is a major portion of Oakeson’s overhead, and the inventories listed above include depreciation in the following amounts:

 

Dec. 31, 2008

Dec. 31, 2007

Depreciation included in work in process inventory.

$15,000

$12,500

Depreciation included in finished goods inventory

26,000

29,000

Oakeson’s net income for 2008 was $90,000. Cost of goods sold for the year included $22,000 in depreciation.

Instructions: Compute net cash flow from operating activities for Oakeson Company for 2008. Assume that the levels of all current assets (except for inventories) and all current liabilities were unchanged from beginning of year to end of year.

impairment deedle company purchased four convenience store buildings on january 1 20 692387

Impairment

Deedle Company purchased four convenience store buildings on January 1,2002,for a total of $26,000,000. The buildings have been depreciated using the straight line method with a 20 year useful life and 5% residual value. As of January 1, 2008, Deedle has converted the buildings into Internet Learning Centers where classes on Internet usage will be conducted six days a week. Because of the change in the use of the buildings, Deedle is evaluating the buildings for possible impairment. Deedle estimates that the buildings have a remaining useful life of 10 years, that their residual value will be zero, that net cash inflow from the buildings will total $1,600,000 per year, and that the current fair value of the four buildings totals $10,000,000.

Instructions:

1. Make the appropriate journal entry, if any, to record an impairment loss as of January 1, 2008.

2. Compute total depreciation expense for 2008.

3. Repeat (1) and (2) assuming that the net cash inflow from the buildings totals $2,200,000 per year. The fair value of the four buildings totals $12,000,000.

summary entries for interest payments santa clarita company reported interest expens 692314

Summary Entries for Interest Payments

Santa Clarita Company reported interest expense in 2008 and 2007 of $470,000 and $410,000, respectively. The balance in Accrued Interest Payable at the end of 2008, 2007, and 2006 was $51,000, $59,000, and $46,000, respectively. In addition, a note to Santa Clarita’s 2008 financial statements included the following: Interest costs related to construction in progress are capitalized as incurred. The Company capitalized $350,000 and $260,000 of interest costs during the years 2008 and 2007, respectively.

Instructions:

1. What summary journal entries would be needed to record all information related to interest in 2008 and 2007?

2. How would interest paid be disclosed in Santa Clarita’s statement of cash flows for 2008 and 2007? Santa Clarita uses the indirect method in reporting cash flow from operating activities.

classifying expenditures as assets or expenses as of december 31 2008 w w cole compa 692315

Classifying Expenditures as Assets or Expenses

As of December 31, 2008, W. W. Cole Company’s total assets were $325 million and total liabilities were $180 million. Net income for 2008 was $38 million. During 2008, W. W. Cole’s chief executive officer had put extreme pressure on employees to meet the profitability goal the CEO had set for them. The goal was to achieve a return on stockholders’ equity in 2008 of 25% (net income/stockholders’ equity). The rumor among Cole’s employees is that to meet this goal, the accounting for some items may have been overly “aggressive.” The following items are of concern:

(a) Research and development costs totaling $18 million were capitalized. None of these costs related to items with alternative uses. The capitalized R&D was assigned a useful life of six years; $3 million was written off during 2008.

(b) During the year, a building was acquired in exchange for 5 million shares of Cole common stock. The building was assigned a value of $27 million by the board of directors. At the time of the exchange, Cole common stock was trading on the New York Stock Exchange for $3 per share.

(c) On December 31, equipment was purchased for $1 million in cash and an agreement to pay $3 million per year for the next eight years, the first payment to be made in one year. The cost of the equipment was recorded at $25 million. The interest rate implicit in the contract was 12%.

(d) Interest of $7 million was capitalized during the year. The only items produced during the year by Cole were routine inventory items.

Instructions:

1. Ignoring any concerns raised by items (a) through (d), did W. W. Cole Company meet its profitability goal for the year?

2. After making any adjustments suggested by items (a) through (d), did W. W. Cole meet its profitability goal? (Ignore income taxes.)

3. What should prevent accounting abuses like those described above?

classifying expenditures as assets or expenses rolitz company completed a program of 692316

Classifying Expenditures as Assets or Expenses

Rolitz Company completed a program of expansion and improvement of its plant during 2008.You are provided with the following information concerning its buildings account:

(a) On October 31, 2008, a 30 foot extension to the present factory building was completed at a contract cost of $329,000.

(b) During the course of construction, the following costs were incurred for the removal of the end wall of the building where the extension was to be constructed.

(1) Payroll costs during the month of April arising from employees’ time spent in removing the wall, $12,360.

(2) Payments to a salvage company for removing unusual debris, $1,520.

(c) The cost of the original structure allocable to the end wall was estimated to be $26,400 with accumulated depreciation thereon of $11,100. Rolitz Company received $5,930 from the construction company for windows and other assorted materials salvaged from the old wall.

(d) The old floor covering was replaced with a new type of long lasting floor covering at a cost of $5,290. The cost of the old floor covering was $12,000 and accumulated depreciation was $5,045. The cost of the old floor covering had been included with the overall building cost even though the floor covering has a different useful life than the building.

(e) The interior of the plant was repainted in new bright colors for a contract price of $8,290.

(f) New and improved shelving was installed at a cost of $3,620. The cost of the old shelving was $2,000 and accumulated depreciation was $840.The cost of the old shelving had been included with the overall building cost even though the shelving has a different useful life than the building.

(g) Old electrical wiring was replaced at a cost of $10,218. Cost of the old wiring was determined to be $4,650 with accumulated depreciation to date of $2,055. Assume that the new wiring has a remaining useful life that is the same as the building.

(h) New electrical fixtures using fluorescent bulbs were installed. The new fixtures were purchased on the installment plan; the schedule of monthly payments showed total payments of $9,300, which included interest and carrying charges of $720. The old fixtures were carried at a cost of $2,790 with accumulated depreciation to date of $1,200. The old fixtures have no scrap value. Assume that the new fixtures have the same remaining useful life as the building.

Instructions: Prepare journal entries for the preceding information. Briefly justify the capitalize or expense decision for each item.

acquisition and valuation of intangibles beecher rsquo s boston barbeque company pur 692317

Acquisition and Valuation of Intangibles

Beecher’s Boston Barbeque Company purchased a customer list and an ongoing research project for a total of $300,000. Beecher uses the expected cash flow approach for estimating the fair value of these two intangibles. The appropriate interest rate is 8%. The potential future cash flows from the two intangibles, and their associated probabilities, are as follows:

Customer List

Outcome 1

20% probability of cash flows of $40,000 at the end of each year for five

 

years.

Outcome 2

30% probability of cash flows of $18,000 at the end of each year for four

 

years.

Outcome 3

50% probability of cash flows of $9,000 at the end of each year for three

 

years.

Ongoing Research Project

Outcome 1

10% probability of cash flows of $450,000 at the end of each year for

10 years.

Outcome 2

20% probability of cash flows of $12,000 at the end of each year for four

 

years.

Outcome 3

70% probability of cash flows of $500 at the end of each year for three

 

years.

Instructions: Prepare the journal entry necessary to record the purchase of the two intangibles.

sample cpa exam questions 1 cole co began constructing a building for its own use in 692319

Sample CPA Exam Questions

1. Cole Co. began constructing a building for its own use in January 2008. During 2008, Cole incurred interest of $50,000 on specific construction debt and $20,000 on other borrowings. The amount of interest that could have been avoided if the building construction expenditures had been used to pay off debt during 2008 was $40,000.What amount of interest cost should Cole capitalize?

(a) $20,000

(b) $40,000

(c) $50,000

(d) $70,000

2. Which of the following costs of goodwill should be capitalized?

 

Maintaining

Developing

 

Goodwill

Goodwill

(a)

Yes

No

(b)

No

No

(c)

Yes

Yes

(d)

No

Yes

is there any goodwill fugate energy corp has recently purchased a small local compan 692320

Is There Any Goodwill?

Fugate Energy Corp. has recently purchased a small local company, Gleave Inc., for $556,950 cash. Fugate’s chief accountant has been given the assignment of preparing the journal entry to record the purchase. An investigation disclosed the following information about the assets of Gleave Inc.:

(a) Gleave owned land and a small manufacturing building. The book value of the property on Gleave’s records was $115,000. An appraisal for fire insurance purposes had been made during the year. The building was appraised by the insurance company at $175,000. Property tax assessment notices showed that the building’s worth was five times the worth of the land.

(b) Gleave’s equipment had a book value of $75,000. It is estimated by Gleave that it would take six times the amount of book value to replace the old equipment with new equipment. The old equipment is, on average, 50% depreciated.

(c) Gleave had a franchise to produce and sell solar energy units from another company in a set geographic area. The franchise was transferred to Fugate as part of the purchase. Gleave carried the asset on its books at $40,000, the unamortized balance of the original cost of $90,000.The franchise is for an unlimited time. Similar franchises are now being sold by the company for $120,000 per geographic area.

(d) Gleave had two excellent research scientists who were responsible for much of the company’s innovation in product development. Each is paid $150,000 per year by Gleave. They have agreed to work for Fugate Energy at the same salary.

(e) Gleave held two patents on its products. Both had been fully amortized and were not carried as assets on Gleave’s books. Gleave believes they could have been sold separately for $75,000 each.

Evaluate each of these items and prepare the journal entry that should be made to record the purchase on Fugate’s books.

how much does a self constructed machine cost bakeman co decides to construct a piec 692321

How Much Does a Self Constructed Machine Cost?

Bakeman Co. decides to construct a piece of specialized machinery using personnel from the maintenance department. This is the first time the maintenance personnel have been used for this purpose, and the cost accountant for the factory is concerned as to the accounting for costs of the machine. Some of the issues raised by the maintenance department management follow:

(a) The maintenance department supervisor has instructed the workers to schedule work so that all the overtime hours are charged to the machinery. Overtime is paid at 150% of the regular rate, or at a 50% premium.

(b) Material used in the production of the machine is charged out from the materials storeroom at 125% of cost, the same markup used when material is furnished to subsidiary companies.

(c) The maintenance department overhead rate is applied on maintenance hours. No extra overhead is anticipated as a result of constructing the machine. (d) Maintenance department personnel are not qualified to test the machine on the production line. This will be done by production employees.

(e) Although the machine will take about one year to build, no extra borrowing of funds will be necessary to finance its construction. The company does, however, have outstanding bonds from earlier financing.

(f) It is expected that the self construction of the machinery will save the company at least $20,000.

What advice can you give the cost accountant to help in the determination of a proper cost for the machine? Ad Dr. ess each individual issue.

but research is our only asset strategy inc was organized by elizabeth durrant and r 692322

But Research Is Our Only Asset!

Strategy, Inc., was organized by Elizabeth Durrant and Ramona Morales, two students working their way through college. Both Elizabeth and Ramona had used the Internet extensively while in high school and had become very proficient Web surfers. Elizabeth had a special ability for designing Web based games that challenged the reasoning power of players. Ramona could see great potential in marketing Elizabeth’s products to other Web users, and so the two began Strategy. Sales have exceeded expectations, and they have added 10 employees to their company to design additional products, debug new programs, and produce and distribute the final software products. Because of its growing size, increased capital is needed for the company. The partners decide to apply for a $100,000 loan to support the growing cost of research. As part of the documentation to obtain the loan, the bank asks for audited financial statements for the past year. After some negotiation, Mark Dawson, CPA, is hired. Strategy had produced a preliminary income statement that reported net income of $35,000. After reviewing the statements, Dawson indicates that the company actually had a $10,000 loss for the year. The major difference relates to $45,000 of wage and material costs that Strategy had capitalized as an intangible asset but that Dawson determined should be expensed. “It’s all research and development,” Dawson insisted. “We’ll easily recoup it in sales next year,” countered Ramona. “I thought you accountants believed in the matching principle. Why do you permit us to capitalize the equipment we’re using, but not our Web development costs? We’ll never look profitable under your requirements!” What major issues are involved in this case? Which position best reflects generally accepted accounting principles?

i found gold can it go on my balance sheet ling company owns several mining claims i 692323

I Found Gold!!! Can It Go on My Balance Sheet?

Ling Company owns several mining claims in Nevada and California. The claims are carried on the books at the cost paid to acquire them 10 years ago. At that time, it was estimated that the claims represented ore reserves valued at $250,000, and the price paid for the properties reflected this value. Subsequent mining and exploration activities have indicated values up to four times the original estimate. Additional capital is needed to pursue the claims, and Ling has decided to issue new shares of common stock. The company wants to report the true value of the claims in the financial statements to make the stock more attractive to potential investors. The accountant, Jennifer Harrison, realizes that the cost basis of accounting does not permit the recording of discovery values. On the other hand, she believes that to ignore the greatly increased value of the claims would be misleading to users. Isn’t there some way the recorded asset values can be increased to better reflect future cash flows arising from the claims? You are hired as an accounting consultant to assist Ling in obtaining additional capital. What recommendations can you make?

the asbestos must go but where do we charge it the fasb rsquo s emerging issues task 692325

The Asbestos Must Go, But Where Do We Charge It?

The FASB’s Emerging Issues Task Force (EITF) considered the question of how the costs incurred in removing asbestos from buildings should be treated (Issue 89–13). This is a widespread issue because studies indicate that some 20% of buildings in the United States contain asbestos. The EITF considered the following specific questions: 1. If a company purchases a building with a known asbestos problem, should the removal costs be expensed or capitalized?

2. If a company discovers an asbestos problem in a building it already owns, should the removal costs be expensed or capitalized?

3. If you had been on the task force, how would you have ruled on these two questions? Why?

why are the costs of buildings different in fasb statement no 34 the fasb called for 692326

Why Are the Costs of Buildings Different?

In FASB Statement No.34, the FASB called for the capitalization of interest costs associated with projects involving the construction or development of assets extending over a significant time period. Interest capitalized is restricted to the amount of interest actually incurred. Consider the case of the following two companies that both constructed a building with a total construction cost of $20 million but chose to finance the construction differently. The costs were incurred evenly over the course of a year; computationally, this is the same as assuming that the entire $20 million was paid halfway through the year.

 

Company A

Company B

Total construction cost of building (excluding interest)                  

$20,000,000

$20,000,000

Company financing (outstanding at year end):

 

 

Construction loan (14%)                                     

20,000,000

0

Common stock issue                                        

0

20,000,000

Total construction loan interest during the year

 

 

(based on average outstanding loan balance)                     

1,400,000

0

As the auditor for both companies, you are asked by your supervisor to prepare a report that calculates the total cost for each building that would be included in each company’s financial statements. Because both companies had the option of purchasing the buildings from a contractor rather than constructing them, your report should include your estimate of the price the contractor would have charged and how you explain the discrepancy in the way cost was determined for the two buildings. Conclude your report by proposing a change in the accounting standards that could eliminate this discrepancy.

brand values on the balance sheet in 1996 financial world magazine estimated and ran 692328

Brand Values on the Balance Sheet?

In 1996, Financial World magazine estimated and ranked the most valuable brand names in the world. Number 11 in the ranking was Gillette with an estimated value of $10.3 billion.

Financial World explained its brand value estimation process for Gillette as follows:

• Estimate the amount of assets used in generating the brand sales. This estimation involves using industry sales to asset ratios. For Gillette, Financial World estimated that 1995 sales of $2.6 billion required the use of $988 million in assets.

• Compute excess return on assets. Financial World assumes that a generic brand name will generate a return on assets of 5%. Gillette’s 1995 operating profit of $961 million exceeds this 5% return by $912 million [$961 million ($988 million assets x 0.05)].

• Estimate after tax return. Financial World puts Gillette’s tax rate at 37%, yielding an after tax excess return of $575 million [$912 million _ (1 _0.37)].

• Multiply the after tax excess return by the brand’s strength multiple. The strength multiple takes into account the brand’s leadership, stability, market size, internationality,trend, and legal protection. Financial World placed Gillette’s strength multiple at17.9 (quite high).This yields the final estimate of the brand value: $10.3 billion ($575million excess after tax return x 17.9).Comment on the relevance and reliability of the $10.3 billion Gillette brand value calculatedby Financial World. Under what circumstances would you be willing to recognizethis value in the financial statements?

deciphering financial statements 3m minnesota mining and manufacturing the 2004 annu 692330

Deciphering Financial Statements (3M: Minnesota Mining and Manufacturing)

The 2004 annual report of Minnesota Mining and Manufacturing (3M) included the following information (all dollar amounts are in millions):

 

2004

2003

From the balance sheet:

 

 

Property, plant, and equipment (net)                                         

$ 5,711

$ 5,609

From the statement of cash flows—operating:

 

 

Depreciation                                                          

835

964

From the statement of cash flows—investing:

 

 

Purchases of property, plant and equipment—outflow                           

(937)

(677)

Proceeds from sale of PP&E and other assets—inflow                          

69

129

From the notes to the financial statements:

 

 

Property, plant, and equipment, at cost                                      

16,290

15,841

Accumulated depreciation                                                

10,579

10,232

1. Using only the net PP&E figures, estimate the book value of the property, plant, and equipment that was sold during the year.

2. Using the individual PP&E and accumulated depreciation accounts, estimate the gain or loss on the disposal of property, plant, and equipment during the year.

researching accounting standards to help you become familiar with the accounting sta 692333

Researching Accounting Standards

To help you become familiar with the accounting standards, this case is designed to take you to the FASB’s Web site and have you access various publications. In this chapter, we discussed the acquisition of an entire business and the accounting ramifications of such transactions. For this case, we will use Statement of FinancialAccounting Standards No.141“Business Combinations.” Open FASB Statement No. 141.

1. Read paragraph 43. The FASB defines goodwill in this paragraph. What is that definition?

2. Read paragraph 45. In the case of negative goodwill, what is to be done with the remaining excess after all designated assets are reduced to zero?

3. Read paragraph 47. What standard outlines the accounting for goodwill and other intangible assets?

ethical dilemma dumping costs into a landfill on st patrick rsquo s day 1992 chamber 692334

Ethical Dilemma (Dumping Costs into a Landfill)

On St. Patrick’s Day 1992, Chambers Development Company, one of the largest landfill and waste management firms in the United States, announced that it had been improperly capitalizing costs associated with landfill development. Chambers announced that it was immediately expensing over $40 million in executive salaries, travel expenses, and public relations costs that had been capitalized as part of the cost of landfills. Wall Street fear over what this move meant for Chambers’ track record of steady earnings growth sent Chambers’ stock price plunging 62% in one day—total market value declined by $1.4 billion. Imagine that it is early 1992 and you have just been assigned to work on the Chambers Development audit. In the course of your audit, you find a number of irregular transactions, including the questionable capitalization of costs as described above. Chambers’ accounting staff tells you that the company has always capitalized these costs. You do a little historical investigation and find that if all the questionable costs had been expensed as you think they should have been, the $362 million expense would completely wipe out all the profit reported by Chambers since it first went public in 1985.You are reluctant to approach your superior, the audit partner on the job, because you know that a large number of the financial staff working for Chambers are former partners in the audit firm you work for. However, you know that ignoring something like this can lead to a catastrophic audit failure. Dr. aft a memo to the audit partner summarizing your findings.

change in estimated units of production on january 1 of year 1 the company purchased 692344

Change in Estimated Units of Production

On January 1 of Year 1, the company purchased a mine for $150,000. At that time, it was estimated that the mine contained 2,000 tons of ore. During Year 1, the company extracted 900 tons of ore from the mine. On January 1 of Year 2,the company spent $60,000 on mine improvements. During Year 2, the company extracted 600 tons of ore. On December 31 of Year 2, it was estimated that the mine contained 700 tons of ore. Compute depletion expense for (1) Year 1 and (2) Year 2.

Q675

Determining Whether a Tangible Asset Is Impaired

The cost and the accumulated depreciation for a piece of equipment are $1,500,000 and $600,000,respectively.Management is concerned that the equipment has become impaired. Management hired several independent appraisers who agreed that the current value of the equipment is $500,000. Management also estimates that the equipment will generate cash inflows of $65,000 per year for the next 14 years. Is the equipment impaired? Explain.

goodwill impairment buyer company acquired target company on january 1 as part of th 692347

Goodwill Impairment

Buyer Company acquired Target Company on January 1. As part of the acquisition, $1,000 in goodwill was recognized; this goodwill was assigned to Buyer’s Manufacturing reporting unit. On December 31, it was estimated that the future cash flows expected to be generated by the Manufacturing reporting unit are $350 at the end of each year for the next 10 years. The appropriate interest rate is 10%. The fair values and book values of the assets and liabilities of the Manufacturing reporting unit are as follows:

 

Book Values

Fair Values

Identifiable assets

$3,500

$4,000

Goodwill 

1,000

?

Liabilities

2,000

2,000

Make the journal entry necessary to recognize any goodwill impairment loss.

classifying an asset as held for sale on october 1 2008 the company has a building w 692348

Classifying an Asset as Held for Sale

On October 1, 2008, the company has a building with a cost of $200,000 and accumulated depreciation of $155,000. The company commits to a plan to sell the building by February 1, 2009. On October 1, 2008, the building has an estimated selling price of $40,000, and it is estimated that selling costs associated with the disposal of the building will be $6,000.On December 31, 2008, the estimated selling price of the building has increased to $60,000, with estimated selling costs remaining at $6,000. Make the journal entries necessary to record (1) the initial classification of the building as held for sale on October 1, 2008, and (2) any adjustment necessary on December 31, 2008. Remember that no depreciation expense is recognized once an asset is classified as held for sale.

inferring useful lives the information that follows is from the balance sheet of ham 692353

Inferring Useful Lives

The information that follows is from the balance sheet of Hampton Company for December 31, 2008, and December 31, 2007.

 

Dec 31, 2008

Dec 31, 2007

Equipment—cost                                           

$ 680,000

$ 680,000

Accumulated depreciation—equipment                          

(250,000)

(160,000)

Buildings—cost                                             

2,450,000

2,450,000

Accumulated depreciation—buildings                            

(340,000)

(230,000)

Hampton did not acquire or dispose of any buildings or equipment during 2008.Hampton uses the straight line method of depreciation. If residual values are assumed to be 10% of asset cost, what is the average useful life of Hampton’s (1) equipment and (2) buildings?

computation of depreciation expense lyman construction purchased a concrete mixer on 692354

Computation of Depreciation Expense

Lyman Construction purchased a concrete mixer on July 15, 2008. Company officials revealed the following information regarding this asset and its acquisition:

Purchase price                                                             

$175,000

Residual value                                                            

$15,000

Estimated useful life                                                        

12 years

Estimated service hours                                                      

40,000

Estimated production in units                                                 

350,000 yards

The concrete mixer was operated by construction crews in 2008 for a total of 4,500 hours, and it produced 41,000 yards of concrete. It is company policy to take a half year’s depreciation on all assets for which it used the straight line or double declining balance depreciation method in the year of purchase. Calculate the resulting depreciation expense for 2008 under each of the following methods, and specify which method allows the greatest depreciation expense.

1. Double declining balance

2. Productive output

3. Service hours

4. Straight line

full cost and successful efforts findit company is an oil and gas exploration firm d 692292

Full Cost and Successful Efforts

Findit Company is an oil and gas exploration firm. During 2008, Findit engaged in 73 different exploratory projects, only 12 of which were successful. The total cost of this exploration effort was $22 million, $4.5 million of which was associated with the successful projects. As of the end of 2008, production had not yet begun at the successful sites.

1. Using the successful efforts method of accounting for oil and gas exploration costs, how much exploration expense would be shown in Findit’s income statement for 2008? How much of the exploration cost will be capitalized and shown as an asset on the company’s balance sheet as of December 31, 2008?

2. Repeat (1) using the full cost method.

classifying expenditures as assets or expenses one of the most difficult problems fa 692293

Classifying Expenditures as Assets or Expenses

One of the most difficult problems facing an accountant is the determination of which expenditures should be capitalized and which should be immediately expensed. What position would you take in each of the following instances?

(a) Painting partitions in a large room recently divided into four sections.

(b) Labor cost of tearing down a wall to permit extension of assembly line.

(c) Replacement of motor on a machine. Life used to depreciate the machine is eight years.

The machine is four years old. Replacement of the motor was anticipated when the machine was purchased.

(d) Cost of grading land prior to construction.

(e) Assessment for street paving.

(f) Cost of tearing down a previously occupied old building in preparation for new construction; old building is fully depreciated.

purchase of a company hull company purchased heaston company for 750 000 cash a sche 692294

Purchase of a Company

Hull Company purchased Heaston Company for $750,000 cash. A schedule of the market values of Heaston’s assets and liabilities as of the purchase date follows.

Heaston Company

Schedule of Asset and Liability Market Values

Assets

 

 

Cash                                                           

$ 5,000

 

Receivables                                                      

78,000

 

Inventory                                                       

136,000

 

Land, buildings, and equipment                                        

436,000

$655,000

Liabilities

 

 

Current liabilities                                                  

$ 80,000

 

Long term debt                                                   

120,000

200,000

Net asset market value                                           

 

 $455,000

1. Make the journal entry necessary for Hull Company to record the purchase.

2. Assume that the purchase price is $385,000 cash. Make the journal entry necessary to record the purchase.

cost classification for a golf course the accountant for stansbury development compa 692300

Cost Classification for a Golf Course

The accountant for Stansbury Development Company is uncertain how to record the following costs associated with the construction of a golf course.

(a) Building artificial lakes.

(b) Moving earth around to enhance the “hilliness” of the course.

(c) Planting fairway grass.

(d) Planting trees and shrubs.

(e) Installing an automatic sprinkler system.

(f) Installing golf cart paths.

(g) Purchasing 50 wooden sand trap rakes (at $1 each).

(h) Paying attorneys’ fees to prepare and file the land title.

(i) Demolishing an old house situated on the site planned for the clubhouse.

Instructions: Indicate which costs should be expensed (E), which should be capitalized and considered to be nondepreciable (CN), and which should be capitalized and depreciated (CD). Include explanations for each classification.

acquisition of land and buildings skyline corporation has decided to expand its oper 692301

Acquisition of Land and Buildings

Skyline Corporation has decided to expand its operations and has purchased land in Salina for construction of a new manufacturing plant. The following costs were incurred in purchasing the property and constructing the building:

Land purchase price                                                           

$ 140,000

Payment of delinquent property taxes                                               

22,000

Title search and insurance                                                       

7,000

City improvements for water and sewer                                            

19,500

Building permit                                                                

6,000

Cost to destroy existing building on land ($5,000 worth of salvaged material

 

used in new building)                                                        

24,000

Contract cost of new building                                                    

1,800,000

Land improvements—landscaping                                                  

76,000

Sidewalks and parking lot                                                        

41,000

Fire insurance on building—1 year                                                 

20,000

The depreciated value of the old building on the books of the company from which the land was purchased was $29,000.The old building was never used by Skyline.

Instructions:

1. Determine the costs of the land and land improvements. Show clearly the elements included in the totals.

2. Determine the cost of the new building. Show clearly the elements included in the total.

acquisition of land and construction of plant bylund corporation was organized in ju 692303

Acquisition of Land and Construction of Plant

Bylund Corporation was organized in June 2008. In auditing its books, you find the following land, buildings, and equipment account:

Account: LAND, BUILDINGS, AND EQUIPMENT

Date

Item

Debit

Credit

Balance

Debit

Credit

2008

 

 

 

 

 

 

June

8

Organization fees paid to the state

21,000

 

21,000

 

 

16

Land site and old building

325,000

 

346,000

 

 

30

Corporate organization costs

40,000

 

386,000

 

July

2

Title clearance fees

15,300

 

401,300

 

Aug.

28

Cost of razing old building

15,000

 

416,300

 

Sept.

1

Salaries of Bylund Corporation executives

100,000

 

516,300

 

 

1

Cost to acquire patent for special equipment

54,000

 

570,300

 

Dec.

12

Stock bonus to corporate promoters, 3,000 shares

 

 

 

 

 

 

of common stock, $50/share market value

150,000

 

720,300

 

 

15

County real estate tax

13,200

 

733,500

 

 

15

Cost of new building completed and occupied

 

 

 

 

 

 

on this date

1,450,000

 

2,183,500

 

An analysis of this account and of other accounts disclosed the following additional information:

(a) The building acquired on June 16, 2008,was valued at $41,000.

(b) The corporation paid $15,000 for the demolition of the old building and then sold the scrap for $7,000 and credited the proceeds to Miscellaneous Revenue.

(c) The corporation executives did not participate in the construction of the new building.

(d) The county real estate tax was for the 6 month period ended December 31, 2008, and was assessed by the county on the land.

Instructions: Prepare journal entries to correct Bylund Corporation’s books.

acquisition of intangible assets in your audit of the books of dyer corporation for 692304

Acquisition of Intangible Assets

In your audit of the books of Dyer Corporation for the year ended September 30, 2008, you found the following items in connection with the company’s patents account:

(a) The company had spent $120,000 during its fiscal year ended September 30, 2007, for research and development costs and debited this amount to its patents account. Your review of the company’s cost records indicated the company had spent a total of $141,500 for the research and development of its patents, of which $21,500 spent in its fiscal year ended September 30, 2007, had been debited to Research and Development Expense.

(b) The patents were issued on April 1,2007. Legal expenses in connection with the issuance of the patents of $14,280 were debited to Legal and Professional Fees Expense.

(c) The company paid a retainer of $15,000 on October 5, 2007, for legal services in connection with a patent infringement suit brought against it. This amount was debited to Deferred Costs.

(d) A letter dated October 15, 2008, from the company’s attorneys in reply to your inquiry as to liabilities of the company existing at September 30, 2008, indicated that a settlement of the patent infringement suit had been arranged. The other party had agreed to Dr. op the suit and to release the company from all future liabilities in exchange for $20,000. Additional fees due to the attorneys amounted to $1,260.

Instructions: From the information given, prepare correcting journal entries as of September 30, 2008.

acquisition of intangible assets transactions during 2008 of the newly organized men 692305

Acquisition of Intangible Assets

Transactions during 2008 of the newly organized Menlove Corporation included the following:

Jan. 2 Paid legal fees of $15,000 and stock certificate costs of $8,300 to complete organization of the corporation.

15 Hired a clown to stand in front of the corporate office for two weeks and hand out pamphlets and candy to create goodwill for the new enterprise. Clown cost, $1,000; pamphlets and candy, $500.

Apr. 1 Patented a newly developed process with costs as follows:

Legal fees to obtain patent   

$42,900

Patent application and licensing fees

6,350

Total                 

$49,250

It is estimated that in six years other companies will have developed improved processes, making the Menlove Corporation process obsolete.

May 1 Acquired both a license to use a special type of container and a distinctive trademark to be printed on the container in exchange for 600 shares of Menlove Corporation no par common stock selling for $50 per share. The license is worth twice as much as the trademark, both of which may be used for six years.

July 1 Constructed a shed for $131,000 to house prototypes of experimental models to be developed in future research projects.

Dec. 31 Incurred salaries for an engineer and chemist involved in product development totaling $175,000 in 2008.

Instructions:

1. Give journal entries to record the preceding transactions. (Ignore amortization of intangible assets.)

2. Present the Intangible Assets section of Menlove Corporation’s balance sheet at December 31, 2008.

basket purchase of noncurrent operating assets wenatcher wholesale company incurred 692306

Basket Purchase of Noncurrent Operating Assets

Wenatcher Wholesale Company incurred the following costs in 2008 for a warehouse acquired on July 1, 2008, the beginning of its fiscal year:

Cost of land                                                                  

$ 90,000

Cost of building                                                               

510,000

Remodeling and repairs prior to occupancy                                           

67,500

Escrow fee                                                                    

10,000

Landscaping

$25,000

Property tax for period prior to acquisition

15,000

Real estate commission 

30,000

The company signed a non interest bearing note for $500,000 on July 1, 2008. The implicit interest rate is 10% compounded semiannually. Payments of $25,000 are to be made semiannually beginning December 31, 2008, for 10 years.

Instructions: Give the required journal entries to record (1) the acquisition of the land and building (assume that cash is paid to equalize the cost of the assets and the present value of the note) and (2) the first two semiannual payments, including amortization of note discount.

income statement for computer software company powersoft company is engaged in devel 692307

Income Statement for Computer Software Company

Powersoft Company is engaged in developing computer software for the small business and home computer market. Most of the computer programmers are involved in developmental work designed to produce software that will perform fairly specific tasks in a userfriendly manner. Extensive testing of the working model is performed before it is released to production for preparation of masters and further testing. As a result of careful preparation, Powersoft has produced several products that have been very successful in the marketplace. The following costs were incurred during 2008:

Salaries and wages of programmers doing research                                     

$265,000

Expenses related to projects prior to establishment of technological feasibility                  

82,200

Expenses related to projects after technological feasibility has been established

 

but before software is available for production                                      

53,800

Amortization of capitalized software development costs from current and prior years            

32,150

Costs to produce and prepare software for sale                                       

49,800

Additional data for 2008:

 

Sales of products for the year                                                     

$675,000

Beginning inventory                                                            

155,000

Portion of goods available for sale sold during year                                     

60%

Instructions: Prepare an income statement for Powersoft for the year 2008. Income tax rate is 35%.

valuation of property at december 31 2007 certain accounts included in the noncurren 692308

Valuation of Property

At December 31, 2007, certain accounts included in the Noncurrent Operating Assets section of Salvino Company’s balance sheet had the following balances:

Land                                                                        

$150,000

Buildings                                                                    

910,000

Leasehold improvements                                                        

500,000

Machinery and equipment                                                        

600,000

During 2008, the following transactions occurred.

(a) Land site 653 was acquired for $1,600,000. Additionally, to acquire the land, Salvino paid a $90,000 commission fee to a real estate agent. Costs of $25,000 were incurred to clear the land. During the course of clearing the land, timber and gravel were recovered and sold for $20,000.

(b) A second tract of land (site 654) with a building was acquired for $700,000.The closing statement indicated that the land value was $510,000 and the building value was $215,000. Shortly after acquisition, the building was demolished at a cost of $30,000. A new building was constructed for $600,000 plus the following costs.

Excavation fees                                                            

$35,000

Architectural design fees                                                     

19,000

Building permit fee                                                         

15,000

Imputed interest on funds used during construction                                 

60,000

(Salvino had no interest bearing debt outstanding during the construction period.)

The building was completed and occupied on September 30, 2008.

(c) A third tract of land (site 655) was acquired for $600,000 and was put on the market for resale.

(d) Extensive work was done to a building occupied by Salvino under a lease agreement that expires on December 31, 2014.The total cost of work was $150,000, which consisted of the following.

Painting ceilings                                  

$ 10,000

(estimated useful life is 1 year)

Performing electrical work                          

60,000

(estimated useful life is 10 years)

Constructing extension to current working area          

80,000 

(estimated useful life is 30 years)

 

$150,000  

 

The lessor paid half of the costs incurred in connection with the extension to the current working area.

(e) During December 2008, costs of $70,000 were incurred to improve leased office space.

The related lease will terminate on December 31, 2010, and is not expected to be renewed.

(f) A group of new machines was purchased under a royalty agreement that provides for payment of royalties based on units of production for the machines. The invoice price of the machines was $90,000, freight costs were $2,000, unloading charges were $2,500, and royalty payments for 2008 were $13,000.

 Instructions:

1. Prepare an analysis of the changes in each of the following balance sheet accounts for 2008. (Disregard the related accumulated depreciation accounts.)

• Land

• Buildings

• Leasehold improvements

• Machinery and equipment

2. List the items in the preceding information that were not used to determine the answer to (1), and indicate where, if at all, these items should be included in Salvino’s financial statements.

capitalization of interest oceanwide enterprises inc is involved in building and ope 692310

Capitalization of Interest

Oceanwide Enterprises, Inc., is involved in building and operating cruise ships. Each ship is identified as a separate discrete job in the accounting records. At the end of 2007, Oceanwide correctly reported $5,400,000 as Construction in Progress on the following jobs.

 

 

Accumulated Costs

 

Completion Date

(including 2007 interest)

Ship

(end of month)

December 31, 2007

340                                  

October 31, 2007*

$2,300,000

341                                  

June 30, 2008

1,150,000

342                                  

September 30, 2008

1,200,000

343                                  

January 31, 2009

750,000

Construction costs for 2008, and the dates the expenditures were made, were as follows:

Ship

Date

Costs

341                                                     

April 1, 2008

$1,200,000

342                                                         

May 1, 2008

1,600,000

343                                                     

July 1, 2008

2,200,000

344                                                     

September 1, 2008

810,000

345                                                     

November 1, 2008

360,000

Ocean wide had the following general liabilities at December 31, 2008:

12%, 5 year note (maturity date—2010)

$2,000,000

10%, 10 year bonds (maturity date—2013)

8,000,000

On January 1, 2008, Ocean wide borrowed $2,000,000 specifically for the construction of ship 343.The loan was for three years with interest at 13%.

Instructions:

1. Compute the maximum interest that can be capitalized in 2008.

2. Compute the weighted average interest rate for the general liabilities for 2008.

3. Compute the interest that Ocean wide should capitalize during 2008.

self construction of equipment american corporation received a 400 000 low bid from 692311

Self Construction of Equipment

American Corporation received a $400,000 low bid from a reputable manufacturer for the construction of special production equipment needed by American in an expansion program. Because its own plant was not operating at capacity, American decided to construct the equipment itself and recorded the following production costs related to the construction:

Services of consulting engineer                                                     

$ 10,000

Work subcontracted                                                           

20,000

Materials                                                                    

200,000

Plant labor normally assigned to production                                          

65,000

Plant labor normally assigned to maintenance                                         

100,000 

Total                                                                     

$395,000  

Management prefers to record the cost of the equipment under the incremental cost method. Approximately 40% of the corporation’s production is devoted to government supply contracts, which are all based in some way on cost. The contracts require that any self constructed equipment be allocated its full share of all costs related to the construction.

The following information also is available.

(a) The preceding production labor was for partial fabrication of the equipment in the plant. Skilled personnel were required and were assigned from other projects. The maintenance labor amount ($100,000) represents the cost of nonproduction plant employees assigned to the construction project. Had these workers not been assigned to construction, the $100,000 cost would still have been incurred for their idle time.

(b) Payroll taxes and employee fringe benefits are approximately 30% of labor cost and are included in manufacturing overhead cost. Total manufacturing overhead for the year was $5,630,000, including the $100,000 maintenance labor used to construct the equipment.

(c) Manufacturing overhead is approximately 50% variable and is applied on the basis of production labor cost. Production labor cost for the year for the corporation’s normal products totaled $6,810,000.

(d) General and administrative expenses include $22,500 of executive salary cost and $10,500 of postage, telephone, supplies, and miscellaneous expenses identifiable with this equipment construction.

Instructions:

1. Compute the amount that should be reported as the full cost of the constructed equipment to meet the requirements of the government contracts.

2. Compute the incremental cost of the constructed equipment.

3. What is the greatest amount that should be capitalized as the cost of the equipment? Why?

asset retirement obligation burns company has purchased land that will serve as a te 692312

Asset Retirement Obligation

Burns Company has purchased land that will serve as a temporary repository for nuclear waste. The site will function for 30 years, at which time Burns will be required to completely decontaminate the land. The purchase price for the land is $500,000. Burns knows that the land will have to be decontaminated but isn’t sure which of several possible approaches will be sufficient to reach the level of decontamination necessary by law. The costs of each approach, and the estimated probability that the approach will be the one used, follow:

Approach 1

10% probability of total decontamination cost of $5,000 at the end of

 

30 years.

Approach 2

20% probability of total decontamination cost of $100,000 at the end of

 

30 years.

Approach 3

70% probability of total decontamination cost of $1,500,000 at the end of

 

30 years.

The appropriate interest rate is 8%.

Instructions: Make the journal entries necessary to record the purchase of the land and the recognition of the asset retirement obligation.

recording goodwill aurora corp acquired payette company on december 31 2008 the foll 692313

Recording Goodwill

Aurora Corp. acquired Payette Company on December 31, 2008. The following information concerning Payette’s assets and liabilities was assembled on the acquisition date:

Per

 

As Adjusted by

Company’s Books

 

Appraisal and Audit

Assets

 

 

Current assets                                 

$ 307,000

$ 340,000

Land, buildings, and equipment (net)                  

179,200

260,000

 

 $ 486,200

 $ 600,000

Liabilities

 

 

Current liabilities                               

(25,000)

(25,000)

Long term liabilities                              

(160,000)

(160,000)

Net assets                                 

 $ 301,200

 $ 415,000

Instructions:

1. Make the journal entry necessary for Aurora Corp. to record the purchase, assuming the purchase price was $1,500,000 in cash.

2. Why might Aurora be willing to pay such a high price for Payette?

3. Repeat (1), assuming the purchase price is $350,000.

4. Repeat (1), assuming the purchase price is $150,000.

cost of self constructed asset brodhead manufacturing company has constructed its ow 692285

Cost of Self Constructed Asset

Brodhead Manufacturing Company has constructed its own special equipment to produce a newly developed product. A bid to construct the equipment by an outside company was received for $1,200,000. The actual costs incurred by Brodhead to construct the equipment were as follows:

Direct material 

$320,000

Direct labor

200,000

It is estimated that incremental overhead costs for construction amount to 140% of direct labor costs. In addition, fixed costs (exclusive of interest) of $700,000 were incurred during the construction period and allocated to production on the basis of total prime costs (direct labor plus direct material). The prime costs incurred to build the new equipment amounted to 35% of the total prime costs incurred for the period. The company follows the policy of capitalizing all possible costs on self construction projects. To assist in financing the construction of the equipment, a $500,000, 10% loan was acquired at the beginning of the 6 month construction period. The company carries no other debt except for trade accounts payable. For simplicity, assume that all construction expenditures took place exactly midway through the project: That is, all expenditures took place with three months remaining in the construction period. Compute the cost to be assigned to the new equipment.

capitalization of interest lodi department stores inc constructs its own stores in t 692286

Capitalization of Interest

Lodi Department Stores, Inc., constructs its own stores. In the past, no cost has been added to the asset value for interest on funds borrowed for construction. Management has decided to correct its policy and desires to include interest as part of the cost of a new store just being completed. Based on the following information, how much interest would be added to the cost of the store (1) in 2008 and (2) in 2009?

Total construction expenditures:

January 2, 2008                                                              

$ 600,000

May 1, 2008                                                                 

600,000

November 1, 2008                                                            

500,000

March 1, 2009                                                               

700,000

September 1, 2009                                                            

400,000

December 31, 2009                                                           

500,000 

 

$3,300,000  

Outstanding company debt:

Mortgage related directly to new store; interest rate, 12%; term,

 

5 years from beginning of construction                                         

$1,000,000

General bond liability:

 

Bonds issued just prior to construction of store; interest rate,

 

10% for 10 years                                                        

$ 500,000

Bonds issued prior to construction; interest rate,

 

8%, mature in 5 years                                                    

$1,000,000

Estimated cost of equity capital                                                   

14%

interest capitalization decision for each of the situations described here indicate 692287

Interest Capitalization Decision

For each of the situations described here, indicate when interest should be capitalized (C) and when it should not be capitalized (NC).

(a) Queen Company is constructing a piece of equipment for its own use. Total construction costs are expected to be $4 million, and the construction period will be 1 month.

(b) Ferney Company is constructing a piece of equipment for sale. Total construction costs are expected to exceed $10 million, and the construction period will be about 15 months. This is a special order. Ferney has never produced a piece of equipment like this before.

(c) Patterson Company is constructing a piece of equipment for its own use. Total construction costs are expected to be $15 million, and the construction period will be about two years. The forecasted total construction cost is only a very rough estimate because Patterson has no system in place to accumulate separately the costs associated with this project.

(d) Savis Company is constructing a piece of equipment for its own use. Total construction costs are expected to be $350, and the construction period will be nine months.

(e) Platt Company is constructing a piece of equipment for sale. Total construction costs are expected to exceed $10 million, and the construction period will be about 15 months.This particular piece of equipment is Platt’s best seller.

(f) Stowell Company is in the process of renovating its corporate office building. The project will cost $7.5 million and will take about 15 months. The building will remain in use throughout the project.

(g) Jackson Company owns a piece of undeveloped land. The land originally cost $21 million. Jackson plans to hold onto the land for three to four years and then develop it into a vacation resort.

research and development costs in 2008 the slidell corporation incurred research and 692290

Research and Development Costs

In 2008, the Slidell Corporation incurred research and development costs as follows:

Materials and equipment                                                        

$160,000

Personnel                                                                   

105,000

Indirect costs                                                                 

60,000 

 

$325,000  

These costs relate to a product that will be marketed in 2009. It is estimated that these costs will be recouped by December 31, 2012.

1. What is the amount of research and development costs that should be expensed in

2008?

2. Assume that of these costs, equipment of $80,000 can be used on other research projects. Estimated useful life of the equipment is five years with no salvage value, and it was acquired at the beginning of 2008. What is the amount of research and development costs that should be expensed in 2008 under these conditions? Assume that depreciation on all equipment is computed on a straight line basis.

what are the r amp d costs pringle company has a substantial research department fol 692291

What Are the R&D Costs?

Pringle Company has a substantial research department. Following are listed, in chronological order, some of the major activities associated with one of Pringle’s research projects.

Project Started

(a) Purchased special equipment to be used solely for this project.

(b) Purchased general equipment that will be usable in Pringle’s normal operations.

(c) Allocated overhead to the project.

Technological Feasibility Established

(d) Purchased more special equipment to be used solely for this project.

(e) Performed tests on an early model of the product.

(f) Allocated overhead to the project.

Product Becomes Ready for Production

(g) Incurred direct production costs.

(h) Allocated overhead to the products.

1. For each activity (a) through (h), indicate whether the cost should be capitalized (C), expensed (E), or included in cost of inventory (I).

2. Repeat (1), assuming that Pringle is a computer software development company.

inventory valuation without records the ma amp pa grocery store has never kept many 692245

Inventory Valuation without Records

The Ma & Pa Grocery Store has never kept many records. The proceeds from sales are used to pay suppliers for goods delivered. When the owners, Donald and Alicia Wride, need some cash, they withDr.aw it from the till without any record of it. The Wrides realize that eventually tax returns must be filed, but for three years, “they just haven’t got around to it.” Finally, the IRS catches up with the Wrides, and an audit of the company records is conducted. The auditor requests the general ledger, special journals, inventory counts, and supporting documentation—very little of which is available. Records of expenditures are extremely sketchy because most expenses are paid in cash. If you were the IRS auditor, what might you do to make a reasonable estimate of income for the company?

silver rsquo s ups and downs in 1979 and 1980 the hunt brothers from texas attempted 692247

Silver’s Ups and Downs

In 1979 and 1980, the Hunt brothers from Texas attempted to corner the world’s silver market. Their hope was to own enough silver to be able to dictate world prices. They made purchase commitments, which locked in the price they would pay for silver. For a while, their plan worked. The price of silver rose, and the Hunt brothers used the silver they owned as collateral to purchase more silver. Their plans were shattered when the price of silver started to decline. From a high in January 1980 of $50.35 an ounce, the price of silver fell to $10.80 in just two months. The silver they were using as collateral decreased in value, requiring the Hunt brothers to provide additional collateral. This collateral was in the form of oil, sugar, and real estate, each of which was faring poorly at the time of the silver crash. At the same time, the purchase commitments they had made required them to buy silver at prices higher than the current market value of silver. The Hunt brothers sought protection in bankruptcy court, and the scheme eventually cost them approximately $4 billion.

1. What are the risks associated with making purchase commitments?

2. Why do accounting standards require that price declines subsequent to the purchase commitment but prior to the actual purchase be recorded immediately?

3. Can firms take any action to reduce their exposure to changing prices?

deciphering financial statements circle k circle k was once one of the largest conve 692250

Deciphering Financial Statements (Circle K)

Circle K was once one of the largest convenience store chains in the United States. Circle K separated its products into two major categories: gasoline and merchandise (Twinkies, beef jerky, soda pop, etc.). Selected financial statement data for the year ended April 30, 1994, follow.

 

Gasoline

Merchandise

Sales

$1,562.5 million

$1,710.3 million

Cost of goods sold

1,372.1 million

1,192.6 million

End of year inventory

26.6 million

93.9 million

1. Compute gross profit percentage for both gasoline and merchandise. Given these numbers, what do you think the attitude of convenience stores is toward automatic pump payment systems that eliminate the need to go into the store to pay for gas?

2. Compute inventory turnover (based on end of year inventory) for both gasoline and merchandise.

3. Compute number of days’ sales in inventory for both gasoline and merchandise. Why do you think the number of days’ sales in gasoline inventory is so much lower than for merchandise?

Selected Circle K financial statement data for the years 1988 and 1993 follow. All numbers are in millions of dollars.

 

 

1993

1988

Sales:

 

 

Gasoline

$1,5041

$ 9646

Merchandise

1,5418

1,6492

Cost of goods sold:

 

 

Gasoline

1,3545

8624

Merchandise

1,0545

1,0308

Ending inventory—total

1312

1910

For 1994, Circle K had total sales of $3,272.8. Purchases for 1994 were $2,554.0.

4. Which set of numbers—1988 or 1993—is likely to give a better estimate of the 1994 gross profit percentage? Explain.

5. Using the gross profit method, estimate Circle K’s inventory as of the end of 1994.

deciphering financial statements 3m minnesota mining and manufacturing company the m 692251

Deciphering Financial Statements (3M: Minnesota Mining and Manufacturing Company)

The Minnesota Mining and Manufacturing Company (3M) gives the following description of its business:

3M’s business has developed from its research and technology in coating and bonding for coated abrasives, the company’s original product. Coating and bonding is the process of applying one material to another, such as abrasive granules to paper or cloth (coated abrasives), adhesives to a backing (pressure sensitive tapes), ceramic coating to granular mineral (roofing granules), glass beads to plastic backing (reflective sheeting), and low tack adhesives to paper (repositionable notes). Familiar 3M products include Scotch tape and the ubiquitous Post it notes. Inventory data from 3M’s 2004 10 K report are as follows (in millions of U.S. dollars):

 

2004

2003

Cost of goods sold                                                          

$9,958

$9,285

Inventories:

 

 

Finished goods                                                           

$ 947

$ 921

Work in process                                                         

614

596

Raw materials                                                           

336

299

Total inventories                                                            

$1,897

$1,816

1. Compute cost of goods manufactured for 2004.

2. Compute total manufacturing costs for 2004.

3. Compute number of days’ sales in inventory for 2004 (use average inventory).Make the calculation using:

(a) Total inventory

(b) Finished goods inventory

4. Of the two numbers you computed in (3), which is more meaningful? Explain.

deciphering financial statements caterpillar and ford motor ford motor automotive an 692252

Deciphering Financial Statements (Caterpillar and Ford Motor)

Ford Motor (automotive) and Caterpillar (heavy equipment) both use the LIFO inventory valuation method. Caterpillar uses it for 80% of its inventories and Ford for 25% of its inventories. Data from the 2004 10 K filings of these two companies follow (in millions of U.S. dollars):

 

Ford

Caterpillar

Cost of goods sold

$135,856

$22,420

LIFO inventory, beginning

9,151

3,047

LIFO inventory, ending

10,766

4,675

LIFO reserve, beginning

996

1,863

LIFO reserve, ending

1,001

2,124

1. For both companies, as of the end of 2004, the existence of a LIFO reserve demonstrates that LIFO inventory is less than it would have been if FIFO had been used. For both companies, compute the ratio of LIFO inventory/FIFO inventory for 2004 ending inventory. Comment on the resulting numbers.

2. For Caterpillar, compute what 2004 cost of goods sold would have been if FIFO had been used.

3. What might have caused Caterpillar’s LIFO reserve to be so much larger than Ford’s?

4. If a company uses FIFO, can you use financial statement data to compute what its cost of goods sold would be using LIFO? Explain.

writing assignment this is not the time for ldquo just in time rdquo you are the ass 692255

Writing Assignment (This is not the time for “Just in Time.”)

You are the assistant controller of Duo Therm Company and are in charge of preparing the financial statements and tax returns. One of your colleagues, the assistant controller in charge of working capital management, has just returned from a 3 day seminar on justin time (JIT) inventory. JIT reduces inventory carrying costs by having arrangements with suppliers to deliver inventory just as it is needed for production or sale. Your colleague is excited about implementing JIT, but you are concerned that not all factors are being considered. Your company has been using LIFO for about 25 years. Prepare a memo to the controller outlining why you think just in time might be a bad idea.

researching accounting standards to help you become familiar with the accounting sta 692256

Researching Accounting Standards

To help you become familiar with the accounting standards, this case is designed to take you to the FASB’s Web site and have you access various publications. In this chapter, we discussed issues relating to inventory. For this case, we will use Statement of Financial Accounting Standards No. 151, “Inventory Costs.” Open FASBStatement No. 151.

1. Read the summary at the beginning of the statement. Previous accounting standards required “idle facility expense, excess spoilage, double freight” etc., to be expensed in the current period if such costs were considered abnormal. How are these costs to be handled with this new accounting standard?

2. In the second paragraph of the summary, the FASB states the primary reason for adDr.essing these inventory costing issues. What is the reason stated?

ethical dilemma lifo and the strategic timing of inventory purchases you have risen 692257

Ethical Dilemma (LIFO and the strategic timing of inventory purchases.)

You have risen fast in Lam Tin Industries and are now in charge of purchasing for the entire company. Lam Tin is a privately held company, and negotiations are currently under way for Lam Tin to be acquired by Kwun Tong Company, a large publicly held firm. It is December, and the final negotiations with Kwun Tong, including the setting of the purchase price, will take place in February after the release of Lam Tin’s audited financial statements for the year ended December 31. You are puzzling over a strange request you received earlier today from Lam Tin’s vice president of finance. She visited your office and asked you to delay your normal December inventory purchases until the first week in January. You explained that this would result in a reduction of year end inventories to less than half their normal year end level. The vice president of finance seemed pleased with this information when she left your office. This request seemed fishy, and you pulled out your copy of Lam Tin’s annual report to check a hunch. Just as you suspected, Lam Tin has been using LIFO for many years and has built up a large LIFO reserve. If you delay the December purchases until January, Lam Tin will liquidate a large portion of its old LIFO layers, resulting in a big increase in reported profit for the year. It is possible that this artificial boost in Lam Tin’s profits might increase the price offered by Kwun Tong in the purchase of Lam Tin. Should you talk over your suspicions with the vice president of finance? With Lam Tin’s independent auditors? With the negotiation team from Kwun Tong? Explain.

cumulative spreadsheet analysis this spreadsheet assignment is a continuation of the 692258

Cumulative Spreadsheet Analysis

This spreadsheet assignment is a continuation of the spreadsheet assignments given in earlier chapters. If you completed those assignments, you have a head start on this one. Refer back to the instructions for preparing the revised financial statements for 2008 as given in (1) of the Cumulative Spreadsheet Analysis assignment in Chapter 3.

1. Skywalker wishes to prepare a forecasted balance sheet, a forecasted income statement, and a forecasted statement of cash flows for 2009. Use the financial statement numbers for 2008 as the basis for the forecast, along with the following additional information.

(a) Sales in 2009 are expected to increase by 40% over 2008 sales of $2,100.

(b) In 2009, Skywalker expects to acquire new property, plant, and equipment costing $240.

(c) The $480 in operating expenses reported in 2008 breaks down as follows: $15 in depreciation expense and $465 in other operating expenses.

(d) No new long term debt will be acquired in 2009.

(e) No cash dividends will be paid in 2009.

(f) New short term loans payable will be acquired in an amount sufficient to make Skywalker’s current ratio in 2009 exactly equal to 2.0.

(g) Skywalker does not anticipate repurchasing any additional shares of stock during 2009.

(h) Because changes in future prices and exchange rates are impossible to predict, Skywalker’s best estimate is that the balance in accumulated other comprehensive income will remain unchanged in 2009.

(i) In the absence of more detailed information, assume that the balances in the investment securities, long term investments, other long term assets, and intangible assets accounts will all increase at the same rate as sales (40%) in 2009.

(j) In the absence of more detailed information, assume that the balance in the other long term liabilities account will increase at the same rate as sales (40%) in 2009.

(k) The investment securities are classified as available for sale securities. Accordingly, cash from the purchase and sale of these securities is classified as an investing activity.

(l) Assume that transactions impacting other long term assets and other long term liabilities accounts are operating activities.

(m) Cash and investment securities accounts will increase at the same rate as sales.

(n) The forecasted amount of accounts receivable in 2009 is determined using the forecasted value for the average collection period. The average collection period for 2009 is expected to be 14.08 days. To make the calculations less complex, this value of 14.08 days is based on forecasted end of year accounts receivable rather than on average accounts receivable.

 

(o) The forecasted amount of inventory in 2009 is determined using the forecasted value for the number of days’ sales in inventory. The number of days’ sales in inventory for 2009 is expected to be 107.6 days. To make the calculations easier, this value of 107.6 days is based on forecasted end of year inventory rather than on average inventory.

(p) The forecasted amount of accounts payable in 2009 is determined using the forecasted value for the number of days’ purchases in accounts payable. The number of days’ purchases in accounts payable for 2009 is expected to be 48.34 days. To make the calculations easier, this value of 48.34 days is based on forecasted end of year accounts payable rather than on average accounts payable.

Clearly state any additional assumptions that you make.

2. Repeat part (1), with the following changes in assumptions:

(a) Number of days’ sales in inventory is expected to be 66.2 days.

(b) Number of days’ sales in inventory is expected to be 150.0 days.

3. Comment on the differences in the forecasted values of cash from operating activities in 2009 under each of the following assumptions about the number of days’ sales in inventory: 107.6 days, 66.2 days, and 150.0 days.

4. Is there any impact on the forecasted level of accounts payable when the number of days’ sales in inventory is changed? Why or why not?

5. What happens to the forecasted level of short term loans payable when the number of days’ sales in inventory is reduced to 66.2 days? Explain.

categories of tangible noncurrent operating assets the following costs were incurred 692262

Categories of Tangible Noncurrent Operating Assets

The following costs were incurred in the most recent year:

(a) Paid $20,000 to purchase a piece of equipment. In addition, paid $1,000 to have the equipment shipped to and installed in its final location. Spent $1,750 to have the equipment tested before beginning its production use. Paid $2,000 for lubrication and normal maintenance during the first year of operation of the equipment.

(b) Paid $100,000 to buy a piece of land. Also paid $10,000 to construct a parking lot and sidewalks.

(c) Paid $50,000 to buy another piece of land. Then paid $10,000 to have an old building demolished and have the land cleared. Paid $125,000 to have a building constructed.

Compute the total cost that should be reported in each of the following categories:

1. Land

2. Buildings

3. Equipment

4. Land Improvements

capitalized interest single year computation the company had the following loans out 692264

Capitalized Interest: Single Year Computation

The company had the following loans outstanding for the entire year:

 

Amount

Interest Rate

Specific construction loan

$100,000

10%

General loan

2,000,000

12

The company began the self construction of a building on January 1. The following expenditures were made during the year:

1 Jan

$100,000

1 May

200,000

1 Nov

300,000

Total

$600,000

Construction was completed on December 31. Compute (1) the amount of interest capitalized during the year and (2) the recorded cost of the building at the end of the year.

capitalized interest journal entry refer to practice 10 6 make the journal entry nec 692265

Capitalized Interest: Journal Entry

Refer to Practice 10 6. Make the journal entry necessary to record total interest paid for the year. Assume that all of the interest was paid in cash on December 31.

Practice 10 6

Capitalized Interest: Single Year Computation

The company had the following loans outstanding for the entire year:

 

Amount

Interest Rate

Specific construction loan

$100,000

10%

General loan

2,000,000

12

The company began the self construction of a building on January 1. The following expenditures were made during the year:

1 Jan

$100,000

1 May

200,000

1 Nov

300,000

Total

$600,000

Construction was completed on December 31. Compute (1) the amount of interest capitalized during the year and (2) the recorded cost of the building at the end of the year.

capitalized interest multiple year computation refer to practice 10 6 assume that co 692266

Capitalized Interest: Multiple Year Computation

Refer to Practice 10 6. Assume that construction was not completed on December 31 of Year 1. Also assume that the same loans were outstanding for all of Year 2.The following expenditure was made during Year 2:

July 1 

$500,000

Final construction was completed on December 31 of Year 2. Compute (1) the amount of interest capitalized during Year 2 and (2) the recorded cost of the building at the end of Year 2.

Practice 10 6

Capitalized Interest: Single Year Computation

The company had the following loans outstanding for the entire year:

 

Amount

Interest Rate

Specific construction loan

$100,000

10%

General loan

2,000,000

12

The company began the self construction of a building on January 1. The following expenditures were made during the year:

1 Jan

$100,000

1 May

200,000

1 Nov

300,000

Total

$600,000

Construction was completed on December 31. Compute (1) the amount of interest capitalized during the year and (2) the recorded cost of the building at the end of the year.

accounting for the acquisition of an entire company james company purchased thomas m 692271

Accounting for the Acquisition of an Entire Company

James Company purchased Thomas Manufacturing for $1,000,000 cash on January 1.The book value and fair value of the assets of Thomas as of the date of the acquisition follow:

 

Book

Fair

 

Value

Value

Cash

$ 10,000

$ 10,000

Accounts receivable

100,000

100,000

Inventory

200,000

300,000

Patent

0

50,000

Property, plant, and equipment

400,000

600,000

Totals

$710,000

$1,060,000

In addition, Thomas had liabilities totaling $400,000 at the time of the acquisition. Thomas has no other separately identifiable intangible assets. Make the journal entry necessary on the books of James Company to record the acquisition.

accounting for negative goodwill refer to practice 10 14 assume that the cash acquis 692272

Accounting for Negative Goodwill

Refer to Practice 10 14. Assume that the cash acquisition price is $500,000 instead of $1,000,000. Make the journal entry necessary on the books of James Company to record the acquisition.

Practice 10 14

Accounting for the Acquisition of an Entire Company

James Company purchased Thomas Manufacturing for $1,000,000 cash on January 1.The book value and fair value of the assets of Thomas as of the date of the acquisition follow:

 

Book

Fair

 

Value

Value

Cash

$ 10,000

$ 10,000

Accounts receivable

100,000

100,000

Inventory

200,000

300,000

Patent

0

50,000

Property, plant, and equipment

400,000

600,000

Totals

$710,000

$1,060,000

In addition, Thomas had liabilities totaling $400,000 at the time of the acquisition. Thomas has no other separately identifiable intangible assets. Make the journal entry necessary on the books of James Company to record the acquisition.

intangibles and a business acquisition buyer company purchased target company for 80 692273

Intangibles and a Business Acquisition

Buyer Company purchased Target Company for $800,000 cash. Target Company had total liabilities of $300,000. Buyer Company’s assessment of the fair values it obtained when it purchased Target Company is as follows:

Cash                                                                       

$100,000

Inventory                                                                     

50,000

In process R&D                                                               

500,000

Assembled workforce                                                           

120,000

Make the journal entry necessary to record this business acquisition.

cost of specific plant items the following expenditures were incurred by peterson en 692276

Cost of Specific Plant Items

The following expenditures were incurred by Peterson Enterprises Co. in 2008:

Purchase of land                                                               

$ 270,000

Land survey                                                                  

4,800

Fees for search of title for land                                                    

500

Building permit                                                                

4,000

Temporary quarters for construction crews                                            

11,200

Payment to tenants of old building for vacating premises                                  

4,450

Razing old building                                                             

41,000

Excavating basement                                                             

13,000

Special assessment tax for street project                                             

2,400

Dividends                                                                   

4,000

Damages awarded for injuries sustained in construction (no insurance was carried)              

8,750

Costs of construction                                                           

2,640,000

Cost of paving parking lot adjoining building                                            

55,000

Cost of shrubs, trees, and other landscaping                                           

36,000

What is the cost of the land, land improvements, and building?

determining cost of patent chen king enterprises inc developed a new machine that re 692277

Determining Cost of Patent

Chen King Enterprises Inc. developed a new machine that reduces the time required to insert the fortunes into its fortune cookies. Because the process is considered very valuable to the fortune cookie industry, Chen King patented the machine. The following expenses were incurred in developing and patenting the machine:

Research and development laboratory expenses                                        

$25,000

Metal used in the construction of the machine                                         

8,000

Blueprints used to design the machine                                               

3,200

Legal expenses to obtain patent                                                   

12,000

Wages paid for the employees’ work on the research, development, and building

 

of the machine (60% of the time was spent in actually building the machine)                

30,000

Expense of Dr. awing required by the patent office to be submitted with the patent application       

1,700

Fee paid to government patent office to process application                               

2,500

One year later, Chen King Enterprises Inc. paid $17,500 in legal fees to successfully defend the patent against an infringement suit by Dr. agon Cookie Co. Give the entries on Chen King’s books indicated by the preceding events. Ignore any amortization of the patent or depreciation of the machine.

purchase on deferred payment contract hi tech industries purchases new electronic eq 692281

Purchase on Deferred Payment Contract

Hi Tech Industries purchases new electronic equipment for its telecommunication system. The contractual arrangement specifies 10 payments of $8,600 each to be made over a 10 year period. If Hi Tech had borrowed money to buy the equipment, it would have paid interest at 9%. Hi Tech’s accountant recorded the purchase as follows:

Equipment

86,000

 

Notes Payable

 

86,000

Prepare the correcting acquisition entry, considering the implicit interest in the purchase.

acquisition of land and building for stock and cash valdilla rsquo s music store acq 692284

Acquisition of Land and Building for Stock and Cash

Valdilla’s Music Store acquired land and an old building in exchange for 50,000 shares of its common stock, par $0.50, and cash of $80,000. The auditor ascertains that the company’s stock was selling for $15 per share when the purchase was made. The following additional costs were incurred to complete the transaction:

Legal cost to complete transaction

$10,000

Property tax for previous year

30,000

Cost of building demolition

21,000

Salvage value of demolished building

(6,000)

What entry should be made to record the acquisition of the property?

lower of cost or market valuation oriental sales co uses the first in first out meth 692220

Lower of Cost or Market Valuation

Oriental Sales Co. uses the first in, first out method in calculating cost of goods sold for three of the products that Oriental handles. Inventories and purchase information concerning these three products are given for the month of August. On August 31, Oriental’s suppliers reduced their prices from the most recent purchase prices by the following percentages: product A, 20%; product B, 10%; product C, 8%. Accordingly, Oriental decided to reduce its sales prices on all items by 10%, effective September 1. Oriental’s selling cost is 10% of sales price. Products A and B have a normal profit (after selling costs) of 30% on sales prices, while the normal profit on product C (after selling cost) is 15% of sales price.

 

 

Product A

Product B

Product C

Aug. 1

Inventory

5,000 units at $6.00

3,000 units at $10.00

6,500 units at $0.90

Aug. 1–15

Purchases

7,000 units at $6.50

4,500 units at $10.50

3,000 units at $1.25

Aug. 16–31

Purchases

3,000 units at $8.00

 

 

Aug. 1–31

Sales

10,500 units

5,000 units

4,500 units

Aug. 31

Sales price

$8.00 per unit

$11.00 per unit

$2.00 per unit

Instructions:

1. Calculate the value of the inventory at August 31, using the lower of cost or market method (applied to individual items).

2. Calculate the FIFO cost of goods sold for August and the amount of inventory write off due to the market decline.

trade ins and repossessed inventory the jamison appliance company began business on 692221

Trade Ins and Repossessed Inventory

The Jamison Appliance Company began business on January 1,2007. The company decided from the beginning to grant allowances on merchandise traded in as partial payment on new sales. During 2008 the company granted trade in allowances of $64,035. The wholesale value of merchandise traded in was $40,875. Trade ins recorded at $39,000 were sold for their wholesale value of $27,000 during the year. The following summary entries were made to record annual sales of new merchandise and trade in sales for 2008.

Accounts Receivable

439,890

 

Trade In Inventory

64,035

 

Sales

 

503,925

Cash

27,000

 

Loss on Trade In Inventory

12,000

 

Trade In Inventory

 

39,000

When a customer defaults on the accounts receivable contract, the merchandise is repossessed. During 2008 the following repossessions occurred:

 

Original Sales
Price

Unpaid Contract
Balance

On 2007 contracts

$37,500

$15,600

On 2008 contracts

24,000

17,800

The wholesale value of these goods is estimated as follows:

(a) Goods repossessed during year of sale are valued at 50% of original sales price.

(b) Goods repossessed in later years are valued at 20% of original sales price.

Instructions:

1. At what values should Jamison Appliance report the trade in and repossessed inventory at December 31, 2008?

2. Give the entry that should have been made to record the repossessions of 2008.

3. Give the entry that is required to correct the trade in summary entries.

inventory transactions mdash journal entries the hansen company values its inventory 692222

Inventory Transactions—Journal Entries

The Hansen Company values its inventory at the lower of FIFO cost or market. The inventory accounts at December 31, 2007, had the following balances.

Raw materials                                                                 

$ 92,000

Work in process                                                               

140,510

Finished goods                                                                 

195,350

The following are some of the transactions that affected the inventory of the Hansen Company during 2008.

Feb. 10 Hansen Company purchases raw materials at an invoice price of $20,000; terms 3/15, n/30. Hansen

Company values inventory at the net invoice price.

Mar. 15 Hansen Company repossesses an inventory item from a customer who was overdue in making payment.

The unpaid balance on the sale is $220. The repossessed merchandise is to be refinished and placed on sale. It is expected that the item can be sold for $350 after estimated refinishing costs of $90. The normal profit for this item is considered to be $65.

Apr. 1 Refinishing costs of $75 are incurred on the repossessed item.

10 The repossessed item is resold for $350 on account, 30% down.

May 30 A sale on account is made of finished goods that have a list price of $670 and a cost of $410. A reduction of $100 off the list price is granted as a trade in allowance. The trade in item is to be priced to sell at $90 as is. The normal profit on this type of inventory is 25% of the sales price.

Dec. 31 The following information is available to adjust the accounts for the annual statements.

(a) The raw materials inventory account has a cost balance of $105,700. Current market value is $99,700.

(b) The finished goods inventory account has a cost balance of $180,250. Current market value is $195,480.

Instructions: Record this information in journal entry form, including any required adjusting entries at December 31, 2008.

inventory fire loss kimbell manufacturing began operations five years ago on august 692223

Inventory Fire Loss

Kimbell Manufacturing began operations five years ago. On August 13, 2008, a fire broke out in the warehouse destroying all inventory and many accounting records relating to the inventory. The information available is presented below. All sales and purchases are on account.

 

January 1, 2008

August 13, 2008

Inventory

$143,850

 

Accounts receivable

130,590

$128,890

Accounts payable

88,140

122,850

Collection on accounts receivable, January 1–August 13

 

753,800

Payments to suppliers, January 1–August 13

 

487,500

Goods out on consignment at August 13, at cost

 

52,900

Summary of previous years’ sales:

 

2005

2006

2007

Sales         

$626,000

$705,000

$680,000

Gross profit on sales

187,800

183,300

231,200

Instructions: Determine the inventory loss suffered as a result of the fire.

interim inventory computation mdash gross profit method the following information wa 692224

Interim Inventory Computation—Gross Profit Method

The following information was taken from the records of Prairie Company.

 

Jan. 1, 2007–Dec. 31, 2007

Jan. 1, 2008–Sept. 30, 2008

Sales

$2,500,000

$1,500,000

Beginning inventory

420,000

785,000

Purchases

2,152,000

1,061,000

Freight in

116,000

72,000

Purchase discounts

30,000

15,000

Purchase returns

40,000

13,000

Purchase allowances

8,000

5,000

Ending inventory

785,000

?

Selling and general expenses

450,000

320,000

Instructions: Using the gross profit method, compute the value to be assigned to the inventory as of September 30, 2008, and prepare an income statement for the 9 month period ending on this date.

inventory theft loss in december 2008 jb masterpiece merchandise inc had a significa 692225

Inventory Theft Loss

In December 2008, JB Masterpiece Merchandise Inc. had a significant portion of its inventory stolen. The company determined the cost of inventory remaining to be $32,400. The following information was taken from the records of the company:

Purchases             

Jan. 1, 2008 to Date of Theft

2007

Purchase returns and allowances

$141,670

$156,430

Sales                 

7,250

6,580

Sales returns and allowances

275,600

283,300

Salaries              

3,400

2,900

Rent         

10,100

12,900

Insurance      

$5,340

$7,120

Utilities       

1,030

1,340

Advertising     

1,115

1,435

Depreciation expense

4,925

3,741

Beginning inventory

1,890

2,106

Purchases             

74,620

69,780

Instructions: Estimate the cost of the stolen inventory.

inventory turnover analysis the following information for valdez industries was take 692227

Inventory Turnover Analysis

The following information for Valdez Industries was taken from the company’s financial statements (amounts in thousands):

 

2008

2007

2006

2005

Sales                                        

$24,000

$18,000

$15,000

$12,000

Cost of goods sold                             

19,600

13,900

10,200

7,200

Inventory                                    

1,400

1,200

910

750

Accounts receivable                            

3,900

3,600

4,100

3,200

Accounts payable                              

2,300

1,200

1,500

1,800

Net income                                  

560

320

510

430

Instructions:

1. Compute the inventory turnover and the number of days’ sales in inventory for the years 2006–2008. Use average inventory in your calculations.

2. Evaluate Valdez’s inventory turnover trend.

computation of lifo inventory with lifo pools the bergman company sells three differ 692228

Computation of LIFO Inventory with LIFO Pools

The Bergman Company sells three different products. Five years ago, management adopted the LIFO inventory method and established three specific pools of goods. Bergman values all incremental layers of inventory at the average cost of purchases within the period. Information relating to the three products for the first quarter of 2008 is given below.

 

Product 400

Product 401

Product 402

Purchases:

 

 

 

January                                 

1,000 @ $1200

500 @ $25

5,000 @ $530

February                               

1,500 @ $1250

250 @ $26

4,850 @ $538

March                                 

1,200 @ $1225

3,500 @ $545

First quarter sales (units)                      

2,850

775

10,750

January 1, 2008, inventory                   

950 @ $1150

155 @ $24

3,760 @ $500

Instructions: Compute the ending inventory value for the first quarter of 2008. (Round unit inventory values to the nearest cent and final inventory values to the nearest dollar.)

dollar value lifo inventory method steve rsquo s repair shop began operations on jan 692229

Dollar Value LIFO Inventory Method

Steve’s Repair Shop began operations on January 1, 2003. After discussing the matter with his accountant, Steve decided dollar value LIFO should be used for inventory costing. Information concerning the inventory of Steve’s Repair Shop is shown below.

 

Inventory at

 

Date

Year End Prices

Year End Index

Dec 31, 2003                                        

$20,500

1.00

Dec 31, 2004                                        

34,000

1.18

Dec 31, 2005                                        

55,600

1.36

Dec 31, 2006                                        

37,800

1.14

Dec 31, 2007                                         

72,250

1.72

Dec 31, 2008                                        

53,900

2.05

Instructions: Compute the inventory value at December 31 of each year under the dollar value

LIFO inventory method, assuming incremental layers are valued using the year end price index.

dollar value lifo retail inventory method in 2005 van hover inc adopted the dollar v 692230

Dollar Value LIFO Retail Inventory Method

In 2005, Van Hover Inc. adopted the dollar value LIFO retail inventory method. The January 1, 2005, price index was 1.00. The following data are available for the 4 year period ending December 31, 2008.

 

Cost

Retail

2005:

 

 

Inventory, January 1                                                  

$148,050

$235,000

Purchases                                                         

393,700

635,000

Sales                                                             

 

590,000

Year end price index                                                 

 

1.12

2006:

 

 

Purchases                                                         

$363,000

$550,000

Sales                                                             

 

579,170

Year end price index                                                 

 

1.08

2007:

 

 

Purchases                                                         

$377,000

$650,000

Sales                                                             

 

641,955

Year end price index                                                 

 

1.09

2008:

 

 

Purchases                                                         

$504,000

$800,000

Sales                                                             

 

762,500

Year end price index                                                 

 

1.12

Instructions: Calculate the inventories to be reported at the end of 2005, 2006, 2007, and 2008. Incremental layers are costed at end of year prices.

purchase commitments on november 17 2008 ur airways entered into a commitment to pur 692231

Purchase Commitments

On November 17, 2008, Ur Airways entered into a commitment to purchase 4,000 barrels of aviation fuel for $180,000 on March 23,2009.Ur entered into this purchase commitment to protect itself against the volatility in the aviation fuel market. By December 31, the purchase price of aviation fuel had fallen to $40 per barrel. However, by March 23, 2009,when Ur took delivery of the 4,000 barrels, the price of aviation fuel had risen to $52 per barrel.

Instructions:

1. Make the journal entry necessary on November 17, 2008, to record the purchase commitment.

2. Make any adjusting entry necessary on December 31, 2008.

3. What type of account (e.g., asset, liability, revenue) is Estimated Loss on Purchase Commitments?

4. Make the journal entry to record the purchase on March 23, 2009. Ur uses a periodic inventory system.

foreign currency transactions charles amp sons a u s computer supplies firm had the 692232

Foreign Currency Transactions

Charles & Sons, a U.S. computer supplies firm, had the following transactions with foreign companies during December 2007:

(a) Goldstar Co., Ltd., a South Korea–based firm, sold 5,000 computer hard Dr. ives to Charles & Sons for 100,000 won per Dr. ive on December 12, 2007. Charles & Sons paid the bill on January 13, 2008.

(b) Charles & Sons sold 2,000 computer hard Dr.ives to a Swiss firm, Lockner Inc., on December 21, 2007. Lockner Inc. agreed to pay $135 per hard Dr. ive. Payment was received by Charles & Sons on February 4, 2008.

(c) Charles & Sons sold 2,400 computer hard Dr.ives to Geopacific, Inc., a company with headquarters in Canada, on December 28, 2007. Geopacific was billed 148 Canadian dollars per Dr.ive. Payment was received on January 10, 2008.

(d) Charles & Sons received 1,000 printers from Printco, a Japanese company, on December 28, 2007. Printco billed Charles & Sons 45,000 yen per printer. Charles & Sons paid the liability on January 14, 2008.

Exchange rates for the above transactions are as follows:

U.S. Dollar Value of 1 Unit of Foreign Currency

 

As of Date of Sale

As of Balance

As of Date of Payment

 

or Purchase

Sheet Date

or Receipt

South Korean won

$0.00103

$0.00112

$0.00115

Swiss franc

0.670

0.632

0.655

Canadian dollar

0.910

0.935

0.905

Japanese yen

0.0075

0.0069

0.0073

Instructions: Prepare the journal entries necessary for Charles & Sons to record each of the above transactions for the following: (1) date of the original transaction, (2) balance sheet date, and (3) date of payment or receipt of cash.

should we adopt lifo you are the controller of the ford steel co the economy enters 692233

Should We Adopt LIFO?

You are the controller of the Ford Steel Co. The economy enters a period of high inflation. Although profits are higher this year than last, you realize that the cost to replace inventory is also higher. You are aware that many companies are changing to the LIFO inventory method to save taxes in the current year, but you are concerned that what goes up will eventually come down, and when prices decline, the LIFO method will result in higher taxes. Because declining prices are usually equated with economic recession, it is likely that the higher taxes will have to be paid at a time when revenues are declining. What factors should you consider before making a change to LIFO? Based on the above considerations, what would you recommend?

what is an inventoriable cost you have been hired by midwestern products co to work 692234

What Is an Inventoriable Cost?

You have been hired by Midwestern Products Co. to work in its accounting department. As part of your assignment, you have been asked to review the inventory costing procedures. In the past, the company has attempted to keep its inventory as low as possible to hedge against future declines in demand. One way of doing this has been to charge off as many costs as can be justified as expenses of the current period. Sales have declined, however, and the controller wants to include as many costs in ending inventory as possible in order to report a better income figure for the current year. Your study shows that the following costs have been consistently treated as period costs for financial reporting purposes:

Depreciation of plant

Fringe payroll benefits for factory personnel

Repairs of equipment

Salaries of supervisors

Warehouse rental for storage of finished products

Pension costs for factory personnel

Training program—all employees

Cafeteria costs—all employees

Interest expense

Depreciation and maintenance of fleet of delivery trucks

Which items do you suggest could be included as part of inventory costs? Evaluate the wisdom and propriety of making the suggested changes.

which method shall we use the white wove corporation began operations in 2008 a summ 692235

Which Method Shall We Use?

The White Wove Corporation began operations in 2008. A summary of the first quarter appears below:

 

Units

Total Cost

Purchases:

 

 

January 2

250

$23,250

February 11

100

9,500

February 20

400

38,400

March 21

200

19,600

March 27

225

22,275

Other data:

 

Sales in

Sales Price

Operating

 

Units

per Unit

Expenses

January                                                     

200

$140

$9,575

February                                                   

225

142

7,820

March                                                      

350

145

7,905

The White Wove Corporation used the LIFO perpetual inventory method and correctly computed an inventory value of $38,300 at the end of the first quarter. Management is considering changing to a FIFO costing method. They have also considered using a periodic system instead of the perpetual system presently being used. You have been hired to assist management in making the decision. What would you advise?

but we do have inventory and it does have problems the mountain top realty company h 692237

But We Do Have Inventory, and It Does Have Problems

The Mountain Top Realty Company has decided to develop the mountain area around Hitown and has purchased several plots of mountainside property. In addition, the company acts as a realtor for existing homes in the area. Greg Hatch has recently graduated from school with an accounting degree and has been hired to work as Mountain Top’s accountant. Greg’s favorite topic in intermediate accounting was inventory, and he’s disappointed that he works for a firm without any inventory and its related problems. Marie Bowman, sales manager, overhears Greg mentioning this to a friend at lunch. “But we do have inventory, Greg, and I think you might be surprised at how many accounting problems a realtor can have with the inventory.”

1. What is the nature of Mountain Top’s inventory?

2. To what problems do you think Marie was referring?

3. Other types of companies have “different” kinds of inventory. What is the strangest kind of “inventory” you have ever heard of?

how well am i really doing fay stocks sells oriental rugs she uses the fifo method o 692238

How Well Am I Really Doing?

Fay Stocks sells oriental rugs. She uses the FIFO method of inventory costing. The inventory available for sale for a particular style of rug is as follows:

Inventory Date

Current Inventory

Cost

June 14

4

@ $1,200 each

June 21

3

@ $1,500 each

July 5

6

@ $1,700 each

On July 31, a wealthy customer purchases three rugs paying $2,600 for each. Fay immediately replaces those rugs with three new rugs at a cost of $2,300 apiece. In addition, Fay immediately pays income tax on the sale at a rate of 40%. (Assume that she has no other expenses.) What is Fay’s net income (after taxes) from the sale of the rugs? What is Fay’s net cash flow from the sale of the rugs, the payment of income taxes, and the subsequent purchase of three new rugs? Why is there a substantial difference between net income and cash flow? What other circumstances can lead to differences like those illustrated in this case?

the steel industry rsquo s lifo problem in the early 1980s the american steel indust 692241

The Steel Industry’s LIFO Problem

In the early 1980s, the American steel industry was experiencing severe financial troubles. An increase in foreign competition as well as advancing technology combined to contribute to the decline of industry profits. The demand for domestic steel was down, and, as a result, many firms laid off workers. However, the use of the LIFO method of accounting for inventory distorted the actual financial position of many firms as illustrated by the following simple example:

USA Steel Co. had the following LIFO inventory layers on January 1, 1982:

Layer 1 (oldest)                                                   

6,000 tons @ $10 per ton

Layer 2                                                        

5,000 tons @ $15 per ton

Layer 3 (newest)                                                 

8,000 tons @ $25 per ton

Assume steel sold for $50 per ton in 1983 and cost $35 per ton to produce. Because of a decrease in demand for domestic steel, USA Steel shut down its production facilities and elected to sell the inventory on hand rather than produce additional inventory.

1. If USA Steel Co. sold 15,000 tons of steel during 1983, what was the gross margin in this simplified example using LIFO?

2. What was USA Steel’s gross margin if the 15,000 tons of steel had been calculated at the current cost of $35 per ton?

3. Does the LIFO gross margin accurately depict the financial situation of USA Steel Co.?

but they won rsquo t buy ducks anymore the bright lite shirt company buys wholesale 692243

But They Won’t Buy Ducks Anymore!

The Bright Lite Shirt Company buys wholesale sweatshirts, nightshirts, T shirts, and other clothing items and, using a novel four color processing system, imprints hunDr.eds of designs on the items. The printed shirts are marketed widely to sports stores, department stores, college campus outlets, variety stores, vacation shops, and so on. Gordon Smith, marketing manager, likes to have a wide variety of products on hand so orders can be promptly met. As the number of designs has grown, so has the inventory. However, the designs often exhibit “fad” characteristics, and the demand for ducks, bears, flowers, or sports heroes can change fairly rapidly. Beverly Patton, the controller, has expressed dismay at the growing inventory and especially the issue of inventory obsolescence. Beverly is now preparing for a meeting with Bright Lite’s external auditor. The auditor is sure to ask for a write down of inventory to the lower of cost or market. Beverly has sent a report to Gordon urging him to reduce his inventory and change his production concept. Gordon is reluctant to change because Bright Lite has developed an excellent reputation for meeting emergency requests for inventory. As Bright Lite’s president, which position will you support: Gordon’s or Beverly’s? Explain.

what value should we place on the clunker the ritchie automobile agency is an exclus 692244

What Value Should We Place on the Clunker?

The Ritchie Automobile Agency is an exclusive agency for the sale of foreign sports cars. As part of its sales strategy, Ritchie allows liberal trade in allowances on the sale of its new cars. A used car division of the company sells these trade ins at a separate location, usually at an amount significantly lower than the trade in allowance. This division is continually showing large losses because the cars are charged to the division at their trade in values. John Lund, manager of the used car division, has requested that the costing procedure be changed and that trade ins be recorded at a price sufficiently below expected retail to allow a reasonable profit to his division. Janet Perry, controller of the agency, acknowledges that some adjustment needs to be made to the inflated trade in values, but she feels that expected retail value should be used without allowance for a profit. What value should be used to record the trade ins?

inventory loss mdash gross profit method on june 30 2008 a flash flood damaged the w 692197

Inventory Loss—Gross Profit Method

On June 30, 2008, a flash flood damaged the warehouse and factory of Dr. ybed Corporation, completely destroying the work in process inventory. There was no damage to either the raw materials or finished goods inventories. A physical inventory taken after the flood revealed the following valuations:

Finished goods

$105,000

Work in process.

0

Raw materials.

51,500

The inventory on January 1, 2008, consisted of the following:

Finished goods                                                              

$118,000

Work in process                                                            

111,000

Raw materials                                                              

54,000 

 

$283,000  

A review of the books and records disclosed that the gross profit margin historically approximated 36% of sales. The sales for the first six months of 2008 were $496,000. Raw materials purchases were $87,000. Direct labor costs for this period were $120,000, and manufacturing overhead has historically been applied at 60% of direct labor. Compute the value of the work in process inventory lost on June 30, 2008.

correction of inventory errors annual income for the stoker co for the period 2004 n 692198

Correction of Inventory Errors

Annual income for the Stoker Co. for the period 2004–2008 appears below. However, a review of the records for the company reveals inventory misstatements as listed. Calculate corrected net income for each year.

 

2004

2005

2006

2007

2008

Reported net income (loss)

$18,000

$13,000

$2,000

$ (5,800)

$16,000

Inventory overstatement, end of year

 

5,500

 

 

3,600

Inventory understatement, end of year

4,500

 

 

10,500

 

effect on net income of inventory errors the martin company reported income before t 692199

Effect on Net Income of Inventory Errors

The Martin Company reported income before taxes of $370,000 for 2007 and $526,000 for 2008. A later audit produced the following information:

(a) The ending inventory for 2007 included 2,000 units erroneously priced at $5.90 per unit. The correct cost was $9.50 per unit.

(b) Merchandise costing $17,500 was shipped to the Martin Company, FOB shipping point, on December 26, 2007. The purchase was recorded in 2007, but the merchandise was excluded from the ending inventory because it was not received until January 4, 2008.

(c) On December 28, 2007, merchandise costing $2,900 was sold to Deluxe Paint Shop. Deluxe had asked Martin in writing to keep the merchandise for it until January 2, when it would come and pick it up. Because the merchandise was still in the store at year end, the merchandise was included in the inventory count. The sale was correctly recorded in December 2007.

(d) Craft Company sold merchandise costing $1,500 to Martin Company. The purchase was made on December 29, 2007, and the merchandise was shipped on December 30. Terms were FOB shipping point. Because the Martin Company bookkeeper was on vacation, neither the purchase nor the receipt of goods was recorded on the books until January 2008.

Assume that all amounts are material and a physical count of inventory was taken every December 31.

1. Compute the corrected income before taxes for each year.

2. By what amount did the total income before taxes change for the two years combined?

3. Assume all errors were found in February 2008, just after the books were closed for 2007.What journal entry would be made? Martin uses a periodic inventory system.

correction of lifo inventory the cardoza products company rsquo s inventory record a 692200

Correction of LIFO Inventory

The Cardoza Products Company’s inventory record appears below.

 

Purchases

Sales

 

Quantity

Unit Cost

Quantity

2006                                              

9,000

$560

6,500

2007                                              

9,500

575

10,000

2008                                              

7,200

582

6,000

The company uses a LIFO cost flow assumption. It reported ending inventories as follows for its first three years of operations:

2006                                                                         

$14,000

2007                                                                        

11,600

2008                                                                        

18,600

Determine if the Cardoza Products Company has reported its inventory correctly.

Assuming that 2008 accounts are not yet closed, make any necessary correcting entries.

inventory turnover the rigby supplement company showed the following data in its fin 692201

Inventory Turnover

The Rigby Supplement Company showed the following data in its financial statements.

 

2008

2007

Cost of goods sold

$1,400,000

$1,125,000

Beginning inventory

275,000

175,000

Ending inventory

405,000

275,000

1. Compute the number of days’ sales in average inventory for both 2007 and 2008. What can you infer from these numbers?

2. How would you interpret the answer to (1) if this company were in the business of selling fresh fruits and vegetables? What if the company sold real estate?

retail inventory method the evening out clothing store values its inventory using th 692202

Retail Inventory Method

The Evening Out Clothing Store values its inventory using the retail inventory method. The following data are available for the month of November 2008:

 

Cost

Retail

Inventory, November 1

$ 53,800

$ 80,000

Purchases

154,304

220,000

Sales

 

244,000

Compute the estimated inventory at November 30, 2008, assuming:

1. FIFO

2. LIFO

3. Average cost

retail inventory method the ivory tower bookstore recently received a shipment of ac 692203

Retail Inventory Method

The Ivory Tower Bookstore recently received a shipment of accounting textbooks from the publisher. Following the receipt of the shipment, the FASB issued a major new accounting standard that related directly to the contents of one chapter of the text. Portions of this chapter became “obsolete” immediately as a result of the FASB’s action. In order to sell the books, the bookstore marked down the selling price and offered a separate supplement covering the new standard, which was provided at no cost by the publisher. Information relating to the cost and selling price of the text for the month of September is given below:

 

Cost

Retail

Beginning inventory                                                     

$ 1,500

$ 1,800

Purchases                                                             

24,000

33,760

Freight in                                                             

1,100

 

Markdowns                                                            

 

2,100

Sales                                                                 

 

27,500

Based on the data given, compute the estimated inventory at the end of the month using the retail inventory method and assuming:

1. Lower of cost or market valuation

2. Average cost valuation

retail inventory method carmel department store uses the retail inventory method on 692204

Retail Inventory Method

Carmel Department Store uses the retail inventory method. On December 31, 2008, the following information relating to the inventory was gathered:

 

Cost

Retail

Inventory, January 1, 2008                                                 

$ 26,550

$ 45,000

Sales                                                                 

 

430,000

Purchases                                                             

309,000

435,000

Purchase discounts                                                      

4,200

 

Freight in                                                             

5,250

 

Markups                                                              

 

30,000

Markdowns                                                            

 

40,000

Sales discounts                                                         

 

5,000

Compute the ending inventory value at December 31, 2008, using:

1. the average cost method.

2. the lower of cost or market method.

computing inventory using lifo pools miller mfg has one lifo pool information relati 692205

Computing Inventory Using LIFO Pools

Miller Mfg. has one LIFO pool. Information relating to the products in this pool is as follows:

Beginning inventory, January 1                                       

60 units

@ $10 each

Purchase, February 12                                             

45 units

@ $12 each

Purchase, February 28                                             

75 units

@ $18 each

Purchase, March 15                                               

65 units @ $1250 each

Sales for the first quarter                                           

 

135 units

Compute the ending LIFO inventory value for the first quarter assuming new layers are valued based on:

1. A FIFO assumption

2. A LIFO assumption

3. An average cost assumption

dollar value lifo inventory method the johnson manufacturing company manufactures a 692206

Dollar Value LIFO Inventory Method

The Johnson Manufacturing Company manufactures a single product. The managers, Ron and Ken Johnson, decided on December 31, 2005, to adopt the dollar value LIFO inventory method. The inventory value on that date using the newly adopted dollar value LIFO method was $500,000. Additional information follows:

 

Inventory at

Year End

Date

Year End Prices

Price Index

Dec 31, 2006                                           

$605,000

1.10

Dec 31, 2007                                           

597,360

1.14

Dec 31, 2008                                           

700,000

1.25

Compute the inventory value at December 31 of each year using the dollar value method, assuming incremental layers are valued at year end prices.

dollar value lifo inventory method jennifer inc adopted dollar value lifo on decembe 692207

Dollar Value LIFO Inventory Method

Jennifer Inc. adopted dollar value LIFO on December 31,2005.Data for 2005–2008 follows:

Inventory and index on the adoption date, December 31, 2005:

Dollar value LIFO inventory                                                    

$250,000

Price index at year end (the base year)                                            

100

Inventory information in succeeding years:

 

Inventory at

Year End

Average

Date

Year End Prices

Price Index

Price Index

Dec 31, 2006                                 

$314,720

1.12

1.04

Dec 31, 2007                                 

361,800

1.20

1.14

Dec 31, 2008                                 

353,822

1.27

1.20

1. Compute the inventory value at December 31 of each year under the dollar value method, assuming new layers are valued using the average price index.

2. Compute the inventory value at December 31, 2008, assuming that dollar value procedures were adopted at December 31, 2006, rather than in 2005. The beginning layer is the December 31, 2006, balance.

dollar value lifo retail method on february 15 2009 rooker madr as amp associates co 692208

Dollar Value LIFO Retail Method

On February 15, 2009, Rooker, MaDr.as & Associates compiled the following information concerning inventory for five years. They used the dollar value LIFO retail inventory method.

 

Year End

 

 

 

 

Price Index

Incremental

Incremental

 

Date

at Retail

Layer Index

Cost Percentage

Inventory

Dec 31, 2004                    

1.00

1.00

71%

$155,000

Dec 31, 2005                    

1.04

1.02

72

188,600

Dec 31, 2006                    

1.14

1.09

64

192,500

Dec 31, 2007                    

1.12

1.11

63

194,200

Dec 31, 2008                    

1.16

1.12

67

195,800

Compute the inventory cost at the end of each year under the dollar value LIFO retail method. (Round all dollar amounts to the nearest dollar.)

foreign currency purchase koreaco produces automobile transmissions which are then s 692210

Foreign Currency Purchase

Koreaco produces automobile transmissions, which are then sent to the United States where they are installed in domestically built cars. CarCo,a U.S. auto company, received a shipment of transmissions on December 15, 2007. The transmissions were subsequently paid for on January 30, 2008. The invoice was denominated in Korean won and totaled 5,000,000 won. The number of Korean won required to purchase 1 U.S. dollar fluctuated as follows:

 

Exchange Rates

December 15, 2007                                                   

800

December 31, 2007                                                   

780

January 30, 2008                                                      

720

Provide the necessary journal entries for CarCo to record the above transactions assuming CarCo’s fiscal year end is December 31.

computing cost of goods sold for a manufacturing firm the following information is a 692211

Computing Cost of Goods Sold for a Manufacturing Firm

The following information is available for Granite Inc.

Products in Sample Inventory

Raw materials:

2008

2007

2006

Beginning inventory                                          

$ 125

$ ?

$ 96

Purchases                                                  

372

410

?

Materials available to use                                      

$ ?

$ ?

$ 463

Ending inventory                                            

136

?

?

Raw materials used                                          

 $ ?

 $ 379

 $ 369

Direct labor                                                   

318

?

307

Manufacturing overhead                                          

398

401

?

Total manufacturing costs                                        

 $ ?

 $1,093

 $1,059

Work in process, January 1                                       

78

?

74

 

 $1,155

 $ ?

 $ ?

Work in process, December 31                                    

?

78

?

Cost of goods manufactured                                      

 $ ?

 $ ?

 $1,052

Finished goods, January 1                                         

?

90

83

 

 $1,167

 $ ?

 $ ?

Finished goods, December 31                                     

?

110

?

Cost of goods sold                                             

 $1,067

 $1,076

 $ ?

Instructions: Compute the missing amounts.

whose inventory is it streuling inc is preparing its 2008 year end financial stateme 692212

Whose Inventory Is It?

Streuling Inc. is preparing its 2008 year end financial statements. Prior to any adjustments, inventory is valued at $76,050. The following information has been found relating to certain inventory transactions:

(a) Goods valued at $11,000 are on consignment with a customer. These goods are not included in the $76,050 inventory figure.

(b) Goods costing $2,700 were received from a vendor on January 5, 2009. The related invoice was received and recorded on January 12, 2009. The goods were shipped on December 31, 2008, terms FOB shipping point.

(c) Goods costing $8,500 were shipped on December 31, 2008, and were delivered to the customer on January 2, 2009. The terms of the invoice were FOB shipping point. The goods were included in ending inventory for 2008 even though the sale was recorded in 2008.

(d) A $3,500 shipment of goods to a customer on December 31, terms FOB destination, was not included in the year end inventory. The goods cost $2,600 and were delivered to the customer on January 8, 2009. The sale was properly recorded in 2009.

(e) An invoice for goods costing $3,500 was received and recorded as a purchase on December 31, 2008. The related goods, shipped FOB destination, were received on January 2, 2009, and thus were not included in the physical inventory.

(f) Goods valued at $6,500 are on consignment from a vendor. These goods are not included in the year end inventory figure.

(g) A $10,500 shipment of goods to a customer on December 30, 2008, terms FOB destination, was recorded as a sale in 2008. The goods, costing $8,200 and delivered to the customer on January 6, 2009, were not included in 2008 ending inventory.

Instructions:

1. Determine the appropriate accounting treatment for each of the preceding items.

Justify your answers.

2. Compute the proper inventory amount to be reported on Streuling Inc.’s balance sheet for the year ended December 31, 2008.

3. By how much would net income have been misstated if no adjustments were made for the above transactions? Ignore income taxes.

inventory computation using different cost flows records of the schwab new products 692214

Inventory Computation Using Different Cost Flows

Records of the Schwab New Products Co. show the following data relative to Product C:

March

2

Inventory                                                 

325 units at $25.50

 

3

Sale                                                      

300 units at $37.50

 

6

Purchase                                                  

300 units at $26.00

 

13

Purchase                                                  

350 units at $27.00

 

20

Sale                                                      

200 units at $35.70

 

25

Purchase                                                  

50 units at $27.50

 

28

Sale                                                      

125 units at $36.00

Instructions: Calculate the inventory balance and the gross profit on sales for the month on each of the following bases.

1. Perpetual FIFO

2. Periodic FIFO

3. Perpetual LIFO

4. Periodic LIFO

5. Perpetual average (Carry calculations to four decimal places and round to three.)

6. Periodic average

inventory calculations mdash lifo and fifo the marci manufacturing co was organized 692215

Inventory Calculations—LIFO and FIFO

The Marci Manufacturing Co. was organized in 2006 to produce a single product. The company’s production and sales records for the period 2006–2008 are summarized below:

 

Units Produced

Sales

 

No of Units

Production Costs

No of Units

Sales Revenue

2006

330,000

$198,000

250,000

$200,000

2007

310,000

201,500

300,000

255,000

2008

290,000

179,800

270,000

210,000

All units produced in a given year are assigned the same average cost.

Instructions: Calculate the gross profit for each of the three years assuming that inventory values are calculated in terms of:

1. LIFO

2. FIFO

computation of inventory from balance sheet and transaction data a portion of the st 692216

Computation of Inventory from Balance Sheet and Transaction Data

A portion of the Stark Company’s balance sheet appears as follows:

 

December 31, 2008

December 31, 2007

Assets:

 

 

Cash                                     

$353,300

$100,000

Notes receivable                            

0

25,000

Inventory                                 

?

199,875

Liabilities:

 

 

Accounts payable                            

?

75,000

Stark Company pays for all operating expenses with cash and purchases all inventory on credit. During 2008, cash totaling $471,700 was paid on accounts payable. Operating expenses for 2008 totaled $220,000. All sales are cash sales. The inventory was restocked by purchasing 1,500 units per month and valued by using periodic FIFO. The unit cost of inventory was $32.60 during January 2008 and increased $0.10 per month during the year. Stark sells only one product. All sales are made for $50 per unit. The ending inventory for 2007 was valued at $32.50 per unit.

Instructions:

1. Compute the number of units sold during 2008.

2. Compute the December 31, 2008, accounts payable balance.

3. Compute the beginning inventory quantity.

4. Compute the ending inventory quantity and value.

5. Prepare an income statement for 2008 (including a detailed Cost of Goods Sold section and ignoring income taxes).

change from fifo to lifo inventory the greenriver manufacturing company manufactures 692218

Change from FIFO to LIFO Inventory

The Greenriver Manufacturing Company manufactures two products: Raft and Float. At December 31, 2007, Greenriver used the FIFO inventory method. Effective January 1, 2008, Greenriver changed to the LIFO inventory method. The retroactive effect of this change is not determinable, and as a result, the ending inventory for 2007 for which the FIFO method was used is also the beginning inventory for 2008 for the LIFO method. Any layers added during 2008 should be costed by reference to the first acquisitions of 2008. The information below was available from Greenriver inventory records for the two most recent years:

 

 

Units

Raft

 

Float

 

 

Unit Cost

Units

Unit Cost

2007 purchases:

 

 

 

 

January 7

5,000

$4.00

22,000

$2.00

April 16

12,000

4.50

 

 

November 8

17,000

5.00

18,500

2.50

December 13

10,000

6.00

 

 

2008 purchases:

 

 

 

 

February 11

3,000

7.00

23,000

3.00

May 20

8,000

7.50

 

 

October 15

20,000

8.00

 

 

December 23

 

 

15,500

3.50

Units on hand:

 

 

 

 

December 31, 2007

15,000

 

14,500

 

December 31, 2008

16,000

 

13,000

 

Instructions: Compute the effect on net income for the year ended December 31, 2008, resulting from the change from the FIFO to the LIFO inventory method. Ignore income taxes.

lower of cost or market valuation witte inc carries four items in inventory the foll 692219

Lower of Cost or Market Valuation

Witte Inc. carries four items in inventory. The following per unit data relate to these items at the end of 2008:

 

 

 

Replacement

Estimated

Selling

Normal

 

Units

Cost

Cost

Sales Price

Cost

Profit

Category 1:

 

 

 

 

 

 

Commodity A

2,500

$10.50

$10.00

$13.00

$1.25

$2.00

Commodity B

1,850

7.00

6.50

9.75

0.75

1.10

Category 2:

 

 

 

 

 

 

Commodity C

4,000

3.00

2.25

4.65

0.75

0.60

Commodity D

2,950

6.50

7.00

7.25

1.25

1.50

Instructions:

1. Calculate the value of the inventory under each of the following methods:

(a) Cost

(b) The lower of cost or market applied to the individual inventory items

(c) The lower of cost or market applied to the inventory categories

(d) The lower of cost or market applied to the inventory as a whole

2. Prepare any journal entries necessary to reflect the proper inventory valuation assuming inventory is valued at:

(a) Cost

(b) The lower of cost or market applied to the individual inventory items

(c) The lower of cost or market applied to the inventory categories

(d) The lower of cost or market applied to the inventory as a whole

markups and markdowns the company reported the following information relating to inv 692167

Markups and Markdowns

The company reported the following information relating to inventory for the month of April:

 

Cost

Retail

Inventory, January 1

$25,000

$50,000

Purchases in January

40,000

70,000

Markups

 

30,000

Markdowns

 

(25,000)

Sales for the month totaled $80,000. Compute the estimated cost of inventory on hand at the end of the month using the average cost assumption.

lifo pools the company has one lifo pool information relating to the products in thi 692168

LIFO Pools

The company has one LIFO pool. Information relating to the products in this pool is as follows:

Beginning inventory, January 1                                           

10 units @ $10 each

Purchase, February 12                                                 

50 units @ $11 each

Purchase, February 28                                                

60 units @ $12 each

Purchase, March 15                                                  

70 units @ $13 each

Sales for the first quarter                                              

160 units

Compute the ending LIFO inventory value for the first quarter assuming new layers are valued based on a LIFO cost assumption.

dollar value lifo retail the company compiled the following information concerning i 692170

Dollar Value LIFO Retail

The company compiled the following information concerning inventory for the current year:

 

Year End

 

 

 

 

Price Index

Incremental

Incremental

Inventory

Date

at Retail

Layer Index

Cost Percentage

at Retail

Jan. 1

1.00

1.00

65%

$ 80,000

Dec. 31

1.10

1.05

70

110,000

Compute the inventory cost at year end using the dollar value LIFO retail method.

identification of inventory costs and categories the records of burtone company cont 692173

Identification of Inventory Costs and Categories

The records of Burtone Company contain the following cost categories. Burtone manufactures exercise equipment and iron weights.

(a) Cost of materials used to repair factory equipment

(b) Depreciation on the fleet of salespersons’ cars

(c) Cost to purchase iron

(d) Salaries of the factory supervisors

(e) Cost of heat, electricity, and insurance for the company office building

(f) Wages of the workers who shape the iron weights

(g) Property taxes on the factory building

(h) Cost of oil for the factory equipment

(i) Salary of the company president

(j) Pension benefits of workers who repair factory equipment

For each category, indicate whether the cost is an inventory cost (I) or if it should be expensed as incurred (E). For each inventory cost, indicate whether the cost is part of direct materials (DM), direct labor (DL), or manufacturing overhead (MOH).

perpetual and periodic inventory systems the following inventory information is for 692174

Perpetual and Periodic Inventory Systems

The following inventory information is for Stevenson Company.

Beginning inventory                                                      

200 units @ $8

Purchases                                                              

350 units @ $8

Ending inventory                                                        

100 units

Sales for the year totaled $5,900. All sales and purchases are on account.

1. Make the journal entries necessary to record purchases and sales during the year assuming a periodic inventory system.

2. Assume that a periodic inventory system is used. Compute cost of goods sold.

3. Assume that a perpetual inventory system is used. The perpetual records indicate that the sales of $5,900 represent 400 units with a total cost of $3,200. Make the journal entries necessary to record purchases, sales, and inventory shrinkage for the year.

inventory computation using different cost flows the webster store shows the followi 692181

Inventory Computation Using Different Cost Flows

The Webster Store shows the following information relating to one of its products.

Inventory, January 1                                                   

300 units @ $1750

Sales, January 8                                                      

200 units

Purchases, January 10                                                   

900 units @ $1800

Sales, January 18                                                     

800 units

Purchases, January 20                                                  

1,200 units @ $1950

Sales, January 25                                                     

1,000 units

What are the values of ending inventory under a periodic inventory system assuming a (1) FIFO, (2) LIFO, and (3) average cost flow? (Round unit costs to three decimal places.)

inventory computation using different cost flows richmond corporation had the follow 692182

Inventory Computation Using Different Cost Flows

Richmond Corporation had the following transactions relating to product AB during September.

Date

 

 

Units

Unit Cost

September

1

Balance on hand

500 units

$50.0

 

6

Purchase

100 units

45.0

 

12

Sale

300 units

 

 

13

Sale

200 units

 

 

18

Purchase

200 units

60.0

 

20

Purchase

200 units

40.0

 

25

Sale

200 units

 

Determine the ending inventory value under each of the following costing methods:

1. FIFO (perpetual)

2. FIFO (periodic)

3. LIFO (perpetual)

4. LIFO (periodic)

comparison of inventory methods dutch truck sales sells semitrailers the current inv 692183

Comparison of Inventory Methods

Dutch Truck Sales sells semitrailers. The current inventory includes the following five semitrailers (identical except for paint color) along with purchase dates and costs:

 

Semitrailer

Purchase Date

Cost

1

April 3, 2008

$73,000

2

April 10, 2008

70,000

3

April 10, 2008

71,000

4

May 4, 2008

77,000

5

May 12, 2008

78,500

On May 20, 2008, a trucking firm purchased semitrailer 3 from Dutch for $86,000.

1. Compute the gross margin on this sale assuming Dutch uses:

(a) FIFO inventory method

(b) LIFO inventory method

(c) Specific identification method

2. Which inventory method do you think Dutch should use? Why?

lifo inventory computation white farm supply rsquo s records for the first three mon 692184

LIFO Inventory Computation

White Farm Supply’s records for the first three months of its existence show purchases of commodity Y2 as follows:

 

Number of Units

Cost

August                                                      

5,500

$28,050

September                                                   

8,000

41,600

October                                                     

5,100

27,030

The inventory of commodity Y2 at the end of October using FIFO is valued at $36,390.

1. Assuming that none of commodity Y2 was sold during August and September, what value would be shown at the end of October if LIFO cost was assumed?

2. If White Farm uses LIFO, what disclosure could it make in its October 31 quarterly report concerning the FIFO value of inventory?

inventory computation from incomplete records a flood recently destroyed many of the 692185

Inventory Computation from Incomplete Records

A flood recently destroyed many of the financial records of Yak Manufacturing Company. Management has hired you to re create as much financial information as possible for the month of July. You are able to find out that the company uses an average cost inventory valuation system. You also learn that Yak makes a physical count at the end of each month in order to determine monthly ending inventory values. By examining various documents you are able to gather the following information:

Ending inventory at July 31                                                  

60,000 units

Total cost of units available for sale in July                                      

$145,210

Cost of goods sold during July                                               

$116,410

Cost of beginning inventory, July 1                                            

$040 per unit

Gross profit on sales for July                                                

$93,590

July purchases:

Date

 

Units

Unit Cost

July

5

55,000

$051

 

11

53,000

050

 

15

45,000

055

 

16

47,000

053

You are asked to provide the following information.

1. Number of units on hand, July 1

2. Units sold during July

3. Unit cost of inventory at July 31

4. Value of inventory at July 31

computation of beginning inventory from ending inventory the killpack company sells 692186

Computation of Beginning Inventory from Ending Inventory

The Killpack Company sells product N. During a move to a new location, the inventory records for product N were misplaced. The bookkeeper has been able to gather some information from the sales records and gives you the data shown below.

July sales:

57,200 units at $10.00

July purchases:

Date

 

Quantity

Unit Cost

July

5

10,000

$650

 

9

12,500

625

 

12

15,000

600

 

25

14,000

620

On July 31, 16,000 units were on hand with a total value of $98,800. Killpack has always used a periodic FIFO inventory costing system. Gross profit on sales for July was $205,875. Reconstruct the beginning inventory (quantity and dollar value) for the month of July.

impact on profit of failure to replace lifo layers harrison lumber company uses a pe 692187

Impact on Profit of Failure to Replace LIFO Layers

Harrison Lumber Company uses a periodic LIFO method for inventory costing. The following information relates to the plywood inventory carried by Harrison Lumber.

Plywood inventory:

Date

 

Quantity

LIFO Costing Layers

May

1

600 sheets

300 sheets at $800

 

 

 

225 sheets at $1100

 

 

 

75 sheets at $1300

Plywood purchases:

May

8                                                         

115 sheets at $1400

 

17                                                         

95 sheets at $1500

 

29                                                         

200 sheets at $1450

All sales of plywood during May were at $20 per sheet. On May 31, there were 360 sheets of plywood in the storeroom.

1. Compute the gross profit on sales for May, as a dollar value and as a percentage of sales.

2. Assume that because of a lumber strike, Harrison Lumber is not able to purchase the May 29 order of lumber until June 10. Assuming sales remained the same, recomputed the gross profit on sales for May, as a dollar value and as a percentage of sales.

3. Compare the results of (1) and (2) and explain the difference.

computation of beginning inventory a note to the financial statements of highland in 692188

Computation of Beginning Inventory

A note to the financial statements of Highland Inc. at December 31, 2008, reads as follows:

Because of the manufacturer’s production problems for our Humdinger Limited line, our inventories were unavoidably reduced. Under the LIFO inventory accounting method currently being used for tax and financial accounting purposes, the net effect of all the inventory changes was to increase pretax income by $1,000,000 over what it would have been had the inventory of Humdinger Limited been maintained at the normal physical levels on hand at the start of the year. The unit purchase price of the merchandise was $25 per unit during the year. Highland Inc. uses the periodic inventory system. Additional data concerning Highland’s inventory were as follows:

Date

Physical Count of Inventory

LIFO Cost of Inventory

1 Jan 08

500,000 units

$ ?

31 Dec 08

400,000 units

$3,600,000

1. What was the unit average cost for the 100,000 units sold from the beginning inventory?

2. What was the reported value for the January 1, 2008, inventory?

income differences mdash fifo vs lifo first in first out has been used for inventory 692189

Income Differences—FIFO vs. LIFO

First in, first out has been used for inventory valuation by the Atwood Co. since it was organized in 2005. Using the data that follow, redetermine the net incomes for each year on the assumption of inventory valuation on the last in, first out basis:

 

2005

2006

2007

2008

Reported net income—FIFO basis

$15,500

$ 40,000

$ 34,250

$ 44,000

Reported ending inventories—FIFO basis

61,500

102,000

126,000

120,000

Ending inventories—LIFO basis

56,500

75,100

95,000

105,000

gross margin differences mdash fifo vs lifo assume the bullock corporation had the f 692190

Gross Margin Differences—FIFO vs. LIFO

Assume the Bullock Corporation had the following purchases and sales of its single product during its first three years of operation.

 

Purchases

Sales

Year

Units

Unit Cost

Units

Unit Price

1                                    

10,000

$10

8,000

$14

2                                    

9,000

12

9,000

17

3                                    

8,000

15

10,000

18

 

 27,000

 

 27,000

 

Cost of goods sold is Bullock’s only expense. The income tax rate is 40%.

1. Determine the net income (after tax) for each of the three years assuming FIFO historical cost flow.

2. Determine the net income (after tax) for each of the three years assuming LIFO historical cost flow.

3. Compare the total net income over the life of the business. How do the different cost flow assumptions affect net income and cash flows over the life of the business? From a cash flow perspective, which cost flow assumption is better? Explain.

lower of cost or market valuation determine the proper carrying value of the followi 692191

Lower of Cost or Market Valuation

Determine the proper carrying value of the following inventory items.

 

 

Replacement

Sales

Selling

Normal

Item

Cost

Cost

Price

Expenses

Profit

Product 561                        

$3.05

$3.00

$3.50

$0.35

$0.20

Product 562                        

0.69

0.72

1.00

0.30

0.04

Product 563                        

0.31

0.24

0.43

0.15

0.07

Product 564                        

0.92

0.70

1.05

0.27

0.05

Product 565                        

0.84

0.82

1.00

0.19

0.09

Product 566                        

1.19

1.25

1.43

0.13

0.09

lower of cost or market valuation the following inventory data are available for nor 692192

Lower of Cost or Market Valuation

The following inventory data are available for Nordic Ski Shop at December 31.

1. Determine the value of ending inventory using the lower of cost or market method applied to (a) individual items and (b) total inventory.

2. Prepare any journal entries required to adjust the ending inventory if lower of cost or market is applied to (a) individual items and (b) total inventory.

 

Cost

Market

Skis

$55,000

$62,000

Boots

42,500

38,000

Ski equipment

18,000

16,500

Ski apparel

10,000

12,000

lower of cost or market valuation newcomer inc values inventories using the lower of 692193

Lower of Cost or Market Valuation

Newcomer, Inc., values inventories using the lower of cost or market method applied to total inventory. Inventory values at the end of the company’s first and second years of operation follow.

 

Cost

Market

Year 1

$58,000

$53,000

Year 2

75,000

73,800

1. Prepare the journal entries necessary to reflect the proper inventory valuation at the end of each year. (Assume Newcomer uses an inventory allowance account.)

2. For Year 1, assume sales were $510,000 and purchases were $440,000.What amount would be reported as cost of goods sold on the income statement for Year 1 if: (a) the inventory decline is reported separately and (b) the inventory decline is not reported separately?

comparison of inventory valuation methods the muhlstein corporation began business o 692194

Comparison of Inventory Valuation Methods

The Muhlstein Corporation began business on January 1, 2008. The following table shows information about inventories, as of December 31, for three consecutive years under different valuation methods. Assume that purchases are $50,000 each year. Using this information and assuming that the same method is used each year, you are to answer each of the questions that follow.

 

 

 

 

Lower of

 

LIFO

FIFO

Market

Cost or Market*

2008                              

$10,200

$10,000

$ 9,600

$ 8,900

2009                              

9,100

9,000

8,800

8,500

2010                              

10,300

11,000

12,000

10,900

1. Which inventory basis would result in the highest net income for 2008?

2. Which inventory basis would result in the highest net income for 2009?

3. Which inventory basis would result in the lowest net income for the three years combined?

4. For the year 2009, how much higher or lower would net income be on the FIFO cost basis than on the lower of cost or market basis?

inventory loss mdash gross profit method on august 15 2008 a hurricane damaged a war 692196

Inventory Loss—Gross Profit Method

On August 15, 2008, a hurricane damaged a warehouse of Rheinhart Merchandise Company. The entire inventory and many accounting records stored in the warehouse were completely destroyed. Although the inventory was not insured, a portion could be sold for scrap. Through the use of the remaining records, the following data are assembled:

Inventory, January 1                                                         

$ 375,000

Purchases, January 1–August 15                                                

1,385,000

Cash sales, January 1–August 15                                                

225,000

Collection of accounts receivable, January 1–August 15                               

2,115,000

Accounts receivable, January 1                                                  

175,000

Accounts receivable, August 15                                                 

265,000

Salvage value of inventory                                                    

5,000

Gross profit percentage on sales                                                 

32%

Compute the inventory loss as a result of the hurricane.

when is the membership fee earned the superb health studio has been operating for fi 692132

When Is the Membership Fee Earned?

The Superb Health Studio has been operating for five years but is presently for sale. It has opened 50 salons in various cities in the United States. The normal pattern for a new opening is to advertise heavily and sell different types of memberships: 1 year, 3 year, and 5 year. For the initial membership fee, members may use the pool, exercise rooms, sauna, and other recreational facilities without charge. If special courses or programs are taken, additional fees are charged; however, members are granted certain privileges, and the fees are less than those charged to outsiders. In addition, $10 a month dues are charged to all members. Nonmembers may use the facilities; however, they must pay a substantial daily charge for services they receive. Your client, Dickson Inc., is considering purchasing the chain of health studios and asks you to give your opinion on its operations. You are provided with financial statements that show a growing revenue and income pattern over the 5 year period. The balance sheet shows that the physical facilities are apparently owned rather than leased. But you are aware that health studios, like all service institutions, have some challenging revenue recognition problems. What questions would you want answered in preparing your report for Dickson?

when is it revenue hertzel advertising agency handles advertising for clients under 692133

When Is It Revenue?

Hertzel Advertising Agency handles advertising for clients under contracts that require the agency to develop advertising copy and layouts and place ads in various media, charging clients a commission of 15% of the media cost as its fee. The agency makes advance billings to its clients of estimate media cost plus its 15% commission. Adjustments to these advances usually are small. Frequently, both the billings and receipt of cash from these billings occur before the period in which the advertising actually appears in the media. A conference meeting is held between officers of the agency and the new firm of CPAs recently engaged to perform annual audits. In this meeting, consideration is given to four possible points for measuring revenue: (1) at the time the advanced billing is made, (2) when payment is received from the client, (3) in the month when the advertising appears in the media, and (4) when the bill for advertising is received from the media, generally in the month following its appearance. The agency has been following the first method for the past several years on the basis that a definite contract exists and the revenue is earned when billed. When the billing is made, an entry is prepared to record the estimated receivable and liability to the media. Estimated expenses related to the contract are also recorded. Adjusting entries are made later for any differences between the estimated and actual amounts. As a member of the CPA firm attending this meeting, how would you react to the agency’s method of recognizing revenue? Discuss the strengths and weaknesses of each of the four methods of revenue recognition, and indicate which one you would recommend for the agency to follow.

which method is appropriate green brothers furniture sells discount furniture and of 692134

Which Method Is Appropriate?

Green Brothers Furniture sells discount furniture and offers easy credit terms. Its margins are not large, but it deals in heavy volume. Its customers are often low income individuals who cannot obtain credit elsewhere. Green Brothers retains the title to the furniture until full payment is received, and it is not uncommon to have 20% of sales be uncollectible. Green Brothers is considering expansion and has hired an independent auditor to review its financial statements prior to obtaining outside funding. The auditor questions the use of accrual accounting as a method for recognizing revenue and suggests that Green Brothers use the installment sales method. The auditor justifies this by stating that because of the high rate of uncollectible, the earnings process is not substantially complete at the point of sale. Financial statements adjusted to the installment sales method result in a 17% decrease in net income for the fiscal year just ended. The chief financial officer for Green Brothers counters that if uncollectible can be estimated, even if that estimate is high, the use of the accrual method is appropriate. The accountant also notes that restated financial statements showing the lower net income figure will make obtaining external funding much more difficult. Which method of revenue recognition would you argue that Green Brothers should use? Why? Remember that your decision could affect this company’s ability to obtain favorable external financing.

the savings amp loan crisis the cost to taxpayers to bail out failed savings and loa 692136

The Savings & Loan Crisis

The cost to taxpayers to bail out failed savings and loan (S&L) companies in the late 1980s has been estimated as high as $500 billion. Reasons for the crisis included mismanagement of resources, management fraud, and unfavorable economic conditions. Another factor contributing to the S&L problems was their revenue recognition techniques. When a loan was made, the associated loan origination fee, often as high as 6% of the loan principal, was recognized immediately as revenue. If a financial institution’s objective was to increase income for the short term, one strategy would be to loan as much money as possible and collect large loan fees. The president of one S&L, Western Savings in Texas, elected to follow this strategy. He enticed investors by promising high yields on certificates of deposit that were federally insured. His telephone operations often netted over $20 million in investments per day. Once the money was received from investors, the president would then loan the money to borrowers and collect loan fees as revenue. These fees and other income were the source of a $3 million dividend to the president over a 2 year period. The problem for taxpayers was that the president was making poor quality loans. Since investors’ deposits were federally insured, the collectability of loans was not a major issue for Western. Million dollar loans were made with no required down payment. Loans were made for more than the full purchase price of properties. As an example, $64 million was loaned to purchase land that two years earlier had sold for $17.2 million. On this particular deal, Western, holding a sixth lien on the property, received $2 million in loan fees. 1. How can a savings and loan company justify recognizing immediately the loan origination fee as revenue rather than recognizing it over the life of the loan? 2. From an accounting point of view, what revenue recognition method should be used when dealing with high risk loans? 3. Why would investors deposit their money in financial institutions that had lending practices like those illustrated in this case? 4. Do external auditors have a responsibility to evaluate the loan practices of the financial institutions that they audit?

college bound was bankruptcy bound high school students know how important it is to 692137

College Bound Was Bankruptcy Bound

High school students know how important it is to perform well on the educational tests required by many colleges and universities as part of the admissions process. In fact, an entire industry has developed to prepare students to take these tests. One company in this industry was College Bound Inc. It was a fast growing company that claimed to be the largest educational counseling firm in the United States with 150 test centers nationally. College Bound was founded by George and Janet Ronkin because they could not find a facility that, in their opinion, could adequately prepare their own son to take the college entrance exams. The company went public in 1988 as a penny stock, and the price of the stock soared to a high of $24 per share in August 1991. In the early months of 1992, however, the SEC began to question many of College Bound’s accounting practices. As a result of its investigations, the SEC determined that much of the rapid growth in revenues reported by College Bound came as a result of “churning bank accounts.” This practice involved transferring funds from the home office’s bank account to various test centers and then back to the home office. College Bound was recognizing as revenue the funds being transferred back from the test centers. The money used for the “churning” was obtained via a convertible note offering in Europe. The result of these practices was to overstate pretax profits for the fiscal year ended August 1991 by 2.5 times, or $5.2 million. The SEC alleged that the Ronkins were transferring large amounts of company money to their personal accounts. In addition to their compensation of $153,846 each, the Ronkins were said to have transferred over $500,000 to Swiss bank accounts during 1991. The court appointed receiver, Joseph Del Raso, who was asked by the courts to monitor College Bound during bankruptcy proceedings, determined that most of the company’s 150 test centers were not profitable by industry standards and closed over 100 centers in May 1992.

1. How would College Bound recognize revenue by simply transferring money from a test center to the home office? What would the journal entry be when the money was transferred from the test centers to the home office?

2. How would the accountant at the home office determine if money being received from a test center was to be recorded as revenue or as repayment of a loan?

keep shipping we need the revenue datarite a maker of computer hardware systems sell 692138

Keep Shipping, We Need the Revenue

Datarite, a maker of computer hardware systems, sells its products to dealers who in turn sell to the final customer. Datarite offers very liberal credit terms and allows its dealers to take up to 90 days to pay. These terms allow dealers to hold larger inventories. As the end of the fiscal year nears, Datarite needs to increase its current ratio and decrease its debt to equity ratio to avoid violating its debt covenants. The president of the company has asked that all dealers be shipped extra inventory. This will increase both sales and accounts receivable, thereby allowing Datarite to remain in compliance with its debt covenants. The chief financial officer remarks that shipping inventory that has not been ordered should be accounted for as consigned inventory rather than revenue. Should the inventory shipments be accounted for as sales or as consigned inventory? Debt covenants exist to protect the interests of creditors. In this instance, are debt covenants effective in monitoring the company’s activities?

deciphering financial statements ben amp jerry rsquo s homemade inc ben amp jerry rs 692143

Deciphering Financial Statements (Ben & Jerry’s Homemade, Inc.)

Ben & Jerry’s Homemade, Inc., an ice cream manufacturer, was acquired by Unilever in 2000. Before that, Ben & Jerry’s was a publicly traded company. Below is the revenue recognition note for Ben & Jerry’s from its 1998 annual report:

Revenue Recognition

The Company recognizes revenue and the related costs when product is shipped. The

Company recognizes franchise fees as income for individual stores when services required by the franchise agreement have been substantially performed and the store opens for business. Franchise fees relating to area franchise agreements are recognized in proportion to the number of stores for which the required services have been substantially performed. Franchise fees recognized as income and included in net sales were approximately $708,000, $553,000, and $301,000 in 1998, 1997, and 1996, respectively.

1. What is the critical event for Ben & Jerry’s sale of ice cream?

2. What is the critical event for Ben & Jerry’s recognition of franchise fee revenue? Note that Ben & Jerry’s deals with two different types of franchise fees.

deciphering financial statements lockheed martin corporation lockheed martin corpora 692144

Deciphering Financial Statements (Lockheed Martin Corporation)

Lockheed Martin Corporation is “engaged in the design, manufacture, integration and operation of a broad array of products and services ranging from aircraft, spacecraft and launch vehicles to energy management, missiles, electronics, and information systems.” As a result,Lockheed has many long term contracts. The following note is taken from Lockheed’s annual report and provides a good summary of the concepts associated with revenue recognition and long term contracts. Sales and earnings—Sales and anticipated profits under long term fixed price production contracts are recorded on a percentage of completion basis, generally using units of delivery as the basis to measure progress toward completing the contract and recognizing revenue. Estimated contract profits are taken into earnings in proportion to recorded sales. Sales under certain long term fixed price contracts which, among other factors, provide for the delivery of minimal quantities or require a substantial level of development effort in relation to total contract value, are recorded upon achievement of performance milestones or using the cost to cost method of accounting where sales and profits are recorded based on the ratio of costs incurred to estimated total costs at completion. Sales under cost reimbursement type contracts are recorded as costs are incurred. Applicable estimated profits are included in earnings in the proportion that incurred costs bear to total estimated costs. Sales of products and services provided essentially under commercial terms and conditions are recorded upon delivery and passage of title. Incentives or penalties related to performance on contracts are considered in estimating sales and profit rates, and are recorded when there is sufficient information to assess anticipated contract performance. Estimates of award fees are also considered in estimating sales and profit rates based on actual awards and anticipated performance. Incentive provisions which increase or decrease earnings based solely on a single significant event are generally not recognized until the event occurs. Amounts representing contract change orders, claims or other items are included in sales only when they can be reliably estimated and realization is probable. When adjustments in contract value or estimated costs are determined, any changes from prior estimates are generally reflected in earnings in the current period. Anticipated losses on contracts are charged to earnings when determined to be probable.

1. How does Lockheed measure its percentage of completion on most of its long term contracts? 2. For the remaining long term contracts, how does Lockheed recognize revenue and profits?

3. If a change in the contract is made, when is that change reflected in revenues?

4. In what periods are the changes in the company’s estimated percentage of completion reflected?

5. If the company determines that the contract will result in a loss, when is that loss recognized?

goods in transit and on consignment the company counted its ending inventory on dece 692151

Goods in Transit and on Consignment

The company counted its ending inventory on December 31. None of the following items were included when the total amount of the company’s ending inventory was computed:

• $15,000 in goods located in the company’s warehouse that are on consignment from another company.

• $20,000 in goods that were sold by the company and shipped on December 30 and were in transit on December 31; the goods were received by the customer on January 2. Terms were FOB destination.

• $30,000 in goods that were purchased by the company and shipped on December 30 and were in transit on December 31; the goods were received by the company on January 2. Terms were FOB shipping point.

• $40,000 in goods that were sold by the company and shipped on December 30 and were in transit on December 31; the goods were received by the customer on January 2. Terms were FOB shipping point.

The company’s reported inventory (before any corrections) was $200,000. What is the correct amount of the company’s inventory on December 31?

schedule of cost of goods manufactured the company reported the following informatio 692152

Schedule of Cost of Goods Manufactured

The company reported the following information for the year:

Ending work in process inventory                                                 

$100,000

Depreciation on factory building                                                 

32,000

Salespersons’ salaries                                                          

27,000

Beginning raw materials inventory                                                 

40,000

Direct labor                                                                

198,000

Factory supervisor’s salary                                                      

56,000

Depreciation on company headquarters building                                      

21,000

Beginning work in process inventory                                               

76,000

Ending raw materials inventory                                                    

34,000

Indirect labor                                                               

36,000

Advertising costs                                                             

50,000

Purchases of raw materials                                                       

230,000

Prepare a schedule of cost of goods manufactured for the year

inventory valuation fifo lifo and average the company reported the following invento 692154

Inventory Valuation: FIFO, LIFO, and Average

The company reported the following inventory data for the year:

 

 

Cost per

 

Units

Unit

Beginning Inventory                                                  

300

$1750

Purchases:

 

 

March 23                                                       

900

1800

September 16                                                   

1,200

1825

Units remaining at year end:                                            

400

 

Compute (1) cost of goods sold and (2) ending inventory assuming (a) FIFO inventory valuation, (b) LIFO inventory valuation, and (c) average cost inventory valuation. The company uses a periodic inventory system.

inventory valuation complications with a perpetual system refer to practice 9 6 assu 692155

Inventory Valuation: Complications with a Perpetual System

Refer to Practice 9 6. Assume that the sales occurred as follows:

 

Units Sold

January 16

100

July 15

600

November 1

1,300

Total

2,000

Compute (1) cost of goods sold and (2) ending inventory assuming (a) FIFO inventory valuation, (b) LIFO inventory valuation, and (c) average cost inventory valuation. The company uses a perpetual inventory system.

Practice 9 6

Inventory Valuation: FIFO, LIFO, and Average

The company reported the following inventory data for the year:

 

 

Cost per

 

Units

Unit

Beginning Inventory                                                  

300

$1750

Purchases:

 

 

March 23                                                       

900

1800

September 16                                                   

1,200

1825

Units remaining at year end:                                            

400

 

Compute (1) cost of goods sold and (2) ending inventory assuming (a) FIFO inventory valuation, (b) LIFO inventory valuation, and (c) average cost inventory valuation. The company uses a periodic inventory system.

lifo layers the company started business at the beginning of year 1 inventory purcha 692156

LIFO Layers

The company started business at the beginning of Year 1. Inventory purchases and sales during the first four years of the company’s business are as follows:

 

Units

Cost

Units

 

Purchased

per Unit

Sold

Year 1                                               

100

$100

80

Year 2                                               

150

150

100

Year 3                                               

150

250

150

Year 4                                               

200

400

160

Compute the company’s ending inventory as of the end of Year 4. The company uses LIFO inventory valuation.

lifo reserve and lifo liquidation refer to practice 9 8 compute the following 1 lifo 692157

LIFO Reserve and LIFO Liquidation

Refer to Practice 9 8. Compute the following:

1. LIFO reserve at the end of Year 4.

2. Cost of goods sold for Year 4.

3. Cost of goods sold for Year 4 assuming that units purchased had been 90 instead of 200.

Practice 9 8

LIFO Layers

The company started business at the beginning of Year 1. Inventory purchases and sales during the first four years of the company’s business are as follows:

 

Units

Cost

Units

 

Purchased

per Unit

Sold

Year 1                                               

100

$100

80

Year 2                                               

150

150

100

Year 3                                               

150

250

150

Year 4                                               

200

400

160

Compute the company’s ending inventory as of the end of Year 4. The company uses LIFO inventory valuation.

lifo and income taxes refer to practice 9 8 assume that the company has no expenses 692158

LIFO and Income Taxes

Refer to Practice 9 8. Assume that the company has no expenses except for cost of goods sold, the selling price per unit is $5 in each year, and that the income tax rate is 40%. Compute the total amount of income taxes owed for Year 1 through Year 4 assuming that (1) the company uses LIFO inventory valuation and (2) the company uses FIFO inventory valuation.

Practice 9 8

LIFO Layers

The company started business at the beginning of Year 1. Inventory purchases and sales during the first four years of the company’s business are as follows:

 

Units

Cost

Units

 

Purchased

per Unit

Sold

Year 1                                               

100

$100

80

Year 2                                               

150

150

100

Year 3                                               

150

250

150

Year 4                                               

200

400

160

Compute the company’s ending inventory as of the end of Year 4. The company uses LIFO inventory valuation.

lower of cost or market the following information pertains to the company rsquo s en 692159

Lower of Cost or Market

The following information pertains to the company’s ending inventory:

 

Original

Selling

Selling

Replacement

Normal

 

Cost

Price

Cost

Cost

Profit

Item A

$ 575

$ 700

$ 50

$ 600

$100

Item B

700

820

80

550

150

Item C

1,180

1,250

100

1,100

300

Apply lower of cost or market accounting to each inventory item individually. What total amount should be reported as inventory in the balance sheet?

lower of cost or market individual vs aggregate refer to practice 9 11 apply lower o 692160

Lower of Cost or Market: Individual vs. Aggregate

Refer to Practice 9 11. Apply lower of cost or market accounting to the inventory as a whole. What total amount should be reported as inventory in the balance sheet?

Lower of Cost or Market

The following information pertains to the company’s ending inventory:

 

Original

Selling

Selling

Replacement

Normal

 

Cost

Price

Cost

Cost

Profit

Item A

$ 575

$ 700

$ 50

$ 600

$100

Item B

700

820

80

550

150

Item C

1,180

1,250

100

1,100

300

Apply lower of cost or market accounting to each inventory item individually. What total amount should be reported as inventory in the balance sheet?

lower of cost or market journal entries the company started business at the beginnin 692161

Lower of Cost or Market Journal Entries

The company started business at the beginning of Year 1. The company applies the lower of cost or market (LCM) rule to its inventory as a whole. Inventory cost and market values as of the end of Year 1 and Year 2 were as follows:

 

Cost

Market
Value

Year 1

$1,000

$800

Year 2

1,700

1,650

The market value numbers already include consideration of the replacement cost, the ceiling, and the floor. Make the journal entry necessary to record the LCM adjustment at the end of (1) Year 1 and (2) Year 2. The company uses an allowance account for any LCM adjustments.

gross profit method on july 23 the company rsquo s inventory was destroyed in a hurr 692163

Gross Profit Method

On July 23, the company’s inventory was destroyed in a hurricane related flood. For insurance purposes, the company must reliably estimate the amount of inventory on hand on

July 23. The company uses a periodic inventory system. The following data have been assembled:

Inventory, January 1                                                          

$1,000,000

Purchases, January 1–July 23                                                    

3,700,000

Sales, January 1–July 23                                                        

5,000,000

Historical gross profit percentages:

 

Last year                                                               

60%

Two years ago                                                           

55%

Estimate the company’s inventory as of July 23 using (1) last year’s gross profit percentage and (2) the gross profit percentage from two years ago.

percentage of completion analysis espiritu construction co has used the cost to cost 692104

Percentage of Completion Analysis

Espiritu Construction Co. has used the cost to cost percentage of completion method of recognizing revenue. Tony Espiritu assumed leadership of the business after the recent death of his father, Howard. In reviewing the records, Tony finds the following information regarding a recently completed building project for which the total contract was $2,000,000.

 

2007

2008

2009

Gross profit (loss)

$75,000

$140,000

($20,000)

Cost incurred

360,000

?

820,000

Espiritu wants to know how effectively the company operated during the last 3 years on this project and, because the information is not complete, has asked for answers to the following questions.

1. How much cost was incurred in 2008?

2. What percentage of the project was completed by the end of 2008?

3. What was the total estimated gross profit on the project by the end of 2008?

4. What was the estimated cost to complete the project at the end of 2008?

percentage of completion accounting the quality construction company was the low bid 692105

Percentage of Completion Accounting

The Quality Construction Company was the low bidder on an office building construction contract. The contract bid was $7,000,000, with an estimated cost to complete the project of $6,000,000.The contract period was 34 months starting January 1, 2007. The company uses the cost to cost method of estimating earnings. Because of changes requested by the customer, the contract price was adjusted downward to $6,700,000 on January 1, 2008. A record of construction activities for the years 2007–2010 is as follows:

 

Actual Cost—

Progress

Cash

Year

Current Year

Billings

Receipts

2007                                          

$2,500,000

$2,100,000

$1,800,000

2008                                         

3,300,000

3,100,000

3,000,000

2009                                         

410,000

1,300,000

1,000,000

2010                                         

 

 

700,000

The estimated cost to complete the contract as of the end of each accounting period is:

2007

$3,500,000

2008

400,000

2009

0

Calculate the gross profit for the years 2007–2009 under the percentage of completion method of revenue recognition.

percentage of completion analysis smokey international inc recently acquired the kur 692106

Percentage of Completion Analysis

Smokey International Inc. recently acquired the Kurtz Builders Company. Kurtz has incomplete accounting records. On one particular project, only the information below is available.

 

2007

2008

2009

Costs incurred during year

$200,000

$250,000

?

Estimated cost to complete

450,000

190,000

$ 0

Recognized revenue

220,000

?

?

Gross profit on contract

?

10,000

(10,000)

Contract price

850,000

 

 

Because the information is incomplete, you are asked the following questions assuming the percentage of completion method is used, an output measure is used to estimate the percentage completed, and revenue is recorded using the actual cost approach.

1. How much gross profit should be reported in 2007?

2. How much revenue should be reported in 2008?

3. How much revenue should be reported in 2009?

4. How much cost was incurred in 2009?

5. What are the total costs on the contract?

6. What would be the gross profit for 2008 if the cost to cost percentage of completion method were used rather than the output measure?

percentage of completion using architect rsquo s estimates central iowa builders inc 692108

Percentage of Completion Using Architect’s Estimates

Central Iowa Builders Inc. entered into a contract to construct an office building and plaza at a contract price of $10,000,000. Income is to be reported using the percentage of completion method as determined by estimates made by the architect. The data below summarize the activities on the construction for the years 2007–2009. For the years 2007–2009, what entries are required to record this information, assuming the architect’s estimate of the percentage completed is used to determine revenue (proportional cost approach)?

 

Actual Cost

Estimated Cost

Percentage Completed—

Project

Collections

Year

Incurred

to Complete

Architect’s Estimate

Billings

on Billings

2007          

        $3,200,000

$6,000,000

25%

$3,300,000

$3,100,000

2008          

        4,300,000

1,600,000

75

4,500,000

2,700,000

2009          

        1,550,000

0

100

2,200,000

4,200,000

percentage of completion method with change orders the build it construction company 692110

Percentage of Completion Method with Change Orders

The Build It Construction Company enters into a contract on January 1, 2008, to construct a 20 story office building for $42,000,000. During the construction period, many change orders are made to the original contract. The following schedule summarizes the changes made in 2008.

Cost Incurred—

 

Estimated

 

2008

 

Cost to Complete

Contract Price

Basic contract                      

$8,000,000

$28,000,000

$42,000,000

Change Order 1                    

50,000

50,000

125,000

Change Order 2                    

0

50,000

0

Change Order 3                    

300,000

300,000

Still to be negotiated;

 

 

 

at least cost

Change Order 4                    

125,000

0

100,000

Compute the revenues, costs, and gross profit to be recognized in 2008, assuming use of the cost to cost method to determine the percentage completed. (Round percentage to two decimal places.)

cost recovery method bailey bats inc had the following sales and gross profit percen 692114

Cost Recovery Method

Bailey Bats Inc. had the following sales and gross profit percentages for the years 2007–2010.

 

Sales

Gross Profit Percentage

2007                                              

$47,000

45%

2008                                              

45,000

42

2009                                              

58,000

47

2010                                              

61,000

49

Historically, 55% of sales are collected in the year of the sale, 30% in the following year, and 10% in the third year. Assuming collections are as projected, give the journal entries for the years 2007–2010, assuming the cost recovery method. (Ignore provision for bad debts.) Prepare a table comparing the gross profit recognized for 2007–2010 using the full accrual method and the cost recovery method.

cost recovery analysis hatch enterprises uses the cost recovery method for all insta 692115

Cost Recovery Analysis

Hatch Enterprises uses the cost recovery method for all installment sales. Complete the following table.

 

2007

2008

2009

Installment sales

$92,000

$103,000

$ (1)

Cost of installment sales

(2)

62,830

74,750

Gross profit percentage

36%

(3)

35%

Cash collections:

 

 

 

2007 sales

27,200

48,300

12,200

2008 sales

 

36,600

(4)

2009 sales

 

 

43,450

Realized gross profit on installment sales

(5)

(6)

19,250

consignment accounting tingey industries sells merchandise on a consignment basis to 692116

Consignment Accounting

Tingey Industries sells merchandise on a consignment basis to dealers. The selling price of the merchandise averages 25% above cost of merchandise. The dealer is paid a 10% commission on the sales price for all sales made. All dealer sales are made on a cash basis. The following consignment sales activities occurred during 2008.

Manufacturing cost of goods shipped on consignment                                  

$250,000

Sales price of merchandise sold by dealers                                          

220,000

Payments made by dealers after deducting commission                                  

139,000

Instructions:

1. Prepare summary entries on the books of the consignor for these consignment sales transactions.

2. Prepare summary entries on the books of the dealer consignee, assuming there is only one dealer involved.

3. Prepare the parts of Tingey Industries’ financial statements at December 31, 2008, that relate to these consignment sales.

contingent rental on january 1 hannah company signed a 1 year rental for a total of 692117

Contingent Rental

On January 1, Hannah Company signed a 1 year rental for a total of $60,000, with quarterly payments of $15,000 due at the end of each quarter. In addition, the renter must pay contingent rent of 4% of all sales in excess of $1,000,000.The contingent rent is paid in one payment on December 31. On March 31, Hannah Company received the first rental payment. At that time, sales for the renter had reached $350,000.The same renter has used the building for the past five years, and in each of those years the renter reached the contingent rent threshold of $1,000,000 in sales. Accordingly, the accountant for Hannah Company recognized total rent revenue of $19,000 for the first quarter—$15,000 collected in cash and another $4,000 in estimated contingent rent. The contingent rent estimate was based on the excess of sales in the quarter over one quarter of the $1,000,000 threshold [($350,000 $250,000) x 0.04]. Sales for the quarter ended June 30 were $300,000, and the accountant for Hannah Company followed the same procedure regarding the contingent rent. Sales in the third quarter were $340,000. However, in the third quarter the accountant for Hannah Company learned that contingent rentals should not be estimated, but instead should be recognized only after the threshold has been reached. The accounting was done correctly in the third quarter, and the appropriate entry was made to correct the mistakes made in the first and second quarters. Sales by the renter in the fourth quarter were $400,000.

Instructions:

Recreate the journal entries made by Hannah Company:

1. In the first quarter.

2. In the second quarter.

3. In the third quarter.

4. In the fourth quarter.

Your entries should include the incorrect entries made in the first and second quarter and the correcting entry made in the third quarter.

construction accounting zamponi rsquo s construction company reports its income for 692118

Construction Accounting

Zamponi’s Construction Company reports its income for tax purposes on a completed contract basis and income for financial statement purposes on a percentage of completion basis. A record of construction activities for 2008 and 2009 follows:

 

 

Cost Incurred—

Estimated Cost

Cost Incurred—

Estimated Cost

Project

Contract Price

2008

to Complete

2009

to Complete

A

$1,450,000

$840,000

$560,000

$480,000

$ 0

B

1,700,000

720,000

880,000

340,000

650,000

C

850,000

160,000

480,000

431,500

58,500

D

1,000,000

 

 

280,000

520,000

General and administrative expenses for 2008 and 2009 were $60,000 for each year and are to be recorded as a period cost.

Instructions:

1. Calculate the income for 2008 and 2009 that should be reported for financial statement purposes.

2. Calculate the income for 2009 to be reported on a completed contract basis.

construction accounting the rushing construction company obtained a construction con 692119

Construction Accounting

The Rushing Construction Company obtained a construction contract to build a highway and bridge over the Snake River. It was estimated at the beginning of the contract that it would take three years to complete the project at an expected cost of $50,000,000. The contract price was $60,000,000. The project actually took four years, being accepted as completed late in 2009. The following information describes the status of the job at the close of production each year.

 

2006

2007

2008

2009

2010

Actual cost incurred                 

$12,000,000

$18,160,000

$14,840,000

$10,000,000

$ 0

Estimated cost to complete            

38,000,000

27,840,000

10,555,555

0

0

Collections on contract               

12,000,000

13,500,000

15,000,000

15,000,000

4,500,000

Billings on contract                  

13,000,000

15,500,000

17,000,000

14,500,000

0

Instructions:

1. What is the revenue, cost, and gross profit recognized for each of the years 2006–2010 under (a) the percentage of completion method and (b) the completed contract method?

2. Give the journal entries for each year assuming that the percentage of completion method is used.

construction accounting the pierson construction corporation contracted with the cit 692121

Construction Accounting

The Pierson Construction Corporation contracted with the City of Plaquemine to construct a dam on the bayou at a price of $14,000,000. Pierson expects to earn $1,270,000 on the contract. The percentage of completion method is to be used, and the completion stage is to be determined by estimates made by the engineer. The following schedule summarizes the activities of the contract for the years 2007–2009.

 

Cost

Engineer’s Estimated

Estimate of

Billings

Collection

Year

Incurred

Cost to Complete

Completion

on Contract

on Billings

2007

$4,300,000

$8,560,000

33%

$4,000,000

$3,600,000

2008

4,100,000

4,700,000

62

5,000,000

5,100,000

2009

4,550,000

0

100

5,000,000

5,300,000

Instructions:

1. Prepare a schedule showing the revenue, cost, and the gross profit earned each year under the percentage of completion method, using the engineer’s estimate as the measure of completion to be applied to revenues and costs.

2. Prepare all journal entries required to reflect the contract.

3. Prepare journal entries for 2009, assuming the completed contract method is used.

4. How would the journal entries in (2) differ if the actual costs incurred were used to calculate cost for the period instead of the engineer’s estimate?

construction accounting jana crebs is a contractor for the construction of large off 692122

Construction Accounting

Jana Crebs is a contractor for the construction of large office buildings. At the beginning of 2008, three buildings were in progress. The following data describe the status of these buildings at the beginning of the year:

 

 

Costs Incurred to

Estimated Cost to

 

Contract Price

Jan 1, 2008

Complete as of Jan 1, 2008

Building 1                

$ 4,000,000

$2,070,000

$1,380,000

Building 2                

9,000,000

6,318,000

1,782,000

Building 3                

13,150,000

3,000,000

9,000,000

During 2008, the following costs were incurred.

Building 1 $930,000 (estimated cost to complete as of December 31, 2008, $750,000)

Building 2 $1,800,000 (job completed)

Building 3 $7,400,000 (estimated cost to complete as of December 31, 2008,$2,800,000)

Building 4 $800,000 (contract price,$2,500,000;estimated cost to complete as of December

31, 2008, $1,200,000)

Instructions:

1. Compute the total revenue, costs, and gross profit in 2008. Assume that Crebs uses the cost to cost percentage of completion method. (Round to two decimal places for percentage completed.)

2. Compute the gross profit for 2008 if Crebs uses the completed contract method.

construction accounting the power construction company was the low bidder on a speci 692123

Construction Accounting

The Power Construction Company was the low bidder on a specialized equipment contract. The contract bid was $6,000,000 with an estimated cost to complete the project of $5,300,000. The contract period was 33 months, beginning January 1, 2007. The company uses the cost to cost method to estimate profits. A record of construction activities for the years 2007–2010 follows:

 

Actual Cost—

Progress

Cash

Year

Current Year

Billings

Receipts

2007                                            

$3,400,000

$3,200,000

$3,000,000

2008                                            

2,550,000

2,000,000

2,000,000

2009                                            

200,000

800,000

600,000

2010                                             

0

0

400,000

The estimated cost to complete the contract at the end of each accounting period is:

2007

$2,100,000

2008

150,000

2009

0

Instructions:

1. What are the revenue, cost, and gross profit recognized for each of the years 2007–2009 under the percentage of completion method?

2. Give the journal entries for each of the years 2007–2009 to record the information from (1).

3. Give the journal entries in 2010 to record any collections and to close out all construction accounts.

construction accounting tuscany boat builders was recently awarded a 17 000 000 cont 692124

Construction Accounting

Tuscany Boat builders was recently awarded a $17,000,000 contract to construct a luxury liner for Queen Cruise liners Inc. Tuscany estimates it will take 42 months to complete the contract. The company uses the cost to cost method to estimate profits. The following information details the actual and estimated costs for the years 2007–2010.

 

Actual Cost—

Estimated Cost

Year

Current Year

to Complete

2007

$6,400,000

$8,700,000

2008

5,200,000

4,600,000

2009

4,100,000

1,500,000

2010

1,000,000

0

Instructions:

1. Compute the revenue, cost, and gross profit to be recognized for each of the years 2007–2010 under the percentage of completion method.

2. Give the journal entries for each of the years 2007–2010 to record the information from (1).

installment sales accounting london corporation has been using the cash method to ac 692125

Installment Sales Accounting

London Corporation has been using the cash method to account for income since its first year of operation in 2008. All sales are made on credit with notes receivable given by the customers. The income statements for 2008 and 2009 included the following amounts:

 

2008

2009

Revenues—collection on principal

$32,000

$50,000

Revenues—interest

3,600

5,500

Cost of goods purchased*

45,200

52,020

The balances due on the notes at the end of each year were as follows:

 

2008

2009

Notes receivable (gross)—2008

$62,000

$36,000

Notes receivable (gross)—2009

0

60,000

Unearned interest revenue—2008

7,167

5,579

Unearned interest revenue—2009

0

8,043

Instructions: Give the journal entries for 2008 and 2009 assuming the installment sales method was used rather than the cash method.

installment sales knight rsquo s furniture sells furniture and electronic items the 692126

Installment Sales

Knight’s Furniture sells furniture and electronic items. The majority of its business is on credit, and the following information is available relating to sales transactions for 2007, 2008, and 2009.

 

2007

2008

2009

Installment sales (net of interest)

$102,000

$111,000

$124,000

Gross profit percentage

37%

40%

39%

Cash collections on installment sales:

 

 

 

Principal—2007

$51,600

$30,150

$16,000

Principal—2008

 

68,520

30,200

Principal—2009

 

 

75,130

Interest—2007

6,720

13,250

2,810

Interest—2008

 

5,460

17,163

Interest—2009

 

 

5,977

Instructions: Prepare the journal entries for the years 2007–2009 assuming Knight’s uses the installment sales method for revenue recognition and records receivables net of interest.

revenue recognition analysis the wasatch construction company entered into a 4 500 0 692127

Revenue Recognition Analysis

The Wasatch Construction Company entered into a $4,500,000 contract in early 2008 to construct a multipurpose recreational facility for the city of Helper. Construction time extended over a 2 year period. The table below describes the pattern of progress payments made by the city of Helper and costs incurred by Wasatch Construction by semiannual periods. Estimated costs of $3,600,000 were incurred as expected.

 

Progress Payments

Progress Cost

Period

for Period

for Period

(1) Jan 1–June 30, 2008                               

$ 750,000

$ 900,000

(2) July 1–Dec 31, 2008                               

1,050,000

1,200,000

(3) Jan 1–June 30, 2009                               

1,950,000

1,080,000

(4) July 1–Dec 31, 2009                               

750,000

420,000

Total                                             

 $4,500,000

 $3,600,000

The Wasatch Construction Company prepares financial statements twice each year,

June 30 and December 31.

Instructions:

1. Based on the foregoing data, compute the amount of revenue, costs, and gross profit for the four semiannual periods under each of the following methods of revenue recognition:

(a) Percentage of completion

(b) Completed contract

(c) Installment sales (gross profit only)

(d) Cost recovery (gross profit only)

2. Which method do you feel best measures the performance of Wasatch on this contract?

sample cpa exam questions 1 which of the following is used in calculating the income 692128

Sample CPA Exam Questions

1. Which of the following is used in calculating the income recognized in the fourth and final year of a contract accounted for by the percentage of completion method?

 

Actual Total

Income Previously

 

Cost

Recognized

(a)

Yes

Yes

(b)

Yes

No

(c)

No

Yes

(d)

No

No

2. When should a lessor recognize in income a nonrefundable lease bonus paid by a lessee upon signing an operating lease?

(a) When received

(b) At the inception of the lease

(c) At the expiration of the lease

(d) Over the life of the lease

let rsquo s spread our losses too the abbott construction company has several contra 692130

Let’s Spread Our Losses, Too!

The Abbott Construction Company has several contracts to build sections of freeways, bridges, and dams. Because most of these contracts require more than one year to complete, the accountant, Dave Allred, has recommended use of the percentage of completion method to recognize revenue and income on these contracts. The president, Kathy Bahr, isn’t quite sure how the accounting method works, and she indicates concern about the impact of this decision on income taxes. Bahr also inquires as to what happens when a contract results in a loss. When told by Allred that any estimated loss must be recognized when it is first identified, Bahr becomes upset. “If it is a percentage of completion method and we are recognizing profits in part as we go along, why shouldn’t we be able to do the same for losses?” How would you, as the accountant, answer Bahr’s concerns?

deciphering financial statements xerox as indicated in the case at the beginning of 691983

Deciphering Financial Statements (Xerox)

As indicated in the case at the beginning of this chapter, Xerox was manipulating income between the years 1997 through 1999. Below are revenue, gross profit, net income, and operating cash flow data for Xerox for the years 1997 through 2000.

(in millions)

2000

1999

1998

1997

Revenues                                             

$18,701

$19,228

$19,447

$18,144

Gross profit                                          

7,601

9,003

9,580

9,036

Net income (loss)                                      

(257)

1,424

395

1,452

Operating cash flow                                    

(663)

1,224

(1,165)

472

Proceeds from securitization

 

 

 

 

of finance receivables                                 

0

1,495

0

0

The securitization of the finance receivables represents the sale of receivables to a third party. The cash inflow from the sale was shown in the Operating Activities section of the statement of cash flows. Using these data, identify evidence that proves Xerox was managing its reported earnings during this period.

writing assignment why did we manage earnings you are the controller for cam ry indu 691984

Writing Assignment (Why did we manage earnings?)

You are the controller for Cam Ry Industries. Your company has recently received a large amount of unfavorable publicity because an SEC investigation uncovered a systematic 2 year effort by Cam Ry’s management to manipulate reported earnings. The primary motivation for this earnings management scheme was to consistently meet analysts’ earnings expectations in order to keep the opinion of your company high in advance of an additional share offering that was to take place next year. The SEC has now formally sanctioned your company and fined it $350,000, and the investor backlash has lowered the company’s share price and resulted in the cancellation of the planned share offering next year. As controller, you were aware of the earnings management scheme. You failed to actively oppose the effort. However, the Dr. iving force behind the scheme was the former chief executive officer (CEO) who has now been replaced. Your former auditor has also been fired. The new CEO is attempting to mend all of the stakeholder relationships that have been strained because of the SEC revelation of the earnings management activity. The new CEO has assigned you to repair relations with Yosef Bank. The bank has provided a line of credit to Cam Ry for over 15 years. You have personally represented Cam Ry in its dealings with the bank. For the past 5 years, you have met frequently with Dee Ann Martinez who is a senior vice president with the bank and the person assigned to the Cam Ry account. Through mutual friends, you have heard that Martinez feels personally betrayed by you and no longer trusts you. You are scheduled to meet with Dee Ann Martinez next week to discuss Cam Ry’s line of credit. In advance of that meeting, you have decided to write a 1 page memo to Martinez in an attempt to mend your relationship. Write a Dr. aft of that memo.

researching accounting standards to help you become familiar with the accounting sta 691985

Researching Accounting Standards

To help you become familiar with the accounting standards, this case is designed to take you to the FASB’s Web site and have you access various publications.

In this chapter, we discussed earnings management and how companies might be tempted to inappropriately increase earnings by either overstating revenues or by understating expenses. For this case, we will use Statement of Financial Accounting Standards No. 48, “Revenue Recognition When Right of Return Exists.” Open FASB Statement No. 48.

1. Read paragraph 4. If a customer returns an item that is defective, do the provisions of this accounting standard apply?

2. Read paragraph 6. Part (f) of this paragraph indicates that the amount of future returns must be reasonably estimable if revenue is to be recognized. However, footnote 3 indicates that certain returns are not considered returns as far as this standard is concerned. What types of returns are excluded?

3. Read paragraph 8. Future returns must be estimable. This paragraph identifies instances when a reliable estimate may be unavailable. Identify two instances where the ability to estimate returns would be impaired.

ethical dilemma what should you do with unpleasant and unwelcome audit evidence you 691986

Ethical Dilemma (What should you do with unpleasant and unwelcome audit evidence?)

You are a manager with Doman & Detmer, a mid sized local accounting firm. You have been with the firm for six years. Currently, you are working on the McMahon Company audit engagement. You are supervising a team of seven staff and senior accountants. Your direct supervisor, Giff Nielsen, is the partner in charge of the engagement. You were involved with the economic analysis of McMahon Company that was undertaken during the audit planning stage. A number of indicators suggest that McMahon has suffered a substantial downturn in its business this year. Accordingly, you are being very careful to see whether this downturn is properly reflected in the reported financial statement numbers. In scrutinizing McMahon’s sales near the end of the fiscal year, your audit team has detected a number of suspicious transactions. It appears that McMahon has shipped goods without receiving customer purchase orders. In addition, in several cases in which McMahon has received purchase orders, the goods shipped were two or three times the quantity ordered. Your audit team thinks that McMahon has been engaging in “channel stuffing,” which is the shipment of excess goods to customers in order to boost reported sales in the current period. You have taken the findings of your audit team to Giff Nielsen, the partner in charge of the audit, and have suggested that substantial additional audit tests be conducted to find out whether McMahon has in fact engaged in channel stuffing. Nielsen instructed you to ignore the channel stuffing evidence and proceed with the rest of the audit program. Nielsen is concerned about keeping the staff hours under budget on this engagement. In addition, Nielsen doesn’t want to upset the senior management team of McMahon. McMahon’s controller has already expressed some concern over the level of detailed testing that the Doman & Detmer audit team has conducted this year. McMahon’s controller has hinted that McMahon is shopping around for a new auditor for next year. Because McMahon is one of the largest clients of Doman & Detmer, Giff Nielsen is afraid that his future with Doman & Detmer will be bleak if he loses McMahon as a client. What should you do with the channel stuffing evidence assembled by your audit team?

present value of series of unequal payments casper sporting goods co is considering 691988

Present Value of Series of Unequal Payments

Casper Sporting Goods Co. is considering a $1 million capital investment that will provide the following expected net receipts at the end of each of the next six years.

Year

Expected Net Receipts

1

$195,000

2

457,000

3

593,000

4

421,000

5

95,000

6

5,000

Casper will make the investment only if the rate of return is greater than 12%.Will Casper make the investment?

reciprocal relationships future and present values determine the amount that must be 691995

Reciprocal Relationships: Future and Present Values

Determine the amount that must be deposited now at compound interest to provide the desired sum for each of the following:

1. Amount to be invested for 10 years at 6% per annum, compounded semiannually, to equal $17,000.

2. Amount to be invested for 21⁄2 years at 8% per annum, compounded quarterly, to equal $5,000.

3. Amount to be invested for 15 years at 12% per annum, compounded semiannually, then reinvested at 16% per annum, compounded quarterly, for five more years to equal $25,000.

4. Amount to be invested at 8% per annum, compounded semiannually for three years, then $5,000 more added and the entire amount reinvested at the same rate for another three years, compounded semiannually, to equal $12,500.

determining unknown quantities determine the unknown quantity for each of the follow 691999

Determining Unknown Quantities

Determine the unknown quantity for each of the following independent situations using the appropriate interest tables:

1. Jeff and Nancy want to start a trust fund for their newborn son, Mark. They have decided to invest $5,000 today. If interest is 8% compounded semiannually, how much will be in the fund when Mark turns 20?

2. Nixon Corporation wants to establish a retirement fund. Management wants to have $1,000,000 in the fund at the end of 40 years. If fund assets will earn 12%,compounded annually, how much will need to be invested now?

3. How many payments would Star, Inc., need to make if it purchases a new building for $100,000 with annual payments made at the end of each year of $16,401.24 and interest of 16%, compounded annually?

4. An investment broker indicates that an investment of $10,000 in a CD for 10 years at the current interest rate will accumulate to $21,589. What is the current annual rate of interest if interest is compounded annually?

determining unknown quantities determine the unknown quantity for each of the follow 692000

Determining Unknown Quantities

Determine the unknown quantity for each of the following independent situations using the appropriate interest tables:

1. Sue wants to have $10,000 saved when she begins college. If Sue enters college in four years and interest is 8% compounded annually, how much will Sue need to save each year assuming equal deposits at the end of each year?

2. XYZ Company has obtained a bank loan to finance the purchase of an automobile for one of its executives. The terms of the loan require monthly payments at the end of each month of $585. If the interest rate is 18% compounded monthly and the car costs $15,850, for how many months will XYZ have to make payments?

3. Diaz Company is offering the following investment plan. If deposits of $250 are made semiannually for the next nine years, $7,726 will accrue. If interest is compounded semiannually, what is the approximate annual rate of interest on the investment?

4. Jack wants to buy a rental unit. For how many periods will he have to make annual deposits of $5,000 in order to accumulate $50,445, the price of the rental unit, if interest is 12% compounded annually? Assume deposits are made at the end of each year.

journal entries for an up front refundable fee the company operates a travel club th 692086

Journal Entries for an Up Front, Refundable Fee

The company operates a travel club through which subscribers can access low rates for air fares, hotel rooms, and rental cars. Each year, subscribers pay a refundable fee of $1,000 that allows them access to the company’s services for that year. A customer may receive a full refund of this fee at any time during the year with no questions asked. The cost to service a customer’s account for a year is $120; these costs are incurred in cash evenly throughout the year. The company can reliably estimate that 30% of customers will ask for a full refund of their subscription fee. On January 1, the company received payments from 1,500 subscribers. Make the journal entries necessary to record (1) the receipt of the subscription fees from these 1,500 customers, (2) the partial recognition of the subscription fees as revenue after the first month (with the associated service cost for the first month), and (3) final recognition of revenue (and associated service cost) for the month of December as well as the payment of full refunds to 30% of the customers (as expected).

journal entries for contingent rent on january 1 owner company signed a 1 year renta 692087

Journal Entries for Contingent Rent

On January 1, Owner Company signed a 1 year rental for a total of $480,000, with monthly payments of $40,000 due at the end of each month. In addition, the renter must pay contingent rent of 2% of all sales in excess of $50 million annually. The contingent rent is paid in one payment on December 31. On January 31, Owner Company received the first rental payment. At that time, sales for the renter had reached $10 million. On May 31, Owner Company received the regular monthly rental payment; by the end of May, the renter had reached a sales level of $55 million. On December 31, Owner received the final monthly rental payment as well as the contingent rental payment. The renter’s sales for the year totaled $80 million, of which $12 million occurred in December. Make the journal entries necessary on the books of Owner Company on (1) January 31, (2) May 31, and (3) December 31.

reporting revenue gross and net online company operates a web grocer customers submi 692088

Reporting Revenue Gross and Net

Online Company operates a Web grocer. Customers submit their orders online to Online Company; Online then forwards the orders to a national grocery chain. The grocery chain arranges for assembly and shipment of the order. Online Company receives 2% of the retail value of all orders it takes. During January, Online Company received orders for groceries with a retail selling price of $300,000. These groceries cost the grocery store chain $210,000. The grocery store chain collected cash of $300,000 from the customers and paid the appropriate commission in cash to Online Company. Based on this information, make all journal entries necessary in January (1) on the books of Online Company and (2) on the books of the grocery store chain. Assume that the grocery store chain uses a perpetual inventory system.

cost to cost method the company signed an 800 000 contract to build an environmental 692089

Cost to Cost Method

The company signed an $800,000 contract to build an environmentally friendly access trail to South Willow Lake. The project was expected to take approximately three years. The following information was collected for each year of the project—Year 1, Year 2, and Year 3:

 

Cost

Expected

Support

Additional

Trail Feet

Additional

 

Expended

Additional

Timbers

Support

Constructed

Trail Feet

 

during

Cost to

Laid during

Timbers

during

to Be

 

the Year

Completion

the Year

to Be Laid

the Year

Constructed

Year 1       

$100,000

$450,000

150

850

3,000

15,200

Year 2        

150,000

280,000

300

520

7,500

8,200

Year 3        

250,000

0

500

0

8,000

0

The company uses the percentage of completion method of computing revenue from long term construction contracts. Assume that the company employs the cost to cost method of estimating the percentage of completion. Compute the amount of revenue to be recognized in (1) Year 1, (2) Year 2, and (3) Year 3.

percentage of completion based on output measures refer to practice 8 8 assume that 692091

Percentage of Completion Based on Output Measures

Refer to Practice 8 8. Assume that the company employs an output measure to estimate the percentage of completion. In particular, the company measures its progress by the number of trail feet that have been completed. Compute the amount of revenue to be recognized in (1) Year 1, (2) Year 2, and (3) Year 3.

Cost to Cost Method

The company signed an $800,000 contract to build an environmentally friendly access trail to South Willow Lake. The project was expected to take approximately three years. The following information was collected for each year of the project—Year 1, Year 2, and Year 3:

 

Cost

Expected

Support

Additional

Trail Feet

Additional

 

Expended

Additional

Timbers

Support

Constructed

Trail Feet

 

during

Cost to

Laid during

Timbers

during

to Be

 

the Year

Completion

the Year

to Be Laid

the Year

Constructed

Year 1       

$100,000

$450,000

150

850

3,000

15,200

Year 2        

150,000

280,000

300

520

7,500

8,200

Year 3        

250,000

0

500

0

8,000

0

The company uses the percentage of completion method of computing revenue from long term construction contracts. Assume that the company employs the cost to cost method of estimating the percentage of completion. Compute the amount of revenue to be recognized in (1) Year 1, (2) Year 2, and (3) Year 3.

basic construction journal entries refer to practice 8 8 in addition to the percenta 692092

Basic Construction Journal Entries

Refer to Practice 8 8. In addition to the percentage of completion information, the following information is available regarding billing and cash collection for the project:

 

Year 1

Year 2

Year 3

Progress billings

$200,000

$200,000

$400,000

Cash collections

180,000

170,000

450,000

Make the journal entries necessary to record the construction cost, the progress billings, and the cash collections in (1) Year 1, (2) Year 2, and (3) Year 3.

Practice 8 8

Cost to Cost Method

The company signed an $800,000 contract to build an environmentally friendly access trail to South Willow Lake. The project was expected to take approximately three years. The following information was collected for each year of the project—Year 1, Year 2, and Year 3:

 

Cost

Expected

Support

Additional

Trail Feet

Additional

 

Expended

Additional

Timbers

Support

Constructed

Trail Feet

 

during

Cost to

Laid during

Timbers

during

to Be

 

the Year

Completion

the Year

to Be Laid

the Year

Constructed

Year 1       

$100,000

$450,000

150

850

3,000

15,200

Year 2        

150,000

280,000

300

520

7,500

8,200

Year 3        

250,000

0

500

0

8,000

0

multiple years of revenues and costs cost to cost method the company signed a 1 200 692093

Multiple Years of Revenues and Costs: Cost to Cost Method

The company signed a $1,200,000 contract to build an environmentally friendly access trail to Deseret Peak. The project was expected to take approximately 3 years. The following information was collected for each year of the project, Year 1, Year 2, and Year 3:

 

Cost

Expected

Trail Feet

Additional

 

Expended

Additional

Constructed

Trail Feet

 

during

Cost to

during

to Be

 

the Year

Completion

the Year

Constructed

Year 1

$200,000

$550,000

8,000

16,200

Year 2

350,000

280,000

12,500

4,100

Year 3

250,000

0

4,000

0

The company uses the percentage of completion method of computing revenue from long term construction contracts. Assume that the company employs the cost to cost method of estimating the percentage of completion. Make the journal entries to record revenue and cost for the construction project—(1) Year 1, (2) Year 2, and (3) Year 3.

journal entries for the proportional performance method the skull valley angels is a 692095

Journal Entries for the Proportional Performance Method

The Skull Valley Angels is a minor league baseball team. The team has 60 home games during a season and sells season tickets for $500 each. For the most recent season, the Angels sold 2,000 season tickets. The total initial direct costs (in cash) related to the season tickets (including product giveaways for signing up early, costs of processing the transactions, and so forth) were $150,000. Direct costs (in cash) are $2 per customer per game. The team’s fiscal year ends on June 30. As of that date, 23 of the home games have been played. Make the journal entries necessary to record (1) the receipt of the cash for the 2,000 season tickets sold, (2) the payment (in cash) for the initial direct costs, and (3) the recognition of all season ticket revenues and expenses for the fiscal year.

accounting for a bill and hold arrangement on december 30 tricky company segregated 692100

Accounting for a Bill and Hold Arrangement

On December 30, Tricky Company segregated goods costing $145,000 for future shipment to one of its customers, Tracking Company. Tracking was billed $210,000. Make the journal entry necessary on Tricky’s books to record this action in each of the following situations. Treat each situation independently.

(a) Tracking is a regular customer, and Tricky has been expecting an order for the past 2 weeks. To make sure that sufficient goods are available when the order from Tracking finally does come, Tricky has segregated the goods.

(b) Normal procedure is for the purchasing agent for Tracking to sign a formal sales agreement as part of each purchase. That agreement is then countersigned by Tricky’s sales manager. The segregation of goods was arranged over the phone; Tricky plans to take care of the formal paperwork next week.

(c) Tracking has requested, in writing, that Tricky segregate the goods. Tracking is conducting temporary repairs to its storage warehouse, so Tracking has arranged to make its shipments directly from Tricky’s warehouse for the duration of the repairs. The goods have been carefully separated so that Tricky employees don’t accidentally ship them to another customer.

(d) The sales agreement between Tricky and Tracking requires that all goods be subjected to a quality control test by Tracking engineers. That quality control test is not expected to occur until early January.

journal entries for an up front nonrefundable fee body tone company sells lifetime h 692101

Journal Entries for an Up Front, Nonrefundable Fee

Body Tone Company sells lifetime health club memberships. For one up front, nonrefundable fee, a customer becomes a lifetime member of Body Tone’s network of health clubs. The fee is $2,000. The fee includes full access to all of the club facilities plus an initial comprehensive physical, mental, and spiritual wellness evaluation. The wellness evaluation is frequently sold separately for $400. Occasionally, Body Tone sells lifetime memberships without the wellness evaluation; the price is $1,750. Body Tone can reliably estimate that the average customer will use the health club facilities for 5 years. On January 1, 2008, Body Tone received lifetime membership payments from 300 new customers. The direct cost of providing a wellness evaluation is $70, and the direct cost of providing health club access for one person for one year is $250. All of the wellness evaluations were completed during 2008.Make all of the journal entries necessary in 2008 in connection with these 300 new memberships. Assume that all costs were incurred in cash.

journal entries for an up front refundable fee accoun tutor company operates a natio 692102

Journal Entries for an Up Front, Refundable Fee

Accoun Tutor Company operates a nationwide online tutorial service for college students taking intermediate financial accounting. Subscribers to the service pay an up front, refundable fee of $200 that allows them access to the company’s services for one year. A subscriber may receive a full refund of this fee at any time during the year. Because the subscribers are accounting students with high ethics, Accoun Tutor has no concern about unscrupulous students using the service for a year and then brazenly asking for a full refund. The initial setup cost (incurred in cash) associated with each subscriber is $40.The direct cost to service a subscriber’s account for a year is $80; these costs are incurred in cash evenly throughout the year. The company can reliably estimate that 20% of subscribers will ask for a full refund of their subscription fee. On January 1, 2008, the company received payments from 20,000 subscribers. No refunds were requested until the end of the fourth quarter of the year when 3,800 subscribers requested and received full refunds. Costs associated with the subscribers who are expected to request refunds are expensed as incurred. Other direct costs are deferred and matched with the associated revenues. Make all summary journal entries necessary:

1. On January 1, 2008.

2. At the end of the first quarter.

3. At the end of the second quarter.

4. At the end of the third quarter.

5. At the end of the fourth quarter.

completed contract method on march 1 2008 bids were submitted for a construction pro 692103

Completed Contract Method

On March 1, 2008, bids were submitted for a construction project to build a new municipal building and fire station. The lowest bid was $4,270,000, submitted by the Harper Construction Company. Harper was awarded the contract. Harper uses the completed contract method to report gross profit. The following data are given to summarize the activities on this contract for 2008 and 2009. Give the entries to record these transactions using the completed contract method.

 

Cost

Estimated Cost

Billings on

Collections of

Year

Incurred

to Complete

Contract

Billings

2008                       

$1,790,000

$2,140,000

$1,750,000

$1,050,000

2009                       

2,090,000

0

2,520,000

3,220,000

ethical dilemma is the price right you are a finance and accounting analyst for buns 691955

Ethical Dilemma (Is the price right?)

You are a finance and accounting analyst for Bunscar Company and have been with the firm for 5 years. Bunscar is a closely held corporation—all of the shares are owned by the founder, Ryan Brown, and by other long time employees. Bunscar is preparing to issue stock for the first time in an initial public offering (IPO). Of great interest is the initial selling price of the stock because that will determine how much Brown and the others will reap from the sale of their shares. The board of directors has put together an analysis proposing that the initial selling price be set at $15 per share. Because Brown and the other insiders intend to sell 10 million shares, this price will bring them $150 million. The analysis relies heavily on the trend in Bunscar’s earnings, which have grown sharply, particularly in the past year. You have the reputation of possessing the best presentation skills in the company. The board of directors has asked you to present the $15 per share proposal to the investment banking firm that will handle Bunscar’s IPO. This is your big chance. As you review the board’s analysis in preparing your presentation, you notice that no mention is made of Bunscar’s cash flow from operations (CFO). CFO has been fairly steady for the past few years; at the same time, earnings have more than doubled. In the past year, when earnings increased 65%, CFO actually declined slightly. After some investigation, you find that Bunscar has become very loose in its assumptions about when revenue should be recognized. In fact, putting the revenue and cash flow numbers together, you conclude that most of Bunscar’s earnings increase has come from questionable revenue that probably will never be collected in cash. It seems clear to you that Bunscar’s accounting assumptions have been manipulated to make reported income look as good as possible to increase the IPO selling price. Your presentation is scheduled for the day after tomorrow. What should you do?

cumulative spreadsheet analysis this spreadsheet assignment is a continuation of the 691956

Cumulative Spreadsheet Analysis

This spreadsheet assignment is a continuation of the spreadsheet assignments given in earlier chapters. If you completed those assignments, you have a head start on this one.

Refer back to the instructions for preparing the revised financial statements for 2008 as given in part (1) of the Cumulative Spreadsheet Analysis assignment in Chapter 3.

1. Skywalker wishes to prepare a forecasted balance sheet, a forecasted income statement, and a forecasted statement of cash flows for 2009. Use the financial statement numbers for 2008 as the basis for the forecast, along with the following additional information.

(a) Sales in 2009 are expected to increase by 40% over 2008 sales of $2,100.

(b) In 2009, Skywalker expects to acquire new property, plant, and equipment costing $240.

(c) The $480 in operating expenses reported in 2008 breaks down as follows: $15 depreciation expense and $465 other operating expenses.

(d) No new long term debt will be acquired in 2009.

(e) No cash dividends will be paid in 2009.

(f) New short term loans payable will be acquired in an amount sufficient to make Skywalker’s current ratio in 2009 exactly equal to 2.0.

(g) Skywalker does not anticipate repurchasing any additional shares of stock during 2009.

(h) Because changes in future prices and exchange rates are impossible to predict, Skywalker’s best estimate is that the balance in accumulated other comprehensive income will remain unchanged in 2009.

(i) In the absence of more detailed information, assume that investment securities, long term investments, other long term assets, and intangible assets will all increase at the same rate as sales (40%) in 2009.

(j) In the absence of more detailed information, assume that other long term liabilities will increase at the same rate as sales (40%) in 2009.

(k) The investment securities are classified as available for sale. Accordingly, cash from the purchase and sale of these securities is classified as an investing activity.

(l) Transactions impacting other long term assets and other long term liabilities accounts are operating activities.

2. Repeat (1) with the following change in assumptions:

(a) Sales growth in 2009 is expected to be 25%.

(b) Sales growth in 2009 is expected to be 50%.

3. Comment on the forecasted values of cash from operating activities in 2009, assuming that sales will grow at 25%, 40%, and 50%, respectively.

should we implement an earnings based bonus plan benjamin vincent is the chief finan 691957

Should We Implement an Earnings Based Bonus Plan?

Benjamin Vincent is the chief financial officer (CFO) of Annie Company. The company’s chief executive officer (CEO) has asked Benjamin to design an incentive scheme that will motivate employees to focus more on the company’s bottom line results. Benjamin is considering a plan that will give each employee a bonus based on the company’s reported net income for the year. Each employee will receive an amount equal to the company’s earnings per share multiplied by either 10,000 times, 50,000 times, or 200,000 times, depending on the employee’s level in the company. Last year, Annie Company’s earnings per share was $1.32. Benjamin Vincent has asked you for your advice. In particular, he wants you to explain the disadvantages of having an earnings based bonus system.

income smoothing and an ipo you are an analyst for an investment fund that invests i 691960

Income Smoothing and an IPO

You are an analyst for an investment fund that invests in initial public offerings (IPOs).You are looking at the financial statements of two companies, Clark Company and Durfee Company, that plan to go public soon. Net income for the past three years for the two companies has been as follows (in thousands):

 

Clark

Durfee

Year

Net Income

Net Income

2005

$10,000

$17,000

2006

14,000

1,000

2007

20,000

26,000

If both companies issue the same number of shares and if the initial share prices are the same, which of the two companies appears to be a more attractive investment? Explain your reasoning. What alternate sources of data would you look at to find out whether the reported earnings amounts accurately portray the business performance of these two companies over the past three years?

managing earnings to avoid political scrutiny flame control company is a publicly tr 691962

Managing Earnings to Avoid Political Scrutiny

Flame Control Company is a publicly traded company based in a heavily forested state in the western United States. Flame Control manufactures equipment used in fighting forest fires. During the past year, many large fires occurred in the forests of Flame Control’s state. Many homes were destroyed, hun Dr. eds of thousands of acres of timber were burned, and the public expenditure on fighting the fires was at least triple what it had been in any other year in history. It was a very successful year financially for Flame Control because it was able to sell every piece of equipment that it was able to manufacture in its factories. There has been some grumbling in the press about price gouging by fire equipment manufacturers. You are Flame Control Company’s chief financial officer (CFO).You are working with the accounting staff to prepare the financial statements for the preceding fiscal year. Flame Control is expected to make a preliminary earnings announcement next week. What issues and what accounting actions might you consider as you prepare for the preliminary earnings announcement?

managing earnings in the jubilee year heidelberg company has been in business for 10 691965

Managing Earnings in the Jubilee Year

Heidelberg Company has been in business for 100 years. The past three years have been trying ones for the company, which has reported operating losses in each of those three years. The board of directors is planning a huge, year long celebration of the company’s centennial year. The board has informed the company’s controller that the company must report a profit in each quarter of the centennial year. The board has not told the controller how this is to be done, but the implication is that if the operating results are not enough to generate a profit, the controller must use accounting assumptions to push the company over the top. The controller has identified three areas in which Heidelberg Company has some flexibility in its accounting assumptions: depreciation, bad debts, and pension accounting. Describe specifically how the controller can use accounting assumptions in these three areas to improve Heidelberg’s reported earnings. Also describe which set of financial statement users is most likely to be influenced by this earnings management in the centennial year financial statements.

how can you justify that change in estimate you are a financial analyst and have bee 691966

How Can You Justify that Change in Estimate?

You are a financial analyst and have been looking at the financial statements of Denethor Company. The notes to the financial statements reveal that Denethor changed its estimated depreciation lives for its manufacturing equipment. You calculate that without this change, Denethor would have had a reported loss instead of a reported profit for the year. The financial statement notes include the following justification for the change in estimate: “The changes in estimated depreciation lives were made to conform the Company’s depreciation estimates to those used by other manufacturers in the Company’s industry and to provide a more equitable allocation of the cost of equipment over their useful lives.” You have just received a call from a long time client who is considering investing in Denethor Company. Given this information, what will you tell this client?

i didn rsquo t do it on purpose you are a senior staff member in the office of the c 691967

I Didn’t Do It on Purpose!

You are a senior staff member in the office of the Chief Accountant of the Securities and Exchange Commission (SEC).You have been supervising a case brought against an audit firm. The audit client used non GAAP accounting practices that allowed it to report annual earnings of $47.3 million instead of a loss of $15.0 million. Earnings in the preceding three years averaged $10 million per year. The auditor explains that this non GAAP accounting practice was not detected during the audit because of innocent mistakes made by staff auditors. Your thorough investigation has not turned up any evidence that the audit firm intentionally allowed the client to use this non GAAP practice. You must decide whether to formally sanction the audit firm or whether to Dr.op the case because of lack of evidence of wrongful intent. What should you do?

earnings management inc john sleaze and mary scum run earnings management inc a cons 691968

Earnings Management, Inc.

John Sleaze and Mary Scum run Earnings Management, Inc., a consulting business. They have the following items in their product line:

1. A database that lists types of depreciable assets and the minimum and maximum depreciation lives that have been accepted by auditors for each type of asset. The listing can be sorted by audit firm, so a client can know the minimums and maximums accepted by each individual audit firm.

2. A detailed analysis of the SEC’s Staff Accounting Bulletin (SAB) 101 on revenue recognition. The analysis reveals loopholes in SAB 101 that companies can use to strategically time the recognition of revenue.

3. A comprehensive list of all accounting issues for which there is no generally accepted standard. This list can be viewed as an identification of all of the fuzzy areas of accounting that a company might exploit if it desired to conduct earnings management.

4. A list of the local offices for each major audit firm that appear to have been the most “flexible” in signing off on aggressive accounting treatments by clients. In some cases, the list includes specific audit partners who have a reputation for being accommodating when a client firm wishes to use aggressive earnings management techniques. You are an FBI agent investigating Earnings Management, Inc., for possible indictment on securities fraud and racketeering charges. Comment on whether you think John and Mary have committed any indictable offenses.

strategically record a business acquisition you are the controller for rosie company 691970

Strategically Record a Business Acquisition

You are the controller for Rosie Company. Rosie has just acquired another company and it is your job to allocate the $10 million overall purchase price to the specific items acquired. The following is a list of the items to which the purchase price must be allocated along with two possible allocations:

Item

Accounting Treatment

Allocation 1

Allocation 2

In process R&D

Immediate expense

$ 500,000

$5,000,000

Building

Depreciation life of 15 to 25 years

4,000,000

2,000,000

Machinery

Depreciation life of 3 to 10 years

5,500,000

3,000,000

Rosie Company’s CEO, who has absolutely no personal ethics, has instructed you to allocate the purchase price to show big earnings growth in the next few years. The CEO doesn’t care what earnings are reported this year because any losses can be blamed on the effort to integrate the newly acquired company. The CEO also wants your allocation to give the company maximum flexibility to manage earnings to show consistently increasing earnings in future years. Which allocation, 1 or 2, and which depreciation lives for the building and machinery should you choose to accomplish the CEO’s directive? (Ignore income tax considerations.) What concerns should you have about this request?

loading up the cookie jar lily company has historically reported a bad debt expense 691971

Loading Up the Cookie Jar!

Lily Company has historically reported a bad debt expense amount of between 1% and 4% of sales. The percentage for any given year is a function of both the business conditions for the year and whether recent experience suggests that the estimates in past years have been too high or too low. For example, if estimates in past years have been too high, a lower amount of bad debt expense is recognized in the current year. Lily Company’s board of directors has met to review the preliminary financial statements for the just completed fiscal year. Assume that the board will decide on a bad debt estimate of either 1% or 4% of sales. Consider the following two scenarios:

Scenario 1. The preliminary earnings number for the year is very high, far higher than expected. However, Lily’s board is concerned about future years; there is some indication of unsettled business conditions ahead.

Scenario 2. The preliminary earnings number for the year is quite low, lower than expected. The board has reason to be optimistic that Lily’s operating performance will turn around next year.

What estimate (1% or 4%) do you think Lily’s board will choose in each of the two scenarios? Explain your choices. What risks are there to Lily Company if the bad debt estimate is chosen using only the type of information given here?

excuse me but what is your audit materiality threshold rex tee is a staff auditor fo 691972

Excuse Me, But What Is Your Audit Materiality Threshold?

Rex Tee is a staff auditor for a large audit firm. As part of the audit planning process for the Kirtland Company audit, he has been informed that the materiality threshold for the audit will be $250,000.This means that audit disagreements about amounts less than $250,000 will not be actively investigated. This threshold has been justified by the fact that Kirtland has annual sales in excess of $50 million. Rex has been trying to establish a friendly working relationship with the accounting staff at Kirtland. In a conversation this morning, Rex made the following statement to the assistant controller: “I found an audit difference of $185,000 yesterday, but we aren’t going to investigate it because our materiality threshold on this audit is $250,000.”What danger is there in Rex’s comment? What danger is there in establishing the $250,000 threshold?

i need to recognize the revenue now the h k clark health club sells lifetime members 691973

I Need to Recognize the Revenue Now!

The H. K. Clark Health Club sells lifetime memberships for $5,000 each. These memberships entitle a person to unlimited access to the club’s weight room, exercise equipment, swimming pool, and sauna. Once a lifetime membership fee is paid, it is not refundable for any reason. According to the provisions of SAB 101, revenue from the sale of a lifetime membership must be deferred and recognized over the average expected time that a member will continue to use the club facilities. However, if the terms of the membership agreement are interpreted very favorably, a substantial portion of the $5,000 initial fee might be able to be recognized as revenue immediately. Kristen Qi and her partners own the health club. To overcome a cash shortage, they intend to seek a new loan from their bank. Kristen and her partners are meeting with their accountant to provide information for preparation of financial statements. What incentives would Kristen and her partners have for recognizing the entire amount of the lifetime membership fee as revenue at the time it is collected? Since the entire amount will ultimately be recognized anyway, what difference does the timing make? Do Kristen and her partners have any economic incentive to go ahead and defer the membership revenue in accordance with SAB 101?

how should i interpret the pro forma number worthington company and millward company 691974

How Should I Interpret the Pro Forma Number?

Worthington Company and Millward Company both reported pro forma earnings numbers in conjunction with their release of results for the most recent quarter. Both announcements included a reconciliation to GAAP earnings. These reconciliations are reproduced here.

Worthington Company

Pro Forma Earnings

(in thousands)

 

GAAP earnings                                                              

$ 50,000

Add back amount expensed for the purchase of in process R&D                           

35,000

Subtract a one time gain from the sale of a building                                     

(17,000)

Pro forma earnings                                                            

 $ 68,000

 

Millward Company

Pro Forma Earnings

(in thousands)

 

GAAP earnings                                                              

$ 50,000

Add back expenses associated with a strategic realignment initiative                        

10,000

Add back employee training expenses                                              

8,000

Pro forma earnings                                                             

 $ 68,000

Which of the two pro forma earnings disclosures do you find to be the more informative?

Explain.

i rsquo m an accountant not a public relations person jacob marley is the controller 691975

I’m an Accountant, Not a Public Relations Person!

Jacob Marley is the controller for Dickens Company. Marley has been with Dickens for more than 30 years. Marley is a dedicated employee and prides himself on the efficiency of his accounting department staff. Over the years, Marley has received many inquiries and suggestions from the board of directors of Dickens Company about appropriate accounting treatments, the magnitude of certain accounting estimates, and so forth, but he has never paid the slightest attention to any of the suggestions. Marley’s view is that the process of generating the financial statement numbers is simply a matter of rigidly applying certain predetermined mathematical rules, and he does not welcome the input of the board of directors or anyone else. Marley also refuses to communicate with analysts, pension fund managers, and business press reporters who call to make inquiries about Dickens. Marley believes that the financial statements speak for themselves and need no clarification or amplification. Comment on the costs and benefits of Jacob Marley’s approach to financial reporting to Dickens Company.

how can i screen my audit clients sarah corning is the managing partner for a large 691978

How Can I Screen My Audit Clients?

Sarah Corning is the managing partner for a large office of a major audit firm. The audit firm has developed an analytical model that is used to evaluate the risk of potential audit clients. The audit firm has learned that the audits of certain types of clients are more likely to result in a failure to detect material misstatements, exposing the audit firm to lawsuits. The analytical model includes factors such as industry, past volatility in the company’s stock price (for publicly traded companies), asset mix, assessment of the character of management, the strength of the company’s internal controls, and so forth. The model rates potential clients on a scale from 1 to 5, with 1 being the safest clients and 5 being the most risky. Some of Sarah’s partners have advocated a policy of rejecting all potential clients with ratings of 5. Comment on this proposal.

do all analysts have the same incentives there are two general types of financial an 691979

Do All Analysts Have the Same Incentives?

There are two general types of financial analysts:

• Buy side analyst. An analyst employed by an entity, such as a mutual fund, which invests on its own accounts. Unlike that of the sell side analysts employed by brokerage firms, research produced by buy side analysts is usually unavailable outside the firm that hired the analyst.

• Sell side analyst. An analyst employed by a brokerage firm or another firm that manages client accounts. Unlike that of the buy side analysts employed by mutual funds, research produced by sell side analysts is usually available to the public. Some financial analysts have been criticized for making optimistic forecasts of the earnings of potential clients in order to curry favor with those potential clients. Do you think that this criticism is directed at buy side analysts or sell side analysts? Explain.

who would report if reporting were voluntary tarazania is a country with a small but 691980

Who Would Report if Reporting Were Voluntary?

Tarazania is a country with a small but active stock market. However, the country has no accounting standards; in fact, the issuance of financial statements is illegal. This odd law stems from the fact that the founding king of Tarazania once took an intermediate accounting course and was so overwhelmed by the chapter on the statement of cash flows that he vowed he would never view another financial statement again. As a result, none of the 100 companies with publicly traded stocks in Tarazania have ever made financial statements available to the public. Of course, each of these companies has prepared financial statements and other reports for use internally for years. Last week Tarazania’s founding king died. His eldest daughter has now ascended to the throne. Her Majesty has been a secret aficionado of financial statements for years. One of her first official acts was to make the public release of financial statements legal but not mandatory. Of the 100 publicly traded companies in Tarazania, which will be the first to release its financial statements to the public? Will all 100 companies do so?

does it pay to lie joseph han has 10 million that he wishes to invest he has identif 691981

Does It Pay to Lie?

Joseph Han has $10 million that he wishes to invest. He has identified two candidate companies: Company A and Company B. Both companies are privately held and have never yet released external financial statements. Joseph Han has some familiarity with the use of financial statements, but his knowledge is not perfect and he can be fooled. However, he has the ability to recognize blatant financial statement manipulation. As Companies A and B prepare their financial statements, they must consider the following three scenarios:

• Scenario 1. Both prepare transparent financial statements that faithfully reflect their underlying business performance. Joseph Han is impressed with both companies and invests $5 million in each.

• Scenario 2. One of the companies prepares deceptive financial statements. These financial statements look so good compared to the transparent financial statements prepared by the other company that Joseph Han instantly decides to invest $8 million in the deceptive company and nothing in the truthful company. To avoid putting all of his eggs in one basket, Joseph Han holds back $2 million and puts it in a bank savings account.

• Scenario 3. Both companies prepare deceptive financial statements. In carefully comparing these two glowing sets of financial statements, Joseph Han realizes that both sets of financial statements have been manipulated. He decides to invest $1 million dollars in each company, as a speculation, and to put the remaining $8 million in a bank savings account. Given these three scenarios, what is the best strategy for Companies A and B—to lie or to tell the truth? Will your answer change if Joseph Han announces his intention to make this same $10 million investment decision with respect to these two companies each year for the next 30 years?

deciphering financial statements the walt disney company in the press release announ 691982

Deciphering Financial Statements (The Walt Disney Company)

In the press release announcing Disney’s results for the quarter ending July 2, 2005, the company stated the following:

The Walt Disney Company today reported earnings for the quarter and nine months ended July 2, 2005. Diluted earnings per share (EPS) for the third quarter increased

41% to $0.41, compared to $0.29 in the prior year quarter. . . .

Current quarter EPS included a $26 million gain on the sale of the Mighty Ducks of Anaheim, a $32 million partial impairment charge for a cable television investment in Latin America, and a $24 million write down related to the Movie Beam venture. In aggregate, these items reduced current quarter EPS by $0.01 per share. The prior year quarter’s EPS included restructuring and impairment charges of $56 million, or $0.02 per share, recorded in connection with the disposition of the Disney Stores North America.

1. How did Disney arrive at the 41% increase?

2. Suppose none of the non operating transactions disclosed in the second paragraph had occurred in the third quarter of 2005 or the third quarter of 2004. Compute the EPS increase.

cash flow ratios following are data from the financial statements for houma company 691920

Cash Flow Ratios

Following are data from the financial statements for Houma Company.

Houma Company

Selected Financial Statement Data

For the Years Ended December 31, 2008 and 2007

 

2008

2007

Net income                                                     

$34,000

$ 65,200

Cash from operating activities                                       

28,900

158,130

Cash paid for purchase of fixed assets                                  

42,000

156,000

Cash paid for interest                                             

26,000

24,000

Cash paid for income taxes                                         

15,000

25,670

Compute the following for both 2007 and 2008:

1. Cash flow to net income ratio

2. Cash flow adequacy ratio

3. Cash times interest earned ratio

forecasted balance sheet income statement and statement of cash flows balance sheet 691922

Forecasted Balance Sheet, Income Statement, and Statement of Cash Flows

Balance Sheet

2008

Cash

$10

Other current assets

250

Property, plant, and equipment, net

800

Total assets

$1,060

Accounts payable

$ 100

Bank loans payable

700

Total stockholders’ equity

260

Total liabilities and stockholders’ equity

$1,060

 

Income Statement

2008

Sales                    

$1,000

Cost of goods sold         

750

Gross profit              

$250

Depreciation expense       

40

Other operating expenses    

80

Operating profit           

$130

Interest expense           

70

Income before income taxes  

$60

Income taxes             

20

Net income              

$40

In addition, Ryan has assembled the following forecasted information regarding 2009.

(a) Sales are expected to increase to $1,500.

(b) Ryan expects to become more efficient at utilizing its property, plant, and equipment in 2009.Therefore, Ryan expects that the sales increase will not require any overall increase in property, plant, and equipment. Accordingly, the year 2009 property, plant, and equipment balance is expected to be $800.

(c) Ryan’s bank has approved a new long term loan of $200. This loan will be in addition to the existing loan payable.

(d) Ryan Company does not anticipate paying any dividends in the coming year.

Instructions:

1. Prepare a forecasted balance sheet for 2009. Clearly state what assumptions you make.

2. Prepare a forecasted income statement for 2009. Clearly state what assumptions you make.

3. Prepare a forecasted statement of cash flows for 2009. Use the indirect method of reporting cash from operating activities.

preparing the operating activities section of the statement of cash flows po dr acer 691923

Preparing the Operating Activities Section of the Statement of Cash Flows

Po Dr. acer Productions provides the following income statement for the year ended December 31, 2008:

Sales                                                                       

$1,530,600

Cost of goods sold                                                            

895,400

Gross margin                                                                

$ 635,200

General expenses                                                              

255,400

Depreciation expense                                                          

23,500

Salaries expense                                                              

114,300

Operating income                                                             

$ 242,000

Interest revenue                                                              

17,250

Interest expense                                                               

(12,500)

Loss on sale of equipment                                                       

(9,500)

Income before income taxes                                                     

$ 237,250

Income tax expense                                                           

85,500

Net income                                                                 

$ 151,750

In addition, Po Dr. acer provides the following balance sheet information:

 

Dec 31, 2008

Dec 31, 2007

Accounts receivable                                      

$250,400

$225,400

Interest receivable                                       

2,100

2,250

Inventory                                              

74,300

59,550

Prepaid general expenses                                  

17,600

14,000

Accounts payable                                        

39,500

46,300

Accrued general expenses                                 

19,500

21,750

Interest payable                                         

900

1,100

Income taxes payable                                     

11,500

9,750

Salaries payable                                         

9,850

5,400

Instructions: Using the simultaneous analysis matrix illustrated in the text, prepare the Operating Activities section of the statement of cash flows using (1) the direct method and (2) the indirect method.

statement of cash flows mdash indirect method comparative balance sheet data for amb 691924

Statement of Cash Flows—Indirect Method

Comparative balance sheet data for Amber Company follow. In addition, new equipment was purchased for $50,000, payment consisting of $25,000 cash and a long term note for $25,000. The short term note payable was arranged with a supplier to finance inventory purchases on credit. Cash dividends of $10,000 were paid in 2008; all other changes to retained earnings were caused by the net income for 2008, which amounted to $83,500.

 

Dec 31, 2008

Dec 31, 2007

Cash and cash equivalents                                  

$ 41,000

$ 28,000

Accounts receivable                                      

94,000

86,000

Inventory                                              

110,000

100,000

Property, plant, and equipment                              

550,000

500,000

Accumulated depreciation—property, plant, and equipment          

(277,500)

(250,000)

Total assets                                            

 $517,500

 $464,000

 

 

Dec 31, 2008

Dec 31, 2007

Short term notes payable                                  

$ —

$ 20,000

Accounts payable                                        

105,000

80,000

Long term notes payable                                   

100,000

75,000

Bonds payable                                          

50,000

100,000

Common stock, $1 par                                    

20,000

20,000

Additional paid in capital                                   

155,000

155,000

Retained earnings                                         

87,500

14,000

Total liabilities and stockholders’ equity                     

 $517,500

 $464,000

Instructions: Prepare a statement of cash flows for the year ended December 31, 2008, using the indirect method.

statement of cash flows mdash indirect method the following information was taken fr 691925

Statement of Cash Flows—Indirect Method

The following information was taken from the records of Glassett Produce Company for the year ended June 30, 2008.

Borrowed on long term notes                                                    

$15,000

Issued capital stock                                                            

45,000

Purchased equipment                                                           

18,000

Net income                                                                 

32,000

Purchased treasury stock                                                        

5,000

Paid dividends                                                                

29,000

Depreciation expense                                                          

21,000

Retired bonds payable                                                          

65,000

Patent amortization                                                             

3,000

Sold long term investment (at cost)                                                

7,200

Increase in cash                                                               

13,300

Decrease in inventories                                                         

6,300

Increase in accounts receivable                                                    

9,200

Increase in accounts payable                                                     

10,000

Cash balance, July 1, 2007                                                       

22,000

Instructions:

1. From the information given, prepare a statement of cash flows using the indirect method.

2. Briefly explain what an interested party would learn from studying the cash flow statement for Glassett Produce Company.

statement of cash flows mdash indirect method the following information was obtained 691926

Statement of Cash Flows—Indirect Method

The following information was obtained from analysis of selected accounts of Orlando Company for the year ended December 31, 2008.

Increase in long term debt                                                       

$ 57,000

Purchase of treasury stock                                                       

52,000

Depreciation and amortization                                                    

197,000

Gain on sale of equipment (included in net income)                                    

6,000

Proceeds from issuance of common stock                                           

184,000

Purchase of equipment                                                         

434,000

Proceeds from sale of equipment                                                   

20,000

Payment of dividends                                                           

49,000

Net income                                                                 

375,000

Increase (decrease) in working capital accounts:

 

Cash                                                                    

$ 45,000

Accounts receivable                                                         

229,000

Inventories                                                               

275,000

Trade notes payable                                                         

167,000

Accounts payable                                                           

124,000

Income taxes payable                                                        

(34,000)

Cash balance, January 1, 2008                                                     

120,000

Instructions: From the information given, prepare a statement of cash flows using the indirect method.

statement of cash flows mdash direct method based on an analysis of the cash and oth 691927

Statement of Cash Flows—Direct Method

Based on an analysis of the cash and other accounts, the following information was provided by the controller of Lumber camp, Inc., a manufacturer of wood burning stoves, for the year 2008.

(a) Cash sales for the year were $150,000; sales on account totaled $180,000.

(b) Cost of goods sold was 50% of total sales.

(c) All inventory is purchased on account.

(d) Depreciation on equipment was $93,000 for the year.

(e) Amortization of patent was $6,000.

(f) Collection of accounts receivable was $114,000.

(g) Payments on accounts payable for inventory equaled $117,000.

(h) Rent expense paid in cash was $33,000.

(i) Cash of $720,000 was obtained by issuing 60,000 shares of $10 par stock.

(j) Land worth $318,000 was acquired in exchange for a $300,000 bond.

(k) Equipment was purchased for cash at a cost of $252,000.

(l) Dividends of $138,000 were declared.

(m) Dividends of $45,000 that had been declared the previous year were paid.

(n) A machine used on the assembly line was sold for $36,000.The machine had a book value of $21,000.

(o) Another machine with a book value of $1,500 was scrapped and was reported as an ordinary loss. No cash was received on this transaction.

(p) The cash account had a balance of $87,000 on January 1, 2008.

Instructions: Use the direct method to prepare a statement of cash flows for Lumber camp,

Inc., for the year ended December 31, 2008.

statement of cash flows mdash indirect method comparative balance sheet data for the 691928

Statement of Cash Flows—Indirect Method

Comparative balance sheet data for the partnership of Bond and Wallin follow.

 

Dec 31, 2008

Dec 31, 2007

Cash                                                 

$ 15,000

$ 12,500

Accounts receivable                                      

24,200

27,000

Inventory                                              

105,400

91,000

Prepaid expenses                                         

4,100

5,350

Furniture and fixtures                                     

65,500

41,000

Accumulated depreciation                                  

(40,250)

(25,250)

Total assets                                         

 $173,950

 $151,600

Accrued expenses                                       

$ 9,000

$ 6,700

Accounts payable                                        

22,425

32,875

Long term note                                          

21,300

Ryan Bond, capital                                       

69,350

56,150

Trent Wallin, capital                                      

51,875

55,875

Total liabilities and stockholders’ equity                     

 $173,950

 $151,600

Net income for the year was $22,000, and this was transferred in equal amounts to the partners’ capital accounts. Additional changes in the capital accounts arose from additional investments and withDr.awals by the partners. The change in the furniture and fixtures account arose from a purchase of additional furniture; part of the purchase price was paid in cash, and a long term note was issued for the balance.

Instructions: Using the indirect method, prepare a statement of cash flows for 2008.

statement of cash flows mdash direct method the table on page 267 shows the account 691930

Statement of Cash Flows—Direct Method

The table on page 267 shows the account balances of Novation, Inc., at the beginning and end of the company’s accounting period.

Debits

Dec 31, 2008

Jan 1, 2008

Cash and Cash Equivalents                                  

$176,400

$ 58,000

Accounts Receivable                                      

32,000

26,600

Inventory                                              

21,000

25,400

Prepaid Insurance                                        

5,600

4,000

Long Term Investments (at cost)                             

6,000

16,800

Equipment                                             

80,000

66,000

Treasury Stock (at cost)                                   

10,000

20,000

Cost of Goods Sold                                      

368,000

 

Operating Expenses                                      

185,000

 

Income Tax Expense                                      

37,600

 

Loss on Sale of Equipment                                  

1,000 

 

Total debits                                         

$922,600

$216,800

 

Credits

Dec 31, 2008

Jan 1, 2008

Accumulated Depreciation—Equipment                        

$ 19,000

$ 18,000

Accounts Payable                                        

7,000

11,200

Interest Payable                                         

1,000

2,000

Income Taxes Payable                                     

12,000

8,000

Notes Payable—Long Term                                 

16,000

24,000

Common Stock                                         

110,000

100,000

Paid In Capital in Excess of Par                              

32,000

30,000

Retained Earnings                                         

19,600*

23,600

Sales                                                 

704,000

 

Gain on Sale of Long Term Investments                        

2,000 

 

Total credits                                         

$922,600

$216,800

The following additional information is available:

(a) All purchases and sales were on account.

(b) Equipment costing $10,000 was sold for $3,000; a loss of $1,000 was recognized on the sale.

(c) Among other items, the operating expenses included depreciation expense of $7,000; interest expense of $2,800; and insurance expense of $2,400.

(d) Equipment was purchased during the year by issuing common stock and by paying the balance ($12,000) in cash.

(e) Treasury stock was sold for $4,000 less than it cost; the decrease in owners’ equity was recorded by reducing Retained Earnings. No dividends were paid during the year.

Instructions:

1. Prepare a statement of cash flows for the year ended December 31, 2008, using the direct method of reporting cash flows from operating activities.

2. Comment on the lack of dividend payment. Does a “no dividend” policy seem appropriate under the current circumstances for Novations, Inc.?

income statement and statement of cash flows mdash indirect method refer to the data 691931

Income Statement and Statement of Cash Flows—Indirect Method

Refer to the data for Novations, Inc., in Problem 5 47.

Instructions:

1. Prepare an income statement for Novations, Inc., for the year ended December 31,2008.

2. Prepare a statement of cash flows for the year ended December 31, 2008, using the indirect method.

Problem 5 47

Statement of Cash Flows—Direct Method

The table on page 267 shows the account balances of Novations, Inc., at the beginning and end of the company’s accounting period.

Debits

Dec 31, 2008

Jan 1, 2008

Cash and Cash Equivalents                                 

$176,400

$ 58,000

Accounts Receivable                                      

32,000

26,600

Inventory                                              

21,000

25,400

Prepaid Insurance                                        

5,600

4,000

Long Term Investments (at cost)                             

6,000

16,800

Equipment                                             

80,000

66,000

Treasury Stock (at cost)                                    

10,000

20,000

Cost of Goods Sold                                      

368,000

 

Operating Expenses                                      

185,000

 

Income Tax Expense                                      

37,600

 

Loss on Sale of Equipment                                 

1,000 

 

Total debits                                         

$922,600

$216,800

 

Credits

Dec 31, 2008

Jan 1, 2008

Accumulated Depreciation—Equipment                        

$ 19,000

$ 18,000

Accounts Payable                                        

7,000

11,200

Interest Payable                                         

1,000

2,000

Income Taxes Payable                                     

12,000

8,000

Notes Payable—Long Term                                 

16,000

24,000

Common Stock                                         

110,000

100,000

Paid In Capital in Excess of Par                              

32,000

30,000

Retained Earnings                                        

19,600*

23,600

Sales                                                 

704,000

 

Gain on Sale of Long Term Investments                        

2,000 

 

Total credits                                          

$922,600

$216,800

articulation the following data are for bond company note all inventory is purchased 691932

Articulation

The following data are for Bond Company. Note: All inventory is purchased on account, and Accounts Payable relates only to the purchase of inventory.

 

Dec 31, 2008

Dec 31, 2007

BALANCE SHEET DATA (partial)

 

 

Accounts receivable                                      

$ 72,000

$65,000

Inventory                                              

54,000

41,000

Prepaid rent                                            

?

8,000

Accounts payable                                        

44,000

52,000

Wages payable                                          

23,000

?

INCOME STATEMENT DATA

 

 

Sales                                                 

485,000

 

Cost of goods sold                                       

?

 

Wages expense                                         

?

 

Rent expense                                           

22,000

 

Other expenses                                         

121,000

 

Net income                                            

?

 

CASH FLOW DATA

 

 

Net Income                                            

?

 

_/_ Change in accounts receivable                          

(7,000)

 

_/_ Change in inventory                                  

(13,000)

 

_/_ Change in accounts payable                            

(8,000)

 

_/_ Change in prepaid rent                                

?

 

_/_ Change in wages payable                              

6,000

 

Cash from operating activities                               

?

 

OTHER DATA

 

 

Cash collected from customers                              

?

 

Cash paid for inventory                                    

?

 

Inventory purchased on account                             

230,000

 

Cash paid for rent                                       

27,000

 

Cash paid for wages                                      

81,000

 

Cash paid for other expenses                               

121,000

 

Instructions:

Compute the following:

1. The ending balance in the prepaid rent account.

2. The beginning balance in the wages payable account.

3. The amount of Cost of Goods Sold for 2008.

4. The amount of Wages Expense for 2008.

5. The amount of reported Net Income for 2008.

6. The amount of cash collected from customers during 2008.

7. The amount of cash paid for inventory during 2008.

cash flow analysis following are data from the financial statements for shang hi com 691935

Cash Flow Analysis

Following are data from the financial statements for Shang Hi Company:

Shang Hi Company

Selected Financial Statement Data

For the Years Ended December 31, 2008 and 2007

(In millions of dollars)

 

2008

2007

Sales

$ 81,000

$73,000

Total assets

101,000

92,000

Stockholders’ equity

30,000

27,000

Net income

7,700

6,800

Cash from operations

10,200

18,500

Cash paid for capital expenditures

12,400

10,600

Cash paid for acquisitions

3,200

500

Cash paid for interest

1,200

1,000

Cash paid for income taxes

3,900

3,500

Instructions:

1. Compute the following for 2007 and 2008.

(a) Return on sales

(b) Return on assets

(c) Return on equity

(d) Cash flow to net income ratio

(e) Cash flow adequacy ratio

(f) Cash times interest earned ratio

2. In which year did Shang Hi Company perform better, 2007 or 2008? Explain your answer.

3. Shang Hi Company intends to sell a large block of newly issued stock to the public in the first half of 2009. Given your computations in (1), what questions would you like to ask of Shang His management before investing in the newly issued stock?

forecasted balance sheet income statement and statement of cash flows balance sheet 691936

Forecasted Balance Sheet, Income Statement, and Statement of Cash Flows

Balance Sheet

2008

Cash                                                                             

$ 40

Other current assets                                                                 

350

Property, plant, and equipment, net                                                       

1,000

Total assets                                                                        

 $1,390

Accounts payable                                                                    

$ 100

Bank loans payable                                                                  

1,000

Paid in capital                                                                      

100

Retained earnings                                                                    

190

Total liabilities and stockholders’ equity                                              

 $1,390

 

Income Statement

2008

Sales                 

$1,000

Cost of goods sold      

350

Gross profit            

$650

Depreciation expense    

200

Other operating expenses 

250

Operating profit        

$200

Interest expense        

120

Income before income taxes

$80

Income taxes           

20

Net income            

$60

In addition, Lorien has assembled the following forecasted information for 2009.

(a) Sales are expected to increase to $1,200.

(b) Lorien does not expect to buy any new property, plant, and equipment during 2009.

(c) Because of adverse banking conditions, Lorien does not expect to receive any new bank loans in 2009.

(d) Lorien plans to pay cash dividends of $15 in 2009.

Instructions:

1. Prepare a forecasted balance sheet, a forecasted income statement, and a forecasted statement of cash flows for 2009. Clearly state what assumptions you make. Use the indirect method for reporting cash from operating activities.

2. If you have constructed your forecasted cash flow statement correctly, you will see that Lorien plans to distribute cash to shareholders through two different means in 2009. Which of these methods involves distributing an equal amount of cash for each share owned? Which of these methods channels the cash to shareholders who are the least optimistic about the prospects of the company?

some accountant you are early in the year 2009 john roberts a recent graduate of sou 691942

Some Accountant You Are!

Early in the year 2009, John Roberts, a recent graduate of Southeast State College, delivers the financial statements shown below to Laura Dennis of Dennis, Inc. After a quick review, Dennis exclaims, “What do you mean I had net income of $20,000? I borrowed $40,000 from the bank and my cash balance decreased by $2,000. I must have had a loss! Some accountant you are!” How should Mr. Roberts answer Ms. Dennis?

Dennis, Inc.

Comparative Balance Sheet

December 31, 2008 and 2007

Assets

 

2008

2007

Cash                                                           

$ 3,000

$ 5,000

Accounts receivable                                                

18,000

8,000

Inventory                                                        

20,000

15,000

Equipment (at cost)                                                

52,000

20,000

Accumulated depreciation                                            

(10,000)

(5,000)

Total assets                                                      

 $83,000

 $43,000

Liabilities and Stockholders’ Equity

 

 

Accounts payable                                                  

$ 4,000

$ 9,000

Notes payable—long term                                           

40,000

Common stock, $1 par                                              

2,000

2,000

Additional paid in capital                                             

18,000

18,000

Retained earnings                                                  

19,000

14,000

Total liabilities and stockholders’ equity                                  

 $83,000

 $43,000

 

Dennis, Inc.

Combined Statement of Income and Retained Earnings

For the Year Ended December 31, 2008

Sales                                                           

 

$240,000

Cost of goods sold                                                 

$150,000

 

Operating expenses (including depreciation of $5,000)                       

70,000

220,000

Net income                                                      

 

 $ 20,000

Add: Retained earnings, January 1, 2008                                  

 

14,000

Deduct: Dividends paid                                              

 

(15,000)

Retained earnings, December 31, 2008                                   

 

 $ 19,000

the secret of cash flow patterns kara nemrow a security analyst for primer mead amp 691945

The Secret of Cash Flow Patterns

Kara Nemrow, a security analyst for Primer Mead & Co., asserts that she can tell more about a company’s financial condition by looking at the trends of the negative or positive cash flows in the three categories than from other information found in the financial statements. She illustrates her theory with the following pattern of cash flows for Atlas Security over the past three years.

 

2008

2007

2006

Net income                                                      

+

+

Cash flows from:

 

Operating activities                                             

 

+

+

Financing activities                                              

+

+

+

Investing activities                                              

+

+

+

How do you think Kara would analyze this pattern? Do you agree that analyzing cash flow patterns provides superior analytical information?

w t grant what is ldquo cash flow rdquo the case of w t grant is a classic in cash f 691946

W. T. Grant: What Is “Cash Flow”?

The case of W. T. Grant is a classic in cash flow analysis. During the 1960s and 1970s, Grant was one of the largest retailers in the United States, with more than 1,200 stores nationwide. Grant was a stable New York Stock Exchange firm that had paid cash dividends every year since 1907.However, the inability of Grant’s operations to generate positive cash flow indicated the existence of serious problems. From 1966 through 1973, while Grant’s net income was steady at about $35 million per year, cash flow from operations was negative in every year except 1968 and 1969, and even in those years the positive cash flow generated was insignificant in amount. The results for the fiscal year ended January 31, 1973, are the most striking. Net income for the year was $38 million. A frequently used measure of “cash flow”(net income and depreciation) suggested that W. T. Grant’s operations generated $48 million in cash. However, actual cash flow generated by operations for the year was a negative $120 million. In October 1975, Grant filed for bankruptcy, and by early 1976, the company was liquidated and ceased to exist. What might have caused the “net income  depreciation” measure of cash flow to be positive when in fact actual cash flow from operations was negative? Under what circumstances is the “net income  depreciation” measure of cash flow a good estimate of actual cash flow from operations? When is it a bad measure?

can operating cash flow be manipulated lump steak company anticipates some difficult 691947

Can Operating Cash Flow Be Manipulated?

Lump steak Company anticipates some difficulty in meeting the operating cash flow level that financial analysts are expecting from the company this year. As a result, the chief financial officer (CFO) has ordered the accounts payable department to make no vendor payments in the month of December but to send assurances to the vendors that these missed payments will be made up in January. In addition, the CFO has instructed the purchasing agent to delay making any new inventory purchases until January. Finally, the CFO has made arrangements with a financing company to package a large number of Lump steak’s accounts receivable and “securitize,” or sell, them to the financing company. This is a way for Lump steak to receive its cash immediately without waiting for customers to pay their accounts. Some companies report the cash proceeds from securitizing accounts such as this as cash from financing activities; Lump steak has determined that the securitization cash inflows will be reported in the Operating Activities section of the statement of cash flows. At the beginning of Chapter 5, it was explained that manipulate reported earnings. Can reported operating cash flow be manipulated? Explain any difference in the actions necessary to manipulate reported earnings compared to the actions necessary to manipulate reported operating cash flow.

deciphering financial statements caterpillar caterpillar is a u s based manufacturer 691949

Deciphering Financial Statements (Caterpillar)

Caterpillar is a U.S. based manufacturer of construction machinery and heavy duty engines. Caterpillar’s consolidated comparative statement of cash flows for 2002, 2003, and 2004 follows. All amounts are in millions of U.S. dollars.

Years ended December 31,

2004

2003

2002

Profit                                                      

$ 2,035

$ 1,099

$ 798

Adjustments for non cash items:

 

 

 

Depreciation and amortization                                

1,397

1,347

1,220

Other                                                  

(113)

(69)

350

Changes in assets and liabilities:

 

 

 

Receivables—trade and other                                 

(7,616)

(8,115)

(6,323)

Inventories                                               

(1,391)

(286)

162

Accounts payable and accrued expenses                          

1,457

542

97

Other—net                                              

240

(129)

(266)

NET CASH USED FOR OPERATING ACTIVITIES                     

 $ (3,991)

 $(5,611)

 $(3,962)

Capital expenditures excluding equipment leased to others               

(926)

(682)

(728)

Expenditures for equipment leased to others                         

(1,188)

(1,083)

(1,045)

Proceeds from disposals of property, plant, and equipment               

673

761

561

Additions to finance receivables                                  

(8,930)

(6,868)

(5,933)

Collections of finance receivables                                 

6,216

5,251

4,569

Proceeds from sale of finance receivables                            

700

661

613

Collections of retained interests in securitized trade receivables           

5,722

7,129

5,917

Investments and acquisitions (net of cash acquired)                     

(290)

(268)

(294)

Proceeds from sale of partnership involvement                        

290

Other—net                                                 

(190)

(17)

(40)

NET CASH PROVIDED BY INVESTING ACTIVITIES                   

 $ 2,077

 $ 4,884

 $ 3,620

Dividends paid                                               

(534)

(491)

(481)

Common stock issued, including Treasury shares reissued                

317

157

10

Treasury shares purchased                                      

(539)

(405)

Proceeds from long term debt issued:

 

 

 

Machinery and Engines                                      

9

128

248

Financial Products                                         

5,079

5,506

3,889

Payments on long term debt:

 

 

 

Machinery and Engines                                      

$ (35)

$ (463)

$ (225)

Financial Products                                         

(2,973)

(3,774)

(3,114)

Short term borrowings—net                                     

$ 550 

$ 87 

$ (102) 

 

NET CASH PROVIDED BY FINANCING ACTIVITIES                  

1,874 

745 

225 

 

Effect of exchange rate changes on cash                            

143 

15 

26 

 

INCREASE (DECREASE) IN CASH AND SHORT TERM INVESTMENTS     

103

33

(91)

 

CASH AND SHORT TERM INVESTMENTS AT BEGINNING OF PERIOD    

342

309

400

 

CASH AND SHORT TERM INVESTMENTS AT END OF PERIOD          

$ 445

$ 342

$ 309

 

1. For each year reported Caterpillar reports a profit and each year cash flow from operating activities is negative. Identify the primary reason for the negative cash from operating activities.

2. How is Caterpillar compensating for the negative cash from operating activities?

3. In your opinion, is Caterpillar’s negative cash from operating activites sustainable over the long term?

deciphering financial statements lockheed martin corporation lockheed martin corpora 691952

Deciphering Financial Statements (Lockheed Martin Corporation)

Lockheed Martin Corporation is a well known producer of advanced aircraft, missiles, and space hardware. Lockheed Martin is most famous for its super secret research and development division, nicknamed the “Skunk Works.” Among the high tech aircraft developed at the Skunk Works are the SR 71 Blackbird spy plane and the F 117A Stealth fighter. The consolidated statement of cash flows from Lockheed Martin’s 2004 annual report is reproduced on page 277.

When investors and analysts use the term cash flow, they can mean a variety of things.

Some common definitions of cash flow are as follows:

(a) Net income + Depreciation

(b) Cash flow from operating activities

(c) Cash flow from operating activities + Cash paid for interest + Cash paid for income taxes

(d) Cash flow from operating activities Capital expenditures Dividends

Lockheed Martin Corporation
CONSOLIDATED STATEMENT OF CASH FLOWS

 

Year ended December 31,

(in millions)

2004

2003

2002

Operating Activities

 

 

 

Earnings from continuing operations                      

$1,266

$1,053

$533

Adjustments to reconcile earnings (loss) from continuing

 

 

 

operations to net cash provided by operating activities:

 

 

 

Depreciation and amortization                        

511

480

433

Amortization of purchased intangible assets              

145

129

125

Deferred federal income taxes                        

(58)

467

(463)

Write down of investments and other charges           

151

42

1,127

Loss from discontinued operations                     

(33)

Changes in operating assets and liabilities:

 

 

 

Receivables                                       

(87)

(258)

394

Inventories                                       

519

(94)

585

Accounts payable                                  

288

330

(317)

Customer advances and amounts in excess of costs incurred

(228)

(285)

(460)

Income taxes                                     

(63)

(16)

44

Other                                              

480

(39)

320

Net cash provided by operating activities                   

2,924

1,809

2,288

Investing Activities

 

 

 

Expenditures for property, plant, and equipment             

(769)

(687)

(662)

Proceeds from divestiture of businesses/investments

 

 

 

in affiliated companies                               

279

234

134

Purchase of short term investments, net                   

(156)

(240)

Acquisition of businesses/investments in affiliated companies    

(91)

(821)

(104)

Other                                              

29

53

93

Net cash used for investing activities                   

(708)

(1,461)

(539)

Financing Activities

 

 

 

Repayments of long term debt                           

(1,089)

(2,202)

(110)

Issuances of long term debt                             

1,000

Long term debt repayment and issuance costs               

(163)

(175)

Issuances of common stock                             

164

44

436

Repurchases of common stock                          

(673)

(482)

(50)

Common stock dividends                              

(405)

(261)

(199)

Net cash (used for) provided by financing activities        

(2,166

(2,076)

77

Net (decrease) increase in cash and cash equivalents          

50

(1,728)

1,826

Cash and cash equivalents at beginning of year              

1,010

2,738

912

Cash and cash equivalents at end of year                

$1,060

$1,010

$2,738

Supplemental Disclosure Information

 

 

 

Cash paid during the year for:

 

 

 

Interest                                          

$420

$519

$586

Taxes                                           

363

170

55

Instructions:

1. Using the data from Lockheed’s statement of cash flows, compute values for the four measures of cash flow defined above for 2002, 2003, and 2004. Use earnings from continuing operations as net income. For capital expenditures, use expenditures for property, plant, and equipment.

2. One of the definitions (a) through (d) is sometimes given the title free cash flow because it indicates the amount of discretionary cash generated by a business. Free cash flow is thought of as the amount of cash that an owner can remove from a business without harming its long term potential. Which of these four definitions do you think applies to free cash flow? Explain.

3. A leveraged buyout (LBO) is the purchase of a company using borrowed money. The idea behind an LBO is to borrow the money, buy the company, and then repay the loan using the cash flow generated by the purchased company. Which of the four definitions of cash flow do you think would be particularly useful to someone considering an LBO? Explain.

writing assignment where is your statement of cash flows you are a senior credit ana 691953

Writing Assignment (Where is your statement of cash flows?)

You are a senior credit analyst for Far West Bank. The president of Moran Auto Sales has asked you for a loan of $2,000,000. Moran’s accountant has compiled and submitted a current balance sheet and income statement. Moran has had moderate income over the past 3 years but has found itself short of cash and therefore in need of the loan. After receiving the statements, you call Moran’s accountant and indicate that the financial statements are not complete; you need to see a statement of cash flows. The accountant argues,“ Everything on a statement of cash flows comes from the other two statements. Why make me do the additional work? Just analyze what we sent.” Your task now is to write a memo to the president of Moran Auto Sales convincing her that a statement of cash flows is essential for you to properly evaluate Moran’s loan application.

complete statement of cash flows from detailed data using the following information 691895

Complete Statement of Cash Flows from Detailed Data

Using the following information, prepare a complete statement of cash flows.

(a)

Cash balance, beginning

$ 1,500

(b)

Cash paid to purchase inventory

7,800

(c)

Cash received from sale of a building

5,600

(d)

Cash paid for interest

450

(e)

Cash paid to repay a loan

1,000

(f)

Cash collected from customers

10,000

(g)

Cash balance, ending

???

(h)

Cash received from issuance of new shares of common stock

1,200

(i)

Cash paid for dividends

780

(j)

Cash paid for income taxes

1,320

(k)

Cash paid to purchase machinery

1,950

operating cash flow restructuring charges using the following information compute ca 691897

Operating Cash Flow: Restructuring Charges

Using the following information, compute cash flow from operating activities.

Decrease in inventory                                                          

$ 300

Increase in wages payable                                                       

170

Restructuring charge                                                           

2,300

Depreciation                                                                 

1,000

Net income                                                                 

500

The restructuring charge consists of two elements: (1) $1,500 for the write down in value of certain assets and (2) $800 for recognition of an obligation to relocate employees; none of the relocation has yet taken place.

operating cash flow deferred income taxes using the following information compute ca 691898

Operating Cash Flow: Deferred Income Taxes

Using the following information, compute cash paid for income taxes.

Reported income tax expense                            

                       $20,000

 

End of Year

Beginning of Year

Income taxes payable

$ 1,250

$ 1,130

Deferred income tax liability                             

43,000

41,750

operating cash flow prepaid operating expenses using the following information compu 691900

Operating Cash Flow: Prepaid Operating Expenses

Using the following information, compute cash paid for operating expenses.

Operating expenses:

Depreciation     

$10,000

Insurance        

7,500

Wages          

14,600

Total operating expenses

$32,100

 

 

End of Year

Beginning of Year

Prepaid insurance

$1,500

$1,430

Wages payable

600

750

computing cash paid to purchase property plant and equipment using the following inf 691901

Computing Cash Paid to Purchase Property, Plant, and Equipment

Using the following information, compute cash paid to purchase property, plant, and equipment.

Depreciation expense

$10,000

 

End of Year

Beginning of Year

Property, plant, and equipment

$112,000

$106,000

Accumulated depreciation

31,000

44,000

During the year, property, plant, and equipment with an original cost of $35,000 was sold for a gain of $4,500.

computing cash received from the sale of property plant and equipment refer to pract 691902

Computing Cash Received from the Sale of Property, Plant, and Equipment

Refer to Practice 5 17. Compute the amount of cash received from the sale of the property, plant, and equipment.

Practice 5 17

Computing Cash Paid to Purchase Property, Plant, and Equipment

Using the following information, compute cash paid to purchase property, plant, and equipment.

Depreciation expense

$10,000

 

End of Year

Beginning of Year

Property, plant, and equipment

$112,000

$106,000

Accumulated depreciation

31,000

44,000

During the year, property, plant, and equipment with an original cost of $35,000 was sold for a gain of $4,500.

computing cash flow ratios using the following information compute the following rat 691904

Computing Cash Flow Ratios

Using the following information, compute the following ratios: (1) cash flow to net income, (2) cash flow adequacy, and (3) cash times interest earned.

Net income                                                                  

$10,000

Cash flow from operating activities                                                 

14,000

Cash paid for capital expenditures                                                 

25,000

Cash paid for acquisitions                                                       

15,000

Cash paid for interest                                                          

5,500

Cash paid for income taxes                                                       

7,500

articulation use the following information to answer the questions listed below divi 691905

Articulation

Use the following information to answer the questions listed below:

Dividends declared and paid                                                      

$ 8,000

Cash from investing activities                                                     

(25,000)

Cash from financing activities                                                     

(8,000)

 

 

End of Year

Beginning of Year

Cash                                               

$ 21,000

$ 12,000

Other assets                                         

210,000

227,000

Liabilities                                           

105,000

117,000

Common stock                                       

21,000

21,000

Retained earnings                                     

105,000

?

Compute the (a) cash from operating activities and (b) net income.

preparing a forecasted statement of cash flows the following balance sheet and incom 691906

Preparing a Forecasted Statement of Cash Flows

The following balance sheet and income statement information includes actual data for 2008 and forecasted data for 2009:

 

Actual 2008

Forecasted 2009

Cash                                              

$ 100

$ 130

Accounts receivable                                    

600

780

Inventory                                           

1,300

1,690

Property, plant, and equipment (net)                        

5,000

6,300

Accounts payable                                     

500

650

Long term debt                                        

4,000

5,000

Paid in capital                                        

1,000

1,400

Retained earnings                                     

1,500

1,850

Sales                                               

$10,000

$13,000

Cost of goods sold                                    

6,000

7,800

Depreciation expense                                  

1,000

1,200

Interest expense                                      

400

500

Income before income taxes                             

 $ 2,600

 $ 3,500

Income tax expense                                   

910

1,225

Net income                                         

 $ 1,690

 $ 2,275

Prepare a forecasted statement of cash flows for 2009. Use the indirect method of reporting cash flow from operating activities.

classification of cash flows indicate whether each of the following items would be c 691907

Classification of Cash Flows

Indicate whether each of the following items would be classified as (1) an operating activity, an investing activity, or a financing activity or (2) as a noncash transaction or noncash item.

(a) Cash collected from customers.

(b) Cash paid to suppliers for inventory.

(c) Cash received for interest on a nontrade note receivable.

(d) Cash received from issuance of stock.

(e) Cash paid for dividends.

(f) Cash received from bank on a loan.

(g) Cash paid for interest on a loan.

(h) Cash paid to retire bonds.

(i) Cash paid to purchase stock of another company as a long term investment.

(j) Cash received from the sale of a business segment.

(k) Cash paid for property taxes.

(l) Cash received for dividend revenue.

(m) Cash paid for wages.

(n) Cash paid for insurance.

(o) Preferred stock retired by issuing common stock.

(p) Depreciation expense for the year.

(q) Cash paid to purchase machinery.

(r) Cash received from the sale of land.

cash receipts and cash payments the accountant for alpine hobby stores prepared the 691909

Cash Receipts and Cash Payments

The accountant for Alpine Hobby Stores prepared the following selected information for the year ended December 31, 2008:

 

Dec 31, 2008

Dec 31, 2007

(a) Equipment                                          

$35,000

$40,000

(b) Accumulated Depreciation                               

11,000

9,500

(c) Long Term Debt                                      

13,000

20,000

(d) Common Stock                                      

20,000

15,000

Equipment with a book value of $20,000 was sold for $17,000 cash. The original cost of the equipment was $25,000. Determine the cash inflows and outflows during 2008 associated with each of the accounts listed. Indicate how the cash flows for each item would be presented on the statement of cash flows.

preparing the operating activities section of the statement of cash flows anakin inc 691910

Preparing the Operating Activities Section of the Statement of Cash Flows

Anakin, Inc., provides the following account balances for 2008 and 2007:

 

Dec 31, 2008

Dec 31, 2007

Accounts Receivable                                      

$ 18,700

$15,500

 

Inventory                                              

25,440

27,200

 

Accounts Payable                                        

21,650

22,400

 

Salaries Payable                                         

1,500

1,350

 

Sales                                                 

278,700

 

 

Cost of Goods Sold                                      

197,000

 

 

Depreciation Expense                                     

16,700

 

 

Salaries Expense                                         

35,200

 

 

Other Expenses                                         

24,300

 

 

         

Using the format presented in the chapter, prepare the Operating Activities section of the statement of cash flows and present that information using (a) the direct method and (b) the indirect method.

preparing the operating activities section of a statement of cash flows sith enterpr 691911

Preparing the Operating Activities Section of a Statement of Cash Flows

Sith Enterprises provides the following income statement for 2008:

Sales                                                                        

$765,200

Cost of goods sold                                                            

375,800

Gross margin                                                                

$389,400

Depreciation expense                                                          

42,000

Salaries expense                                                              

115,250

Interest expense                                                               

10,500

Other expenses                                                              

82,150

Income taxes expense                                                          

39,000

Net income                                                                  

$100,500

In addition, the following balance sheet information is available:

 

Dec 31, 2008

Dec 31, 2007

Accounts receivable                                      

$43,000

$39,000

Inventory                                              

68,200

65,400

Prepaid other expenses                                   

5,400

7,300

Accounts payable                                        

46,300

47,500

Interest payable                                         

900

1,100

Income taxes payable                                     

2,850

4,100

Using the format presented in the chapter, prepare the Operating Activities section of the statement of cash flows and present that information using (a) the direct method and (b) the indirect method.

format of statement of cash flows with indirect method from the following informatio 691912

Format of Statement of Cash Flows with Indirect Method

From the following information for Carter Corporation, prepare a statement of cash flows for the year ended December 31, 2008, using the indirect method.

Amortization of patent                                                          

$ 4,000

Depreciation expense                                                          

7,000

Issuance of common stock                                                       

25,000

Issuance of new bonds payable                                                    

30,000

Net income                                                                 

55,000

Payment of dividends                                                           

22,500

Purchase of equipment                                                         

33,200

Retirement of long term debt                                                     

40,000

Sale of land (includes $6,000 gain)                                                 

35,000

Decrease in accounts receivable                                                   

2,100

Increase in inventory                                                            

1,200

Increase in accounts payable                                                     

1,500

Increase in cash                                                               

56,700

Cash balance, January 1, 2008                                                     

82,800

cash flow from operations mdash indirect method the following information was taken 691913

Cash Flow from Operations—Indirect Method

The following information was taken from the books of Tap water Company. Compute the amount of net cash provided by (used in) operating activities during 2008 using the indirect method.

 

Dec 31, 2008

Dec 31, 2007

Accounts receivable                                      

$18,900

$16,750

Accounts payable                                        

11,500

14,000

Accumulated depreciation (no plant assets retired during year)       

29,000

22,000

Inventories                                             

24,500

20,000

Other current liabilities                                   

5,000

3,000

Prepaid insurance                                        

1,200

2,000

Net income                                            

35,500

cash flow from operations mdash indirect method the following information was taken 691915

Cash Flow from Operations—Indirect Method

The following information was taken from the comparative financial statements of Tulip Corporation:

Net income for year                                                           

$ 75,000

Sales revenue                                                                

450,000

Cost of goods sold (except depreciation)                                            

275,000

Depreciation expense for year                                                    

50,000

Amortization of intangible assets for year                                             

20,000

Interest expense on short term debt for year                                         

5,200

Dividends declared and paid during year                                             

35,000

Selected account balances:

 

Beginning of Year

End of Year

Accounts Receivable                                   

$22,000

$15,000

Inventory                                           

35,000

40,000

Accounts Payable                                     

47,500

52,000

Interest Payable                                      

1,200

400

Using the indirect method, compute the net amount of cash provided by (used in) operating activities for the year.

statement of cash flows mdash indirect method following is information for goulding 691917

Statement of Cash Flows—Indirect Method

Following is information for Goulding Manufacturing Company:

(a) Long term debt of $500,000 was retired at face value.

(b) New machinery was purchased for $62,000.

(c) Common stock with a par value of $100,000 was issued for $160,000.

(d) Dividends of $22,000 declared in 2007 were paid in January 2008, and dividends of $30,000 were declared in December 2008, to be paid in 2009.

(e) Net income was $450,700. Included in the computation were depreciation expense of $70,000 and intangible assets amortization of $10,000.

 

Dec 31, 2008

Dec 31, 2007

Current assets:

 

 

Cash and cash equivalents

$189,200

$130,000

Accounts receivable

175,000

156,000

Inventory

178,000

160,000

Current liabilities:

 

 

Accounts payable

64,000

87,400

Dividends payable

30,000

22,000

Interest payable

12,900

7,000

Wages payable

24,000

17,000

Prepare a statement of cash flows for the year ended December 31, 2008, using the indirect method.

articulation the following information is available for brimley inc note all invento 691918

Articulation

The following information is available for Brimley Inc. Note: All inventory is purchased on account, and Accounts Payable relates only to the purchase of inventory.

 

Dec 31, 2008

Dec 31, 2007

Accounts receivable                                      

$ ?

$50,000

Inventory                                              

65,000

70,000

Accounts payable                                        

47,000

40,000

Sales                                                 

483,000

 

Cost of goods sold                                       

?

 

Cash collected from customers                              

471,000

 

Cash paid for inventory                                   

?

 

Inventory purchased on account                             

295,000

 

Compute the following for 2008:

1. The ending balance in accounts receivable

2. The amount of cash paid for inventory

3. The amount of cost of goods sold

when should revenue be recognized stan crowfoot is a renowned sculptor who specializ 691869

When Should Revenue Be Recognized?

Stan Crowfoot is a renowned sculptor who specializes in Native American sculptures. Typically, a cast is prepared for each work to permit the multiple reproduction of the pieces. A limited number of copies are made for each sculpture, and the mold is destroyed after the number is reached. Limiting the number of pieces enhances the price, and most of the pieces have initially sold for $2,000 to $4,000. To encourage sales, Stan has a liberal return policy that permits customers to return any unwanted piece for a period of up to one year from the date of sale and receive a full refund. Do you think Stan should recognize revenue (1) when the piece is produced and cast in bronze, (2) when the goods are delivered to the customer, or (3) when the period of return has passed? Justify your answer in terms of the FASB conceptual framework.

we just changed our minds management for marlowe manufacturing company decided in 20 691871

We Just Changed Our Minds

Management for Marlowe Manufacturing Company decided in 2007 to discontinue one of its unsuccessful product lines. (The product line does not meet the definition of a business component.) The planned discontinuance involved obsolete inventory, assembly lines, and packaging and advertising supplies. It was estimated that a loss of $250,000 would result from the decision, and this estimate was recorded as a restructuring charge in the 2007 income statement. In 2008, new management was appointed, and it was decided that the unsuccessful product line could be turned around with a more aggressive marketing policy.

The change was made, and indeed the product began to make money. The new management wants to reverse the adjustment made the previous year and remove the liability for the estimated loss. How should the 2007 estimated loss be reported in the 2008 income statement? How should the 2008 reversal of the 2007 action be reported in the 2008 financial statements?

what was last year rsquo s income walesco corporation has decided to discontinue an 691874

What Was Last Year’s Income?

Walesco Corporation has decided to discontinue an entire component of its business effective November 1, 2008. It hopes to sell the assets involved and convert the physical plant to other uses within the manufacturing division. The CPA auditing the books indicates that GAAP requires separate identification of the revenues and expenses related to the component to be sold and their removal from the continuing revenue and expense amounts. The controller objects to this change. “We have already distributed last year’s numbers. If we change them now, one year later, confidence in our financial statements will be greatly eroded.” What are the pros and cons of identifying separately the costs related to the discontinued component?

accrual accounting near the end of the fiscal year preliminary financial results rev 691875

Accrual Accounting

Near the end of the fiscal year, preliminary financial results revealed that Stancomb Wills Company was in danger of not meeting corporate performance goals. According to an article in the business press, top executives at Stancomb Wills responded by deferring many expenses “beyond accepted accounting norms, and revenue was inappropriately booked far in advance.” These practices had the effect of “making the current quarter look more profitable. ”The top executives of Stancomb Wills were hoping that an upturn in the economy would spur sales that would provide additional profits to cover the deceptive accounting practices.

1. How are expenses deferred and revenues booked (recorded) in advance? What would the journal entries be?

2. Why would top executives encourage these misleading accounting practices?

3. None of the top executives who ordered the misstatements actually made the journal entries. If you were Stancomb Wills’ accountant, what would you have done?

4. Is Stancomb Wills’ independent auditor responsible for detecting these types of misstatements?

revenue recognition a common method for inflating revenues and profits is to ship mo 691876

Revenue Recognition

A common method for inflating revenues and profits is to ship more inventory to customers than they order. Business Week illustrates two instances in which the revenue recognition criteria may have been compromised. Using a practice known as “trade loading,”

RJR Nabisco, the second largest cigarette producer in America, would ship more inventory to wholesalers than the wholesalers could resell. The excess inventory would eventually be returned, but RJR would book the revenue and profit when the cigarettes were originally shipped. Management stopped this practice in 1988, and the result was a $360 million decrease in operating profits for 1989. Another company, Regina Co., took trade loading several steps further. In a hurried effort to compete in the upright vacuum cleaner market, Regina skipped proper testing of its product, the Housekeeper. The result was that 40,000 units, or 16% of sales, were returned. Regina’s solution was to lease a building to store the returned items and make no entries to record the returns. In a continued effort to make Regina’s stock attractive, the firm began to record sales when goods were ordered, not when they were shipped. Furthermore, to ensure that projected sales figures were achieved for the fiscal year ending June 30, 1988, the company generated $5.4 million of fictitious sales invoices for the last three business days of the year. SOURCES: Wafecia Konrad,“RJR Nabisco,” Business Week, February 19, 1990; John A. Byrne, “Regina,” Business Week, February 12, 1990.

1. Do these transactions of RJR Nabisco and Regina satisfy the revenue recognition criteria as set forth by the FASB?

2. If RJR Nabisco has open contracts with distributors that require distributors to attempt to sell all inventory shipped to them, does trade loading violate the revenue recognition criteria?

3. Regina recorded revenue when goods were ordered rather than when the goods were shipped. Does it really make a difference when the journal entry is made?

4. As Regina’s accountant, what would you do if the president of the company who was fined $50,000 and sentenced to one year in jail asked for your assistance in “cooking the books”?

financial statement analysis mdash ratios shawn o rsquo neil owns two businesses a d 691877

Financial Statement Analysis—Ratios

Shawn O’Neil owns two businesses, a Dr.ug store and a retail department store.

Dr.ug Store

Department Store

Net sales

$1,050,000

$670,000

Cost of goods sold

950,000

560,000

Other expenses

39,500

66,500

Which business earns more income? Which business has the higher gross profit percentage?

Return on sales? Which business would you consider more profitable?

deciphering financial statements the walt disney company locate the 2004 financial s 691878

Deciphering Financial Statements (The Walt Disney Company)

Locate the 2004 financial statements for The Walt Disney Company on the Internet.

1. Did Disney have any below the line items in 2004? Explain.

2. Disney’s net income increased from $1,267 million in 2003 to $2,345 million in 2004.

Identify the major reasons for the increase.

3. Imagine that you are a financial analyst asked to generate a forecast of Disney’s net income for 2005.You know that generally the best place to start in forecasting next year’s net income is this year’s net income. Given this starting point, look at the items in Disne ’s 2004 income statement and make a forecast of 2005 net income.

4. In its income statement, Disney separates reported net income into earnings or loss attributed to Disney common stock and to the Internet Group common stock. However, Disney reports the following in its 10 K filing: “During the year the Company converted all of its outstanding Internet Group common stock into Disney common stock and changed the reporting structure of the various components of the Internet Group. Accordingly, the Company no longer reports separate results for the Internet Group.” This statement can be confirmed by looking at Disney’s balance sheet; the September 30, 2004, balance for Internet Group common stock is zero. Why do you think that Disney reported separate results for the Internet Group? Why do you think that Disney decided to stop reporting the separate results?

5. What was Disney’s comprehensive income for 2004?

6. Of Disney’s four major segments—media networks, parks and resorts, studio entertainment, and consumer products—which generated the most revenue in 2004? The most operating income? Which had the highest operating profit margin (operating income/revenue)?

7. What percent of total revenue does Disney generate within the United States and Canada?

8. How does Disney recognize revenue from broadcast advertising? From advance theme park ticket sales?

9. Does Disney expense its film and television costs using direct matching, systematic and rational allocation, or immediate recognition?

10. How does Disney expense its parks, resorts, and other properties?

deciphering financial statements pfizer pfizer is one of the largest pharmaceutical 691879

Deciphering Financial Statements (Pfizer)

Pfizer is one of the largest pharmaceutical and consumer healthcare products companies in the world. Familiar products sold by Pfizer include Sudafed, Zantac, Bena Dr. yl, Listerine, and Viagra. The company’s highest selling product is Lipitor, which is designed to help reduce high cholesterol. Pfizer’s income statement for 2004 follows.

Pfizer Inc. and Subsidiary Companies

CONSOLIDATED STATEMENT OF INCOME

 

Year Ended December 31

(millions, except per share data)

2004

2003

2002

Revenues                                                   

$52,516

$44,736

$32,294

 

Costs and expenses:

 

 

 

 

Cost of sales                                              

7,541

9,589

4,014

 

Selling, informational and administrative expenses                    

16,903

15,108

10,829

 

Research and development expenses                            

7,684

7,487

5,208

 

Amortization of intangible assets                               

3,364

2,187

22

 

Merger related in process research and development charges           

1,071

5,052

 

Merger related costs                                        

1,193

1,058

630

 

Other income—net                                         

753 

1,009 

(175) 

 

Income from continuing operations before provision for taxes

 

 

 

 

on income and minority interests                               

14,007

3,246

11,766

 

Provision for taxes on income                                    

2,665

1,614

2,599

 

Minority interests                                             

10 

 

Income from continuing operations before cumulative effect

 

 

 

 

of change in accounting principles                               

11,332

1,629

9,161

 

Discontinued operations:

 

 

 

 

Income/(loss) from operations of discontinued business

 

 

 

 

and product lines—net of tax                               

(22)

26

298

 

Gains on sales of discontinued businesses and product

 

 

 

 

lines—net of tax                                        

51

2,285

77

 

Discontinued operations—net of tax                               

29

2,311

375

 

Income before cumulative effect of change in accounting

 

 

 

 

principles                                                

11,361

3,940

9,536

 

Cumulative effect of change in accounting principles—net

 

 

 

 

of tax                                                   

— 

(30) 

(410) 

 

Net income                                                  

$11,361 

$ 3,910 

$ 9,126 

 

EARNINGS PER COMMON SHARE—BASIC:

 

 

 

 

Income from continuing operations before cumulative effect

 

 

 

 

of change in accounting principles                               

$ 151

$ 022

$ 149

 

Discontinued operations                                        

032

006

 

Income before cumulative effect of change in accounting

 

 

 

 

principles                                                

151

054

155

 

Cumulative effect of change in accounting principles                    

— 

— 

(007) 

 

Net income                                                 

$ 151

$ 054

$ 148

 

EARNINGS PER COMMON SHARE—DILUTED:

 

 

 

 

Income from continuing operations before cumulative effect

 

 

 

 

of change in accounting principles                               

$ 149

$ 022

$ 147

 

Discontinued operations                                        

032

006

 

Income before cumulative effect of change in accounting

 

 

 

 

principles                                                

149

054

153

 

Cumulative effect of change in accounting principles                    

— 

— 

(007) 

 

Net income                                                 

$ 149

$ 054

$ 146

 

The following information came from Pfizer’s statement of stockholders’ equity:

Dividends                                                   

$5,251

$4,771

$3,313

Currency translation adjustment                                  

1,961

2,070

85

Net unrealized gain (loss) on available for sale securities                 

128

68

(32)

Minimum pension liability                                       

(6)

(68)

(179)

In addition, in the notes to its financial statements, Pfizer reports that advertising expenses in 2002, 2003, and 2004 were $2,298 million, $2,936 million, and $3,490 million, respectively. Advertising expense is reported as part of selling, informational, and administrative expenses.

1. Compute the following for each of the years 2002–2004:

(a) Net income/Revenues

(b) Cost of sales/Revenues

(c) Research and development expenses/Revenues

(d) Advertising expense/Revenues

2. Comment on the ratios you computed in part (1). Make particular mention of any trends.

3. Compute Pfizer’s effective tax rate (on continuing operations) for each year.

4. For 2004, estimate the average number of basic and diluted shares outstanding.

5. Compute comprehensive income for each of the years 2002–2004.

deciphering financial statements wells fargo amp company wells fargo amp company is 691880

Deciphering Financial Statements (Wells Fargo & Company)

Wells Fargo & Company is the fourth largest bank in the United States (based on total assets as of December 31, 2004).Wells Fargo is the successor to the banking and stagecoach company founded by Henry Wells and William G. Fargo in 1852.The company’s consolidated statement of income follows.

Wells Fargo & Company and Subsidiaries
Consolidated Statement of Income

 

For the Years Ended December 31

(in millions, except per share amounts)

2004

2003

2002

INTEREST INCOME

 

 

 

Trading assets                             

$145

$156

$169

Securities available for sale                    

1,883

1,816

2,424

Mortgages held for sale                      

1,737

3,136

2,450

Loans held for sale                          

292

251

252

Loans                                   

16,781

13,937

13,045

Other interest income                       

129

122

119

Total interest income                     

20,967

19,418

18,459

INTEREST EXPENSE

 

 

 

Deposits                                 

1,827

1,613

1,919

Short term borrowings                      

353

322

536

Long term debt                            

1,637

1,355

1,404

Guaranteed preferred beneficial interests in Company’s

 

 

 

subordinated debentures                   

121

118

Total interest expense                     

3,817

3,411

3,977

NET INTEREST INCOME                      

$17,150

$16,007

$14,482

Provision for credit losses                      

1,717

1,722

1,684

Net interest income after provision for credit losses   

15,433

14,285

12,798

NONINTEREST INCOME

 

 

 

Service charges on deposit accounts               

2,417

2,297

2,134

Trust and investment fees                       

2,116

1,937

1,875

Card fees                                  

1,230

1,079

977

Other fees                                 

1,779

1,560

1,372

Mortgage banking                             

1,860

2,512

1,713

Operating leases                             

836

937

1,115

Insurance                                  

1,193

1,071

997

Net gains (losses) on debt securities available for sale  

(15)

4

293

Net gains (losses) from equity investments          

394

55

(327)

Other                                     

1,099

930

618

Total noninterest income                    

12,909

12,382

10,767

NONINTEREST EXPENSE

 

 

 

Salaries                                    

5,393

4,832

4,383

Incentive compensation                        

1,807

2,054

1,706

Employee benefits                            

1,724

1,560

1,283

Equipment                                  

1,236

1,246

1,014

Net occupancy                              

1,208

1,177

1,102

Operating leases                             

633

702

802

Other                                     

5,572

5,619

4,421

Total noninterest expense                    

17,573

17,190

14,711

INCOME BEFORE INCOME TAX EXPENSE AND EFFECT

 

 

 

OF CHANGE IN ACCOUNTING PRINCIPLE     

10,769

9,477

8,854

Income tax expense                           

3,755

3,275

3,144

NET INCOME BEFORE EFFECT OF CHANGE IN

 

 

 

ACCOUNTING PRINCIPLE                  

7,014

6,202

5,710

Cumulative effect of change in accounting principle    

(276)

NET INCOME                               

$7,014

$6,202

$5,434

EARNINGS PER COMMON SHARE BEFORE EFFECT OF

 

 

 

CHANGE IN ACCOUNTING PRINCIPLE

 

 

 

Earnings per common share                  

$415.00

$369.00

$335.00

Diluted earnings per common share            

409

365

332

EARNINGS PER COMMON SHARE

 

 

 

Earnings per common share                  

$415.00

$369.00

$319.00

Dilute earnings per common share             

409

365

316

DIVIDENDS DECLARED PER COMMON SHARE     

$186.00

$15.00

$186.00

1. How is this income statement different from all the other income statements illustrated in this chapter?

2. For a merchandising firm, gross profit represents sales less cost of goods sold. For Wells Fargo, what component of the income statement would be similar to gross profit?

3. Compute the following ratios for each of the years 2002–2004:

(a) Total interest expense/Total interest income

(b) Incentive compensation/Salaries

(c) Employee benefits/Salaries

4. Comment on the ratios you computed in part (3).Make particular mention of any trends.

5. The average loans receivable balance for Wells Fargo during 2004 was $266,503 million. The average amount of deposits during 2004 was $261,193 million. Using the income statement data, comment on the average interest rate Wells Fargo pays to its depositors, the average interest rate Wells Fargo earns on its loans receivable, and the spread between these two rates.

6. The market value of Wells Fargo’s stock at the end of each year was $62.15, $58.89, and $46.87 for the years 2004, 2003, and 2002, respectively. Compute the firm’s price earnings ratio for each year. Use diluted earnings per share. Is it increasing or decreasing over time?

deciphering financial statements the reader rsquo s digest association inc reader rs 691882

Deciphering Financial Statements (Ford Motor Company)

The consolidated statement of income for Ford Motor Company appears at the top of the following page.

1. What is the first thing you notice about the way revenues and expenses are partitioned?

2. For the Automotive division, compute the ratio (Cost of sales/Sales) for each of the three years presented. Interpret the results.

3. Look at the operating results for the Automotive division. Is there any good news for Ford in these results?

4. Depreciation expense is reported by the Financial Services division but not by the Automotive division. Explain why the Automotive division does not report depreciation expense.

5. Which of the company’s two divisions seems to be performing better over time?

6. Is Ford a car company that finances automobiles or a finance company that makes cars?

Ford Motor Company and Subsidiaries Consolidated Statement of Income

 

For the Years Ended December 31

 

2004

2003

2002

(in millions)

 

 

 

AUTOMOTIVE

 

 

 

Sales                                             

$147,134

$138,260

$134,120

Costs and Expenses

 

 

 

Cost of Sales                                       

135,856

129,685

125,027

Selling, administrative and other expenses                  

11,455 

10,131 

9,697 

Total Costs and Expenses                           

147,311 

139,816 

134,724 

Operating Income(loss)                               

177

1,556

604

Interest Expense                                    

1,221

1,323

1,333

Interest Income and other non operating income

 

 

 

(expense), net                                   

988

897

974

Equity in net income(loss) of affiliated companies             

255

74 

91 

Income(loss) before income taxes—automotive             

155

1,908

1,054

FINANCIAL SERVICES

 

 

 

Financial Services revenues                             

24,518

26,078

28,138

Costs and Expenses

 

 

 

Interest Expense                                    

5,850

6,320

7,468

Depreciation                                       

6,618

8,771

10,154

Operating and other expenses                          

5,830

5,492

5,345

Provision for credit and insurance losses                   

1,212 

2,248 

3,053 

Total Costs and Expenses                           

19,510 

22,831 

26,020 

Income(loss) before income taxes—financial services         

5,008

3,247

2,118

TOTAL COMPANY

 

 

 

Income(loss) before income taxes                       

4,853

1,339

1,064

Provision for(benefit from) income taxes                  

937 

123 

342 

INCOME(LOSS) BEFORE MINORITY INTERESTS           

3,916

1,216

722

Minority interests in net income(loss) of subsidiaries          

282 

314 

367 

INCOME FROM CONTINUING OPERATIONS             

3,634

902

355

Income(loss) from discontinued operations                 

147

143

333

Cumulative effect of change in accounting principle           

— 

264 

1,002 

Net income (loss)                                   

3,487  

495  

980  

ethical dilemma far from being an exact science accounting involves estimation and j 691883

Ethical Dilemma

Far from being an exact science, accounting involves estimation and judgment. Consider the case of Dwight Nelson, chief financial officer of Pilot Enterprises. Pilot is a relatively young, privately held company with thoughts of going public in the near future. The owners of the business would like to include in the prospectus (a document containing information about the company and its past performance) financial statements that support their assertion that Pilot is a successful company with a bright future. And the problem is this—the income statement for the past year shows a slight decrease in income from the prior period. When Dwight presented this information to the board of directors of Pilot, he was told that the income statement would have to be revised. He was specifically counseled to review his estimates associated with bad debt expense, warranty expense, and estimated useful life of depreciable assets. He was invited to present his “revised” income statement to the board of directors when it showed a 5% increase over last period’s net income—anything less would not do. After reviewing the assumptions made regarding uncollectible, warranties, and depreciation, Dwight found that he could revise his estimates and obtain the 5% target increase in income. But he did not feel that the revised income statement properly reflected the performance of Pilot for the period.

1. What are the risks to Dwight of revising the income statement to meet the target figure?

2. What are the risks to Dwight of not revising the income statement to meet the target figure?

cumulative spreadsheet analysis this spreadsheet assignment is a continuation of the 691884

Cumulative Spreadsheet Analysis

This spreadsheet assignment is a continuation of the spreadsheet assignment given in Chapter 3. If you completed that assignment, you have a head start on this one. Refer back to the instructions for preparing the revised financial statements for 2008 as given in part (1) of the Cumulative Spreadsheet Analysis assignment in Chapter 3. Clearly state any additional assumptions that you make. Skywalker wishes to prepare a forecasted balance sheet and a forecasted income statement for 2009. Use the financial statement numbers for 2008 [given in part (1) of the Cumulative Spreadsheet Analysis assignment in Chapter 3] as the basis for the forecast, along with the following additional information.

(a) Sales in 2009 are expected to increase by 40% over 2008 sales of $2,100.

(b) In 2009, Skywalker expects to acquire new property, plant, and equipment costing $240.

(c) The $480 in operating expenses reported in 2008 breaks down as follows: $15 depreciation expense and $465 other operating expenses.

(d) No new long term debt will be acquired in 2009.

(e) No cash dividends will be paid in 2009.

(f) New short term loans payable will be acquired in an amount sufficient to make Skywalker’s current ratio in 2009 exactly equal to 2.0.

(g) Skywalker does not anticipate repurchasing any additional shares of stock during 2009.

(h) Because changes in future prices and exchange rates are impossible to predict, Skywalker’s best estimate is that the balance in accumulated other comprehensive income will remain unchanged in 2009.

(i) In the absence of more detailed information, assume that investment securities, long term investments, other long term assets, and intangible assets will all increase at the same rate as sales (40%) in 2009.

(j) In the absence of more detailed information, assume that other long term liabilities will increase at the same rate as sales (40%) in 2009.

cash flow patterns identify which of the following cash flow patterns most likely be 691887

Cash Flow Patterns

Identify which of the following cash flow patterns most likely belongs to (1) a start up, high growth company, (2) a steady state company, and (3) a cash cow.

 

Operating

Investing

Financing

 

Cash Flow

Cash Flow

Cash Flow

Company A

$(10,000)

$(27,000)

$ 40,000

Company B

40,000

(27,000)

(10,000)

Company C

30,000

(27,000)

(1,500)

               

noncash investing and financing activities combining the following information compu 691888

Noncash Investing and Financing Activities

Combining the following information, compute the total amount of (1) cash flow from investing activities and (2) cash flow from financing activities.

(a) Purchased a building for $120,000.Paid $40,000 and signed a mortgage with the seller for the remaining $80,000.

(b) Executed a debt equity swap: replaced a $67,000 loan by giving the lender shares of common stock worth $67,000 on the date the swap was executed.

(c) Purchased land for $100,000. Signed a note for $35,000 and gave shares of common stock worth $65,000.

(d) Borrowed $56,000 under a long term loan agreement. Used the cash from the loan proceeds as follows: $15,000 for purchase of additional inventory, $30,000 to pay cash dividends, and $11,000 to increase the cash balance.

general format for a statement of cash flows organize the following summary informat 691889

General Format for a Statement of Cash Flows

Organize the following summary information into the proper format for a statement of cash flows.

Cash balance, beginning of year                                                   

$ 3,200

Cash flow from financing activities                                                 

10,000

Total stockholders’ equity, end of year                                               

23,000

Cash flow from operating activities                                                

4,300

Cash balance, end of year                                                       

2,500

Cash flow from investing activities                                                 

(15,000)

Total stockholders’ equity, beginning of year                                          

20,000

cash collected from customers using the following information compute cash collected 691890

Cash Collected from Customers

Using the following information, compute cash collected from customers.

Sales                                                                       

$10,000

Cost of goods sold                                                            

5,300

Operating expenses                                                            

3,800

 

 

End of Year

Beginning of Year

Prepaid operating expenses                              

$1,000

$ 700

Accounts payable                                     

1,350

1,200

Inventory                                           

2,500

2,100

Accounts receivable                                   

1,400

1,375

cash paid for inventory purchases refer to the information in practice 5 6 compute c 691891

Cash Paid for Inventory Purchases

Refer to the information in Practice 5 6. Compute cash paid for inventory purchases.

Practice 5 6          

General Format for a Statement of Cash Flows

Organize the following summary information into the proper format for a statement of cash flows.

Cash balance, beginning of year                                                   

$ 3,200

Cash flow from financing activities                                                 

10,000

Total stockholders’ equity, end of year                                              

23,000

Cash flow from operating activities                                                

4,300

Cash balance, end of year                                                        

2,500

Cash flow from investing activities                                                 

(15,000)

Total stockholders’ equity, beginning of year                                          

20,000

 

cash paid for operating expenses refer to the information in practice 5 6 compute ca 691892

Cash Paid for Operating Expenses

Refer to the information in Practice 5 6. Compute cash paid for operating expenses.

Practice 5 6          

General Format for a Statement of Cash Flows

Organize the following summary information into the proper format for a statement of cash flows.

Cash balance, beginning of year                                                   

$ 3,200

Cash flow from financing activities                                                 

10,000

Total stockholders’ equity, end of year                                              

23,000

Cash flow from operating activities                                                

4,300

Cash balance, end of year                                                        

2,500

Cash flow from investing activities                                                 

(15,000)

Total stockholders’ equity, beginning of year                                          

20,000

direct method using the following income statement and cash flow adjustment informat 691893

Direct Method

Using the following income statement and cash flow adjustment information, prepare the operating cash flow section of the statement of cash flows using the direct method.

Sales              

$4,000

Cost of goods sold   

1,700

Interest expense     

350

Depreciation expense 

800

Net income        

$1,150

Adjustments:

(a) Interest payable decreased by $60.

(b) Accounts receivable decreased by $290.

(c) Inventory increased by $500.

(d) Accounts payable decreased by $130.

indirect method refer to practice 5 9 prepare the operating cash flow section of the 691894

Indirect Method

Refer to Practice 5 9. Prepare the operating cash flow section of the statement of cash flows using the indirect method.

Practice 5 9

Direct Method

Using the following income statement and cash flow adjustment information, prepare the operating cash flow section of the statement of cash flows using the direct method.

Sales              

$4,000

Cost of goods sold   

1,700

Interest expense     

350

Depreciation expense 

800

Net income        

$1,150

Adjustments:

(a) Interest payable decreased by $60.

(b) Accounts receivable decreased by $290.

(c) Inventory increased by $500.

(d) Accounts payable decreased by $130.

intraperiod income tax allocation nephi corporation reported the following income it 691842

Intraperiod Income Tax Allocation

Nephi Corporation reported the following income items before tax for the year 2008:

Income from continuing operations before income taxes

$260,000

Loss from operations of a discontinued business component

70,000

Gain from disposal of a business component 

40,000

Extraordinary gain

110,000

The income tax rate is 35% on all items. Prepare the portion of the income statement beginning with Income from continuing operations before income taxes for the year ended December 31, 2008, after applying proper intraperiod income tax allocation procedures.

discontinued operations on june 30 2008 top management of garrison manufacturing co 691843

Discontinued Operations

On June 30, 2008, top management of Garrison Manufacturing Co. decided to dispose of an unprofitable business component. An operating loss of $130,000 associated with the component was incurred during the year. The plant facilities associated with the business segment were sold on December 1, and a $15,000 gain was realized on the sale of the plant assets.

(a) Assuming a 30% tax rate, prepare the discontinued operations section of Garrison Manufacturing Co.’s income statement for the year ending December 31, 2008.

(b) What additional information about the discontinued segment would be provided by Garrison Manufacturing if it were reporting using the accounting standards of the United Kingdom?

discontinued operations jason bond company operates two restaurants one in valencia 691844

Discontinued Operations

Jason Bond Company operates two restaurants, one in Valencia and one in Saugus. The operations and cash flows of each of the two restaurants are clearly distinguishable. During 2008, Jason Bond decided to close the restaurant in Saugus and sell the property; it is probable that the disposal will be completed early next year. The revenues and expenses of Jason Bond for 2008 and for the preceding two years are as follows:

 

2008

2007

2006

Sales—Valencia

$60,000

$48,000

$40,000

Cost of goods sold—Valencia

26,000

22,000

18,000

Other expenses—Valencia

14,000

13,000

12,000

Sales—Saugus

23,000

30,000

52,000

Cost of goods sold—Saugus

14,000

19,000

20,000

Other expenses—Saugus

17,000

16,000

15,000

The other expenses do not include income tax expense. During the later part of 2008, Jason Bond sold much of the kitchen equipment of the Saugus restaurant and recognized a pretax gain of $15,000 on the disposal. The income tax rate for Jason Bond is 35%. Prepare the 3 year comparative income statement for 2006–2008.

reporting items on financial statements under what classification would you report e 691846

Reporting Items on Financial Statements

Under what classification would you report each of the following items on the financial statements?

(a) Revenue from sale of obsolete inventory.

(b) Loss on sale of the fertilizer production division of a lawn supplies manufacturer.

(c) Loss stemming from expropriation of assets by a foreign government.

(d) Gain resulting from changing asset balances to adjust for the effect of excessive depreciation charged in error in prior years.

(e) Loss resulting from excessive accrual in prior years of estimated revenues from long term contracts.

(f) Costs incurred to purchase a valuable patent.

(g) Net income from the discontinued dune buggy operations of a custom car designer.

(h) Costs of rearranging plant machinery into a more efficient order.

(i) Error made in capitalizing advertising expense during the prior year.

(j) Gain on sale of land to the government.

(k) Loss from destruction of crops by a hailstorm.

(l) Additional depreciation resulting from a change in the estimated useful life of an asset.

(m) Gain on sale of long term investments.

(n) Loss from spring flooding.

(o) Sale of obsolete inventory at less than book value.

(p) Additional federal income tax assessment for prior years.

(q) Loss resulting from the sale of a portion of a business component.

(r) Costs associated with moving a U.S. business to Japan.

(s) Loss resulting from a patent that was recently determined to be worthless.

multiple step income statement from the following list of accounts prepare a multipl 691847

Multiple Step Income Statement

From the following list of accounts, prepare a multiple step income statement in good form showing all appropriate items properly classified, including disclosure of earnings per share data. (No monetary amounts are to be reported.)

Accounts Payable

Accumulated Depreciation—Office Building

Accumulated Depreciation—Office Furniture and Fixtures

Advertising Expense

Allowance for Bad Debts

Bad Debt Expense

Cash

Common Stock, $1 par (10,000 shares outstanding)

Depreciation Expense—Office Building

Depreciation Expense—Office Furniture and Fixtures

Dividend Revenue

Dividends Payable

Dividends Receivable

Extraordinary Gain (net of income taxes)

Federal Unemployment Tax Payable

Freight In

Goodwill

Income Tax Expense

Income Taxes Payable

Insurance Expense

Interest Expense—Bonds

Interest Expense—Other

Interest Payable

Interest Receivable

Interest Revenue

Inventory—beginning

Inventory—ending

Loss from Discontinued Operations (net of income taxes)

Miscellaneous General Expense

Miscellaneous Selling Expense

Office Salaries Expense

Office Supplies

Office Supplies Expense

Officers’ Salaries Expense

Property Taxes Expense

Purchase Discounts

Purchase Returns and Allowances

Purchases

Retained Earnings

Royalties Received in Advance

Royalty Revenue

Salaries and Wages Payable

Sales

Sales Discounts

Sales Returns and Allowances

Sales Salaries and Commissions

Sales Taxes Payable

single step income statement and statement of retained earnings jacksonville window 691848

Single Step Income Statement and Statement of Retained Earnings

Jacksonville Window Co. reports the following for 2008:

Retained earnings, January 1                                                     

$335,200

Selling expenses                                                              

$290,200

Sales revenue                                                                 

$1,420,000

Interest expense                                                              

$14,100

General and administrative expenses                                               

$224,800

Cost of goods sold                                                             

$772,000

Dividends declared this year                                                     

$40,000

Tax rate for all items                                                          

40%

Average shares of common stock outstanding during the year                             

30,000

Prepare a single step income statement (including earnings per share data) and a statement of retained earnings for Jacksonville.

statement of comprehensive income svedin incorporated provides the following informa 691850

Statement of Comprehensive Income

Svedin Incorporated provides the following information relating to 2008:

Net income                                                                 

$17,650

Unrealized losses on available for sale securities                                       

1,285

Foreign currency translation adjustment                                             

287

Minimum pension liability adjustment                                               

315

The foreign currency adjustment resulted from a weakening in the currencies of Svedin’s foreign subsidiaries relative to the U.S. dollar. The minimum pension liability adjustment required an increase in the pension liability with a resulting decrease in equity.

(Note: These items represent the results of events occurring during 2008, not the cumulative result of events in prior years.)

1. Determine the effect that each of these items would have when computing comprehensive income for 2008. Explain your rationale.

2. Prepare a statement of comprehensive income for Svedin Incorporated for 2008.

forecasted income statement han company wishes to forecast its net income for the ye 691851

Forecasted income Statement

Han Company wishes to forecast its net income for the year 2009. Han has assembled balance sheet and income statement data for 2008 and has also done a forecast of the balance sheet for 2009. In addition, Han has estimated that its sales in 2009 will rise to $2,200.This information is summarized in the following table.

 

 

2009

Balance Sheet

2008

Forecasted

Cash                                                      

$ 20

$ 22

Other current assets                                           

500

550

Property, plant, and equipment (net)                                

600

800

Total assets                                                 

 $1,120

 $1,372

 

 

 

Accounts payable                                             

$ 200

$ 220

Bank loans payable                                            

600

500

Total stockholders’ equity                                       

320

652

Total liabilities and stockholders’ equity                             

 $1,120

 $1,372

 

 

2009

Income Statement

2008

Forecasted

Sales                                                      

$2,000

$2,200

Cost of goods sold                                            

700

 

Gross profit                                                 

 $1,300

 

Depreciation expense                                          

120

 

Other operating expenses                                       

1,010 

 

Operating profit                                              

$ 170

 

Interest expense                                              

90

 

Income before taxes                                           

 $ 80

 

Income taxes                                                

30

 

Net income                                                 

 $ 50

 

Instructions:

Prepare a forecasted income statement for 2009. Clearly state what assumptions you make.

forecasted balance sheet and income statement ryan company wishes to prepare a forec 691852

Forecasted Balance Sheet and Income Statement

Ryan Company wishes to prepare a forecasted income statement and a forecasted balance sheet for 2009. Ryan’s balance sheet and income statement for 2008 follow.

Balance Sheet

2008

Cash                                                                      

$ 10

Other current assets                                                           

250

Property, plant, and equipment, net                                                

800 

Total assets                                                                 

$ 1,060 

Accounts payable                                                             

 $ 100

Bank loans payable                                                            

700

Total stockholders’ equity                                                       

260 

Total liabilities and stockholders’ equity                                             

$ 1,060 

Income Statement

2008

Sales                                                                      

$ 1,000

Cost of goods sold                                                             

750 

Gross profit                                                                 

$ 250

Depreciation expense                                                          

40

Other operating expenses                                                       

80

Operating profit                                                              

 $ 130

Interest expense                                                              

70 

Income before taxes                                                           

$ 60

Income taxes                                                                

20

Net income                                                                 

 $ 40

In addition, Ryan has assembled the following forecasted information regarding 2009:

(a) Sales are expected to increase to $1,500.

(b) Ryan expects to become more efficient at utilizing its property, plant, and equipment in 2009. Therefore, Ryan expects that the sales increase will not require any increase in property, plant, and equipment. Accordingly, the year 2009 property, plant, and equipment balance is expected to be $800.

(c) Ryan’s bank has approved a new long-term loan of $200. This loan will be in addition to the existing loan payable.

Instructions:

Prepare a forecasted balance sheet and a forecasted income statement for 2009. Clearly state what assumptions you make.

single step income statement mcgrath co on june 30 2008 reported a retained earnings 691853

Single Step Income Statement

McGrath Co. on June 30, 2008, reported a retained earnings balance of $1,475,000 before closing the books. The books of the company showed the following account balances on June 30, 2008:

Sales                                                                      

$2,870,000

Inventory: July 1, 2007                                                          

150,000

June 30, 2008                                                        

175,000

Sales Returns and Allowances                                                    

120,000

Purchases                                                                   

1,542,000

Purchase Discounts                                                            

32,000

Dividends Declared                                                            

300,000

Selling and General Expenses                                                     

283,000

Interest Revenue                                                             

72,000

Income Taxes                                                                 

270,900

Instructions:

Prepare a single step income statement and a retained earnings statement. McGrath Co. has 275,000 shares of common stock outstanding.

revenue recognition and preparation of income statement richmond company manufacture 691854

Revenue Recognition and Preparation of Income Statement

Richmond Company manufactures and sells robot type toys for chil Dr. en. Under one type of agreement with the dealers, Richmond is to receive payment upon shipment to the dealers. Under another type of agreement, Richmond receives payments only after the dealer makes the sale. Under this latter agreement, toys may be returned by the dealer. Richmond’s president desires to know how the income statement would differ under these two methods over a 2 year period.

The following information is made available for making the computations:

Sales price per unit:

 

If paid after shipment                                                         

$5

If paid after sale, with right of return                                            

$6

Cost to produce per unit (assume fixed quantity of toys is produced)                       

$3

Expected bad debt percentage of sales if revenue recognized at time of shipment               

5%

Expected bad debt percentage of sales if revenue recognized at time of sale                   

1/2%

Selling expenses—2008                                                          

$25,000

Selling expenses—2009                                                         

$15,000

General and administrative expenses—2008 and 2009                                  

$22,000

 

Quantity Shipped and Sold

 

2009

2008

Units shipped to dealers

30,000

25,000

Units sold by dealers

22,000

14,000

Instructions:

1. Prepare comparative income statements for 2008 and 2009 for each of the two types of dealer agreements assuming the company began operations in 2008.

2. Discuss the implications of the revenue recognition method used for each of the dealer agreements.

discontinued operations in process in 2008 laetner industries decided to discontinue 691858

Discontinued Operations in Process

In 2008, Laetner Industries decided to discontinue its Laminating Division, a separately identifiable component of Laetner’s business. At December 31, Laetner’s year end, the division has not been completely sold. However, negotiations for the final and complete sale are progressing in a positive manner, and it is probable that the disposal will be completed within a year. Analysis of the records for the year disclosed the following relative to the Laminating Division.

Operating loss for the year                                                      

$89,900

Loss on disposal of some Laminating Division assets during 2008                           

5,000

Expected operating loss in 2009 preceding final disposal                                 

45,000

Expected gain in 2009 on disposal of division                                          

20,000

Instructions:

Assuming a 35% tax rate, prepare the Discontinued Operations section of Laetner Industries’ income statement for the year ending December 31, 2008.

financial statement analysis mdash ratios the following financial statement informat 691859

Financial Statement Analysis—Ratios

The following financial statement information for Tronics Inc. is available:

(In thousands)

2008

2007

2006

Sales                                                      

$6,041

$5,872

$5,324

Cost of goods sold                                            

3,202

2,877

2,396

Operating expenses                                           

1,991

1,779

1,578

Income taxes                                                

165

222

280

The following information relates to the firm’s common stock for the same period:

 

2008

2007

2006

Shares outstanding          

1,000

1,000

1,000

Market value per share at year end

$813.00

$1,225.00

$1,532.00

Instructions:

1. For each year compute

(a) Gross profit percentage.

(b) Return on sales.

(c) Price earnings ratio.

2. Do you notice any significant trends as a result of this analysis?

income and retained earnings statements selected account balances of connell company 691860

Income and Retained Earnings Statements

Selected account balances of Connell Company for 2008 along with additional information as of December 31 are as follows:

Bad Debt Expense                                                             

$ 32,000

Delivery Expense                                                             

425,000

Depreciation Expense—Delivery Trucks                                             

29,000

Depreciation Expense—Office Building                                              

25,000

Depreciation Expense—Office Equipment                                            

10,000

Depreciation Expense—Store Equipment                                             

25,000

Dividend Revenue                                                             

35,000

Dividends                                                                   

150,000

Employee Pension Expense                                                      

190,000

Freight In                                                                   

145,000

Gain on Sale of Office Equipment                                                  

8,000

Income Taxes, 2008                                                            

427,425

Interest Revenue                                                             

10,000

Inventory, January 1, 2008                                                        

775,000

Loss on Sale of Investment Securities                                               

20,000

Loss on Write Down of Obsolete Inventory                                         

75,000

Miscellaneous General Expenses                                                  

45,000

Miscellaneous Selling Expenses                                                    

50,000

Officers’ and Office Salaries                                                       

550,000

Property Taxes Expense                                                        

100,000

Purchase Discounts                                                            

47,700

Purchases                                                                   

4,633,200

Retained Earnings, January 1, 2008                                                 

550,000

Sales                                                                       

8,125,000

Sales Discounts                                                              

55,000

Sales Returns and Allowances                                                    

95,000

Sales Salaries                                                                 

521,000

(a) Inventory was valued at year end as follows:

Cost                   

$825,000

Write down of obsolete inventory

75,000

 

$750,000

(b) Number of Connell shares of stock outstanding: 60,000

Instructions:

Prepare a multiple step income statement and statement of retained earnings for the year ended December 31, 2008.

corrected income statement a newly hired staff accountant prepared the pre audit inc 691861

Corrected Income Statement

A newly hired staff accountant prepared the pre audit income statement of Jericho Recreation Incorporated for the year ending December 31, 2008.

Net revenues                    

 

$797,000

Cost of goods sold                

 

300,800

Gross profit                     

 

$496,200

Expenses:

 

 

Sales salaries and commissions     

$160,000

 

Officers’ and office salaries        

210,000

 

Depreciation                  

56,000

 

Advertising expense             

13,400

 

Other general and administrative expenses

38,80

 

 

 

478,200

Income from continuing operations             

 

$18,000

Discontinued operations:

 

 

Gain on disposal of business segment         

 

40,000

Income before income taxes               

 

$58,000

Income taxes (30%)                      

 

17,400

Net income                           

 

$40,600

Earnings per common share (10,000 shares outstanding)

 

$406.00

The following information was obtained by Jericho’s independent auditor.

(a) Net revenues in the income statement included the following items.

Sales returns and allowances                                                

$ 9,500

Interest revenue                                                         

6,600

Interest expense                                                         

10,600

Loss on sale of short term investment                                         

3,000

Extraordinary gain                                                        

16,000

(b) Of the total depreciation expense reported in the income statement, 60% relates to stores and store equipment, 40% to office building and equipment.

(c) At the beginning of 2008, management decided to close one of Jericho’s retail stores. Jericho is a large company and does not attempt to prepare complete financial reports for each individual store. The inventory and equipment were moved to another Jericho store, and the land and building were sold on July 1, 2008, at a pretax gain of $40,000.This amount has been reported under discontinued operations.

(d) The income tax rate is 30%.

Instructions:

Prepare a corrected multiple step income statement for the year ended December 31, 2008.

analysis of income items mdash multiple step income statement preparation on decembe 691862

Analysis of Income Items—Multiple Step Income Statement Preparation

On December 31, 2008, analysis of Sayer Sporting Goods’ operations for 2008 revealed the following.

(a) Total cash collections from customers, $105,260.

(b) December 31, 2007, inventory balance, $12,180.

(c) Total cash payments, $92,450.

(d) Accounts receivable, December 31, 2007, $22,150.

(e) Accounts payable, December 31, 2007, $10,830.

(f) Accounts receivable, December 31, 2008, $18,920.

(g) Accounts payable, December 31, 2008, $7,120.

(h) General and administrative expenses total 25% of sales. This amount includes the depreciation on store and equipment.

(i) Selling expenses of $12,352 total 20% of gross profit.

(j) No general and administrative or selling expense liabilities existed at December 31,

2008.

(k) Wages and salaries payable at December 31, 2007, $4,450.

(l) Depreciation expense on store and equipment total 12% of general and administrative expenses.

(m) Shares of stock issued and outstanding, 5,000.

(n) The income tax rate is 40%.

Instructions:

Prepare a multiple step income statement for the year ended December 31, 2008.

comprehensive income statement the following information for the year ending decembe 691864

Comprehensive Income Statement

The following information for the year ending December 31, 2008, has been provided for Rexburg Company.

Sales                                                                  

$470,000

Cost of goods sold                                                       

287,000

Foreign translation adjustment (net of income taxes)                               

43,000 (Cr)

Selling expenses                                                          

72,100

Extraordinary gain (net of income taxes)                                        

41,200

Correction of inventory error (net of income taxes)                                

29,720 (Cr)

General and administrative expenses                                           

61,240

Income tax expense                                                       

18,500

Gain on sale of investment                                                  

7,300

Proceeds from sale of land at cost                                            

78,000

Dividends                                                              

10,900

Instructions:

Prepare a statement of comprehensive income for Rexburg Company.

forecasted balance sheet and income statement lorien company wishes to prepare a for 691865

Forecasted Balance Sheet and Income Statement

Lorien Company wishes to prepare a forecasted income statement and a forecasted balance sheet for 2009. Lorien’s balance sheet and income statement for 2008 follow.

Balance Sheet

2008

Cash                                                                      

$ 40

Other current assets                                                          

350

Property, plant, and equipment, net                                                

1,000

Total assets                                                                 

$1,390

Accounts payable                                                              

$ 100

Bank loans payable                                                            

1,000

Paid in capital                                                                

100

Retained earnings                                                              

190

Total liabilities and stockholders’ equity                                             

$1,390

Income Statement

2008

Sales                                                                      

$1,000

Cost of goods sold                                                            

350

Gross profit                                                                 

$ 650

Depreciation expense                                                          

200

Other operating expenses                                                       

250

Operating profit                                                              

$ 200

Interest expense                                                              

120

Income before taxes                                                           

$ 80

Income taxes                                                                

20

Net income                                                                 

$ 60

In addition, Lorien has assembled the following forecasted information regarding 2009:

(a) Sales are expected to increase to $1,200.

(b) Lorien does not expect to buy any new property, plant, and equipment during 2009.

(c) Because of adverse banking conditions, Lorien does not expect to receive any new bank loans in 2009.

(d) Lorien plans to pay cash dividends of $15 in 2009.

Instructions:

1. Prepare a forecasted balance sheet and a forecasted income statement for 2009. Clearly state what assumptions you make.

2. If you construct your forecasted balance sheet in (1) correctly, total forecasted paid in capital for 2009 should be negative. Is this possible? Explain.

sample cpa exam question during january 2008 doe corp agreed to sell the assets and 691866

Sample CPA Exam Question

During January 2008, Doe Corp. agreed to sell the assets and product line of its Hart division. The sale was completed on January 15, 2009; on that date, Doe recognized a gain on disposal of $900,000. Hart’s operating losses were $600,000 for 2008 and $50,000 for the period January 1 through January 15, 2009.The income tax rate is 40%.What amount of net gain (loss) from discontinued operations should be reported in Doe’s comparative 2009 and 2008 income statements?

 

2009

2008

a.

$ 0

$ 150,000

b.

150,000

0

c.

510,000

(360,000)

d.

540,000

(390,000)

how can my company have income but no cash max stevenson owns a local dr ug store du 691868

How Can My Company Have Income but No Cash?

Max Stevenson owns a local Dr.ug store. During the past few years, the economy has experienced a period of high inflation. Stevenson has had the policy of with Dr. awing cash from his business equal to 80% of the company’s reported net income. As the business has grown, he has had a CPA prepare the company’s financial statements and tax returns. The following is a summary of the company’s income statement for the current year:

Revenue                 

$565,000

Cost of goods sold (Dr. ugs, etc)

395,000

Gross profit on items sold   

$170,000

Operating expenses (including taxes)

110,000

Net income              

$60,000

Even though the business has reported net income each year, it has experienced severe cash flow shortages. The company has had to pay higher prices for its inventory as the company has tried to maintain the same quantity and quality of its goods. For example, last year’s cost of goods sold had a historical cost of $250,000 and a replacement cost of $295,000. The current year’s cost of goods sold has a replacement cost of $440,000. Stevenson’s personal cash outflows have also grown faster than his with Dr. awals from the company due to increasing personal demands. Stevenson asks you as a financial advisor how the company can have income of $60,000 yet he and the company still have a shortage of cash.

gains and losses on extraordinary items use the following information to compute inc 691826

Gains and Losses on Extraordinary Items

Use the following information to compute income from continuing operations and net income. Assume that the income tax rate on all items is 40%.

Cost of goods sold

$ 11,000

Interest expense

2,100

Loss from an unusual but frequent event

(1,000)

Selling and administrative expense

1,750

Loss from an unusual and infrequent event

(400)

Sales

20,000

Gain from a normal but infrequent event

1,250

Dividends

700

cumulative effect of a change in accounting principle the company started business i 691827

Cumulative Effect of a Change in Accounting Principle

The company started business in 2006. In 2008, the company decided to change its method of computing oil and gas exploration expense. The company has only two expenses: oil and gas exploration expense and income tax expense. The following sales and oil and gas exploration expense information are for 2006–2008:

 

2008

2007

2006

Sales

$5,000

$3,000

$2,000

Oil and gas exploration expense—old method

1,000

600

400

Oil and gas exploration expense—new method

700

1,200

1,500

Prepare the 2008 comparative income statement. The income tax rate for all items is 30%.

price earnings p e ratio refer to practice 4 16 use that information to compute the 691831

Price Earnings (P/E) Ratio

Refer to Practice 4 16. Use that information to compute the price earnings ratio.

Practice 4 16

Return on Sales

Use the following information to compute return on sales.

Earnings per share

$1.67

Cost of goods sold 

$10,000

Cash

$550

Sales

$13,000

Market price per share

$20

Net income

$200

Total stockholders’ equity

$6,700

forecasted balance sheet the following balance sheet asset information is for 2008 c 691833

Forecasted Balance Sheet

The following balance sheet asset information is for 2008:

Cash

$ 100

Accounts receivable

500

Inventory

1,000

Land

 2,500

Plant and equipment (net)

5,000

Sales are expected to increase by 25% in 2009. No new land will be needed to support this increased level of sales. This sales increase will require significantly expanded production capacity; net plant and equipment will increase by 40%. Prepare a forecast of the Assets section of the 2009 balance sheet.

forecasted income statement the following balance sheet information represents actua 691834

Forecasted Income Statement

The following balance sheet information represents actual data for 2008 and forecasted data for 2009:

 

Actual 2008

Forecasted 2009

Current assets                                         

$2,000

$2,600

Property, plant, and equipment (net)                         

5,000

6,000

Accounts payable                                       

500

650

Long term debt                                        

4,000

5,000

Total stockholders’ equity                                  

2,500

2,950

The actual income statement for 2008 is as follows:

Sales

$10,000

Cost of goods sold

6,000

Depreciation expense

1,000

Interest expense

400

Income before income taxes

$ 2,600

Income tax expense

910

Net income

$ 1,690

Sales are expected to increase by 30% in 2009. Prepare a forecasted income statement for 2009.

calculation of net income changes in the balance sheet account balances for the beec 691835

Calculation of Net Income

Changes in the balance sheet account balances for the Beecher Sales Co. during 2008 follow. Dividends declared during 2008 were $20,000. Calculate the net income for the year assuming that no transactions other than the dividends affected retained earnings.

Increase

(Decrease)

Cash

$ 65,200

Accounts Receivable

82,000

Inventory

(25,000)

Buildings and Equipment (net)

170,000

Patents

(5,000)

Accounts Payable

(85,000)

Bonds Payable

120,000

Capital Stock

50,000

Additional Paid In Capital

50,000

revenue recognition for each of the following transactions events or circumstances i 691836

Revenue Recognition

For each of the following transactions, events, or circumstances, indicate whether the recognition criteria for revenues and gains are met and provide support for your answer.

(a) An order of $25,000 for merchandise is received from a customer.

(b) The value of timberlands increases by $40,000 for the year due to normal growth.

(c) Accounting services are rendered to a client on account.

(d) A 1991 investment was made in land at a cost of $80,000. The land currently has a fair market value of $107,000.

(e) Cash of $5,600 is collected from the sale of a gift certificate that is redeemable in the next accounting period.

(f) Cash of $7,500 is collected from subscribers for subscription fees to a monthly magazine.

The subscription period is 2 years.

(g) You owe a creditor $1,500, payable in 30 days. The creditor has cash flow difficulties and has agreed to allow you to retire the debt in full with an immediate payment of $1,200.

revenue recognition indicate which of the following transactions or events gives ris 691837

Revenue Recognition

Indicate which of the following transactions or events gives rise to the recognition of revenue in 2008 under the accrual basis of accounting. If revenue is not recognized, what account, if any, is credited?

(a) On December 15, 2008, Howe Company received $20,000 as rent revenue for the 6 month period beginning January 1, 2009.

(b) Monroe Tractor Co., on July 1, 2008, sold one of its tractors and received $10,000 in cash and a note for $50,000 at 12% interest, payable in one year. The fair market value of the tractor is $60,000.

(c) Oswald, Inc., issued additional shares of common stock on December 10, 2008, for $30,000 above par value.

(d) Balance Company received a purchase order in 2008 from an established customer for $10,200 of merchandise. The merchandise was shipped on December 20, 2008. The company’s credit policy allows the customer to return the merchandise within 30 days and a 3% discount if paid within 20 days from shipment.

(e) Gloria, Inc., sold merchandise costing $2,000 for $2,500 in August 2008.The terms of the sale are 15% down on a 12 month conditional sales contract, with title to the goods being retained by the seller until the contract price is paid in full.

(f) On November 1, 2008, Jones & Whitlock entered into an agreement to audit the 2008 financial statements of Lehi Mills for a fee of $35,000. The audit work began on December 15, 2008, and will be completed around February 15, 2009.

expense recognition for each of the following items indicate whether the expense sho 691838

Expense Recognition

For each of the following items, indicate whether the expense should be recognized using

(1) direct matching, (2) systematic and rational allocation, or (3) immediate recognition.

Provide support for your answer.

(a) Johnson & Smith, Inc., conducts cancer research. The company’s hope is to develop a cure for the deadly disease. To date, its efforts have proven unsuccessful. It is testing a new Dr.ug, Ebzinene, which has cost $400,000 to develop.

(b) Sears, Roebuck and Co. warranties many of the products it sells. Although the warranty periods range from days to years, Sears can reasonably estimate warranty costs.

(c) Stocks Co. recently signed a 2 year lease agreement on a warehouse. The entire cost of $15,000 was paid in advance.

(d) John Clark assembles chairs for the Stone Furniture Company. The company pays Clark on an hourly basis.

(e) Hardy Co. recently purchased a fleet of new delivery trucks. The trucks are each expected to last for 100,000 miles.

(f) Taylor Manufacturing Inc. regularly advertises in national trade journals. The objective is to acquire name recognition, not to promote a specific product.

physical capital maintenance refer to practice 4 1 assets with the same productive c 691817

Physical Capital Maintenance

Refer to Practice 4 1. Assets with the same productive capacity as the assets comprising the $400,000 beginning asset balance had a current cost of $465,000 at the end of the year. Using the physical capital maintenance concept, determine the company’s income for the year.

Practice 4 1

The company had the following total asset and total liability balances at the beginning and the end of the year:

 

Beginning

Ending

Total assets

$400,000

$625,000

Total liabilities

230,000

280,000

During the year, the company received $100,000 in new investment funds contributed by the owners. Using the financial capital maintenance concept, determine the company’s income for the year.

computation of income using matching the company sells custom designed engineering e 691818

Computation of Income Using Matching

The company sells custom designed engineering equipment. During the most recent year, the company received the following customer orders:

For Machine A, selling price = $150,000, production cost = $79,000

For Machine B, selling price = $270,000, production cost = $163,000

For Machine C, selling price = $91,000, production cost = $46,000

For Machine D, selling price = $400,000, production cost = $231,000

Machines A and C were completed and shipped during the year; the total revenue from the sale of these machines will be reported in the income statement for the year. Machines B and D have not yet been completed; the total production cost incurred so far for these two machines is $350,000. The revenue from the sale of these two machines will not be reported in the income statement for the year. Using the transaction approach (the matching method), compute the company’s income for the year.

revenue recognition the following information describes the company rsquo s sales fo 691819

Revenue Recognition

The following information describes the company’s sales for the year:

(a) A sale for $100,000 was made on March 23. As of the end of the year, all work associated with the sale has been completed. Unfortunately, the customer is a significant credit risk and the collection of the cash for the sale is very uncertain. No cash has been collected as of the end of the year.

(b) A sale for $130,000 was made on July 12. The $130,000 cash for the sale was collected in full on July 12. The work associated with the sale has not yet begun but is expected to be completed early next year.

(c) A sale for $170,000 was made on November 17. No cash has been collected as of the end of the year, but all of the cash is expected to be collected early next year. As of the end of the year, all of the work associated with the sale has been completed. How much revenue should be recognized for the year?

expense recognition the following information describes the company rsquo s costs in 691820

Expense Recognition

The following information describes the company’s costs incurred during the year:

               

Amount of

Expense

Length of

Matched

 

Amount of

Recognition

Allocation

Revenue

 

Cost

Method

Period

Recognized?

(a)

$30,000

Direct matching

Not applicable

Yes

(b)

70,000

Immediate recognition

Not applicable

Not applicable

(c)

15,000

Rational allocation

3 years

Not applicable

(d)

27,000

Immediate recognition

Not applicable

Not applicable

(e)

45,000

Rational allocation

5 years

Not applicable

(f)

50,000

Direct matching

Not applicable

No

How much expense should be recognized for the year?

multiple step income statement refer to the information in practice 4 6 use that inf 691822

Multiple Step Income Statement

Refer to the information in Practice 4 6. Use that information to prepare a multiple step income statement.

Practice 4 6

Single Step Income Statement

Using the following information, prepare a single step income statement.

Cost of goods sold

$ 6,000

Interest expense

1,100

Selling and administrative expense

750

Cash

400

Sales

10,000

Accrued wages payable

250

Dividends

700

Retained earnings (beginning)

1,000

Income tax expense

1,200

computation of income from continuing operations use the following information to co 691823

Computation of Income from Continuing Operations

Use the following information to compute income from continuing operations. Assume that the income tax rate on all items is 40%.

Cost of goods sold

$ 4,000

Interest expense

1,100

Income (loss) from discontinued operations

(1,000)

Selling and administrative expense

1,750

Extraordinary loss

(400)

Sales

10,000

Dividends

700

Loss on sale of discontinued operations

(200)

computation of income from discontinued operations fleming company has two divisions 691824

Computation of Income from Discontinued Operations

Fleming Company has two divisions, E and N. Both qualify as business components. In 2008, the firm decides to dispose of the assets and liabilities of Division N; it is probable that the disposal will be completed early next year. The revenues and expenses of Fleming for 2007 and 2008 are as follows:

 

2008

2007

Sales—E

$5,000

$4,600

Total nontax expenses—E

4,400

4,100

Sales—N

3,500

5,100

Total nontax expenses—N

3,900

4,500

During the later part of 2008, Fleming disposed of a portion of Division N and recognized a pretax loss of $2,000 on the disposal. The income tax rate for Fleming Company is 30%. Prepare the 2008 comparative income statement.

computation of income from discontinued operations refer to the data in practice 4 1 691825

Computation of Income from Discontinued Operations

Refer to the data in Practice 4 11. Repeat the exercise, assuming that Division E is being discontinued. Also assume that instead of a $2,000 pretax loss on the disposal, there was a $1,500 pretax gain.

Practice 4 11

Computation of Income from Discontinued Operations

Fleming Company has two divisions, E and N. Both qualify as business components. In 2008, the firm decides to dispose of the assets and liabilities of Division N; it is probable that the disposal will be completed early next year. The revenues and expenses of Fleming for 2007 and 2008 are as follows:

 

2008

2007

Sales—E

$5,000

$4,600

Total nontax expenses—E

4,400

4,100

Sales—N

3,500

5,100

Total nontax expenses—N

3,900

4,500

During the later part of 2008, Fleming disposed of a portion of Division N and recognized a pretax loss of $2,000 on the disposal. The income tax rate for Fleming Company is 30%. Prepare the 2008 comparative income statement.

corrected balance sheet the following balance sheet is submitted to you for inspecti 691791

Corrected Balance Sheet

The following balance sheet is submitted to you for inspection and review.

Appalachian Freight Company

Balance Sheet

December 31, 2008

Assets

 

Cash

$ 45,050

Accounts receivable

112,500

Inventories

204,000

Prepaid insurance

8,800

Property, plant, and equipment

376,800

Total assets

$747,150

Liabilities and Owners’ Equity

 

Miscellaneous liabilities

$ 3,600

Loan payable

76,200

Accounts payable

75,250

Capital stock

134,000

Paid in capital

458,100

Total liabilities and owners’ equity

$747,150

In the course of the review, you find the following data:

(a) The possibility of uncollectible accounts on accounts receivable has not been considered.

It is estimated that uncollectible accounts will total $4,800.

(b) The amount of $45,000 representing the cost of a large scale newspaper advertising campaign completed in 2008 has been added to the inventories because it is believed that this campaign will benefit sales of 2009. It is also found that inventories include merchandise of $16,250 received on December 31 that has not yet been recorded as a purchase.

(c) The books show that property, plant, and equipment have a cost of $556,800 with depreciation of $180,000 recognized in prior years. However, these balances include fully depreciated equipment of $85,000 that has been scrapped and is no longer on hand.

(d) Miscellaneous liabilities of $3,600 represent salaries payable of $9,500, less noncurrent advances of $5,900 made to company officials.

(e) Loan payable represents a loan from the bank that is payable in regular quarterly installments of $6,250.

(f) Tax liabilities not shown are estimated at $18,250.

(g) Deferred income tax liability arising from temporary differences totals $44,550.This liability was not included in the balance sheet.

(h) Capital stock consists of 6,250 shares of preferred 6% stock, par $20, and 9,000 shares of common stock, stated value $1.

(i) Capital stock had been issued for a total consideration of $283,600; the amount received in excess of the par and stated values of the stock has been reported as paid in capital. Net income and dividends were recorded in Paid In Capital.

Instructions: Prepare a corrected balance sheet with accounts properly classified.

december 31 2008 current assets 53 415 less current liabilities 29000 working capita 691792

Corrected Balance Sheet

The accountant for Delicious Bakery prepares the following condensed balance sheet.

Delicious Bakery
Condensed Balance Sheet
December 31, 2008

Current assets

$53,415

Less: Current liabilities

29000

Working capital

$24,415

Add: Other assets

75120

 

$99,535

Less: Other liabilities

3600

Investment in business

$95,935

A review of the account balances disclosed the following data.

(a) An analysis of the current asset grouping revealed the following:

Cash

$10,600

Accounts receivable (fully collectible)

12,500

Notes receivable (notes of customer who has been declared bankrupt and

 

is unable to pay anything on the obligations)

1,000

Investment securities—trading, at cost (market value $2,575)

4,250

Inventory

20,965

Cash surrender value of insurance on officers’ lives

4100

Total current assets

$53,415

The inventory account was found to include supplies costing $425, a delivery truck acquired at the end of 2008 at a cost of $2,100, and fixtures at a depreciated value of $10,400.The fixtures had been acquired in 2002 at a cost of $12,500.

(b) The total for other assets was determined as follows.

Land and buildings at cost of acquisition, July 1, 2006

$92,000

Less balance due on mortgage, $16,000, and accrued interest on mortgage,

$880

(mortgage is payable in annual installments of $4,000 on July 1 of each year

 

together with interest for the year at that time at 11%)

16880

Total other assets

$75,120

It was estimated that the land at the time of the purchase was worth $30,000. Buildings as of December 31, 2008,were estimated to have a remaining life of 171⁄2 years.

(c) Current liabilities represented balances that were payable to trade creditors.

(d) Other liabilities consisted of withholding, payroll, real estate, and other taxes payable to the federal, state, and local governments. However, no recognition was given the accrued salaries, utilities, and other miscellaneous items totaling $350.

(e) The company was originally organized in 2001 when 5,000 shares of no par stock with a stated value of $5 per share were issued in exchange for business assets that were recognized on the books at their fair market value of $55,000.

Instructions: Prepare a corrected balance sheet with the items properly classified.

classified balance sheet lane peterson incorporated his concrete manufacturing opera 691793

Classified Balance Sheet

Lane Peterson incorporated his concrete manufacturing operations on January 1, 2008, by issuing 10,000 shares of $1 par common stock to himself. The following balance sheet for the new corporation was prepared.

Outrigger Corporation

Balance Sheet

January 1, 2008

Cash

$ 10,000

Accounts receivable

75,000

Inventory

85,000

Equipment

125,000

Total

 $295,000

Accounts payable—suppliers

$ 45,000

Capital stock, $1 par

10,000

Additional paid in capital

240,000

Total

$295,000

During 2008, Outrigger Corporation engaged in the following transactions.

(a) Outrigger Corporation produced concrete costing $320,000.Concrete costs consisted of the following: $220,000, raw materials purchased; $45,000, labor; and $55,000, overhead. Outrigger Corporation paid the $45,000 owed to suppliers as of January 1 and $150,000 of the $220,000 of raw materials purchased during the year. All labor, except for $4,500, and recorded overhead were paid in cash during the year. Other operating expenses of $18,000 were incurred and paid in 2008.

(b) Concrete costing $280,000 was sold during 2008 for $360,000. All sales were made on credit, and collections on receivables were $325,000.

(c) Outrigger Corporation purchased machinery (fair market value = $210,000) by trading in old equipment costing $80,000 and paying $130,000 in cash. There is no accumulated depreciation on the old equipment as it was revalued when the new corporation was formed.

(d) Outrigger Corporation issued an additional 5,000 shares of common stock for $25 per share and declared a dividend of $2.50 per share to all stockholders of record as of December 31, 2008, payable on January 15, 2009.

(e) Depreciation expense for 2008 was $32,000. The allowance for bad debts after yearend adjustments is $1,500.

Instructions: Prepare a properly classified balance sheet in account form for Outrigger Corporation as of December 31, 2008.

sample cpa exam questions 1 which of the following is the true purpose of informatio 691794

Sample CPA Exam Questions

1. Which of the following is the true purpose of information presented in notes to the financial statements?

(a) To provide disclosures required by generally accepted accounting principles.

(b) To correct improper presentation in the financial statements.

(c) To provide recognition of amounts not included in the totals of the financial statements.

(d) To present management’s responses to auditor comments.

2. Which of the following information should be included in Melay, Inc.’s 2008 summary of significant accounting policies?

(a) Property, plant, and equipment is recorded at cost with depreciation computed principally by the straight line method.

(b) During 2008, the Delay Segment was sold.

(c) Business segment 2008 sales are Alay $1M, Belay $2M, and Celay $3M.

(d) Future common share dividends are expected to approximate 60% of earnings.

the ten largest companies in the united states forbes annually provides a list of th 691795

The Ten Largest Companies in the United States

Forbes annually provides a list of the most valuable companies in the world. The top 10 most valuable companies in the United States, from the 2005 Forbes 2000, follow.

(In billions of U.S. dollars)

 

Market

Total

Net

Company Name Industry

Value

Assets

Income

ExxonMobil

Oil & gas operations

$405.25

$ 195.26

$25.33

General Electric

Conglomerates

372.14

750.33

16.59

Microsoft

Software & services

273.75

64.94

10.00

Citigroup

Banking

247.66

1,484.10

17.05

Wal Mart

Retailing

218.56

120.62

10.27

Pfizer

Dr. ugs & biotechnology

197.99

123.68

11.36

Johnson & Johnson

Dr. ugs & biotechnology

194.68

47.59

8.51

Bank of America

Banking

188.77

1,110.46

14.14

American International

 

 

 

 

Group

Insurance

173.99

776.42

10.91

IBM

Technology hardware & equipment

152.76

109.18

8.43

As an analyst for a securities broker, you are asked the following questions concerning some of the figures.

1. Microsoft has total assets of $65 billion but a stock market value of $274 billion. How can a company be worth more than its total assets?

2. Compute for each company the ratio of total assets to total market value. What factors must the market be considering in valuing these companies that are not captured on the companies’ balance sheets?

3. The price earnings ratio, often called the P/E ratio, is defined as market price per share divided by earnings per share. Alternatively, the P/E ratio can be computed as total market value divided by net income. Compute the P/E ratios for the preceding companies. What factors do you think influence P/E ratios?

which company is which following are summaries of the balance sheets of five compani 691799

Which Company Is Which?

Following are summaries of the balance sheets of five companies. The amounts are all stated as a percentage of total assets. The five companies are

• BankAmerica, a large bank

• Kelly Services, a firm that provides temporary employees

• Yahoo!, an Internet company

• McDonald’s, a fast food company

• Consolidated Edison, a utility serving New York City

 

A

B

C

D

E

Receivables

59.1

4.0

4.0

62.9

3.1

Inventories

0.0

1.9

0.0

0.0

0.4

Other current assets

20.6

1.8

71.1

23.8

3.2

Land, buildings, and equipment

11.7

79.3

2.4

1.5

81.1

Other long term assets

8.6

13.1

22.4

11.8

12.3

Short term payables

42.1

6.6

12.9

26.8

8.5

Other current liabilities

0.0

1.3

0.0

57.8

4.1

Long term liabilities

0.0

48.4

0.9

8.0

39.5

Equity

57.9

43.6

86.2

7.4

47.8

Match each balance sheet summary (A–E) with the appropriate company. Justify your choices.

how can we live with debt covenant requirements bohr company has a credit agreement 691800

How Can We Live with Debt Covenant Requirements?

Bohr Company has a credit agreement with a syndicate of banks. In order to impose some limitations on Bohr’s financial riskiness, the credit agreement requires Bohr to maintain a current ratio of at least 1.4 and a debt ratio of 0.55 or less. The following summary data reflect a projection of Bohr’s balance sheet for the coming year end.

Current assets

 $1,200,000

Long term assets

1,800,000

Current liabilities

900,000

Long term liabilities

800,000

Equity

1,300,000

The following information has also been prepared.

(a) If Bohr were to use FIFO instead of LIFO for inventory valuation, ending inventory would increase by $50,000.

(b) The amounts listed for long term assets and liabilities include the anticipated purchase (and associated mortgage payable) of a building costing $100,000, or Bohr can lease the building instead. The lease would qualify for treatment as an operating lease.

(c) Projected amounts include a planned declaration of cash dividends totaling $40,000 to be paid next year. Bohr has consistently paid dividends of equivalent amounts. As a consultant to Bohr, you are asked to respond to the following two questions.

1. What steps can Bohr take to avoid violating the current ratio constraint?

2. What steps can Bohr take to avoid violating the debt ratio constraint? Of the steps that you propose, which ones do you think the banks had in mind when they imposed the loan covenants? If you had assisted the banks in Dr. awing up the loan covenants, how would you have written them differently to avoid unintended consequences?

what should we tell the stockholders technology unlimited inc uses a fiscal year end 691802

What Should We Tell the Stockholders?

Technology Unlimited, Inc., uses a fiscal year ending June 30. The auditors completed their review of the 2008 financial statements on September 8, 2008. They discovered the following subsequent events between June 30 and September 8.

(a) Technology split its common stock 2 for 1 on August 15. Prior to the split, Technology had outstanding 100,000 shares of $1 par common stock.

(b) A major customer, Diatride Company, declared bankruptcy on August 1.The customer owed Technology $75,000 on June 30.No payment had been received as of September

8. It is estimated that creditors will receive only 15% of outstanding claims.

(c) Technology completed negotiations to purchase Liston Development Labs on July 18.

The purchase price was $525,000 in cash and a 4 year, $250,000, 10% note.

(d) A $750,000 lawsuit against Technology was filed on August 15. It is too early to measure the loss potential.

(e) A general decline in stock market values for technology stocks occurred during the first week of September. Technology Unlimited’s market value per share Dr. opped from $42.50 to $28.00 in this week.

The auditors have requested that you prepare the subsequent event note that should accompany the financial statements for the year ending June 30, 2008. Only those events that require disclosure should be included in your note. Justify the exclusion of any events from your note.

what does this british balance sheet mean jonathan atwood a student from england sho 691803

What Does this British Balance Sheet Mean?

Jonathan Atwood, a student from England, shows you the following balance sheet from his father’s British company. Jonathan knows that you are studying accounting and asks you to look at the statement. You immediately recognize some differences between this statement and the ones you have been studying in your textbook.

 

 

Group

31 December

Company

31 December

 

 

2008

2007

2008

2007

 

NOTES

£m

 £m

£m

 £m

Fixed Assets

 

_____

_____

_____

_____

Intangibles

13

304.0

307.4

 

 

Tangible assets

14

978.8

822.5

16.8

16.2

Investments

15

16.7

25.2

938.9

679.3

 

 

1,299.5

1,155.1

955.7

695.5

Current Assets

 

 

 

 

 

Stock

16

328.2

334.8

 

 

Debtors

17

554.1

548.2

113.4

210.8

Investments short term loans

 

 

 

 

 

and deposits

 

118.0

33.3

5.1

23.5

Cash at bank and in hand

 

62.6

57.4

 

 

 

 

1,062.9 

973.7 

118.5 

234.3 

Creditors: amounts falling due within one year

 

 

 

 

 

Borrowings

18

(136.3)

(133.7)

(175.0)

(74.6)

Other

19

(825.9) 

(809.2) 

(98.4) 

(234.7) 

Net current assets (liabilities)

 

100.7 

30.8 

(154.9) 

(75.0) 

Total assets less current liabilities

 

1,400.2 

1,185.9 

800.8 

620.5 

Other Liabilities

 

 

 

 

 

Creditors: amounts falling due after more than one year

 

 

 

 

 

Borrowings

18

(407.9)

(381.4)

(54.2)

(80.4)

Other

19

(12.0)

(8.5)

(26.4)

(42.1)

Provisions for liabilities and charges 

20

(96.4) 

(115.5) 

0.5 

1.2 

 

 

(516.3) 

(505.4) 

(80.1) 

(121.3) 

 

 

883.9

680.5

720.7

499.2

Capital and Reserves

 

 

 

 

 

Called up share capital

21

174.7

173.6

174.7

173.6

Share premium account

22

381.6

217.4

381.6

217.4

Revaluation reserve

22

95.8

36.7

2.4

1.1

Profit and loss account

22

115.8

167.6

162.0

107.1

 

 

767.9

595.3

720.7

499.2

Minority interests

 

116.0 

85.2 

 

 

 

 

883.9

680.5

720.7

499.2

1. Identify the differences that exist between this British statement and those prepared using the standards and conventions of the United States.

2. Evaluate the differences, identifying strengths and weaknesses of each nation’s approach.

are banks backward the following is an excerpt from an article dealing with accounti 691804

Are Banks Backward?

The following is an excerpt from an article dealing with accounting and banks in The Wall

Street Journal, “GAO Says Accountants Auditing Thrifts Are Hiding Behind Outdated

Standards” (February 6, 1989) p.  C21. Congress deregulated the left side of the balance sheet [liabilities] by permitting thrifts to get into high risk business but kept regulation and deposit insurance for the right side of the balance sheet [assets].

1. From an accounting perspective, what is wrong with this quote?

2. As a bank depositor, do you care about the balance sheet of the bank where you deposit your money? Why or why not? How might your attitude change if the U.S. federal government were to abolish deposit insurance?

3. Consider your account at a bank—does the bank view your account as an asset or as a liability?

why is our book to market ratio so high aiga company is a leading manufacturer of ho 691805

Why Is Our Book to Market Ratio So High?

Aiga Company is a leading manufacturer of household plumbing materials. Aiga does notmake the high profile faucets and fixtures; instead, it makes the pipes and other connectionsthat are usually out of sight under kitchen and bathroom sinks. You are AigaCompany’s chief financial officer. You are scheduled to meet with an irate group of stockholders.

These stockholders read a recent business press article that explained that the average book to market ratio for the 10 most valuable companies in the United States is below 0.10. The article then claimed that companies with book to market ratios above 0.50 are probably run by mediocre managers who are unable to inspire market confidence in their companies. Aiga has a book to market ratio of 0.65. What will you say to the irate stockholders?

what do we want off our balance sheet kuanysh company is considering purchasing a la 691806

What Do We Want Off Our Balance Sheet?

Kuanysh Company is considering purchasing a large retail location. The retail site includes a large parking lot, loading dock facilities, and a warehouse sized store suitable for sale of both general merchandise and groceries. The retail site is in a prime location and costs $15 million. Kuanysh has arranged to borrow the entire $15 million purchase price from a local bank. When the transaction is completed, Kuanysh will have total reported assets of $65 million and total reported liabilities of $40 million. Kuanysh has been approached by a real estate company that has offered to buy the property and then lease it to Kuanysh under a long term, noncancelable lease contract. If the lease contract is carefully designed, neither the $15 million real estate asset nor the $15 million loan obligation will appear on Kuanysh’s balance sheet. Why might Kuanysh want to enter into this lease contract rather than simply borrowing the money and buying the location itself?

deciphering financial statements the walt disney company locate the 2004 financial s 691807

Deciphering Financial Statements (The Walt Disney Company)

Locate the 2004 financial statements for The Walt Disney Company on the Internet.

1. Compute a current ratio for Disney as of September 30, 2004. How does this current ratio compare with the prior year’s current ratio?

2. Compute Disney’s asset turnover for 2004.Was the company more or less efficient in

2004 compared to 2003?

3. What method of inventory valuation does Disney use?

4. What method of depreciation does Disney use?

5. What material commitments and contingencies does Disney report in the notes to its 2004 financial statements?

6. What percentage of Disney’s 2004 operating income was generated in the “United States and Canada” geographic segment?

deciphering financial statements boston celtics with all due respect to michael jord 691808

Deciphering Financial Statements (Boston Celtics)

With all due respect to Michael Jordan and the Chicago Bulls, the Boston Celtics are the most successful team in professional basketball history. Teams led by Bill Russell, Dave Cowens, John Havlicek, and Larry Bird have won a total of 16 NBA championships. The Celtics are also an unusual professional sports team because ownership shares in the Celtics were at one time publicly traded (on the New York Stock Exchange as “Boston Celtics Limited Partnership”). As such, the Celtics were required to file financial statements with the SEC each quarter. The June 30, 2001, balance sheet of “Celtics Basketball Holdings” follows.

BOSTON CELTICS LIMITED PARTNERSHIP

and Subsidiaries

Consolidated Balance Sheets

 

June 30,

June 30,

 

2001

2000

ASSETS

 

 

CURRENT ASSETS

 

 

Cash and cash equivalents                                    

$12,572,324

$14,941,632

Accounts receivable                                         

3,250,212

5,799,898

Prepaid expenses and other current assets                        

601,184

636,551

TOTAL CURRENT ASSETS                                      

16,423,720

21,378,081

PROPERTY AND EQUIPMENT, net                                

1,200,556

1,144,785

NATIONAL BASKETBALL ASSOCIATION FRANCHISE, net of

 

 

amortization of $2,776,318 in 2001 and $2,622,078 in 2000            

3,393,263

3,547,503

INVESTMENT IN NBA MEDIA VENTURES, LLC                      

5,018,420

4,263,420

OTHER ASSETS                                              

125,060

776,815

 

$26,161,019

$31,110,604

LIABILITIES AND PARTNERS’ CAPITAL (DEFICIT)

 

 

CURRENT LIABILITIES

 

 

Accounts payable and accrued expenses           

$23,506,664

$24,478,303

Deferred game revenues                      

6,498,726

9,204,607

Deferred compensation—current portion          

1,226,316

1,278,410

TOTAL CURRENT LIABILITIES                    

31,231,706

34,961,320

NOTES PAYABLE TO BANK                      

50,000,000

50,000,000

DEFERRED COMPENSATION—noncurrent portion     

5,182,821

6,369,646

OTHER NONCURRENT LIABILITIES               

 

708,000

PARTNERS’ CAPITAL (DEFICIT)

 

 

Celtics Basketball Holdings, LP—General Partner     

1,015

1,008

Celtics Pride GP—Limited Partner               

29,111,174

29,437,209

Castle Creek Partners, LP—Limited Partner        

31,144,430

31,493,235

 

60,254,589

60,929,436

Celtics Basketball, LP—General Partner

1,081

1,074

TOTAL PARTNERS’ CAPITAL (DEFICIT)

60,253,508

60,928,362

 

$26,161,019

$31,110,604

1. From June 2000 to June 2001, the Celtics’ total assets decreased by approximately $5 million. What assets accounted for most of the decrease? Of course, total liabilities and equity also decreased by $5 million; what liability or equity items accounted for most of the decrease?

2. As of June 30, 2001, the Celtics have their NBA franchise recorded, net of amortization, at approximately $3.393 million. What original value was recorded for the NBA franchise? Over how many years is the NBA franchise being amortized? In what year was the NBA franchise originally recorded?

3. Partners’ capital as of June 30, 2001, is about negative $60.3 million. How can partners’ capital become negative?

4. The Celtics reported a liability for deferred compensation totaling $6,409,137 ($1,226,316 +$5,182,821).However, the notes to the financial statements revealed the following:“ Celtics Basketball has employment agreements with officers, coaches, and players of the Boston Celtics basketball team. Certain of the contracts provide for guaranteed payments which must be paid even if the employee is injured or terminated.” The Celtics then disclose that the total amount of these guaranteed payments is $254.585 million. Explain the vast difference between the $6.4 million deferred compensation liability reported in the balance sheet and the $254.585 million compensation obligation disclosed in the notes.

deciphering financial statements diageo diageo is a united kingdom uk consumer produ 691809

Deciphering Financial Statements (Diageo)

Diageo is a United Kingdom (UK) consumer products firm, best known in the United States for the following brand names: Smirnoff, Johnnie Walker, J&B, Gordon’s, Seagram’s, and Guinness. Diageo’s 2004 consolidated balance sheet follows.

Diageo

Consolidated Balance Sheet

30 June 2004

(In millions of pounds)

Fixed assets

 

 

Intangible assets                                              

 

4,012

Tangible assets                                               

 

1,976

Investments in associates                                       

 

1,263

Other investments                                             

 

1,772

 

 

9,023

Current assets

 

 

Stocks                                                     

2,176

 

Debtors—due within one year                                   

1,573

 

Debtors—due after one year                                    

151

 

Cash at bank and liquid resources                                 

1,167

 

 

5,067

 

Creditors—due within one year

 

 

Borrowings                                                  

(2,001)

 

Other creditors                                              

(3,022)

 

 

(5,023)

 

Net current assets (liabilities)                                 

 

44

Total assets less current liabilities                              

 

9,067

Creditors—due after more than one year

 

 

Borrowings                                                  

(3,316)

 

Other creditors                                              

(109)

 

 

 

(3,425)

Provisions for liabilities and charges                            

 

(709)

Net assets before post-employment assets and liabilities           

 

4,933

Post-employment assets                                        

7

 

Post-employment liabilities                                       

(757)

 

 

 

(750)

Net assets                                                 

 

4,183

Capital and reserves

 

 

Called up share capital                                         

 

885

Share premium account                                         

1,331

 

Revaluation reserve                                            

113

 

Capital redemption reserve                                      

3,058

 

Profit and loss account                                         

(1,695)

 

Reserves attributable to equity shareholders                          

 

2,807

Shareholder funds                                           

 

3,692

Minority interests

 

 

Equity                                                     

179

 

Non-equity                                                  

312

 

 

 

491

 

 

4,183

Re-create Diageo’s June 30, 2004, balance sheet using U.S. terminology and a standard U.S. format. (Note: Two of the reserve items have no counterpart in the United States. The revaluation reserve is the amount by which tangible assets have been written up to reflect an increase in market value. The capital redemption reserve is recorded when a company repurchases, or redeems, its own shares. Accounting for share repurchases is discussed in Chapter 13; for purposes of this exercise, add the capital redemption reserve to “called up share capital.”)

deciphering financial statements safeway albertson rsquo s and a amp p safeway opera 691810

Deciphering Financial Statements (Safeway, Albertson’s, and A&P)

Safeway operates 1,802 supermarkets in the United States and Canada. In the United States, Safeway is located principally in the Western, Southwestern, Rocky Mountain, Midwestern, and Mid Atlantic regions. Albertson’s operates 2,503 stores in 37 Northeastern, Western, Midwestern, and Southern states. The Great Atlantic & Pacific Tea Company (A&P) operates 649 stores in the Northeast and in Canada. Selected financial statement information for 2004 for these three companies follows (in millions of U.S. dollars).

 

Safeway

Albertson’s

A&P

Inventory                                

$ 2,741

$ 3,119

$ 654

Total current assets                        

3,598

4,295

1,146

Property, plant, and equipment               

8,689

10,472

1,449

Total assets                              

15,377

18,311

2,751

Total current liabilities                     

3,792

4,085

1,074

Total liabilities                            

11,071

12,890

2,365

Sales                                    

35,823

39,897

10,812

Cost of goods sold                        

25,228

28,711

7,883

Net income                              

560

444

(147)

1. For each of the three companies, compute the following ratios:

(a) Current ratio

(b) Debt ratio

(c) Asset turnover

(d) Return on equity

2. Which company uses its inventory most efficiently? Which company uses its property, plant, and equipment most efficiently?

3. What dangers might there be in making ratio comparisons without viewing the financial statement notes for the individual companies?

writing assignment unrecorded assets should stay unrecorded you are a member of the 691812

Writing Assignment (Unrecorded assets should stay unrecorded)

You are a member of the most popular student club on campus, the Accounting Ant defamation Organization. Recently, the field of accounting was savagely attacked in an article written by a militant economics student group and published in the student newspaper. The article charged that the balance sheet is stupid, outdated, and useless and cited as an example the accounting practice of not recognizing many intangible assets. As a specific illustration, the article claimed that the name recognition, reputation, and goodwill of the Coca Cola trademark are worth over $67 billion, but these assets are not recorded in Coca Cola’s balance sheet. You have been asked by the editor of the student newspaper to respond in writing to this vicious assault by the economics students. Don’t cave in to the pressure—argue persuasively why these unrecorded assets should stay unrecorded.

ethical dilemma dodging a loan covenant violation you are on the accounting staff of 691814

Ethical Dilemma (Dodging a loan covenant violation)

You are on the accounting staff of Chisos Manufacturing Company. Chisos has a $100 million loan with Rio Grande National Bank. One of the covenants associated with the loan is that Chisos must maintain a current ratio greater than 1.5. As of January 20, 2008, preliminary financial statement numbers for the year ended December 31, 2007, have been compiled. It looks like Chisos will violate the current ratio loan covenant. Violation could be very costly in two ways. First, Rio Grande National Bank has historically raised the interest rate one half of a point on loans with covenant violations. Second, a violation will increase the perceived riskiness of Chisos and make future borrowing more costly.

The 2007 financial statement numbers are just preliminary, and the senior accounting staff of Chisos has discussed the following two options to avoid violation:

1. Reclassify “long term investment property” as “short term property held for sale. Doing this would require a statement from management that the intention is to sell the property within one year. Actually, Chisos intends to hold the property for several more years, and the property classification would be changed back to long term next year when the threat of covenant violation has hopefully disappeared.

2. Reclassify certain short term loans as long term on the basis that Chisos will refinance the loans. Technically, this is true. However, Chisos has no formal refinancing commitment and will not have one until sometime in June. You have been chosen to present the findings of the accounting staff to the board of directors. What points will you emphasize in your presentation?

cumulative spreadsheet analysis this spreadsheet assignment is a continuation of the 691815

Cumulative Spreadsheet Analysis

This spreadsheet assignment is a continuation of the spreadsheet assignment given in Chapter 2. If you completed that assignment, you have a head start on this one.

1. Refer back to the financial statement numbers for Skywalker Enterprises for 2008 (given in part 1 of the Cumulative Spreadsheet Analysis assignment in Chapter 2). Revise those financial statements by making the following changes:

• Change the paid in capital amount from $150 to $200.

• In the Equity section of the balance sheet, insert a treasury stock amount of –$60.

The remaining amount of the “other equity” mentioned in Chapter 2 is accumulated other comprehensive income.

• Increase amount of long term debt from $621 to $671.

• In the Asset section of the balance sheet, insert an intangible asset amount of $100.

Using the revised balance sheet and income statement, create spreadsheet cell formulas to compute and display values for the following ratios.

• Current ratio

• Debt ratio

• Asset turnover

• Return on assets

• Return on equity

2. Determine the impact of each of the following transactions on the ratio values computed in Question 1.Treat each transaction independently; that is, before determining the impact of each new transaction you should reset the financial statement values to their original amounts. The transactions that follow are assumed to occur on December 31, 2008.

(a) Collected $60 cash from customer receivables.

(b) Purchased $90 in inventory on account.

(c) Purchased $300 in property, plant, and equipment. The entire amount of the purchase was financed with a mortgage. Principal repayment for the mortgage is due in 10 years.

(d) Purchased $300 in property, plant, and equipment. The entire amount of the purchase was financed with new stockholder investments.

(e) Borrowed $60 with a short term loan payable. The $60 was paid out as a dividend to stockholders.

(f) Received $60 as an investment from stockholders. The $60 was paid out as a dividend to stockholders.

(g) The long term debt amount of $671 includes $90 in short term loans payable that Skywalker hopes to refinance. Skywalker has no explicit agreement with the bank to refinance the loan and does not expect to finalize the refinancing until the last quarter of 2009.

(h) During the first week in January 2009, Skywalker learned that, of the $459 reported as inventory as of December 31, 2008, $45 is completely obsolete and worthless. The inventory had become obsolete during the last quarter of 2008, but the facts had not been verified until early 2009.

financial capital maintenance the company had the following total asset and total li 691816

Financial Capital Maintenance

The company had the following total asset and total liability balances at the beginning and the end of the year:

 

Beginning

Ending

Total assets

$400,000

$625,000

Total liabilities

230,000

280,000

During the year, the company received $100,000 in new investment funds contributed by the owners. Using the financial capital maintenance concept, determine the company’s income for the year.

asset mix use the information in practice 3 2 to compute the proportion of total ass 691762

Asset Mix

Use the information in Practice 3 2 to compute the proportion of total assets in each of the following asset categories. Assume that the list contains all the asset items.

(a) Inventory

(b) Property, Plant, and Equipment

Current Liabilities

Using the following information, compute total current liabilities:

Accrued Income Taxes Payable                                                    

$ 9,000

Notes Payable (due in 14 months)                                                 

1,100

Paid In Capital                                                               

1,750

Treasury Stock                                                               

400

Current Portion of Long Term Debt                                               

10,000

Unearned Revenue                                                            

250

Accounts Payable                                                             

700

Retained Earnings                                                             

1,000

Additional Paid In Capital                                                         

4,000

return on assets refer to practice 3 9 net income for the year totaled 2 000 compute 691764

Return on Assets

Refer to Practice 3 9. Net income for the year totaled $2,000. Compute return on assets.

Practice 3 9

Format of Foreign Balance Sheet

Following is a balance sheet presented in standard U.S. format. Rearrange this balance sheet to be in standard British format. Don’t worry about differences in terminology; use the U.S. labels, but present the information in the British format.

Current Assets:

 

Cash                                                                    

$ 500

Inventory                                                                

2,000

Total current assets                                                         

$ 2,500

Noncurrent Assets:

 

Property, plant, and equipment                                                 

$ 8,000

Long term investments                                                       

1,700

Total noncurrent assets                                                      

$ 9,700

Total assets                                                               

$12,200

Current Liabilities:

 

Accounts payable                                                           

$ 300

Short term loans payable                                                     

1,100

Total current liabilities                                                        

$ 1,400

Noncurrent Liabilities:

 

Long term debt                                                            

$ 3,000

Stockholders’ Equity:

 

Common stock, at par                                                       

$ 50

Additional paid in capital                                                     

2,000

Retained earnings                                                          

5,750

Total stockholders’ equity                                                     

$ 7,800

Total liabilities and stockholders’ equity                                           

$12,200

book to market ratio refer to practice 3 9 as of the end of the year the total marke 691768

Book to Market Ratio

Refer to Practice 3 9. As of the end of the year, the total market value of shares outstanding was $10,000. Compute the book to market ratio.

Practice 3 9

Format of Foreign Balance Sheet

Following is a balance sheet presented in standard U.S. format. Rearrange this balance sheet to be in standard British format. Don’t worry about differences in terminology; use the U.S. labels, but present the information in the British format.

Current Assets:

 

Cash                                                                    

$ 500

Inventory                                                                 

2,000

Total current assets                                                         

$ 2,500

Noncurrent Assets:

 

Property, plant, and equipment                                                 

$ 8,000

Long term investments                                                       

1,700

Total noncurrent assets                                                      

$ 9,700

Total assets                                                               

$12,200

Current Liabilities:

 

Accounts payable                                                           

$ 300

Short term loans payable                                                     

1,100

Total current liabilities                                                       

$ 1,400

Noncurrent Liabilities:

 

Long term debt                                                            

$ 3,000

Stockholders’ Equity:

 

Common stock, at par                                                        

$ 50

Additional paid in capital                                                     

2,000

Retained earnings                                                          

5,750

Total stockholders’ equity                                                     

$ 7,800

Total liabilities and stockholders’ equity                                           

$12,200

balance sheet classification a balance sheet contains the following classifications 691769

Balance Sheet Classification

A balance sheet contains the following classifications:

(a) Current assets

(b) Investments

(c) Property, plant, and equipment

(d) Intangible assets

(e) Other noncurrent assets

(f) Current liabilities

(g) Long term debt

(h) Other noncurrent liabilities

(i) Capital stock

(j) Additional paid in capital

(k) Retained earnings

Indicate by letter how each of the following accounts would be classified. Place a minus sign ( ) for all accounts representing offset or contra balances.

1. Discount on Bonds Payable

2. Stock of Subsidiary Corporation

3. 12% Bonds Payable (due in 6 months)

4. U.S. Treasury Notes

5. Income Taxes Payable

6. Sales Taxes Payable

7. Estimated Claims under Warranties for Service and Replacements

8. Par Value of Stock Issued and Outstanding

9. Unearned Rent Revenue (6 months in advance)

10. Long Term Advances to Officers

11. Interest Receivable

12. Preferred Stock Retirement Fund

13. Trademarks

14. Allowance for Bad Debts

15. Dividends Payable

16. Accumulated Depreciation

17. Trading Securities

18. Prepaid Rent

19. Prepaid Insurance

20. Deferred Income Tax Asset

balance sheet classification state how each of the following accounts should be clas 691770

Balance Sheet Classification

State how each of the following accounts should be classified on the balance sheet.

(a) Treasury Stock

(b) Retained Earnings

(c) Vacation Pay Payable

(d) Foreign Currency Translation Adjustment

(e) Allowance for Bad Debts

(f) Liability for Pension Payments

(g) Investment Securities (Trading)

(h) Paid In Capital in Excess of Stated Value

(i) Leasehold Improvements

(j) Goodwill

(k) Receivables—U.S. Government Contracts

(l) Advances to Salespersons

(m) Premium on Bonds Payable

(n) Inventory

(o) Patents

(p) Unclaimed Payroll Checks

(q) Income Taxes Payable

(r) Subscription Revenue Received in Advance

(s) Interest Payable

(t) Deferred Income Tax Asset

(u) Tools

(v) Deferred Income Tax Liability

liability definition using the definition of a liability from fasb concepts statemen 691772

Liability Definition

Using the definition of a liability from FASB Concepts Statement No. 6, indicate whether each of the following should be listed as a liability by Pauli Company:

(a) Pauli was involved in a highly publicized lawsuit last year. Pauli lost and was ordered to pay damages of $125 million. The payment has been made.

(b) In exchange for television advertising services that Pauli received last month, Pauli is obligated to provide the television station with building maintenance service for the next four months.

(c) Pauli contractually guarantees to replace any of its stain resistant carpets if they are stained and can’t be cleaned.

(d) Pauli estimates that its total payroll for the coming year will exceed $35 million.

(e) In the past, Pauli has suffered frequent vandalism at its storage warehouses. Pauli estimates that losses due to vandalism during the coming year will total $3 million.

preparation of corrected balance sheet the following balance sheet was prepared for 691775

Preparation of Corrected Balance Sheet

The following balance sheet was prepared for Jared Corporation as of December 31, 2008.

Jared Corporation

Balance Sheet

December 31, 2008

Assets

 

Liabilities and Owners’ Equity

 

Current assets:

 

Current liabilities:

 

Cash

$ 12,500

Accounts payable

$ 3,400

Investment securities

8,000

Other current liabilities

2,000

Accounts receivable, net

21,350

Total current liabilities

$ 5,400

Inventory

31,000

Long term liabilities

32,750

Other current assets

14,200

Total liabilities

$ 38,150

Total current assets

$ 87,050

 

 

Noncurrent assets:

 

Owners’ equity:

 

Property, plant, and equipment, net

$ 64,800

Common stock

$ 50,000

Treasury stock

4,500

Retained earnings

81,800

Other noncurrent assets

13,600

Total owners’ equity

$131,800

Total noncurrent assets

$ 82,900

 

 

Total assets

$169,950

Total liabilities and owners’ equity

$169,950

The following additional information relates to the December 31, 2008, balance sheet.

(a) Cash includes $4,000 that has been restricted to the purchase of manufacturing equipment

(a noncurrent asset).

(b) Investment securities include $2,750 of stock that was purchased in order to give the company significant ownership and a seat on the board of directors of a major supplier.

(c) Other current assets include a $4,000 advance to the president of the company. No due date has been set.

(d) Long term liabilities include bonds payable of $10,000. Of this amount, $2,500 represents bonds scheduled to be redeemed in 2009.

(e) Long term liabilities also include a $7,000 bank loan. On May 15, the loan will become due on demand.

(f) On December 21, dividends in the amount of $15,000 were declared to be paid to shareholders of record on January 25.These dividends have not been reflected in the financial statements.

(g) Cash in the amount of $19,000 has been placed in a restricted fund for the redemption of preferred stock in 2009. Both the cash and the stock have been removed from the balance sheet.

(h) Property, plant, and equipment includes land costing $8,000 that is being held for investment purposes and that is scheduled to be sold in 2009.

Based on the information provided, prepare a corrected balance sheet.

balance sheet relationships on the clark and company inc balance sheet indicate the 691776

Balance Sheet Relationships

On the Clark and Company Inc. balance sheet, indicate the amount that should appear for each of the items (a) through (n) on the balance sheet.

Clark and Company Inc.

Consolidated Balance Sheet

December 31, 2008

Assets

 

 

 

Current assets:

 

 

 

Cash                                               

 

$ 24,250

 

Investment securities                                   

 

(a)

 

Accounts and notes receivable                            

$ (b)

 

 

Allowance for doubtful accounts and notes receivable           

7,851

121,664

 

Inventories                                           

 

197,682

 

Other current assets                                   

 

14,227

 

Total current assets                                    

 

 

$ (c)

Noncurrent assets:

 

 

 

Property, plant, and equipment                             

$694,604

 

 

Accumulated depreciation                                

(d)

$398,832

 

Other noncurrent assets                                 

 

13,217

 

Total noncurrent assets                                  

 

 

412,049

Total assets                                             

 

 

$792,514

Liabilities and Owners’ Equity

 

 

 

Current liabilities:

 

 

 

Accounts payable                                      

 

$ (e)

 

Payable to banks                                       

 

34,236

 

Income taxes payable                                   

 

9,211

 

Current installments of long-term debt                      

 

6,341

 

Accrued expenses                                     

 

7,100

 

Total current liabilities                                  

 

 

$ (f)

Noncurrent liabilities:

 

 

 

Long-term debt                                       

 

$ (g)

 

Deferred income tax liability                              

 

41,218

 

Minority interest in subsidiaries                            

 

4,201

 

Total noncurrent liabilities                                

 

 

205,410

Total liabilities                                           

 

 

$350,782

Contributed capital:

 

 

 

Preferred stock, no par value (authorized

 

 

 

1,618 shares; issued 1,115 shares)                        

$ 12,392

 

 

Common stock, $1 par value per share

 

 

 

(authorized 60,000 shares; issued 21,842 shares)              

(h)

 

 

Additional paid-in capital                                 

(i)

(j)

 

Total contributed capital                                 

 

$ (k)

 

Retained earnings                                        

 

390,625

 

Total contributed capital and retained earnings                   

 

$ (l)

 

Less: Treasury stock, at cost (1,229 shares)                      

 

27,038 

 

Total owners’ equity                                      

 

 

$ (m)

Total liabilities and owners’ equity                             

 

 

$ (n)

balance sheet schedules in its annual report to stockholders crantz inc presents a c 691777

Balance Sheet Schedules

In its annual report to stockholders, Crantz Inc. presents a condensed balance sheet with detailed data provided in supplementary schedules.

1. From the adjusted trial balance of Crantz, prepare the following sections of the balance sheet, properly classifying all accounts as to balance sheet categories:

(a) Current assets

(b) Property, plant, and equipment

(c) Intangible assets

(d) Total assets

(e) Current liabilities

(f) Noncurrent liabilities

(g) Owners’ equity

(h) Total liabilities and owners’ equity

2. Compute the current ratio and debt ratio for Crantz.

Crantz Inc.

Adjusted Trial Balance

December 31, 2008

 

Debit

Credit

Cash                                                       

$ 33,900

 

Investment securities (trading)                                    

20,000

 

Notes receivable—trade debtors                                  

18,000

 

Accrued interest on notes receivable                               

1,800

 

Accounts receivable                                           

88,400

 

Allowance for doubtful accounts                                  

 

$ 4,300

Inventory                                                   

56,900

 

Prepaid expenses                                              

6,100

 

Accounts payable                                              

 

31,500

Notes payable—trade creditors                                   

 

16,000

Accrued interest on notes payable                                 

 

800

Land                                                       

80,000

 

Buildings                                                    

170,000

 

Accumulated depreciation—buildings                               

 

34,000

Equipment                                                  

48,000

 

Accumulated depreciation—equipment                              

 

7,600

Patents                                                     

15,000

 

Franchises                                                   

10,000

 

Bonds payable, 8%—issue 1 (mature 12/31/10)                        

 

50,000

Bonds payable, 12%—issue 2 (mature 12/31/14)                       

 

100,000

Accrued interest on bonds payable                                

 

8,000

Premium on bonds payable—issue 1                                

 

1,500

Discount on bonds payable—issue 2                                

10,500

 

Mortgage payable                                             

 

57,500

Accrued interest on mortgage payable                              

 

2,160

Capital stock, par value $1; 10,000 shares authorized;

 

 

4,000 shares issued                                         

 

4,000

Additional paid-in capital                                        

 

112,800

Retained earnings                                             

 

139,440

Treasury stock—at cost (500 shares)                               

11,000

 

 

$569,600

$569,600

computation of financial ratios the following data are from the financial statements 691778

Computation of Financial Ratios

The following data are from the financial statements of Borg Company.

Current assets                                 

                             $ 70,000

Total assets                                   

                             150,000

Current liabilities                               

                             30,000

Total liabilities                                 

                             80,000

Net income                                   

                             10,000

Sales                                        

                             300,000

Compute Borg’s current ratio, debt ratio, asset turnover, return on assets, and return on equity.

computation of financial ratios schlofman company has the following assets cash 20 0 691779

Computation of Financial Ratios

Schlofman Company has the following assets.

Cash

$ 20,000

Accounts receivable

60,000

Inventory

105,000

Property, plant, and equipment

220000

Total assets

$405,000

Companies in Schlofman’s industry typically have the following asset mix: cash, 7%; accounts receivable, 15%; inventory, 18%; property, plant, and equipment, 60%. Compared to other companies in its industry, Schlofman has too much of one asset. Which one? Show your computations.

classification of subsequent events the following events occurred after the end of t 691780

Classification of Subsequent Events

The following events occurred after the end of the company’s fiscal year but before the annual audit was completed. Classify each event as to its impact on the financial statements, that is, (1) reported by changing the amounts in the financial statements, (2) reported in notes to the financial statements, or (3) does not require reporting. Include support for your classification.

(a) Major customer went bankrupt due to a deteriorating financial condition.

(b) Company sustained extensive hurricane damage to one of its plants.

(c) Company lost a major lawsuit that had been pending for two years.

(d) Increasing U.S. trade deficit may have impact on company’s overseas sales.

(e) Company sold a large block of preferred stock.

(f) Preparation of current year’s income tax return disclosed that an additional $25,000 is due on last year’s return.

(g) Company’s controller resigned and was replaced by an audit manager from the company’s audit firm.

reporting financial information for each of the following items indicate whether the 691781

Reporting Financial Information

For each of the following items, indicate whether the item should be reflected in the 2008 financial statements for Tindall Company. If the item should be reflected, indicate whether it should be reported in the financial statements themselves or by note disclosure.

(a) As of December 31,2008, the company holds $12.1 million of its own stock that it purchased in the open market and is holding for possible reissuance.

(b) As of December 31, 2008, the company was in violation of certain loan covenants. The violation does not cause the loans to be callable immediately but does increase the interest charge by 2.0%.

(c) The company’s reported Provision for Income Taxes includes $4.2 million in current taxes and $7.8 million in deferred taxes.

(d) As of December 31, 2008, accounts receivable in the amount of $7.1 million are estimated to be uncollectible.

(e) The Environmental Protection Agency is investigating the company’s procedures for disposing of toxic waste. Outside consultants have estimated that the company may be liable for fines of up to $10 million.

(f) During 2008, the company had a gain on the sale of manufacturing assets.

(g) During 2008, a long term insurance agreement was signed. The company paid five years of insurance premiums in advance.

(h) The company uses straight line depreciation for all tangible, long term assets.

(i) During 2008, the company hired three prominent research chemists away from its chief competitor.

(j) Reported long term debt is composed of senior subordinated bonds payable, convertible bonds payable, junior subordinated bonds payable, and capital lease obligations.

(k) Early in 2009, a significant Dr. op in raw material prices caused the company’s stock price to rise in anticipation of sharply increased profits for the year.

book to market ratio the following information relates to two companies designated c 691783

Book to Market Ratio

The following information relates to two companies, designated Company A and Company B. One of the companies is a traditional steel manufacturer. The other is a successful Internet retailer. Using the following information, identify which is which, and explain your answer.

Reported

 

Total Market Value

Stockholders’ Equity

 

of Equity

Company A                                

$10,000

$75,000

Company B                                

10,000

8,000

computing balance sheet components denton equipment inc furnishes you with the follo 691784

Computing Balance Sheet Components

Denton Equipment Inc. furnishes you with the following list of accounts.

Accounts Payable                                                             

$ 66,000

Accounts Receivable                                                           

40,000

Accumulated Depreciation                                                       

44,000

Advances to Salespersons                                                       

10,000

Advertising Expense                                                           

72,000

Allowance for Bad Debts                                                        

10,000

Bonds Payable                                                               

80,000

Cash                                                                      

22,000

Certificates of Deposit                                                         

16,000

Common Stock (par)                                                          

100,000

Deferred Income Tax Liability                                                     

46,000

Equipment                                                                  

215,500

Inventory                                                                   

55,000

Investment in Rowe Oil Co Stock (40% of outstanding stock owned for control purposes)        

76,500

Investment in Siebert Co Stock (trading securities)                                     

21,000

Paid In Capital in Excess of Par                                                   

42,500

Premium on Bonds Payable                                                      

6,000

Prepaid Insurance                                                             

6,000

Rent Revenue                                                                

37,000

Rent Revenue Received in Advance (4 months)                                        

12,000

Retained Earnings                                                             

97,500

Taxes Payable                                                                 

10,000

Tools                                                                      

52,000

Instructions:

1. From the preceding list of accounts, determine working capital, total assets, total liabilities, and owners’ equity per share of stock (75,000 shares outstanding).

2. Assume net income of $20,000. Compute current ratio, debt ratio, and return on equity.

classified balance sheet following is a list of account titles and balances for wait 691785

Classified Balance Sheet

Following is a list of account titles and balances for Waite Investment Corporation as of January 31, 2008.

Accounts Payable                                                             

$ 87,900

Accounts Receivable                                                           

161,200

Accumulated Depreciation—Buildings                                               

149,700

Accumulated Depreciation—Machinery and Equipment                                  

121,300

Additional Paid In Capital—Common Stock                                          

$612,000

Allowance for Doubtful Notes and Accounts Receivable                                 

19,700

Buildings                                                                   

370,000

Cash Fund for Stock Redemption                                                  

22,500

Cash in Banks                                                               

10,320

Cash on Hand                                                               

86,250

Claim for Income Tax Refund                                                     

5,100

Common Stock, $1 par                                                         

60,000

Employees’ Income Taxes Payable                                                  

4,260

Income Taxes Payable                                                          

19,900

Interest Payable                                                              

6,890

Interest Receivable                                                             

1,200

Inventory                                                                   

176,000

Investment Securities (trading)                                                    

98,750

Investments in Undeveloped Properties                                             

183,000

Land                                                                      

201,000

Machinery and Equipment                                                       

145,000

Miscellaneous Supplies Inventory                                                  

5,600

Notes Payable (current)                                                         

52,320

Notes Payable (due in 2013)                                                     

41,000

Notes Receivable (current)                                                      

25,960

Preferred Stock, $5 par                                                          

305,000

Prepaid Insurance                                                             

2,800

Retained Earnings                                                             

6,010

Salaries and Wages Payable                                                      

8,700

Instructions:

1. Prepare a properly classified balance sheet.

2. Assume net income of $200,000 and sales of $5,000,000. Compute the current ratio, debt ratio, and asset turnover.

classified balance sheet mdash including notes adjusted account balances and supplem 691786

Classified Balance Sheet—Including Notes

Adjusted account balances and supplemental information for Brock bank Research Corp. as of December 31, 2008, are as follows:

Accounts Payable

$ 32,160

Accounts Receivable—Trade                                                      

57,731

Accumulated Depreciation—Leasehold Improvements and Equipment                       

579,472

Additional Paid In Capital                                                       

265,000

Allowance for Bad Debts                                                       

1,731

Automotive Equipment                                                         

132,800

Cash                                                                      

25,600

Cash Fund for Bond Retirement                                                  

3,600

Common Stock                                                              

35,000

Deferred Income Tax Liability                                                    

45,000

Dividends Payable                                                             

37,500

Franchises                                                                   

12,150

Furniture, Fixtures, and Store Equipment                                            

769,000

Insurance Claims Receivable                                                     

120,000

Inventories                                                                   

201,620

Investment in Unconsolidated Subsidiary                                            

80,000

Land                                                                      

6,000

Leasehold Improvements                                                        

65,800

7 1⁄2%–12% Mortgage Notes Payable                                                

200,000

Notes Payable—Banks (due in 2009)                                                

12,000

Notes Payable—Trade                                                          

63,540

Patent Licenses                                                               

57,402

Prepaid Insurance                                                              

5,500

Profit Sharing, Payroll, and Vacation Payable                                           

40,000

Retained Earnings                                                             

225,800

Supplemental information is as follows:

(a) Depreciation is provided by the straight line method over the estimated useful lives of the assets.

(b) Common stock is $1 par, and 35,000 of the 100,000 authorized shares were issued and are outstanding.

(c) The cost of an exclusive franchise to import a foreign company’s ball bearings and a related patent license are being amortized on the straight line method over their remaining lives: franchise, 10 years; patents, 15 years.

(d) Inventories are stated at the lower of cost or market; cost was determined by the specific identification method.

(e) Insurance claims based on the opinion of an independent insurance adjustor are for property damages at the central warehouse. These claims are estimated to be two thirds collectible in the following year and one third collectible thereafter.

(f) The company leases all of its buildings from various lessors. Estimated fixed lease obligations are $50,000 per year for the next 10 years. The leases do not meet the criteria for capitalization.

(g) The company is currently in litigation over a claimed overpayment of income tax of $13,000. In the opinion of counsel, the claim is valid. The company is contingently liable on guaranteed notes worth $12,000.

Instructions: Prepare a properly classified balance sheet. Include all notes and parenthetical notations necessary to properly disclose the essential financial data.

classification of liabilities the accountant for sierra corp prepared the following 691787

Classification of Liabilities

The accountant for Sierra Corp. prepared the following schedule of liabilities as of December 31, 2008.

Accounts payable

$ 65,000

Notes payable—trade

19,000

Notes payable—bank

80,000

Wages and salaries payable

1,500

Interest payable

14,300

Mortgage note payable—10%

60,000

Mortgage note payable—12%

150,000

Bonds payable

200000

Total

$589,800

The following additional information pertains to these liabilities.

(a) All trade notes payable are due within six months of the balance sheet date.

(b) Bank notes payable include two separate notes payable to First Interstate Bank.

(1) A $30,000, 8% note issued March 1, 2006, payable on demand. Interest is payable every six months.

(2) A 1 year, $50,000,111⁄2% note issued January 2, 2008.On December 30,2008, Sierra negotiated a written agreement with First Interstate Bank to replace the note with a 2 year, $50,000, 10% note to be issued January 2, 2009.

(c) The 10% mortgage note was issued October 1, 2005, with a term of 10 years. Terms of the note give the holder the right to demand immediate payment if the company fails to make a monthly interest payment within 10 days of the date the payment is due. As of December 31, 2008, Sierra is three months behind in paying its required interest payment.

(d) The 12% mortgage note was issued May 1, 2002, with a term of 20 years. The current principal amount due is $150,000. Principal and interest are payable annually on April 30. A payment of $22,000 is due April 30, 2009. The payment includes interest of $18,000.

(e) The bonds payable are 10 year, 8% bonds, issued June 30, 1999.

Instructions: Prepare the Liabilities section of the December 31, 2008, classified balance sheet for Sierra Corp. Include notes as appropriate. Assume the interest payable accrual has been computed correctly.

corrected balance sheet the following balance sheet was prepared by the accountant f 691788

Corrected Balance Sheet

The following balance sheet was prepared by the accountant for Tippetts Company.

Tippetts Company

Balance Sheet

June 30, 2008

Assets

 

Cash                                                                      

$ 32,200

Investment securities—Trading (includes long term investment of $250,000 in stock of

 

Pine Valley Developers)                                                      

298,000

Inventories (net of amount still due suppliers of $75,000)                                

605,400

Prepaid expenses (includes a deposit of $15,000 made on inventories to be delivered

 

in 18 months)                                                             

48,000

Property, plant, and equipment (excluding $70,000 of equipment still in use,

 

but fully depreciated)                                                        

240,000

Goodwill (based on estimate by the president of Tippetts Company)                        

90,000

Total assets                                                                

$1,313,600

Liabilities and Owners’ Equity

 

Notes payable ($70,000 due in 2010)                                               

$ 140,000

Accounts payable (not including amount due to suppliers of inventory—see above)             

135,000

Long term liability under pension plan                                              

55,000

Retained earnings restricted for building expansion                                     

115,000

Accumulated depreciation                                                        

78,000

Taxes payable                                                                

42,500

Bonds payable (net of discount of $20,000)                                          

280,000

Deferred income tax liability                                                      

62,000

Common stock (20,000 shares, $1 par)                                             

20,000

Additional paid in capital                                                        

237,500

Unrestricted retained earnings                                                    

148,600

Total liabilities and owners’ equity                                              

$1,313,600

Instructions: Prepare a corrected classified balance sheet using appropriate account titles.

corrected balance sheet the bookkeeper for reliable computers inc reports the follow 691790

Corrected Balance Sheet

The bookkeeper for Reliable Computers, Inc., reports the following balance sheet amounts as of June 30, 2008.

Current assets                                                               

$233,400

Other assets                                                                

667,100

Current liabilities                                                              

146,820

Other liabilities                                                               

100,000

Owners’ equity                                                              

653,680

A review of account balances reveals the following data.

(a) An analysis of current assets discloses the following:

Cash

$ 47,500

Investment securities—trading

55,000

Accounts receivable

51,900

Inventories, including advertising supplies of $2,000

79,000

 

$233,400

 (b) Other assets include the following:

Property, plant, and equipment:

 

Depreciated book value (cost, $670,000)                                 

$574,000

Deposit with a supplier for merchandise ordered for August delivery               

5,200

Goodwill recorded on the books to cancel losses incurred by the company in prior years

87900

 

$667,100

(c) Current liabilities include the following:

Payroll payable                                  

$ 8,250

Taxes payable                                   

4,670

Rent payable                                   

13,200

Accounts payable

 

Total owed to suppliers on account                

$104,700

 

Less: 6 month note received from a supplier who purchased

 

 

some used equipment on June 29, 2008           

2000

102,700

Notes payable                                  

 

18000

   

$146,820

(d) Other liabilities include the following:

10% mortgage on property, plant, and equipment, payable in semiannual

 

installments of $10,000 through June 30, 2013            

$100,000

(e) Owners’ equity includes the following:

Preferredstock:20,000sharesoutstanding($20parvalue)

$400,000

Commonstock:150,000sharesat$1statedvalue

. 150,000

Additional paid in capital

103,680

 

$653,680

(f) Common shares were originally issued for $394,000, but the losses of the company for the past years were charged against additional paid in capital.

Instructions: Using the account balances and related data, prepare a corrected balance sheet showing individual asset, liability, and owners’ equity balances properly classified.

cash to accrual adjusting entries and income statement gee enterprises records all t 691733

Cash to Accrual Adjusting Entries and Income Statement

Gee Enterprises records all transactions on the cash basis. Greg Gee, company accountant, prepared the following income statement at the end of the company’s first year of operations:

Gee Enterprises
Income Statement
For the Year Ended December 31, 2008

Sales                   

 

$252,000

Selling and administrative expenses:

 

 

Salaries expense        

$78,000

 

Rent expense          

45,000

 

Utilities expense        

29,000

 

Equipment             

30,000

 

Commission expense     

37,800

 

Insurance expense       

6,000

 

Interest expense        

3,000

228,800

Net income              

 

$ 23,200

You have been asked to prepare an income statement on the accrual basis. The following information is given to you to assist in the preparation:

(a) Amounts due from customers at year end were $28,000. Of this amount, $3,000 will probably not be collected.

(b) Salaries of $11,000 for December 2008 were paid on January 5, 2009. Ignore payroll taxes.

(c) Gee rents its building for $3,000 a month, payable quarterly in advance. The contract was signed on January 1, 2008.

(d) The bill for December’s utility costs of $2,700 was paid January 10, 2009.

(e) Equipment of $30,000 was purchased on January 1, 2008. The expected life is five years, no salvage value. Assume straight line depreciation.

(f) Commissions of 15% of sales are paid on the same day cash is received from customers.

(g) A 1 year insurance policy was issued on company assets on July 1, 2008. Premiums are paid annually in advance.

(h) Gee borrowed $50,000 for one year on May 1,2008.Interest payments based on an annual rate of 12% are made quarterly, beginning with the first payment on August 1, 2008.

(i) The income tax rate is 40%. No prepayments of income taxes were made during 2008.

Instructions:

1. Prepare adjusting entries to convert the books from a cash to an accrual basis.

2. Prepare the income statement for the year ended December 31, 2008, based on the entries in (1).

adjusting and closing entries account balances taken from the ledger of builders rsq 691734

Adjusting and Closing Entries

Account balances taken from the ledger of Builders’ Supply Corporation on December 31,

2008, before adjustment, follow information relating to adjustments on December 31, 2008:

(a) Allowance for Bad Debts is to be increased to a balance of $3,000.

(b) Buildings are depreciated at the rate of 5% per year.

(c) Accrued selling expenses are $3,840.

(d) There are supplies of $780 on hand.

(e) Prepaid insurance relating to 2009 totals $720.

(f) Accrued interest on long term investments is $240.

(g) Accrued real estate and payroll taxes are $900.

(h) Accrued interest on the mortgage is $480.

(i) Income taxes are estimated to be 20% of the income before income taxes.

Cash                       

$ 24,000

Accounts Receivable            

72,000

Allowance for Bad Debts        

1,380

Inventory                   

87,570

Long Term Investments          

15,400

Land                       

69,600

Buildings                    

72,000

Accumulated Depreciation—Buildings

19,800

Accounts Payable              

35,000

Mortgage Payable              

68,800

Capital Stock, $10 par           

180,000

Retained Earnings, December 31, 2007

14,840

Dividends                    

13,400

Sales                       

246,000

Sales Returns                 

4,360

Sales Discounts               

5,400

Cost of Goods Sold            

114,370

Selling Expenses               

49,440

Office Expenses               

21,680

Insurance Expense             

1,440

Supplies Expense              

5,200

Taxes—Real Estate and Payroll    

7,980

Interest Revenue              

660

Interest Expense               

2,640

Instructions:

1. Prepare a trial balance.

2. Journalize the adjustments.

3. Journalize the closing entries.

4. Prepare a post closing trial balance.

adjusting and closing entries and post closing trial balance data for adjustments at 691735

Adjusting and Closing Entries and Post Closing Trial Balance

Data for adjustments at December 31, 2008, are as follows:

(a) Taipei International uses a perpetual inventory system.

(b) An analysis of Accounts Receivable reveals that the appropriate year end balance in

Allowance for Bad Debts is $750.

(c) Equipment depreciation for the year totaled $32,000.

(d) A recheck of the inventory count revealed that goods costing $5,600 were wrongly excluded from ending inventory. The goods in question were not shipped until January 3, 2009. A related receivable for $8,200 was also mistakenly recorded.

(e) Interest on the note payable has not been accrued. The note was issued on March 1, 2008, and the interest rate is 12%.

(f) The balance in Insurance Expense represents $3,000 that was paid for a 1 year policy on October 1. The policy went into effect on October 1.

(g) Dividends totaling $7,800 were declared on December 25.The dividends will not be paid until January 15, 2009. No entry was made.

Taipei International Corporation
Unadjusted Trial Balance
December 31, 2008

 

Debit

Credit

Cash                       

$ 31,500

 

Accounts Receivable            

25,000

 

Allowance for Bad Debts        

 

$ 250

Inventory                    

41,700

 

Equipment                   

190,000

 

Accumulated Depreciation—Equipment

 

51,000

Accounts Payable              

 

31,000

Notes Payable                

 

70,000

Wages Payable                

 

8,000

Income Taxes Payable           

 

6,500

Common Stock               

 

40,000

Retained Earnings              

 

34,100

Sales Revenue                 

 

310,000

Interest Revenue               

 

12,000

Cost of Goods Sold            

205,250

 

Wages Expense               

45,000

 

Interest Expense               

3,200

 

Utilities Expense               

6,000

 

Insurance Expense             

3,000

 

Advertising Expense            

5,000

 

Income Tax Expense            

7,200

 

Totals                     

$562,850

$562,850

Instructions:

1. Journalize the necessary adjusting entries. (Ignore income tax effects.)

2. Journalize the necessary closing entries.

3. Prepare a post closing trial balance.

4. Can a company pay dividends in a year in which it has a net loss? Can a company owe income taxes in a year in which it has a net loss?

preparation of work sheet account balances taken from the ledger of royal distributi 691736

Preparation of Work Sheet

Account balances taken from the ledger of Royal Distributing Co. on December 31, 2008, follow:

Cash                      

$ 35,000

Accounts Receivable           

91,000

Allowance for Bad Debts       

1,800

Inventory                   

92,000

Long Term Investments         

27,500

Land                      

53,400

Buildings                   

112,500

Accumulated Depreciation—Buildings

26,780

Accounts Payable             

47,300

Mortgage Payable             

99,500

Capital Stock, $5 par          

175,000

Retained Earnings, December 31, 2007

14,840

Dividends                   

9,670

Sales                      

359,000

Sales Returns                

12,890

Sales Discounts               

7,540

Cost of Goods Sold           

158,520

Selling Expenses              

62,350

Office Expenses              

38,900

Insurance Expense            

14,000

Supplies Expense             

4,800

Taxes—Real Estate and Payroll

$ 9,500

Interest Revenue        

550

Interest Expense         

3,200

Information relating to adjustments on December 31, 2008, follows:

(a) Allowance for Bad Debts is to be increased by $2,000.

(b) Buildings have a salvage value of $7,500. They are being depreciated at the rate of 10% per year.

(c) Accrued selling expenses are $8,600.

(d) There are supplies of $1,250 on hand.

(e) Prepaid insurance relating to 2009 totals $4,000.

(f) Total interest revenue earned in 2008 is $1,400.

(g) Accrued real estate and payroll taxes are $2,340.

(h) Accrued interest on the mortgage is $1,780.

(i) Income tax is estimated to be 40% of income.

Instructions: Prepare a work sheet showing the net income and balance sheet totals for the year ending December 31, 2008.

preparation of work sheet and adjusting and closing entries the following account ba 691737

Preparation of Work Sheet and Adjusting and Closing Entries

The following account balances are taken from the general ledger of Whitni Corporation on December 31, 2008, the end of its fiscal year. The corporation was organized January 2, 2002.

Cash                                 

$ 40,250

Notes Receivable                       

16,500

Accounts Receivable                     

63,000

Allowance for Bad Debts (credit balance)      

650

Inventory, December 31, 2008              

94,700

Land                                

80,000

Buildings                             

247,600

Accumulated Depreciation—Buildings        

18,000

Furniture and Fixtures                   

15,000

Accumulated Depreciation—Furniture and Fixtures

9,000

Notes Payable                         

18,000

Accounts Payable                       

72,700

Common Stock, $100 par                 

240,000

Retained Earnings                       

129,125

Sales                                

760,000

Sales Returns and Allowances              

17,000

Cost of Goods Sold                     

465,800

Utilities Expense                        

16,700

Property Tax Expense                    

10,200

Salaries and Wages Expense                

89,000

Sales Commissions Expense               

73,925

Insurance Expense                      

18,000

Interest Revenue                       

2,600

Interest Expense                        

2,400

Data for adjustments at December 31, 2008, are as follows:

(a) Depreciation (to nearest month for additions): furniture and fixtures, 10%; buildings, 4%.

(b) Additions to the buildings costing $150,000 were completed June 30, 2008.

(c) Allowance for Bad Debts is to be increased to a balance of $2,500.

(d) Accrued expenses: sales commissions, $700; interest on notes payable, $45; property taxes, $6,000.

(e) Prepaid expenses: insurance, $3,200.

(f) Accrued revenue: interest on notes receivable, $750.

(g) The following information is also to be recorded:

(1) On December 30, the board of directors declared a quarterly dividend of $1.50 per share on common stock, payable January 25, 2009, to stockholders of record January 15, 2009.

(2) Income taxes for 2008 are estimated at $15,000.

(3) The only charges to Retained Earnings during the year resulted from the declaration of the regular quarterly dividends.

Instructions:

1. Prepare an 8 column spreadsheet. There should be a pair of columns each for trial balance, adjustments, income statement, and balance sheet.

2. Prepare all the journal entries necessary to record the effects of the foregoing information and to adjust and close the books of the corporation.

to record or not to record explain why each of the following hypothetical events wou 691739

To Record or Not to Record

Explain why each of the following hypothetical events would not be recorded in a journal entry.

1. A famous and much beloved movie star is secretly filmed by an investigative news team using your company’s product when she in fact has an endorsement contract with your company’s major competitor.

2. Two of your firm’s top vice presidents have a bitter argument and will probably never speak to each other again.

3. Your company’s chief research chemist is killed in a plane crash.

4. Because of unfavorable economic news, consumer confidence is shaken, and the stock market falls by 10%.

5. You, a small business owner, buy a sofa for your home. You pay with a check Dr. awn on your personal, not your business, checking account.

6. Disney decides to build the next Walt Disney World near a large piece of property you own.

when cash basis is different from accrual basis alice guth operates a low impact aer 691741

When Cash Basis Is Different from Accrual Basis

Alice Guth operates a low impact aerobics studio. Alice has been in business for 3 years and has always had her financial statements prepared on a cash basis. This year, Alice’s accountant has suggested that accrual based financial statements would give a more accurate picture of the performance of the business. Alice’s friend Frank Geller tells her that, in his experience, accrual based financial statements tell pretty much the same story as cash basis statements.  Under what circumstances would the cash basis and the accrual basis of accounting yield quite different pictures of a firm’s operating performance? Under what circumstances would the cash basis and the accrual basis show approximately the same picture?

working capital using the following information compute working capital cost of good 691748

Working Capital

Using the following information, compute working capital:

Cost of Goods Sold                                                            

$ 9,000

Accounts Payable                                                             

1,100

Paid In Capital                                                               

1,750

Cash                                                                      

400

Sales                                                                      

10,000

Accrued Wages Payable                                                         

250

Dividends                                                                   

700

Retained Earnings (beginning)                                                     

1,000

Inventory                                                                   

4,000

current assets using the following information compute total current 691749

Current Assets

Using the following information, compute total current assets:

Goodwill                                                                   

$ 9,000

Prepaid Expenses                                                             

1,100

Paid In Capital                                                               

1,750

Cash                                                                      

400

Property, Plant, and Equipment                                                     

10,000

Investment Securities (trading)                                                    

250

Accounts Receivable                                                           

700

Retained Earnings                                                              

1,000

Inventory                                                                   

4,000

current liabilities using the following information compute total current liabilitie 691750

Current Liabilities

Using the following information, compute total current liabilities:

Accrued Income Taxes Payable                                                    

$ 9,000

Notes Payable (due in 14 months)                                                 

1,100

Paid In Capital                                                                

1,750

Treasury Stock                                                               

400

Current Portion of Long Term Debt                                               

10,000

Unearned Revenue                                                            

250

Accounts Payable                                                             

700

Retained Earnings                                                             

1,000

Additional Paid In Capital                                                        

4,000

classification of short term loans to be refinanced the company has the following th 691751

Classification of Short Term Loans to Be Refinanced

The company has the following three loans payable scheduled to be repaid in June of next year. As of December 31 of this year, identify which of the three should be classified as current and which should be classified as noncurrent. 

(a) The company intends to repay Loan A when it comes due in June. In the following September, the company intends to get a new loan of equal amount from the same bank. 

(b) The company intends to refinance Loan B when it comes due. The refinancing contract will be signed in May after the financial statements for this year have been released. 

(c) The company intends to refinance Loan C when it comes due. The refinancing contract will be signed in January before the financial statements for this year have been released.

contingent liabilities the company has the following three potential obligations des 691753

Contingent Liabilities

The company has the following three potential obligations. Describe how each will be reported in the financial statements.

(a) The company has promised to make fixed pension payments to employees after they retire. The company is not certain how long the employees will work or how long they will live after they retire.

(b) The company has been sued by a group of shareholders who claim that they were deceived by the company’s financial reporting practices. It is possible that the company will lose this lawsuit.

(c) The company is involved in litigation over who must clean up a toxic waste site near one of the company’s factories. It is probable, but not certain, that the company will be required to pay for the cleanup.

stockholders rsquo equity using the following information compute total contributed 691754

Stockholders’ Equity

Using the following information, compute: (a) total contributed capital, (b) ending retained earnings, and (c) total stockholders’ equity:

Additional Paid In Capital, Common                                                

$ 9,000

Accounts Payable                                                             

1,100

Total Expenses                                                               

7,800

Preferred Stock, at par                                                         

1,750

Common Stock, at par                                                         

400

Sales                                                                      

10,000

Treasury Stock                                                               

250

Dividends                                                                   

700

Retained Earnings (beginning)                                                     

1,000

Additional Paid In Capital, Preferred                                                

50

stockholders rsquo equity using the following information compute total contributed 691755

Stockholders’ Equity

Using the following information, compute (a) total contributed capital, (b) total accumulated other comprehensive income, and (c) total stockholders’ equity:

Additional Paid In Capital, Common                                                

$9,000

Common Stock, at par                                                         

400

Cumulative Translation Adjustment (equity reduction), ending                              

2,000

Treasury Stock                                                                

700

Retained Earnings (post closing, or ending)                                           

1,500

Cumulative Unrealized Gain on Available for Sale Securities, ending                          

1,100

format of foreign balance sheet following is a balance sheet presented in standard u 691756

Format of Foreign Balance Sheet

Following is a balance sheet presented in standard U.S. format. Rearrange this balance sheet to be in standard British format. Don’t worry about differences in terminology; use the U.S. labels, but present the information in the British format.

Current Assets:

 

Cash                                                                    

$ 500

Inventory                                                                

2,000

Total current assets                                                         

$ 2,500

Noncurrent Assets:

 

Property, plant, and equipment                                                 

$ 8,000

Long term investments                                                       

1,700

Total noncurrent assets                                                       

$ 9,700

Total assets                                                               

$12,200

Current Liabilities:

 

Accounts payable                                                           

$ 300

Short term loans payable                                                     

1,100

Total current liabilities                                                       

$ 1,400

Noncurrent Liabilities:

 

Long term debt                                                             

$ 3,000

Stockholders’ Equity:

 

Common stock, at par                                                       

$ 50

Additional paid in capital                                                     

2,000

Retained earnings                                                          

5,750

Total stockholders’ equity                                                     

$ 7,800

Total liabilities and stockholders’ equity                                           

$12,200

current ratio use the following information to compute the current ratio 691757

Current Ratio

Use the following information to compute the current ratio:

Accounts Payable                                                             

$ 1,100

Paid In Capital                                                               

1,750

Cash                                                                      

400

Sales                                                                      

10,000

Accrued Wages Payable                                                         

250

Inventory                                                                   

4,000

quick ratio use the following information to compute the quick ratio long term 691758

Quick Ratio

Use the following information to compute the quick ratio:

Long Term Loan Payable                                                        

$ 1,100

Accounts Receivable                                                           

1,750

Cash                                                                      

400

Cost of Goods Sold                                                           

10,000

Accrued Wages Payable                                                         

250

Inventory                                                                    

4,000

debt ratio use the information in practice 3 3 to compute the debt ratio assume that 691759

Debt Ratio

Use the information in Practice 3 3 to compute the debt ratio. Assume that the list includes all liability and equity items.

Practice 3 3

Current Liabilities

Using the following information, compute total current liabilities:

Accrued Income Taxes Payable                                                    

$ 9,000

Notes Payable (due in 14 months)                                                 

1,100

Paid In Capital                                                                

1,750

Treasury Stock                                                               

400

Current Portion of Long Term Debt                                               

10,000

Unearned Revenue                                                             

250

Accounts Payable                                                             

700

Retained Earnings                                                             

1,000

Additional Paid In Capital                                                        

4,000

debt ratio use the information in practice 3 7 to compute the debt ratio assume that 691760

Debt Ratio

Use the information in Practice 3 7 to compute the debt ratio. Assume that the list includes all liability and equity items.

Practice 3 7

Stockholders’ Equity

Using the following information, compute: (a) total contributed capital, (b) ending retained earnings, and (c) total stockholders’ equity:

Additional Paid In Capital, Common                                                

$ 9,000

Accounts Payable                                                             

1,100

Total Expenses                                                               

7,800

Preferred Stock, at par                                                         

1,750

Common Stock, at par                                                         

400

Sales                                                                      

10,000

Treasury Stock                                                               

250

Dividends                                                                   

700

Retained Earnings (beginning)                                                     

1,000

Additional Paid In Capital, Preferred                                                 

50

asset mix use the information in practice 3 9 to compute the proportion of total ass 691761

Asset Mix

Use the information in Practice 3 9 to compute the proportion of total assets in each of the following asset categories.

(a) Inventory

(b) Property, Plant, and Equipment

Practice 3 9

Format of Foreign Balance Sheet

Following is a balance sheet presented in standard U.S. format. Rearrange this balance sheet to be in standard British format. Don’t worry about differences in terminology; use the U.S. labels, but present the information in the British format.

Current Assets:

 

Cash                                                                     

$ 500

Inventory                                                                

2,000

Total current assets                                                         

$ 2,500

Noncurrent Assets:

 

Property, plant, and equipment                                                 

$ 8,000

Long term investments                                                       

1,700

Total noncurrent assets                                                      

$ 9,700

Total assets                                                               

$12,200

Current Liabilities:

 

Accounts payable                                                            

$ 300

Short term loans payable                                                     

1,100

Total current liabilities                                                       

$ 1,400

Noncurrent Liabilities:

 

Long term debt                                                            

$ 3,000

Stockholders’ Equity:

 

Common stock, at par                                                       

$ 50

Additional paid in capital                                                      

2,000

Retained earnings                                                          

5,750

Total stockholders’ equity                                                     

$ 7,800

Total liabilities and stockholders’ equity                                           

$12,200

but we need only one accounting standard mdash fairness in the 1970s a leader in the 691698

But We Need Only One Accounting Standard—Fairness

In the 1970s, a leader in the accounting profession proposed that there really needed to be only one underlying standard to govern the establishment of generally accepted accounting principles. That standard was identified as fairness. Financial statements should be prepared so that they are fair to all users: management, labor, investors, creditors. As changes occur in society, financial reporting should change to fairly reflect each user’s needs. Because the financial statements are the responsibility of management, such a standard would require management to determine what reporting methods would be fair. What advantages do you see to this proposal? What would be management’s most serious problem in applying a fairness standard?

deciphering financial statements mcdonald rsquo s corporation the following informat 691701

Deciphering Financial Statements (McDonald’s Corporation)

The following information comes from the 2004 financial statements of McDonald’s Corporation.  Individual franchise arrangements generally include a lease and a license and provide  for payment of initial fees, as well as continuing rent and service fees to the Company  based upon a percent of sales with minimum rent payments that parallel the  Company’s underlying leases and escalations (on properties that are leased).  McDonald’s franchisees are granted the right to operate a restaurant using the McDonald’s System and, in most cases, the use of a restaurant facility, generally for a period of 20 years. Franchisees pay related occupancy costs including property taxes, insurance, and maintenance. In addition, franchisees outside the United States generally pay a refundable, non interest bearing security deposit. Foreign affiliates and developmental licensees pay a royalty to the Company based upon a percent of sales.  The results of operations of restaurant businesses purchased and sold in transactions with franchisees, affiliates, and others were not material to the consolidated financial statements for periods prior to purchase and sale. Revenues from franchised and affiliated restaurants consisted of the following:

(In millions)

2004

2003

2002

Rents and service fees                       

$48,048.00

$43,021.00

$3,855.00

Initial fees                                

36.1

43.0

51.1

Revenues from franchised and affiliated restaurants

$48,409

$43,451

$39,061

 

Future minimum rent payments due to the Company under existing franchise arrangements are as follows:

 

(In millions)

Owned Sites

Leased Sites

Total

2005

$10,634.00

$8,117.00

$18,751.00

2006

10,389.00

790.3

18,292.00

2007

10,067.00

772.1

17,788.00

2008

972.2

751.3

17,235.00

2009

933.0

722.9

16,559.00

Thereafter

72,417.00

55,317.00

127,734.00

Total minimum payments

$122,559.00

$9,380.00

$216,359.00

This $21.6 billion amount represents the future minimum payments that McDonald’s expected to receive from its franchisees as of December 31, 2004.

1. Using the element definition from the conceptual framework, should this $21.6 billion be recorded as an asset in McDonald’s 2004 balance sheet? Why or why not?

2. If your answer in part (1) is yes, what measurement attributes should be used in reporting the asset?

researching accounting standards to help you become familiar with the accounting sta 691703

Researching Accounting Standards

To help you become familiar with the accounting standards, this case is designed to take you to the FASB’s Web site and have you access various publications. Click on“ FASB Pronouncements.”

For this case, we will use Statement of Financial Accounting Concepts No. 1. Open Concepts Statement No. 1.

1. Read paragraph 28. Based on information in this paragraph, what group of users benefits most from financial information and why?

2. Read paragraph 34. Based on information in this paragraph, what is assumed about the background and/or education of those who are using financial accounting information?

3. Read paragraph 43.Based on information in this paragraph, those interested in an enterprise’s future cash flows should pay particular attention to information contained in which primary financial statement?

posting the beginning balance in the accounts payable account was 8 000 during the m 691706

Posting

The beginning balance in the accounts payable account was $8,000.During the month, the following four journal entries (involving accounts payable) were recorded:

a.

Inventory

2,775

 

 

Accounts Payable

 

2,700

b.

Accounts Payable

6,500

 

 

Cash 

 

6,500

c.

Accounts Payable

200

 

 

Inventory

 

200

d.

Inventory

3,000

 

 

Cash

 

450

 

Accounts Payable

 

2,550

Create an Accounts Payable T account and post the entries to this account. Compute an ending balance.

income statement prepare two income statements one using the information in practice 691709

Income Statement

Prepare two income statements, one using the information in Practice 2–8 and the other using the information in Practice 2–9.

Practice 2–8

Trial Balance

Use the following account balance information to construct a trial balance:

Cost of Goods Sold

$ 9,000

Accounts Payable

1,100

Paid In Capital

2,000

Cash       

400

Sales

10,000

Dividends

700

Retained Earnings (beginning)

1,000

Inventory

4,000

Practice 2–9

Trial Balance

Use the following account balance information to construct a trial balance:

Salary Expense

$18,000

Unearned Service Revenue

4,700

Paid In Capital

2,000

Cash

800

Service Revenue

20,000

Rent Expense

6,400

Retained Earnings (beginning)

1,500

Prepaid Rent Expense

3,000

balance sheet prepare two balance sheets one using the information in practice 2 nda 691710

Balance Sheet

Prepare two balance sheets, one using the information in Practice 2–8 and the other using the information in Practice 2–9.

Practice 2–8

Trial Balance

Use the following account balance information to construct a trial balance:

Cost of Goods Sold

$ 9,000

Accounts Payable

1,100

Paid In Capital

2,000

Cash       

400

Sales

10,000

Dividends

700

Retained Earnings (beginning)

1,000

Inventory

4,000

Practice 2–9

Trial Balance

Use the following account balance information to construct a trial balance:

Salary Expense

$18,000

Unearned Service Revenue

4,700

Paid In Capital

2,000

Cash

800

Service Revenue

20,000

Rent Expense

6,400

Retained Earnings (beginning)

1,500

Prepaid Rent Expense

3,000

recording transactions in t accounts georgia supply corporation a merchandising firm 691714

Recording Transactions in T Accounts

Georgia Supply Corporation, a merchandising firm, prepared the following trial balance as of October 1:

 

Debit

Credit

Cash              

$150,000

 

Accounts Receivable   

21,540

 

Inventory           

32,680

 

Land              

15,400

 

Building            

14,000

 

Accounts Payable     

 

$ 9,190

Mortgage Payable     

 

23,700

Common Stock      

 

140,000

Retained Earnings

 

60,730

Totals            

$233,620

$233,620

Georgia Supply engaged in the following transactions during October 2008. The company records inventory using the perpetual system.

Oct. 1 Sold merchandise on account to the Tracker Corporation for $12,000; terms 2/10, n/30, FOB shipping point. Tracker paid $350 freight on the goods. The merchandise cost $6,850.

5 Purchased inventory costing $10,250 on account; terms n/30.

7 Received payment from Tracker for goods shipped October 1.

15 The payroll paid for the first half of October was $22,000. (Ignore payroll taxes.)

18 Purchased a machine for $8,600 cash.

22 Declared a dividend of $0.45 per share on 45,000 shares of common stock outstanding.

27 Purchased building and land for $125,000 in cash and a $225,000 mortgage payable, due in 30 years. The land was appraised at $150,000 and the building at $300,000.

1. Prepare T accounts for all items in the October 1 trial balance and enter the initial balances.

2. Record the October transactions directly to the T accounts.

3. Prepare a new trial balance as of the end of October.

adjusting and correcting entries upon inspecting the books and records for wernli co 691716

Adjusting and Correcting Entries

Upon inspecting the books and records for Wernli Company for the year ended December

31, 2008, you find the following data:

(a) A receivable of $640 from Hatch Realty is determined to be uncollectible. The company maintains an allowance for bad debts for such losses.

(b) A creditor, E. F. Bowcutt Co., has just been awarded damages of $3,500 as a result of breach of contract by Wernli Company during the current year. Nothing appears on the books in connection with this matter.

(c) A fire destroyed part of a branch office. Furniture and fixtures that cost $12,300 and had a book value of $8,200 at the time of the fire were completely destroyed. The insurance company has agreed to pay $7,000 under the provisions of the fire insurance policy.

(d) Advances of $950 to salespersons have been previously recorded as sales salaries expense.

(e) Machinery at the end of the year shows a balance of $19,960. It is discovered that additions to this account during the year totaled $4,460, but of this amount, $760 should have been recorded as repairs. Depreciation is to be recorded at 10% on machinery owned throughout the year but at one half this rate on machinery purchased or sold during the year.

Record the entries required to adjust and correct the accounts. (Ignore income tax consequences.)

reconstructing adjusting entries for each situation reconstruct the adjusting entry 691717

Reconstructing Adjusting Entries

For each situation, reconstruct the adjusting entry that was made to arrive at the ending balance. Assume statements and adjusting entries are prepared only once each year.

1.

Prepaid Insurance:

 

 

Balance beginning of year

$5,600

 

Balance end of year

6,400

During the year, an additional business insurance policy was purchased. A 2–year premium of $2,500 was paid and charged to Prepaid Insurance.

2.

Accumulated Depreciation:

 

 

Balance beginning of year

$85,200

 

Balance end of year

88,700

During the year, a depreciable asset that cost $7,500 and had a carrying value of $1,600 was sold for $2,400.The disposal of the asset was recorded correctly.

3.

Unearned Rent:

 

 

Balance beginning of year

$11,000

 

Balance end of year

15,000

Warehouse quarterly rent received in advance is $18,000. During the year, equipment was rented to another company at an annual rent of $9,000. The quarterly rent payments were credited to Rent Revenue; the annual equipment rental was credited to Unearned Rent.

4.

Salaries Payable:

 

 

Balance beginning of year

$42,860

 

Balance end of year

34,760

Salaries are paid biweekly. All salary payments during the year were debited to Salaries Expense.

adjusting and closing entries and post closing trial balance accounts of pioneer hea 691718

Adjusting and Closing Entries and Post Closing Trial Balance

Accounts of Pioneer Heating Corporation at the end of the first year of operations showed the following balances. In addition, prepaid operating expenses are $4,000, and accrued sales commissions payable are $5,900. Investment revenue receivable is $1,000. Depreciation for the year on buildings is $4,500 and on machinery, $5,000. Federal and state income taxes for the year are estimated at $18,100.

 

Debit

Credit

Cash                

$ 39,000

 

Inventory             

50,000

 

Investment            

50,000

 

Land                

70,000

 

Buildings              

180,000

 

Machinery            

100,000

 

Accounts Payable       

 

$ 65,000

Common Stock        

 

320,000

Additional Paid In Capital 

 

40,000

Sales                

 

590,000

Cost of Goods Sold     

230,000

 

Sales Commissions      

200,000

 

General Operating Expenses

101,000

 

Investment Revenue     

 

5,000

Totals              

$1,020,000

$1,020,000

1. Prepare the necessary entries to adjust and close the books.

2. Prepare a post closing trial balance.

analysis of journal entries for each of the following journal entries write a descri 691720

Analysis of Journal Entries

For each of the following journal entries, write a description of the underlying event.

1.

Cash              

 300

 

 

Accounts Receivable 

 

 300

2.

Accounts Payable     

 400

 

 

Inventory         

 

 400

3.

Cash              

 5,000

 

 

Loan Payable       

 

 5,000

4.

Cash              

 200

 

 

Accounts Receivable   

 700

 

 

Sales             

 

 900

 

Cost of Goods Sold   

 550

 

 

Inventory         

 

550 

5.

Prepaid Insurance     

 200

 

 

Cash             

 

 200

6.

Dividends           

 250

 

 

Dividends Payable   

 

 250

7.

Retained Earnings     

 1,000

 

 

Dividends         

 

 1,000

8.

Insurance Expense     

 50

 

 

Prepaid Insurance   

 

 50

9.

Inventory           

 600

 

 

Cash             

 

 150

 

Accounts Payable    

 

 450

10.

Allowance for Bad Debts

 46

 

 

Accounts Receivable 

 

 46

11.

Interest Expense   

 125

 

 

Interest Payable  

 

 125

12.

Wages Payable    

 130

 

 

Wages Expense    

 75

 

 

Cash          

 

 205

13.

Accounts Payable  

 500

 

 

Cash          

 

 490

 

Purchase Discounts

 

 10

analyzing adjusting entries guide com consulting company initially records prepaid i 691723

Analyzing Adjusting Entries

Guide com Consulting Company initially records prepaid items as assets and unearned items as liabilities. Selected account balances at the end of the current and prior year follow. Accrued expenses and revenues are adjusted only at year end.

 

Adjusted Balances,
December 31, 2007

Adjusted Balances,
December 31, 2008

Prepaid Rent

$ 5,100

$3,400

Salaries and Wages Payable

2,100

4,700

Unearned Consulting Fees

18,200

7,800

Interest Receivable

800

2,100

During 2008, Guide com Consulting paid $14,000 for rent and $40,000 for wages. It received $112,000 for consulting fees and $3,200 as interest.

1. Provide the entries that were made at December 31, 2008, to adjust the accounts to the year end balances shown above.

2. Determine the proper amount of Rent Expense, Salaries and Wages Expense, Consulting Fees Revenue, and Interest Revenue to be reported on the current year income statement.

closing entries an accountant for jolley inc a merchandising enterprise has just fin 691724

Closing Entries

An accountant for Jolley, Inc., a merchandising enterprise, has just finished posting all yearend adjusting entries to the ledger accounts and now wishes to close the appropriate account balances in preparation for the new period.

1. For each of the accounts listed, indicate whether the year end balance should be

(a) carried forward to the new period, (b) closed by debiting the account, or (c) closed by crediting the account.

(a)

Cash

$ 25,000

(b)

Sales

75,000

(c)

Dividends

3,500

(d)

Inventory

7,500

(e)

Selling Expenses

7,900

(f)

Capital Stock

100,000

(g)

Wages Expense

14,400

(h)

Dividends Payable

4,000

(i)

Cost of Goods Sold

26,500

(j)

Accounts Payable

12,000

(k)

Accounts Receivable

140,000

(l)

Prepaid Insurance

16,000

(m)

Interest Receivable

1,500

(n)

Sales Discounts

4,200

(o)

Interest Revenue

6,500

(p)

Supplies

8,000

(q)

Retained Earnings

6,500

(r)

Accumulated Depreciation

2,000

(s)

Depreciation Expense

1,800

2. Give the necessary closing entries.

3. What was Jolley’s net income (loss) for the period?

accrual errors loring tools inc failed to make year end adjustments to record accrue 691727

Accrual Errors

Loring Tools, Inc., failed to make year end adjustments to record accrued salaries and recognize interest receivable on investments over the last three years as follows:

 

2006

2007

2008

Accrued salaries

$25,000

$19,000

$32,000

Interest receivable

10,500

8,500

13,200

What impact would the correction of these errors have on the net income for these three years? Ignore income taxes.

journal entries selfish gene company is a merchandising firm the following events oc 691728

Journal Entries

Selfish Gene Company is a merchandising firm. The following events occurred during the month of May. (Note: Selfish Gene maintains a perpetual inventory system.)

May 1 Received $40,000 cash as new stockholder investment.

3 Purchased inventory costing $8,000 on account from Dawkins Company; terms 2/10, n/30.

4 Purchased office supplies for $500 cash.

4 Held an office party for the retiring accountant. Balloons, hats, and refreshments cost $150 and were paid for with office staff contributions.

5 Sold merchandise costing $7,500 on account for $14,000 to Richard Company; terms 3/15, n/30.

8 Paid employee wages of $2,000. Gross wages were $2,450; taxes totaling $450 were withheld.

9 Hired a new accountant; agreed to a first year salary of $28,000.

9 Paid $1,500 for newspaper advertising.

10 Received payment from Richard Company.

12 Purchased a machine for $6,400 cash.

May 15 Declared a cash dividend totaling $25,000.

18 Sold merchandise costing $13,000 for $3,000 cash and $21,000 on account to Feynman Company; terms n/30.

19 Paid Dawkins Company account in full.

22 Company executives appeared on the cover of a national newsmagazine.

Related article extolled Selfish Gene’s labor practices, environmental concerns, and customer service.

23 Market value of Selfish Gene’s common stock rose by $150,000.

25 Purchased a building for $15,000 cash and a $135,000 mortgage payable.

29 Paid dividends declared on May 15.

Instructions:

1. Record the preceding events in general journal form.

2. Which event do you think had the most significant economic impact on Selfish Gene Company? Are all economically relevant events recorded in the financial records?

adjusting entries on december 31 wright company noted the following transactions tha 691730

Adjusting Entries

On December 31, Wright Company noted the following transactions that occurred during

2008, some or all of which might require adjustment to the books.

(a) Payment of $3,100 to suppliers was made for purchases on account during the year and was not recorded.

(b) Building and land were purchased on January 2 for $210,000.The building’s fair market value was $150,000 at the time of purchase. The building is being depreciated over a 30 year life using the straight line method, assuming no salvage value.

(c) Of the $40,000 in Accounts Receivable, 5% is estimated to be uncollectible. Currently,

Allowance for Bad Debts shows a debit balance of $350.

(d) On August 1, $60,000 was loaned to a customer on a 12 month note with interest at an annual rate of 12%.

(e) During 2008, Wright received $12,500 in advance for services, 80% of which will be performed in 2009.The $12,500 was credited to sales revenue.

(f) The interest expense account was debited for all interest charges incurred during the year and shows a balance of $1,400. However, of this amount, $500 represents a discount on a 60 day note payable, due January 30, 2009.

Instructions:

1. Give the necessary adjusting entries to bring the books up to date.

2. Indicate the net change in income as a result of the foregoing adjustments.

adjusting entries the bookkeeper for allen wholesale electric co records all revenue 691732

Adjusting Entries

The bookkeeper for Allen Wholesale Electric Co. records all revenue and expense items in nominal accounts during the period. The following balances, among others, are listed on the trial balance at the end of the fiscal period, December 31, 2008, before accounts have been adjusted:

 

Dr.. (Cr.)

Accounts Receivable

$148,000

Allowance for Bad Debts

(3,000)

Interest Receivable

2,300

Discount on Notes Payable

400

Prepaid Real Estate and Personal Property Tax

1,700

Salaries and Wages Payable

(5,200)

Discount on Notes Receivable

(2,600)

Unearned Rent Revenue

(3,300)

Inspection of the company’s records reveals the following as of December 31, 2008:

(a) Uncollectible accounts are estimated at 4% of the accounts receivable balance.

(b) The accrued interest on investments totals $2,900.

(c) The company borrows cash by discounting its own notes at the bank. Discounts on notes payable at the end of 2008 are $1,100.

(d) Prepaid real estate and personal property taxes are $1,700,the same as at the end of 2007.

(e) Accrued salaries and wages are $6,700.

(f) The company accepts notes from customers, giving its customers credit for the face of the note less a charge for interest. At the end of each period, any interest applicable to the succeeding period is reported as a discount. Discounts on notes receivable at the end of 2008 are $1,800.

(g) Part of the company’s properties had been sublet on September 15, 2007, at a rental of $2,500 per month. The arrangement was terminated at the end of one year.

Instructions: Give the adjusting entries required to bring the books up to date.

internet use is very important to a number of segments of the hospitality industry e 691639

Internet use is very important to a number of segments of the hospitality industry, especially for lodging and travel booking. Increasingly, more and more restaurants also employ the Internet for reservations. Some fast food places even offer “ordering” for pick up or delivery through the Internet. It’s no wonder that the proper Internet domain name can make or break a company and that buying a domain name makes a hot market. While the cost of registration is negligible, if a company has to purchase its name from a cyber squatter—people who register names in the hopes of selling them for a profit—the cost can rise quickly. When eBay, Inc., the world’s largest online auction house, recently tried to register www.ebay.ca in Canada, it discovered that the name had been registered previously by an entrepreneur. eBay then had two options to consider. Since eBay is a registered trademark around the world, (1) the company could take legal action; or (2) it could negotiate the domain name www.ebaycanada.ca, which also had been registered previously by a self described “Internet entrepreneur.” This entrepreneur said that he hoped to make some quick money when he registered www.ebaycanada.ca. He eventually gave up the name without a fight rather than go to court and face high legal bills.

on january 1 2008 skyline hotel co purchased a passenger van for transporting guests 691641

On January 1, 2008, Skyline Hotel Co. purchased a passenger van for transporting guests to and from airports and nearby shopping areas at an acquisition cost of $28,000. The vehicle has been depreciated by the straight line method using a four year service life and a $4,000 salvage value. The company’s fiscal year ends on December 31.

Instructions

Prepare the journal entry or entries to record the disposal of the van assuming that it was

(a) Retired and scrapped with no salvage value on January 1, 2012.

(b) Sold for $5,000 on July 1, 2011.

(c) Traded in on a new van on January 1, 2011. The fair market value of the old vehicle was $9,000, and $22,000 was paid in cash.

(d) Traded in on a new van on January 1, 2011. The fair market value of the old vehicle was $11,000, and $22,000 was paid in cash.

mia fitness club was organized on january 1 during the first year of operations the 691648

Mia Fitness Club was organized on January 1. During the first year of operations, the following plant asset expenditures and receipts were recorded in random order

Debits

1. Accrued real estate taxes paid at time of purchase of real estate

$ 2,000

2. Real estate taxes on land paid for the current year

3,000

3. Full payment to building contractor

600,000

4. Excavation costs for new building

25,000

5. Cost of real estate purchased as a plant site (land $100,000 and

building $25,000)

125,000

6. Cost of parking lots and driveways

15,000

7. Architect’s fees on building plans

10,000

8. Installation cost of fences around property

4,000

9. Cost of demolishing building to make land suitable for construction

of new building

21,000

 

$805,000

Credit

10. Proceeds from salvage of demolished building

$ 2,500

Instructions

Analyze the foregoing transactions using the following column headings. Insert the number of each transaction in the item space, and insert the amounts in the appropriate columns. For amounts in the Other Accounts column, also indicate the account title. Item Land Building Other Accounts

at the beginning of 2008 bellamy seafood acquired equipment costing 60 000 it was es 691649

At the beginning of 2008, Bellamy Seafood acquired equipment costing $60,000. It was estimated that this equipment would have a useful life of six years and a residual value of $6,000 at that time. The straight line method of depreciation was considered the most appropriate to use with this type of equipment. Depreciation is to be recorded at the end of each year. During 2010 (the third year of the equipment’s life), the company’s engineers reconsidered their expectations and estimated that the equipment’s useful life would probably be seven years (in total) instead of six years. The estimated residual value was not changed at that time. However, during 2013 the estimated residual value was reduced to $3,000.

Instructions

Indicate how much depreciation expense should be recorded for this equipment each year by completing the following table.

Year

Depreciation Expense

Accumulated Depreciation

2008

 

 

2009

 

 

2010

 

 

2011

 

 

2012

 

 

2013

 

 

2014

 

 

foxx saber resorts was organized on january 1 during the first year of operations th 691650

Foxx Saber Resorts was organized on January 1. During the first year of operations, the following plant asset expenditures and receipts were recorded in random order.

Debits

 

1. Cost of filling and grading the land

$ 4,000

2. Full payment to building contractor

700,000

3. Real estate taxes on land paid for the current year

5,000

4. Cost of real estate purchased as a plant site (land $100,000 and building  $45,000

145,000

5. Excavation costs for new building

30,000

6. Architect’s fees on building plans

10,000

7. Accrued real estate taxes paid at time of purchase of real estate

2,000

8. Cost of parking lots and driveways

14,000

9. Cost of demolishing building to make land suitable for construction of new  building

20,000

Credit

10. Proceeds from salvage of demolished building

$ 3,500

Instructions

Analyze the foregoing transactions using the following column headings. Insert the number of each transaction in the Item space, and insert the amounts in the appropriate columns. For amounts entered in the Other Accounts column, also indicate the account titles.

when a group of investors in a company is unhappy with a company rsquo s performance 691652

When a group of investors in a company is unhappy with a company’s performance, it sometimes tries to elect new members to the board of directors at the company’s annual stockholder meeting. This is referred to as a proxy fight. Usually these efforts fail because it has been very expensive to get in contact with all of the company’s shareholders to try to convince them to vote for your group of nominees.But the Internet has changed that, says James Heard, chief executive of Proxy Monitor, a New York firm that consults institutional shareholders on how to vote on corporate governance issues. “Increasingly the Internet is being used as a tool of communication among shareholders to pressure managements,” he said. One recent case involved an effort by a shareholder at Lube’s to get four new people elected to that company’s board of directors. That shareholder attracted considerable support from other Lube’s shareholders by posting messages on a Yahoo! message board.

in the wake of enron rsquo s collapse the members of enron rsquo s board of director 691653

In the wake of Enron’s collapse, the members of Enron’s board of directors have been questioned and scrutinized to determine what they knew and when they knew it. A Wall Street Journal story reported that Enron’s board contends it was “kept in the dark” by management and by Arthur Andersen—Enron’s longtime auditors—and didn’t learn about the company’s troublesome accounting until October 2001. But the Wall Street Journal reported that according to outside attorneys, “directors on at least two occasions waived Enron’s ethical code of conduct to approve partnership between Enron and its chief financial officer. That partnership kept significant debt off of Enron’s books and masked actual company finances.” Was Enron’s board of directors fulfilling its role in a corporate organization when it waived Enron’s ethical code on two occasions?

it is not necessary for a corporation to have an office in the state in which it inc 691654

It is not necessary for a corporation to have an office in the state in which it incorporates. In fact, more than 50 percent of the Fortune 500 corporations are incorporated in Delaware. A primary reason is the Delaware courts’ long standing “business judgment rule.” The rule provides that as long as directors exercise “due care” in the interests of stockholders, their actions will not be second guessed by the courts. The rule has enabled directors to reject hostile takeover offers and to spurn takeovers simply because they did not want to sell the company. However, new interpretations are emerging. In a recent case, the state court ruled for a company that made a hostile takeover bid. On appeal, the Delaware Supreme Court ruled for the directors but gave the following guideline to the state courts: “Was the board’s response reasonable in the light of the threat posed?”

the holman hotel corporation is authorized to issue 1 million shares of 5 par value 691666

The Holman Hotel Corporation is authorized to issue 1 million shares of $5 par value common stock. In its first year, 2008, the company has the following stock transactions:

Jan. 10

Issued 400,000 shares of stock at $8 per share.

July 1

Issued 100,000 shares of stock for land. The land had an asking price of $900,000. The stock is currently selling on a national exchange at $8.25 per share

Sept. 1

Purchased 10,000 shares of common stock for the treasury at $9 per share.

Dec. 1

Sold 4,000 shares of the treasury stock at $10 per share.

Instructions

(a) Journalize the transactions.

(b) Prepare the stockholders’ equity section assuming the company had retained earnings of $200,000 at December 31, 2008.

on june 1 meyers hotel corporation had 80 000 shares of no par common stock issued a 691672

On June 1, Meyers Hotel Corporation had 80,000 shares of no par common stock issued and outstanding. The stock has a stated value of $10 per share. During the year, the following occurred:

 

Jul. 1

Issued 10,000 additional shares of common stock.

Aug. 1

Declared a cash dividend of $1.50 per share to stockholders of record on Au gust 30.

Oct. 1

Paid the $1.50 per share cash dividend.

Nov. 3

Issued 2,500 additional shares of common stock.

Dec. 3

Declared a cash dividend on outstanding shares of $1.60 per share to stock holders of record on December 31.

Instructions

(a) Prepare the entries, if any, on each of the three dividend dates.

(b) How are dividends and dividends payable reported in the financial statements prepared at December 31?

aspects of the fasb rsquo s conceptual framework determine whether the following sta 691676

Aspects of the FASB’s Conceptual Framework

Determine whether the following statements are true or false. If a statement is false, explain why.

1. Comprehensive income includes changes in equity resulting from distributions to owners.

2. Timeliness and predictive value are both characteristics of relevant information.

3. The tendency to recognize favorable events early is an example of conservatism.

4. The conceptual framework focuses primarily on the needs of internal users of financial information.

5. The seven Statements of Financial Accounting Concepts are considered part of generally accepted accounting principles.

6. The overriding objective of financial reporting is to provide information for making economic decisions.

7. The term recognition is synonymous with the term disclosure.

8. Once an accounting method is adopted, it should never be changed.

conceptual framework terminology match the numbered statements below with the letter 691677

Conceptual Framework Terminology

Match the numbered statements below with the lettered terms. An answer (letter) may be used more than once, and some terms require more than one answer (letter).

1. Key ingredients in quality of relevance.

2. Traditional assumptions that influence the FASB’s conceptual framework.

3. The idea that information should represent what it purports to represent.

4. An important constraint relating to costs and benefits.

5. An example of conservatism.

6. The availability of information when it is needed.

7. Recording an item in the accounting records.

8. Determines the threshold for recognition.

9. Implies consensus.

10. Transactions between independent parties.

(a) Cost effectiveness

(b) Representational faithfulness

(c) Recognition

(d) Verifiability

(e) Time periods

(f) Unrealized

(g) Completeness

(h) Timeliness

(i) Materiality

(j) Predictive value

(k) Economic entity

(l) Lower of cost or market rule

(m) Phrenology

(n) Arm’s length transactions

objectives of financial reporting for each of the following independent situations i 691678

Objectives of Financial Reporting

For each of the following independent situations, identify the relevant objective(s) of financial reporting that the company could be overlooking. Discuss each of these objectives.

1. The president of Coventry, Inc., believes that the financial statements should be prepared for use by management only, because they are the primary decision makers.

2. Cascade Carpets Co. believes that financial statements should reflect only the present financial standing and cash position of the firm and should not provide any future oriented data.

3. The vice president of Share Enterprises, Inc., believes that the financial statements are to present only current year revenues and expenses, not to disclose assets, liabilities, and owners’ equity.

4. Cruz Co. has a policy of providing disclosures of only its assets, liabilities, and owner ’equity.

5. Marty Manufacturing, Inc., always discloses the assets, liabilities, and owners’ equity of the firm along with the revenues and expenses. Marty’s management believes that these items provide all of the information relevant to investing decisions.

applications of accounting characteristics and concepts for each situation listed in 691679

Applications of Accounting Characteristics and Concepts

For each situation listed, indicate by letter the appropriate qualitative characteristic(s) or accounting concept(s) applied. A letter may be used more than once, and more than one characteristic or concept may apply to a particular situation.

1. Goodwill is recorded in the accounts only when it arises from the purchase of another entity at a price higher than the fair market value of the purchased entity’s identifiable assets.

2. Land is valued at cost.

3. All payments out of petty cash are debited to Miscellaneous Expense.

4. Plant assets are classified separately as land or buildings, with an accumulated depreciation account for buildings.

5. Periodic payments of $1,500 per month for services of H. Hay, who is the sole proprietor of the company, are reported as with Dr. awals.

6. Small tools used by a large manufacturing firm are recorded as expenses when purchased.

7. Investments in equity securities are initially recorded at cost.

8. A retail store estimates inventory rather than taking a complete physical count for purposes of preparing monthly financial statements.

9. A note describing the company’s possible liability in a lawsuit is included with the financial statements even though no formal liability exists at the balance sheet date.

10. Depreciation on plant assets is consistently computed each year by the straight line method.

(a) Understandability

(b) Verifiability

(c) Timeliness

(d) Representational faithfulness

(e) Neutrality

(f) Relevance

(g) Going concern

(h) Economic entity

(i) Historical cost

(j) Measurability

(k) Materiality

(l) Comparability

elements of financial reporting for each of the following items identify the financi 691681

Elements of Financial Reporting

For each of the following items, identify the financial statement element being discussed.

1. Changes in equity during a period except those resulting from investments by owners and distributions to owners.

2. The net assets of an entity.

3. The result of a transaction requiring the future transfer of assets to other entities.

4. An increase in assets from the delivery of goods that constitutes the entity’s ongoing central operations.

5. An increase in an entity’s net assets from incidental transactions.

6. An increase in net assets through the issuance of stock.

7. Decreases in net assets from peripheral transactions of an enterprise.

8. The payment of a dividend.

9. Outflows of assets from the delivery of goods or services.

10. Items offering future value to an entity.

assumptions of financial reporting in each of the following independent situations a 691682

Assumptions of Financial Reporting

In each of the following independent situations, an example is given involving one of the five traditional assumptions of the accounting model. For each situation, identify the assumption involved (briefly explain your answer). 

1. A subsidiary of Parent Inc. was exhibiting poor earnings performance for the year. In an effort to increase the subsidiary’s reported earnings, Parent Inc. purchased products from the subsidiary at twice the normal markup. 

2. When preparing the financial statements for Mac Neil & Sons, the accountant included certain personal assets of Mac Neil and his sons. 

3. The operations of Uintah Savings & Loan are being evaluated by the federal government.  During their investigations, government officials have determined that numerous loans made by top management were unwise and have seriously endangered the future existence of the savings and loan. 

4. Pine Valley Ski Resort has experienced a Dr. astic reduction in revenues because of light snowfall for the year. Rather than produce financial statements at the end of the fiscal year, as is traditionally done, management has elected to wait until next year and present results for a two year period. 

5. Colobri Inc. has equipment that was purchased in 1996 at a cost of $150,000. Because of inflation, that same equipment, if purchased today, would cost $225,000. Management  would like to report the asset on the balance sheet at its current value.

sample cpa exam questions 1 one of the elements on a financial statement is comprehe 691684

Sample CPA Exam Questions

1. One of the elements on a financial statement is comprehensive income. Comprehensive income excludes changes in equity resulting from which of the following?

(a) Loss from discontinued operations

(b) Prior period error correction

(c) Dividends paid to stockholders

(d) Unrealized loss on investments in noncurrent marketable equity securities

2. According to the FASB conceptual framework, the objectives of financial reporting for business enterprises are based on

(a) generally accepted accounting principles.

(b) reporting on management’s stewardship.

(c) the need for conservatism.

(d) the needs of users of the information.

3. Statements of financial accounting concepts are intended to establish

(a) generally accepted accounting principles in financial reporting by business enterprises.

(b) the meaning of “Present fairly in accordance with generally accepted accounting principles.”

(c) the objectives and concepts for use in developing standards of financial accounting and reporting.

(d) the hierarchy of sources of generally accepted accounting principles.

4. According to statements of financial accounting concepts, neutrality is an ingredient of reliability? Of relevance?

(a) Yes Yes

(b) Yes No

(c) No Yes

(d) No No

5. According to the FASB conceptual framework, which of the following statements conforms to the realization concept?

(a) Equipment depreciation was assigned to a production department and then to product unit costs.

(b) Depreciated equipment was sold in exchange for a note receivable.

(c) Cash was collected on accounts receivable.

(d) Product unit costs were assigned to cost of goods sold when the units were sold.

the advantage of internal users emilio valdez worked for several years as a loan ana 691686

The Advantage of Internal Users

Emilio Valdez worked for several years as a loan analyst for a large bank. He recently left the bank and took a management position with Positron, a high tech manufacturing firm. Emilio prepared for his first management meeting by extensively analyzing Positron’s external financial statements. However, in the meeting, the other managers referred to lots of information that Emilio hadn’t found in the financial statements. In addition to using the financial statements, the other managers were using computer printouts and reports unlike anything Emilio had seen in his years at the bank. After the meeting, Monique Vo, one of Emilio’s associates, offered the following advice:“ Emilio, you have to remember that you are an internal user now, not an external user.” What does Monique mean?

we aren rsquo t getting what we expect quality enterprises inc issued its 2007 finan 691687

We Aren’t Getting What We Expect

Quality Enterprises Inc. issued its 2007 financial statements on February 22, 2008. The auditors expressed a “clean” opinion in the audit report. On July 14, 2008, the company filed for bankruptcy as a result of the inability to meet currently maturing long term debt obligations.  Reasons cited for the action include (1) large losses on inventory due to overproduction of product lines that did not sell, (2) failure to collect on a large account receivable due to the customer’s bankruptcy, and (3) a deteriorating economic environment caused by a  severe recession in the spring of 2008. Joan Stevens, a large stockholder with a large number of Quality shares, is concerned about the fact that a company with a clean audit opinion could have financial difficulty leading to bankruptcy just four months after the audit report was issued.“ Where were the auditors?” she inquired. In reply, the auditors contend that on December 31, 2007, the date of the financial statements, the statements were presented in accordance with GAAP. What is an auditor’s responsibility for protecting users from losses? Are auditors and investors in agreement on what an audit should provide?

let rsquo s play by the irs rules little attempt is made to reconcile the accounting 691692

Let’s Play by the IRS Rules

Little attempt is made to reconcile the accounting standard differences between the IRS and the FASB. These differences are recognized as arising from differences in the objectives of the two bodies. However, the existence of differences requires companies to keep two different sets of records in some areas: records that follow the FASB pronouncements and those that follow the IRS rules and regulations.  In many foreign countries, such as Japan and Germany, the financial accounting standards closely follow the tax rules established by the respective government. What applies for taxes often applies for the balance sheet and the income statement as well. Should the United States follow the practice of many foreign competitors? What are the advantages of merging accounting standards for taxes and financial reporting? What are the disadvantages? What would it take to change a system so deeply ingrained in the business fabric of either the United States or other countries?

harry smith owns and manages harry rsquo s restaurant a twenty four hour restaurant 691602

Harry Smith owns and manages Harry’s Restaurant, a twenty four hour restaurant near the city’s medical complex. Harry employs nine full time employees and sixteen part time employees. He pays all the full time employees by check, the amounts of which are determined by Harry’s public accountant, Pam Web. Harry pays all his part time employees in cash. He computes their wages and withdraws the cash directly from his cash register. Pam has repeatedly urged Harry to pay all employees by check. But, as Harry has told his competitor and friend, Steve Hill, who owns the Greasy Diner, “First of all, my part time employees prefer the cash over a check; and second, I don’t withhold or pay any taxes or workmen’s compensation insurance on those wages because they go totally unrecorded and unnoticed.”

Instructions

(a) Who are the stakeholders in this situation?

(b) What are the legal and ethical considerations regarding Harry’s handling of his payroll?

(c) Pam Web is aware of Harry’s payment of the part time payroll in cash. What are her ethical responsibilities in this case?

(d) What internal control principle is violated in this payroll process?

interest rates on most credit cards are quite high averaging 18 8 percent as a resul 691603

Interest rates on most credit cards are quite high, averaging 18.8 percent. As a result, consumers often look for companies that charge lower rates. Be careful—some companies offer lower interest rates but have eliminated the standard 25 day grace period before finance charges are incurred. Other companies encourage consumers to get more in debt by advertising that only a $1 minimum payment is due on a $1,000 account balance. The less you pay off, the more interest they earn! One bank markets a credit card that allows cardholders to skip a payment twice a year. However, the outstanding balance continues to incur interest. Other credit card companies calculate finance charges initially on two month, rather than one month, averages, a practice that often translates into higher interest charges. In short, read the fine print.

in the hotel industry we use two terms to describe trade receivables mdash guest led 691604

In the hotel industry, we use two terms to describe trade receivables—guest ledger and city ledger. The guest ledger is a list of all outstanding amounts (receivables) owed to the hotel by the guest who did not depart yet, and the city ledger is a list of all outstanding amounts owed by the departed guest who has billing privileges. Usually, the guest ledger is not a challenge for the simple reason that most outstanding amounts will be paid by credit card or cash once the guest checks out. A few accounts will be transferred to the city ledger. Accordingly, most of our efforts are used to control the outstanding receivables in the city ledger. For the same reason, in the hotel business, city ledger and trade receivables are used interchangeably; and the perfect receivables is one that is relatively low in dollars, reasonably current, without a doubt collectable, and ultimately produces no or little write off.

although many individuals pay for their hotel stay with a credit card there are many 691605

Although many individuals pay for their hotel stay with a credit card, there are many others, especially business groups that ask to be billed to a master account. These invoices are often thousands of dollars. An association holding an annual convention in a hotel might have twenty to thirty rooms blocked for its officers, meeting space rentals, breakfasts, luncheons, dinners, and breaks, all charged to a single master bill. It is therefore imperative that hotel controllers try to collect the funds as soon as possible. A number of hotel companies send bills to their customers via Federal Express or other courier services if such accounts are more than $25,000.Why? The customer will have to sign for the delivery of the bill. This eliminates the excuse some customers use, saying they never “received” the bill so that they are not at fault for delinquent payment.

one important component of a note is its interest rate if you are issuing a note rec 691609

One important component of a note is its interest rate. If you are issuing a note receivable, you sure would like to have it at a higher rate, thereby earning more interest payment. On the contrary, if you are writing a note payable, you would want to negotiate the note at the lowest rate possible so that you will not need to pay that much interest. Interest rates go through cycles, with the highest prime rate recorded at 20.50 percent in August 1981 in the United States. In the early 2000s, interest rates still were relatively low, dropping from 9.0 percent to as low as 4.0 percent. However, around 2005, interest rates in the United States began inching higher and higher. The lowest prime rate was recorded at 4.0 percent in 2003 and 2004. In June 2006, the prime rate broke 8.0 percent, went up to 8.25 percent in July and stayed at the same rate and dropped back to 7.50 percent effective October 31, 2007. So shop before you sign any note or borrow any funds for your operations.

can a hotel company make money selling notes receivable absolutely in 2005 dallas ba 691611

Can a hotel company make money selling notes receivable? Absolutely. In 2005, Dallas based Silver leaf Resorts, Inc., reported a net income of $12.9 million in its third quarterly report. Included in that figure is a gain of sale in its notes receivable of $5.8 million. Together with other adjustments, its third quarter adjusted net income was $5.0 million. This also holds true for Hilton Grand Vacations in June 2002, when Hilton’s timeshare segment sold approximately $52 million of its timeshare notes receivable to a subsidiary of GE Capital, resulting in a gain of approximately $2 million in the second quarter. Starwood Hotel and Resorts also reported gains from the sales of notes receivable of $25 million and $14 million in 2005 and 2004, respectively, primarily owing to the sale of approximately $221 million and $113 million of vacation ownership receivables during the years ended December 31, 2005 and 2004.

the following selected transactions relate to falsetto souvenirs company mar 1 sold 691613

The following selected transactions relate to Falsetto Souvenirs Company:

Mar. 1

Sold $20,000 of merchandise to Potter Hotels, terms 2/10, n/30.

11

Received payment in full from Potter Hotels for balance due.

12

Accepted Juno Resorts $20,000, 6 month, 12% note for balance due.

15

Made American Express credit sales totaling $6,700. A 5% service fee is charged by American Express.

30

Received payment in full from American Express Company.

Apr. 11

Sold accounts receivable of $8,000 to Hartco Factor. Hartco Factor assesses a service charge of 2% of the amount of receivables sold.

May. 10

Wrote off as uncollectible $16,000 of accounts receivable. falsetto uses the percentage of sales basis to estimate bad debts.

June. 30

Credit sales for the first 6 months total $2,000,000. The bad debt percentage is 1% of credit sales. At June 30, the balance in the allowance account is $3,500.

Jul. 16

One of the accounts receivable written off in May was from J. Simon, who pays the amount due, $4,000, in full.

Instructions

Prepare the journal entries for the transactions.

presented here are two independent situations on january 6 bennett catering sells fo 691616

Presented here are two independent situations.

(a) On January 6, Bennett Catering sells food and services on account to Jackie, Inc., for $7,000, terms 2/10, n/30. On January 16, Jackie, Inc. pays the amount due. Prepare the entries on Bennett’s books to record the sale and related collection.

(b) On January 10, Erin By bee uses her Sheridan Co. credit card to purchase kitchen supplies from Sheridan Co. for $9,000. On February 10, By bee is billed for the amount due of $9,000. On February 12, By bee pays $6,000 on the balance due. On March 10, By bee is billed for the amount due, including interest at 2 percent per month on the unpaid balance as of February 12. Prepare the entries on Sheridan Co.’s books related to the transactions that occurred on January 10, February 12, and March 10.

the ledger of elburn grill at the end of the current years shows accounts receivable 691617

The ledger of Elburn Grill at the end of the current years shows Accounts Receivable $110,000; Sales $840,000; and Sales Returns and Allowances $28,000.

Instructions

(a) If Elburn uses the direct write off method to account for uncollectible accounts, journalize the adjusting entry at December 31, assuming Elburn determines that Capp’s $1,400 balance is uncollectible.

(b) If Allowance for Doubtful Accounts has a credit balance of $2,100 in the trial balance, journalize the adjusting entry at December 31, assuming bad debts are expected to be (1) 1 percent of net sales and (2) 10 percent of accounts receivable.

(c) If Allowance for Doubtful Accounts has a debit balance of $200 in the trail balance, journalize the adjusting entry at December 31, assuming bad debts are expected to be (1) 0.75 percent of net sales and (2) 6 percent of accounts receivable.

leland hotels has accounts receivable of 98 100 at march 31 an analysis of the accou 691618

Leland Hotels has accounts receivable of $98,100 at March 31. An analysis of the accounts shows the following:

Month of Sale

Balance, March 31

March

$65,000

February

17,600

January

8,500

Prior to January

7,000

 

$98,100

Credit terms are 2/10, n/30. On March 31, Allowance for Doubtful Accounts has a credit balance of $1,200 prior to adjustment. The company uses the percentage of receivables basis for estimating uncollectible accounts. The company’s estimate of bad debts is as follows:

 

Age of Accounts

Estimated  Percentage Uncollectible

1–30 days

2.0%

30–60 days

5.0%

60–90 days

30.0%

Over 90 days

50.0%

Instructions

(a) Determine the total estimated uncollectible.

(b) Prepare the adjusting entry at March 31 to record bad debts expense.

presented here are two independent situations on april 2 julie keiser uses her kitch 691621

Presented here are two independent situations.

On April 2, Julie Keiser uses her Kitchen art Company credit card to purchase kitchen supply from a Kitchen art store for $1,800. On May 1, Keiser is billed for the $1,800 amount due. Keiser pays $700 on the balance due on May 3. On June 1, Keiser receives a bill for the amount due, including interest at 1 percent per month on the unpaid balance as of May 3. Prepare the entries on Kitchen art books related to the transactions that occurred on April 2, May 3, and June 1.

On July 4, Newark’s Restaurant accepts an American Express card for a $350 dinner bill. American Express charges a 4 percent service fee. On July 10, American Express pays Newark $336. Prepare the entries on Newark’s books related to the transactions.

record the following transactions for sandwich co in the general journal 2006 may 1 691623

Record the following transactions for Sandwich, Co., in the general journal:

2006

May 1

1 Received an $8,700, one year, 10 percent note in exchange for Linda Anderson’s outstanding accounts receivable.

Dec. 31

Accrued interest on the Anderson note.

Dec. 31

Closed the interest revenue account.

2007

May 1

Received principal plus interest on the Anderson note. (No interest has been accrued in 2009.)

 

at december 31 2008 sycamore international foods reported the following information 691624

At December 31, 2008, Sycamore International Foods reported the following information on its balance sheet:

During 2009, the company had the following transactions related to receivables

Accounts receivable

$1,020,000

Less: Allowance for doubtful accounts

60,000

1. Sales on account

$2,670,000

2. Sales returns and allowances

40,000

3. Collections of accounts receivable

2,300,000

4. Write offs of accounts receivable deemed uncollectible

65,000

5. Recovery of bad debts previously written off as uncollectible.

20,000

 

Instructions

(a) Prepare the journal entries to record each of these five transactions. Assume that no cash discounts were taken on the collections of accounts receivable.

(b) Enter the January 1, 2009, balances in Accounts Receivable and Allowance for Doubtful Accounts. Post the entries to the two accounts (use T accounts), and determine the balances.

(c) Prepare the journal entry to record bad debts expense for 2009, assuming that an aging of accounts receivable indicates that estimated bad debts are $95,000.

(d) Compute the accounts receivable turnover ratio for the year 2009.

brantley food supply co has the following transactions related to notes receivable d 691625

Brantley Food Supply Co. has the following transactions related to notes receivable during the last two months of the year:

Nov. 1

Loaned $15,000 cash to Bayou Bend Restaurant on a one year, 12 percent note.

Dec. 11

Sold goods to Walker BBQ receiving a $8,000, ninety day, 12 percent note.

16

Received a $5,000, 180 day, 10 percent note on account from Jane’s Bakery.

31

Accrued interest revenue on all notes receivable.

Accrued interest revenue on all notes receivable.

record the following transactions for amy ice co in the general journal 2008 may 1 r 691626

Record the following transactions for Amy Ice Co. in the general journal:

2008

May 1

Received a $12,000, one year, 10 percent note on account from Rosa’s Hamburger.

 

Dec. 31

Accrued interest on the note.

Dec. 31

Closed the interest revenue account.

2009

May 1

Received principal plus interest on the note. (No interest has been accrued in 2009.)

the securities exchange act of 1934 requires any firm that is listed on one of the n 691628

The Securities Exchange Act of 1934 requires any firm that is listed on one of the national exchanges to file annual reports (Form 10 K), financial statements, and quarterly reports (Form 10 Q) with the Securities and Exchange Commission (SEC). This exercise demonstrates how to search and access available SEC filings through the Internet. Journalize entries for notes receivable. (SO 7, 8) Journalize entries for notes receivable transactions. (SO 6, 7) Journalize entries to record transactions related to bad debts. (SO 2, 3)

Instructions

Answer the following questions:

(a) Which SEC filings were available for the company you selected?

(b) In the company’s quarterly report (SEC Form 10 Q), what was one key point discussed in the “Management’s Discussion and Analysis of Results of Operations and Financial Condition”?

(c) What was the net income for the period selected?

financial reporting problem pepsico the financial statements of pepsico and the note 691630

Financial Reporting Problem PepsiCo

The financial statements of PepsiCo and the Notes to Consolidated Financial statements appear in Appendix A.

Instructions Refer to PepsiCo’s financial statements, and answer the following questions about current and long term liabilities: (a) what were PepsiCo’s total current liabilities at December 30, 2006? What was the increase/ decrease in PepsiCo’s total current liabilities from the prior year? (b) In PepsiCo’s Note 2 (“Our Significant Accounting Policies”), the company explains the nature of its contingencies. Under what conditions does PepsiCo recognize (record and report) liabilities for contingencies? (c) What were the components of total current liabilities on December 30, 2006? (d) What was PepsiCo’s total long term debt (excluding deferred income taxes) at December 30, 2006? What was the increase/decrease in total long term debt (excluding deferred income taxes) from the prior year? What does Note 9 to the financial statements indicate about the composition of PepsiCo’s long term debt obligation? (e) What are the total long term contractual commitments that PepsiCo reports as of December 30, 2006?

feature story trellis is a spectacular spa within the tranquil setting of the eighte 691631

Feature Story

Trellis is a spectacular spa within the tranquil setting of the eighteen acre property of The Houstonian Hotel, Club & Spa in Houston, Texas. A Mediterranean style, two story, residential style building surrounds a courtyard that offers a beautiful natural garden environment that can be seen through large windows located throughout the spa. Outside the second story of Trellis, there is a balcony overlooking the Houstonian’s lush landscape. This 17,000 square foot spa houses nineteen treatment rooms, including ten rooms for body treatments, six facial rooms, two couples’ rooms (each including two treatment tables and couples’ showers and tubs), and a hydrotherapy room. In addition, there are hair styling stations, manicure and pedicure stations, luxurious locker rooms with Jacuzzi and steam room, an indoor float pool, a comfortable upstairs lounge with a fireplace, and a quiet room for before and after treatments. How much does it cost to build such a luxurious sanctuary just next to the Galleria and business center in Houston? $5.6 million. This includes $4.26 million in construction; $450,000 in furniture, fixtures, and equipment; $124,000 in computing; $380,000 in architecture and consultant fees; and other costs of $392,000. Why did the Houstonian decide to go with this investment? Al Gallo, the chief financial officer of Redstone Hospitality, which owns and operates the Houstonian Hotel, Club & Spa, offers the following: First, the existing spa of 5,000 square feet that serves the members of the fitness club and the hotel and catering patrons is too small. It had long appointment waiting lists; and the facility itself, although somewhat historic, had outlived its usefulness. In addition, though it was owned by Redstone, it was managed by an outside spa company. Mr. Gallo notes that Redstone feels that it can manage and own the spa and can do a better job of controlling member and guest expectations. Finally, demolition of the old spa building made room for the latest addition, the new family resort style swimming pool. The new spa was relocated across the street, a few yards away, on a vacant parcel of land previously used as green space and some parking. The name Trellis was chosen in order to incorporate the luscious outdoor landscaping element of the property, and some elements of water were added for a feel of tranquility and relaxation. The entire project from ground breaking to grand opening took sixteen months, from September 2002 through December 2003. The hiring and training were done two months in advance of opening, and Redstone offered complimentary spa treatments to staff and members for the month prior to opening. Given the clientele, Redstone is quite selective in its hiring of licensed massage therapists. All the new hires go through “Seven Habits” training, a Franklin Covey–style leadership training program. The guests have given it rave reviews, as have the members Trellis is the only Mobil four star spa in the city of Houston. Although 50 percent of the guests are active fitness club members and about 10 percent are hotel guests, 40 percent of the clients are from the local Houston area. Determining the correct balance of treatment rooms to locker rooms and leisure space is a challenge. To accomplish this, Redstone enlisted outside spa consultants. With the current demand, Trellis can use an additional two to three rooms beyond what it has today and therefore has plans already to expand in the near future. Is it different to be the CFO of a hotel than to be one of a hotel, a club, and a spa all at the same time? Mr. Gallo says that the key is to understand the accounting for the hotel. He has found if one can understand the nuances of hotel accounting, the club and/or spa accounting is much easier to digest because they are large businesses but viewed as additional outlets to the hotel. He did have to learn much more about the spa business with regard to payroll, such as hourly payments versus flat fees for each serve

[Feature Story Continued]

Ice performed. The volume of work increases significantly within the accounting department itself because the spa is a large generator of invoices to pay, employees on staff, gift certificates to track, and software to keep up to date and interfaced with the other hotel and club systems. By setting the club and spa up similar to, say, an outlet of the hotel, staff can process the payables, payroll, and receivables in a similar fashion without adding a whole

 width=

the houstonian hotel club amp spa welcomes 86 000 guests per year and has a member l 691633

The Houstonian Hotel, Club & Spa welcomes 86,000 guests per year and has a member list of about 10,000. It also hosts meetings and interviews for President George W. Bush. Therefore, anything that is done at the Houstonian is always first class. However, with buildings that depreciate, renovations and upkeeps, and building a new spa, how does the Houstonian deal with the issue of depreciation? Mr. Al Gallo explains: “We capitalize our assets and break them out in terms of hard construction costs, soft costs—or those costs associated with architectural fees, project managers and consultants, also furniture fixtures and equipment—and finally we categorize separately our computer equipment and security and surveillance equipment. Each category listed above has a unique life established. We would employ MACRS [Modified Accelerated Cost Recovery System] for tax and simple depreciation for book as well. After the project is finished, we would move the ‘work in progress’ amounts into the ‘fixed assets’ and depreciate them monthly in order to properly match revenues and expenses

in what could become one of the largest accounting frauds in history worldcom announ 691636

In what could become one of the largest accounting frauds in history, WorldCom announced the discovery of $7 billion in expenses improperly booked as capital expenditures, a gimmick that boosted profit over a recent five quarter period. If these expenses had been recorded properly, WorldCom, one of the biggest stock market stars of the 1990s, would have reported a net loss for 2001, as well as for the first quarter of 2002. Instead, WorldCom reported a profit of $1.4 billion for 2001 and $130 million for the first quarter of 2002. As a result of these problems, WorldCom declared bankruptcy, to the dismay of its investors and creditors. What erroneous accounting entries (accounts debited and credited) were made by WorldCom? What is the correcting entry that should be recorded, and what is its effect on WorldCom’s financial statements?

if control is so important in the hospitality business and if technology is supposed 691566

If control is so important in the hospitality business and if technology is supposed to make things better, is technology used in helping operators and businesses control their costs? Chevys, a restaurant chain that owns 131 Chevys Fresh Mix restaurants, 33 Rio Bravos, and 9 Fazio Universal Pasta restaurants, has the answer. They installed a system that has resulted in at least a 1 percent savings in food costs. With average sales at a Chevys Fresh Mix of over $2.5 million a year, 1 percent of sales translates to $25,000 and thus close to $3.3 million just for the Fresh Mix brand. The system allows management to do correct pricing with updated information from the vendor. There are many available technology systems of which restaurateurs, big or small, can take advantage and better operate their businesses. They can track menu additions, product mix, takeout orders, complimentary meals, voids, specialty requests, and so forth, so that management and owners can stay competitive.

unfortunately computer related frauds have become a major concern the average comput 691567

Unfortunately, computer related frauds have become a major concern. The average computer fraud loss is $650,000, compared with an average loss of only $19,000 resulting from other types of white collar crime. Computer fraud can be perpetrated almost invisibly and with electronic speed. Psychologically, stealing with impersonal computer tools can seem far less criminal to some people. Therefore, the moral threshold to commit computer fraud is lower than that for fraud involving person to person contact. Preventing and detecting computer fraud represent a major challenge. One of the best ways for a company to minimize the likelihood of computer fraud is to have a good system of internal control that allows the benefits of computerization to be gained without opening the possibility for rampant fraud. Is a computer capable of committing fraud? Discus

a study by the association of certified fraud examiners indicates that businesses wi 691568

A study by the Association of Certified Fraud Examiners indicates that businesses with fewer than 100 employees are most at risk for employee theft. Also, the average loss per incident for small companies—$127,500—was actually higher than the average loss for larger companies. The high degree of trust often found in small companies makes them more vulnerable to dishonest employees. For example, in one small company, the employee responsible for paying bills would intentionally ask the owner to sign checks only when the owner was extremely busy. The employee would slip in one check that was made out to himself, and the owner didn’t notice because he was too busy to carefully review each check. Which principles of internal control should have prevented such fraud?

during a class lecture mr alan e gallo chief financial officer of redstone hospitali 691571

During a class lecture, Mr. Alan E. Gallo, chief financial officer of Redstone Hospitality, shared some controls incidents with the students. He mentioned that a hotel received a complaint letter and a laundry bill from a guest. This guest claimed that food was spilled on his wife’s dress, and he would like the hotel to pay for the laundry bill of an amount less than $10. To make the case more convincing, the guest also mentioned that he was a friend of another frequent guest of this establishment. When investigating the matter, the controller called the number on the laundry bill, trying to confirm the incident with the cleaner, and found that the cleaner did not exist. Most people will pay the small bill to take care of the complaint and move on to the next issue. The moral of the story: It is not the amount that matters; it is the principle. Follow set procedures and catch those crooks.

utrillo beds and mattresses sells beddings to resort and lodging operations its bank 691574

Utrillo Beds and Mattresses sells beddings to resort and lodging operations. Its bank statement for May 2008 shows the following data:

Balance 5/1

$12,650

Balance 5/31

$14,280

Debit memorandum:

NSF check

$175

Credit memorandum:

Collection of note receivable

$505

The cash balance per books at May 31 is $13,319. Your review of the data reveals the

Following:

1. The NSF check was from Hop Hotel Co., a customer.

2. The note collected by the bank was a $500, three month, 12 percent note. The bank charged a $10 collection fee. No interest has been accrued.

3. Outstanding checks at May 31 total $2,410.

4. Deposits in transit at May 31 total $1,752.

5. A Utrillo check for $352 dated May 10 cleared the bank on May 25. This check, which was a payment on account, was journalized for $325.

Instructions

(a) Prepare bank reconciliation at May 31.

(b) Journalize the entries required by the reconciliation.

the following control procedures are used at sandwich company for over the counter c 691578

The following control procedures are used at Sandwich Company for over the counter cash receipts.

1. To minimize the risk of robbery, cash in excess of $100 is stored in an unlocked attaché case in the stock room until it is deposited in the bank.

2. All over the counter receipts are registered by three clerks who use a cash register with a single cash drawer.

3. The company accountant makes the bank deposit and then records the day’s receipts.

4. At the end of each day, the total receipts are counted by the cashier on duty and reconciled to the cash register total.

5. Cashiers are experienced; they are not bonded.

Instructions

(a) For each procedure, explain the weakness in internal control, and identify the control principle that is violated.

(b) For each weakness, suggest a change in procedure that will result in good internal control.

the following control procedures are used in morgan rsquo s fishing excursions for c 691579

The following control procedures are used in Morgan’s Fishing Excursions for cash disbursements:

1. The company accountant prepares the bank reconciliation and reports any discrepancies to the owner.

2. The store manager personally approves all payments before signing and issuing checks.

3. Each week, Morgan leaves 100 company checks in an unmarked envelope on a shelf behind the cash register.

4. After payment, bills are filed in a paid invoice folder.

5. The company checks are unnumbered.

Instructions

(a) For each procedure, explain the weakness in internal control, and identify the internal control principle that is violated.

(b) For each weakness, suggest a change in the procedure that will result in good internal control.

lisa cesar is unable to reconcile the bank balance for her catering company on janua 691580

Lisa Cesar is unable to reconcile the bank balance for her catering company on January 31. Lisa’s reconciliation is as follows:

Cash balance per bank

$3,660.20

Add: NSF check

590.00

Less: Bank service charge

25.00

Adjusted balance per bank

$4,225.20

Cash balance per books

$3,875.20

Less: Deposits in transit

530.00

Add: Outstanding checks

930.00

Adjusted balance per books

$4,275.20

Instructions

(a) Prepare correct bank reconciliation.

(b) Journalize the entries required by the reconciliation.

on april 30 the bank reconciliation of ottawa travel shows three outstanding checks 691581

On April 30, the bank reconciliation of Ottawa Travel shows three outstanding checks: no. 254, $650; no. 255, $720; and no. 257, $410. The May bank statement and the May cash payments journal show the following:

Bank Statement

Checks Paid

Date

Check No.

Amount

5/4

254

650

5/2

257

410

5/17

258

159

5/12

259

275

5/20

261

500

5/29

263

480

5/30

262

750

 

Cash Payments Journal

Checks Issued

Date

Check No.

Amount

5/2

258

159

5/5

259

275

5/10

260

790

5/15

261

500

5/22

262

750

5/24

263

480

5/29

264

560

Instructions

Using step 2 in the reconciliation procedure, list the outstanding checks on May 31.

the following information relates to the cash account in the ledger of dick wasson c 691583

The following information relates to the Cash account in the ledger of Dick Wasson Company:

Balance September 1

$17,150

Cash deposited

$64,000

Balance September 30

$17,404

Checks written

$63,746

The September bank statement shows a balance of $16,422 on September 30 and the following memoranda:

 

Credits

 

Debits

 

Collection of $1,500 note plus interest$30

$1,530

NSF check: J. E. Hoover

$725

Interest earned on checking account

$45

Safety deposit box rent

$65

On September 30, deposits in transit were $4,150, and outstanding checks totaled $2,383.

Instructions

(a) Prepare the bank reconciliation at September 30.

(b) Prepare the adjusting entries at September 30, assuming (1) the NSF check was from a customer on account, and (2) no interest had been accrued on the note.

you are the assistant controller in charge of general ledger accounting at springtim 691586

You are the assistant controller in charge of general ledger accounting at Springtime Bottling Company. Your company has a large loan from an insurance company. The loan agreement requires that the company’s cash account balance be maintained at $200,000 or more, as reported monthly. On June 30, the cash balance is $80,000, which you report to Anne Shirley, the financial vice president. Anne excitedly instructs you to keep the cash receipts book open for one additional day for purposes of the June 30 report to the insurance company. Anne says, “If we don’t get that cash balance over $200,000, we’ll default on our loan agreement. They could close us down, put us all out of our jobs!” Anne continues, “I talked to Oconto Distributors (one of Spring time’s largest customers) this morning. They said they sent us a check for $150,000 yesterday. We should receive it tomorrow. If we include just that one check in our cash balance, we’ll be in the clear. It’s in the mail!”

Instructions

(a) Who will suffer negative effects if you do not comply with Anne Shirley’s instructions? Who will suffer if you do comply?

(b) What are the ethical considerations in this case?

(c) What alternatives do you have?

no it is not halloween but you can get spooked by a ghost employee in the old days w 691587

No, it is not Halloween, but you can get spooked by a ghost employee! In the old days, when businesses were small and payroll was given in cash or paychecks were given to employees in person, this was not a problem. With technology and direct deposits, ghost employees seem to surface. Who are ghost employees? They are people who do not exist in your casino, cruise ship, theme park, or hotel but are being paid. They are fictitious employees created by the perpetrator or terminated or deceased employees who are not removed from payroll records. Some dishonest person continues to collect and cash paychecks for people who do not exist. What can be done? Detecting fraud takes a team. Always check bank account numbers, Social Security or identification numbers, addresses, deductions, work location and department, and the like. It may not be a bad idea to also look at sick leave and vacation. Even ghosts need a break. If someone on your payroll is not taking the normal level of sick leave and vacations, you either have a very dedicated employee or a dead one. Either way, it is good for you to find out so that you can reward the former or delete the latter.

employee earnings records for brantley restaurants reveal the following gross earnin 691593

Employee earnings records for Brantley Restaurants reveal the following gross earnings for four employees through the pay period of December 15:

 

C. Mays

$83,500

D. Delgado

$86,100

L. Jeter

$85,600

T. Rollin

$87,000

For the pay period ending December 31, each employee’s gross earnings is $3,000. The FICA tax rate is 8 percent on gross earnings of $87,900.

Instructions

Compute the FICA withholdings that should be made for each employee for the December 31 pay period. (Show computations.)

marissa rsquo s sushi has the following data for the weekly payroll ending january 3 691594

Marissa’s Sushi has the following data for the weekly payroll ending January 31:

Hours

Employee

M

T

W

T

F

S

M. Hindi

8

8

9

8

10

3

E. Benson

8

8

8

8

8

2

K. Estes

9

10

8

8

9

0

 

 

Hourly

Rate

Federal

Income Tax Withholding

Health

Insurance

$11

$34

$10

13

37

15

14

58

15

Employees are paid 11⁄2 times the regular hourly rate for all hours worked in excess of 40 hours per week. FICA taxes are 8 percent on the first $87,900 of gross earnings. Marissa’s Sushi is subject to 5.4 percent state unemployment taxes on the first $9,800 and 0.8 percent federal unemployment taxes on the first $7,000 of gross earnings.

Instructions

(a) Prepare the payroll register for the weekly payroll.

(b) Prepare the journal entries to record the payroll and Marissa’s payroll tax expense.

the payroll procedures used by three different companies are 1 in brewer cafe each e 691595

The payroll procedures used by three different companies are:

1. In Brewer Cafe, each employee is required to mark on a clock card the hours worked. At the end of each pay period, the employee must have this clock card approved by the department manager. The approved card is then given to the payroll department by the employee. Subsequently, the treasurer’s department pays the employee by check.

2. In Evan’s Steak House, clock cards and time clocks are used. At the end of each pay period, the department manager initials the cards, indicates the rates of pay, and sends them to payroll. A payroll register is prepared from the cards by the payroll department. Cash equal to the total net pay in each department is given to the department manager, who pays the employees in cash.

3. In Alien’s Noodles, employees are required to record hours worked by “punching” clock cards in a time clock. At the end of each pay period, the clock cards are collected by the department manager. The manager prepares a payroll register in duplicate and forwards the original to payroll. In payroll, the summaries are checked for mathematical accuracy, and a payroll supervisor pays each employee by check.

Instructions (a) indicate the weakness (est.) in internal control in each company.

(b) For each weakness, describe the control procedure(s) that will provide effective internal control. Use the following format for your answer:

joe rsquo s sandwich shop has four employees who are paid on an hourly basis plus ti 691596

Joe’s Sandwich Shop has four employees who are paid on an hourly basis plus time and a half for all hours worked in excess of 40 a week. Payroll data for the week ended February 15, 2010, are presented below:

Employees

 

Hours

Worked

 

Hourly

Rate

 

Federal

Income Tax

Withholdings

United

Fund

 

L. Leis

39

$14.00

$ ?

$ —

S. Bjork

42

$12.00

?

5.00

M. Cape

44

$12.00

61

7.50

L. Wild

48

$12.00

52

5.00

Leis and Bjork are married. They claim two and four withholding allowances, respectively. The following tax rates are applicable: FICA 8 percent, state income taxes 3 percent, state unemployment taxes 5.4 percent, and federal unemployment taxes 0.8 percent. The first three employees are sales clerks (store wage expense). The fourth employee performs administrative duties (office wages expense).

Instructions

(a) Prepare a payroll register for the weekly payroll. (Use the wage bracket withholding table in the text for federal income tax withholdings.)

(b) Journalize the payroll on February 15, 2010, and the accrual of employer payroll taxes.

(c) Journalize the payment of the payroll on February 16, 2010.

(d) Journalize the deposit in a Federal Reserve Bank on February 28, 2010, of the FICA and federal income taxes payable to the government.

the following payroll liability accounts are included in the ledger of milton rsquo 691597

The following payroll liability accounts are included in the ledger of Milton’s Brewery and Grille on January 1, 2010:

FICA Taxes Payable

$ 662.20

Federal Income Taxes Payable

1,254.60

State Income Taxes Payable

102.15

Federal Unemployment Taxes Payable

312.00

State Unemployment Taxes Payable

1,954.40

Union Dues Payable

250.00

U.S. Savings Bonds Payable

350.00

 

In January, the following transactions occurred:

Jan.10

Sent check for $250.00 to union treasurer for union dues.

12

Deposited check for $1,916.80 in Federal Reserve bank for FICA taxes and

federal income taxes withheld

15

Purchased U.S. Savings Bonds for employees by writing check for $350.00.

17

Paid state income taxes withheld from employees.

20

Paid federal and state unemployment taxes.

31

Completed monthly payroll register, which shows office salaries $17,600;

store wages $27,400; FICA taxes withheld $3,600; federal income taxes

payable $1,770; state income taxes payable $360; union dues payable $400;

United Fund contributions payable $1,800; and net pay $37,070.

31

Prepared payroll checks for the net pay, and distributed checks to

Employees.

On January 31, the company also makes the following accrual for employer payroll taxes: FICA taxes 8 percent,  tare unemployment taxes 5.4 percent, and federal unemployment taxes 0.8 percent.

Instructions

(a) Journalize the January transactions.

(b) Journalize the adjustments pertaining to employee compensation at January 31.

for the year ended december 31 2010 r vishal company reports the following summary p 691598

For the year ended December 31, 2010, R. Vishal Company reports the following summary payroll data:

 

Gross earnings

 

Administrative salaries

$180,000

Electricians’ wages

320,000

Total

$500,000

 

Deductions

 

FICA taxes

$35,200

Federal income taxes withheld

153,000

State income taxes withheld (2.6%)

13,000

United Fund contributions payable

25,000

*Hospital insurance premiums

15,800

Total

$242,000

R. Vishal Company’s payroll taxes are FICA 8 percent, state unemployment 2.5 percent (due to a stable employment record), and federal unemployment 0.8 percent. Gross earnings subject to FICA taxes total $440,000, and unemployment taxes total $110,000.

Wages, Tips,

Federal Income

State Income

FICA

FICA Tax

Other Compensation

Tax Withheld

Tax Withheld

Wages

Withheld

Complete the required data for the following employees:

Employee

Gross Earnings

Federal Income Tax Withheld

R. Lopez

$60,000

$27,500

K. Kirk

27,000

11,000

nugent rsquo s vietnamese sandwiches employs seven waitpersons with one bus person w 691599

Nugent’s Vietnamese Sandwiches employs seven waitpersons with one bus person, with whom the waitpersons will share their tips. The information regarding their gross receipts, number of hours worked, and tips reported are detailed in the table below. The owner, Mrs. Twang Nguyen, is not sure that all the waitpersons report the required 8 percent of gross receipts. Using both the gross receipts and the hours worked methods; please help Mrs. Nguyen identify the tip amounts that need to be allocated to each employee, if any.

 

Employee

Gross

Receipts

Hours

Worked

Tips

Reported

1

$28,000

30

$1,600

2

35,000

40

2,670

3

17,500

20

1,250

4

31,230

40

3,150

5

40,100

40

3,870

6

42,290

38

3,775

7

34,300

40

2,200

kishwaukee catering company provides catering services for banquets events and other 691601

Kishwaukee Catering Company provides catering services for banquets, events, and other catering functions within a university community. The work is fairly steady throughout the year but peaks significantly in December and May as a result of holiday parties and graduation events. Two years ago, the company attempted to meet the peak demand by hiring part time help. However, this led to numerous errors and considerable customer dissatisfaction. A year ago, the company hired four experienced employees on a permanent basis instead of using part time help. This proved to be much better in terms of productivity and customer satisfaction. But it has caused an increase in annual payroll costs and a significant decline in annual net income. Recently, Valarie Flynn, a sales representative of Harrington Services, Inc., has made a proposal to the company. Under her plan, Harrington Services will provide up to four experienced workers at a daily rate of $110 per person for an eight hour workday. Harrington workers are not available on an hourly basis. Kishwaukee Catering would have to pay only the daily rate for the workers used. The owner of Kishwaukee, Martha Bell, asks you, as the company’s accountant, to prepare a report on the expenses that are pertinent to the decision. If the Harrington plan is adopted, Martha will terminate the employment of two permanent employees and will keep two permanent employees. At the moment, each employee earns an annual income of $30,000. Kishwaukee pays 8 percent FICA taxes, 0.8 percent federal unemployment taxes, and 5.4 percent state unemployment taxes. The unemployment taxes apply to only the first $7,000 of gross earnings. In addition, Kishwaukee Catering pays $40 per month for each employee for medical and dental insurance. Martha indicates that if the Harrington Services plan is accepted, Kishwaukee’s needs for Harrington’s workers will be as follows.

 

Months

Number

 

Working

Days per Month

January–March

2

20

April–May

3

25

June–October

2

18

November–December

3

23

Instructions

With the class divided into groups, answer the following:

(a) Prepare a report showing the comparative payroll expense of continuing to employ permanent workers compared to adopting the Harrington Services, Inc., plan.

(b) What other factors should Martha consider before finalizing her decision?

michael davidson started his own hospitality consulting firm star company on june 1 691479

Michael Davidson started his own hospitality consulting firm, Star Company, on June 1, 2008. Star Company performs feasibility studies to determine if restaurants should be opened at certain locations. The trial balance at June 30 is as follows.

STAR COMPANY

Trial Balance

June 30, 2008

Account Number

 

Debit

Credit

 

101

Cash

$7,700

   

110

Accounts Receivable

4,000

   

120

Prepaid Insurance

2,400

   

130

Supplies

1,500

   

135

Office Equipment

11,000

   

200

Accounts Payable

 

$3,500

 

230

Unearned Service Revenue

 

3,000

 

311

Common Stock

 

19,100

 

320

Retained Earnings

 

–0–

 

400

Service Revenue

 

6,000

 

510

Salaries Expense

4,000

   

520

Rent Expense

1,000

   
   

$31,600

$31,600

 

 

In addition to those accounts listed on the trial balance, the chart of accounts for Star Company also contains the following accounts and account numbers: No. 136 Accumulated Depreciation— Office Equipment, No. 210 Utilities Payable, No. 220 Salaries Payable, No. 530 Depreciation Expense, No. 540 Insurance Expense, No. 550 Utilities Expense, and No. 560 Supplies Expense.

Other data:

1. $500 of supplies has been used during the month.

2. A utility bill for $200 has not been recorded and will not be paid until next month.

3. The insurance policy is for two years.

4. $1,000 of unearned service revenue has been earned at the end of the month.

5. Salaries of $600 are accrued at June 30.

6. The office equipment has a five year life with no salvage value. It is being depreciated at $200 per month for 60 months.

7. Invoices representing $1,000 of services performed during the month have not been recorded as of June 30.

Instructions

(a) Prepare the adjusting entries for the month of June. Use J3 as the page number for your journal.

(b) Post the adjusting entries to the ledger accounts. Enter the totals from the trial balance as beginning account balances, and place a check mark in the posting reference column.

(c) Prepare an adjusted trial balance at June 30, 2008.

the thayer motel inc opened for business on may 1 2008 its trial balance before adju 691480

The Thayer Motel, Inc., opened for business on May 1, 2008. Its trial balance before adjustment on May 31 is as follows.

In addition to those accounts listed on the trial balance, the chart of accounts for Thayer Mo

THAYER MOTEL, INC.

Trial Balance

May 31, 2008

Account Number

 

                           Debit

 

                           Credit

 

101

Cash

$2,500

$2,500

126

Supplies

$1,900

1,900

130

Prepaid Insurance

2,400

2,400

140

Land

15,000

15,000

141

Lodge

70,000

 

149

Furniture

16,800

 

201

Accounts Payable

 

$5,300

208

Unearned Rent

 

4,600

275

Mortgage Payable

 

35,000

311

Common Stock

 

60,000

332

Dividends

1,000

9,200

429

Rent Revenue

 

 

610

Advertising Expense

500

 

726

Salaries Expense

3,000

 

732

Utilities Expense

1,000

 
   

$114,100

$114,100

tell also contains the following accounts and account numbers: No. 142 Accumulated Depreciation— Lodge, No. 150 Accumulated Depreciation—Furniture, No. 212 Salary Payable, No. 230 Interest Payable, No. 320 Retained Earnings, No. 619 Depreciation Expense—Lodge, No. 621 Depreciation Expense—Furniture, No. 631 Supplies Expense, No. 718 Interest Expense, and No. 722 Insurance Expense.

Other data:

1. Insurance expires at the rate of $200 per month.

2. A count of supplies shows $900 of unused supplies on May 31.

3. Annual depreciation is $2,400 on the lodge and $3,000 on furniture.

4. The mortgage interest rate is 12%. (The mortgage was taken out on May 1.)

5. Unearned rent of $2,500 has been earned.

6. Salaries of $800 are accrued and unpaid at May 31.

Instructions

(a) Journalize the adjusting entries on May 31.

(b) Prepare a ledger using the three column form of account. Enter the trial balance amounts,

And post the adjusting eateries. (Use J1 as the posting reference.)

(c) Prepare an adjusted trial balance on May 31.

(d) Prepare an income statement and a retained earnings statement for the month of May and

a balance sheet at May 31

ethic case die hard company is a pest control company providing services to hotels 691483

{{Ethic Case}}

Die Hard Company is a pest control company providing services to hotels and clubs. Its sales declined greatly this year due to the passage of legislation outlawing the sale of several of Die Hart’s chemical pesticides. In the coming year, Die Hard will have environmentally safe and competitive chemicals to replace these discontinued products. Sales in the next year are expected to greatly exceed any prior year’s. The decline in sales and profits appears to be a one year aberration. But even so, the company president fears a large dip in the current year’s profits. He believes that such a dip could cause a significant drop in the market price of Die Hart’s stock and make the company a takeover target. To avoid this possibility, the company president calls in Becky Freeman, controller, to discuss this period’s year end adjusting entries. He urges her to accrue every possible revenue and to defer as many expenses as possible. He says to Becky, “We need the revenues this year, and next year can easily absorb

expenses deferred from this year. We can’t let our stock price be hammered down!” Becky didn’t get around to recording the adjusting entries until January 17, but she dated the entries December 31 as if they were recorded then. Becky also made every effort to comply with the president’s request.

Instructions

(a) Who are the stakeholders in this situation?

(b) What are the ethical considerations of (1) the president’s request and (2) Becky’s dating the adjusting entries December 31?

(c) Can Becky accrue revenues and defer expenses and still be ethical?

accounting in action business insight the river oaks country club is an exclusive pr 691487

Accounting In Action Business Insight

The River Oaks Country Club is an exclusive private club in the Houston metropolitan area. It has more than 1,500 members, and monthly billing can be a big project without the use of computers. Mel Samuelson, chief financial officer of the club, comments that he could not imagine how cumbersome the system would be if month end closing and billing had to be done manually. “It only takes us one day to do our month end closing with computers,” said Samuelson. “If every transaction for all 1,530 members has to be done by hand, it may take at least a week just for the closing.” Mr. Samuelson also adds that with the use of computers, all postings are updated on a daily basis, making information accessible and real time. As Mr. Samuelson suggests, computers do provide more accurate and updated information. More important, this also means that the associates in the accounting office can now use the time saved to do other, more meaningful analysis of data, thereby providing better information to management and better service to its members.

a look at our feature story refer back to the feature story about rhino foods inc at 691489

A Look At Our Feature Story

Refer back to the Feature Story about Rhino Foods, Inc. at the beginning of this chapter and answer the following questions:

1. What is the lesson of the Rhino Foods story and Ted Castle’s innovations?

2. How did Rhino’s employees’ knowledge of financial statements, especially  production reports and the income statement, contribute to their effectiveness as  employees?

When Ted Castle was a hockey coach at the University of Vermont, his players were self motivated by their desire to win. Hockey was a game you either won or lost. But at Rhino Foods, Inc., a specialty bakery foods company he founded in Burlington, Vermont, he discovered that manufacturing line workers were not so self motivated. Ted thought, what if he turned the food making business into a game, with rules, strategies, and trophies? Ted knew that in a game, knowing the score is all important. He felt that only if the employees know the score—know exactly how the business is doing daily, weekly, monthly—could he turn food making into a game. But Rhino is a closely held, family owned business, and its financial statements and profits were confidential. Should Ted open Rhino’s books to the employees? A consultant he was working with put Ted’s concerns in perspective. perspective. The consultant said, “Imagine you’re playing touch football. You play for an hour or two, and the whole time I’m sitting there with a book, keeping score. All of a sudden I blow the whistle, and I say, ‘OK, that’s it. Everybody go home.’ I close my book and walk away. How would you feel?” Ted opened his books and revealed the financial statements to his employees. The next step was to teach employees the rules and strategies of how to win at making food. The first lesson: “Your opponent at Rhino is expenses. You must cut and control expenses.” Ted and his staff distilled those lessons into daily scorecards (production reports and income statements) that keep Rhino’s employees up to date on the game. At noon each day Ted posts the previous day’s results at the entrance entrance to the production room. Everyone checks whether they made or lost money on what they produced the day before. And it’s not just an academic exercise; there’s a bonus check for each employee at the end of every four week “game” that meets profitability guidelines. Everyone can be a winner! Rhino has flourished since the first game. Employment has increased from 20 to 130 people, while both revenues and profits have grown dramatically.

[[Demonstration  Problem]]

At the end of its first month of operations, Watson Island Tour Service, Inc., has the following unadjusted trial balance:

 

WATSON ISLAND TOUR SERVICE, INC.

August 31, 2008

Trial Balance

 

Debit

Credit

Cash

$ 5,400

 

Accounts Receivable

8,800

 

Prepaid Insurance

2,400

 

Supplies

1,300

 

Equipment

60,000

 

Notes Payable

 

$40,000

Accounts Payable

 

2,400

Common Stock

 

30,000

Dividends

1,000

 

Service Revenue

 

10,900

Salaries Expense

3,200

 

Utilities Expense

800

 

Advertising Expense

400

 

 

$83,300

$83,300

Other data consist of the following:

1. Insurance expires at the rate of $200 per month.

2. There are $1,000 of supplies on hand at August 31.

3. Monthly depreciation on the equipment is $900.

4. Interest of $500 on the notes payable has accrued during August.

Instructions

(a) Prepare a work sheet.

(b) Prepare a classified balance sheet assuming that $35,000 of the notes payable are long term.

(c) Journalize the closing entries.

the balance sheet debit column of the work sheet for empanada supreme includes the f 691495

The balance sheet debit column of the work sheet for Empanada Supreme includes the following accounts: Prepaid Insurance $4,600; Cash $24,080; Supplies $6,200; Short term Investments $7,240; and Accounts Receivable $15,670. Prepare the current assets section of the balance sheet, listing the accounts in proper sequence.

CAJUN COMPANY

Work Sheet (partial)

For the Month Ended April 30, 2008

 

Adjusted Trial Balance

Income Statement

Balance Sheet

Account Titles

Dr.

Cr.

Dr.

Cr.

Dr.

Cr.

Cash

14,752

         

Accounts Receivable

7,840

         

Prepaid Rent

2,280

         

Equipment

23,050

         

Accumulated Depreciation

 

4,921

       

Notes Payable

 

5,700

       

Accounts Payable

 

5,972

       

Common Stock

 

25,000

       

Retained Earnings

 

8,960

       

Dividends

3,650

         

Service Revenue

 

12,590

       

Salaries Expense

9,840

         

Rent Expense

760

         

Depreciation Expense

671

         

Interest Expense

57

         

Interest Payable

 

57

       

Totals

62,900

62,900

       

the adjusted trial balance of lana company at the end of its fiscal year is as follo 691496

The adjusted trial balance of Lana Company at the end of its fiscal year is as follows:

LANZA COMPANY

Adjusted Trial Balance

July 31, 2008

No

Account Titles

Debit

Credit

       

101

Cash

$14,840

 

112

Accounts Receivable

8,780

 

157

Equipment

15,900

 

167

Accumulated

 

$5,400

201

Accounts Payable

 

4,220

208

Unearned Rent

 

1,800

311

Common Stock

 

20,000

320

Retained Earnings

 

25,200

332

Dividends

16,000

 

404

Commission

 

67,000

429

Rent Revenue

 

6,500

711

Depreciation

4,000

 

720

Salaries Expense

55,700

 

732

Utilities Expense

14,900

 
   

$130,120

$130,120

 

Instructions

(a) Prepare the closing entries using page J15.

(b) Post to Retained Earnings and No. 350 Income Summary accounts. (Use the three column form.)

(c) Prepare a post closing trial balance on July 31.

(d) Prepare an income statement and a retained earnings statement for the year ended July 31, 2008. There were no issuances of stock during the year.

(e) Prepare a classified balance sheet on July 31.

john claris began operations as an event consultant on january 1 2008 the trial 691497

John Claris began operations as an event consultant on January 1, 2008. The trial balance      columns of the work sheet on March 31 are as follows.

JOHN CLAIS, INC.

Work Sheet

For the Quarter Ended March 31, 2008

 

 

 

Trial Balance

 

Account Titles

                               Dr.

Cr.

Cash

12,400

 

Accounts Receivable

2,620

 

Supplies

1,250

 

Prepaid Insurance

2,000

 

Equipment

32,000

 

Notes Payable

 

10,000

Accounts Payable

 

12,350

Common Stock

 

20,000

Dividends

600

 

Service Revenue

 

13,620

Salaries Expense

3,200

 

Travel Expense

900

 

Rent Expense

1,000

 
 

55,970

55,970

Other data:

1. Supplies on hand total $500.

2. Depreciation is $250 per quarter.

3. Interest accrued on a six month note payable, issued January 1, $400.

4. Insurance expires at the rate of $180 per month.

5. Services provided but unbilled at March 31 total $1,750.

Instructions

(a) Enter the trial balance on a work sheet, and complete the work sheet.

(b) Prepare an income statement and a retained earnings statement for the quarter and a classified  balance sheet on March 31.

(c) Journalize the adjusting entries from the adjustments columns of the work sheet.

(d) Journalize the closing entries from the financial statement columns of the work sheet.

Final Reporting problem PepsiCo:

as the controller of telltales ice cream company you discover a misstatement that si 691500

As the controller of Telltales Ice Cream Company, you discover a misstatement that significantly overstated net income in the prior year’s financial statements. The misleading financial statements appear in the company’s annual report, which was issued to banks and other creditors less than a month ago. After much thought about the consequences of telling the president, Eddie Lineman, about this misstatement, you gather your courage to inform him. Eddie says, “Hey! What they don’t know won’t hurt them. But just so we set the record straight, we’ll adjust this year’s financial statements for last year’s misstatement. We can absorb that misstatement better in this year than in last year anyway! Just don’t make such a mistake again.”

Instructions

(a) Who are the stakeholders in this situation?

(b) What are the ethical issues in this situation?

(c) What would you do as the controller in this situation?

after enron many companies were forced by increased investor criticism and regulator 691501

After Enron, many companies were forced by increased investor criticism and regulator scrutiny to improve the clarity of their financial disclosures. For example, IBM announced that it would begin providing more detail regarding its “Other gains and losses.” It had previously included these items in its selling, general, and administrative expenses, with little disclosure. Disclosing other gains and losses in a separate line item on the income statement won’t have any effect on bottom line income. However, analysts complained that burying these details in the selling, general, and administrative expense line reduced their ability to fully understand how well IBM was performing. For example, previously, if IBM sold off one of its buildings at a gain, it would include this gain in the selling, general, and administrative expense line item, thus reducing that expense. This made it appear that the company had done a better job of controlling operating expenses than it actually had. Other companies recently announcing changes to increase the in formativeness of their income statements included PepsiCo, Krispy Kreme Doughnuts, and General Electric. Why have investors and analysts demanded more accuracy in isolating “Other gains and losses” from operating items?

presented below is information related to reynolds gourmet foods company use it to p 691504

Presented below is information related to Reynolds Gourmet Foods Company. Use it to Prepare a statement of cash flows using the indirect method.

REYNOLDS GOURMET FOODS COMPANY

Comparative Balance Sheets

December 31

Assets

2009

2008

Change Increase/Decrease

Cash

$54,000

$37,000

$ 17,000 Increase

Accounts receivable

68,000

26,000

42,000 Increase

Inventories

54,000

–0–

54,000 Increase

Prepaid expenses

4,000

6,000

2,000 Decrease

Land

45,000

70,000

25,000 Decrease

Buildings

200,000

200,000

–0–

Accumulated depreciation—buildings

21,000

11,000

10,000 Increase

Equipment

193,000

68,000

125,000 Increase

Accumulated depreciation—equipment

28,000

10,000

18,000 Increase

Totals

$569,000

$386,000

 

Liabilities and Stockholders’ Equity

 

Accounts payable

$23,000

$40,000

$ 17,000 Decrease

Accrued expenses payable

10,000

–0–

10,000 Increase

Bonds payable

110,000

150,000

40,000 Decrease

Common stock ($1 par)

220,000

60,000

160,000 Increase

Retained earnings

206,000

136,000

70,000 Increase

Totals

$569,000

$386,000

 

 

 

REYNOLDS GOURMET FOODS COMPANY

Income Statement

For the Year Ended December 31, 2009

Revenues

 

$890,000

Cost of goods sold

$465,000

 

Operating expenses

221,000

 

Interest expense

12,000

 

Loss on sale of equipment

2,000

700,000

Income from operations

190,000

 

Income tax expense

65,000

 

Net income

$125,000

 

       

Additional information:

1. Operating expenses include depreciation expense of $33,000 and charges from prepaid expenses of $2,000.

2. Land was sold at its book value for cash.

3. Cash dividends of $55,000 were declared and paid in 2009.

4. Interest expense of $12,000 was paid in cash.

5. Equipment with a cost of $166,000 was purchased for cash. Equipment with a cost of $41,000 and a book value of $36,000 was sold for $34,000 cash.

6. Bonds of $10,000 were redeemed at their book value for cash. Bonds of $30,000 were converted into common stock.

7. Common stock ($1 par) of $130,000 was issued for cash. 8. Accounts payable pertain to merchandise suppliers.

the income statement for the year ended december 31 2008 for tuscany bay resort cont 691505

The income statement for the year ended December 31, 2004, for Tuscany Bay Resort contains the following condensed information.
 
border=0
 
 
Included in operating expenses is a $24,000 loss resulting from the sale of guest room furniture for $270,000 cash. Guest room furniture was purchased at a cost of $750,000.
The following balances are reported on Tuscany Bay’s comparative balance sheets at
 
December 31.
border=0
 
Income tax expense of $353,000 represents the amount paid in 2004. Dividends declared
and paid in 2004 totaled $200,000.
 
Instructions
(a) Prepare the statement of cash flows using the indirect method.
OR
(b) Prepare the statement of cash flows using the direct method.

 

in its income statement for the year ended december 31 2008 bach resort amp spa repo 691506

In its income statement for the year ended December 31, 2008, Bach Resort & Spa reported the following condensed data.

Administrative expenses

$ 435,000

Selling expenses

$ 490,000

Cost of goods sold

1,289,000

Loss on sale of equipment

10,000

Interest expense

70,000

Net sales

2,342,000

Interest revenue

28,000

 

 

Instructions

(a) Prepare a multiple step income statement.

(b) Prepare a single step income statement.

the t accounts for equipment and the related accumulated depreciation for stone kitc 691507

The T accounts for Equipment and the related Accumulated Depreciation for Stone Kitchen Equipment Company at the end of 2008 are as follows.

Equipment

Equipment

Accumulated Depreciation

Equipment

             

Beg. bal.

80,000

Disposals

22,000

Disposals

5,500

Beg. bal.

44,500

Acquisitions

41,600

       

Depr.

12,000

End. bal.

99,600

       

End. bal.

51,000

Stone Kitchen Equipment Company’s income statement reported a loss on the sale of equipment of $4,900.What amount was reported on the statement of cash flows as “cash flow from sale of equipment

antoine winery had the following transactions during 2008 1 issued 50 000 par value 691511

Antoine Winery had the following transactions during 2008:

1. Issued $50,000 par value common stock for cash.

2. Collected $11,000 of accounts receivable.

3. Declared and paid a cash dividend of $25,000.

4. Sold a long term investment with a cost of $15,000 for $15,000 cash.

5. Issued $200,000 par value common stock upon conversion of bonds having a face value of $200,000.

6. Paid $14,000 on accounts payable.

7. Purchased a machine for $30,000, giving a long term note in exchange

Instructions

Analyze the seven transactions, and indicate whether each transaction resulted in a cash flow from (a) operating activities, (b) investing activities, (c) financing activities, or (d) noncash investing and financing activities.

gerald d engle hart steakhouse has the following inventory purchases and sales data 691557

Gerald D. Engle hart Steakhouse has the following inventory, purchases, and sales data on its premium individually packaged beef for the month of March:

Inventory:

 

March 1

 

200 lbs @ $4.00

 

$ 800

 

Purchases:

 

 

 

 

 

March 10

500 lbs @ $4.50

2,250

 

March 20

400 lbs @ $4.75

1,900

 

March 30

300 lbs @ $5.00

1,500

Sales:

 

 

 

 

 

March 15

500 lbs

 

 

March 25

400 lbs

 

 

The physical inventory count on March 31 shows 500 pounds on hand.

Instructions

Under a periodic inventory system, determine the cost of inventory on hand at March 31 and the cost of goods sold for March under the (a) first in, first out (FIFO) method, (b) last in, first out (LIFO) method, and (c) average cost method.

abbott rsquo s liquor store uses a perpetual inventory system data for the texas cha 691560

Abbott’s Liquor Store uses a perpetual inventory system. Data for the Texas Chardonnay include the following purchases:

Date

Number of Bottles

Unit Price

May 7

50

$10

July 28

30

13

On June 1, Abbott’s sold 30 bottles, and on August 27, 35 more bottles. Prepare the perpetual inventory calculations for the above transactions using (a) FIFO, (b) LIFO, and (c) average cost.

sharpers carry a special french wine that is popular with wine enthusiasts below is 691561

Sharpers carry a special French wine that is popular with wine enthusiasts. Below is information relating to Sharper’s purchases of this wine during September? During the same month, 124 bottles were sold. A sharper uses a periodic inventory system.

 

Date

Explanation

Bottle

Unit cost

Total Cost

Sept. 1

Inventory

26

$ 97

$ 2,522

Sept. 12

Purchases

45

102

4,590

Sept. 19

Purchases

20

104

2,080

Sept. 26

Purchases

50

105

5,250

 

Totals

141

 

$14,442

Instructions

(a) Compute the ending inventory at September 30 using the FIFO and the LIFO methods. Prove the amount allocated to cost of goods sold under each method.

(b) For both FIFO and LIFO, calculate the sum of ending inventory and cost of goods sold. What do you notice about the answers you found for each method?

zambia pastry uses a periodic inventory system its records show the following for th 691562

Zambia Pastry uses a periodic inventory system. Its records show the following for the month of May, in which 70 units were sold.

   

Unit

Unit Cost

Total Cost

1 May

Inventory

30

$8

$240

15

Purchases

25

11

275

24

Purchases

35

12

420

 

Totals

90

 

$935

Instructions

Compute the ending inventory at May 31 using the FIFO and the LIFO methods. Prove the amount allocated to cost of goods sold under each method.

j k leaks wholesale corp uses the lifo method of inventory costing in the current ye 691564

J. K. Leaks Wholesale Corp. uses the LIFO method of inventory costing. In the current year, profit at J. K. Leaks is running unusually high. The corporate tax rate is also high this year, but it is scheduled to decline significantly next year. In an effort to lower the current year’s net income and to take advantage of the changing income tax rate, the president of J. K. Leaks Wholesale instructs the accountant to recommend to the purchasing department a large purchase of inventory for delivery three days before the end of the year. The price of the inventory to be purchased has doubled during the year, and the purchase will represent a major portion of the ending inventory value.

Instructions

(a) What is the effect of this transaction on this year and next year’s income statement and income tax expense? Why?

(b) If J. K. Leaks Wholesale had been using the FIFO method of inventory costing, would the president give the same directive?

(c) Should the plant accountant order the inventory purchase to lower income? What are the ethical implications of this order?

w s blaskowski foodservices entered into the following transactions during may 2008 691444

W. S. Blaskowski Foodservices entered into the following transactions during May 2008.

1. Purchased kitchen equipment for $30,000 from General Appliances on account.

2. Paid $3,000 cash for May rent.

3. Received $5,000 cash from customers for services billed in April.

4. Provided services to the Osborne’ 50th wedding  anniversary.

5. Paid World Web Services $5,000 cash for advertising in May.

6. Stockholders invested an additional $50,000 in the business.

7. Paid General Appliances for the equipment purchased in transaction (1).

8. Incurred consulting expense for May of $2,000 on account.

Instructions

Indicate with the appropriate letter whether each of the preceding transactions results in

(a) an  increase in assets and a decrease in assets.

(b) an  increase in assets and an increase in stockholders’ equity.

(c) an increase in assets and an increase in liabilities.

(d) a decrease in assets and a decrease in stockholders’ equity.

(e) a  decrease in assets and a decrease in liabilities.

(f) an increase in liabilities and a decrease in stockholders’ equity.

(g) an increase in stockholders’ equity and a decrease in liabilities.

on june 1 maui travel agency inc was established the following transactions were com 691445

On June 1, Maui Travel Agency, Inc., was established. The following transactions were completed during the month.

1. Stockholders invested $40,000 cash, receiving common stock in exchange 

2. Paid $1,000 cash for June office rent.

3. Purchased office equipment for $5,000 cash.

4. Paid $1,000 cash for office supplies.

5. Incurred $500 of advertising costs in the Daily Maui, on account.

6. Earned $25,000 for services rendered: Cash of $12,000 is received from customers, and the balance of $13,000 is billed to customers on account.

7. Paid $750 cash dividend.

8. Paid Daily Maui amount due in transaction (4).

9. Paid employees’ salaries $2,500.

10. Cash of $10,000 is received from customers who have previously been billed in transaction (6).

raisin buns inc recorded the following transactions in a general journal during the 691451

Raisin Buns, Inc., recorded the following transactions in a general journal during the month of March.

Cash

2,280

 

Service Revenue

 

2,280

Wages Expense

400

 

Cash

 

400

Utilities Expense

92

 

Cash

 

92

Post these entries to the Cash account of the general ledger to determine the ending balance in cash. The beginning balance in cash on March 1 was $600.

Q24

Refer back to the Feature Story about The Mug and Musket at the beginning of this

Chapter, and answer the following questions. 1. What accounting entries would Tanis likely make to record?

(a) the receipt of cash from a customer in payment of his or her bill, ( b) payment of a utility bill, and (c)payment of wages for the waiters?

2. How did Tanis’s job as director of finance help in her studies as she finished her business administration degree?

bob sample and other student investors opened the campus laundromat inc on september 691452

Bob Sample and other student investors opened the Campus Laundromat, Inc., on September 1, 2008. During the first month of operations, the following transactions occurred:

Sept. 1

 

Stockholders invested $20,000 cash in the business.

 

2

Paid $1,000 cash for store rent for the month of September.

3

 

Purchased washers and dryers for $25,000, paying $10,000 in cash and signing a $15,000, 6 month, 12% note payable.

4

Paid $1,200 for a one year accident insurance policy.

10

Received bill from the Daily News for advertising the opening of the Laundromat $200.

20

Declared and paid a cash dividend to stockholders $700.

30

 

Determined that cash receipts for laundry fees for the month were $6,200.

 

The chart of accounts for the company is the same as for Premier Staffing Agency, Inc., except for the following: No. 154 Laundry Equipment and No. 610 Advertising Expense.

Instructions

(a) Journalize the September transactions. (Use J1 for the journal page number.)

(b) Open ledger accounts and post the September transactions.

(c) Prepare a trial balance at September 30, 2008.

transactions for milton rsquo s wings and pizza for the month of june are presented 691453

Transactions for Milton’s Wings and Pizza for the month of June are presented below. Identify the accounts to be debited and credited for each transaction, and journalize the transactions.

June 1

Milton Wolfe invests $20,000 cash in exchange for shares of common stock in a small printing corporation

2

Buys equipment  on account for $8,600

3

Pays $1,200 to  landlord for June rent

12

$890  for 12 Bills B. J. Chang a pizza party

oruro johnson has the following transactions during august of the current year indic 691454

Oruro Johnson has the following transactions during August of the current year. Indicate (a) the basic analysis and (b) the debit–credit analysis illustrated on pages 82–86 of the text. Journalize the transactions.

Aug. 1

 Opens an office as a travel agent, investing $25,000 in cash in exchange for common stock.

4

Pays insurance in advance for 6 months, $1,200.

16

Receives $5,000 from clients for arranging travel services

27

Pays secretary $1,500 salary 

selected transactions for y k catering are presented in journal form below post the 691455

Selected transactions for Y. K. Catering are presented in journal form below. Post the transactions to T accounts.

Date

 

Account Titles and Explanation

 

Ref.

 

Debit

 

Credit

 

May 10

Accounts Receivable

 

7,200

 

 

Service Revenue

 

 

7,200

15

Cash

 

5,500

 

 

Accounts Receivable

 

 

5,500

20

Cash

 

2,500

 

 

Service Revenue

 

 

2,500

an inexperienced bookkeeper prepared the following trial balance that does not balan 691456

An inexperienced bookkeeper prepared the following trial balance that does not balance. Prepare a correct trial balance, assuming all account balances are normal.

ELIZABETH BAKERY

Trial Balance

December 31, 2008

 

Debit

Credit

Cash

$33,600

 

Prepaid Insurance

 

$4,900

Accounts Payable

 

7,500

Unearned Fees

5,800

 

Common Stock

20,000

 

Dividends

 

4,000

Service Revenue

 

29,200

Salaries Expense

17,000

 

Rent Expense

 

3,000

 

$76,400

$48,600

evergreen campground was started on june 1 by jolene and david collier the following 691458

Evergreen Campground was started on June 1 by Jolene and David Collier. The following selected events and transactions occurred during June.

June 1

Invested $150,000 cash in the business in exchange for common stock.

4

Purchased land costing $40,000 for cash.

8

Incurred advertising expense of $3,500 on account.

11

Paid salaries to employees $1,500.

12

Hired Campground manager at a salary of $5,000 per month, effective July 15.

13

Paid $2,000 for a one year insurance policy.

20

Received $9,000 in cash for admission fees.

30

Received $5,000 in cash for admission fees.

30

Paid $1,000 on account for advertising incurred on June 8.

Evergreen uses the following accounts: Cash; Prepaid Insurance; Land; Accounts Payable; Unearned Admissions; Common mission Revenue Advertising Expense; and Salaries Expense.

Instructions Journalize the April transactions.

alyson clark is a licensed incorporated cpa during the first month of operations of 691459

Alyson Clark is a licensed incorporated CPA. During the first month of operations of the business, the following events and transactions occurred:

Sept. 1

Invested $40,000 cash in exchange for common stock.

2

Hired a secretary receptionist at a salary of $2,480 per month.

3

Purchased $1,500 of supplies on account from Ross Supply Company.

7

Paid office rent of $1,500 for the month.

11

Completed a tax assignment and billed client $1,900 for services rendered.

12

Received $7,000 advance on a management consulting engagement.

17

Received cash of $2,350 for services completed for Mutter Company.

31

Paid secretary receptionist $2,480 salary for the month.

31

Paid 40% of balance due Ross Supply Company.

The company uses the following chart of accounts: No. 101 Cash, No. 112 Accounts Receivable, No. 126 Supplies, No. 201 Accounts Payable, No. 205 Unearned Revenue, No. 311 Common Stock, No. 400 Service Revenue, No. 726 Salaries Expense, and No. 729 Rent Expense.

Instructions

(a) Journalize the transactions.

 (c) Prepare a trial balance on September 30, 2008.

financial reporting problem pepsico the financial statements of pepsico are presente 691461

FINANCIAL REPORTING PROBLEM: PepsiCo

The financial statements of PepsiCo are presented in Appendix A. The statements contain the following selected accounts for the year ended, 2006 stated in millions of dollar

Accounts Payable

$6,496

Income Taxes Payable

$90

Accounts Receivable

3,725

Interest Expense

239

Property, Plant, and Equipment

9,687

Inventory

1,926

Instructions

(a) Answer the following questions.

(1) What is the increase and decrease side for each account?

(2) What is the normal balance for each account?

(b) Identify the probable other account in the transaction and the effect on that account when:

(1) Accounts Receivable is decreased.

(2) Accounts Payable is decreased.

(3) Inventory is increased.

(c) Identify the other accounts (s) that ordinarily would be involved when:

(1) Interest Expense is increased.

(2) Property, Plant, and Equipment is increased.

Remember to go back to the Navigator box on the chapter opening page and

accounting in action insight suppose you are a filmmaker like george lucas and spend 691462

Accounting In Action Insight

Suppose you are a filmmaker like George Lucas and spend $11 million to produce a film such as Star Wars. Over what period should the cost be expensed? It should be expensed over the economic life of the film. But what is its economic life? The filmmaker must estimate how much revenue will be earned from box office sales, video sales, television, and games and toys—a period that could be less than a year or more than twenty years, as is the case for Twentieth Century Fox’s Star Wars. Originally released in 1977 and rereleased in 1997, domestic revenues total nearly $500 million for Star Wars and continue to grow. This situation demonstrates the difficulty of properly matching expenses to revenues.

What accounting principle does this example illustrate? How will financial results be affected if the expenses are recognized over a period that is less than that used for revenues? What if the expenses are recognized over a period that is longer than that used for revenues?

accounting and action business insight in the past the costs of media advertising fo 691463

Accounting And Action Business Insight In the past, the costs of media advertising for burgers, bleaches, athletic shoes,

and such products were sometimes recorded as assets and expensed in subsequent periods as sales took place. The reasoning behind this treatment was that long ad campaigns provided benefits over multiple accounting periods. Today,

this treatment is no longer allowed. Instead, advertising costs must be expensed when the advertising takes place. The issue is important because the outlays for advertising can be substantial. Recent big spenders: Coca Cola spent $2 billion; PepsiCo, Inc., $1.7 billion; Campbell Soup Company, $1.7 billion: and JC Penney Company, $947 million.

Why do you think current accounting rules require that the cost of long ad campaigns be expensed rather than recorded as an asset?

accounting matter ethic insight companies would rather report steadily increasing pr 691464

Accounting Matter! Ethic Insight

Companies would rather report steadily increasing profits than fluctuating profits. To “smooth” earnings, companies sometimes shift the reporting of revenues or expenses between periods. A Wall Street Journal article reported that Microsoft

Corp. agreed to settle Securities and Exchange Commission charges that it misstated its earnings in some years by illegally maintaining different “reserve” accounts for such expenses as marketing and obsolete inventory. The settlement did not require Microsoft to pay a fine. Microsoft accepted the commission’s order without admitting or denying wrongdoing and agreed not to commit accounting violations. “The SEC said Microsoft maintained undisclosed reserve accounts totaling between $200 million and $900 million between 1994 and 1998 and didn’t maintain proper internal controls to document them or substantiate

their size.” The SEC said that the improper use of these reserve accounts resulted in “material inaccuracies” in the financial reports filed with the SEC. What accounting principles do you think Microsoft violated? What did the SEC mean

by “material inaccuracies”? Why would a company prefer to report steadily increasing profits rather than fluctuating profits?

the ledger of hammond inc on march 31 2008 includes the following selected accounts 691465

The ledger of Hammond, Inc., on March 31, 2008, includes the following selected accounts before adjusting entries.

 

Debit

Credit

Prepaid Insurance

3,600

 

Office Supplies

2,800

 

Office Equipment

25,000

 

Accumulated Depreciation—Office Equipment

 

5,000

Unearned Revenue

 

9,200

An analysis of the accounts shows the following:

1. Insurance expires at the rate of $100 per month.

2. Supplies on hand total $800.

3. The office equipment depreciates $200 a month.

4. One half of the unearned revenue was earned in March.

Prepare the adjusting entries for the month of March.

a look at our feature story refer back to the feature story about lake swan resort j 691468

A Look At Our Feature Story

Refer back to the Feature Story about Lake Swan Resort, Juneau Country Club, Cuisine on the Go, and Hospitality Purveyors at the beginning of this chapter, and answer the following questions: 1. What are the purposes of dusting entries? 2. What do these four companies have in common relative to accrual accounting? 3. What other types of adjusting entries do you believe these companies might make?

A Look At Our Feature Story

In Chapter 1 you learned a neat little formula: Net income = Revenues – Expenses. And in Chapter 3 you learned some nice, orderly rules for recording corporate revenue and expense transactions. Guess what? Things are not really that nice and neat. In fact, it is often difficult to determine in what time period some revenues and expenses should be reported. And, in measuring net income, timing is everything.

There are rules that give guidance on these issues. But occasionally, these rules are overlooked, misinterpreted, or even intentionally ignored. Think what the results would be if the following cases were true.

• Lake Swan Resort pays its general property insurance on a six month basis. The total of $18,000 for a six month period, or $3,000 per month, is paid on May 30 for June to November. The entire amount of $18,000 is recorded as an expense for the month of May because it is paid on May 30. Thus the company has a loss of $7,533 for that month.

• Juneau Country Club gives its members an option to pay the entire year’s membership with a 5 percent discount. Dues that are received in December 2008 for the year 2009 are all entered at December 2008 as dues income.

• Cuisine on the Go receives in October a 50 percent deposit for a Christmas party in the amount of $7,525. When the accountant records the transaction, it is entered as catering income for the month of October.

• Hospitality Purveyors has its annual convention in Houston. One event is an off site event to NASA Johnson Space Center for a tour. Coach buses are contracted for this outing with Luxury Coach, Inc. A check for $3,685 is written to Luxury in May for its services to be provided in September. Luxury records the $3,685 as income for the month of May. In each case, accrual accounting concepts were violated. That is, revenues or expenses were not recorded in the proper period, which has a substantial impact on reported income. Their timing is off!

 

 width=

accounting in action business insight suppose you are a filmmaker like george lucas 691469

Accounting In Action Business Insight

Suppose you are a filmmaker like George Lucas and spend $11 million to produce a film such as Star Wars. Over what period should the cost be expensed? It should be expensed over the economic life of the film. But what is its economic life? The filmmaker must estimate how much revenue will be earned from box office sales, video sales, television, and games and toys—a period that could be less than a year or more than twenty years, as is the case for Twentieth Century Fox’s Star Wars. Originally released in 1977 and rereleased in 1997, domestic revenues total nearly $500 million for Star Wars and continue to grow. This situation demonstrates the difficulty of properly matching expenses to revenues. What accounting principle does this example illustrate? How will financial results be affected if the expenses are recognized over a period that is less than that used for revenues? What if the expenses are recognized over a period that is longer than that used for revenues?

accounting matter ethic insight companies would rather report steadily increasing pr 691471

Accounting Matter Ethic Insight

Companies would rather report steadily increasing profits than fluctuating profits. To “smooth” earnings, companies sometimes shift the reporting of revenues or expenses between periods’ Wall Street Journal article reported that Microsoft Corp. agreed to settle Securities and Exchange Commission charges that it misstated its earnings in some years by illegally maintaining different “reserve” accounts for such expenses as marketing and obsolete inventory. The settlement did not require Microsoft to pay a fine. Microsoft accepted the commission’s order without admitting or denying wrongdoing and agreed not to commit accounting violations. “The SEC said Microsoft maintained undisclosed reserve accounts totaling between $200 million and $900 million between 1994 and 1998 and didn’t maintain proper internal controls to document them or substantiate their size.” The SEC said that the improper use of these reserve accounts resulted in “material inaccuracies” in the financial reports filed with the SEC. What accounting principles do you think Microsoft violated? What did the SEC mean by “material inaccuracies”? Why would a company prefer to report steadily increasing profits rather than fluctuating profits?

spring river resort inc opened for business on june 1 with eight air conditioned uni 691477

Spring River Resort Inc. opened for business on June 1 with eight air conditioned units. Its trial balance before adjustment on August 31 is as follows

SPRING RIVER RESORT INC.

Trial Balance

August 31, 2008

Account

Number

     

 

 

Debit

Credit

 

101

Cash

$19,600

 

 

126

Supplies

3,300

 

 

130

Prepaid Insurance

6,000

 

 

140

Land

25,000

 

 

143

Cottages

125,000

 

 

149

Furniture

26,000

 

 

201

Accounts Payable

 

$6,500

208

Unearned Rent

 

7,400

275

Mortgage Payable

 

80,000

311

Common Stock

 

100,000

320

Retained Earnings

 

–0–

332

Dividends

5,000

 

 

429

Rent Revenue

 

80,000

622

Repair Expense

3,600

 

 

726

Salaries Expense

51,000

 

 

732

Utilities Expense

9,400

 

 

   

$273,900

$273,900

 

           

In addition to those accounts listed on the trial balance, the chart of accounts for Spring River Resort also contains the following accounts and account numbers: No. 112 Accounts Receivable, No 144 Accumulated Depreciation—Cottages, No. 150 Accumulated Depreciation— Furniture, No. 212 Salaries Payable, No. 230 Interest Payable, No. 620 Depreciation Expense— Cottages, No. 621 Depreciation Expense—Furniture, No. 631 Supplies Expense, No. 718 Interest Expense, and No. 722 Insurance Expense.

Other data:

1. Insurance expires at the rate of $400 per month.

 2. A count of August 31 shows $900 of supplies on hand.

3. Annual depreciation is $3,600 on cottages and $2,400 on furniture.

4. Unearned rent of $4,100 was earned prior to August 31.

5. Salaries of $400 were unpaid at August 31.

6. Rentals of $800 were due from tenants at August 31. (Use  Accounts Receivable.)

7. The mortgage interest rate is 9 percent per year. (The mortgage was taken out on  August 1.)

Instructions

Journalize the adjusting entries on August 31 for the three month period June 1–

August 31.

jan spears opened her decorating company on january 1 2008 during the first month of 691478

Jan Spears opened her decorating company on January 1, 2008. During the first month of operations, the following transactions occurred:

1. Performed services for country club clients. On January 31, $2,300 of such services was earned but not yet billed to the clubs.

2. Utility expenses incurred but not paid prior to January 31 totaled $650.

3. Purchased decorating supplies on January 1 for $50,000, paying $10,000 in cash and signing a $40,000, three year note payable. Interest is $300 per month.

4. Purchased a one year fire insurance policy on January 1 for $6,000.

5. Purchased a computer at $2,100. On January 31, determined that $200 of the computer had been depreciated.

Instructions

Prepare the adjusting entries on January 31. Account titles are: Accumulated Depreciation— Furniture, Depreciation Expense, Service Revenue, Accounts Receivable, Insurance Expense, Interest Expense, Interest Payable, Prepaid Insurance, Supplies, Supplies Expense, Utilities Expense, and Utilities Payable

flexible budget oak hill township operates a motor pool with 20 vehicles the motor p 691411

Flexible Budget

Oak Hill Township operates a motor pool with 20 vehicles. The motor pool furnishes gasoline, oil, and other supplies for the cars and hires one mechanic who does routine maintenance and minor repairs. Major repairs are done at a nearby commercial garage. A supervisor manages the operations.

Each year, the supervisor prepares a master budget for the motor pool. Depreciation on the automobiles is recorded in the budget to determine the costs per mile.

The following schedule presents the master budget for the year and for the month of July.

OAK HILL TOWNSHIP

Motor Pool

Budget Report for July

 

Annual

One Month

 

Over or

 

Master

Master

July

(Under )

 

Budget

Budget

Actual

Budget

Gasoline

$ 81,000

$ 6,750

$ 8,515

$1,765

Oil, minor repairs, parts, and supplies

7,200

600

760

160

Outside repairs

5,400

450

100

(350)

Insurance

12,000

1,000

1,050

50

Salaries and benefits

60,000

5,000

5,000

–0–

Depreciation

52,800

4,400

4,620

220

Total cost

$218,400

$18,200

$20,045

$1,845

Total miles

900,000

75,000

94,500

 width=

Cost per mile

$0.2427

$0.2427

$0.2121

 

Number of automobiles

20

20

21

 

The annual budget was based on the following assumptions:

1. Automobiles in the pool: 20.

2. Miles per year per automobile: 45,000.

3. Miles per gallon per automobile: 20.

4. Gas per gallon: $1.80.

5. Oil, minor repairs, parts, and supplies per mile: $0.008.

6. Outside repairs per automobile per year: $270.

The supervisor is unhappy with the monthly report, claiming that it unfairly presents his performance for July. His previous employer used flexible budgeting to compare actual costs to budgeted amounts.

Required

a. What is the gasoline monthly flexible budget and the resulting amount over or underbudget? (Use miles as the activity base.)

b. What is the monthly flexible budget for the oil, minor repairs, parts, and supplies and the amount over or underbudget? (Use miles as the activity base.)

c. What is the monthly flexible budget for salaries and benefits and the resulting amount over or underbudget?

d. What is the major reason for the cost per mile to decrease from $0.2427 budgeted to $0.2121 actual?

racketeer inc comprehensive overview of budgets and variance ldquo i just don rsquo 691412

Racketeer, Inc. (Comprehensive Overview of Budgets and Variance)

“I just don’t understand these financial statements at all!” exclaimed Mr. Elmo Knapp. Mr. Knapp explained that he had turned over management of Racketeer, Inc., a division of American Recreation Equipment, Inc., to his son, Otto, the previous month. Racketeer, Inc., manufactures tennis rackets.

“I was really proud of Otto,” he beamed. “He was showing us all the tricks he learned in business school, and if I do say so myself, I think he was doing a rather good job for us. For example, he put together this budget for Racketeer, which makes it very easy to see how much profit we’ll make at any sales volume (Exhibit 17.8). As best as I can figure it, in March we expected to have a volume of 8,000 units and a profit of $14,500 on our rackets. But we did much better than that! We sold 10,000 rackets, so we should have made almost $21,000 on them.”

Exhibit 17.8 Profit Graph—Racketeer, Inc.

 width=

Exhibit 17.9

Standard Costsa— Racketeer, Inc.

 

Per Racket

Raw material

 

Frame (one frame per racket)

$3.15

Stringing materials: 20 feet at 3¢ per foot

 

Direct labor

0.60

Skilled: 1/8 hour at $9.60 per hour

1.20

Unskilled: 1/8 hour at $5.60 per hour

0.70

Plant overhead

 

Indirect labor

0.10

Power

0.03

Supervision

0.12b

Depreciation

0.20b

Other

0.15b

Total standard cost per frame

$6.25

“Another one of Otto’s innovations is this standard cost system,” said Mr. Knapp proudly. “He sat down with our production people and came up with a standard production cost per unit (see  Exhibit 17.9). He tells me this will let us know how well our production people are performing. Also, he claims it will cut down on our clerical work.”

Mr. Knapp continued, “But one thing puzzles me. My calculations show that we should have earned profit of nearly $21,000 in March. However, our accountants came up with less than $19,000 in the monthly income statement (Exhibit 17.10). This bothers me a great deal. Now, I’m sure our accountants are doing their job properly. But still, it appears to me that they’re about $2,200 short.

“As you can probably guess,” Mr. Knapp concluded, “we are one big happy family around here. I just wish I knew what those accountants were up to . . . coming in with a low net income like that.”

Required

Prepare a report for Mr. Elmo Knapp and Mr. Otto Knapp that reconciles the profit graph with the actual results for March (see Exhibit 17.11). Show the source of each variance from the original plan (8,000 rackets) in as much detail as you can and evaluate Racketeer’s performance in March. Recommend improvements in Racketeer’s profit planning and control methods.

Exhibit 17.10 Income Statement, March—Racketeer, Inc.

RACKETEER, INC.

Income Statement,

For the Month of March—Actual

 

Sales

 

10,000 rackets at $9

$90,000

Standard cost of goods sold 10,000 rackets at $6.25

62,500

Gross profit after standard costs

$27,500

Variances

 

Materials variance

(490)

Labor variance

(392)

Overhead variance

(660)

Gross profit

$25,958

Selling and administrative expenses

7,200

Operating profit

$18,758

Exhibit 17.11 Actual Production Data for March—Racketeer, Inc.

Direct materials purchased and used

Stringing materials

175,000 feet at 2.5¢ per foot

Frames (some frames were ruined during production)

7,100 at $3.15 per frame

Labor

 

Skilled ($9.80 per hour)

900 hours

Unskilled ($5.80 per hour)

840 hours

Overhead

 

Indirect labor

$ 800

Power

$ 250

Depreciation

$1,600

Supervision

$ 960

Other

$1,250

Production

7,000 rackets

jyc investments selected six performance measures link the six measures to the four 691414

JYC Investments selected six performance measures. Link the six measures to the four perspectives of the balanced scorecard by filling in the following blanks.

1. Financial

Employee satisfaction

Linked to Perspective Number?

2. Customer

Branch costs

_________________________

3. Learning and growth

Branch profitability

_________________________

4. Internal business processes

Employee learning

_________________________

 

Regulatory violations

_________________________

 

Customer satisfaction

_________________________

balanced scorecards and strategy maps tech masters inc has the following mission sta 691416

Balanced Scorecards and Strategy Maps

Tech Masters, Inc., has the following mission statement:

To be the leader in innovation and new products for the industrial control instruments market.

Tech Master’s CFO, who has just returned from an executive seminar on performance measurement, tells you that she has developed the following initial balanced scorecard as part of an exercise during class:

Perspective

Objectives

Perspective

Objectives

Financial

• Increased profitability

Internal

• Increased cycle

 

 

 

efficiency

 

• Increased market share

 

 

Customer

• Repeat orders

Learning and growth

• Employee satisfaction

 

• Customer referrals

 

• Employee retention

Required

Comment on the scorecard and make recommendations about adding or deleting measures to align the scorecard with the company’s strategy as defined by its mission statement.

functional measures for each category of functional measures listed in exhibit 18 7 691417

Functional Measures

For each category of functional measures listed in Exhibit 18.7, add one additional specific measurement that is not already listed.

Exhibit 18.7 Functional Measures of Performance

Accounting Quality

• Percentage of late reports

• Percentage of errors in reports

• Percentage of errors in budget predictions

• Degree of manager satisfaction with accounting reports

Clerical Quality

• Number of errors per typed page

• Number of times messages are not delivered

• Quality of product/development engineering

• Percentage of errors in cost estimates

• Degree to which product meets customer expectations

Forecasting Quality

• Percentage of errors in sales forecasts

• Number of forecasting assumption errors

• Decision maker satisfaction with forecasts

Procurement/Purchasing Quality

• Percentage of supplies delivered on schedule

• Average time to fill emergency orders

Production Control Quality

• Time required to incorporate engineering changes

• Time that assembly line is down due to materials shortage

Quality Assurance Quality

• Wait time on customer inquiries

• Wait time on customer support calls

• Time to answer customer complaints

Sources: Most of these measures are drawn from much longer lists in G. Fellers, The Deming Vision: SPC/TQM for Administrators (Milwaukee, WI: ASQC Quality Press, 1992); and D. Talley, Total Quality Management (Milwaukee, WI: ASQC Quality Press, 1991). Some of these measures were developed by the authors.

partial productivity measures the food services manager at north central university 691418

Partial Productivity Measures

The Food Services manager at North Central University (NCU) was looking at data from cafeterias in four different academic schools at NCU: Business, Engineering, Humanities & Sciences (H&S), and Music. She was curious to see how the different cafeterias were performing. She had the following data from the previous semester:

 

Business

Engineering

H&S

Music

Labor hours

14,000

22,000

31,000

2,100

Meals served

38,000

72,000

114,000

4,500

Required

a. Compute the partial productivity measures for labor for the four schools.

b. Comment on the results. Are there factors other than efficiency that might affect the results?

partial productivity measures as the cost accounting manager at mckinley industries 691419

Partial Productivity Measures

As the cost accounting manager at McKinley Industries (MI), you are responsible for compiling and reporting various performance measures to the senior managers. MI instituted many efficiency improvement programs recently and the CFO has asked you to measure and report partial productivity for both labor and materials. Data for the last two years follow:

 

Year 2

Year 1

Gallons input

45,000

40,000

Labor hours

50,000

38,000

Output (gallons)

25,000

20,000

Required

a. Compute the partial productivity measures for labor for year 1 and year 2.

b. Compute the partial productivity measures for material for year 1 and year 2.

c. Comment on the results. Have the efficiency improvement programs resulted in greater productivity?

balanced scorecards and strategy maps hill street company hsc manufactures plastic p 691421

Balanced Scorecards and Strategy Maps

Hill Street Company (HSC) manufactures plastic parts for the home construction industry. The market is extremely competitive and margins are thin. The company recently adopted a balanced scorecard for performance evaluation. As part of that exercise, managers at HSC developed the following strategy map.

 width=

Required

a. Using the strategy map, comment on the performance measures used for each of the four perspectives of the scorecard. Would you recommend any changes to the measures? If so, what changes would you make? Why?

b. What are the strengths and weaknesses of the strategy map as developed by HSC?

c. Recommend changes to the map that will better communicate the strategy for HSC and incorporate your recommendations from requirement ( ).

balanced scorecards and strategy maps monroe corporation makes precision parts for b 691422

Balanced Scorecards and Strategy Maps

Monroe Corporation makes precision parts for boats and aircraft. Quality is an important competitive advantage in the industry and the company prides itself on the quality of its products. The company recently adopted a balanced scorecard for performance evaluation. Monroe uses the following strategy map to communicate its strategy to its managers and line employees:

 width=

Required

a. Using the strategy map, comment on the performance measures used for each of the four perspectives of the scorecard. Would you recommend any changes to the measures? If so, what changes would you make? Why?

b. What are the strengths and weaknesses of the strategy map as developed by Monroe Corporation?

c. Recommend changes to the map that will better communicate the strategy for Monroe Corporation and incorporate your recommendations from requirement (a).

performance measures drawing a business model garnet electronics discloses to you th 691423

Performance Measures, Drawing a Business Model

Garnet Electronics discloses to you that it uses a balanced scorecard with the following performance measures:

Profit

Training hours

Percentage of defects

Employee turnover

Customer satisfaction

Patents awarded

Required

a. Link the measures to the perspectives of the balanced scorecard by filling in the following table:

 width=

b. Present a business model as in Exhibit 18.1 that would lead the company to choose this set of measures.

Exhibit 18.1 Example Business Model—JYC Investments

 width=

operational performance measures zuma company manufactures surfboards the controller 691424

Operational Performance Measures

Zuma Company manufactures surfboards. The controller prepares a weekly production efficiency report and sends it to corporate headquarters. The data compiled in these reports for a recent six week period follow:

ZUMA COMPANY

Production Efficiency Report

 

Week

1

2

3

4

5

6

Percentage of manufacturing cycle efficiency

70

69

72

65

66

62

Percentage of on time deliveries

98

95

96

92

94

90

Number of customer complaints

20

18

22

25

23

27

Required

a. Write a memo to the company president evaluating the plant’s performance.

b. If you identify any areas of concern in your memo, indicate an appropriate action for management to take. Indicate any additional information you would like to have to make your evaluation.

operational performance measures mid states metal finishers produces steel tubing at 691426

Operational Performance Measures

Mid States Metal Finishers produces steel tubing at its Akron plant. The plant’s quality assurance officer prepares a monthly report and sends it to headquarters. The data compiled in these reports for a recent six month period follow:

MID STATES METAL FINISHERS

Production Efficiency Report

 

Month

Jan

Feb

Mar

Apr

May

Jun

Percentage of orders filled on time

87

86

88

92

91

92

Number of defective units, as a percent of total

7

6

2

1

3

2

Number of customer returns

20

21

18

10

6

5

Required

a. Write a memo to the company president evaluating the plant’s performance.

b. If you identify any areas of concern in your memo, indicate an appropriate action for management to take. Indicate any additional information you would like to have to make your evaluation.

productivity measures refer to the data in exercise 18 29 mckinley industries from t 691427

Productivity Measures

Refer to the data in Exercise 18 29 (McKinley Industries). From the accounting records, you also gather the following information for the two years:

 width=

Required

a. Compute the total factor productivity measures for year 1 and year 2 based on the three inputs (material, labor, and overhead).

b. Comment on the results.

c. Describe briefly the advantages and disadvantages of the total factor productivity relative to the partial measures computed in Exercise 18 29. Would it be useful to report both?

Exercise 18 29: Partial Productivity Measures

As the cost accounting manager at McKinley Industries (MI), you are responsible for compiling and reporting various performance measures to the senior managers. MI instituted many efficiency improvement programs recently and the CFO has asked you to measure and report partial productivity for both labor and materials. Data for the last two years follow:

 

Year 2

Year 1

Gallons input

45,000

40,000

Labor hours

50,000

38,000

Output (gallons)

25,000

20,000

Required

a. Compute the partial productivity measures for labor for year 1 and year 2.

b. Compute the partial productivity measures for material for year 1 and year 2.

c. Comment on the results. Have the efficiency improvement programs resulted in greater productivity?

employee involvement the interaction between customers and line employees is often m 691428

Employee Involvement

The interaction between customers and line employees is often more direct in service industries than in manufacturing fi rms. At the same time, we often observe that employees in many service firms (for example, in hotels and airlines) are given authority to respond to customer concerns without having to seek approval from their supervisors. Desk clerks in hotels or gate agents for airlines, for example, are often allowed to offer upgrades, early check ins, late check outs, special seats, and so on, as a way to apologize for or remedy problems. It is also a way to proactively build customer satisfaction.

Required

a. How can policies that allow employees to make these decisions help the organization? How might they harm the organization?

b. You observe these policies less frequently in other service organizations such as banks and other financial institutions. Why might this be the case?

balanced scorecards and strategy maps following several years of tight budgets admin 691429

Balanced Scorecards and Strategy Maps

Following several years of tight budgets, administrators at the University of California, Davis, looked for ways “to do more with less.” Janet Hamilton, vice chancellor of administration, researched books and articles, met with consultants, and talked to her counterparts at universities across the United States to find new management methods that could change the university from a bureaucratic to a customer oriented organization. She learned about reengineering, total quality, and a variety of other management techniques. None of the management techniques appealed to her, until she came across articles about the balanced scorecard. She believed that the balanced scorecard was the right tool for the Davis campus, and she set about implementing it.

At first, Hamilton did not call her approach a “balanced scorecard,” because she feared that employees would think of this as just another management fad to endure until the administration went on to something new. Instead, she pilot tested the balanced scorecard ideas in one service department, environmental health and safety (EHS), until it worked. With the success of EHS behind her, she moved to implement the balanced scorecard in other service departments, such as police, fire, and printing services.

Each department developed its own particular performance measures to achieve the following objectives (we have shortened the list to save space):

 width=

Required

a. Was the vice chancellor overly cautious in not calling her approach a “balanced scorecard”?

b. Comment on the wisdom of beginning a balanced scorecard with a pilot project. Would it bepossible to extrapolate the experience of a service department, such as environmental health and safety, to an academic unit, such as a college of business?

c. What opportunities and difficulties do you see in applying a balanced scorecard to a university setting?

accounting matters international insight research and development costs are an examp 691437

ACCOUNTING MATTERS! International insight

Research and development costs are an example of different international accounting standards. Compare how four countries account for research and development (R&D):

Country

Accounting Treatment

United States

Expenditures are expensed.

United Kingdom

Certain expenditures may be capitalized.

Germany

Expenditures are expensed.

Japan

Expenditures may be capitalized and written off over 5 years.  

Thus, an R&D expenditure of $100 million is charged totally to expense in the current period in the United States and Germany. This same expense could range from zero to $100 million in the United Kingdom and from $20 million to $100 million in Japan!

What would be the advantage of similar accounting standards for all countries? How can the financial and operating performance of international companies be compared?

a tabular analysis of the transactions made by roberta mendez amp co a decorating co 691438

A tabular analysis of the transactions made by Roberta Mendez & Co., a decorating consulting firm, for the month of August is shown below. Each increase and decrease in stockholders’ equity is explained.

Assets

=

Liabilities

+

Stockholders’ Equity

 

Cash

+

Office
Equipment

=

Accounts Payable

+

Common Stock 

+

Retained

1.

+25,000

         

+25,000

 

Earnings

2.

   

+7,000

           

3.

+8,000

             

+8,000

4.

850

             

850

Describe each transaction that occurred for the month.

ACTION PLAN

• Analyze the tabular analysis to determine the nature and effect of each transaction.

• Keep the accounting equation always in balance.

• Remember that a change in an asset will require a change in another asset, a liability, or stockholders’ equity.

attend feature story of pepsico financial reporting a matter of trust in recent year 691439

Attend

Feature Story of PepsiCo.

Financial Reporting:

A Matter of Trust

In recent years the financial press has been full of articles about financial scandals and accounting misdeeds.

It started with Enron, but then spread to Xerox, Qwest, Global Crossing, and WorldCom, among others. Many of the articles expressed concern that as an increasing number of misdeeds came to public attention, a mistrust of financial reporting in general was developing. These articles made clear just how important accounting and financial reporting are to the U.S. and world financial markets and to society as a whole. Without financial reports, managers would not be able to evaluate how well their company is doing or to make decisions about the best way to make their company grow in the future. Without financial reports, investors and lenders could not make informed decisions about how to allocate their funds. There is no doubt that a sound, well functioning economy depends on accurate and dependable financial reporting— accounting matters!

In order to make financial decisions as either an investor or a manager, you need to know how to read financial reports. In this book you will learn about financial reporting and some basic tools used to evaluate financial reports. In the first chapter we introduce you to the real financial statements of a company whose products most of you probably are familiar with—PepsiCo, Inc. We have chosen the financial statements of PepsiCo because they are a good example from the real world. Appendix A contains the statements in their entirety, and a copy of the PepsiCo, Inc., 2006 Annual Report accompanies this text. PepsiCo manufactures Pepsi Cola, the number two soft drink beverage in the world. PepsiCo also manufactures the number one bottled water (Aquafina), the number one sports drink (Gatorade), the number one ready to drink tea (Lipton), and the number one ready to drink coffee (Frappuccino). In addition, PepsiCo is the largest manufacturer of snack foods in the world. Its Frito Lay chips dominate the U.S. market with 59% of all snack chip sales and the world market with over 32%. In all, PepsiCo ranks among the world’s largest packaged goods and beverage companies, with over $25 billion in sales, $23 billion in assets, and 140,000 employees. PepsiCo is not only large; it is also quite profitable, ranking twenty eighth among all U.S. companies, with $3.3 billion in net income.

Refer above to the Feature Story about PepsiCo, and answer the following questions.

1. If you were interested in investing in PepsiCo, what would the balance sheet and income statement tell you?

2. Would you request audited financial statements? Explain.

3. Will the financial statements show the market value of the company? Explain.

hospitality legal services inc which provides contract services for caterers and the 691440

Hospitality Legal Services, Inc., which provides contract services for caterers and their clients, was incorporated on July 1, 2008. During the first month of operations, the following transactions occurred:

1. Stockholders invested $10,000 in cash in exchange for shares of stock.

2. Paid $800 for July rent on office space.

3. Purchased office equipment on account, $3,000.

4. Provided legal services to clients for cash, $1,500 (use Service Revenue).

5. Borrowed $700 cash from a bank on a note payable.

6. Performed legal services for client on account, $2,000.

7. Paid monthly expenses: salaries $500; utilities $300; and telephone $100.

Instructions

(a) Prepare a tabular summary of the transactions.

(b) Prepare the income statement, retained earnings statement, and balance sheet at July 31 for Hospitality Legal Services, Inc.

presented below are a number of operational guidelines and practices that have devel 691441

Presented below are a number of operational guidelines and practices that have developed over time.

Instructions

Identify the accounting assumption, accounting principle, or reporting constraint that most appropriately justifies these procedures and practices. Use only one item per description.

(a) The first note, “Summary of Significant Accounting Policies,” presents information on the sub classification of plant assets and discusses the company’s depreciation methods.

(b) The local hamburger restaurant expenses all spatulas, French fry baskets, and other cooking utensils when purchased.

(c) Retailers recognize revenue at the point of sale.

(d) Green Grow Landscape, Inc., includes an estimate of warranty expense in the year in which it sells its lawn mowers, which carry a 2 year warranty.

(e) Companies present sufficient financial information so that creditors and reasonably prudent investors will not be misled.

(f) Companies listed on U.S. stock exchanges report audited financial information annually and report unaudited information quarterly.

(g) Beach Resorts, Inc., does not record the 2004 value of $1.5 million for a piece of beach front property it purchased in 1989 for $500,000.

(h) Restaurant Supplies, Inc., takes a $32,000 loss on a number of older ovens in its inventory; it paid the manufacturer $107,000 for them but can sell them for only $75,000.

(i) Frito Lay is a wholly owned subsidiary of PepsiCo, Inc., and Frito Lay’s operating results and financial condition are included in the consolidated financial statements of PepsiCo. (Do not use full disclosure.)

comprehensive variance problem sweetwater company manufactures two products mountain 691382

Comprehensive Variance Problem

Sweetwater Company manufactures two products, Mountain Mist and Valley Stream. The company prepares its master budget on the basis of standard costs. The following data are for March:

Standards

Mountain Mist

Valley Stream

Direct materials

3 ounces at $15 per ounce

4 ounces at $16.50 per ounce

Direct labor

5 hours at $60 per hour

6 hours at $75 per hour

Variable overhead (per direct labor hour)

$48

$52.50

Fixed overhead (per month)

$335,340

$397,800

Expected activity (direct labor hours)

5,750

7,800

Actual results

 

 

Direct material (purchased and used)

3,100 ounces at $13.50 per ounce

4,700 ounces at $17.25 per ounce

Direct labor

4,900 hours at $60.75 per hour

7,400 hours at $76.50 per hour

Variable overhead

$242,550

$378,510

Fixed overhead

$313,950

$396,000

Units produced (actual)

1,000 units

1,200 units

Required

a. Prepare a variance analysis for each variable cost for each product.

b. Prepare a fixed overhead variance analysis for each product like the one in Exhibit 16.13.

Exhibit 16.13 Fixed Overhead Variances, August— Bayou Division

 width=

In this situation, a $40,000 unfavorable production volume variance exists. It is unfavorable because less overhead was applied than was budgeted; production was lower than the average monthly estimate. This variance is a result of the full absorption costing system; it does not occur in variable costing.

This $160,000 applied fixed overhead equals $2 per frame multiplied by 80,000 units actually produced (see Exhibit 16.14). If the $40 rate per direct labor hour had been used, the amount applied to the 80,000 units produced would still be $160,000 (= $40 x 0.05 x 80,000).

A variance occurs if the number of units actually produced differs from the number of units used to estimate the fixed cost per unit. Again, this variance is commonly referred to as a production volume variance (also called a capacity variance , an idle capacity variance , or a denominator variance ).

Our example has a production volume variance because the 80,000 frames actually produced during the month do not equal the 100,000 estimated for the month. Consequently, production is charged $160,000 (point A in Exhibit 16.14) instead of $200,000 (point B in Exhibit 16.14). The $40,000 difference is the production volume variance

Exhibit 16.14 Fixed Overhead Variances, Graphic Presentation—Bayou Division

 width=

because it is caused by a deviation in production volume level (number of frames produced) from that estimated to arrive at the standard cost.

If Bayou had estimated 80,000 frames per month instead of 100,000 frames, the standard cost would have been $2.50 per frame (= $200,000 ÷ 80,000 frames). Thus, $200,000 (= $2.50 x 80,000 frames) would have been applied to units produced, and there would have been no production volume variance.

The production volume variance applies only to fixed costs; it occurs because we are allocating a fixed period cost to units on a predetermined basis. It does not represent resources spent or saved. This is unique to full absorption costing. The benefits of calculating the variance for control purposes are questionable. Although the production volume variance signals a difference between expected and actual production levels, so does a simple production report of actual versus expected production quantities.

Compare with the Fixed Production Cost Price Variance The fixed production cost price variance is the difference between actual and budgeted fixed production costs. Unlike the production volume variance, the price variance commonly is used for control purposes because it is a measure of differences between actual and budgeted period costs.

Exhibits 16.13 and 16.14 summarize the computation of the fixed production price (spending) and production volume variances. Reviewing them will help you see the relationship between actual, budgeted, and applied fixed production costs.

agm com performance measurement and variances i thought the internet would be an ide 691383

agm.com: Performance Measurement and Variances

I thought the Internet would be an ideal way to distribute our products. We’ve had a lot of success with our direct sales, but now we can reach a much larger audience. The baskets we make and sell appeal to people everywhere. I thought about opening stores in other towns or maybe even franchising, but the Web offers me a way to expand without losing control.

That’s why the results for the first quarter of our Web based unit are so disappointing. We expected a small loss, because of marketing and other start up expenses, but I was not prepared for the beating we took.

Maya McCrum,

President and CEO

AGM Enterprises

Organization

AGM Enterprises is a small, family owned and managed company that produces and sells wooden baskets. The company was founded in 1947 in California by Autumn McCrum as a way of supplementing the family income. The business remained small until 1990, when Maya McCrum took it over from her mother. Until that time, all orders were taken by the senior Ms. McCrum and all baskets werehandmade by her. Ten years ago, Maya moved to a model of having “dealers” take orders and opened a small workshop where part time labor produced the baskets. The dealers were also looking to supplement their incomes and, supplied with a small display inventory, displayed the baskets at home or at parties, and took orders. Order fulfillment was handled directly by AGM Enterprises personnel, who shipped finished baskets directly to customers. Little production inventory was kept.

Last year, Maya McCrum evaluated the costs and benefits of two alternative distribution channels in an attempt to expand the business beyond the West Coast. One alternative was to franchise the business. Maya was concerned that she and the managers of AGM would lose control, especially control over quality, which she felt distinguished AGM baskets. The other alternative was to begin taking orders over the Internet. Maya chose the Internet option. The company added a new managerial position, Chief Technology Officer (CTO), and established a subsidiary, agm.com , to handle the new business. In an unusual move for the company, Maya went outside the small circle of family and friends and hired as the CTO Mary Brown, who had experience on both the technical and management side of a local Internet start up. Mary was looking for something new where she could be in charge of an entire operation and was excited that she could combine this with her interest in basket weaving. It was agreed that if she could meet or exceed her budget for the first year of operation, she would be given a substantial piece of agm.com .

The executives of AGM Enterprises considered the initial foray into the Internet to be an experiment to see if the “anonymous” approach would be effective in selling baskets. Until this time, AGM considered its network of dealers to be crucial in the growth it had experienced in the last several years. To this end, a separate workshop (factory) was established in Pennsylvania. One of the reasons for selecting Pennsylvania was the availability of part time labor at lower costs than in California. Another was to attempt to penetrate the East Coast market by locating a workshop there, taking advantage of more immediate access to local market tastes and trends. It was decided that the Pennsylvania operation would produce exclusively for agm.com business and the California workshop would continue to handle the orders from dealers.

Most of the staff functions for agm.com were provided and controlled by AGM Enterprises. Mary and Donna Cunha, the senior vice president of marketing for the parent company, jointly decided the marketing budget. While the budget was decided jointly, media decisions and advertising campaigns were run directly from the parent organization. Personnel and financial services were also centralized.

Mary contracted with a major telecommunications company to provide Web hosting services for the operation. She wanted to go with a telecommunications company rather than a local Internet Service Provider (ISP) for reasons of reliability. The back office operations (billing, payroll, etc.) would be maintained on personal computers at the agm.com office.

The Initial Plan

AGM Enterprises (and agm.com ) have a July 1 fiscal year and the launch of agm.com was designed to coincide with the beginning of fiscal year 1. Maya and Mary decided that agm.com would initially offer only one of the company’s many baskets for sale. Company managers believed this would simplify production scheduling and help maintain quality control for the workforce. The basket to be offered was the round basket, one of the company’s most popular. The standard cost sheet for the basket is shown in Exhibit 16.16.

The cost accounting system at AGM Enterprises and the one adopted for agm.com is a full absorption, standard cost system. Overhead is assigned to products (at standard cost) and not recognized in income until the product is sold. Variable overhead is allocated on the basis of direct labor hours and fixed overhead on the number of units. The fixed overhead rate is based on an estimated production level for the quarter. All variances from standard are recognized in the period recorded.

Because of the uncertainty surrounding the demand for baskets using this new channel, the first quarter budget was designed to be “easy” to meet. In addition, relatively large marketing expenses were budgeted for promoting the new channel at related Web sites and in craft publications. This

Exhibit 16.16 Standard Cost Sheet— agm.com

Selling price

 

 

 

$25.00

Materials

 

 

 

 

Reed (pounds per unit)

0.4 pounds @ $5

$2.00

$6.10

 

Handle

 

4.10

 

 

Direct labor

0.5 hours @ $12 0.5

 

6.00

 

Variable overhead

hours @ $1

 

0.50

 

Fixed overhead

 

 

2.00

 

Total standard cost

 

 

 

14.60

Standard gross profit per unit

 

 

 

$10.40

Exhibit 16.17 Operating Budget, First Quarter—agm.com

Budgeted sales and production

 

8,000 baskets

Revenue

 

$200,000

Variable costs

 

 

Materials

$48,800

100,800

Labor

 48,000

 

Variable overhead

 4,000

 

Budgeted contribution margin

 

$ 99,200

Fixed overhead

 

 16,000

Budgeted gross profit

 

$ 83,200

Marketing and administration

 

 90,000

Operating profit

 

$ (6,800)

was especially important in some of the East Coast publications because AGM had a small share in these markets. The first quarter operating budget is shown in Exhibit 16.17. The marketing and administration budget included the costs incurred by the parent for providing these services, as well as the cost of the small staff assisting Mary and Jeff Lancaster, the production manager at agm.com

First Quarter Results

At first, things went well for agm.com . Sales in July were sufficiently strong that managers thought the initial sales forecast might have been too limiting. Beginning in mid August, however, events turned against the new operation. Workers at the telecommunications company went on strike. At first, there was little impact. On August 9, however, a phone line leading to the server was damaged. Because of the strike, the site went off the air. It was one week before supervisors were able to get the site back up. Although difficult to estimate, Mary suggested in a message to AGM Enterprises that the company lost about 5 percent in unit sales (i.e., about 400 baskets). She based this estimate on the fact that lines were down 7 days of the quarter (about 7.7 percent) but that some of the customers that were not able to connect would return when service was restored. Others would simply click on the next site their search engine identified.

In order to try and counteract some of the negative publicity that had occurred, agm.com offered some concessions to customers. One concession was free shipping on all orders over $100. (Initially, shipping was billed to the customer at cost.) This added $13,000 to the Marketing and Administration expenses for the quarter. Also, at Mary’s request, additional marketing campaigns costing $32,000 were launched in craft magazines and on cable television. These efforts helped make up for the lost sales.

As sales were falling, the company was also hit by the booming economy in the state when the basket makers were finding better part time employment in the local industries. As a result, agm. com had to increase the wage rate simply to maintain production.

Not all the news was bad, however. Mary had immediately identified a modification in the production process at the Pennsylvania workshop that reduced the scrap on each basket by 20 percent. This modification was used on all baskets produced in the quarter. (In the original process, scrap occurred in the initial cutting of the material and, therefore, no labor was lost because of the scrap.) In addition, she maintained the level of quality, so the company received no returns and many comments about future purchases. Still, she was concerned that this poor first quarter showing was going to be difficult to make up.

I came here because I wanted to work at a company that, first, I had a significant ownership stake in and, second, that would allow me to pursue my interest in the craft of basket weaving full time. I’m afraid that, because of the strike, I won’t meet the first year budget and will lose my bonus shares. I think Maya is a fair person, but she has to answer to the other owners. They might not be so willing to assume that these results are because of events out of my control.

Exhibit 16.18 shows the actual results for the quarter. The actual direct (materials and labor) production inputs are shown in Exhibit 16.19. Actual total variable overhead for the quarter was $5,760 and actual fixed overhead was $16,000.

Next Steps

As Maya contemplates the future of the new distribution channel, she is concerned as well about the effect of the first quarter on her agreement with Mary.

I would really like the answer to just one question: Should we rewrite our agreement? From what I have seen, Mary is really dedicated to the business. On the other hand, an agreement is an agreement. If we revise it now, what kind of problems will we have in the future?

Exhibit 16.18 Actual Results, First Quarter—agm.com

Actual sales and production

8,000

Revenue

$176,000

Standard cost of goods sold

 116,800

Gross profit

$ 59,200

Production cost variances

 12,960

Marketing and administration

 135,000

Operating profit

$ (88,760)

Exhibit 16.19 Actual Direct Production Quantities and Costs— agm.com

Input

Quantity

Total Actual Cost

Materialsa

 

 

Reed

2,400 pounds

$11,520

Handle

8,000 handles

31,200

Direct labor

4,800 hours

65,280

a All materials used in production. There are no materials or work in process inventories.

Required

a. What were the factors that caused actual quarterly income to be less than budgeted? Quantify the effect of each of these factors. Be as specific as possible.

b. For which of these factors, if any, should Mary be held responsible?

c. Should Maya rewrite the agreement with Mary?

turlock tube inc manufactures metal tubing the standard materials usage for each uni 691384

Turlock Tube, Inc., manufactures metal tubing. The standard materials usage for each unit is 0.5 pounds. The standard cost of metal is $4 per pound. Actual results for September follow:

Production

100,000 units

Materials used

52,000 pounds

Materials purchased (67,000 pounds)

$261,300

Compute the material purchase price variance and the material efficiency variance for September.

duluth castings company makes a product x tol from two materials ticon and vf the st 691386

Duluth Castings Company makes a product, X Tol, from two materials, Ticon and VF. The standard prices and quantities are as follows:

 

Ticon

VF

Price per pound

$12

$18

Pounds per unit of X Tol

6 pounds

3 pounds

In May, Duluth produced 35,000 units of X Tol using the following actual prices and quantities of materials:

 

Ticon

VF

Price per pound

$11.40

$16.80

Pounds purchased and used

216,000

114,000

a. Compute materials price and efficiency variances.

b. Compute materials mix and yield variances.

sales mix and quantity variances aaa electronics sells two models of electronic orga 691391

Sales Mix and Quantity Variances

AAA Electronics sells two models of electronic organizers. The budgeted price per unit for the regular model is $109.50 and the budgeted price per unit for the wireless model is $249.50. The master budget called for sales of 20,000 regular models and 9,000 wireless models during the current year. Actual results showed sales of 15,000 regular models, with a price of $112.90 per unit, and 10,000 wireless models, with a price of $253.90 per unit. The standard variable cost per unit is $50 for a regular model and $100 for a wireless model.

Required

a. Compute the activity variance for these data.

b. Break down the activity variance into mix and quantity parts.

sales mix and quantity variances renee rsquo s rings manufactures college rings two 691392

Sales Mix and Quantity Variances

Renee’s Rings manufactures college rings. Two models are produced: The Spirit model with a budgeted price of $500 and a standard variable cost of $200. The Chancellor model has a budgeted price of $1,200 and a standard variable cost of $400. At the beginning of the year, Renee estimated that she would sell 2,000 Chancellor rings and 8,000 Spirit rings. The actual results for the year showed that 2,400 Chancellor rings were sold for total revenues of $2,520,000. A total of 8,800 Spirit rings were sold for revenues of $4,620,000.

Required

a. Compute the activity variance for Renee’s Rings for the year.

b. Compute the mix and quantity variances for the year.

sales mix and quantity variances tapas by tom tbt is a high end restaurant tbt offer 691393

Sales Mix and Quantity Variances

Tapas By Tom (TBT) is a high end restaurant. TBT offers two dining options. A fixed price menu has a budgeted meal price of $60. An a la carte menu has a budgeted price of $40 for a meal. TBTexpects that one third of its diners will order the fixed price menu. The fixed price menu has a budgeted variable cost of $20 and the a la carte meal averages $16 per meal in budgeted variable cost. TBT estimates that 1,200 people will order a meal in any month.

For July, TBT served a total of 1,080 meals, including 324 fixed price meals. Total revenues were $17,820 for fixed price meals and $34,020 for a la carte meals.

Required

a. Compute the activity variance for TBT for July.

b. Compute the mix and quantity variances for July.

materials mix and yield variances huron group had the following direct materials dat 691394

Materials Mix and Yield Variances

Huron Group had the following direct materials data for its product:

Standard costs for one unit of output

 

Material Twinkle

20 units of input at $20

Material Star

40 units of input at $30

During October, the company had the following results:

Units of output produced 2,000 units

 

Materials purchased and used

 

Material Twinkle

44,000 units at $18

Material Star

76,000 units at $32

Required

a. Compute materials price and efficiency variances.

b. Compute materials mix and yield variances.

labor mix and yield variances matt rsquo s eat lsquo n run has two categories of dir 691396

Labor Mix and Yield Variances

Matt’s Eat ‘N Run has two categories of direct labor, unskilled, which costs $7.50 per hour, and skilled, which costs $15 per hour. Management has established standards per “equivalent friendly meal,” which has been defined as a typical meal consisting of a hamburger, a drink, and French fries. Standards have been set as follows:

Skilled labor

2 minutes per equivalent meal

Unskilled labor

6 minutes per equivalent meal

For the year, Matt’s sold 180,000 equivalent friendly meals and incurred the following labor costs:

Skilled labor

6,000 hours

$ 92,000

Unskilled labor

15,000 hours

180,000

Required

a. Compute labor price and efficiency variances.

b. Compute labor mix and yield variances.

sales activity variance service organization refer to the data in exercise 17 24 pre 691398

Sales Activity Variance, Service Organization

Refer to the data in Exercise 17 24. Prepare a sales activity variance analysis like the one in Exhibit 16.4 of the previous chapter.

Exercise 17 24: Flexible Budgeting, Service Organization

Lowe & Rent is a law firm that specializes in probate work. Last year, the firm billed more hours than expected, but, as the following data show, profits were lower than anticipated.

 

Reported Income

Master

 

Statement

Budget

Billable hoursa

6,900

6,000

Revenue

$825,000

$750,000

Professional salaries (all variable)

465,000

375,000

Other variable costs (e.g., supplies, computer services)

108,000

102,000

Fixed costs

174,000

180,000

Profit

$ 78,000

$ 93,000

Required

Prepare a flexible budget for Lowe & Rent. Use billable hours as the measure of output (that is, units produced).

Exhibit 16.4 Flexible and Master Budget, August—Bayou Division

 

A

 

B

C

D

E

 

 

 

 

Flexible Budget

Sales Activity

 

Master Budget

 

 

 

 

(based on

Variance

 

(based on

 

 

 

 

actual activity

(based on

 

planned

 

 

 

 

of 80,000

variance in

 

activity of

 

1

 

 

units)

sales volume)

 

100,000 units)

 

2

Sales units

 

80,000

20,000

 

100,000

 

3

 

 

 

 

 

 

 

4

Sales revenue

 

$ 800,000

$ 200,000

U

$ 1,000,000

 

5

Less

 

 

 

 

 

 

6

Variable costs

 

 

 

 

 

 

7

Variable manufacturing costs

 

304,000

76,000

F

380,000

 

8

Variable selling and administrative

 

72,000

18,000

F

90,000

 

9

Total variable costs

 

$ 376,000

$

94,000

F

$

470,000

 

10

Contribution margin

 

$ 424,000

$ 106,000

U

$

530,000

 

11

Fixed costs

 

 

 

 

 

 

 

12

Fixed manufacturing overhead

 

200,000

–0–

 

 

200,000

 

13

Fixed selling and administrative costs

 

140,000

–0–

 

 

140,000

 

14

Total fixed costs

 

$ 340,000

–0–

 

$

340,000

 

15

Profit

 

$

84,000

$ 106,000

U

$

190,000

 

16

 

 

 

 

 

 

 

 

 

                     

profit variance analysis service organization refer to the data in exercise in 17 24 691399

Profit Variance Analysis, Service Organization

Refer to the data in Exercise in 17 24. Prepare a profit variance analysis for Lowe & Rent like the one in Exhibit 16.5 of the previous chapter.

Exercise 17 24: Flexible Budgeting, Service Organization

Lowe & Rent is a law firm that specializes in probate work. Last year, the firm billed more hours than expected, but, as the following data show, profits were lower than anticipated.

 

Reported Income

Master

 

Statement

Budget

Billable hoursa

6,900

6,000

Revenue

$825,000

$750,000

Professional salaries (all variable)

465,000

375,000

Other variable costs (e.g., supplies, computer services)

108,000

102,000

Fixed costs

174,000

180,000

Profit

$ 78,000

$ 93,000

Required

Prepare a flexible budget for Lowe & Rent. Use billable hours as the measure of output (that is, units produced).

Exhibit 16.5 Profit Variance Analysis, August—Bayou Division

 

A

B

C

D

E

F

G

H

I

J

K

L

M

 

1

 

(1)

(2)

 

 

(3)

 

(4)

 

(5)

(6)

 

(7)

 

 

 

 

 

 

 

 

 

 

 

Flexible

 

 

Master Budget

 

 

 

Actual (based

 

 

 

 

 

 

 

Budget (based

 

 

(based on

 

 

 

on actual

 

 

 

Marketing and

 

 

 

on actual

Sales

 

planned

 

 

 

activity of

Manufacturing

 

 

Administrative

 

Sales Price

 

activity of

Activity

 

activity of

 

2

 

80,000 units)

Variances

 

 

Variances

 

Variance

 

80,000 units)

Variance

 

100,000 units)

 

3

Sales revenue

$ 840,000

 

 

 

 

 

$ 40,000

F

$ 800,000

$ 200,000

U

$ 1,000,000

 

4

Less

 

 

 

 

 

 

 

 

 

 

 

 

 

5

Variable costs

 

 

 

 

 

 

 

 

 

 

 

 

 

6

Variable manufacturing costs

329,680

$ 25,680

U

a

 

 

 

 

304,000

76,000

F

380,000

 

7

Variable selling and administrative

68,000

 

 

 

$ 4,000

F

 

 

72,000

18,000

F

90,000

 

8

Contribution margin

$ 442,320

$ 25,680

U

 

$ 4,000

F

$ 40,000

F

$ 424,000

$ 106,000

U

$

530,000

 

9

Fixed costs

 

 

 

 

 

 

 

 

 

 

 

 

 

 

10

Fixed manufacturing overhead

195,500

4,500

F

 

 

 

 

 

200,000

–0–

 

 

200,000

 

11

Fixed selling and administrative costs

132,320

 

 

 

7,680

F

 

 

140,000

–0–

 

 

140,000

 

12

Profit

$ 114,500

$ 21,180

U

 

$ 11,680

F

$ 40,000

F

$

84,000

$ 106,000

U

$

190,000

 

13

 

 

 

 

 

 

 

 

 

 

 

 

 

 

 

 

14

aThe individual cost variances are shown in Exhibit 16.11.

 

 

 

 

 

 

 

 

 

 

 

 

 

 

15

 

 

 

 

 

 

 

 

 

 

 

 

 

 

 

 

 

 

 

 

 

 

 

 

 

 

 

 

 

 

 

 

 

Exhibit 16.11 Variable Manufacturing Cost Variance Summary, August—Bayou Division

 width=

in Exhibit 16.5. The cost variance analysis just completed is a more detailed analysis of the variable production cost variance derived in Exhibit 16.5.

A summary of this nature is useful for reporting variances to high level managers. It provides both an overview of variances and their sources. When used for reporting, the computations at the right of Exhibit 16.11 usually are replaced with a brief explanation of the cause of the variance.

Management might want more detailed information about some of the variances. Extending each variance branch in Exhibit 16.11 to show variances by product line, department, or other categories can provide this additional detail.

sales price and activity variances dylan amp father operate a tax accounting practic 691400

Sales Price and Activity Variances

Dylan & Father operate a tax accounting practice with partners and staff members. Each billable hour of partner time has a $770 budgeted price and $364 budgeted variable cost. Each billable hour of staff time has a budgeted price of $182 and a budgeted variable cost of $98. This month, the partnership budget called for 5,100 billable partner hours and 20,790 staff hours. Actual results were as follows:

Partner revenue

$3,612,000

4,800 hours

Staff revenue

$3,738,000

20,400 hours

Required

Compute the sales price and activity variances for these data. Also compute the mix and quantity variances.

sales mix and quantity variances mattie rsquo s vineyards is a producer and wholesal 691402

Sales Mix and Quantity Variances

Mattie’s Vineyards is a producer and wholesaler of three varieties of white wine. Sales data for November are given below:

 

Sauvignon

 

 

 

 

Blanc

Chardonnay

Riesling

Total

Budgeted selling price

$7.00

$8.25

$6.75

 

Budgeted variable cost

$5.00

$6.00

$4.75

 

Budgeted selling quantity

10,400

3,900

11,700

26,000

Actual selling price

$7.25

$8.10

$7.10

 

Actual variable cost

$5.10

$6.10

$5.10

 

Actual selling quantity

8,000

6,000

11,000

25,000

Required

a. Compute the sales price variance for all three wines.

b. Compute the activity variance for Mattie’s Vineyards for the year.

c. Compute the mix and quantity variances for Mattie’s Vineyards for the year.

analyze performance for a restaurant doug rsquo s diner is planning to expand operat 691403

Analyze Performance for a Restaurant

Doug’s Diner is planning to expand operations and is concerned that its reporting system might need improvement. The master budget income statement for the Downtown Doug’s, which contains a delicatessen and restaurant operation, follows (in thousands):

 

Delicatessen

Restaurant

Total

Gross sales

$1,000

$2,500

$3,500

 

 

 

 

Costs

 

 

 

Purchases

600

1,000

1,600

Hourly wages

50

876

926

Franchise fee

30

76

106

Advertising

100

200

300

Utilities

70

126

196

Depreciation

50

76

126

Lease cost

30

50

80

Salaries

30

50

80

Total costs

$ 960

$2,454

$3,414

Operating profit

$ 40

$ 46

$ 86

The company uses the following performance report for management evaluation:

DOWNTOWN DOUG’S

Net Income for the Year

($000)

Actual Results

Actual Results

Delicatessen

Restaurant

Total

Budget

Over or (Under ) Budgeta

Gross sales

1200

$2,000

$3,200

$3,500

$(300)

Costs

         

Purchases

780

800

1,580

1,600

$ (20)

Hourly wagesb

60

700

760

926

(166)

Franchise feeb

36

60

96

106

(10)

Advertising

100

200

300

300

 

Utilitiesb

76

100

176

196

(20)

Depreciation

50

76

126

126

 

Lease cost

30

50

80

80

 

Salaries

30

50

80

80

 

Total costs

1162

$2,036

$3,198

$3,414

$(216)

Operating profit

38

$ (36)

$ 2

$ 86

$ (84)

Required

Prepare a profit variance analysis for the delicatessen segment.

nonmanufacturing cost variances springfield bank originates mortgage loans for resid 691404

Nonmanufacturing Cost Variances

Springfield Bank originates mortgage loans for residential housing. The company charges a service fee for processing loan applications. This fee is set twice a year based on the cost of processing a loan application. For the first half of this year, the bank estimated that it would process 225 loans. Correspondence, credit reports, supplies, and other materials that vary with each loan are estimated to cost $45 per loan. The company hires a loan processor at an estimated cost of $60,000 per year and an assistant at an estimated cost of $50,000 per year. The cost to lease office space and pay utilities and other related costs is estimated to be $130,000 per year.

During the first six months of this year, Springfield Bank processed 240 loans. Cost of materials, credit reports, and other items related to loan processing were 12 percent higher than expected for the volume of loans processed.

The loan processor and her assistant cost $55,000 for the six months. Leasing and related office costs were $63,000 for the six months.

Required

Prepare an analysis of the variances for Springfield Bank.

performance evaluation in service industries bay area bank estimates that its overhe 691405

Performance Evaluation in Service Industries

Bay Area Bank estimates that its overhead costs for policy administration should be $30 for each new account obtained and $0.45 per year for each $1,000 of deposits. The company set a budget of selling 20,000 new accounts during the coming period. In addition, it estimated that the total deposits for the period would average $43,200,000.

During the period, actual costs related to new accounts amounted to $572,250. The bank sold a total of 19,200 new accounts.

The cost of maintaining existing accounts was $18,000. Had these costs been incurred at the same prices as were in effect when the budget was prepared, the costs would have been $17,700, however, some costs changed. Also, deposits averaged $45,000,000 during the period.

Required

Prepare a schedule to show the differences between master budget and actual costs.

revenue analysis using industry data and multiple product lines peninsula candy comp 691406

Revenue Analysis Using Industry Data and Multiple Product Lines

Peninsula Candy Company makes three types of candy bars: Chewy, Chunky, and Choco Lite (Lite). Sales volume for the annual budget is determined by estimating the total market volume for candy bars and then applying the company’s prior year market share, adjusted for planned changes due to company programs for the coming year. Volume is apportioned among the three bars based on the prior year’s product mix, again adjusted for planned changes for the coming year.

The following are the company budget and the results of operations for July.

Budget

Chewy

Chunky

Choco Lite

Total

Sales units (in thousands)

2,000 bars

2,000 bars

4,000 bars

8,000 bars

Sales dollars (in thousands)

$200

$400

$600

$1,200

Variable costs

140

320

460

920

Contribution margin

$ 60

$ 80

$140

$ 280

Manufacturing fixed cost

40

40

60

140

Product margin

$ 20

$ 40

$ 80

$ 140

Marketing and administrative

       

costs (all fixed)

     

50

Operating profit

     

$ 90

Actual

       

Sales units (in thousands)

1,600 bars

2,000 bars

4,200 bars

7,800 bars

Sales dollars (in thousands)

$162

$400

$600

$1,162

Variable costs

112

322

464

898

Contribution margin

$ 50

$ 78

$136

$ 264

Manufacturing fixed cost

42

44

63

149

Product margin

$ 8

$ 34

$ 73

$ 115

Marketing and administrative

       

costs (all fixed)

     

55

Operating profit

     

$ 60

Industry volume was estimated at 80 million bars for budgeting purposes. Actual industry volume for July was 76 million bars.

Required

a. Prepare an analysis to show the effects of the sales price and sales activity variances.

b. Break down the sales activity variance into the parts caused by industry volume and market share.

sales mix and quantity variances refer to the data for the peninsula candy company p 691407

Sales Mix and Quantity Variances

Refer to the data for the Peninsula Candy Company (Problem 17 33). Break down the total activity variance into sales mix and quantity parts.

Problem 17 33: Revenue Analysis Using Industry Data and Multiple Product Lines

Peninsula Candy Company makes three types of candy bars: Chewy, Chunky, and Choco Lite (Lite). Sales volume for the annual budget is determined by estimating the total market volume for candy bars and then applying the company’s prior year market share, adjusted for planned changes due to company programs for the coming year. Volume is apportioned among the three bars based on the prior year’s product mix, again adjusted for planned changes for the coming year.

The following are the company budget and the results of operations for July.

Budget

Chewy

Chunky

Choco Lite

Total

Sales units (in thousands)

2,000 bars

2,000 bars

4,000 bars

8,000 bars

Sales dollars (in thousands)

$200

$400

$600

$1,200

Variable costs

140

320

460

920

Contribution margin

$ 60

$ 80

$140

$ 280

Manufacturing fixed cost

40

40

60

140

Product margin

$ 20

$ 40

$ 80

$ 140

Marketing and administrative

       

costs (all fixed)

     

50

Operating profit

     

$ 90

Actual

       

Sales units (in thousands)

1,600 bars

2,000 bars

4,200 bars

7,800 bars

Sales dollars (in thousands)

$162

$400

$600

$1,162

Variable costs

112

322

464

898

Contribution margin

$ 50

$ 78

$136

$ 264

Manufacturing fixed cost

42

44

63

149

Product margin

$ 8

$ 34

$ 73

$ 115

Marketing and administrative

       

costs (all fixed)

     

55

Operating profit

     

$ 60

Industry volume was estimated at 80 million bars for budgeting purposes. Actual industry volume for July was 76 million bars.

Required

a. Prepare an analysis to show the effects of the sales price and sales activity variances.

b. Break down the sales activity variance into the parts caused by industry volume and market share.

materials mix and yield variances houston corporation manufactures a wide variety of 691408

Materials Mix and Yield Variances

Houston Corporation manufactures a wide variety of chemical compounds and liquids for industrial uses. The standard mix for producing a single batch of 1,000 liters of Rust Off is as follows:

Input Chemical

Quantity (in liters)

Cost (per liter)

Total Cost

z Alpha

600

$ 9

$ 5,400

z Beta

450

12

5,400

z Gamma

200

24

4,800

 

1,250

 

$15,600

There is a 20 percent loss in liquid volume during processing due to evaporation. The finished liquid is put into 5 liter bottles for sale. Thus, the standard material cost for a 5 liter bottle is $78.

The actual quantities of direct materials and the cost of the materials placed in production during March were as follows (materials are purchased and used at the same time):

Input Chemical

Quantity (in liters)

Total Cost

z Alpha

50,400

$ 423,360

z Beta

37,040

400,464

z Gamma

16,960

417,216

 

104,400

$1,241,040

A total of 16,000 bottles (80,000 liters) were produced during March.

Required

Calculate the total direct material variance for the liquid product for the month of March and then further analyze the total variance into:

a. Materials price and efficiency variances.

b. Materials mix and yield variances.

derive amounts for profit variance analysis aqua clean inc operates a pool cleaning 691410

Derive Amounts for Profit Variance Analysis

Aqua Clean, Inc., operates a pool cleaning service. Aqua Clean wants to compare this month’s results with those for last month, which is believed to be a typical “base period.” Assume that the following information is provided:

 

Last Month

This Month

Number of cleanings

140

161

Revenues

$22,680

$22,800

Variable costs

4,620

5,220

Contribution margin

$18,060

$17,580

Required

Compute the flexible budget and sales activity variance and prepare a profit variance analysis (like the one in Exhibit 16.5 of the previous chapter) in as much detail as possible.

Exhibit 16.5 Profit Variance Analysis, August—Bayou Division

 

A

B

C

D

E

F

G

H

I

J

K

L

M

 

1

 

(1)

(2)

 

 

(3)

 

(4)

 

(5)

(6)

 

(7)

 

 

 

 

 

 

 

 

 

 

 

Flexible

 

 

Master Budget

 

 

 

Actual (based

 

 

 

 

 

 

 

Budget (based

 

 

(based on

 

 

 

on actual

 

 

 

Marketing and

 

 

 

on actual

Sales

 

planned

 

 

 

activity of

Manufacturing

 

 

Administrative

 

Sales Price

 

activity of

Activity

 

activity of

 

2

 

80,000 units)

Variances

 

 

Variances

 

Variance

 

80,000 units)

Variance

 

100,000 units)

 

3

Sales revenue

$ 840,000

 

 

 

 

 

$ 40,000

F

$ 800,000

$ 200,000

U

$ 1,000,000

 

4

Less

 

 

 

 

 

 

 

 

 

 

 

 

 

5

Variable costs

 

 

 

 

 

 

 

 

 

 

 

 

 

6

Variable manufacturing costs

329,680

$ 25,680

U

a

 

 

 

 

304,000

76,000

F

380,000

 

7

Variable selling and administrative

68,000

 

 

 

$ 4,000

F

 

 

72,000

18,000

F

90,000

 

8

Contribution margin

$ 442,320

$ 25,680

U

 

$ 4,000

F

$ 40,000

F

$ 424,000

$ 106,000

U

$

530,000

 

9

Fixed costs

 

 

 

 

 

 

 

 

 

 

 

 

 

 

10

Fixed manufacturing overhead

195,500

4,500

F

 

 

 

 

 

200,000

–0–

 

 

200,000

 

11

Fixed selling and administrative costs

132,320

 

 

 

7,680

F

 

 

140,000

–0–

 

 

140,000

 

12

Profit

$ 114,500

$ 21,180

U

 

$ 11,680

F

$ 40,000

F

$

84,000

$ 106,000

U

$

190,000

 

13

 

 

 

 

 

 

 

 

 

 

 

 

 

 

 

 

14

aThe individual cost variances are shown in Exhibit 16.11.

 

 

 

 

 

 

 

 

 

 

 

 

 

 

15

 

 

 

 

 

 

 

 

 

 

 

 

 

 

 

 

 

 

 

 

 

 

 

 

 

 

 

 

 

 

 

 

 

Exhibit 16.11 Variable Manufacturing Cost Variance Summary, August—Bayou Division

 width=

in Exhibit 16.5. The cost variance analysis just completed is a more detailed analysis of the variable production cost variance derived in Exhibit 16.5.

A summary of this nature is useful for reporting variances to high-level managers. It provides both an overview of variances and their sources. When used for reporting, the computations at the right of Exhibit 16.11 usually are replaced with a brief explanation of the cause of the variance.

Management might want more detailed information about some of the variances. Extending each variance branch in Exhibit 16.11 to show variances by product line, department, or other categories can provide this additional detail.

appendix used in part b variable cost variances information on thurmster corporatio 691354

(Appendix used in Part b) Variable Cost Variances

Information on Thurmster Corporation’s direct materials costs follows:

Actual quantities of direct materials used

7,500

Actual costs of direct materials used

$98,550

Standard price per unit of direct materials

$12.60

Flexible budget for direct materials

$89,775

Thurmster Corporation has no materials inventories.

Required

a. Prepare a short report for management showing Thurmster Corporation’s direct materials price and efficiency variances.

b. (Appendix) Prepare the journal entries to record the purchase and use of the direct materials using standard costing.

appendix used in part b variable cost variances information on canyon chemical rsqu 691355

(Appendix used in Part b) Variable Cost Variances

Information on Canyon Chemical’s direct materials costs follows:

Quantities of chemical Y purchased and used

28,800 gallons

Actual cost of chemical Y used

$640,000

Standard price per gallon of chemical Y

$22.50

Standard quantity of chemical Y allowed

26,400 gallons

Required

a. What were Canyon Chemical’s direct materials price and efficiency variances?

b. (Appendix) Prepare the journal entries to record the purchase and use of chemical Y using standard costing.

fixed cost variances information on carney company rsquo s fixed overhead costs foll 691356

Fixed Cost Variances

Information on Carney Company’s fixed overhead costs follows:

Overhead applied

$360,000

Actual overhead

385,500

Budgeted overhead

369,000

Required

What are the fixed overhead price and production volume variances? (Refer to Exhibit 16.13 for the format to use.)

Exhibit 16.13 Fixed Overhead Variances, August— Bayou Division

 width=

In this situation, a $40,000 unfavorable production volume variance exists. It is unfavorable because less overhead was applied than was budgeted; production was lower than the average monthly estimate. This variance is a result of the full absorption costing system; it does not occur in variable costing.

This $160,000 applied fixed overhead equals $2 per frame multiplied by 80,000 units actually produced (see Exhibit 16.14). If the $40 rate per direct labor hour had been used, the amount applied to the 80,000 units produced would still be $160,000 (= $40 x 0.05 x 80,000).

A variance occurs if the number of units actually produced differs from the number of units used to estimate the fixed cost per unit. Again, this variance is commonly referred to as a production volume variance (also called a capacity variance , an idle capacity variance , or a denominator variance ).

Our example has a production volume variance because the 80,000 frames actually produced during the month do not equal the 100,000 estimated for the month. Consequently, production is charged $160,000 (point A in Exhibit 16.14) instead of $200,000 (point B in Exhibit 16.14). The $40,000 difference is the production volume variance

Exhibit 16.14 Fixed Overhead Variances, Graphic Presentation—Bayou Division

 width=

because it is caused by a deviation in production volume level (number of frames produced) from that estimated to arrive at the standard cost.

If Bayou had estimated 80,000 frames per month instead of 100,000 frames, the standard cost would have been $2.50 per frame (= $200,000 ÷ 80,000 frames). Thus, $200,000 (= $2.50 x 80,000 frames) would have been applied to units produced, and there would have been no production volume variance.

The production volume variance applies only to fixed costs; it occurs because we are allocating a fixed period cost to units on a predetermined basis. It does not represent resources spent or saved. This is unique to full absorption costing. The benefits of calculating the variance for control purposes are questionable. Although the production volume variance signals a difference between expected and actual production levels, so does a simple production report of actual versus expected production quantities.

Compare with the Fixed Production Cost Price Variance The fixed production cost price variance is the difference between actual and budgeted fixed production costs. Unlike the production volume variance, the price variance commonly is used for control purposes because it is a measure of differences between actual and budgeted period costs.

Exhibits 16.13 and 16.14 summarize the computation of the fixed production price (spending) and production volume variances. Reviewing them will help you see the relationship between actual, budgeted, and applied fixed production costs.

graphical presentation refer to the data in exercise 16 32 management would like to 691357

Graphical Presentation

Refer to the data in Exercise 16 32. Management would like to see results reported graphically.

Required

Prepare a graph like that shown in Exhibit 16.14.

Exercise 16 32: Fixed Cost Variances

Information on Carney Company’s fixed overhead costs follows:

Overhead applied

$360,000

Actual overhead

385,500

Budgeted overhead

369,000

Required

What are the fixed overhead price and production volume variances? (Refer to Exhibit 16.13 for the format to use.)

Exhibit 16.13 Fixed Overhead Variances, August— Bayou Division

 width=

In this situation, a $40,000 unfavorable production volume variance exists. It is unfavorable because less overhead was applied than was budgeted; production was lower than the average monthly estimate. This variance is a result of the full absorption costing system; it does not occur in variable costing.

This $160,000 applied fixed overhead equals $2 per frame multiplied by 80,000 units actually produced (see Exhibit 16.14). If the $40 rate per direct labor hour had been used, the amount applied to the 80,000 units produced would still be $160,000 (= $40 x 0.05 x 80,000).

A variance occurs if the number of units actually produced differs from the number of units used to estimate the fixed cost per unit. Again, this variance is commonly referred to as a production volume variance (also called a capacity variance , an idle capacity variance , or a denominator variance ).

Our example has a production volume variance because the 80,000 frames actually produced during the month do not equal the 100,000 estimated for the month. Consequently, production is charged $160,000 (point A in Exhibit 16.14) instead of $200,000 (point B in Exhibit 16.14). The $40,000 difference is the production volume variance

Exhibit 16.14 Fixed Overhead Variances, Graphic Presentation—Bayou Division

 width=

because it is caused by a deviation in production volume level (number of frames produced) from that estimated to arrive at the standard cost.

If Bayou had estimated 80,000 frames per month instead of 100,000 frames, the standard cost would have been $2.50 per frame (= $200,000 ÷ 80,000 frames). Thus, $200,000 (= $2.50 x 80,000 frames) would have been applied to units produced, and there would have been no production volume variance.

The production volume variance applies only to fixed costs; it occurs because we are allocating a fixed period cost to units on a predetermined basis. It does not represent resources spent or saved. This is unique to full absorption costing. The benefits of calculating the variance for control purposes are questionable. Although the production volume variance signals a difference between expected and actual production levels, so does a simple production report of actual versus expected production quantities.

Compare with the Fixed Production Cost Price Variance The fixed production cost price variance is the difference between actual and budgeted fixed production costs. Unlike the production volume variance, the price variance commonly is used for control purposes because it is a measure of differences between actual and budgeted period costs.

Exhibits 16.13 and 16.14 summarize the computation of the fixed production price (spending) and production volume variances. Reviewing them will help you see the relationship between actual, budgeted, and applied fixed production costs.

appendix used in part c comprehensive cost variance analysis wagner inc manufacture 691359

(Appendix used in Part c) Comprehensive Cost Variance Analysis

Wagner, Inc., manufactures truck tires. The following information is available for the last operating period.

• Wagner produced and sold 46,000 tires for $60 each. Budgeted production was 50,000 tires.

• Standard variable costs per tire follow:

Direct materials: 4 pounds at $3

$12.00

Direct labor: 0.8 hours at $13.50

 10.80

Variable production overhead: 0.18 machine hours at $15 per hour

2.70

Total variable costs

$25.50

• Fixed production overhead costs:

Monthly budget

$1,000,000

• Fixed overhead is applied at the rate of $20 per tire.

• Actual production costs:

Direct materials purchased and used: 192,000 pounds at $2.70

$ 518,400

Direct labor: 35,200 hours at $13.80

485,760

Variable overhead: 8,640 machine hours at $15.30 per hour

132,192

Fixed overhead

1,075,000

Required

a. Prepare a cost variance analysis for each variable cost for Wagner.

b. Prepare a fixed overhead cost variance analysis.

c. Prepare the journal entries to record the activity for the last period using standard costing. Assume that all variances are closed to Cost of Goods Sold at the end of the operating period.

overhead variances three bridges inc shows the following overhead information for th 691361

Overhead Variances

Three Bridges, Inc., shows the following overhead information for the current period:

Actual overhead incurred

$638,400 ($182,400 fixed and $456,000 variable)

Budgeted fixed overhead

$187,200 (12,000 direct labor hours budgeted)

Standard variable overhead rate

 

per direct labor hour

$36

Standard hours allowed for

 

actual production

14,000 hours

Actual labor hours used

13,200 hours

Required

What are the variable overhead price and efficiency variances and the fixed overhead price variance?

solve for master budget given actual results a new accounting intern at gibson corpo 691362

Solve for Master Budget Given Actual Results

A new accounting intern at Gibson Corporation lost the only copy of this period’s master budget. The CFO wants to evaluate performance for this period but needs the master budget to do so. Actual results for the period follow:

Sales volume

120,000 units

Sales revenue

$672,000

Variable costs

 

Manufacturing

147,200

Marketing and administrative

61,400

Contribution margin

$463,400

Fixed costs

 

Manufacturing

205,000

Marketing and administrative

113,200

Operating profit

$145,200

The company planned to produce and sell 108,000 units for $5 each. At that volume, the contribution margin would have been $380,000. Variable marketing and administrative costs are budgeted at 10 percent of sales revenue. Manufacturing fixed costs are estimated at $2 per unit at the normal volume of 108,000 units. Management notes, “We budget an operating profit of $1 per unit at the normal volume.”

Required

a. Construct the master budget for the period.

b. Prepare a profit variance analysis like the one in Exhibit 16.5.

Exhibit 16.5 Profit Variance Analysis, August—Bayou Division

 

A

B

C

D

E

F

G

H

I

J

K

L

M

 

1

 

(1)

(2)

 

 

(3)

 

(4)

 

(5)

(6)

 

(7)

 

 

 

 

 

 

 

 

 

 

 

Flexible

 

 

Master Budget

 

 

 

Actual (based

 

 

 

 

 

 

 

Budget (based

 

 

(based on

 

 

 

on actual

 

 

 

Marketing and

 

 

 

on actual

Sales

 

planned

 

 

 

activity of

Manufacturing

 

 

Administrative

 

Sales Price

 

activity of

Activity

 

activity of

 

2

 

80,000 units)

Variances

 

 

Variances

 

Variance

 

80,000 units)

Variance

 

100,000 units)

 

3

Sales revenue

$ 840,000

 

 

 

 

 

$ 40,000

F

$ 800,000

$ 200,000

U

$ 1,000,000

 

4

Less

 

 

 

 

 

 

 

 

 

 

 

 

 

5

Variable costs

 

 

 

 

 

 

 

 

 

 

 

 

 

6

Variable manufacturing costs

329,680

$ 25,680

U

a

 

 

 

 

304,000

76,000

F

380,000

 

7

Variable selling and administrative

68,000

 

 

 

$ 4,000

F

 

 

72,000

18,000

F

90,000

 

8

Contribution margin

$ 442,320

$ 25,680

U

 

$ 4,000

F

$ 40,000

F

$ 424,000

$ 106,000

U

$

530,000

 

9

Fixed costs

 

 

 

 

 

 

 

 

 

 

 

 

 

 

10

Fixed manufacturing overhead

195,500

4,500

F

 

 

 

 

 

200,000

–0–

 

 

200,000

 

11

Fixed selling and administrative costs

132,320

 

 

 

7,680

F

 

 

140,000

–0–

 

 

140,000

 

12

Profit

$ 114,500

$ 21,180

U

 

$ 11,680

F

$ 40,000

F

$

84,000

$ 106,000

U

$

190,000

 

13

 

 

 

 

 

 

 

 

 

 

 

 

 

 

 

 

14

aThe individual cost variances are shown in Exhibit 16.11.

 

 

 

 

 

 

 

 

 

 

 

 

 

 

15

 

 

 

 

 

 

 

 

 

 

 

 

 

 

 

 

 

 

 

 

 

 

 

 

 

 

 

 

 

 

 

 

 

Exhibit 16.11 Variable Manufacturing Cost Variance Summary, August—Bayou Division

 width=

in Exhibit 16.5. The cost variance analysis just completed is a more detailed analysis of the variable production cost variance derived in Exhibit 16.5.

A summary of this nature is useful for reporting variances to high-level managers. It provides both an overview of variances and their sources. When used for reporting, the computations at the right of Exhibit 16.11 usually are replaced with a brief explanation of the cause of the variance.

Management might want more detailed information about some of the variances. Extending each variance branch in Exhibit 16.11 to show variances by product line, department, or other categories can provide this additional detail.

ethical issues in managing reported profits a new ceo has come in from the outside t 691365

Ethical Issues in Managing Reported Profits

A new CEO has come in from the outside to turn struggling Doak Industries into a profitable organization. Relying on market research, she wanted to focus production on the two specific product lines produced by the papers division.

Market research proved correct, and, by the end of the year, the papers division had exceeded budgeted profits by 18 percent. The controller, Ray Green, knew that his annual bonus depended on exceeding budgeted profit and that his bonus would plateau at 10 percent above budgeted profit. Ray expected that next year’s profit plan would be similar but that next year’s budget would consider the changes in the product lines. Ray discovered that he could accrue some of next year’s expenses and defer some of this year’s revenue while still exceeding budgeted profit by 10 percent.

Required

Why would Ray Green, Doak’s controller, want to defer revenue but accrue expenses? Is this ethical?

profit variance analysis use the information for tolstoy corporation in problem 16 4 691368

Profit Variance Analysis

Use the information for Tolstoy Corporation in Problem 16 43 to prepare a profit variance analysis like the one in Exhibit 16.5.

Problem 16 43: Prepare Flexible Budget

The following information is provided concerning the operations of Tolstoy Corporation for the current period:

 

Actual (based on actual of 360 units)

Master Budget (based on budgeted 400 units)

Sales revenue

$ 55,200

$ 60,000

Less

 

 

Manufacturing costs Direct labor

8,520

9,000

Materials

7,200

8,400

Variable overhead

4,920

6,000

Marketing

3,180

3,600

Administrative

3,000

3,000

Total variable costs

$26,820

$30,000

Contribution margin

$28,380

$30,000

Fixed costs

 

 

Manufacturing

2,916

3,000

Marketing

6,240

6,000

Administrative

5,976

6,000

Total fixed costs

$15,132

$15,000

Operating profits

$13,248

$15,000

There are no inventories.

Required

Prepare a flexible budget for Tolstoy Corporation.

Exhibit 16.5 Profit Variance Analysis, August—Bayou Division

 

A

B

C

D

E

F

G

H

I

J

K

L

M

 

1

 

(1)

(2)

 

 

(3)

 

(4)

 

(5)

(6)

 

(7)

 

 

 

 

 

 

 

 

 

 

 

Flexible

 

 

Master Budget

 

 

 

Actual (based

 

 

 

 

 

 

 

Budget (based

 

 

(based on

 

 

 

on actual

 

 

 

Marketing and

 

 

 

on actual

Sales

 

planned

 

 

 

activity of

Manufacturing

 

 

Administrative

 

Sales Price

 

activity of

Activity

 

activity of

 

2

 

80,000 units)

Variances

 

 

Variances

 

Variance

 

80,000 units)

Variance

 

100,000 units)

 

3

Sales revenue

$ 840,000

 

 

 

 

 

$ 40,000

F

$ 800,000

$ 200,000

U

$ 1,000,000

 

4

Less

 

 

 

 

 

 

 

 

 

 

 

 

 

5

Variable costs

 

 

 

 

 

 

 

 

 

 

 

 

 

6

Variable manufacturing costs

329,680

$ 25,680

U

a

 

 

 

 

304,000

76,000

F

380,000

 

7

Variable selling and administrative

68,000

 

 

 

$ 4,000

F

 

 

72,000

18,000

F

90,000

 

8

Contribution margin

$ 442,320

$ 25,680

U

 

$ 4,000

F

$ 40,000

F

$ 424,000

$ 106,000

U

$

530,000

 

9

Fixed costs

 

 

 

 

 

 

 

 

 

 

 

 

 

 

10

Fixed manufacturing overhead

195,500

4,500

F

 

 

 

 

 

200,000

–0–

 

 

200,000

 

11

Fixed selling and administrative costs

132,320

 

 

 

7,680

F

 

 

140,000

–0–

 

 

140,000

 

12

Profit

$ 114,500

$ 21,180

U

 

$ 11,680

F

$ 40,000

F

$

84,000

$ 106,000

U

$

190,000

 

13

 

 

 

 

 

 

 

 

 

 

 

 

 

 

 

 

14

aThe individual cost variances are shown in Exhibit 16.11.

 

 

 

 

 

 

 

 

 

 

 

 

 

 

15

 

 

 

 

 

 

 

 

 

 

 

 

 

 

 

 

 

 

 

 

 

 

 

 

 

 

 

 

 

 

 

 

 

Exhibit 16.11 Variable Manufacturing Cost Variance Summary, August—Bayou Division

 width=

in Exhibit 16.5. The cost variance analysis just completed is a more detailed analysis of the variable production cost variance derived in Exhibit 16.5.

A summary of this nature is useful for reporting variances to high level managers. It provides both an overview of variances and their sources. When used for reporting, the computations at the right of Exhibit 16.11 usually are replaced with a brief explanation of the cause of the variance.

Management might want more detailed information about some of the variances. Extending each variance branch in Exhibit 16.11 to show variances by product line, department, or other categories can provide this additional detail.

prepare flexible budget hayden corporation provides you with the following informati 691369

Prepare Flexible Budget

Hayden Corporation provides you with the following information for the month of June:

 

Actual (based on

Master Budget (based

 

actual sales of

on budgeted sales of

 

21,000 units)

20,000 units)

Sales revenues

$1,155,000

$1,200,000

Less

 

 

Variable costs

 

 

Direct material

147,000

120,000

Direct labor

136,500

160,000

Variable overhead

158,500

160,000

Marketing

51,000

50,000

Administration

47,000

50,000

Total variable costs

$ 540,000

 $ 540,000

 

 

 

Contribution margin

$ 615,000

$ 660,000

Fixed costs

 

 

Manufacturing

256,000

250,000

Marketing

57,000

50,000

Administration

203,000

$ 200,000

 

 

 

Total fixed costs

$ 516,000

$ 500,000

Operating profits

$ 99,000

$ 160,000

Required

Prepare a flexible budget for Hayden Corporation for June.

sales activity variance refer to the data in problem 16 46 prepare a sales activity 691370

Sales Activity Variance

Refer to the data in Problem 16 46. Prepare a sales activity variance analysis for Hayden Corporation like the one in Exhibit 16.4.

Problem 16 46: Prepare Flexible Budget

Hayden Corporation provides you with the following information for the month of June:

 

Actual (based on

Master Budget (based

 

actual sales of

on budgeted sales of

 

21,000 units)

20,000 units)

Sales revenues

$1,155,000

$1,200,000

Less

 

 

Variable costs

 

 

Direct material

147,000

120,000

Direct labor

136,500

160,000

Variable overhead

158,500

160,000

Marketing

51,000

50,000

Administration

47,000

50,000

Total variable costs

$ 540,000

 $ 540,000

 

 

 

Contribution margin

$ 615,000

$ 660,000

Fixed costs

 

 

Manufacturing

256,000

250,000

Marketing

57,000

50,000

Administration

203,000

$ 200,000

 

 

 

Total fixed costs

$ 516,000

$ 500,000

Operating profits

$ 99,000

$ 160,000

Required

Prepare a flexible budget for Hayden Corporation for June.

Exhibit 16.4 Flexible and Master Budget, August—Bayou Division

 

A

 

B

C

D

E

 

 

 

 

Flexible Budget

Sales Activity

 

Master Budget

 

 

 

 

(based on

Variance

 

(based on

 

 

 

 

actual activity

(based on

 

planned

 

 

 

 

of 80,000

variance in

 

activity of

 

1

 

 

units)

sales volume)

 

100,000 units)

 

2

Sales units

 

80,000

20,000

 

100,000

 

3

 

 

 

 

 

 

 

4

Sales revenue

 

$ 800,000

$ 200,000

U

$ 1,000,000

 

5

Less

 

 

 

 

 

 

6

Variable costs

 

 

 

 

 

 

7

Variable manufacturing costs

 

304,000

76,000

F

380,000

 

8

Variable selling and administrative

 

72,000

18,000

F

90,000

 

9

Total variable costs

 

$ 376,000

$

94,000

F

$

470,000

 

10

Contribution margin

 

$ 424,000

$ 106,000

U

$

530,000

 

11

Fixed costs

 

 

 

 

 

 

 

12

Fixed manufacturing overhead

 

200,000

–0–

 

 

200,000

 

13

Fixed selling and administrative costs

 

140,000

–0–

 

 

140,000

 

14

Total fixed costs

 

$ 340,000

–0–

 

$

340,000

 

15

Profit

 

$

84,000

$ 106,000

U

$

190,000

 

16

 

 

 

 

 

 

 

 

 

                     

profit variance analysis refer to the data in problem 16 46 prepare a profit varianc 691371

Profit Variance Analysis

Refer to the data in Problem 16 46. Prepare a profit variance analysis for Hayden Corporation like the one in Exhibit 16.5.

Problem 16 46: Prepare Flexible Budget

Hayden Corporation provides you with the following information for the month of June:

 

Actual (based on

Master Budget (based

 

actual sales of

on budgeted sales of

 

21,000 units)

20,000 units)

Sales revenues

$1,155,000

$1,200,000

Less

 

 

Variable costs

 

 

Direct material

147,000

120,000

Direct labor

136,500

160,000

Variable overhead

158,500

160,000

Marketing

51,000

50,000

Administration

47,000

50,000

Total variable costs

$ 540,000

 $ 540,000

 

 

 

Contribution margin

$ 615,000

$ 660,000

Fixed costs

 

 

Manufacturing

256,000

250,000

Marketing

57,000

50,000

Administration

203,000

$ 200,000

 

 

 

Total fixed costs

$ 516,000

$ 500,000

Operating profits

$ 99,000

$ 160,000

Required

Prepare a flexible budget for Hayden Corporation for June.

Exhibit 16.5 Profit Variance Analysis, August—Bayou Division

 

A

B

C

D

E

F

G

H

I

J

K

L

M

 

1

 

(1)

(2)

 

 

(3)

 

(4)

 

(5)

(6)

 

(7)

 

 

 

 

 

 

 

 

 

 

 

Flexible

 

 

Master Budget

 

 

 

Actual (based

 

 

 

 

 

 

 

Budget (based

 

 

(based on

 

 

 

on actual

 

 

 

Marketing and

 

 

 

on actual

Sales

 

planned

 

 

 

activity of

Manufacturing

 

 

Administrative

 

Sales Price

 

activity of

Activity

 

activity of

 

2

 

80,000 units)

Variances

 

 

Variances

 

Variance

 

80,000 units)

Variance

 

100,000 units)

 

3

Sales revenue

$ 840,000

 

 

 

 

 

$ 40,000

F

$ 800,000

$ 200,000

U

$ 1,000,000

 

4

Less

 

 

 

 

 

 

 

 

 

 

 

 

 

5

Variable costs

 

 

 

 

 

 

 

 

 

 

 

 

 

6

Variable manufacturing costs

329,680

$ 25,680

U

a

 

 

 

 

304,000

76,000

F

380,000

 

7

Variable selling and administrative

68,000

 

 

 

$ 4,000

F

 

 

72,000

18,000

F

90,000

 

8

Contribution margin

$ 442,320

$ 25,680

U

 

$ 4,000

F

$ 40,000

F

$ 424,000

$ 106,000

U

$

530,000

 

9

Fixed costs

 

 

 

 

 

 

 

 

 

 

 

 

 

 

10

Fixed manufacturing overhead

195,500

4,500

F

 

 

 

 

 

200,000

–0–

 

 

200,000

 

11

Fixed selling and administrative costs

132,320

 

 

 

7,680

F

 

 

140,000

–0–

 

 

140,000

 

12

Profit

$ 114,500

$ 21,180

U

 

$ 11,680

F

$ 40,000

F

$

84,000

$ 106,000

U

$

190,000

 

13

 

 

 

 

 

 

 

 

 

 

 

 

 

 

 

 

14

aThe individual cost variances are shown in Exhibit 16.11.

 

 

 

 

 

 

 

 

 

 

 

 

 

 

15

 

 

 

 

 

 

 

 

 

 

 

 

 

 

 

 

 

 

 

 

 

 

 

 

 

 

 

 

 

 

 

 

 

Exhibit 16.11 Variable Manufacturing Cost Variance Summary, August—Bayou Division

 width=

in Exhibit 16.5. The cost variance analysis just completed is a more detailed analysis of the variable production cost variance derived in Exhibit 16.5.

A summary of this nature is useful for reporting variances to high level managers. It provides both an overview of variances and their sources. When used for reporting, the computations at the right of Exhibit 16.11 usually are replaced with a brief explanation of the cause of the variance.

Management might want more detailed information about some of the variances. Extending each variance branch in Exhibit 16.11 to show variances by product line, department, or other categories can provide this additional detail.

overhead variances lima parts inc shows the following overhead information for the c 691373

Overhead Variances

Lima Parts, Inc., shows the following overhead information for the current period:

Actual overhead incurred

$58,800, 2/3 of which is variable

Budgeted fixed overhead

$17,280

Standard variable overhead rate per

 

direct labor hour

$18

Standard hours allowed for

 

actual production

2,350 hours

Actual labor hours used

2,200 hours

Required

What are the variable overhead price and efficiency variances and fixed overhead price variance?

manufacturing variances clemson company prepares its budgets on the basis of standar 691374

Manufacturing Variances

Clemson Company prepares its budgets on the basis of standard costs. A responsibility report is prepared monthly showing the differences between master budget and actual results. Variances are analyzed and reported separately. There are no materials inventories.

The following information relates to the current period:

Standard costs (per unit of output)

 

Direct materials, 2 gallons @ $6.00 per gallon

$12

Direct labor, 4 hours @ $24 per hour

96

Factory overhead

 

Variable (25% of direct labor cost)

24

Total standard cost per unit

$132

Actual costs and activities for the month follow:

Materials used

4,200 gallons at $5.40 per gallon

Output

1,900 units

Actual labor costs

6,400 hours at $30 per hour

Actual variable overhead

$54,000

Required

Prepare a cost variance analysis for the variable costs.

overhead cost and variance relationships fargo corporation reported a 400 favorable 691375

Overhead Cost and Variance Relationships

Fargo Corporation reported a $400 favorable price variance for variable overhead and a $4,000 favorable price variance for fixed overhead. The flexible budget had $256,800 variable overhead based on 21,400 direct labor hours; only 21,200 hours were worked. Total actual overhead was $434,800. The number of estimated hours for computing the fixed overhead application rate totaled 22,000 hours.

Required

a. Prepare a variable overhead analysis like the one in Exhibit 16.10.

b. Prepare a fixed overhead analysis like the one in Exhibit 16.13.

Exhibit 16.10 Variable Overhead Variances, August (80,000 Frames)—Bayou Division

 width=

At Bayou Division, the $880 unfavorable price variance for August (see Exhibit 16.10) was attributed to waste in using supplies and recent price increases for petroleum products used to maintain the machines.

Exhibit 16.13 Fixed Overhead Variances, August— Bayou Division

 width=

In this situation, a $40,000 unfavorable production volume variance exists. It is unfavorable because less overhead was applied than was budgeted; production was lower than the average monthly estimate. This variance is a result of the full absorption costing system; it does not occur in variable costing.

This $160,000 applied fixed overhead equals $2 per frame multiplied by 80,000 units actually produced (see Exhibit 16.14). If the $40 rate per direct labor hour had been used, the amount applied to the 80,000 units produced would still be $160,000 (= $40 x 0.05 x 80,000).

A variance occurs if the number of units actually produced differs from the number of units used to estimate the fixed cost per unit. Again, this variance is commonly referred to as a production volume variance (also called a capacity variance , an idle capacity variance , or a denominator variance ).

Our example has a production volume variance because the 80,000 frames actually produced during the month do not equal the 100,000 estimated for the month. Consequently, production is charged $160,000 (point A in Exhibit 16.14) instead of $200,000 (point B in Exhibit 16.14). The $40,000 difference is the production volume variance

Exhibit 16.14 Fixed Overhead Variances, Graphic Presentation—Bayou Division

 width=

because it is caused by a deviation in production volume level (number of frames produced) from that estimated to arrive at the standard cost.

If Bayou had estimated 80,000 frames per month instead of 100,000 frames, the standard cost would have been $2.50 per frame (= $200,000 ÷ 80,000 frames). Thus, $200,000 (= $2.50 x 80,000 frames) would have been applied to units produced, and there would have been no production volume variance.

The production volume variance applies only to fixed costs; it occurs because we are allocating a fixed period cost to units on a predetermined basis. It does not represent resources spent or saved. This is unique to full absorption costing. The benefits of calculating the variance for control purposes are questionable. Although the production volume variance signals a difference between expected and actual production levels, so does a simple production report of actual versus expected production quantities.

Compare with the Fixed Production Cost Price Variance The fixed production cost price variance is the difference between actual and budgeted fixed production costs. Unlike the production volume variance, the price variance commonly is used for control purposes because it is a measure of differences between actual and budgeted period costs.

Exhibits 16.13 and 16.14 summarize the computation of the fixed production price (spending) and production volume variances. Reviewing them will help you see the relationship between actual, budgeted, and applied fixed production costs.

analysis of cost reports mary is the production manager of the cabot plant a divisio 691376

Analysis of Cost Reports

Mary is the production manager of the Cabot plant, a division of the larger corporation, Triparte, Inc. She has complained several times to the corporate office that the cost reports used to evaluate her plant are misleading. She states, “I know how to get good quality product out. Over a number of years, I’ve even cut the amount of raw materials used to do it. The cost reports don’t show any of this; they’re always negative, no matter what I do. There’s no way I can win with accounting or the people at headquarters who use these reports.”

A copy of the latest report follows.

CABOT PLANT

Cost Report

For the Month of March

($000)

 

Master Budget

Actual Cost

Excess Cost

Raw material

$1,200

$1,311

$111

Direct labor

1,680

1,620

(60)

Overhead

300

402

102

Total

$3,180

$3,333

$153

Required

Identify and explain at least three changes to the report that would make the cost information more meaningful and less threatening to the production managers.

change of policy to improve productivity osage electronics has been experiencing dec 691377

Change of Policy to Improve Productivity

Osage Electronics has been experiencing declining profit margins and has been looking for ways to increase operating income. It cannot raise selling prices for fear of losing business to its competitors. It must either cut costs or improve productivity.

Osage uses a standard cost system to evaluate the performance of the soldering department. It investigates all unfavorable variances at the end of the month. The soldering department rarely completes the operations in less time than the standard allows (which would result in a favorable variance). In most months, the variance is zero or slightly unfavorable. Reasoning that the application of lower standard costs to the products manufactured will result in improved profit margins, the production manager has recommended that all standard times for soldering operations be drastically reduced. The production manager has informed the soldering personnel that she expects the soldering department to meet these new standards.

Required

Will the lowering of the standard costs (by reducing the time of the soldering operations) result in improved profit margins and increased productivity?

ethics and standard costs farmer frank rsquo s produces items from local farm produc 691378

Ethics and Standard Costs

Farmer Frank’s produces items from local farm products and distributes them to supermarkets. Over the years, price competition has become increasingly important, so Susan Kramer, the company’s controller, is planning to implement a standard cost system for Farmer Frank’s. She asked her cost accountant, Margaret Chang, to gather cost information on the production of blueberry preserves (Farmer Frank’s most popular product). Margaret reported that blueberries cost $.75 per quart, the price she intends to pay to her good friend who has been operating a blueberry farm that has been unprofitable for the last few years. Because of an oversupply in the market, the price for blueberries has dropped to $.60 per quart. Margaret is sure that the $.75 price will be enough to pull her friend’s farm out of the red and into the black.

Required

Is Margaret’s behavior regarding the cost information she provided to Susan unethical? Explain your answer.

comprehensive variance problem trenton manufacturing company manufactures one produc 691379

Comprehensive Variance Problem

Trenton Manufacturing Company manufactures one product, with a standard cost detailed as follows:

Direct materials, 20 yards at $6 per yard

$120

Direct labor, 25 hours at $5 per hour

125

Factory overhead applied at 80% of

 

direct labor (variable costs = $75; fixed costs = $25)

100

Variable selling and administrative

80

Fixed selling and administrative

50

Total unit costs

$475

Standards have been computed based on a master budget activity level of 14,400 direct labor hours per month. Actual activity for the past month was as follows:

Materials used

9,500 yards at $6.15 per yard

Direct labor

12,600 hours at $5.10 per hour

Total factory overhead

$55,500

Production

500 units

Required

Prepare variance analyses for the variable and fixed costs. Indicate which variances cannot be computed. Materials are purchased as they are used.

find actual and budget amounts from variances assume that time killer inc manufactur 691380

Find Actual and Budget Amounts from Variances

Assume that Time killer, Inc., manufactures a new electronic game console. The current standard costs sheet for a game console follows:

Direct materials, ? kilograms at $4 per kilogram

$ ? per game

Direct labor, 0.75 hours at ? per hour

? per game

Overhead, 0.75 hours at ? per hour

? per game

Total costs

$39 per game

Assume that the following data appeared in Time killer’s records at the end of the past month:

Actual production

24,000 units

Actual sales

15,000 units

Materials (112,500 kilograms)

$472,500

Materials price variance

22,500 U

Materials efficiency variance

18,000 U

Direct labor price variance

13,680 U

Direct labor (17,100 hours)

287,280

Under applied overhead (total)

9,000 U

There are no materials inventories.

Required

a. Prepare a variance analysis for direct materials and direct labor and complete the standard cost sheet.

b. Assume that all production overhead is fixed and that the $9,000 under applied is the only overhead variance that can be computed. What are the actual and applied overhead amounts?

prepare flexible budget data 2 go manufactures and sells flash drives the company pr 691346

Prepare Flexible Budget

Data 2 Go manufactures and sells flash drives. The company produces only when it receives orders and, therefore, has no inventories. The following information is available for the current month:

 

Actual

(based on actual of

750,000 units)

Master Budget (based on budgeted 800,000 units)

 

 

Sales revenue

$6,210,000

$6,000,000

Less

 

 

Variable costs

 

 

Blank flash drives

1,800,000

1,800,000

Direct labor

345,000

420,000

Variable overhead

843,000

780,000

Variable marketing and administrative

585,000

600,000

Total variable costs

$3,573,000

$3,600,000

Contribution margin

$2,637,000

$2,400,000

Less

 

 

Fixed costs

 

 

Manufacturing overhead

1,236,000

1,200,000

Marketing

360,000

360,000

Administrative

255,000

225,000

Total fixed costs

$1,851,000

$1,785,000

Operating profits

$ 786,000

$ 615,000

Required

Prepare a flexible budget for Data 2 Go.

sales activity variance refer to the data in exercise 16 22 prepare a sales activity 691347

Sales Activity Variance

Refer to the data in Exercise 16 22. Prepare a sales activity variance analysis for Data 2 Go like the one in Exhibit 16.4.

Exercise 16 22: Prepare Flexible Budget

Data 2 Go manufactures and sells flash drives. The company produces only when it receives orders and, therefore, has no inventories. The following information is available for the current month:

 

Actual

(based on actual of

750,000 units)

Master Budget (based on budgeted 800,000 units)

 

 

Sales revenue

$6,210,000

$6,000,000

Less

 

 

Variable costs

 

 

Blank flash drives

1,800,000

1,800,000

Direct labor

345,000

420,000

Variable overhead

843,000

780,000

Variable marketing and administrative

585,000

600,000

Total variable costs

$3,573,000

$3,600,000

Contribution margin

$2,637,000

$2,400,000

Less

 

 

Fixed costs

 

 

Manufacturing overhead

1,236,000

1,200,000

Marketing

360,000

360,000

Administrative

255,000

225,000

Total fixed costs

$1,851,000

$1,785,000

Operating profits

$ 786,000

$ 615,000

Required

Prepare a flexible budget for Data 2 Go.

Exhibit 16.4 Flexible and Master Budget, August—Bayou Division

 

A

 

B

C

D

E

 

 

 

 

Flexible Budget

Sales Activity

 

Master Budget

 

 

 

 

(based on

Variance

 

(based on

 

 

 

 

actual activity

(based on

 

planned

 

 

 

 

of 80,000

variance in

 

activity of

 

1

 

 

units)

sales volume)

 

100,000 units)

 

2

Sales units

 

80,000

20,000

 

100,000

 

3

 

 

 

 

 

 

 

4

Sales revenue

 

$ 800,000

$ 200,000

U

$ 1,000,000

 

5

Less

 

 

 

 

 

 

6

Variable costs

 

 

 

 

 

 

7

Variable manufacturing costs

 

304,000

76,000

F

380,000

 

8

Variable selling and administrative

 

72,000

18,000

F

90,000

 

9

Total variable costs

 

$ 376,000

$

94,000

F

$

470,000

 

10

Contribution margin

 

$ 424,000

$ 106,000

U

$

530,000

 

11

Fixed costs

 

 

 

 

 

 

 

12

Fixed manufacturing overhead

 

200,000

–0–

 

 

200,000

 

13

Fixed selling and administrative costs

 

140,000

–0–

 

 

140,000

 

14

Total fixed costs

 

$ 340,000

–0–

 

$

340,000

 

15

Profit

 

$

84,000

$ 106,000

U

$

190,000

 

16

 

 

 

 

 

 

 

 

 

                     

profit variance analysis use the information from exercise 16 22 to prepare a profit 691348

Profit Variance Analysis

Use the information from Exercise 16 22 to prepare a profit variance analysis for Data 2 Go like the one in Exhibit 16.5.

Exercise 16 22: Prepare Flexible Budget

Data 2 Go manufactures and sells flash drives. The company produces only when it receives orders and, therefore, has no inventories. The following information is available for the current month:

 

Actual

(based on actual of

750,000 units)

Master Budget (based on budgeted 800,000 units)

 

 

Sales revenue

$6,210,000

$6,000,000

Less

 

 

Variable costs

 

 

Blank flash drives

1,800,000

1,800,000

Direct labor

345,000

420,000

Variable overhead

843,000

780,000

Variable marketing and administrative

585,000

600,000

Total variable costs

$3,573,000

$3,600,000

Contribution margin

$2,637,000

$2,400,000

Less

 

 

Fixed costs

 

 

Manufacturing overhead

1,236,000

1,200,000

Marketing

360,000

360,000

Administrative

255,000

225,000

Total fixed costs

$1,851,000

$1,785,000

Operating profits

$ 786,000

$ 615,000

Required

Prepare a flexible budget for Data 2 Go.

Exhibit 16.5 Profit Variance Analysis, August—Bayou Division

 

A

B

C

D

E

F

G

H

I

J

K

L

M

 

1

 

(1)

(2)

 

 

(3)

 

(4)

 

(5)

(6)

 

(7)

 

 

 

 

 

 

 

 

 

 

 

Flexible

 

 

Master Budget

 

 

 

Actual (based

 

 

 

 

 

 

 

Budget (based

 

 

(based on

 

 

 

on actual

 

 

 

Marketing and

 

 

 

on actual

Sales

 

planned

 

 

 

activity of

Manufacturing

 

 

Administrative

 

Sales Price

 

activity of

Activity

 

activity of

 

2

 

80,000 units)

Variances

 

 

Variances

 

Variance

 

80,000 units)

Variance

 

100,000 units)

 

3

Sales revenue

$ 840,000

 

 

 

 

 

$ 40,000

F

$ 800,000

$ 200,000

U

$ 1,000,000

 

4

Less

 

 

 

 

 

 

 

 

 

 

 

 

 

5

Variable costs

 

 

 

 

 

 

 

 

 

 

 

 

 

6

Variable manufacturing costs

329,680

$ 25,680

U

a

 

 

 

 

304,000

76,000

F

380,000

 

7

Variable selling and administrative

68,000

 

 

 

$ 4,000

F

 

 

72,000

18,000

F

90,000

 

8

Contribution margin

$ 442,320

$ 25,680

U

 

$ 4,000

F

$ 40,000

F

$ 424,000

$ 106,000

U

$

530,000

 

9

Fixed costs

 

 

 

 

 

 

 

 

 

 

 

 

 

 

10

Fixed manufacturing overhead

195,500

4,500

F

 

 

 

 

 

200,000

–0–

 

 

200,000

 

11

Fixed selling and administrative costs

132,320

 

 

 

7,680

F

 

 

140,000

–0–

 

 

140,000

 

12

Profit

$ 114,500

$ 21,180

U

 

$ 11,680

F

$ 40,000

F

$

84,000

$ 106,000

U

$

190,000

 

13

 

 

 

 

 

 

 

 

 

 

 

 

 

 

 

 

14

aThe individual cost variances are shown in Exhibit 16.11.

 

 

 

 

 

 

 

 

 

 

 

 

 

 

15

 

 

 

 

 

 

 

 

 

 

 

 

 

 

 

 

 

 

 

 

 

 

 

 

 

 

 

 

 

 

 

 

 

Exhibit 16.11 Variable Manufacturing Cost Variance Summary, August—Bayou Division

 width=

in Exhibit 16.5. The cost variance analysis just completed is a more detailed analysis of the variable production cost variance derived in Exhibit 16.5.

A summary of this nature is useful for reporting variances to high level managers. It provides both an overview of variances and their sources. When used for reporting, the computations at the right of Exhibit 16.11 usually are replaced with a brief explanation of the cause of the variance.

Management might want more detailed information about some of the variances. Extending each variance branch in Exhibit 16.11 to show variances by product line, department, or other categories can provide this additional detail.

assigning responsibility wallace manufacturing produces engine parts for auto manufa 691349

Assigning Responsibility

Wallace Manufacturing produces engine parts for auto manufacturers. Recently, one of the major auto firms rejected a load of manifolds as being defective. Wallace’s purchasing department had ordered from a new supplier with a much lower price. Unfortunately, the quality was much lower as well. Now the company must produce replacement parts, and the customer will not reimburse Wallace for the original cost.

The manufacturing manager argues that the purchasing department should bear all of the cost of the additional production. The purchasing department manager says that the manufacturing department should have checked the quality of the material when it was delivered.

Required

As the plant manager, how would you assign responsibility?

assigning responsibility davidson communications produces mobile phones in building 691350

Assigning Responsibility

Davidson Communications produces mobile phones. In Building 404, the phones are assembled and then sent to Building 405 where they are inspected, packaged, and shipped to the customer. On Thursday, the production supervisor in Building 405 asked the Building 404 manager to stop production. One of the electronic testing machines needed emergency maintenance. The Building 404 manager refused, saying, “I’m paid to maintain production. If I stop, I’m penalized and could lose my bonus.”

As a result, the testers in Building 405 tested a smaller sample than usual. Because of this, the return rate on the day’s production was much higher than usual. The cost of returns and warranty repairs was $75,000.

Required

As top management, to whom would you assign the responsibility for the $75,000 of returns and warranty costs?

variable cost variances the following data reflect the current month rsquo s activit 691352

Variable Cost Variances

The following data reflect the current month’s activity for Sills, Inc.:

Actual total direct labor

$546,000

Actual hours worked

26,000

Standard labor hours allowed for actual output (flexible budget)

27,000

Direct labor price variance

$19,500 U

Actual variable overhead

$132,600

Standard variable overhead rate per standard direct labor hour

$5.25

Variable overhead is applied based on standard direct labor hours allowed.

Required

Compute the labor and variable overhead price and efficiency variances.

variable cost variances the records of simon company show the following for february 691353

Variable Cost Variances

The records of Simon Company show the following for February:

Standard labor hours allowed per unit of output

1.5

Standard variable overhead rate per

 

standard direct labor hour

$30

Good units produced

60,000

Actual direct labor hours worked

92,000

Actual total direct labor

$1,975,000

Direct labor efficiency variance

$40,000 U

Actual variable overhead

$2,560,000

Required

Compute the direct labor and variable overhead price and efficiency variances.

transfer pricing performance evaluation issues cochise corporation rsquo s southern 691329

Transfer Pricing: Performance Evaluation Issues

Cochise Corporation’s Southern Division is operating at capacity. It has been asked by Northern Division to supply it a thermal switch, which Southern sells to its regular customers for $60 each. Northern, which is operating at 70 percent capacity, is willing to pay $40 each for the switch. Northern will put the switch into a kitchen appliance that it is manufacturing on a cost plus basis for the Army. Southern has a $34 variable cost of producing the switch.

The cost of the kitchen appliance as built by Northern follows:

Purchased parts—outside vendors

$180

Southern thermal switch

 40

Other variable costs

112

Fixed overhead and administration

64

Total cost

$396

Northern believes that the price concession is necessary to get the job.

The company uses ROI and dollar profits in evaluating the division and divisional manager’s performance.

Required

a. If you were Southern’s division controller, would you recommend supplying the switch to Northern? (Ignore any income tax issues.) Why or why not?

b. Would it be to the short run economic advantage of Cochise Corporation for Southern to supply Northern with the switch at $40 each? (Ignore any income tax issues.) Explain your answer.

c. Discuss the organizational and managerial behavior difficulties, if any, inherent in this situation.

As Cochise’s controller, what would you advise the corporation’s president to do in this situation?

evaluate transfer price system western states supply inc wss consists of three 691330

Evaluate Transfer Price System

Western States Supply, Inc. (WSS), consists of three divisions—California, Northwest, and Southwest—that operate as if they were independent companies. Each division has its own sales force and production facilities. Each division manager is responsible for sales, cost of operations, acquisition and financing of divisional assets, and working capital management. WSS corporate management evaluates the performance of each division and its managers on the basis of ROI.

Southwest has just been awarded a contract for a product that uses a component manufactured by outside suppliers as well as by Northwest, which is operating well below capacity. Southwest used a cost figure of $37 for the component in preparing its bid for the new product. Northwest supplied this cost figure in response to Southwest’s request for the average variable cost of the component; it represents the standard variable manufacturing cost and variable marketing costs.

Northwest’s regular selling price for the component that Southwest needs is $65. Northwest’s management indicated that it could supply Southwest the required quantities of the component at the regular selling price less variable selling and distribution expenses. Southwest management responded by offering to pay standard variable manufacturing cost plus 25 percent.

The two divisions have been unable to agree on a transfer price. Corporate management has never established a transfer price policy. The corporate controller suggested a price equal to the standard full manufacturing cost (that is, no selling and distribution expenses) plus a 20 percent markup. The two division managers rejected this price because each considered it grossly unfair.

The unit cost structure for the Northwest component and the suggested prices follow.

Costs

 

Standard variable manufacturing cost

$32

Standard fixed manufacturing cost

13

Variable selling and distribution expenses

5

Total cost

$50

Alternative transfer prices

 

Regular selling price

$65

Regular selling price less variable selling and distribution expenses ($65 $5)

$60

Variable manufacturing plus 25% ($32 x 1.25)

$40

Standard full manufacturing cost plus 20% ($45 x 1.20)

$54

Required

a. Discuss the effect that each of the proposed prices could have on the attitude of Northwest’s management toward intracompany business.

b. Is the negotiation of a price between Northwest and Southwest a satisfactory method to solve the transfer price problem? Explain your answer.

c. Should WSS’s corporate management become involved in this transfer price controversy? Explain your answer.

transfer prices and tax regulations ethical issues gage corporation has two operatin 691331

Transfer Prices and Tax Regulations: Ethical Issues

Gage Corporation has two operating divisions in a semiautonomous organizational structure. Adams Division, located in the United States, produces a specialized electrical component that is an input to Bute Division, located in the south of England. Adams uses idle capacity to produce the component, which has a domestic market price of $36. Its variable costs are $15 per unit. Gage’s U.S. tax rate is 40 percent of income.

In addition to the transfer price for each component received from Adams, Bute pays a $9 per unit shipping fee. The component becomes a part of its assembled product, which costs an additional $6 to produce and sells for an equivalent of $69. Bute could purchase the component from a Manchester (England) supplier for $30 per unit. Gage’s English tax rate is 70 percent of income. Assume that English tax laws permit transferring at either variable cost or market price.

Required

a. What transfer price is economically optimal for Gage Corporation? Show computations.

b. Is it ethical to choose a transfer price for tax purposes that is different from the transfer price used to evaluate a business unit’s performance?

c. Suppose Gage had a third operating division, Case, in Singapore, where the tax rate is below that of the United States. Would it be ethical for Gage to use different transfer prices for transactions between Adams and Bute and between Adams and Case?

segment reporting midwest entertainment has four operating divisions bus charters lo 691332

Segment Reporting

Midwest Entertainment has four operating divisions: Bus Charters, Lodging, Concerts, and Ticket Services. Each division is a separate segment for financial reporting purposes. Revenues and costs related to outside transactions were as follows for the past year (dollars in thousands):

 

Bus Charters

Lodging

Concerts

Ticket Services

Revenues

$12,250

$5,300

$4,450

$1,600

Costs

7,850

3,550

3,300

1,500

Bus Charters Division participates in a frequent guest program with Lodging Division. During the past year, Bus Charters reported that it traded lodging award coupons for travel that had a retail value of $1.3 million, assuming that the travel was redeemed at full fares. Concerts Division offered 20 percent discounts to Midwest’s bus passengers and lodging guests. These discounts to bus passengers were estimated to have a retail value of $350,000. Midwest’s lodging guests redeemed $150,000 in concert discount coupons. Midwest’s hotels also provided rooms for Bus Charter’s employees (drivers and guides). The value of the rooms for the year was $650,000.

The Ticket Services Division sold chartered tours for Bus Charters valued at $200,000 for the year. This service for intracompany lodging was valued at $100,000. It also sold concert tickets for Concerts; tickets for intracompany concert admission were valued at $50,000.

While preparing all of these data for financial statement presentation, Lodging Division’s controller stated that the value of the bus coupons should be based on their differential and opportunity costs, not on the full fare. This argument was supported because travel coupons are usually allocated to seats that would otherwise be empty or that are restricted similar to those on discount tickets. If the differential and opportunity costs were used for this transfer price, the value would be $250,000 instead of $1.3 million. Bus Charter’s controller made a similar argument concerning the concert discount coupons. If the differential cost basis were used for the concert coupons, the transfer price would be $50,000 instead of the $350,000.

Midwest reports assets in each division as follows (dollars in thousands):

Bus Charters

$47,750

Lodging

19,250

Concerts

16,050

Ticket Services

3,250

Required

a. Using the retail values for transfer pricing for segment reporting purposes, what are the operating profits for each Midwest division?

b. What are the operating profits for each Midwest division using the differential cost basis for pricing transfers?

c. Rank each division by ROI using the transfer pricing methods in ( ) and ( ). What difference does the transfer pricing system have on the rankings?

two part transfer prices mathes corporation manufactures paper products the company 691333

Two Part Transfer Prices

Mathes Corporation manufactures paper products. The company operates a landfill, which it uses to dispose of nonhazardous trash. The trash is hauled from the two nearby manufacturing facilities in trucks that can carry up to 5 tons of trash in a load. The landfill operation requires certain preparation activities regardless of the amount of trash in a truck (i.e., for each load). The budget for the landfill for next year follows:

Volume of trash

1,500 tons (300 loads)

Preparation costs (varies by loads)

$ 45,000

Other variable costs (varies by tons)

45,000

Fixed costs

110,000

Total budgeted costs

$200,000

Mathes is considering making the landfill a profit center and charging the manufacturing plants for disposing of the trash. The landfill has sufficient capacity to operate for at least the next 20 years. Other landfills are available in the area (both private and municipal), and each plant would be free to decide which landfill to use.

Required

a. What transfer pricing rule should Mathes implement at the landfill so that its plant managers would independently make decisions regarding landfill use that would be in the company’s best interests?

b. Illustrate your rule by computing the transfer price that would be applied to a 4 ton load of trash from one of the plants.

budget versus actual costs refer to the data in problem 15 35 at the end of the year 691334

Budget versus Actual Costs

Refer to the data in Problem 15 35. At the end of the year, the following data are available on actual operations at the landfill.

Volume of trash

1,250 tons (400 loads)

Preparation costs (per load)

$ 56,000

Other variable costs (per ton)

38,750

Fixed costs

108,250

Total budgeted costs

$203,000

Required

Based on the actual activities and costs, would you change the recommendation you made in Problem 15 35? Why or why not?

Problem 15 35: Two Part Transfer Prices

Mathes Corporation manufactures paper products. The company operates a landfill, which it uses to dispose of nonhazardous trash. The trash is hauled from the two nearby manufacturing facilities in trucks that can carry up to 5 tons of trash in a load. The landfill operation requires certain preparation activities regardless of the amount of trash in a truck (i.e., for each load). The budget for the landfill for next year follows:

Volume of trash

1,500 tons (300 loads)

Preparation costs (varies by loads)

$ 45,000

Other variable costs (varies by tons)

45,000

Fixed costs

110,000

Total budgeted costs

$200,000

Mathes is considering making the landfill a profit center and charging the manufacturing plants for disposing of the trash. The landfill has sufficient capacity to operate for at least the next 20 years. Other landfills are available in the area (both private and municipal), and each plant would be free to decide which landfill to use.

Required

a. What transfer pricing rule should Mathes implement at the landfill so that its plant managers would independently make decisions regarding landfill use that would be in the company’s best interests?

b. Illustrate your rule by computing the transfer price that would be applied to a 4 ton load of trash from one of the plants.

two part transfer prices chs is a large multi division firm one division health serv 691335

Two Part Transfer Prices

CHS is a large multi division firm. One division, Health Services, is well known inside CHS for it efficient information technology (IT). A smaller division, Optics, has approached Health Services with a proposal that it provide IT support in the form of machine time for some of Optic’s billing and administrative work.

After an analysis of the demands that Optics would place on the system, the IT manager of Health Services notes that Health Services would have to lease a new server because of the additional load. The lease rates for the current server are a fixed annual lease of $3,200 and it averages machine time of 2,800 hours annually. The new server leases for an annual rate of $5,000. Because the new server is a faster machine, Health Services can complete its current requirements in only 2,000 hours. The work for Optics is estimated to be 1,000 hours.

In addition to leasing a new server, there are two other changes Health Services would have to make in IT. First, they will have to upgrade their server support position. The IT manager estimates that it will cost an additional $20,000 per year to get an individual with the necessary advanced training. In addition, Health Services has a contract for service from the machine vendor. The support contract is a fixed price contract of $1 per hour of machine usage. The current lease contract can be canceled at no cost if Health Services leases a more expensive machine.

Required

a. Assume that no outside market exists for this service and that Health Services would have excess capacity on the new server. What is the optimal transfer price rule Health Services should use to charge Optics?

b. Suppose Optics uses 1,000 hours on the new machine. What is the average cost per hour Optics would pay using the rule you developed in part (a)?

c. Suppose Optics uses 100 hours on the new machine. What is the average cost per hour Optics would pay using the rule you developed in part (a)?

two part transfer prices refer to problem 15 37 suppose health services could sell t 691336

Two Part Transfer Prices

Refer to Problem 15 37. Suppose Health Services could sell time on the machine to other companies in the area on a per hour basis. Further, it can sell all the time available for $30 per hour.

Required

a. What is the optimal transfer price rule Health Services should use to charge Optics?

b. Suppose Optics uses 1,000 hours on the new machine. What is the average cost per hour Optics would pay using the rule you developed in part (a)?

c. Suppose Optics uses 100 hours on the new machine. What is the average cost per hour Optics would pay using the rule you developed in part (a)?

Problem 15 37: Two Part Transfer Prices

CHS is a large multi division firm. One division, Health Services, is well known inside CHS for it efficient information technology (IT). A smaller division, Optics, has approached Health Services with a proposal that it provide IT support in the form of machine time for some of Optic’s billing and administrative work.

After an analysis of the demands that Optics would place on the system, the IT manager of Health Services notes that Health Services would have to lease a new server because of the additional load. The lease rates for the current server are a fixed annual lease of $3,200 and it averages machine time of 2,800 hours annually. The new server leases for an annual rate of $5,000. Because the new server is a faster machine, Health Services can complete its current requirements in only 2,000 hours. The work for Optics is estimated to be 1,000 hours.

In addition to leasing a new server, there are two other changes Health Services would have to make in IT. First, they will have to upgrade their server support position. The IT manager estimates that it will cost an additional $20,000 per year to get an individual with the necessary advanced training. In addition, Health Services has a contract for service from the machine vendor. The support contract is a fixed price contract of $1 per hour of machine usage. The current lease contract can be canceled at no cost if Health Services leases a more expensive machine.

Required

a. Assume that no outside market exists for this service and that Health Services would have excess capacity on the new server. What is the optimal transfer price rule Health Services should use to charge Optics?

b. Suppose Optics uses 1,000 hours on the new machine. What is the average cost per hour Optics would pay using the rule you developed in part (a)?

c. Suppose Optics uses 100 hours on the new machine. What is the average cost per hour Optics would pay using the rule you developed in part (a)?

custom freight systems a transfer pricing ldquo we can rsquo t drop our prices below 691337

Custom Freight Systems (A): Transfer Pricing

“We can’t drop our prices below $210 per hundred pounds,” exclaimed Greg Berman, manager of Forwarders, a division of Custom Freight Systems. “Our margins are already razor thin. Our costs just won’t allow us to go any lower. Corporate rewards our division based on our profitability and I won’t lower my prices below $210.”

Custom Freight Systems is organized into three divisions: Air Cargo provides air cargo services, Logistics Services operates distribution centers and provides truck cargo services, and Forwarders provides international freight forwarding services (see Exhibit 15.6). Freight forwarders typically buy space on planes from international air cargo companies. This is analogous to a charter company that books seats on passenger planes and resells them to passengers. In many cases, freight forwarders hire trucking companies to transport the cargo from the plane to the domestic destination.

Management believes that the three divisions integrate well and are able to provide customers with one stop transportation services. For example, a Forwarders branch in Singapore would receive cargo from a shipper, prepare the necessary documentation, and then ship the cargo on Air Cargo to a domestic Forwarders station. The domestic Forwarders station would ensure that the cargo passes through customs and would ship it to the final destination with Logistics Services as in Exhibit 15.6.

Management evaluates each division separately and rewards divisional managers based on profit and return on investment (ROI). Responsibility and decision making authority are decentralized. Similarly, each division has a sales and marketing organization. Divisional salespeople report

Exhibit 15.6 Custom Freight Systems’s Operations

 width=

Exhibit 15.7 Organization Chart— Custom Freight Systems

 width=

to the vice president of Operations for Custom Freight Systems. See Exhibit 15.7. Custom Freight Systems believes that it successfully motivates divisional managers by paying bonuses for high divisional profits.

Recently, the Logistics division needed to prepare a bid for a customer. The customer had freight to import from an overseas supplier and wanted Logistics to submit a bid for a distribution package that included supplying air freight, receiving the freight and providing customs clearance services at the airport, warehousing, and then distributing packages to customers.

Because this was a contract for international shipping, Logistics needed to contact different freight forwarders for shipping quotes. Logistics requested quotes from Forwarders and United Systems, a competing freight forwarder. Divisions of Custom Freight Systems are free to use the most appropriate and cost effective suppliers.

Logistics received bids of $210 per hundred pounds from Forwarders and $185 per hundred pounds from United Systems. Forwarders specified in its bid that it will use Air Cargo, a division of Custom Freight Systems. Forwarder’s variable costs were $175 per hundred pounds, which included the cost of subcontracting air transportation. Air Cargo, which was experiencing a period of excess capacity, quoted Forwarders the market rate of $155. Typically, Air Cargo’s variable costs are 60 percent of the market rate.

The price difference between the two different bids alarmed Susan Burns, a contract manager at Logistics. She knows this is a competitive business and is concerned because the difference between the high and low bids was at least $1 million (current projections for the contract estimated 4,160,000 pounds during the first year). Susan contacted Greg Berman, the manager of Forwarders, and discussed the quote. “Don’t you think full markup is unwarranted due to the fact that you and the airlines have so much excess capacity?” she asked.

Susan soon realized that Greg was not going to drop the price quote. “You know how small the margins in this business are. Why should I cut my margins even smaller just to make you look good?” he asked.

Susan went to Bennie Espinosa, vice president of Operations for Custom Freight Systems and chairperson for the corporate strategy committee. “That does sound strange,” he said. “I need to examine the overall cost structure and talk to Greg. I’ll get back to you by noon Monday.”

Required

a. Which bid should the Logistics division accept: the internal bid from the Forwarders division or the external bid from United Systems?

b. What should be the transfer price on this transaction?

c. What should Bennie Espinosa do?

d. Do the reward systems for divisional managers support the best interests of both the Forwarders division and Custom Freight Systems? Give examples that support your conclusion.

custom freight systems b transfer pricing assume that all of the information is the 691338

Custom Freight Systems (B): Transfer Pricing

Assume that all of the information is the same as in Integrative Case 15 39, but instead of receiving only one outside bid, Logistics receives two. The new bid is from World Services for $195 per hundred pounds. World has offered to use Air Cargo for transporting packages. Air Cargo will charge World $155 per hundred pounds. The bids from Forwarders and United Systems remain the same as in Integrative Case 15 39, $210 and $185, respectively.

Required

Which bid should Logistics Division take? Why?

Integrative Case 15 39: Custom Freight Systems (A): Transfer Pricing

“We can’t drop our prices below $210 per hundred pounds,” exclaimed Greg Berman, manager of Forwarders, a division of Custom Freight Systems. “Our margins are already razor thin. Our costs just won’t allow us to go any lower. Corporate rewards our division based on our profitability and I won’t lower my prices below $210.”

Custom Freight Systems is organized into three divisions: Air Cargo provides air cargo services, Logistics Services operates distribution centers and provides truck cargo services, and Forwarders provides international freight forwarding services (see Exhibit 15.6). Freight forwarders typically buy space on planes from international air cargo companies. This is analogous to a charter company that books seats on passenger planes and resells them to passengers. In many cases, freight forwarders hire trucking companies to transport the cargo from the plane to the domestic destination.

Management believes that the three divisions integrate well and are able to provide customers with one stop transportation services. For example, a Forwarders branch in Singapore would receive cargo from a shipper, prepare the necessary documentation, and then ship the cargo on Air Cargo to a domestic Forwarders station. The domestic Forwarders station would ensure that the cargo passes through customs and would ship it to the final destination with Logistics Services as in Exhibit 15.6.

Management evaluates each division separately and rewards divisional managers based on profit and return on investment (ROI). Responsibility and decision making authority are decentralized. Similarly, each division has a sales and marketing organization. Divisional salespeople report

Exhibit 15.6 Custom Freight Systems’s Operations

 width=

Exhibit 15.7 Organization Chart— Custom Freight Systems

 width=

to the vice president of Operations for Custom Freight Systems. See Exhibit 15.7. Custom Freight Systems believes that it successfully motivates divisional managers by paying bonuses for high divisional profits.

Recently, the Logistics division needed to prepare a bid for a customer. The customer had freight to import from an overseas supplier and wanted Logistics to submit a bid for a distribution package that included supplying air freight, receiving the freight and providing customs clearance services at the airport, warehousing, and then distributing packages to customers.

Because this was a contract for international shipping, Logistics needed to contact different freight forwarders for shipping quotes. Logistics requested quotes from Forwarders and United Systems, a competing freight forwarder. Divisions of Custom Freight Systems are free to use the most appropriate and cost effective suppliers.

Logistics received bids of $210 per hundred pounds from Forwarders and $185 per hundred pounds from United Systems. Forwarders specified in its bid that it will use Air Cargo, a division of Custom Freight Systems. Forwarder’s variable costs were $175 per hundred pounds, which included the cost of subcontracting air transportation. Air Cargo, which was experiencing a period of excess capacity, quoted Forwarders the market rate of $155. Typically, Air Cargo’s variable costs are 60 percent of the market rate.

The price difference between the two different bids alarmed Susan Burns, a contract manager at Logistics. She knows this is a competitive business and is concerned because the difference between the high and low bids was at least $1 million (current projections for the contract estimated 4,160,000 pounds during the first year). Susan contacted Greg Berman, the manager of Forwarders, and discussed the quote. “Don’t you think full markup is unwarranted due to the fact that you and the airlines have so much excess capacity?” she asked.

Susan soon realized that Greg was not going to drop the price quote. “You know how small the margins in this business are. Why should I cut my margins even smaller just to make you look good?” he asked.

Susan went to Bennie Espinosa, vice president of Operations for Custom Freight Systems and chairperson for the corporate strategy committee. “That does sound strange,” he said. “I need to examine the overall cost structure and talk to Greg. I’ll get back to you by noon Monday.”

Required

a. Which bid should the Logistics division accept: the internal bid from the Forwarders division or the external bid from United Systems?

b. What should be the transfer price on this transaction?

c. What should Bennie Espinosa do?

d. Do the reward systems for divisional managers support the best interests of both the Forwarders division and Custom Freight Systems? Give examples that support your conclusion.

prepare a flexible budget for bayou division for august with the same master budget 691339

Prepare a flexible budget for Bayou Division for August with the same master budget as in Exhibit 16.4 but assuming that 110,000 units were actually sold.

Exhibit 16.4 Flexible and Master Budget, August—Bayou Division

 

A

 

B

C

D

E

 

 

 

 

Flexible Budget

Sales Activity

 

Master Budget

 

 

 

 

(based on

Variance

 

(based on

 

 

 

 

actual activity

(based on

 

planned

 

 

 

 

of 80,000

variance in

 

activity of

 

1

 

 

units)

sales volume)

 

100,000 units)

 

2

Sales units

 

80,000

20,000

 

100,000

 

3

 

 

 

 

 

 

 

4

Sales revenue

 

$ 800,000

$ 200,000

U

$ 1,000,000

 

5

Less

 

 

 

 

 

 

6

Variable costs

 

 

 

 

 

 

7

Variable manufacturing costs

 

304,000

76,000

F

380,000

 

8

Variable selling and administrative

 

72,000

18,000

F

90,000

 

9

Total variable costs

 

$ 376,000

$

94,000

F

$

470,000

 

10

Contribution margin

 

$ 424,000

$ 106,000

U

$

530,000

 

11

Fixed costs

 

 

 

 

 

 

 

12

Fixed manufacturing overhead

 

200,000

–0–

 

 

200,000

 

13

Fixed selling and administrative costs

 

140,000

–0–

 

 

140,000

 

14

Total fixed costs

 

$ 340,000

–0–

 

$

340,000

 

15

Profit

 

$

84,000

$ 106,000

U

$

190,000

 

16

 

 

 

 

 

 

 

 

 

                     

last month the following events took place at superior supplies bull produced 100 00 691340

Last month, the following events took place at Superior Supplies:

• Produced 100,000 “leather like” digital music player cases.

• Had standard variable costs per unit (that is, per case):

Direct materials: 3 pounds at $1.50

$ 4.50

Direct labor: 0.20 labor hours at $22.50

 4.50

Variable production overhead: .20 labor hours at $10.00

2.00

Total per case

$11.00

• Incurred actual production costs:

Direct materials purchased and used:

 

325,000 pounds at $1.40

$455,000

Direct labor: 19,000 labor hours at $25

475,000

Variable overhead

209,000

Compute the direct materials, labor, and variable production overhead price and efficiency variances.

this question follows up self study question 2 assume that the fixed production cost 691341

This question follows up Self Study Question 2. Assume that the fixed production cost budget for the month was $320,000, and actual fixed production overhead costs were $332,000. The estimated monthly production was 80,000 cases (or 16,000 standard labor hours). Compute the fixed production overhead price variance and the fixed production overhead production volume variance.

Self Study Question 2: Last month, the following events took place at Superior Supplies:

• Produced 100,000 “leather like” digital music player cases.

• Had standard variable costs per unit (that is, per case):

Direct materials: 3 pounds at $1.50

$ 4.50

Direct labor: 0.20 labor hours at $22.50

 4.50

Variable production overhead: .20 labor hours at $10.00

2.00

Total per case

$11.00

• Incurred actual production costs:

Direct materials purchased and used:

 

325,000 pounds at $1.40

$455,000

Direct labor: 19,000 labor hours at $25

475,000

Variable overhead

209,000

Compute the direct materials, labor, and variable production overhead price and efficiency variances.

economic value added suwon pharmaceuticals invests heavily in research and developme 691307

Economic Value Added

Suwon Pharmaceuticals invests heavily in research and development (R&D), although it must currently treat its R&D expenditures as expenses for financial accounting purposes. To encourage investment in R&D, Suwon evaluates its division managers using EVA. The company adjusts accounting income for R&D expenditures by assuming these expenditures create assets with a two year life. That is, the R&D expenditures are capitalized and then amortized over two years.

BK division of Suwon shows after tax income of $2.5 million for year 2. R&D expenditures in year 1 amounted to $1 million and in year 2, R&D expenditures were $1.6 million. For purposes of computing EVA, Suwon assumes all R&D expenditures are made at the beginning of the year. Before adjusting for R&D, BK division shows assets of $10 million at the beginning of year 2 and current liabilities of $200,000. Suwon computes EVA using divisional investment at the beginning of the year and a 12 percent cost of capital.

Required

Compute EVA for BK division for year 2.

economic value added biddle company uses eva to evaluate the performance of division 691308

Economic Value Added

Biddle Company uses EVA to evaluate the performance of division managers. For the Wallace Division, after tax divisional income was $400,000 in year 3.

The company adjusts the after tax income for advertising expenses. First, it adds the annual advertising expenses back to after tax divisional income. Second, the company managers believe that advertising has a three year positive effect on the sale of the company’s products, so it amortizes advertising over three years. Advertising expenses in year 1 will be expensed 50 percent, 40 percent in year 2, and 10 percent in year 3. Advertising expenses in year 2 will be expensed 50 percent, 40 percent in year 3, and 10 percent in year 4. Advertising expenses in year 3 will be amortized 50 percent, 40 percent in year 4, and 10 percent in year 5. Third, unamortized advertising expenses become part of the divisional investment in the EVA calculations. Wallace Division had incurred advertising expenses of $100,000 in year 1 and $200,000 in year 2. It incurred $240,000 of advertising in year 3.

Before considering the unamortized advertising, the Wallace Division had total assets of $4,200,000 and current liabilities of $600,000 at the beginning of year 3. Biddle Company calculates EVA using the divisional investment at the beginning of the year. The company uses a 10 percent cost of capital to compute EVA.

Required

Compute the EVA for the Wallace Division for year 3. Is the division adding value to shareholders?

barrows consumer products a i thought evaluating performance would be easier than th 691309

Barrows Consumer Products (A)

I thought evaluating performance would be easier than this. I have three vice presidents, operating the same business in three different countries. I need to be able to compare them in order to prepare compensation recommendations to the board. The problem is that there are so many variables that each of the managers can make some claim to having the best performance. I hope our consultant can help me sort this out.

Alice Karlson, Executive Vice President

Southeast Asia Emerging Markets Sector

Barrows Consumer Products

Organization

Barrows Consumer Products is a large, multinational consumer products firm based in the United States. In the mid 1990s, Barrows made a strategic decision to enter the transitional and emerging markets. Each of the new markets was led by an executive vice president and organized along country lines. Barrows believed this form of organization made it easier to evaluate each country and also made it easier to exit from a country it identified as unprofitable.

One of the new markets developed by Barrows was Southeast Asia. Although there was significant competition in the region from other Asian and European competitors, the management of Barrows believed its advantage was in its portfolio of products with widely recognized brand names. Barrows chose three countries to enter initially: Indonesia, the Philippines, and Vietnam. At the time of the decision, all three appeared to represent significant growth opportunities.

Barrows’s policy in these new markets was to install a Barrows manager originally from the country who was willing to return and manage the business. Barrows believed that this policy resulted in additional goodwill and also allowed the managers to use their knowledge of local business customs. (It also hoped to take advantage of any personal ties the managers might have in business and government, but this was not included in its policy statement.) A simplified organization chart for the Southeast Asia Emerging Markets Sector is provided in Exhibit 14.12.

Although all three countries could be classified as emerging or transitional economies, there are considerable differences among them. Indonesia has a very large population, while the Philippines and Vietnam are smaller. The Philippines, however, has a higher level of per capita income; Vietnam is the poorest of the three countries. Selected demographic data for the three countries are shown in Exhibit 14.13.

Exhibit 14.12 Organization Chart, Southeast Asia Emerging Markets Sector—Barrows Consumer Products

 width=

Exhibit 14.13 Selected Demographic Data, Southeast Asia Emerging Markets Sector

 width=

Performance Evaluation

Barrows has a well developed set of performance measures that is used for managerial evaluation. The two primary measures that are used for groups in the United States, Canada, Western Europe, and Japan are division (or country) profit and return on investment (ROI). Return on investment is computed by dividing division (or country) operating income (essentially, income before taxes) by division (or country) total assets. While profit and ROI are commonly used in much of the company, the executive vice presidents in emerging market sectors are given considerable leeway in evaluating their individual country vice presidents. This performance evaluation is important to these managers. Compensation in the Southeast Asia Sector consists of salary and bonus. The bonus pool for the three managers is dictated by corporate headquarters of Barrows in the United States. The bonus pool formula is not explicitly defined although there is a clear correlation between the size of the pool and the profitability of the sector, however measured.

The allocation of the pool to the individual country managers is at the discretion of Ms. Karlson, the sector executive vice president. In March of year 9, the financial results from the three countries for year 8 have been tabulated and she is now evaluating them. Because this is her first year in this position, she has not had to perform this task in the past. She has hired a local compensation consultant to advise her on the relative performance of the three managers.

The financial staff at sector headquarters receives the financial statements from the controller’s staff in each of the three countries and ensures that the statements are consistently prepared in a common currency. The income statements for year 8 are shown in Exhibit 14.14. The balance sheets as of the beginning of year 8 are shown in Exhibit 14.15. Ms. Karlson discusses the source of her concern.

When I look at the financial statements, I can see immediately that Ade [Darmadi, V.P.—Indonesia] has outperformed Isadore [Real, V.P.—Philippines]. But Indonesia is a much larger market than the Philippines. So I calculate ROI to try and adjust for size and now Isadore is outperforming Ade. When I mention this to Ade, she counters that although Indonesia is larger, it is also poorer and geographically dispersed, leading to higher distribution costs. The only thing I can say for sure is that Binh has not developed much of a market.

I also wonder whether headquarters is looking at the right performance measure. I recently attended a seminar on new performance evaluation measures and the seminar speaker spent quite a bit of time on something called

Exhibit 14.14 Country Level Income Statements, Emerging Markets Sector—Barrows Consumer Products

 width=

Exhibit 14.15 Country Level Balance Sheets, Emerging Markets Sector—Barrows Consumer Products

 width=

economic value added (EVA). The way I understand it, EVA adjusts profit and subtracts a capital charge from it. The capital charge is the cost of capital multiplied by the net assets (total assets less current liabilities) employed. I guess I would use the cost of capital of 20 percent after tax that corporate policy requires I use for investment decisions. The problem I have is I am not sure how to adjust income, which is an accounting measure, into something more meaningful. We don’t do any R&D here, so the only item on the statements that was mentioned at the seminar is advertising. (Note: Advertising expenses for the previous three years are shown in Exhibit 14.16.) When I was working in the United States, I came across a study stating that advertising expenditures in our industry have an expected life of about three years. If that’s true, then clearly the way we account for advertising is wrong and I should adjust these results accordingly.

There are other issues that I think are more ambiguous. For one thing, Binh developed a new approach for delivering products that cut distribution costs in Vietnam. At our annual retreat, he shared his ideas with Ade and Isadore about how they could adapt this to their markets. In addition, many customers want their stores in Vietnam and Indonesia to be entirely served from Indonesia, so Binh receives no credit for that business.

Exhibit 14.16 Historical Advertising Expenses, Emerging Markets Sector—Barrows Consumer Products

 width=

Required

a. What are some of the factors causing the problems in measuring performance in the Southeast Asia Sector?

b. Rank the three countries using each of the following measures of performance:

1. Country profit.

2. Return on investment.

3. Economic value added (EVA).

c. Are there other performance measures you would suggest? How would you measure these?

d. Write a one page memo to Ms. Karlson explaining which country performed best. Be sure to explain your reasoning.

capital investment analysis and decentralized performance measurement the following 691310

Capital Investment Analysis and Decentralized Performance Measurement

The following exchange occurred just after the finance staff at Diversified Electronics rejected a capital investment proposal.

David Parker (Product Development) : I just don’t understand why you rejected my proposal. We can expect to make $230,000 on it before tax.

Shannon West (Finance) : David, get real. This product proposal does not meet our short term ROI target of 15 percent after tax.

David : I’m not so sure about the ROI target, but it is profitable—$230,000 worth.

Shannon : We believe that a company like Diversified Electronics should have a return on investment of 15 percent after tax. The Professional Services division consistently comes in with a 15 percent or better ROI, while your division, Residential Products, has managed to get only 10 percent. The performance of the Aerospace Products division has been especially dismal, with an ROI of only 6 percent. We expect divisions in the future to carry their share of the load.

Diversified Electronics, a growing company in the electronics industry, had grown to its present size of more than $140 million in sales. (See Exhibits 14.17 and 14.18 for Diversifier’s year 1 and year 2 income statements and balance sheets, respectively.) Diversified Electronics has three divisions, Residential Products, Aerospace Products, and Professional Services, each of which accounts for about one third of Diversified Electronics sales. Residential Products, the oldest division, produces furnace thermostats and similar products. The Aerospace Products division is a large job shop that builds electronic devices to customer specifications. A typical job or batch takes several months to complete. About one half of Aerospace Product’s sales are to the U.S. Defense Department. The newest of the three divisions, Professional Services, provides consulting engineering services. This division has grown tremendously since Diversified Electronics acquired it seven years ago.

Exhibit 14.17 Income Statements— Diversified Electronics

 width=

Exhibit 14.18 Balance Sheets— Diversified Electronics

 width=

Each division operates independently of the others, and corporate management treats each as a separate entity. Division managers make many of the operating decisions. Corporate management coordinates the activities of the various divisions, including the review of all investment proposals over $400,000.

Diversified Electronics measures return on investment as the division’s net income divided by total assets. Each division’s expenses include the allocated portion of corporate administrative expenses. Since each of Diversified Electronic’s divisions is located in a separate facility, management can easily attribute most assets, including receivables, to specific divisions. Management allocates the corporate office assets, including the centrally controlled cash account, to the divisions on the basis of divisional revenues.

Exhibit 14.19 shows the details of David Parker’s rejected product proposal.

Required

a. Was the decision to reject the new product proposal the right one? If top management used the discounted cash flow (DCF) method instead, what would the results be? The company uses a 15 percent after tax cost of capital (i.e., discount rate) in evaluating projects such as these.

b. Evaluate the manner in which Diversified Electronics has implemented the investment center concept. What pitfalls did it apparently not anticipate? What, if anything, should be done with regard to the investment center approach and the use of ROI as a measure of performance?

c. What conflicting incentives for managers can occur when yearly ROI is used as a performance measure and DCF is used for capital budgeting?

Exhibit 14.19 Data—New Product Proposal

 width=

a Annual capacity of 120,000 units.

b Includes straight line depreciation on new plant and equipment, depreciated for eight years with no net salvage value at the end of eight years.

elmhurst enterprises consists of two divisions flavorings and foods flavorings divis 691311

Elmhurst Enterprises consists of two divisions: Flavorings and Foods. Flavorings Division manufactures a food flavoring that can be used in the packaged dinners that Foods Division produces and sells. Both divisions are considered profit centers, and the division managers are evaluated and compensated based on divisional profits. The following data are available concerning the flavoring and the two divisions:

 

Flavorings

Foods

 

Division

Division

Average units produced

200,000

 

Average units sold

 

200,000

Variable manufacturing

 

 

cost per unit

$1

 

Variable finishing

 

 

cost per unit

 

$4

Fixed divisional costs

 

 

(unavoidable)

$50,000

$200,000

Flavorings Division can sell all of its output to other food manufacturers for $2 per unit. Foods Division can buy flavorings from other firms (of the same quality, etc.) for $2. Foods Division sells its dinners for $10 per unit.

a. What is the optimal transfer price in this case?

b. If both division managers are given the decision authority to decide where to buy and sell flavoring, are there likely to be many disputes about the transfer price?

consider the same facts as in self study question 1 but assume there is no intermedi 691312

Consider the same facts as in Self Study Question 1 but assume there is no intermediate market for flavorings.

a. If the transfer price for flavoring is set at $2 per unit, what is the minimum price that Foods Division can charge for its product and still cover its differential costs?

b. What is the optimal transfer price?

c. What profit will the two divisions report at the optimal transfer price from part (b)?

Self Study Question 1: Elmhurst Enterprises consists of two divisions: Flavorings and Foods. Flavorings Division manufactures a food flavoring that can be used in the packaged dinners that Foods Division produces and sells. Both divisions are considered profit centers, and the division managers are evaluated and compensated based on divisional profits. The following data are available concerning the flavoring and the two divisions:

 

Flavorings

Foods

 

Division

Division

Average units produced

200,000

 

Average units sold

 

200,000

Variable manufacturing

 

 

cost per unit

$1

 

Variable finishing

 

 

cost per unit

 

$4

Fixed divisional costs

 

 

(unavoidable)

$50,000

$200,000

Flavorings Division can sell all of its output to other food manufacturers for $2 per unit. Foods Division can buy flavorings from other firms (of the same quality, etc.) for $2. Foods Division sells its dinners for $10 per unit.

a. What is the optimal transfer price in this case?

b. If both division managers are given the decision authority to decide where to buy and sell flavoring, are there likely to be many disputes about the transfer price?

suppose that peggy o rsquo brien as the controller of padre papers decides to use a 691313

Suppose that Peggy O’Brien, as the controller of Padre Papers, decides to use a dual transfer pricing policy. Wood Division will sell wood to Paper Division for $50, and Paper Division will buy wood from Wood Division for $20. Paper can sell its product for $120 per unit, and all other data are unchanged from Exhibit 15.1. What is income for each of the divisions and for Padre Papers under this transfer pricing policy?

Exhibit 15.1 Cost and Production Data—Padre Papers

 

A

B

C

1

 

Wood

Paper

2

Average units produced

100,000

 

3

Average units sold

 

100,000

4

Variable manufacturing cost per unit

$ 20

 

5

Variable finishing cost per unit

 

$ 30

6

Fixed divisional costs (unavoidable)

$ 2,000,000

$ 4,000,000

7

 

 

 

apply transfer pricing rules best practices inc is a management consulting firm its 691314

Apply Transfer Pricing Rules

Best Practices, Inc., is a management consulting firm. Its Corporate Division advises private firms on the adoption and use of cost management systems. Government Division consults with state and local governments. Government Division has a client that is interested in implementing an activity based costing system in its public works department. The division’s head approached the head of Corporate Division about using one of its associates. Corporate Division charges clients $450 per hour for associate services, the same rate other consulting companies charge.

The Government Division head complained that it could hire its own associate at an estimated variable cost of $150 per hour, which is what Corporate pays its associates.

Required

a. What is the minimum transfer price that Corporate Division should obtain for its services, assuming that it is operating at capacity?

b. What is the maximum price that Government Division should pay?

c. Would your answers in (a) or (b) change if Corporate Division had idle capacity? If so, which answer would change, and what would the new amount be?

evaluate transfer pricing system mississippi company has two decentralized divisions 691315

Evaluate Transfer Pricing System

Mississippi Company has two decentralized divisions, Illinois and Iowa. Illinois always has purchased certain units from Iowa at $60 per unit. Because Iowa plans to raise the price to $80 per unit, Illinois is considering buying these units from outside suppliers for $60 per unit. Iowa’s costs follow:

Variable costs per unit

$56

Annual fixed costs

$100,000

Annual production of these units

5,000 units

Required

If Illinois buys from an outside supplier, the facilities that Iowa uses to manufacture these units will remain idle. What will be the result if Mississippi enforces a transfer price of $80 per unit between Illinois and Iowa?

evaluate transfer pricing system division a offers its product to outside markets fo 691317

Evaluate Transfer Pricing System

Division A offers its product to outside markets for $30. It incurs variable costs of $11 per unit and fixed costs of $75,000 per month based on monthly production of 4,000 units. Division B can acquire the product from an alternate supplier for $31 per unit or from Division A for $30 plus $2 per unit in transportation costs in addition to the transfer price charged by Division A.

Required

a. What are the costs and benefits of the alternatives available to Division A and Division B with respect to the transfer of Division A’s product? Assume that Division A can market all that it can produce.

b. How would your answer change if Division A had idle capacity sufficient to cover all of Division B’s needs?

evaluate transfer pricing system seattle transit ltd operates a local mass transit s 691318

Evaluate Transfer Pricing System

Seattle Transit Ltd. operates a local mass transit system. The transit authority is a state governmental agency. It has an agreement with the state government to provide rides to senior citizens for 50 cents per trip. The government will reimburse Seattle Transit for the “cost” of each trip taken by a senior citizen.

The regular fare is $2 per trip. After analyzing its costs, Seattle Transit figured that with its operating deficit, the full cost of each ride on the transit system is $4. Routes, capacity, and operating costs are unaffected by the number of senior citizens on any route.

Required

a. What alternative prices could be used to determine the governmental reimbursement to Seattle Transit?

b. Which price would Seattle Transit prefer? Why?

c. Which price would the state government prefer? Why?

d. If Seattle Transit provides an average of 150,000 trips for senior citizens in a given month, what is the monthly value of the difference between the prices in (b) and (c)?

evaluate transfer pricing system carmen seville and don turco jointly own bright gre 691319

Evaluate Transfer Pricing System

Carmen Seville and Don Turco jointly own Bright Green Temp Services (BGTS). Carmen owns 60 percent and Don owns 40 percent. The company provides temporary clerical services at a rate of $40 per hour. During the past year, its clients used 14,000 hours of temporary services.

Big City Developers purchased 2,800 hours of temporary services from BGTS last year. Carmen has a 20 percent interest in Big City Developers, and Don has a 60 percent interest in it. At the end of the year, Don suggested that BGTS give Big City Developers a 10 percent reduction in the hourly rate charged next year in recognition of its large purchases and desirability as a client.

Required

Assuming that Big City Developers purchases the same number of hours and that all other costs and activities remain the same in the coming year, what effect would the price reduction have on BGTS’s operating profits that accrue to Carmen and to Don for the coming year?

international transfer prices ethical issues trans atlantic metals has two operating 691320

International Transfer Prices: Ethical Issues

Trans Atlantic Metals has two operating divisions. Its forging operation in Finland forges raw metal, cuts it, and then ships it to the United States where the company’s Gear Division uses the metal to produce finished gears. Operating expenses amount to $10 million in Finland and $30 million in the United States exclusive of the costs of any goods transferred from Finland. Revenues in the United States are $75 million.

If the metal were purchased from one of the company’s U.S. forging divisions, the costs would be $15 million. However, if it had been purchased from an independent Finnish supplier, the cost would be $20 million. The marginal income tax rate is 60 percent in Finland and 40 percent in the United States.

Required

a. What is the company’s total tax liability to both jurisdictions for each of the two alternative transfer pricing scenarios ($15 million and $20 million)?

b. Is it ethical to choose the transfer price based on the impact on taxes?

transfer pricing policies ethical issues refer to the data in exercise 15 ndash 16 s 691321

Transfer Pricing Policies: Ethical Issues

Refer to the data in Exercise 15–16. Suppose that Government Division will charge the client interested in implementing an activity based costing system by the hour based on cost plus a fixed fee, where the cost is primarily the consultant’s hourly pay. Assume also that Government Division cannot hire additional consultants. That is, if it is to do this job, it will need to use a consultant from Corporate Division.

Required

a. What is the minimum transfer price that Corporate Division should obtain for its services, assuming that it is operating at capacity? Would this be an ethical price to charge the Government client? Explain.

b. What is the transfer price you would recommend if Corporate Division was not operating at capacity? Would this be an ethical price to charge the Government client? Explain.

Exercise 15 16: Apply Transfer Pricing Rules

Best Practices, Inc., is a management consulting firm. Its Corporate Division advises private firms on the adoption and use of cost management systems. Government Division consults with state and local governments. Government Division has a client that is interested in implementing an activity based costing system in its public works department. The division’s head approached the head of Corporate Division about using one of its associates. Corporate Division charges clients $450 per hour for associate services, the same rate other consulting companies charge.

The Government Division head complained that it could hire its own associate at an estimated variable cost of $150 per hour, which is what Corporate pays its associates.

Required

a. What is the minimum transfer price that Corporate Division should obtain for its services, assuming that it is operating at capacity?

b. What is the maximum price that Government Division should pay?

c. Would your answers in (a) or (b) change if Corporate Division had idle capacity? If so, which answer would change, and what would the new amount be?

segment reporting leapin rsquo larry rsquo s pre owned cars has two divisions operat 691322

Segment Reporting

Leapin’ Larry’s Pre Owned Cars has two divisions, Operations and Financing. Operations is responsible for selling Larry’s inventory as quickly as possible and purchasing cars for future sale. Financing Division takes loan applications and packages loans into pools and sells them in the financial markets. It also services the loans. Both divisions meet the requirements for segment disclosures under accounting rules.

Operations Division had $51 million in sales last year. Costs, other than those charged by Financing Division, totaled $39 million. Financing Division earned revenues of $12 million from servicing loans and incurred outside costs of $10 million. In addition, Financing charged Operations $6 million for loan related fees. Operation’s manager complained to Larry that Financing was charging twice the commercial rate for loan related fees and that Operations would be better off sending its buyers to an outside lender.

Financing’s manager replied that although commercial rates could be lower, servicing Larry’s loans is more difficult, thereby justifying the higher fees.

Required

a. What are the reported segment operating profits for each division, ignoring income taxes and using the $6 million transfer price for the loan related fees?

b. What are the reported segment operating profits for each division, ignoring income taxes and using a $3 million commercial rate as the transfer price for the loan related fees?

segment reporting perth corporation has two operating divisions a casino and a hotel 691323

Segment Reporting

Perth Corporation has two operating divisions, a casino and a hotel. The two divisions meet the requirements for segment disclosures. Before transactions between the two divisions are considered, revenues and costs are as follows:

 

Casino

Hotel

Revenues

$80,000,000

$55,000,000

Costs

45,000,000

40,000,000

The casino and the hotel have a joint marketing arrangement by which the hotel gives coupons redeemable at casino slot machines and the casino gives discount coupons good for stays at the hotel. The value of the coupons for the slot machines redeemed during the past year totaled $12,000,000. The discount coupons redeemed at the hotel totaled $5,000,000. As of the end of the year, all coupons for the current year expired.

Required

What are the operating profits for each division considering the effects of the costs arising from the joint marketing agreement?

transfer pricing with imperfect markets roi evaluation normal costing athena company 691324

Transfer Pricing with Imperfect Markets: ROI Evaluation, Normal Costing

Athena Company has two divisions. Spartan Division, which has an investment base of $8,400,000, produces and sells 450,000 units of a product at a market price of $28 per unit. Its variable costs total $8 per unit. The division also charges each unit $14 of fixed costs based on a capacity of 500,000 units.

Trojan Division wants to purchase 100,000 units from Spartan. However, it is willing to pay only $16 per unit because it has an opportunity to accept a special order at a reduced price. The order is economically justifiable only if Trojan can acquire Spartan’s output at a reduced price.

Required

a. What is the ROI for Spartan without the transfer to Trojan?

b. What is Spartan’s ROI if it transfers 100,000 units to Trojan at $16 each?

c. What is the minimum transfer price for the 100,000 unit order that Spartan would accept if it were willing to maintain the same ROI with the transfer as it would accept by selling its 450,000 units to the outside market?

transfer pricing with imperfect markets ri evaluation normal costing refer to the da 691325

Transfer Pricing with Imperfect Markets: RI Evaluation, Normal Costing.

Refer to the data in Problem 15 26. Division managers are evaluated using residual income using a 15 percent cost of capital.

Required

a. What is the residual income for Spartan without the transfer to Trojan?

b. What is Spartan’s residual income if it transfers 100,000 units to Trojan at $16 each?

c. What is the minimum transfer price for the 100,000 unit order that Spartan would accept if it were willing to maintain the same residual income with the transfer as it would accept by selling its 450,000 units to the outside market?

Problem 15 26: Transfer Pricing with Imperfect Markets: ROI Evaluation, Normal Costing

Athena Company has two divisions. Spartan Division, which has an investment base of $8,400,000, produces and sells 450,000 units of a product at a market price of $28 per unit. Its variable costs total $8 per unit. The division also charges each unit $14 of fixed costs based on a capacity of 500,000 units.

Trojan Division wants to purchase 100,000 units from Spartan. However, it is willing to pay only $16 per unit because it has an opportunity to accept a special order at a reduced price. The order is economically justifiable only if Trojan can acquire Spartan’s output at a reduced price.

Required

a. What is the ROI for Spartan without the transfer to Trojan?

b. What is Spartan’s ROI if it transfers 100,000 units to Trojan at $16 each?

c. What is the minimum transfer price for the 100,000 unit order that Spartan would accept if it were willing to maintain the same ROI with the transfer as it would accept by selling its 450,000 units to the outside market?

evaluate profit impact of alternative transfer decisions amazon beverages produces a 691326

Evaluate Profit Impact of Alternative Transfer Decisions

Amazon Beverages produces and bottles a line of soft drinks using exotic fruits from Latin America and Asia. The manufacturing process entails mixing and adding juices and coloring ingredients at the bottling plant, which is a part of Mixing Division. The finished product is packaged in a company produced glass bottle and packed in cases of 24 bottles each.

Because the appearance of the bottle heavily influences sales volume, Amazon developed a unique bottle production process at the company’s container plant, which is a part of Container Division. The Mixing Division uses all of the container plant’s production. Each division (Mixing and Container) is considered a separate profit center and evaluated as such. As the new corporate controller, you are responsible for determining the proper transfer price to use for the bottles produced for Mixing Division.

At your request, Container Division’s general manager asked other bottle manufacturers to quote a price for the number and sizes demanded by Mixing Division. These competitive prices follow:

Volume

Total Price

Price per Case

400,000 equivalent casesa

$2,880,000

$7.20

800,000

5,000,000

6.25

1,200,000

6,480,000

5.40

a An equivalent case represents 24 bottles.

Container Division’s cost analysis indicates that it can produce bottles at these costs:

Volume

Total Cost

Cost per Case

400,000 equivalent cases

$2,400,000

$6.00

800,000

4,000,000

5.00

1,200,000

5,600,000

4.67

These costs include fixed costs of $800,000 and variable costs of $4 per equivalent case. These data have caused considerable corporate discussion as to the proper price to use in the transfer of bottles from Container Division to Mixing Division. This interest is heightened because a significant portion of a division manager’s income is an incentive bonus based on profit center results.

Mixing Division has the following costs in addition to the bottle costs:

Volume

Total Cost

Cost per Case

400,000 equivalent cases

$1,800,000

$4.50

800,000

2,600,000

3.25

1,200,000

3,400,000

2.83

The corporate marketing group has furnished the following price–demand relationship for the finished product:

 

Total Sales

Sales Price

Sales Volume

Revenue

per Case

400,000 equivalent cases

$ 8,000,000

$20

800,000

14,400,000

18

1,200,000

18,000,000

15

Required

a. Amazon Beverages has used market price transfer prices in the past. Using the current market prices and costs and assuming a volume of 1.2 million cases, calculate operating profits for:

(1) Container Division.

(2) Mixing Division.

(3) Amazon Beverages.

b. Is this production and sales level the most profitable volume for:

(1) Container Division?

(2) Mixing Division?

(3) Amazon Beverages?

Explain.

analyze transfer pricing data valencia products is a decentralized organization that 691328

Analyze Transfer Pricing Data

Valencia Products is a decentralized organization that evaluates divisional management based on measures of divisional contribution margin. Consumer Audio (CA) Division and Auto Electronics (AE) Division both sell audio equipment. CA focuses on home and personal audio equipment; AE focuses on components for automobile stereo systems. CA produces an audio player that it can sell to the outside market for $60 per unit. The outside market can absorb up to 70,000 units per year. These units require 3 direct labor hours each.

If CA modifies the units with an additional hour of labor time, it can sell them to AE for $67.50 per unit. AE will accept up to 60,000 of these units per year.

If AE does not obtain 60,000 units from CA, it purchases them for $70 each from the outside. AE incurs $30 of additional labor and other out of pocket costs to convert the player into one that fits in the dashboard and integrates with the automobile’s audio system. The units can be sold to the outside market for $170 each.

CA estimates that its total costs are $825,000 for fixed costs, $12 per direct labor hour, and $6 per audio player for materials and other variable costs besides direct labor. Its capacity is limited to 300,000 direct labor hours per year.

Required

Determine the following:

a. Total contribution margin to CA if it sells 70,000 units outside.

b. Total contribution margin to CA if it sells 60,000 units to AE.

c. The costs to be considered in determining the optimal company policy for sales by CA.

d. The annual contributions and costs for CA and AE under the optimal policy.

prepare cash budget for service organization the board of directors of the cortez be 691284

Prepare Cash Budget for Service Organization

The board of directors of the Cortez Beach Yacht Club (CBYC) is developing plans to acquire more equipment for lessons and rentals and to expand club facilities. The board plans to purchase about $50,000 of new equipment each year and wants to begin a fund to purchase a $600,000 piece of property for club expansion.

The club manager is concerned about the club’s capability to purchase equipment and expand its facilities. One club member has agreed to help prepare the following financial statements and help the manager ascertain whether the plans are realistic. Additional information follows the financial statements.

CORTEZ BEACH YACHT CLUB

Statement of Income (Cash Basis)

For the Year Ended October 31

 

Year 9

Year 8

Cash revenues

   

Annual membership fees

$ 710,000

$600,000

Lesson and class fees

468,000

360,000

Miscellaneous

4,000

3,000

Total cash received

$1,182,000

$963,000

Cash costs

   

Manager’s salary and benefits

$ 72,000

$ 72,000

Regular employees’ wages and benefits

380,000

380,000

Lesson and class employees’ wages and benefits

390,000

300,000

Supplies

32,000

31,000

Utilities (heat and light)

44,000

30,000

Mortgage interest

70,200

75,600

Miscellaneous

4,000

3,000

Total cash costs

$ 992,200

$891,600

Cash income

$ 189,800

$ 71,400

Additional Information

1. Other financial information as of October 31, Year 9:

a. Cash in checking account, $14,000.

b. Petty cash, $600.

c. Outstanding mortgage balance, $720,000.

d. Accounts payable for supplies and utilities unpaid as of October 31, Year 9, and due in November Year 9, $5,000.

2. The club purchased $50,000 worth of sailing equipment during the current fiscal year (ending October 31, Year 9). Cash of $20,000 was paid on delivery, with the balance due on October 1, which had not been paid as of October 31, Year 9.

3. The club began operations in year 3 in rental quarters. In October Year 5, it purchased its current property (land and building) for $1,200,000, paying $240,000 down and agreeing to pay $60,000 plus 6 percent interest annually on the previously unpaid loan balance each November 1, starting November 1, Year 6.

4. Membership rose 3 percent during year 9, approximately the same annual rate of increase the club has experienced since it opened and that is expected to continue in the future.

5. Membership fees were increased by 15 percent in year 9. The board has tentative plans to increase them by 10 percent in year 10.

6. Lesson and class fees have not been increased for three years. The number of classes and lessons has grown significantly each year; the percentage growth experienced in year 9 is expected to be repeated in year 10.

7. Miscellaneous revenues are expected to grow in year 10 (over year 9) at the same percentage as experienced in year 9 (over year 8).

8. Lesson and class employees’ wages and benefits will increase to $604,650. The wages and benefits of regular employees and the manager will increase 15 percent. Equipment depreciation and supplies, utilities, and miscellaneous expenses are expected to increase 25 percent.

Required

a. Construct a cash budget for year 10 for Cortez Beach Yacht Club.

b. Identify any operating problem(s) that this budget discloses for CBYC. Explain your answer.

c. Is the manager’s concern that the board’s goals are unrealistic justified? Explain your answer.

home furnishings inc is a nationwide retailer of home furnishings it is organized in 691285

Home Furnishings, Inc., is a nationwide retailer of home furnishings. It is organized into two divisions, Kitchen Products and Bath Products. Selected information on performance for year 2 follows:

a. Compute after tax divisional income for the two divisions.

The tax rate is 35 percent. Comment on the results.

b. Using the information from requirement (a), assess the relative performance of the two division managers at Home Furnishings, Inc.

 

Kitchen

Bath

 

($000)

Revenue

$10,000

$5,000

Cost of sales

5,400

3,000

Allocated corporate overhead

460

200

Local advertising

2,000

500

Other general and admin

500

260

       

consider the case of home furnishings inc which was described in self study question 691287

Consider the case of Home Furnishings, Inc., which was described in Self Study Question 1. Divisional assets are $8,200,000 in Kitchen Products and $4,000,000 in Bath Products.

a. Compute ROI for the two divisions.

b. Assess the relative performance of the two division managers at Home Furnishings, Inc., using ROI.

Case Study Question 1: Home Furnishings, Inc., is a nationwide retailer of home furnishings. It is organized into two divisions, Kitchen Products and Bath Products. Selected information on performance for year 2 follows:

a. Compute after tax divisional income for the two divisions.

The tax rate is 35 percent. Comment on the results.

b. Using the information from requirement (a), assess the relative performance of the two division managers at Home Furnishings, Inc.

 

Kitchen

Bath

 

($000)

Revenue

$10,000

$5,000

Cost of sales

5,400

3,000

Allocated corporate overhead

460

200

Local advertising

2,000

500

Other general and admin

500

260

       

winter division of seasons inc acquired depreciable assets costing 4 million the cas 691288

Winter Division of Seasons, Inc., acquired depreciable assets costing $4 million. The cash flows from these assets for three years were as follows:

Year

Cash Flow

1

$1,000,000

2

1,200,000

3

1,420,000

Depreciation of these assets was 10 percent per year; the assets have no salvage value after 10 years. The denominator in the ROI calculation is based on end of year asset values. If replaced with identical new assets, these assets would cost $5,000,000 at the end of year 1, $6,250,000 at the end of year 2, and $7,800,000 at the end of year 3.

Compute the ROI for each year under each of the following methods (ignore holding gains and losses):

a. Historical cost, net book value.

b. Current cost, net book value.

c. Historical cost, gross book value.

d. Current cost, gross book value.

compute divisional income eastern merchants shows the following information for its 691289

Compute Divisional Income

Eastern Merchants shows the following information for its two divisions for year 1:

 

Eastern

Western

Revenue

$1,200,000

$3,800,000

Cost of sales

769,500

1,900,000

Allocated corporate overhead

72,000

228,000

Other general and administration

158,500

1,100,000

Required

Compute divisional operating income for the two divisions. Ignore taxes. How well have these divisions performed?

compare alternative measures of division performance the following data are availabl 691291

Compare Alternative Measures of Division Performance

The following data are available for two divisions of Solomons Company:

 

North Division

South Division

Division operating profit

$ 7,000,000

$ 39,000,000

Division investment

28,000,000

260,000,000

The cost of capital for the company is 10 percent. Ignore taxes.

Required

a. If Solomons measures performance using ROI, which division had the better performance?

b. If Solomons measures performance using economic value added, which division had the better performance? (The divisions have no current liabilities.)

c. Would your evaluation change if the company’s cost of capital were 20 percent?

impact of new asset on performance measures ocean division currently earns 780 000 a 691292

Impact of New Asset on Performance Measures

Ocean Division currently earns $780,000 and has divisional assets of $3.9 million. The division manager is considering the acquisition of a new asset that will add to profit. The investment has a cost of $675,000 and will have a yearly cash flow of $168,000. The asset will be depreciated using the straight line method over a six year life and is expected to have no salvage value. Divisional performance is measured using ROI with beginning of year net book values in the denominator. The company’s cost of capital is 15 percent. Ignore taxes.

Required

a. What is the divisional ROI before acquisition of the new asset?

b. What is the divisional ROI in the first year after acquisition of the new asset?

impact of leasing on performance measures refer to the data in exercise 14 24 the di 691293

Impact of Leasing on Performance Measures

Refer to the data in Exercise 14 24. The division manager learns that he has the option to lease the asset on a year to year lease for $148,000 per year. All depreciation and other tax benefits would accrue to the lessor. What is the divisional ROI if the asset is leased?

Exercise 14 24: Impact of New Asset on Performance Measures

Ocean Division currently earns $780,000 and has divisional assets of $3.9 million. The division manager is considering the acquisition of a new asset that will add to profit. The investment has a cost of $675,000 and will have a yearly cash flow of $168,000. The asset will be depreciated using the straight line method over a six year life and is expected to have no salvage value. Divisional performance is measured using ROI with beginning of year net book values in the denominator. The company’s cost of capital is 15 percent. Ignore taxes.

Required

a. What is the divisional ROI before acquisition of the new asset?

b. What is the divisional ROI in the first year after acquisition of the new asset?

impact of an asset disposal on performance measures noonan division has total assets 691294

Impact of an Asset Disposal on Performance Measures

Noonan Division has total assets (net of accumulated depreciation) of $2,200,000 at the beginning of year 1. One of the assets is a machine that has a net book value of $200,000. Expected divisional income in year 1 is $330,000 including $28,000 in income generated by the machine (after depreciation). Noonan’s cost of capital is 12 percent. Noonan is considering disposing of the asset today (the beginning of year 1).

Required

a. Noonan computes ROI using beginning of the year net assets. What will the divisional ROI be for year 1 assuming Noonan retains the asset?

b. What would divisional ROI be for year 1 assuming Noonan disposes of the asset for its book value (there is no gain or loss on the sale)?

c. Noonan computes residual income using beginning of the year net assets. What will the divisional residual income be for year 1 assuming Noonan retains the asset?

d. What would divisional residual income be for year 1 assuming Noonan disposes of the asset for its book value (there is no gain or loss on the sale)?

impact of an asset disposal on performance measures refer to the facts in exercise 1 691295

Impact of an Asset Disposal on Performance Measures

Refer to the facts in Exercise 14 27, but assume that Noonan has been leasing the machine for $40,000 annually. Assume also that the machine generates income of $28,000 annually after the lease payment. Noonan can cancel the lease on the machine without penalty at any time.

Required

a. Noonan computes ROI using beginning of the year net assets. What will the divisional ROI be for year 1 assuming Noonan retains the asset?

b. What would divisional ROI be for year 1 assuming Noonan disposes of the asset?

c. Noonan computes residual income using beginning of the year net assets. What will the divisional residual income be for year 1 assuming Noonan retains the asset?

d. What would divisional residual income be for year 1 assuming Noonan disposes of the asset for its book value (there is no gain or loss on the sale)?

Exercise 14 27: Impact of an Asset Disposal on Performance Measures

Noonan Division has total assets (net of accumulated depreciation) of $2,200,000 at the beginning of year 1. One of the assets is a machine that has a net book value of $200,000. Expected divisional income in year 1 is $330,000 including $28,000 in income generated by the machine (after depreciation). Noonan’s cost of capital is 12 percent. Noonan is considering disposing of the asset today (the beginning of year 1).

Required

a. Noonan computes ROI using beginning of the year net assets. What will the divisional ROI be for year 1 assuming Noonan retains the asset?

b. What would divisional ROI be for year 1 assuming Noonan disposes of the asset for its book value (there is no gain or loss on the sale)?

c. Noonan computes residual income using beginning of the year net assets. What will the divisional residual income be for year 1 assuming Noonan retains the asset?

d. What would divisional residual income be for year 1 assuming Noonan disposes of the asset for its book value (there is no gain or loss on the sale)?

compare historical cost net book value to gross book value the caribbean division of 691296

Compare Historical Cost, Net Book Value to Gross Book Value

The Caribbean Division of Mega Entertainment Corporation just started operations. It purchased depreciable assets costing $30 million and having a four year expected life, after which the assets can be salvaged for $6 million. In addition, the division has $30 million in assets that are not depreciable. After four years, the division will have $30 million available from these non depreciable assets. This means that the division has invested $60 million in assets with a salvage value of $36 million. Annual depreciation is $6 million. Annual operating cash flows are $15 million. In computing ROI, this division uses end of year asset values in the denominator. Depreciation is computed on a straight line basis, recognizing the salvage values noted. Ignore taxes.

Required

a. Compute ROI, using net book value for each year.

b. Compute ROI, using gross book value for each year.

compare roi using net book and gross book values refer to the data in exercise 14 29 691297

Compare ROI Using Net Book and Gross Book Values

Refer to the data in Exercise 14 29. Assume that the division uses beginning of year asset values in the denominator for computing ROI.

Required

a. Compute ROI, using net book value.

b. Compute ROI, using gross book value.

c. If you worked Exercise 14 29, compare those results with those in this exercise. How different is the ROI computed using end of year asset values, as in Exercise 14 29, from the ROI using beginning of year values in this exercise?

Exercise 14 29: Compare Historical Cost, Net Book Value to Gross Book Value

The Caribbean Division of Mega Entertainment Corporation just started operations. It purchased depreciable assets costing $30 million and having a four year expected life, after which the assets can be salvaged for $6 million. In addition, the division has $30 million in assets that are not depreciable. After four years, the division will have $30 million available from these non depreciable assets. This means that the division has invested $60 million in assets with a salvage value of $36 million. Annual depreciation is $6 million. Annual operating cash flows are $15 million. In computing ROI, this division uses end of year asset values in the denominator. Depreciation is computed on a straight line basis, recognizing the salvage values noted. Ignore taxes.

Required

a. Compute ROI, using net book value for each year.

b. Compute ROI, using gross book value for each year.

effects of current cost on performance measurements upper division of lower company 691299

Effects of Current Cost on Performance Measurements

Upper Division of Lower Company acquired an asset with a cost of $600,000 and a four year life. The cash flows from the asset, considering the effects of inflation, were scheduled as follows:

Year

Cash Flow

1

$225,000

2

255,000

3

285,000

4

300,000

The cost of the asset is expected to increase at a rate of 10 percent per year, compounded each year. Performance measures are based on beginning of year gross book values for the investment base. Ignore taxes.

Required

a. What is the ROI for each year of the asset’s life, using a historical cost approach?

b. What is the ROI for each year of the asset’s life if both the investment base and depreciation are determined by the current cost of the asset at the start of each year?

equipment replacement and performance measures oscar clemente is the manager of forb 691300

Equipment Replacement and Performance Measures

Oscar Clemente is the manager of Forbes Division of Pitt, Inc., a manufacturer of biotech products. Forbes Division, which has $4 million in assets, manufactures a special testing device. At the beginning of the current year, Forbes invested $5 million in automated equipment for test machine assembly. The division’s expected income statement at the beginning of the year was as follows:

Sales revenue

$16,000,000

Operating costs

 

Variable

2,000,000

Fixed (all cash)

7,500,000

Depreciation

 

New equipment

1,500,000

Other

1,250,000

Division operating profit

$ 3,750,000

A sales representative from LSI Machine Company approached Oscar in October. LSI has for $6.5 million a new assembly machine that offers significant improvements over the equipment Oscar bought at the beginning of the year. The new equipment would expand division output by 10 percent while reducing cash fixed costs by 5 percent. It would be depreciated for accounting purposes over a three year life. Depreciation would be net of the $500,000 salvage value of the new machine. The new equipment meets Pitt’s 20 percent cost of capital criterion. If Oscar purchases the new machine, it must be installed prior to the end of the year. For practical purposes, though, Oscar can ignore depreciation on the new machine because it will not go into operation until the start of the next year.

The old machine, which has no salvage value, must be disposed of to make room for the new machine.

Pitt has a performance evaluation and bonus plan based on ROI. The return includes any losses on disposal of equipment. Investment is computed based on the end of year balance of assets, net book value. Ignore taxes.

Required

a. What is Forbes Division’s ROI if Oscar does not acquire the new machine?

b. What is Forbes Division’s ROI this year if Oscar acquires the new machine?

c. If Oscar acquires the new machine and it operates according to specifications, what ROI is expected for next year?

evaluate trade offs in return measurement oscar clemente problem 14 ndash 33 is stil 691301

Evaluate Trade Offs in Return Measurement

Oscar Clemente (Problem 14–33) is still assessing the problem of whether to acquire LSI’s assembly machine. He learns that the new machine could be acquired next year, but if he waits until then, it will cost 15 percent more. The salvage value would still be $500,000. Other costs or revenue estimates would be apportioned on a month by month basis for the time each machine (either the current machine or the machine Oscar is considering) is in use. Fractions of months may be ignored. Ignore taxes.

Required

a. When would Oscar want to purchase the new machine if he waits until next year?

b. What are the costs that must be considered in making this decision?

Problem 14 33: Equipment Replacement and Performance Measures

Oscar Clemente is the manager of Forbes Division of Pitt, Inc., a manufacturer of biotech products. Forbes Division, which has $4 million in assets, manufactures a special testing device. At the beginning of the current year, Forbes invested $5 million in automated equipment for test machine assembly. The division’s expected income statement at the beginning of the year was as follows:

Sales revenue

$16,000,000

Operating costs

 

Variable

2,000,000

Fixed (all cash)

7,500,000

Depreciation

 

New equipment

1,500,000

Other

1,250,000

Division operating profit

$ 3,750,000

A sales representative from LSI Machine Company approached Oscar in October. LSI has for $6.5 million a new assembly machine that offers significant improvements over the equipment Oscar bought at the beginning of the year. The new equipment would expand division output by 10 percent while reducing cash fixed costs by 5 percent. It would be depreciated for accounting purposes over a three year life. Depreciation would be net of the $500,000 salvage value of the new machine. The new equipment meets Pitt’s 20 percent cost of capital criterion. If Oscar purchases the new machine, it must be installed prior to the end of the year. For practical purposes, though, Oscar can ignore depreciation on the new machine because it will not go into operation until the start of the next year.

The old machine, which has no salvage value, must be disposed of to make room for the new machine.

Pitt has a performance evaluation and bonus plan based on ROI. The return includes any losses on disposal of equipment. Investment is computed based on the end of year balance of assets, net book value. Ignore taxes.

Required

a. What is Forbes Division’s ROI if Oscar does not acquire the new machine?

b. What is Forbes Division’s ROI this year if Oscar acquires the new machine?

c. If Oscar acquires the new machine and it operates according to specifications, what ROI is expected for next year?

economic value added refer to the facts in problem 14 ndash 33 assume that pitt rsqu 691302

Economic Value Added

Refer to the facts in Problem 14–33. Assume that Pitt’s performance measurement and bonus plans are based on residual income instead of ROI. Pitt uses a cost of capital of 12 percent in computing residual income.

Required

a. What is Forbes Division’s residual income if Oscar does not acquire the new machine?

b. What is Forbes Division’s residual income this year if Oscar acquires the new machine?

c. If Oscar acquires the new machine and operates it according to specifications, what residual income is expected for next year?

Problem 14 33: Equipment Replacement and Performance Measures

Oscar Clemente is the manager of Forbes Division of Pitt, Inc., a manufacturer of biotech products. Forbes Division, which has $4 million in assets, manufactures a special testing device. At the beginning of the current year, Forbes invested $5 million in automated equipment for test machine assembly. The division’s expected income statement at the beginning of the year was as follows:

Sales revenue

$16,000,000

Operating costs

 

Variable

2,000,000

Fixed (all cash)

7,500,000

Depreciation

 

New equipment

1,500,000

Other

1,250,000

Division operating profit

$ 3,750,000

A sales representative from LSI Machine Company approached Oscar in October. LSI has for $6.5 million a new assembly machine that offers significant improvements over the equipment Oscar bought at the beginning of the year. The new equipment would expand division output by 10 percent while reducing cash fixed costs by 5 percent. It would be depreciated for accounting purposes over a three year life. Depreciation would be net of the $500,000 salvage value of the new machine. The new equipment meets Pitt’s 20 percent cost of capital criterion. If Oscar purchases the new machine, it must be installed prior to the end of the year. For practical purposes, though, Oscar can ignore depreciation on the new machine because it will not go into operation until the start of the next year.

The old machine, which has no salvage value, must be disposed of to make room for the new machine.

Pitt has a performance evaluation and bonus plan based on ROI. The return includes any losses on disposal of equipment. Investment is computed based on the end of year balance of assets, net book value. Ignore taxes.

Required

a. What is Forbes Division’s ROI if Oscar does not acquire the new machine?

b. What is Forbes Division’s ROI this year if Oscar acquires the new machine?

c. If Oscar acquires the new machine and it operates according to specifications, what ROI is expected for next year?

roi and management behavior ethical issues division managers at asher company are gr 691303

ROI and Management Behavior: Ethical Issues

Division managers at Asher Company are granted a wide range of decision authority. With the exception of managing cash, which is done at corporate headquarters, divisions are responsible for sales, pricing, production, costs of operations, and management of accounts receivable, inventories, accounts payable, and use of existing facilities.

If divisions require funds for investment, division executives present investment proposals to corporate management, which analyzes and documents them. The final decision to commit funds for investment purposes rests with corporate management.

The corporation evaluates divisional executive performance by using the ROI measure. The asset base is composed of fixed assets employed plus working capital, exclusive of cash. The ROI performance of a division executive is the most important appraisal factor for salary changes. In addition, each executive’s annual performance bonus is based on ROI results, with increases in ROI having a significant impact on the amount of the bonus.

Asher adopted the ROI performance measure and related compensation procedures about 10 years ago and seems to have benefited from it. The ROI for the corporation as a whole increased during the first years of the program. Although the ROI continued to increase in each division, corporate ROI has declined in recent years. The corporation has accumulated a sizable amount of short term marketable securities in the past three years.

Corporate management is concerned about the increase in the short term marketable securities. A recent article in a financial publication suggested that some companies have overemphasized the use of ROI, with results similar to those experienced by Asher.

Required

a. Describe the specific actions that division managers might have taken to cause the ROI to increase in each division but decrease for the corporation. Illustrate your explanation with appropriate examples.

b. Using the concepts of goal congruence and motivation of division executives, explain how the overemphasis on the use of the ROI measure at Asher Company might have resulted in the recent decline in the company’s ROI and the increase in cash and short term marketable securities.

c. What changes could be made in Asher Company’s compensation policy to avoid this problem? Explain your answer.

d. Is it ethical for a manager to take actions that increase her ROI but decrease the firm’s ROI?

impact of decisions to capitalize or expense on performance measurement ethical issu 691304

Impact of Decisions to Capitalize or Expense on Performance Measurement: Ethical Issues

Pharmaceutical firms, oil and gas companies, and other ventures inevitably incur costs on unsuccessful investments in new projects (e.g., new drugs or new wells). For oil and gas firms, a debate continues over whether those costs should be written off as period expense or capitalized as part of the full cost of finding profitable oil and gas ventures. For pharmaceutical firms, GAAP in the United States is clear that R&D costs are to be expensed when incurred.

Pharm It has been writing R&D costs off to expense as incurred for both financial reporting and internal performance measurement. However, this year a new management team was hired to improve the profit of Pharm It’s Cardiology Division. The new management team was hired with the provision that it would receive a bonus equal to 10 percent of any profits in excess of base year profits of the division. However, no bonus would be paid if profits were less than 20 percent of end of year investment. The following information was included in the performance report for the division:

 

 

 

Increase over

 

This Year

Base Year

Base Year

Sales revenues

$ 20,500,000

$20,000,000

 

Costs incurred

 

 

 

R&D Expense

0

4,000,000

 

Depreciation and other amortization

3,900,000

3,750,000

 

Other costs

8,000,000

7,750,000

 

Division profit

$ 8,600,000

$ 4,500,000

$4,100,000

End of year investment

$40,500,000a

$34,500,000

 

During the year, the new team spent $5 million on R&D activities, of which $4,500,000 was for unsuccessful ventures. The new management team has included the $4,500,000 in the current end of year investment base because “You can’t invent successful drugs without missing on a few unsuccessful ones.”

Required

a. What is the ROI for the base year and the current year? Ignore taxes.

b. What is the amount of the bonus that the new management team is likely to claim? Is this ethical?

c. If you were on Pharm It’s board of directors, how would you respond to the new management’s claim for the bonus?

evaluate performance evaluation system behavioral issues several years ago seville c 691305

Evaluate Performance Evaluation System: Behavioral Issues

Several years ago, Seville Company acquired Salvador Components. Prior to the acquisition, Salvador manufactured and sold automotive components products to third party customers. Since becoming a division of Seville, Salvador has manufactured components only for products made by Seville’s Luxo Division.

Seville’s corporate management gives the Salvador Division management considerable latitude in running the division’s operations. However, corporate management retains authority for decisions regarding capital investments, product pricing, and production quantities.

Seville has a formal performance evaluation program for all division managements. The evaluation program relies substantially on each division’s ROI. Salvador Division’s income statement provides the basis for the evaluation of Salvador’s management. (See the following income statement.)

The corporate accounting staff prepares the divisional financial statements. Corporate general services costs are allocated on the basis of sales dollars, and the computer department’s actual costs are apportioned among the divisions on the basis of use. The net divisional investment includes divisional fixed assets at net book value (cost less depreciation), divisional inventory, and corporate working capital apportioned to the divisions on the basis of sales dollars.

SEVILLE COMPANY

Salvador Division

Income Statement

For the Year Ended October 31

($000)

Sales revenue

 

$32,000

Costs and expenses

   

Product costs

   

Direct materials

$ 4,000

 

Direct labor

8,800

 

Factory overhead

10,400

 

Total

$23,200

 

Less increase in inventory

2,800

$20,400

Engineering and research

 

960

Shipping and receiving

 

1,920

Division administration

   

Manager’s office

$1,680

 

Cost accounting

320

 

Personnel

656

2,656

Corporate cost

   

General services

$1,840

 

Computer

384

2,224

Total costs and expenses

 

$28,160

Divisional operating profit

 

$ 3,840

Net plant investment

 

$12,800

Return on investment

 

30%

Required

a. Discuss Seville Company’s financial reporting and performance evaluation program as it relates to the responsibilities of Salvador Division.

b. Based on your response to requirement (a), recommend appropriate revisions of the financial information and reports used to evaluate the performance of Salvador’s divisional management. If revisions are not necessary, explain why.

roi eva and different asset bases house station inc is a nationwide hardware and fur 691306

ROI, EVA, and Different Asset Bases

House Station, Inc., is a nationwide hardware and furnishings chain. The manager of the House Station Store in Portland is evaluated using ROI. House Station headquarters requires an ROI of 10 percent of assets. For the coming year, the manager estimates revenues will be $2,340,000, cost of goods sold will be $1,467,000, and operating expenses for this level of sales will be $234,000. Investment in the store assets throughout the year is $1,687,500 before considering the following proposal.

A representative of Sharp’s Appliances approached the manager about carrying Sharp’s line of appliances. This line is expected to generate $675,000 in sales in the coming year at the Portland House Station store with a merchandise cost of $513,000. Annual operating expenses for this additional merchandise line total $76,500. To carry the line of goods, an inventory investment of $495,000 throughout the year is required. Sharp’s is willing to floor plan the merchandise so that the House Station store will not have to invest in any inventory. The cost of floor planning would be $60,750 per year. House Station’s marginal cost of capital is 10 percent. Ignore taxes.

Required

a. What is the Portland House Station store’s expected ROI for the coming year if it does not carry Sharp’s appliances?

b. What is the store’s expected ROI if the manager invests in Sharp’s inventory and carries the appliance line?

c. What would the store’s expected ROI be if the manager elected to take the floor plan option?

d. Would the manager prefer (a), (b), or (c)? Why?

e. Would your answers to any of the above change if EVA was used to evaluate performance? For purposes of this problem, assume no current liabilities.

this question is based on the previous self study questions in this chapter and on t 691256

This question is based on the previous Self Study Questions in this chapter and on the Santiago Pants example in the text. Prepare a cash budget given the revised figures for Santiago Pants provided in Self Study Questions 1 and 2. Assume, however, that cash collections will decrease by the same amount as the decrease in sales except that the ending accounts receivable level will decrease by another $20,000. Lower payments for purchases of materials equal to the reduction in purchases will be required, but ending accounts payable also will decrease by $2,000. Payments for income taxes will decrease to $300,000.

Self Study Questions 1: The self study questions in this chapter provide a comprehensive budgeting problem based on data from the Santiago Pants example in the chapter. Refer to the data for Santiago Pants in the chapter example. Assume that the sales forecast was decreased to 150,000 units with no change in price. Managers revised their estimate of property taxes to $51,500 (reduced from $60,000) based on a recent property tax law change. The new target ending inventories follow

Self Study Questions 2: Recall that Santiago Pants has a sales forecast of 150,000 units and new target ending inventories as follows:

Finished goods

8,000 units

Cotton

7,000 yards

Fine cotton

400 yards

In addition, you learn that income tax expense is $500,000. Variable marketing costs change proportionately with volume; that is, the amount now is (150,000 ÷ 160,000) of the amount in the text example. Prepare a budgeted schedule of marketing and administrative costs and a budgeted income statement.

estimate sales revenues ez credit inc has 80 million in consumer loans with an avera 691258

Estimate Sales Revenues

EZ Credit, Inc., has $80 million in consumer loans with an average interest rate of 13.5 percent. The bank also has $64 million in home equity loans with an average interest rate of 9 percent. Finally, the company owns $6 million in corporate securities with an average rate of 6 percent.

EZ Credit estimates that next year its consumer loan portfolio will rise to $84 million and the interest rate will fall to 12 percent. Its home equity loans will fall to $60 million with an average interest rate of 8 percent, and its corporate securities portfolio will increase to $16 million with an average rate of 7 percent.

Required

Estimate EZ Credit’s revenues for the coming year.

estimate sales revenues starlite company manufactures office products last year it s 691259

Estimate Sales Revenues

Starlite Company manufactures office products. Last year, it sold 45,000 electric staplers for $10 per unit. The company estimates that this volume represents a 30 percent share of the current electric stapler market. The market is expected to increase by 10 percent next year. Marketing specialists have determined that as a result of new competition, the company’s market share will fall to 25 percent (of this larger market). Due to changes in prices, the new price for the electric staplers will be $11 per unit. This new price is expected to be in line with the competition and have no effect on the volume estimates.

Required

Estimate Starlite’s sales revenues from electric staplers for the coming year.

estimate production and materials requirements the casings plant of wyoming machines 691261

Estimate Production and Materials Requirements

The Casings Plant of Wyoming Machines makes plastics shells for the company’s calculators. (Each calculator requires one shell.) For each of the next two years, Wyoming expects to sell 160,000 calculators. The beginning finished goods inventory of shells at the Casings Plant is 20,000 units. However, the target ending finished goods inventory for each year is 5,000 units.

Each unit (shell) requires 6 ounces of plastic. At the beginning of the year, 60,000 ounces of plastic are in inventory. Management has set a target to have plastic on hand equal to two months’ sales requirements. Sales and production take place evenly throughout the year.

Required

a. Compute the total targeted production of the finished product for the coming year.

b. Compute the required amount of plastic to be purchased for the coming year.

estimate purchases and cash disbursements white products wishes to purchase goods in 691263

Estimate Purchases and Cash Disbursements

White Products wishes to purchase goods in one month for sale in the next. On March 31, the company has 8,000 portable music players in stock, although sales for the next month (April) are estimated to total 8,600 players. Total sales of players are expected to be 7,000 in May and 7,400 in June.

Portable music players are purchased at a wholesale price of $45. The supplier has a financing arrangement by which White Products pays 60 percent of the purchase price in the month when the players are delivered and 40 percent in the following month. White purchased 10,000 players in March.

Required

a. Estimate purchases (in units) for April and May.

b. Estimate the cash required to make purchases in April and May.

estimate cash disbursements ashland corporation a merchandising firm is preparing it 691264

Estimate Cash Disbursements

Ashland Corporation, a merchandising firm, is preparing its cash budget for October. The following information is available concerning its inventories:

Inventories at beginning of October

$135,000

Estimated purchases for October

660,000

Estimated cost of goods sold for October

675,000

Estimated payments in October for purchases in September

165,000

Estimated payments in October for purchases prior to September

30,000

Estimated payments in October for purchases in October

70%

Required

What are the estimated cash disbursements in October?

estimate cash collections duluth company is preparing its cash budget for december t 691265

Estimate Cash Collections

Duluth Company is preparing its cash budget for December. The following information is available concerning its accounts receivable:

Estimated credit sales for December

$300,000

Actual credit sales for November

$225,000

Estimated collections in December for credit sales in December

25%

Estimated collections in December for credit sales in November

70%

Estimated collections in December for credit sales prior to November

$ 24,000

Estimated write offs in December for uncollectible credit sales

$ 12,000

Estimated provision for bad debts in December for credit sales in December

$ 10,500

Required

What is the estimated amount of cash receipts from accounts receivable collections in December?

estimate cash receipts scare 2 b u s2bu specializes in costumes for all occasions th 691267

Estimate Cash Receipts

Scare 2 B U (S2BU) specializes in costumes for all occasions. The average price of each of its costumes is $160. For each occasion, S2BU receives a 20 percent deposit two months before the occasion, 50 percent the month before, and the remainder on the day the costume is delivered. Based on information at hand, managers at S2BU expect to make costumes for the following number of occasions during the coming months:

April

75

May

45

June

30

July

60

August

75

September

165

Required

a. What are the expected revenues for S2BU for each month, April through September? Revenues are recorded in the month of the occasion.

b. What are the expected cash receipts for each month, April through July?

estimate cash receipts varmit b gone is a pest control service that operates in a su 691268

Estimate Cash Receipts

Varmit B Gone is a pest control service that operates in a suburban neighborhood. The company attempts to make service calls at least once a month to all homes that subscribe to its service. It makes more frequent calls during the summer. The number of subscribers also varies with the season. The number of subscribers and the average number of calls to each subscriber for the months of interest follow:

 

Subscribers

Service Calls (per subscriber)

March

600

0.6

April

700

0.9

May

1,400

1.5

June

1,600

2.5

July

1,600

3.0

August

1,500

2.4

The average price charged for a service call is $80. Of the service calls, 30 percent are paid in the month the service is rendered, 60 percent in the month after the service is rendered, and 8 percent in the second month after. The remaining 2 percent is uncollectible.

Required

What are Varmit B Gone’s expected cash receipts for May, June, July, and August?

prepare budgeted financial statements refer to the data in exercise 13 28 varmit b g 691269

Prepare Budgeted Financial Statements

Refer to the data in Exercise 13 28. Varmit B Gone estimates that the number of subscribers in September should fall 10 percent below August levels, and the number of service calls per subscriber should decrease by an estimated 20 percent. The following information is available for costs incurred in August. All costs except depreciation are paid in cash.

Service costs

 

Variable costs

$ 24,000

Maintenance and repair

22,000

Depreciation (fixed)

42,000

Total

$ 88,000

Marketing and administrative costs

Marketing (variable)

$ 14,500

Administrative (fixed)

 55,000

Total

$ 69,500

Total costs

$157,500

Variable service and marketing costs change with volume. Fixed depreciation will remain the same, but fixed administrative costs will increase by 5 percent beginning September 1. Maintenance and repair are provided by contract, which calls for a 1 percent increase in September.

Required

Prepare a budgeted income statement for September.

Exercise 13 28: Estimate Cash Receipts

Varmit B Gone is a pest control service that operates in a suburban neighborhood. The company attempts to make service calls at least once a month to all homes that subscribe to its service. It makes more frequent calls during the summer. The number of subscribers also varies with the season. The number of subscribers and the average number of calls to each subscriber for the months of interest follow:

 

Subscribers

Service Calls (per subscriber)

March

600

0.6

April

700

0.9

May

1,400

1.5

June

1,600

2.5

July

1,600

3.0

August

1,500

2.4

The average price charged for a service call is $80. Of the service calls, 30 percent are paid in the month the service is rendered, 60 percent in the month after the service is rendered, and 8 percent in the second month after. The remaining 2 percent is uncollectible.

incentives and sales forecasts ethical issues the controller of northwest hardware h 691272

Incentives and Sales Forecasts: Ethical Issues

The controller of Northwest Hardware has just received two forecasts for sales in the Montana District for the coming year. Based on an econometric analysis of consumer spending and economic trends, a marketing research firm estimates sales of $1 million for next year. Lloyd Sutter, the district sales manager, estimates sales of $900,000. The controller seeks your advice on the estimate that should be used in developing next year’s budget.

Required

a. What are two possible explanations for the difference between the marketing firm’s estimate and Lloyd’s?

b. Suppose that instead of $900,000, Lloyd estimates $1.1 million in sales. What are two possible explanations for the difference between the marketing firm’s estimate and Lloyd’s?

c. Do any of these explanations suggest unethical behavior by Lloyd?

budget revisions ethical issues elizabeth jablonski is the director of research and 691273

Budget Revisions: Ethical Issues

Elizabeth Jablonski is the director of Research and Development for Galaxy Electronics. Last week, she submitted the following funding request as part of the annual budget process.

Project

Funding Request

1. Portable audio project

$2,500,000

2. Aircraft guidance system

1,800,000

3. Automobile navigation system

3,000,000

4. Miniature DVD player

1,200,000

Total request

$8,500,000

The aircraft guidance system is a project the company has publicly announced and is marketing strongly to manufacturers. The miniature DVD project has received little support in the company but is one of Elizabeth’s favorites. The chief financial officer requests all groups to revise their budgets because of lower than expected sales for the company. Department heads have been asked to submit new budgets that are 10 percent below their original submissions.

Knowing the company’s commitment to the aircraft guidance system, Elizabeth submits the following revision noting that it is 10 percent lower as requested.

Project

Revised Funding Request

1. Portable audio project

$2,450,000

2. Aircraft guidance system

1,000,000

3. Automobile navigation system

3,000,000

4. Miniature DVD player

1,200,000

Total request

$7,650,000

Required

a. What do you think Elizabeth is trying to accomplish with the revised request? Is this ethical?

b. Is Elizabeth likely to be successful in achieving the goal you identified in requirement (a)?

sensitivity analysis sanjana rsquo s sweet shoppe operates on the boardwalk of a new 691274

Sensitivity Analysis

Sanjana’s Sweet Shoppe operates on the boardwalk of a New England coastal town. The store only opens for the summer season and the business is heavily dependent on the weather and the economy in addition to new competition. Sanjana Sweet, the owner, prepares a budget each year after reading long term weather forecasts and estimates of summer tourism. The budget is a first step in planning whether she will need any loans and whether she needs to consider adjustments to store staffing. Based on expertise and experience, she develops the following:

 

Gross Margin

 

 

per Customer

Number of

Scenario

(Price Cost of Goods)

Customers

Good

$5

30,000

Fair

4

20,000

Poor

2

15,000

Sanjana assumes, for simplicity, that the gross margin and the estimated number of customers are independent. Thus, she has nine possible scenarios. In addition to the cost of the products sold, Sanjana estimates staffing costs to be $25,000 plus $2 for every customer in excess of 20,000. The marketing and administrative costs are estimated to be $10,000 plus 3 percent of the gross margin.

Required

Use a spreadsheet to prepare an analysis of the possible operating income for Sanjana similar to that in Exhibit 13.15. What is the range of operating incomes?

Exhibit 13.15 Spreadsheet Analysis of Alternative Budgeting Scenarios

 

A

B

C

D

E

F

G

 

1

Sales

Sales

 

Cost of

Gross

Marketing

Operating

 

2

Price

Quantity

Revenue

Goods Sold

Margin

& Admin.

Profit

 

3

 

 

 

 

 

 

 

 

4

$ 40

150,000

$ 6,000,000

$ 3,600,000

$ 2,400,000

$ 1,483,500

$ 916,500

 

5

$ 45

150,000

$ 6,750,000

$ 3,600,000

$ 3,150,000

$ 1,483,500

$ 1,666,500

 

6

$ 50

150,000

$ 7,500,000

$ 3,600,000

$ 3,900,000

$ 1,483,500

$ 2,416,500

 

7

 

 

 

 

 

 

 

 

8

$ 40

160,000

$ 6,400,000

$ 3,840,000

$ 2,560,000

$ 1,506,000

$ 1,054,000

 

9

$ 45

160,000

$ 7,200,000

$ 3,840,000

$ 3,360,000

$ 1,506,000

$ 1,854,000

 

10

$ 50

160,000

$ 8,000,000

$ 3,840,000

$ 4,160,000

$ 1,506,000

$ 2,654,000

 

11

 

 

 

 

 

 

 

 

12

$ 40

170,000

$ 6,800,000

$ 4,080,000

$ 2,720,000

$ 1,528,500

$ 1,191,500

 

13

$ 45

170,000

$ 7,650,000

$ 4,080,000

$ 3,570,000

$ 1,528,500

$ 2,041,500

 

14

$ 50

170,000

$ 8,500,000

$ 4,080,000

$ 4,420,000

$ 1,528,500

$ 2,891,500

 

15

 

 

 

 

 

 

 

 

 

 

 

 

 

 

 

 

 

Spreadsheets are extremely helpful in preparing budgets, which require considerable sensitivity, or what if, thinking. Spreadsheets help link the various what if scenarios to changes in financial variables and to financial consequences.

For example, the simple spreadsheet in Exhibit 13.15 shows the nine scenarios reflecting the estimated sales prices and sales quantities just described. Each scenario is associated with estimated changes in cost of goods sold and in marketing and administrative costs. (Row 9 presents the budget used in the text ignoring inventories; assume that the other scenarios were worked out by management and presented to us.) Note that the amount shown for operating profits varies considerably between the worst scenario in row 4 and the best scenario in row 14. This analysis alerts management that Santiago Pants will earn less than half the expected profits under the worst scenario. Given the company’s precarious cash position, this analysis will motivate managers at Santiago Pants to develop contingency plans for obtaining cash to meet operating expenses in the event that the more pessimistic scenarios are realized.

These are only a few of the numerous scenarios that management could develop. Furthermore, managers could develop alternative scenarios for any of the budgets that we have discussed. Large companies usually develop complex financial models to deal with the numerous interactions of the budget and these models involve considerably more than just the sensitivity analysis illustrated in Exhibit 13.15.

For example, a decision support system model has been developed to help managers assess the trade offs of different business approaches. Most budgeting activities involve decisions having more than one strategic objective. A company could have two objectives, to maximize income and minimize labor overtime. As the number of objectives increases and they begin to conflict, the decision making process becomes more complex. An interactive, multiple objective, programming model allows managers to deal with often conflicting objectives by using a straightforward set of equations and constraints. The result is a solution that maximizes each objective.

sensitivity analysis bay area limos operates transportation services to bay city air 691275

Sensitivity Analysis

Bay Area Limos operates transportation services to Bay City airport. The price of service is fixed at a flat rate for each trip and most costs of providing the service are fixed for each trip. Betty Smith, the owner, forecasts income by estimating two factors that fluctuate with the economy: the fuel cost associated with the trip and the number of customers who would take trips. Looking at next year, Betty develops the following estimates of contribution margin (price less variable costs, including fuel) for the estimated number of customers. For simplicity, she assumes that the fuel costs (therefore the contribution margin per ride) and the number of customers are independent.

 

Contribution Margin

 

 

per Ride

 

Scenario

(Price = Variable cost)

Number of Customers

Excellent

$50

4,800

Fair

30

3,000

Poor

20

2,000

In addition to the costs of a ride, Betty estimates that other service costs are $45,000 plus $5 for each customer (ride) in excess of 3,000 rides. Annual administrative and marketing costs are estimated to be $20,000 plus 10 percent of the contribution margin.

Required

Use a spreadsheet to prepare an analysis of the possible operating income for Bay Area Limos similar to that in Exhibit 13.5. What is the range of possible operating incomes?

Exhibit 13.5 Manufacturing Overhead Budget

 

A

B

C

D

E

1

SANTIAGO PANTS

 

2

Schedule of Budgeted Manufacturing Overhead

 

3

For the Budget Year Ended December 31

 

4

 

Variable

For Total

 

 

5

 

Overhead

Production

 

 

6

 

per Unit

(Exhibit 13.2)

 

 

7

Units to be produced (from the production budget Exhibit 13.2)

 

170,000

units

 

8

Variable overhead

 

 

 

 

9

Indirect materials and supplies

$ 0.30

$

51,000

 

 

10

Materials handling

0.40

 

68,000

 

 

11

Other indirect labor

0.10

$

17,000

$ 136,000

 

12

 

 

 

 

 

 

13

Fixed manufacturing overhead

 

 

 

 

 

14

Supervisory labor

 

$ 102,000

 

 

15

Maintenance and repairs

 

50,000

 

 

16

Plant administration

 

85,000

 

 

17

Utilities

 

55,000

 

 

18

Depreciation

 

140,000

 

 

19

Insurance

 

30,000

 

 

20

Property taxes

 

60,000

 

 

21

Other

 

22,000

544,000

 

22

Total manufacturing overhead

 

 

$ 680,000

 

23

 

 

 

 

 

 

 

 

 

 

 

prepare budgeted financial statements the following information is available for yea 691276

Prepare Budgeted Financial Statements

The following information is available for year 1 for Dancer Components:

Revenues (300,000 units)

$5,700,000

Manufacturing costs

 

Materials

$ 336,000

Variable cash costs

284,800

Fixed cash costs

655,200

Depreciation (fixed)

1,998,000

Marketing and administrative costs

 

Marketing (variable, cash)

844,800

Marketing depreciation

299,200

Administrative (fixed, cash)

1,018,400

Administrative depreciation

149,600

Total costs

$5,586,000

Operating profits

$ 114,000

All depreciation charges are fixed and are expected to remain the same for year 2. Sales volume is expected to increase by 18 percent, but prices are expected to fall by 5 percent. Material costs per unit are expected to decrease by 8 percent. Other unit variable manufacturing costs are expected to decrease by 2 percent per unit. Fixed manufacturing costs are expected to increase by 5 percent.

Variable marketing costs will change with volume. Administrative cash costs are expected to increase by 10 percent. Inventories are kept at zero. Dancer operates on a cash basis.

Required

Prepare a budgeted income statement for year 2.

prepare budgeted financial statements cameron parts has the following data from year 691277

Prepare Budgeted Financial Statements

Cameron Parts has the following data from year 1 operations, which are to be used for developing year 2 budget estimates:

Revenues (12,500 units)

$ 1,119,000

Manufacturing costs

 

Materials

$ 199,500

Variable cash costs

271,350

Fixed cash costs

108,000

Depreciation (fixed)

133,500

Marketing and administrative costs

 

Marketing (variable, cash)

142,500

Marketing depreciation

33,900

Administrative (fixed, cash)

135,165

Administrative depreciation

2,600

Total costs

$1,036,515

Operating profits

$ 82,485

All depreciation charges are fixed. Old manufacturing equipment with an annual depreciation charge of $14,550 will be replaced in year 2 with new equipment that will incur an annual depreciation charge of $21,000. Sales volume and prices are expected to increase by 12 percent and 6 percent, respectively. On a per unit basis, expectations are that materials costs will increase by 10 percent and variable manufacturing costs will decrease by 4 percent. Fixed manufacturing costs are expected to decrease by 7 percent.

Variable marketing costs will change with volume. Administrative cash costs are expected to increase by 8 percent. Inventories are kept at zero. Cameron operates on a cash basis.

Required

Prepare a budgeted income statement for year 2.

prepare a production budget chander inc manufactures cloth shopping bags the control 691278

Prepare a Production Budget

Chander, Inc., manufactures cloth shopping bags. The controller is preparing a budget for the coming year and asks for your assistance. The following costs and other data apply to bag production:

Direct materials per bag

 

1.0 yard cotton at $6 per yard

 

0.2 yards canvas finish at $20 per yard

 

Direct labor per bag

 

0.5 hour at $32 per hour

 

Overhead per bag

 

Indirect labor

$1.00

Indirect materials

0.30

Power

0.60

Equipment costs

2.50

Building occupancy

 1.60

Total overhead per unit

$6.00

You learn that equipment costs and building occupancy are fixed and are based on a normal production of 300,000 units per year. Other overhead costs are variable. Plant capacity is sufficient to produce 375,000 units per year.

Labor costs per hour are not expected to change during the year. However, the cotton supplier has informed Chander that it will impose a 20 percent price increase at the start of the coming budget period. No other costs are expected to change.

During the coming budget period, Chander expects to sell 270,000 bags. Finished goods inventory is targeted to increase from the current balance of 60,000 units to 105,000 units to prepare for an expected sales increase the year after next. Production will occur evenly throughout the year. Inventory levels for cotton and canvas are expected to remain unchanged throughout the year. There is no work in process inventory.

Required

Prepare a production budget and estimate the materials, labor, and overhead costs for the coming year.

sales expense budget capstone corporation has just received its sales expense report 691280

Sales Expense Budget

Capstone Corporation has just received its sales expense report for January, which follows.

Item

Amount

Sales commissions

$607,500

Sales staff salaries

144,000

Telephone and mailing

72,900

Building lease payment

90,000

Utilities

18,450

Packaging and delivery

123,300

Depreciation

56,250

Marketing consultants

88,650

You have been asked to develop budgeted costs for the coming year. Because this month is typical, you decide to prepare an estimated budget for a typical month in the coming year and you uncover the following additional data:

• Sales volume is expected to increase by 10 percent.

• Sales prices are expected to increase by 5 percent.

• Commissions are based on a percentage of sales revenue.

• Sales staff salaries will increase 4 percent next year regardless of sales volume.

• Building rent is based on a five year lease that expires in three years.

• Telephone and mailing expenses are scheduled to increase by 8 percent even with no change in sales volume. However, these costs are variable with the number of units sold, as are packaging and delivery costs.

• Utilities costs are scheduled to increase by 15 percent regardless of sales volume.

• Depreciation includes furniture and fixtures used by the sales staff. The company has just acquired an additional $85,500 in furniture that will be received at the start of next year and will be depreciated over a 10 year life using the straight line method.

• Marketing consultant expenses were for a special advertising campaign that runs from time to time. During the coming year, these costs are expected to average $157,500 per month.

Required

Prepare a budget for sales expenses for a typical month in the coming year.

comprehensive budget plan brighton inc manufactures kitchen tiles the company recent 691282

Comprehensive Budget Plan

Brighton, Inc., manufactures kitchen tiles. The company recently expanded, and the controller believes that it will need to borrow cash to continue operations. It began negotiating for a one month bank loan of $500,000 starting May 1. The bank would charge interest at the rate of 1 percent per month and require the company to repay interest and principal on May 31. In considering the loan, the bank requested a projected income statement and cash budget for May.

The following information is available:

• The company budgeted sales at 600,000 units per month in April, June, and July and at 450,000 units in May. The selling price is $4 per unit.

• The inventory of finished goods on April 1 was 120,000 units. The finished goods inventory at the end of each month equals 20 percent of sales anticipated for the following month. There is no work in process.

• The inventory of raw materials on April 1 was 57,000 pounds. At the end of each month, the raw materials inventory equals no less than 40 percent of production requirements for the following month. The company purchases materials in quantities of 62,500 pounds per shipment.

• Selling expenses are 10 percent of gross sales. Administrative expenses, which include depreciation of $2,500 per month on office furniture and fixtures, total $165,000 per month.

• The manufacturing budget for tiles, based on normal production of 500,000 units per month, follows:

Materials (1/4 pound per tile, 125,000 pounds, $4 per pound)

$ 500,000

Labor

400,000

Variable overhead

200,000

Fixed overhead (includes depreciation of $200,000)

400,000

Total

$1,500,000

Required

a. Prepare schedules computing inventory budgets by months for

(1) Production in units for April, May, and June.

(2) Raw materials purchases in pounds for April and May.

b. Prepare a projected income statement for May. Cost of goods sold should equal the variable manufacturing cost per unit times the number of units sold plus the total fixed manufacturing cost budgeted for the period. Assume cash discounts of 1 percent and bad debt expense of 0.5 percent.

comprehensive budget plan panther corporation appeared to be experiencing a good yea 691283

Comprehensive Budget Plan

Panther Corporation appeared to be experiencing a good year. Sales in the first quarter were one third ahead of last year, and the sales department predicted that this rate would continue throughout the entire year. The controller asked Janet Nomura, a summer accounting intern, to prepare a draft forecast for the year and to analyze the differences from last year’s results. She based the forecast on actual results obtained in the first quarter plus the expected costs of production to be completed in the remainder of the year. She worked with various department heads (production, sales, and so on) to get the necessary information. The results of these efforts follow:

PANTHER CORPORATION

Expected Account Balances for December 31, Year 2

Cash

$ 4,800

 

Accounts receivable

320,000

 

Inventory (January 1, Year 2)

192,000

 

Plant and equipment

520,000

 

Accumulated depreciation

 

$ 164,000

Accounts payable

 

180,000

Notes payable (due within one year)

 

200,000

Accrued payables

 

93,000

Common stock

 

280,000

Retained earnings

 

432,800

Sales

 

2,400,000

Other income

 

36,000

Manufacturing costs

 

 

Materials

852,000

 

Direct labor

872,000

 

Variable overhead

520,000

 

Depreciation

20,000

 

Other fixed overhead

31,000

 

Marketing

 

 

Commissions

80,000

 

Salaries

64,000

 

Promotion and advertising

180,000

 

Administrative

 

 

Salaries

64,000

 

Travel

10,000

 

Office costs

36,000

 

Income taxes

 

Dividends

20,000

__________

 

$3,785,800

 $3,785,800

Adjustments for the change in inventory and for income taxes have not been made. The scheduled production for this year is 450,000 units, and planned sales volume is 400,000 units. Sales and production volume was 300,000 units last year. The company uses a full absorption costing and FIFO inventory system and is subject to a 40 percent income tax rate. The actual income statement for last year follows:

PANTHER CORPORATION

Statement of Income and Retained Earnings

For the Budget Year Ended December 31, Year 1

Revenues

     

Sales

$1,800,000

$1,860,000

Other income

60,000

   

Expenses

     

Cost of goods sold

     

Materials

$ 528,000

   

Direct labor

540,000

   

Variable overhead

324,000

   

Fixed overhead

48,000

   
 

$1,440,000

 

Beginning inventory

192,000

   
 

$1,632,000

 

Ending inventory

192,000

$1,440,000

Selling

     

Salaries

$ 54,000

   

Commissions

60,000

   

Promotion and advertising

126,000

240,000

 

General and administrative.

     

Salaries

$ 56,000

   

Travel

8,000

   

Office costs

32,000

96,000

 

Income taxes

 

33,600

1,809,600

Operating profit

   

50,400

Beginning retained earnings

   

402,400

Subtotal

   

$ 452,800

Less dividends

   

20,000

Ending retained earnings

   

$ 432,800

Required

Prepared a budgeted income statement and balance sheet.

we can use the analysis in exhibit 11 6 starting with administration panel a proport 691223

We can use the analysis in Exhibit 11.6 , starting with Administration:

Panel A: Proportions

 

 

Service Department

Department’s  Direct Costs

Information Systems

Administration

Hilltop

Mine

Pacific Mine

Total

Administration

$5,000,000

20%

–0–%

50%

30%

100%

Information Systems

800,000

–0–

–0–

20

80

100

 

$5,800,000

 

 

 

 

 

 

Panel B: Step Method Allocation

 

Cost Allocation to

From

Information Systems

Administration

Hilltop Mine

Pacific Mine

Direct costs

$ 800,000

$5,000,000

$ –0–

$ –0–

Administration

1,000,000

(5,000,000)

2,500,000

1,500,000

Information Systems

(1,800,000)

–0–

360,000

1,440,000

Total

$ –0–

$ –0–

$2,860,000

 $2,940,000

Exhibit 11.6 Service Department Cost Allocation Computations: Step Method—Carlyle Coal Company

 

A

B

C

D

E

F

G

 

1

 

Service Department

 

 

 

2

 

Information Systems (S1)

Administration (S2)

 

 

 

3

 

Usage

Percent

Usage

Percent

 

 

 

4

Departments

(hours)

of Total

(employees)

of Total

 

 

 

5

Administration

100,000

50%

–0–

0%

 

 

 

6

Information Systems

–0–

0

2,000

20

 

 

 

7

Hilltop Mine (P1)

20,000

10

5,000

50

 

 

 

8

Pacific Mine (P2)

80,000

40

3,000

30

 

 

 

9

Total usage

200,000

100%

10,000

100%

 

 

 

10

 

 

 

 

 

 

 

 

11

 

 

 

 

 

 

 

 

12

Step Method:

 

Percent Allocable to

 

 

13

 

Department

Information

 

Hilltop

Pacific

 

 

14

 

Direct Cost

Systems

Administration

Mine

Mine

Total

 

15

Service Department

 

 

 

 

 

 

 

16

Information Systems (S1)

$

800,000

0.0%

50.0%

10.0%

40.0%

100.0%

 

17

Administration (S2)

5,000,000

0.0%

0.0%

62.5%

37.5%

100.0%

 

18

 

$ 5,800,000

 

 

 

 

 

 

19

 

 

 

 

 

 

 

 

20

 

 

Amount Allocable to

 

 

21

 

 

Information

 

Hilltop

Pacific

 

 

22

 

 

Systems

Administration

Mine

Mine

 

 

23

From

 

 

 

 

 

 

 

24

Direct department costs

 

$ 800,000

$ 5,000,000

$ –0–

$

–0–

 

 

25

Information Systems (S1)

 

(800,000)

400,000

80,000

320,000

 

 

26

Administration (S2)

 

–0–

(5,400,000)

3,375,000

2,025,000

 

 

27

Total

 

 

$ –0–

$ –0–

$3,455,000

$ 2,345,000

 

 

28

 

 

 

 

 

 

 

 

 

 

 

 

 

 

 

 

 

 

                                         

we can write the equations for the service department costs as follows substituting 691224

We can write the equations for the service department costs as follows:

 width=

Substituting the first equation into the second yields

 width=

Substituting the value of S2 back into the first equation gives

S1 = $100,000 + 0.30 ($40,000)

S1 = $112,000

We now use the values for S1 ($112,000) and S2 ($40,000) to allocate costs simultaneously to all the departments, as in Exhibit 11.9.

Panel A: Proportions

Service Department

Department’s Total Costs

Maintenance

Cafeteria

Finishing

Assembly

Total

Maintenance

$112,000

–0–%

20%

50%

30%

100%

Cafeteria

40,000

30

–0–

20

50

100

 

$152,000

 

 

 

 

 

 

Panel B: Reciprocal Method Allocation

 

Cost Allocation to

From

Maintenance

Cafeteria

Finishing

Assembly

Direct costs

$100,000

$17,600

$ –0–

$ –0–

Maintenance

112,000

22,400

56,000

33,600

Cafeteria

12,000

40,000

8,000

20,000

Total allocated cost

$ –0–

$ –0–

$64,000

$53,600

Production department cost

 

 

1,200,000

640,000

Total cost

   

$1,264,000

$693,600

Exhibit 11.9 Service Department Cost Allocation Computations: Reciprocal Method—Carlyle Coal Company

 

A

B

C

D

E

F

G

1

 

Service Department

 

 

2

 

Information Systems (S1)

Administration (S2)

 

 

3

 

Usage

Percent

Usage

Percent

 

 

4

Departments

(hours)

of Total

(employees)

of Total

 

 

5

Administration

100,000

50%

–0–

0%

 

 

6

Information Systems

–0–

0

2,000

20

 

 

7

Hilltop Mine (P1)

20,000

10

5,000

50

 

 

8

Pacific Mine (P2)

80,000

40

3,000

30

 

 

9

Total usage

200,000

100%

10,000

100%

 

 

10

 

 

 

 

 

 

 

11

 

 

 

 

 

 

 

12

Reciprocal Method:

 

Percent Allocable to

 

13

 

Department

Information

 

Hilltop

Pacific

 

14

 

Total Cost

Systems

Administration

Mine

Mine

Total

15

Service Department

 

 

 

 

 

 

16

Information Systems (S1)

$2,000,000

0.0%

50.0%

10.0%

40.0%

100.0%

17

Administration (S2)

6,000,000

20.0%

0.0%

50.0%

30.0%

100.0%

18

 

$8,000,000

 

 

 

 

 

19

 

 

 

 

 

 

 

20

 

 

Amount Allocable to

 

21

 

 

Information

 

Hilltop

Pacific

 

22

 

 

Systems

Administration

Mine

Mine

 

23

From

 

 

 

 

 

 

24

Direct department costs

 

$ 800,000

$ 5,000,000

$ –0–

$ –0–

 

25

Information Systems (S1)

 

(2,000,000)

1,000,000

200,000

800,000

 

26

Administration (S2)

 

1,200,000

(6,000,000)

3,000,000

1,800,000

 

27

Total

 

$ –0–

$ –0–

$ 3,200,000

$2,600,000

 

28

 

 

 

 

 

 

 

ten tons of sugar beets yields 2 tons of sugar 10 tons x 0 2 and 4 tons of feed 10 t 691225

Ten tons of sugar beets yields 2 tons of sugar (10 tons x 0.2) and 4 tons of feed (10 tons x 0.4)

 

Sugar

Feed

Total

Final sales value

$800

$800

$1,600

Less additional processing costs

–0–

–0–

–0–

Net realizable value at split off point

$800

$800

$1,600

Proportionate share

 

 

 

$800 + 1,600

50%

 

 

$800 + 1,600

 

50%

 

Allocated joint costs

 

 

 

($60 + $40) x 10 tons x 50%

$500

 

 

($60 + $40) x 10 tons x 50%

 

$500

 

consider the proposed dual rate method for global electronics suppose that next year 691227

Consider the proposed dual rate method for Global Electronics. Suppose that next year’s targets are as follows:

Revenue ($000)

 

Latin America division

$ 75,000

Total Global Electronics revenue

$600,000

Fixed corporate costs ($000)

$ 18,000

Variable cost as a percentage of revenue

2.5%

a. Fixed corporate costs are allocated on the basis of relative revenue. What is the target corporate cost for the Latin America division?

b. Suppose that actual Latin America division revenues next year are $80 million, actual corporate revenues are $800 million, and actual corporate costs are $35,000 million. Compare the corporate costs that would have been allocated under the old method at Global Electronics and under the dual rate method.

evaluating management control systems mdash ethical considerations magnolia manufact 691232

Evaluating Management Control Systems—Ethical Considerations

Magnolia Manufacturing makes wing components for large aircraft. Kevin Choi is the production manager, responsible for manufacturing, and Michelle Michaels is the marketing manager. Both managers are paid a flat salary and are eligible for a bonus. The bonus is equal to 1 percent of their base salary for every 10 percent profit that exceeds a target. The maximum bonus is 5 percent of salary. Kevin’s base salary is $150,000 and Michelle’s is $200,000.

The target profit for this year is $5 million. Kevin has read about a new manufacturing technique that would increase annual profit by 20 percent. He is unsure whether to employ the new technique this year, wait, or not employ it at all. Using the new technique will not affect the target.

Required

a. Suppose that profit without using the technique this year will be $5 million. By how much will Kevin’s bonus change if he decides to employ the new technique? By how much will Michelle’s bonus change if Kevin decides to employ the new technique?

b. Suppose that profit without using the technique this year will be $7 million. By how much will Kevin’s bonus change if he decides to employ the new technique? By how much will Michelle’s bonus change if Kevin decides to employ the new technique?

c. Suppose that profit without using the technique this year will be $4 million. By how much will Kevin’s bonus change if he decides to employ the new technique? By how much will Michelle’s bonus change if Kevin decides to employ the new technique?

d. Is it ethical for Kevin to consider the impact of the new technique on his bonus when deciding whether or not to use it? Explain.

e. Assess the management control system used at Magnolia Manufacturing and provide recommendations for changes, if any are required. Be sure to discuss:

• Decision authority

• Performance measures

• Compensation

management control systems and incentives a company that we call ldquo dc rdquo is a 691233

Management Control Systems and Incentives

A company that we call “DC” is a Fortune 100 diversified conglomerate with operations in many industries around the world. Top management focuses on the annual earnings in evaluating the performance of division managers. Each year is a new ballgame for division managers.

The incentive plan includes an annual bonus that ranges from 7 to 40 percent of division managers’ salaries. There is an element of relative performance evaluation in that the target earnings for each year are based on how well companies in the same industry are performing. Once the target is set, it is not changed during the year.

Failing to meet a division’s target has serious consequences for the division manager. First, the manager loses some or all of the potential bonus. Second, a manager who misses a target will find her job in jeopardy. Missing a target two years in a row generally means that the manager will be fi red.

Required

a. What incentives does this plan give to division managers?

b. Is this a good plan? Would you want to be a division manager in this company?

management control systems and incentives a fortune 500 company that we shall call l 691234

Management Control Systems and Incentives

A Fortune 500 company that we shall call “Heavy” is a manufacturer of machinery and engines. This company is headquartered in a small city in the Midwestern region of the United States. This company’s products have a well respected brand name and receive a premium price in the market. The unionized work force is well paid and does quality work.

This company faces challenges from foreign companies that pay lower wages and have more modern and more efficient production equipment. Consequently, it is seeking ways to cut costs without reducing quality.

The company recently introduced a profit sharing arrangement whereby workers receive a share of profits in profitable years. The workers gave up a wage increase to obtain this profit sharing arrangement.

Required

Evaluate the advantages and disadvantages of giving the workers a profit sharing bonus instead of a wage increase.

alternative allocation bases service packages 2 go has two divisions air express and 691235

Alternative Allocation Bases: Service

Packages 2 Go has two divisions, air express and ground service, that share the common costs of the company’s communications network, which are $10,000,000 a year. You have the following information about the two divisions and the common communications network:

 

Calls (thousands)

Time on Network (hours)

Air express

240,000

200,000

Ground service

160,000

800,000

Required

a. What is the communications network cost that is charged to each division if the number of calls is used as the allocation basis?

b. What is the communications network cost to each division using time on network as the allocation basis?

c. The cost of the communications network is necessary regardless of which division uses it. Why is the method of allocation important?

single versus dual rates required refer to data for packages 2 go in exercise 12 21 691236

Single versus Dual Rates

Required

Refer to data for Packages 2 Go in Exercise 12 21. Determine the cost allocation if $5.2 million of the communications network costs are fixed and allocated on the basis of time on network, and the remaining costs, which are variable, are allocated on the basis of the number of calls.

Exercise 12 21: Alternative Allocation Bases: Service

Packages 2 Go has two divisions, air express and ground service, that share the common costs of the company’s communications network, which are $10,000,000 a year. You have the following information about the two divisions and the common communications network:

 

Calls (thousands)

Time on Network (hours)

Air express

240,000

200,000

Ground service

160,000

800,000

Required

a. What is the communications network cost that is charged to each division if the number of calls is used as the allocation basis?

b. What is the communications network cost to each division using time on network as the allocation basis?

c. The cost of the communications network is necessary regardless of which division uses it. Why is the method of allocation important?

single versus dual rates using the data for the consulting firm in exercise 12 23 wh 691238

Single versus Dual Rates

Using the data for the consulting firm in Exercise 12 23, what is the cost allocation if fixed computer costs of $4 million are allocated on the basis of number of consultants and the remaining costs (all variable) are allocated on the basis of the number of gigabytes of storage used by the department?

Exercise 12 23: Single versus Dual Rates: Ethical Considerations

A consulting firm has two departments, Corporate and Government. Computer support is common to both departments. The cost of computer support is $6 million. The following information is given:

 

Gigabytes of Storage

Number of Consultants

Corporate

75,000

150

Government

50,000

69

Required

a. What is the cost charged to each department if the allocation is based on the number of gigabytes of storage?

b. What is the cost charged to each department if number of consultants is the allocation basis?

c. Most of the business in the Corporate Department is priced on a fixed fee basis, and most of the work in the Government Department is priced on a cost plus fixed fee basis. Will this affect the choice of the allocation base? Should it?

single versus dual rates ajax manufacturing repairs aircraft engines the company rsq 691239

Single versus Dual Rates

Ajax Manufacturing repairs aircraft engines. The company’s purchasing department supports its two departments, Defense and Commercial. The Defense division has contracts with the Department of Defense and the Commercial division works primarily with domestic airlines and air freight companies. The cost of the purchasing department is $1 million annually.

Information on the activity of the purchasing department for the last year follows:

 

Number of Purchase Orders

Dollar Amount of Purchases

Defense

5,000

$100,000,000

Commercial

20,000

150,000,000

Required

a. What is the cost charged to each division if Ajax allocates Purchasing department costs based on the number of purchase orders?

b. What is the cost charged to each division if Ajax allocates Purchasing department costs based on the dollar amount of the purchases?

c. Contracts with the Defense Department are on a cost plus fixed fee basis, meaning the price is based on the cost of repairing an engine, including any overhead assigned to the division. Contracts with commercial airlines and air freight companies are almost all fixed price, meaning the price does not depend directly on the cost. Will this affect Ajax’s choice of an allocation base? Should it?

internal controls one of the authors of this book has a favorite sandwich shop where 691242

Internal Controls

One of the authors of this book has a favorite sandwich shop where one person makes the sandwich and another person rings up the sale and takes the customer’s cash. At first, this author thought that having two people involved had something to do with him. After carefully observing the sandwich shop’s operations, he observed that two employees were involved in every sandwich production and sale. The person who made the sandwich did not ring up the sale or take the money from the sale.

Required

a. What type of internal control is provided in this example? Why is the shop manager/owner providing that internal control?

b. Is there an even better internal control?

c. Could the employees get around this internal control?

evaluating management control systems spg company manufactures and sells metal produ 691243

Evaluating Management Control Systems

SPG Company manufactures and sells metal products that are used in many manufacturing operations. The management at SPG believes strongly in decentralized decision making and using performance evaluation and compensation to encourage high performing managers. Marilyn Conner’s is the manager of the manufacturing operations, which produces and transfers the product to the marketing division. Jack Schwartz is the manager of marketing. Marilyn is evaluated on manufacturing cost relative to a budget for good output. Marilyn makes all production decisions. Jack is evaluated on company profit relative to a target. If a manager meets his or her target, they receive a bonus equal to 100 percent of salary.

Information on performance last year follows:

Manufacturing Cost

Company Profit

Target cost per unit

$2.37

Target profit

$10,000,000

Actual cost per unit

2.25

Actual profit

9,232,000

Required

What recommendations would you suggest for changes to the SPG management control system, if any? Discuss the delegation of decision authority, performance evaluation and measurement, and compensation design in your response.

analyze performance report for decentralized organization hall o rsquo fame products 691244

Analyze Performance Report for Decentralized Organization

Hall O’ Fame Products is a nationwide sporting goods manufacturer. The company operates with a widely based manufacturing and distribution system that has led to a highly decentralized management structure. Each division manager is responsible for producing and distributing corporate products in one of eight geographical areas of the country.

Division managers are evaluated using a performance measure that is calculated as the division’s contribution to corporate profits before taxes less a 20 percent investment charge on the division’s investment base. The investment base of each division is the sum of its year end balances of accounts receivable, inventories, and net plant fixed assets (cost less accumulated depreciation). Corporate policies dictate that divisions minimize their investments in receivables and inventories. Investments in fixed plant assets are decisions jointly made by the division and corporate based on proposals made by division plant managers, available corporate funds, and general corporate policy.

James Davenport, division manager for the California sector, prepared the year 2 and preliminary year 3 budgets for his division late in year 1. Final approval of the year 3 budget took place in late year 2 after adjustments for trends and other information developed during year 2. Preliminary work on the year 4 budget also took place at that time. In early October of year 3, Davenport asked the division controller to prepare a report that presents performance for the first nine months of year 3. The report follows:

 

Annual

Nine Month

Nine Month

Annual

Actual

 

Budget

Budgeta

Actual

Budget

Results

Sales

$19,600

$14,700

$15,400

$17,500

$17,010

Divisional costs and expenses

 

 

 

 

 

Direct materials and labor

$ 7,448

$ 5,586

$ 6,965

$ 6,300

$ 6,230

Supplies

308

231

245

245

301

Maintenance and repairs

1,400

1,050

420

1,225

1,120

Plant depreciation

840

630

630

770

770

Administration

840

630

630

630

700

 

 

 

 

 

 

Total divisional costs and expenses

$10,836

$8,127

$8,890

$9,170

$9,121

Divisional margin

$ 8,764

$ 6,573

$ 6,510

$ 8,330

$ 7,889

Allocated corporate fixed costs

2,520

1,890

1,680

2,380

2,240

Divisional profits

$ 6,244

$4,683

$4,830

$5,950

$5,649

 

Budgeted

Balance

12/31/Year 3

Budgeted

Balance

9/30/Year 3

Actual

Balance

9/30/Year 3

Budgeted

Balance

12/31/Year 2

Actual

Balance

12/31/Year 2

 

 

Divisional investment

 

 

 

 

 

Accounts receivable

$ 1,960

$ 2,030

$ 1,750

$ 1,750

$ 1,750

Inventories

3,500

3,500

4,550

3,150

3,325

Plant fixed assets (net)

9,240

9,450

7,700

8,050

7,700

Total

$14,700

$14,980

$14,000

$12,950

$12,775

Required

a. Evaluate the performance of James Davenport for the nine months ending September 30, Year 3. Support your evaluation with pertinent facts from the problem.

b. Identify the features of Hall O’ Fame’s division performance measurement reporting and evaluation system that need to be revised if it is to effectively reflect the responsibilities of the divisional managers.

divisional performance measurement behavioral issues paulista corporation rsquo s di 691245

Divisional Performance Measurement: Behavioral Issues

Paulista Corporation’s division managers have been expressing growing dissatisfaction with the methods the organization uses to measure division performance. Division operations are evaluated every quarter by comparing them with a budget prepared during the prior year. Division managers claim that many factors that are completely out of their control are included in this comparison, resulting in an unfair and misleading performance evaluation.

The managers have been particularly critical of the process used to establish budgets. The annual budget, stated by quarters, is prepared six months prior to the beginning of the operating year. Pressure by top management to reflect increased earnings has often caused divisional managers to overstate revenues and/or understate expenses. In addition, after the budget is established, divisions must “live with it.” Frequently, the budgets that top management has supplied to the divisions have not recognized external factors such as the state of the economy, changes in consumer preferences, and actions of competitors. The credibility of the performance review is damaged when the budget cannot be adjusted to incorporate these changes.

Recognizing these problems, top management has agreed to establish a committee to review the situation and to make recommendations for a new performance evaluation system. The committee consists of each division manager, the corporate controller, and the executive vice president. At the first meeting, one division manager outlined an achievement of objectives system (AOS). This performance evaluation system evaluates division managers according to three criteria:

• Doing better than last year . Various measures are compared to the same measures for the prior year.

• Planning realistically . Actual performance for the current year is compared to realistic plans and/or goals.

• Managing current assets . Various measures are used to evaluate division management’s achievements and reactions to changing business and economic conditions.

One division manager believes that this system would overcome many of the inconsistencies of the current system because divisions could be evaluated from three different viewpoints. In addition, managers would have the opportunity to show how they would react and account for changes in uncontrollable external factors.

Another manager cautions that the success of a new performance evaluation system will be limited unless it has top management’s complete support.

Required

a. Explain whether the proposed AOS would be an improvement over the evaluation system of division performance currently used by Paulista Corporation.

b. Develop specific performance measures for each of the three criteria in the proposed AOS that could be used to evaluate division managers.

c. Discuss the motivational and behavioral aspects of the proposed performance system. Also recommend specific programs that could be instituted to promote morale and give incentives to divisional management.

cost allocation for travel reimbursement your company has a travel policy that reimb 691247

Cost Allocation for Travel Reimbursement

Your company has a travel policy that reimburses employees for the “ordinary and necessary” costs of business travel. Employees often mix a business trip with pleasure by either extending the time at the destination or traveling from the business destination to a nearby resort or other personal destination. When this happens, an allocation must be made between the business and personal portions of the trip. However, the travel policy is unclear on the allocation method to follow.

Consider this example. An employee obtained a business class ticket for $9,537 and traveled the following itinerary:

 

 

 

One Way

 

 

 

 

Regular

 

From

To

Miles

Fare

Purpose

Chicago

Paris

4,140

$3,650a

Business

Paris

Rio de Janeiro

5,700

4,320

Personal

Rio de Janeiro

Chicago

5,300

3,250

Return

a A restricted round trip fare of $4,900 was available on these dates.

Required

a. Compute the business portion of the airfare and state the basis for the indicated allocation that is appropriate according to each of the following independent scenarios:

(1) Based on the maximum reimbursement for the employee.

(2) Based on the minimum cost to the company.

b. Write a short report to management explaining the method that you think should be used and why. You do not have to restrict your recommendation to either of the methods in requirement (a).

pepsi and old bottles when the fraud at pepsico occurred the company had five somewh 691250

Pepsi and Old Bottles

When the fraud at PepsiCo occurred, the company had five somewhat diverse groups of divisions: food products, such as Frito Lay, Inc.; transportation, such as north American Van Lines, Inc.; sporting goods, such as Wilson Sporting Goods Co.; food service, such as Pizza Hut, Inc., and Taco Bell; and its primary business, beverages. The beverage group included United Beverages International (UBI), a company that bottled soft drinks in 11 foreign countries.

The fraud was committed by employees in the UBI subsidiary in two countries: Mexico and the Philippines. These employees used numerous techniques to falsify income, including keeping inventories of broken or unusable bottles on the books, failing to write off uncollectible accounts receivable, writing up the value of bottle inventory above cost, and falsifying expense accounts. These activities required extensive collusion. In the Philippines, employees kept more than $45 million of obsolete bottles on the books to satisfy the country’s debt to equity requirements. (Writing off the bottle inventory would have reduced both assets and equity, thus creating a problem with the country’s debt to equity requirements.)

PepsiCo’s net income was overstated by a total of approximately $92 million over a five year period from these fraudulent activities. At its highest, the overstatement was $36 million, which was 12 percent of PepsiCo’s net income from all five of its main groups.

Consistent with its management style of granting considerable autonomy to division managers, PepsiCo’s Internal Audit Department acted less like a watchdog and more like a management consultant. For example, at PepsiCo, the Internal Audit Department did not conduct surprise audits but notified division managers in advance of its visits to ensure that key employees were present.

Despite their role as consultants, PepsiCo’s internal auditors uncovered the fraudulent activities at PepsiCo’s Mexico and Philippines operations. After discovering the fraud, the Internal Audit Department at PepsiCo became less consulting oriented and started conducting surprise audits. Some people in the company believe that the reorientation of internal audit away from consulting was a major negative repercussion of the fraud.

During the period in which the fraud was committed, PepsiCo portrayed itself as an aggressive, high performance, results oriented company. Prior to the fraud, PepsiCo prided itself on the company’s morale and sense of community. Its policy of decentralization supported the notion that the company had aggressive, hard working, and trustworthy employees. After the fraud was discovered, PepsiCo’s top management was distressed about the conspiracy among those trusted employees who committed the fraud.

In all, the Securities and Exchange Commission fi led formal complaints against 12 employees in the two countries. PepsiCo terminated the people involved, as well as the U.S. based manager of the bottling unit of UBI.

Required

What factors contributed to the fraud at PepsiCo?

business environment performance measures compensation and ethics in the late 1980s 691251

Business Environment, Performance Measures, Compensation, and Ethics

In the late 1980s, General Electric Company (GE), whose CEO at the time was Jack Welch, acquired Kidder Peabody, an investment banking firm founded in 1824. In 1991, Kidder hired a bond trader named Joseph Jett. Jett’s job was trading STRIPS, which are securities linked to U.S. Treasury bonds.

The trades work as follows. Assume you own a 20 year Treasury bond with a face value of $1,000 and an interest rate of 12 percent, payable semiannually. This bond entitles you to 40 payments (20 years x 2 payments per year) of $60 (= $1,000 x 12% x 1/2). For various reasons, some companies and individuals want the payment stream to follow a different pattern. It is possible to convert the single bond described above into 41 separate zero coupon bonds. (A zero coupon bond is one without an explicit interest rate and no payments before maturity.) The resulting bonds are called STRIPS (Separate Trading of Interest and Principal of Securities). The reverse transaction––converting separate bonds into a coupon bond—is referred to as a RECON, or reconstitution of the security.

This transaction has been compared to going to the bank and changing a dollar bill for four quarters. This transaction was done with the Federal Reserve Bank (Fed). Kidder made money on the business through fees and trading profits associated with the inventory of bonds it kept for transactions. As you might expect, there should be no profit in the transaction with the Fed.

Although at first he struggled in his job, Jett was soon generating enormous profits and earning large bonuses. He was able to do this because of an error in the internal Kidder accounting system that recorded the transaction improperly. Because the error would eventually correct itself (as the interest payment date approached), Jett was forced to trade larger and larger volumes. At the time this was discovered, approximately 95 percent of Jett’s trades were with the Fed.

Jett earned a bonus of $2 million in 1992 and $9 million in 1993, in addition to being named Kidder’s “Employee of the Year.” In 1994, Jett was generating in one month the profit he earned for the entire year in 1992 and Kidder executives began to investigate. Jett was fi red in April 1994 and GE was forced to take a $350 million pretax charge against earnings.

Required

a. Suppose you were Jett and you realized the accounting system used to record your performance was fl awed. What steps would you take?

b. Suppose that you are unable to convince your superiors that the accounting system is fl awed (in other words, that it encourages individual actions not in the best interests of the company). What should you do?

c. In his autobiography, Jack: Straight from the Gut , Jack Welch discusses the Kidder case and the differences between the GE and Kidder environments with respect to bonuses: Frankly, the bonus numbers knocked us off our pins when we saw them. At the time, GE’s total bonus pool was just under $100 million for the year for a company making $4 billion in profit. Kidder’s bonus pool was actually higher––at $140 million––for a company that was earning only one twentieth of our income. How might the different business environments and industries lead to such a large difference in the amount of contingent based (bonus) compensation?

d. In the same autobiography, Welch compares the cultures of the two companies: The response of our business leaders to the crisis [the write down of $350 million] was typical of the GE culture. Even though the books had closed on the quarter, many immediately offered to pitch in to cover the Kidder gap. Some said they could find an extra $10 million, $20 million, and even $30 million from their businesses to offset the surprise. Though it was too late, their willingness to help was a dramatic contrast to the excuses I had been hearing from the Kidder people.

(1) What does Welch mean when he says that GE’s business leaders offered to help by finding an extra $10 million, $20 million, and even $30 million to offset the surprise?

(2) What would be alternative uses of the extra $10 (or $20 or $30) million in those businesses?

(3) Would such help be ethical?

a the decision is delegated because the real estate agent has superior local knowled 691252

a. The decision is delegated because the real estate agent has superior local knowledge about what is available on the market. If the person is looking for housing in a new city, the real estate agent also has better local knowledge about neighborhoods and other factors. This information is costly for the new person to obtain.

b. The person looking for housing is the principal. The real estate agent is, appropriately enough, the agent.

c. The agent might be paid a commission based on the rental rate or sales price, so he or she has an incentive to show the most expensive housing he or she thinks the individual can afford. Of course, this incentive is offset somewhat by the agent’s desire for repeat business.

a target fixed corporate costs 000 18 000 target variable cost as a percentage of re 691253

a.

Target fixed corporate costs ($000)

$18,000

Target variable cost as a percentage of revenue

 

(2.5% x $600,000)

15,000

Target total corporate cost

$33,000

Target Latin America division revenues as a percentage of

 

total corporate revenues (= $75 million ÷ $600 million)

x 12.5%

Target allocation of corporate cost to Latin America division

$ 4,125

b.

 

Old Method

Actual revenue as percentage

 

 

of total ($000)

($80 ÷ $800)

10.0%

Corporate cost

 

$35,000

Allocation to Latin America division

 

 

(10% x $35,000)

 

$ 3,500

 

 

Dual Rate

 

Target revenue as a percentage of total

($75 ÷ $600)

12.5%

Target fixed corporate cost ($000)

$18,000

 

Allocated fixed costs

12.5% x $18,000

$2,250

Actual division revenue

$80,000

 

Allocated variable cost

2.5% x $80,000

2,000

Total

 

$4,250

net realizable value method with by products grand company manufactures products alp 691199

Net Realizable Value Method with By Products

Grand Company manufactures products Alpha and Beta from a joint process, which also yields a by product, Gamma. Grand accounts for the revenues from its by product sales as other income. Additional information follows:

 

Alpha

Beta

Gamma

Total

Units produced

45,000

27,000

18,000

90,000

Allocated joint costs

?

?

?

$280,800

Sales value at split off

$300,000

$240,000

$60,000

$600,000

Required

Assuming that joint product costs are allocated using the net realizable value at split off approach, what was the joint cost allocated to product Beta?

net realizable value method douglas manufactures four grades of lubricant w 10 w 20 691200

Net Realizable Value Method

Douglas manufactures four grades of lubricant, W 10, W 20, W 30, and W 40, from a joint process. Additional information follows:

 

 

 

If Processed Further

 

Units

Sales Value

Additional

Sales

Product

Produced

at Split Off

Costs

Values

W 10

35,000

$210,000

$22,500

$270,000

W 20

25,000

180,000

18,000

210,000

W 30

20,000

120,000

12,000

150,000

W 40

20,000

90,000

7,500

90,000

 

100,000

$600,000

$60,000

$720,000

Required

Assuming that total joint costs of $240,000 were allocated using the sales value at split off (net realizable value method), what joint costs were allocated to each product?

physical quantities method the following questions relate to kyle company which manu 691201

Physical Quantities Method

The following questions relate to Kyle Company, which manufactures products KA, KB, and KC from a joint process. Joint product costs were $126,000. Additional information follows:

 

 

 

If Processed Further

 

Units

Sales Value

Sales

Additional

Product

Produced

at Split Off

Values

Costs

KA

56,000

$160,000

$220,000

$36,000

KB

40,000

140,000

180,000

28,000

KC

16,000

100,000

160,000

20,000

Required

a. Assuming that joint product costs are allocated using the physical quantities (units produced) method, what was the total cost of product KA (including $36,000 if processed further)?

b. Assuming that joint product costs are allocated using the sales value at split off (net realizable value method), what was the total cost of product KB (including the $28,000 if processed further)?

step method with three service departments model inc produces model automobiles made 691203

Step Method with Three Service Departments

Model, Inc., produces model automobiles made from metal. It operates two production departments, Molding and Painting, and has three service departments, Administration, Accounting, and Maintenance. The accumulated costs in the three service departments were $270,000, $375,000, and $198,000, respectively. Management is concerned that the costs of its service departments are getting too high. In particular, managers would like to keep the costs of service departments under $3.50 per unit on average. You have been asked to allocate service department costs to the two production departments and compute the unit costs.

The company decided that Administration costs should be allocated on the basis of square footage used by each production and service department. Accounting costs are allocated on the basis of number of employees. Maintenance costs are allocated on the basis of the dollar value of the equipment in each department. The use of each base by all departments during the current period follows:

 

Used by

Allocation Base

Administration

Accounting

Maintenance

Molding

Painting

Building area

10,000

30,000

20,000

360,000

90,000

Employees

18

10

12

70

100

Equipment value

(in thousands)

 

 

 

 

 

$6.00

$120.00

$17.50

$312.00

$162.00

Direct costs of the Molding Department included $237,500 in direct materials, $337,500 in direct labor, and $112,500 in overhead. The Painting Department’s direct costs consisted of $210,000 in direct materials, $200,000 in direct labor, and $75,000 in overhead.

Required

a. Using the step method, determine the allocated costs and the total costs in each of the two producing departments. Ignore self usage (for example, ignore work done by Administration for itself). Rank orders the allocation as follows: (1) Maintenance, (2) Accounting, and (3) Administration.

b. Assume that 100,000 units were processed through these two departments. What is the unit cost for the sum of direct materials, direct labor, and overhead (1) for Molding, (2) for Painting, and (3) in total?

c. Compute the cost per unit for the service department costs allocated to the production departments. Did the company meet management’s standards of keeping service department costs below $3.50 per unit?

comparison of allocation methods gb service corporation has two service departments 691204

Comparison of Allocation Methods

GB Service Corporation has two service departments, Administration and Accounting, and two operating departments, East and West. Administration costs are allocated on the basis of employees, and Accounting costs are allocated on the basis of number of transactions. A summary of GB operations follows:

 

Administration

Accounting

East

West

Employees

25

15

60

Transactions

50,000

10,000

40,000

Department direct costs

$60,000

$24,000

$156,000

$600,000

Required

a. Allocate the cost of the service departments to the operating departments using the direct method.

b. Allocate the cost of the service departments to the operating departments using the step method. Start with Administration.

c. Allocate the cost of the service departments to the operating departments using the reciprocal method.

d. Comment on the results.

solve for unknowns pat rsquo s print shops has a warehouse that supplies paper and o 691205

Solve for Unknowns

Pat’s Print Shops has a warehouse that supplies paper and other supplies to its store locations. It has two service departments, Information Services (S1) and Operation Support (S2), and two operating departments, Printing (P1) and Customer Service (P2). As an internal auditor, you are checking the company’s procedures for cost allocation. You find the following cost allocation results for September:

Costs allocated to P1

$32,000 from S1

? from S2

Costs allocated to P2

? from S1

$18,000 from S2

Total costs for the two service departments are $80,000. Operation Support’s services are provided as follows:

• 50 percent to Printing.

• 30 percent to Customer Service.

The direct method of allocating costs is used.

Required

a. What are the total service department costs (S1 + S2) allocated to P2?

b. Complete the following:

 

To

From

Printing

Customer Service

Information Services

$32,000

?

Operation Support

?

$18,000

c. What proportion of S1’s costs were allocated to P1 and P2?

cost allocation step method with analysis and decision making steamco corporation is 691206

Cost Allocation: Step Method with Analysis and Decision Making

Steamco Corporation is reviewing its operations to see what additional energy saving projects it might adopt. The company’s manufacturing plant generates its own electricity using a process capturing steam from its production processes. A summary of the use of service departments by other service departments as well as by the two producing departments at the plant follows:

 

Services Used by

Service Department

Steam Generation

Fixed Costs

Variable Costs

Equipment

Maintenance

Alpha

Beta

Steam Generation

–0–

–0–

0.40

–0–

0.10

0.50

Electric Generating

 

 

 

 

 

 

Fixed costs

0.10

–0–

–0–

0.10

0.30

0.50

Variable costs

0.10

–0–

–0–

0.05

0.55

0.30

Equipment Maintenance

0.20

0.10

0.05

–0–

0.50

0.15

Direct costs (in thousands) in the various departments follow:

Department

Direct Cost

Steam Generation (S1)

$ 210

Electric Generating:

 

Fixed costs (S2)

90

Variable costs (S3)

240

Equipment Maintenance (S4)

144

Production

 

Alpha (P1)

1,800

Beta (P2)

1,320

Steamco currently allocates costs of service departments to production departments using the step method. The local power company indicates that it would charge $480,000 per year for the electricity that Steamco now generates internally. Management rejected switching to the power company on the grounds that its rates would cost more than the $330,000 ($90,000 + $240,000) cost of the present, company owned, system.

Required

a. What costs of electric service did management use to prepare the basis for its decision to continue generating power internally?

b. Prepare for management an analysis of the costs of the company’s own electric generating operations. (Use the step method.) The rank order of allocation is (1) S1, (2) S4, (3) S2, and (4) S3.

c. Add a section to your analysis to management that you prepared for requirement (b) to indicate whether your answer there would change if the company could realize $174,000 per year from the sale of the steam now used for electric generating. (Assume no selling costs.)

cost allocation step and reciprocal methods manzano bank has two operating departmen 691207

Cost Allocation: Step and Reciprocal Methods

Manzano Bank has two operating departments (Branches and Electronic) and three service departments (Processing, Administration, and Maintenance). During November, the following costs and service department usage ratios were recorded:

Supplying

Using Department

Department

Processing

Administration

Maintenance

Branches

Electronic

Processing

–0–

50%

–0–

10%

40%

Administration

–0–

–0–

–0–

60%

40%

Maintenance

10%

20%

–0–

20%

50%

Direct cost

$120,000

$750,000

$330,000

$6,000,000

$2,250,000

Required

a. Allocate the service department costs to the two operating departments using the reciprocal method.

b. Now allocate the service department costs to the two operating departments using the step method, allocating maintenance costs first, followed by processing, and then administration. How does your answer differ from what you obtained in (a)? Why?

cost allocation step and reciprocal methods farmington components has two production 691208

Cost Allocation: Step and Reciprocal Methods

Farmington Components has two production departments (Fabrication and Assembly) and three service departments (Engineering, Administration, and Maintenance). During July, the following costs and service department usage ratios were recorded:

Supplying

 

Using Department

 

 

 

 

 

 

Department

Engineering

Administration

Maintenance

Fabrication

Assembly

Engineering

–0–

50%

–0–

10%

40%

Administration

20%

–0–

10%

50%

20%

Maintenance

–0–

20%

–0–

30%

50%

Direct cost

$200,000

$950,000

$250,000

$1,750,000

$500,000

Required

Allocate the service department costs to the two operating departments using the reciprocal method.

allocate service department costs direct and step methods not a mega bank has three 691209

Allocate Service Department Costs: Direct and Step Methods

Not A Mega Bank has three service departments (Administration, Communications, and Facilities), and two production departments (Deposits and Loans). A summary of costs and other data for each department prior to allocation of service department costs for the year ended December 31 follows:

 

Administration

Communications

Facilities

Deposits

Loans

Direct costs

$160,000

$240,000

$203,200

$6,730,000

$4,850,000

Employee hours

15,500

21,000

13,500

281,250

218,750

Number of

 

 

 

 

 

employees

6

10

4

140

100

Square footage

 

 

 

 

 

occupied

3,500

9,600

4,000

176,000

144,000

The costs of the service departments are allocated on the following bases: Administration, employee hours; Communications, number of employees; and Facilities, square footage occupied.

Required

Round all final calculations to the nearest dollar.

a. Assume that Not A Mega Bank elects to distribute service department costs directly to production departments using the direct method. What amount of Communications Department costs is allocated to the Deposits Department?

b. Assume the same method of allocation as in requirement (a). What amount of Administration Department costs is allocated to the Loans Department?

c. Assuming that Not A Mega Bank elects to distribute service department costs to other departments using the step method (starting with Facilities and then Communications), what amount of Facilities Department costs is allocated to the Communications Department?

d. Assume the same method of allocation as in requirement (c). What amount of Communication Department costs is allocated to Facilities?

allocate service department costs ethical issues fifth street publishing fsp was fou 691210

Allocate Service Department Costs: Ethical Issues

Fifth Street Publishing (FSP) was founded many years ago as a printing cooperative offering printing services to members. Most members were charitable and religious organizations. Ten years ago, FSP became a for profit corporation, although it retained its commitment to the original member groups. It has two production departments, Member and Commercial. The Member Department handles printing jobs for nonprofit groups and Commercial serves the remaining customers. FSP is organized this way to facilitate its billing: FSP charges nonprofit t customers a price equal to the full cost of job, including allocated overhead; commercial jobs are priced based on what the market will bear. FSP has two service departments, Accounting and Computer Services (CS). Accounting costs are allocated based on number of employees and CS costs are allocated based on computer time (hours). Selected percentage use data follow:

 

Using Department

Supplying Department

Accounting

Computer Services

Member

Commercial

 

 

 

 

 

Accounting (employees)

0%

20%

40%

40%

Computer Services (hours)

50

–0–

10

40

Direct cost

$16,000

$61,600

$192,000

$576,000

Required

a. Suppose FSP allocated service department costs using the direct method. What is the amount of service department costs that will be allocated to each of the production departments?

b. Suppose that, after reviewing the results, the controller tells the cost accountant to change the percentage of computer time so 20 percent is shown as used by the Member Department and 30 percent is shown as used by the Commercial Department. Would this be ethical?

c. Suppose the controller tells the cost accountant to use employee wages instead of number of employees to allocate accounting department cost. Employees in the Commercial Department earn lower salaries. Would this be ethical?

d. Although FSP’s policy is to allocate service department costs using the direct method, the controller asks the cost accountant to allocate the costs using the step method, allocating Computer Service Department costs first. What is the amount of service department costs that will be allocated to each of the production departments?

e. The controller next asks the cost accountant to allocate the costs using the reciprocal method. What is the amount of service department costs that will be allocated to each of the production departments?

f. Suppose the controller tells the cost accountant to use each month the method that allocates the highest cost to the Member Department. Would this be ethical?

reciprocal cost allocation mdash outsourcing a service department refer to the facts 691212

Reciprocal Cost Allocation—Outsourcing a Service Department

Refer to the facts in Problem 11 46. The cost accountant at Manzano Bank estimates that the cost structures in their departments are as follows:

 

Processing

Administration

Maintenance

Branches

Electronic

Variable costs

$ 80,000

$240,000

$180,000

$2,500,000

$1,200,000

Fixed costs

40,000

510,000

150,000

3,500,000

1,050,000

Total costs

$120,000

$750,000

$330,000

$6,000,000

$2,250,000

Avoidable fixed costs

$ 10,000

$403,000

$120,000

$2,112,500

$ 910,000

Required

a. If Manzano outsources the Processing Department, what is the maximum they can pay an outside vendor without increasing total costs?

b. If Manzano outsources the Administration Department, what is the maximum they can pay an outside vendor without increasing total costs?

c. If Manzano outsources the Maintenance Department, what is the maximum they can pay an outside vendor without increasing total costs?

Problem 11 46: Cost Allocation: Step and Reciprocal Methods

Manzano Bank has two operating departments (Branches and Electronic) and three service departments (Processing, Administration, and Maintenance). During November, the following costs and service department usage ratios were recorded:

Supplying

Using Department

Department

Processing

Administration

Maintenance

Branches

Electronic

Processing

–0–

50%

–0–

10%

40%

Administration

–0–

–0–

–0–

60%

40%

Maintenance

10%

20%

–0–

20%

50%

Direct cost

$120,000

$750,000

$330,000

$6,000,000

$2,250,000

Required

a. Allocate the service department costs to the two operating departments using the reciprocal method.

b. Now allocate the service department costs to the two operating departments using the step method, allocating maintenance costs first, followed by processing, and then administration. How does your answer differ from what you obtained in (a)? Why?

reciprocal cost allocation mdash outsourcing a service department refer to the facts 691213

Reciprocal Cost Allocation—Outsourcing a Service Department

Refer to the facts in Problem 11 47. Cost records obtained from Farmington Components show the following cost structure in their departments:

 

Engineering

Administration

Maintenance

Fabrication

Assembly

Variable costs

$100,000

$320,000

$130,000

$700,000

$380,000

Fixed costs

100,000

630,000

120,000

1,050,000

120,000

Total costs

$200,000

$950,000

$250,000

$1,750,000

$500,000

Avoidable

 width=

 width=

 width=

 width=

 

fixed costs

$ 70,000

$400,000

$ 70,000

$ 912,500

$ 75,000

Required

a. If Farmington outsources the Engineering Department, what is the maximum they can pay an outside vendor without increasing total costs?

b. If Farmington outsources the Administration Department, what is the maximum they can pay an outside vendor without increasing total costs?

c. If Farmington outsources the Maintenance Department, what is the maximum they can pay an outside vendor without increasing total costs?

Problem 11 47:Cost Allocation: Step and Reciprocal Methods

Farmington Components has two production departments (Fabrication and Assembly) and three service departments (Engineering, Administration, and Maintenance). During July, the following costs and service department usage ratios were recorded:

Supplying

 

Using Department

 

 

 

 

 

 

Department

Engineering

Administration

Maintenance

Fabrication

Assembly

Engineering

–0–

50%

–0–

10%

40%

Administration

20%

–0–

10%

50%

20%

Maintenance

–0–

20%

–0–

30%

50%

Direct cost

$200,000

$950,000

$250,000

$1,750,000

$500,000

Required

Allocate the service department costs to the two operating departments using the reciprocal method.

net realizable value of joint products toledo chemical company buys a 123 for 2 40 a 691214

Net Realizable Value of Joint Products

Toledo Chemical Company buys A 123 for $2.40 a gallon. At the end of distilling in Department 1, A 123 splits off into three products: B 1, B 2, and B 3. Toledo sells B 1 at the split off point, with no further processing; it processes B 2 and B 3 further before they can be sold. B 2 is fused in Department 2, and B 3 is solidified in Department 3. Following is a summary of costs and other related data for the year ended June 30.

Department

(1) Distilling

(2) Fusing

(3) Solidifying

Cost of A 123

$288,000

–0–

–0–

Direct labor

72,000

$135,000

$195,000

Manufacturing overhead

60,000

63,000

162,000

 

Products

B 1

B 2

B 3

Gallons sold

45,000

90,000

135,000

Gallons on hand at year end

30,000

–0–

45,000

Sales

$90,000

$288,000

$425,250

Toledo had no beginning inventories on hand at July 1 and no A 123 on hand at the end of the year on June 30. All gallons on hand on June 30 were complete as to processing. Toledo uses the net realizable value method to allocate joint costs.

Required

Compute the following:

a. The net realizable value of B 1 for the year ended June 30.

b. The joint costs for the year ended June 30 to be allocated.

c. The cost of B 2 sold for the year ended June 30.

d. The value of the ending inventory for B 1.

finding missing data net realizable value athens inc manufactures argon xon and zeon 691216

Finding Missing Data: Net Realizable Value

Athens, Inc., manufactures argon, xon, and zeon from a joint process. Each gas can be liquefied and sold for a higher price. Data on the process are as follows:

Product

Argon

Xon

Zeon

Total

Units produced

16,000

8,000

4,000

28,000

Joint costs

$ 60,000a

(b)

(a)

$120,000

Sales value at split off

(c)

(d)

$30,000

200,000

Additional costs to liquefy

14,000

$10,000

6,000

30,000

Sales value if liquefied

140,000

60,000

40,000

240,000

Required

Determine the value for each lettered item.

joint costing in a process costing context estimated net realizable value method wes 691217

Joint Costing in a Process Costing Context: Estimated Net Realizable Value Method

West Coast Designs produces three products: super, deluxe, and generic. Super and deluxe are its main products; generic is a by product of super. Information on the past month’s production processes follows:

• In Department A, 330,000 units of the raw material X 1 are processed at a total cost of $783,000. After processing in Department A, 60 percent of the units are transferred to Department B, and 40 percent of the units (now unprocessed deluxe) are transferred to Department C.

• In Department B, the materials received from Department A are processed at an additional cost of $228,000. Seventy percent of the units become super and are transferred to Department D. The remaining 30 percent emerge as generic and are sold at $4.20 per unit. The additional processing costs to make generic salable are $48,600.

• In Department C, deluxe is processed at an additional cost of $990,000. A normal loss of 10 percent of the units of good output of deluxe occurs in this department. The remaining good output is then sold for $24 per unit.

• In Department D, super is processed at an additional cost of $98,880. After this processing, super can be sold for $10 per unit.

Required

Prepare a schedule showing the allocation of the $783,000 joint cost to super and deluxe using the estimated net realizable value approach. Revenue from the sale of by products should be credited to the manufacturing costs of the related main product (method 1 in the text).

find maximum input price estimated net realizable value method ticon corporation rsq 691218

Find Maximum Input Price: Estimated Net Realizable Value Method

Ticon Corporation’s manufacturing operation produces two joint products. Product delta sells for $24 per unit at the split off point. After an additional $225,000 of processing costs are incurred, product omega sells for $81 per unit. In a typical month, 76,000 units are processed; 60,000 units become product delta and 16,000 units become product omega.

The joint process has only variable costs. In a typical month, the conversion costs of the joint products amount to $421,000. Materials prices are volatile, and if prices are too high, the company stops production.

Required

Management has asked you to determine the maximum price that the company should pay for the materials.

a. Calculate the maximum price that Ticon should pay for the materials.

b. Write a brief memo to management explaining how you arrived at your answer in requirement (a).

effect of by product versus joint cost accounting black corporation processes xyrex 691219

Effect of By Product versus Joint Cost Accounting

Black Corporation processes xyrex into three outputs: Xy 1, Xy 2, and Xy 3. Xy 1 accounts for 60 percent of the net realizable value at the split off point, Xy 2 accounts for 30 percent, and Xy 3 accounts for the balance. The joint costs total $365,500. If Xy 3 is accounted for as a by product, its $37,600 net realizable value at split off is credited to the joint manufacturing costs using method 1 described in the text, which credits the by product’s net realizable value as a reduction in the joint costs.

Required

a. What are the allocated joint costs for the three outputs

(1) If Xy 3 is accounted for as a joint product?

(2) If Xy 3 is accounted for as a by product?

b. Management does not understand why joint costs are allocated to Xy 3 differently when it is accounted for as a by product. Write a brief memo explaining why this occurs.

joint cost allocation and product profitability western woods inc processes logs int 691220

Joint Cost Allocation and Product Profitability

Western Woods, Inc., processes logs into grade A and grade B lumber. Logs cost $12,000 per load. The milling process produces 4,000 units of grade A with a market value of $28,000, and 12,000 units of grade B with a market value of $4,000. The cost of the milling process is $5,120 per load.

Required

a. If the costs of the logs and the milling process are allocated on the basis of units of output, what cost will be assigned to each product?

b. If the costs of the logs and the milling process are allocated on the basis of the net realizable value, what cost will be assigned to each product?

c. How much profit or loss does the grade B lumber provide using the data in this problem and your analysis in requirement (a)? Is it really possible to determine which product is more profitable? Explain why or why not.

effect of cost allocation on pricing and make versus buy decisions ag coop is a larg 691221

Effect of Cost Allocation on Pricing and Make versus Buy Decisions

Ag Coop is a large farm cooperative with a number of agriculture related manufacturing and service divisions. As a cooperative, it pays no federal income taxes. The company owns a fertilizer plant that processes and mixes petrochemical compounds into three brands of agricultural fertilizer: greenup, maintane, and winterizer. The three brands differ with respect to selling price and the proportional content of basic chemicals.

Ag Coop’s Fertilizer Manufacturing Division transfers the completed product to the cooperative’s Retail Sales Division at a price based on the cost of each type of fertilizer plus a markup.

The Manufacturing Division is completely automated so that the only costs it incurs are the costs of the petrochemical feedstock’s plus overhead that is considered fixed. The primary feedstock costs $1.50 per pound. Each 100 pounds of feedstock can produce either of the following mixtures of fertilizer.

Output Schedules (in pounds)

 

A

B

Greenup

50

60

Maintane

30

10

Winterizer

20

30

Production is limited to the 750,000 kilowatt hours monthly capacity of the dehydrator. Due to different chemical makeup, each brand of fertilizer requires different dehydrator use. Dehydrator usage in kilowatt hours per pound of product follows:

 

Kilowatt Hour Usage

Product

per Pound

Greenup

32

Maintane

20

Winterizer

40

Monthly fixed costs are $81,250. The company currently is producing according to output schedule A. Joint production costs including fixed overhead are allocated to each product on the basis of weight.

The fertilizer is packed into 100 pound bags for sale in the cooperative’s retail stores. The sales price for each product charged by the cooperative’s Retail Sales Division follows:

 

Sales Price per Pound

Greenup

$10.50

Maintane

9.00

Winterizer

10.40

Selling expenses are 20 percent of the sales price.

The Retail Sales Division manager has complained that the prices charged by the Manufacturing Division are excessive and that he would prefer to purchase from another supplier.

The Manufacturing Division manager argues that the processing mix was determined based on a careful analysis of the costs of each product compared to the prices charged by the Retail Sales Division.

Required

a. Assume that joint production costs including fixed overhead are allocated to each product on the basis of weight. What is the cost per pound of each product, including fixed overhead and the feedstock cost of $1.50 per pound, given the current production schedule?

b. Assume that joint production costs including fixed overhead are allocated to each product on the basis of net realizable value if sold through the cooperative’s Retail Sales Division. What is the allocated cost per pound of each product, given the current production schedule?

c. Assume that joint production costs including fixed overhead are allocated to each product on the basis of weight. Which of the two production schedules, A or B, produces the higher operating profit to the firm as a whole?

d. Would your answer to requirement (c) be different if joint production costs including fixed overhead were allocated to each product on the basis of net realizable value? If so, by how much?

cost of quality financial reporting issues consider adapting the cost of quality fra 691163

Cost of Quality: Financial Reporting Issues

Consider adapting the cost of quality framework to financial reporting issues. Assign costs to one of four categories: prevention (P), appraisal (A), internal failure (IF), and external failure (EF), where the categories refer to financial reporting activities and the consequences of poor, or even illegal, financial reporting.

Required

Classify the following costs incurred for financial reporting activities into the four categories.

a. Extra work done by external auditors to complete the audit because new employees made a lot of errors.

b. Effects of bad publicity on stock prices because publication of financial statements was delayed in order to correct errors in the statements.

c. Employee training: new accounting regulations.

d. Drop in stock price from bad publicity after the chief executive gets sentenced to 10 years in prison.

e. Fines for failing to comply with accounting regulations.

f. Design of information systems to keep out hackers.

g. Design of internal control systems to minimize errors in data entry.

h. Internal auditors’ review of internal controls in item (g) above.

activity based reporting service organization allcott computer services acs provides 691164

Activity Based Reporting: Service Organization

Allcott Computer Services (ACS) provides computer training and repair services for schools and local businesses. Sales for year 1 totaled $1,350,000. Information regarding resources for the year includes the following:

 

Resources Used

Resources Supplied

Marketing

$112,000

$120,000

Depreciation

87,000

89,500

Training personnel

45,000

54,000

Energy

80,000

85,500

Short term labor

225,000

310,000

Long term labor

415,000

425,000

Administrative

70,000

79,000

In addition, ACS spent $42,000 on 500 repair verifications with a cost driver rate of $75.

Required

Management has requested that you do the following:

a. Prepare a traditional income statement.

b. Prepare an activity based income statement.

c. Write a short report to management explaining why the activity based income statement provides useful information to managers. Use the information from requirements (a) and (b) to develop examples for your report.

customer profitability skiblu ltd divides its customers into gold customers and silv 691165

Customer Profitability

SkiBlu, Ltd., divides its customers into Gold customers and Silver customers. The company has one full time customer representative per 1,000 Gold customers and one full time customer representative per 10,000 Silver customers. Customer representatives receive salaries plus bonuses of 10 percent of customer gross margin. SkiBlu spends 90 percent of its promotion costs on Gold customers to encourage their loyalty.

Customer Costs

Total

Gold

Silver

Number of customers

100,000

30,000

70,000

Average customer representative salary

 

$35,000

$35,000

Promotion costs

$4,000,000

 

 

Average gross margin per customer

 

$ 220

$ 70

Required

a. What is the excess of gross margin over customer costs for each category of customer?

b. Write a short memo that evaluates customer profitability.

customer profitability lighthouse company divides its customers into premium custome 691166

Customer Profitability

Lighthouse Company divides its customers into Premium customers and Standard customers. The company has one full time customer representative per 100 Premium customers and one full time customer representative per 1,000 Standard customers. Customer representatives receive salaries plus bonuses of 1 percent of customer gross margin. Lighthouse spends 80 percent of its promotion costs on Premium customers to encourage their loyalty.

Customer Costs

Total

Premium

Standard

Number of customers

10,000

2,000

8,000

Average customer representative salary

 

$60,000

$60,000

Promotion costs

$1,000,000

 

 

Average gross margin per customer

 

$ 1,500

$ 150

Required

a. What is the excess of gross margin over customer costs for each category of customer?

b. Write a short memo that evaluates customer profitability.

activity based costing of suppliers jfi foods produces processed foods their basic i 691167

Activity Based Costing of Suppliers

JFI Foods produces processed foods. Their basic ingredient is a feedstock that is mixed with other ingredients to produce the final packaged product. JFI purchases the feedstock from two suppliers, Rex Materials and Red Oak Chemicals. The quality of the final product depends directly on the quality of the feedstock. If the feedstock is not correct, JFI has to dispose of the entire batch. All feedstock in this business is occasionally “bad,” so JFI measures what they call the “yield,” which is measured as,

Yield = Gold output ÷ Input

where the output and inputs are both measured in tons. As a benchmark, JFI expects to get 8 tons of good output for every 10 tons of feedstock purchased for a yield of 80 percent (= 8 tons of output ÷ 10 tons of feedstock). Data on the two suppliers for the past year follow:

 

Rex

Red Oak

 

 

Materials

Chemicals

Total

Total inputs purchased

1,350

2,250

3,600

Good output (tons)

1,242

1,548

2,790

Average price (per ton)

$90.00

$70.00

$77.50

Required

Assume that the average quality, measured by the yield, and prices from the two companies will continue as in the past. What is the effective price for feedstock from the two companies when quality is considered?

activity based reporting manufacturing leidenheimer corporation manufactures small a 691168

Activity Based Reporting: Manufacturing

Leidenheimer Corporation manufactures small airplane propellers. Sales for year 2 totaled $1,700,000. Information regarding resources for the month follows:

 

Resources Used

Resources Supplied

Parts management

$ 60,000

$ 70,000

Energy

100,000

100,000

Quality inspections

90,000

100,000

Long term labor

50,000

70,000

Short term labor

40,000

48,000

Setups

140,000

200,000

Materials

300,000

300,000

Depreciation

120,000

200,000

Marketing

140,000

150,000

Customer service

20,000

40,000

Administrative

100,000

140,000

In addition, Leidenheimer spent $50,000 on 50 engineering changes with a cost driver rate of $1,000 and $60,000 on eight outside contracts with a cost driver rate of $7,500.

Required

Management has requested that you do the following:

a. Prepare a traditional income statement.

b. Prepare an activity based income statement.

c. Write a short report explaining why the activity based income statement provides useful information to managers. Use the information from requirements (a) and (b) to develop examples for your report.

quality improvement iport products makes cases for portable music players in two pro 691172

Quality Improvement

IPort Products makes cases for portable music players in two processes, cutting and sewing. The

cutting process has a capacity of 100,000 units per year; sewing has a capacity of 120,000 units per

year. Cost information follows:

Inspection and testing costs

$ 60,000

Scrap costs (all in the cutting dept.)

195,000

Demand is very strong. At a sales price of $20 per case, the company can sell whatever output it can produce.

IPort Products can start only 100,000 units into production in the cutting department because of capacity constraints. Defective units are detected at the end of production in the cutting department. At that point, defective units are scrapped. Of the 100,000 units started at the cutting operation, 15,000 units are scrapped. Unit costs in the cutting department for both good and defective units equal $13 per unit, including an allocation of the total fixed manufacturing costs of $600,000 per year to units.

Direct materials (variable)

$ 5

Direct manufacturing, setup, and materials handling labor (variable)

2

Depreciation, rent, and other overhead (fixed)

6

 

$13

The fixed cost of $6 per unit is the allocation of the total fixed costs of the cutting department to each unit, whether good or defective. (The total fixed costs are the same whether the units produced in the cutting department are good or defective.)

The good units from the cutting department are sent to the sewing department. Variable manufacturing costs in the sewing department are $3 per unit and fixed manufacturing costs are $50,000 per year. There is no scrap in the sewing department. Therefore, the company’s total sales quantity equals the cutting department’s good output. The company incurs no other variable costs.

The company’s designers have discovered a new type of direct material that would reduce scrap in the cutting department to 5,000 units. However, using the new material would increase the direct materials costs to $7.25 per unit in the cutting department for all 100,000 units. Recall that only 100,000 units can be started each year.

Required

a. Should IPort use the new material and improve quality? Assume that inspection and testing costs will be reduced by $20,000 if the new material is used. Fixed costs in the sewing department will remain the same whether 85,000 or 95,000 units are produced.

b. What other nonfinancial and qualitative factors should management of IPort Products consider in making the decision?

quality improvement metallic inc produces metal gates in two processes bending in wh 691173

Quality Improvement

Metallic, Inc., produces metal gates in two processes: bending, in which metal is bent to the correct shape, and welding, in which the bent metal pieces are welded into gates. The bending process has a capacity of 10,000 units per year; welding has a capacity of 14,000 units per year. Demand is strong. At a sales price of $500 per unit, the company can sell whatever output it can produce.

Metallic can start only 10,000 units into production in the bending department because of capacity constraints. At present, 1,500 units are found to be defective in the bending department each year. Defective units are not detected until the end of production in the bending department. At that point, the 1,500 defective units are scrapped. So, of the 10,000 units started in the bending operation, 1,500 units are scrapped. Unit costs in the bending department for both good and defective units equal $250 per unit, including an allocation of the total fixed manufacturing costs of $750,000 per year to units.

Direct materials (variable)

$125

Direct manufacturing, setup, and materials handling labor (variable)

50

Depreciation, rent, and other overhead (fixed)

75

 

$250

The fixed cost of $75 per unit is the allocation of total fixed costs of the bending department to each unit, whether good or defective. (The total fixed costs are the same whether the units produced in the bending department are good or defective.)

The good units from the bending department are sent to the welding department. Variable manufacturing costs in the welding department are $75 per unit and fixed manufacturing costs are $500,000 per year. There is no scrap in the welding department. Therefore, the company’s total sales quantity equals the bending department’s good output. The company incurs no other variable costs.

The company’s designers have discovered that, by using a new type of direct material, the company could reduce scrap in the bending department from 1,500 units to 500 units. Using the new material would increase the direct materials costs to $180 per unit in the bending department for all 10,000 units. Recall that only 10,000 units can be started each year.

Required

a. Should Metallic use the new material and improve quality? Assume that inspection and testing costs of $120,000 per year will be reduced by $20,000 with the new materials. Fixed costs in the bending department will remain the same whether 8,500 or 9,500 units are produced.

b. What other nonfinancial and qualitative factors should management of Metallic consider in making the decision?

unused capacity the grape cola caper assume that all of the facts still hold except 691175

Unused Capacity: The Grape Cola Caper

Assume that all of the facts still hold except that the practical capacity of the machinery is 20,000 hours instead of 10,000 hours.

Required

a. Recompute the unit costs for each of the cola products: Diet, Regular, Cherry, and Grape.

b. What is the cost of unused capacity? What do you recommend that Rockness Bottling do with this unused capacity?

c. Now assume that Rockness is considering producing a fifth product: Vanilla cola. Because Vanilla cola is in high demand in Rockness Bottling’s market, assume that it would use 10,000 hours of machine time to make 100,000 units. (Recall that the machine capacity in this case is 20,000 hours, while Diet, Regular, Cherry, and Grape consume only 10,000 hours.) Vanilla cola’s per unit costs would be identical to those of Diet cola except for the machine usage costs. What would be the cost of Vanilla cola? Calculate on a per unit basis, and then in total.

modoc bank is a small retail bank with two branches downtown and mall it has three s 691177

Modoc Bank is a small retail bank with two branches, Downtown and Mall. It has three service departments: Personnel, Finance, and Building Occupancy. The service departments provide support to both branches as well as to the other service departments. However, the branches are considered the only two profit centers, and the branch managers are evaluated on branch profits after allocation of service department costs.

During the current period, the direct costs incurred in each of the departments follow:

Department

Direct Cost

Personnel

$ 202,500

Finance

126,000

Building Occupancy

150,000

Downtown

950,000

Mall

425,000

Total

$1,853,500

Personnel costs are allocated on the basis of number of employees. Finance costs are allocated on the basis of billable transactions. Building Occupancy costs are allocated on the basis of the number of square feet in each user department. For the current period, the following table summarizes the usage of services by other service cost centers and other departments:

Service Department

 

 

Building

Occupancy

(square feet)

 

Personnel

(employees)

Finance

(transactions)

Departments

Personnel

–0–

13,000

15,000

Finance

30

–0–

10,000

Building

 

 

 

Occupancy

15

1,000

–0–

Downtown

60

60,000

30,000

Mall

30

24,000

45,000

Total

135

98,000

100,000

Using the direct method for service cost allocations, what is the total cost for each branch that will be used for determining branch profits?

some firms choose the order of allocation based on the costs in the individual servi 691178

Some firms choose the order of allocation based on the costs in the individual service departments. Consider the case of CCC where Administration is the service department with the higher direct costs. Compute the service cost allocated to each mine (Hilltop and Pacific) using the step method. Start by allocating Administration costs first. Recall that Administration’s direct cost is $5,000,000 and Information Systems’ is $800,000. See Exhibit 11.2 for service department use data.

Exhibit 11.2 Basic Data for Service Department Cost Allocation—Carlyle Coal Company

 

A

B

C

D

E

 

1

 

Service Department

 

2

 

Information Systems (S1)

Administration (S2)

 

3

 

Usage

Percent

Usage

Percent

 

4

Departments

(hours)

of Total

(employees)

of Total

 

5

Administration

100,000

50%

–0–

0%

 

6

Information Systems

–0–

0

2,000

20

 

7

Hilltop Mine (P1)

20,000

10

5,000

50

 

8

Pacific Mine (P2)

80,000

40

3,000

30

 

9

Total

200,000

100%

10,000

100%

 

10

 

 

 

 

 

 

 

 

 

 

 

 

 

Management systems that an important aspect of cost allocation is deciding which allocation base to use. Because we have already spent a great deal of time on the choice of cost allocation bases, we simply specify that CCC has determined that the best allocation base for Information Systems is computer hours and the best allocation base for Administration is number of employees.

williston machining is a small manufacturing firm with two production departments fi 691179

Williston Machining is a small manufacturing firm with two production departments, Finishing and Assembly. Its two service departments, Maintenance and the Cafeteria, serve both production departments.

During the current period, the direct costs incurred in each department follow:

Department

Direct Cost

Maintenance

$ 100,000

Cafeteria

 17,600

Finishing

1,200,000

Assembly

640,000

Total

$1,957,600

Maintenance costs are allocated on the basis of repair hours. Cafeteria costs are allocated on the basis of the number of employees in each department. For the current period, the following table summarizes the usage of services by other service cost centers and other departments:

Service Department

 

Maintenance (S1)

(repair hours)

Cafeteria (S2)

(employees)

Departments

Maintenance

–0–

30

Cafeteria

3,000

–0–

Finishing (P1)

7,500

20

Assembly (P2)

4,500

50

Total

15,000

100

Using the reciprocal method for service cost allocations, what are the total costs in each of the two production departments, Finishing (P1) and Assembly (P2)?

assume that the sugar cannot be sold at split off but requires additional processing 691181

Assume that the sugar cannot be sold at split off but requires additional processing. The additional processing costs $100 per ton, at which point the sugar can be sold for $450 per ton. Allocate the joint costs to the two products using the estimated net realizable value method.

Self study Questions 4: Thumb Beets, Inc., grows sugar beets. After the beets are harvested, they are processed into sugar and livestock feed. One ton of sugar beets yield 0.2 tons of sugar and 0.4 tons of feed. The sugar can be sold for $400 per ton and the feed for $200 per ton at the split off point. The cost of the sugar beets is $60 per ton (2,000 pounds). Processing each ton of beets up to the split off point costs $40 in labor and overhead. Compute the joint cost allocated to sugar and feed produced from 10 tons of sugar beets using the net realizable value method.

why are costs allocated mdash ethical issues you are the division president of stabl 691185

Why Are Costs Allocated?—Ethical Issues

You are the division president of Stable Division of Giga Corp. Your friend, Ligia, is the division president of Giga Corp.’s Turmoil Division. These are the only divisions. Each division has 5,000 employees. Last year, Stable Division had a turnover of 1,000 employees (1,000 employees left and 1,000 were hired). Turmoil Division had a turnover of 4,000 employees. There were no transfers between divisions.

Giga Corp.’s Personnel Department only provides services to Stable and Turmoil and only when an employee leaves or is hired. The total cost of the Personnel Department last year was $100,000.

Required

a. As the Stable Division president, how would you recommend the cost of the Personnel Department be allocated? What arguments would you use to support your claim?

b. As the Turmoil Division president, how do you think Ligia will recommend allocating the cost of the Personnel Department? Why?

c. You are going to be transferred to Turmoil Division, but before the transfer, Giga Corp.’s CEO asks you to recommend an allocation method. How would you recommend the cost of the Personnel Department be allocated? What arguments would you use?

d. Is it ethical to recommend different allocation methods depending on which division you will be heading?

cost allocation direct method warren ltd has two production departments building a a 691186

Cost Allocation: Direct Method

Warren Ltd. has two production departments, Building A and Building B, and two service departments, Maintenance and Cafeteria. Direct costs for each department and the proportion of service costs used by the various departments for the month of June follow:

Proportion of Services Used by

Department

Direct Costs

Maintenance

Cafeteria

Building A

Building B

 

 

 

 

 

 

Building A

$990,000

 

 

 

 

Building B

644,000

 

 

 

 

Maintenance

400,000

0.2

0.5

0.3

Cafeteria

320,000

0.8

0.1

0.1

Required

Compute the allocation of service department costs to producing departments using the direct method.

allocating service department costs first to production departments and then to jobs 691187

Allocating Service Department Costs First to Production Departments and Then

to Jobs

Refer to the facts in Exercise 11 22. Assume that both Building A and Building B work on just two jobs during the month of June: RW 12 and RW 13. Costs are allocated to jobs based on labor hours in Building A and machine hours in Building B. The number of labor and machine hours worked in each department are as follows:

 

 

Building A

Building B

Job RW 12:

Labor hours

160

20

 

Machine hours

20

40

Job RW 13

Labor hours

20

20

 

Machine hours

20

180

Required

How much of the service department costs allocated to Building A and Building B in the direct method should be allocated to Job RW 12? How much should be allocated to Job RW 13?

Exercise 11 22 Cost Allocation: Direct Method

Warren Ltd. has two production departments, Building A and Building B, and two service departments, Maintenance and Cafeteria. Direct costs for each department and the proportion of service costs used by the various departments for the month of June follow:

Proportion of Services Used by

Department

Direct Costs

Maintenance

Cafeteria

Building A

Building B

 

 

 

 

 

 

Building A

$990,000

 

 

 

 

Building B

644,000

 

 

 

 

Maintenance

400,000

0.2

0.5

0.3

Cafeteria

320,000

0.8

0.1

0.1

Required

Compute the allocation of service department costs to producing departments using the direct method.

cost allocation step method refer to the data for warren ltd in exercise 11 22 requi 691189

Cost Allocation: Step Method

Refer to the data for Warren Ltd. in Exercise 11 22.

Required

Use the step method to allocate the service costs, using the following:

a. The order of allocation starts with Maintenance.

b. The allocations are made in the reverse order (starting with Cafeteria).

Exercise 11 22 Cost Allocation: Direct Method

Warren Ltd. has two production departments, Building A and Building B, and two service departments, Maintenance and Cafeteria. Direct costs for each department and the proportion of service costs used by the various departments for the month of June follow:

Proportion of Services Used by

Department

Direct Costs

Maintenance

Cafeteria

Building A

Building B

 

 

 

 

 

 

Building A

$990,000

 

 

 

 

Building B

644,000

 

 

 

 

Maintenance

400,000

0.2

0.5

0.3

Cafeteria

320,000

0.8

0.1

0.1

Required

Compute the allocation of service department costs to producing departments using the direct method.

cost allocation reciprocal method refer to the data for university printers in exerc 691192

Cost Allocation: Reciprocal Method

Refer to the data for University Printers in Exercise 11 24.

Required

Allocate the service department costs using the reciprocal method. (Matrix algebra is not required because there are only two service departments.)

Exercise 11 24 Cost Allocation: Direct Method            

University Printers has two service departments (Maintenance and Personnel) and two operating departments (Printing and Developing). Management has decided to allocate maintenance costs on the basis of machine hours in each department and personnel costs on the basis of labor hours worked by the employees in each.

The following data appear in the company records for the current period:

 

Maintenance

Personnel

Printing

Developing

Machine hours

1,000

1,000

3,000

Labor hours

500

500

2,000

Department direct costs

$15,000

$36,000

$45,000

$30,000

Required

Use the direct method to allocate these service department costs to the operating departments

reciprocal cost allocation mdash outsourcing a service department refer to the facts 691193

Reciprocal Cost Allocation—Outsourcing a Service Department

Refer to the facts in Exercise 11 22. Warren estimates that the variable costs in the Maintenance Department total $145,000, and in the Cafeteria variable costs total $160,000. Avoidable fixed costs in the Maintenance Department are $90,000.

Required

If Warren outsources the Maintenance Department, what is the maximum they can pay an outside vendor without increasing total costs?

Exercise 11 22 Cost Allocation: Direct Method

Warren Ltd. has two production departments, Building A and Building B, and two service departments, Maintenance and Cafeteria. Direct costs for each department and the proportion of service costs used by the various departments for the month of June follow:

Proportion of Services Used by

Department

Direct Costs

Maintenance

Cafeteria

Building A

Building B

 

 

 

 

 

 

Building A

$990,000

 

 

 

 

Building B

644,000

 

 

 

 

Maintenance

400,000

0.2

0.5

0.3

Cafeteria

320,000

0.8

0.1

0.1

Required

Compute the allocation of service department costs to producing departments using the direct method.

reciprocal cost allocation mdash outsourcing a service department refer to the facts 691194

Reciprocal Cost Allocation—Outsourcing a Service Department

Refer to the facts in Exercise 11 24. University Printers estimates that the variable costs in the Personnel Department total $20,000 and in the Maintenance Department variable costs total $8,750. Avoidable fixed costs in the Personnel Department are $6,000. If Warren outsources the Personnel Department functions, what is the maximum they can pay an outside vendor without increasing total costs?

Exercise 11 24 Cost Allocation: Direct Method            

University Printers has two service departments (Maintenance and Personnel) and two operating departments (Printing and Developing). Management has decided to allocate maintenance costs on the basis of machine hours in each department and personnel costs on the basis of labor hours worked by the employees in each.

The following data appear in the company records for the current period:

 

Maintenance

Personnel

Printing

Developing

Machine hours

1,000

1,000

3,000

Labor hours

500

500

2,000

Department direct costs

$15,000

$36,000

$45,000

$30,000

Required

Use the direct method to allocate these service department costs to the operating departments

benefits of activity based costing cawker products has two manufacturing facilities 691139

Benefits of Activity Based Costing

Cawker Products has two manufacturing facilities—Lucas plant and Russell plant—that produce the same product. Until recently, the production process in both plants has been the same. Last year, the Russell production supervisor, Ann Tyler, determined that she could use lower cost utility labor in place of the skilled direct labor to bring raw materials to the assembly line and move finished products to the warehouse. While the total time required remained the same, the new material handling process was included in overhead rather than being considered as direct labor. Staffing at the Russell plant was adjusted to reflect the change. 

In looking over the production plans for next year, Jason Hunter, the CEO of Cawker Products, is surprised by the cost estimates for the two plants. Specifically, he notes that the overhead rate, which had been comparable between the two plants, is now much higher at the Russell plant. He suggests moving some of the production to Lucas to save money.

Required

Prepare a report that states how an activity based costing system might benefit Cawker Products and clear up the CEO’s confusion.

decision making and distorted costs ace industries is a manufacturer of machined par 691141

Decision Making and Distorted Costs

ACE Industries is a manufacturer of machined parts located in midwestern United States. Its primary customers are suppliers to the automobile industry, although it has diversified its customer base in recent years to reduce its dependence on any one sector. It is considered to be a well run and profitable company.

In July, the company appointed Stephen Smale as Chief Financial Officer. Smale had a reputation for having a strong financial focus and a ruthless approach to cost cutting. His appointment was the result of a change in strategy along these lines that coincided with the downturn in the economy. His view was that financial success was attained by focusing on products that were highly profitable rather than being a full line producer. Thus, he imposed a requirement that any product that had a budgeted profit margin (product line profit divided by product line revenue) below 10 percent be dropped.

The product line managers are currently preparing their budgets for the next year. This will be the first year that Smale’s approach to running the business (requiring a minimum 10 percent profit  margin) will be tested. Product line profits are computed by first deducting estimated direct labor and direct material costs from estimated product revenues. Manufacturing overhead is then allocated to products based on estimated direct labor costs. Corporate overhead is not allocated to products and is not included in the 10 percent requirement. Estimated product line revenues and direct costs follow:

 

 

Direct

Direct

 

 

Material

Labor

Product

Revenue

Costs

Costs

A

$270,000

$ 70,000

$20,000

C

165,000

65,000

12,000

E

305,000

145,000

28,000

Estimated annual manufacturing overhead is $240,000.

Required

a. Assuming all products are sold and all revenues and costs are as budgeted, what will operating profit t be next year?

b. Based on the criterion set by the CFO, which product(s) will the company drop?

c. Regardless of your answer to requirement (b), assume the company decided to drop product E. The controller estimates that if product E is dropped, manufacturing overhead will decline to $208,000. Assuming all other product line revenues and costs are incurred as budgeted, what will operating profit t be?

d. The company again goes through the exercise with the two products to see if any should be dropped. Based on the criterion set by the CFO, which product(s) will the company drop?

e. Regardless of your answer to requirement (d), assume the company decides to drop product C. The controller estimates that if product C is dropped, manufacturing overhead will decline to $190,000. Assuming all other product line revenues and costs are incurred as budgeted, what will operating profit t be? What will the profit t margin be?

f. What is the cause of what you observe? Is reported profit t margin a useful way to make the decision of what products to keep in this situation?

cost allocation and environmental processes mdash ethical issues california circuits 691142

Cost Allocation and Environmental Processes—Ethical Issues

California Circuits Company (3C) manufactures a variety of components. Its Valley plant specializes in two electronic components used in circuit boards. These components serve the same function and perform equally well. The difference in the two products is the raw material. The XL D chip is the older of the two components and is made with a metal that requires a wash prior to assembly. Originally, the plant released the wastewater directly into a local river. Several years ago, the company was ordered to treat the wastewater before its release, and it installed relatively expensive equipment. While the equipment is fully depreciated, annual operating expenses of $250,000 are still incurred for wastewater treatment.

Two years ago, company scientists developed an alloy with all of the properties of the raw materials used in XL D that generates no wastewater. Some prototype components using the new material were produced and tested and found to be indistinguishable from the old components in every way relating to their fitness for use. The only difference is that the new alloy is more expensive than the old raw material. The company has been test marketing the newer version of the component, referred to as XL C, and is currently trying to decide its fate. Manufacturing both components begins in the Production Department and is completed in the Assembly Department. No other products are produced in the plant. The following provides information for the two components:

 

XL D

XL C

Units produced

100,000

25,000

Raw material costs per unit

$ 12

$ 14

Direct labor hours per unit—Production

0.1

0.1

Direct labor hours per unit—Assembly

0.4

0.4

Direct labor rate per hour—all labor

$ 20

$ 20

Machine hours per unit—Production

1.6

1.6

Machine hours per unit—Assembly

0.4

0.4

Testing hours per unit (all in production)

3.0

3.0

Shipping weight per unit (pounds)

1.0

1.6

Wastewater generated per unit (gallons)

10.0

0.0

Annual overhead costs for the two departments follow:

 

Production

Assembly

 

Department

Department

Supervision

$ 100,000

$240,000

Material handling

93,000

40,000

Testing

150,000

–0–

Wastewater treatment

250,000

–0–

Depreciation on equipment

400,000

100,000

Shipping

7,000

120,000

Total

$1,000,000

$500,000

The company president believes that it’s foolish to continue producing two essentially equivalent products. At the same time, the corporate image is somewhat tarnished because of a toxic dump found at another site (not the Valley plant). The president would like to be able to point to the Valley plant as an example of company R&D working to provide an environmentally friendly product. The controller points out to the president that the company’s financial position is shaky, and it cannot afford to make products in any way other than the most cost efficient one.

Required

a. California Circuits Company’s current cost accounting system charges overhead to products based on direct labor cost using a single plant wide rate. What product costs will it report for the two products if the current allocation system is used?

b. The controller recently completed an executive education course describing the two stage allocation procedure. Assume that the first stage allocates costs to departments and the second stage allocates costs to products. The controller believes that the costs will be more accurate if machine hours are used to allocate Production Department costs and labor hours are used to allocate Assembly Department costs. What product costs will be reported for the two products if the two stage allocation process is used?

c. Explain the results found in requirements (a) and ( b ).

d. The president argues that an activity based costing system would provide even better costs. The company decides to compute product costs assuming an ABC system is implemented only in the Production Department. Overhead in Assembly will continue to be allocated based on direct labor cost. The cost drivers selected for the activity based costing system are:

Overhead Item

Driver

Supervision

Direct labor hours

Material handling

Material cost

Testing

Testing hours

Wastewater treatment

Wastewater generated

Depreciation on equipment

Machine hours

Shipping

Weight

What product costs would be reported if this ABC system were implemented? Assume that the production mix and costs would remain as originally planned.

e. Because the two products are identical in their use, the controller argues that the decision should be made on cost alone. Do you agree? Explain.

distortions caused by inappropriate overhead allocation base chocolate bars inc cbi 691143

Distortions Caused by Inappropriate Overhead Allocation Base

Chocolate Bars, Inc. (CBI), manufactures creamy deluxe chocolate candy bars. The firm has developed three distinct products: Almond Dream, Krispy Krackle, and Creamy Crunch. CBI is profitable, but management is quite concerned about the profitability of each product and the product costing methods currently employed. In particular, management questions whether the overhead allocation base of direct labor hours accurately reflects the costs incurred during the production process of each product.

In reviewing cost reports with the marketing manager, Steve Hoffman, who is the cost accountant, notices that Creamy Crunch appears exceptionally profitable and that Almond Dream appears to be produced at a loss. This surprises both him and the manager, and after much discussion, they are convinced that the cost accounting system is at fault and that Almond Dream is performing very well at the current market price. Steve decides to hire Jean Sharpe, a management consultant, to study the firm’s cost system over the next month and present her findings and recommendations to senior management. Her objective is to identify and demonstrate how the cost accounting system might be distorting the firm’s product costs.

Jean begins her study by gathering information and documenting the existing cost accounting  system. It is rather simplistic, using a single overhead allocation base—direct labor hours—to calculate and apply overhead rates to all products. The rate is calculated by summing variable and fixed overhead costs and then dividing the result by the number of direct labor hours. The product cost is determined by multiplying the number of direct labor hours required to manufacture the product by the overhead rate and adding this amount to the direct labor and direct material costs.

 

Almond

Krispy

Creamy

 

Dream

Krackle

Crunch

Product costs

 

 

 

Labor hours per case

7

3

1

Total cases produced

1,000

1,000

1,000

Material cost per case

$ 8

$ 2

$ 9

Direct labor cost per case

$ 42

$ 18

$ 6

Labor hours per product

7,000

3,000

1,000

Total overhead = $69,500

 

 

 

Total labor hours = 11,000

 

 

 

Direct labor costs per hour = $6

 

 

 

Allocation rate per labor hour = (a) .

 

 

 

Costs of products

 

 

 

Material cost per case

$ 8

$ 2

$ 9

Direct labor cost per case

42

18

6

Allocated overhead per case

 

 

 

(to be computed)

(b)

(c)

(d )

Product cost

(e)

(f )

(g )

 CBI engages in two distinct production processes for each product. Process 1 is labor intensive, using a high proportion of direct materials and labor. Process 2 uses special packing equipment that  wraps each individual candy bar and then packs it into a box of 24 bars. The boxes are then packaged into cases, each of which has six boxes. Special packing equipment is used on all three products and   has a monthly capacity of 3,000 cases, each containing 144 candy bars (= 6 boxes × 24 bars). To illustrate the source of the distortions to senior management, Jean collects the cost data for the three products, Almond Dream, Krispy Krackle, and Creamy Crunch (see Exhibit 9.21). CBI recently adopted a general policy to discontinue all products whose gross profit margin percentages [(Gross margin ÷ Selling price) × 100] were less than 10 percent. By comparing the selling prices to the firm’s costs and then calculating the gross margin percentages, Jean could determine which products, under the current cost system, should be dropped. The current selling prices of Almond Dream, Krispy Krackle, and Creamy Crunch are $85, $55, and $35 per case, respectively. Overhead will remain $69,500 per month under all alternatives.

Required

a. Complete Exhibit 9.21 under the current cost system and determine which product(s), if any, should be dropped.

b. What characteristic of the product that should be dropped makes it appear relatively unprofitable?

c. Assume that CBI drops the product(s) identified in requirement (a) above. Calculate the gross profit t margin percentage for the remaining products. Assume that CBI can sell all products that it manufactures and that it will use the excess capacity from dropping a product to produce more of the most profit table product. If CBI maintains its current rule about dropping products, which additional products, if any, should CBI drop under the existing cost system?

d. Assume that CBI drops the products identified in requirements (a) and ( c ) above. Recalculate the gross profit t margin percentage for the remaining product(s) and ascertain whether any additional product(s) should be dropped.

e. Discuss the outcome and any recommendations you might make to management regarding the

 current cost system and decision policies.

multiple allocation bases refer to problem 9 46 jean sharpe decides to gather additi 691144

Multiple Allocation Bases

Refer to Problem 9 46. Jean Sharpe decides to gather additional data to identify the cause of overhead costs and figure out which products are most profitable. She notices that $30,000 of the overhead originated from the equipment used. She decides to incorporate machine hours into the overhead allocation base to determine the effect on product profitability. Almond Dream requires 2 machine hours per case, Krispy Krackle requires 7 hours per case, and Creamy Crunch requires 6 hours per case. Additionally, Jean notices that the $15,000 per month spent to rent 10,000 square feet of factory space accounts for almost 22 percent of the overhead. The assignment of square feet is 1,000 to Almond Dream, 4,000 to Krispy Krackle, and 5,000 to Creamy Crunch. Jean decides to incorporate this into the allocation base for the rental costs.

Because labor hours are still an important cost driver for overhead, Jean decides that she should use labor hours to allocate the remaining $24,500.

CBI still plans to produce 1,000 cases each of Almond Dream, Krispy Krackle, and Creamy Crunch. Assume that CBI can sell all products it manufactures and that if it drops any products, it will use excess capacity to produce additional cases of the most profitable product. Overhead will remain $69,500 per month under all alternatives.

Required

a. Based on the additional data, determine the product cost and gross profit t margin percentages of each product using the three allocation bases (labor hours, machine hours, and square feet)to determine the allocation assigned to each product.

b. Would management recommend dropping any product based on the criterion of dropping products with less than 10 percent gross profit t margin?

c. Based on the recommendation you make in requirement ( b ), recalculate the allocations and profit t margins to determine whether any of the remaining products should be dropped from the product line. If so, substantiate the profitability of remaining products.

activity based costing the grape cola caper howard rockness was worried his company 691145

Activity Based Costing: The Grape Cola Caper Howard Rockness was worried. His company, Rockness Bottling, showed declining profits over the past several years despite an increase in revenues. With profits declining and revenues increasing, Rockness knew there must be a problem with costs.

Rockness sent an e mail to his executive team under the subject heading, “How do we get Rockness Bottling back on track?” Meeting in Rockness’s spacious office, the team began brainstorming solutions to the declining profits problem. Some members of the team wanted to add products. (These were marketing people.) Some wanted to fire the least efficient workers. (These were finance people.) Some wanted to empower the workers. (These people worked in the human resources department.) And some people wanted to install a new computer system. (It should be obvious who these people were.)

Rockness listened patiently. When all participants had made their cases, Rockness said, “We made money when we were a smaller, simpler company. We have grown, added new product lines, and added new products to old product lines. Now we are going downhill. What’s wrong with this picture?” Rockness continued, “Here, look at this report. This is last month’s report on the cola bottling line. What do you see here?” He handed copies of the following report to the people assembled in his office.

 width=

 

Rockness asked, “Do you see any problems here? Should we drop any of these products? Should we reprice any of these products?” The room was silent for a moment, and then everybody started talking at once. Nobody could see any problems based on the data in the report, but they all made suggestions to Rockness ranging from “add another cola product” to “cut costs across the board” to “we need a new computer system so that managers can get this information more quickly.” A not so patient Rockness stopped the discussion abruptly and adjourned the meeting.

He then turned to the quietest person in the room—his son, Rocky—and said, “I am suspicious of these cost data, Rocky. Here we are assigning indirect costs to these products using a 260 percent rate. I really wonder whether that rate is accurate for all products. I want you to dig into the indirect cost data, figure out what drives those costs, and see whether you can give me  more accurate cost numbers for these products.”

Rocky first learned from production that the process required four activities: (1) setting up production runs, (2) managing production runs, and (3) managing products. The fourth activity did not require labor; it was simply the operation of machinery. Next, he went to the accounting records to get a breakdown of indirect costs. Here is what he found:

 

A

B

1

Indirect labor

$ 20,000

2

Fringe benefits on indirect labor

8,000

3

Information technology

10,000

4

Machinery depreciation

8,000

5

Machinery maintenance

4,000

6

Energy

2,000

7

Total

$ 52,000

8

 

 

 

 

 

Then, he began a series of interviews with department heads to see how to assign these costs to cost pools. He found that 40 percent of indirect labor was for scheduling or for handling production  runs, including purchasing, preparing the production run, releasing materials for the production run, and performing a first time inspection of the run. Another 50 percent of indirect labor was used to set up machinery to produce a particular product. The remaining 10 percent of indirect labor was spent maintaining records for each of the four products, monitoring the supply of raw materials required for each product, and improving the production processes for each product. This 10 percent of indirect labor was assigned to the cost driver “number of products.”

Interviews with people in the information technology department indicated that $10,000 was allocated to the cola bottling line. Eighty percent of this $10,000 information technology cost was for scheduling production runs. Twenty percent of the cost was for recordkeeping for each of the four products.

Fringe benefits were 40 percent of labor costs. The rest of the overhead was used to supply machine capacity of 10,000 hours of productive time.

Rocky then found the following cost driver volumes from interviews with production personnel.

• Setups: 560 labor hours for setups.

• Production runs: 110 production runs.

• Number of products: 4 products.

• Machine hour capacity: 10,000 hours.

Diet cola used 200 setup hours, 40 production runs, and 5,000 machine hours to produce 50,000 units. Regular cola used 60 setup hours, 30 production runs, and 4,000 machine hours to produce 40,000 units. Cherry cola used 240 setup hours, 30 production runs, and 900 machine hours to produce 9,000 units. Grape cola used 60 setup hours, 10 production runs, and 100 machine hours to produce 1,000 units. Rocky learned that the production people had a difficult time getting the taste just right for the Cherry and Grape colas, so these products required more time per setup than either the Diet or Regular colas.

Required

a. Compute cost driver rates for each of the four cost drivers.

b. Compute unit costs for each of the cola products: Diet, Regular, Cherry, and Grape.

c. Prepare a new “Monthly Report on Cola Bottling Line,” but with your revised indirect cost numbers for each product.

d. Prepare a memorandum to Howard Rockness recommending what to do.

cost hierarchy for a not for profit below are various resources and activities for a 691146

Cost Hierarchy for a Not for Profit

Below are various resources and activities for a social services agency that helps homeless people get housing.

Required

Indicate whether each of the following are likely to be unit level or facility level activities.

a. Utilities—building’s electrical power, natural gas, phone, and sanitation services.

b. Salaried caseworkers—personnel who interview and service clients.

c. Supplies—materials for office personnel.

d. Library—regulations, research, and court documents to support child, homeless, and elderly citizen services.

e. Information technology—desktop and mobile computing equipment and personnel to develop and maintain the agency’s information system.

f. Management—salaries and support for administrators.

g. Building—offices of employees.

h. Automobiles—transportation for caseworkers to meet clients.

driver identification below are various activities for a commercial loan company req 691147

Driver Identification

Below are various activities for a commercial loan company.

Required

Suggest a feasible cost driver base for each of the following, and explain why each selected cost driver base is feasible.

a. Sales calls—new commercial customers.

b. Commercial loan negotiation.

c. Commercial loan review.

d. Customer fi le maintenance.

e. Community involvement.

f. Employee relations.

g. Commercial loan customer service.

h. Consumer loan customer service.

i. Consumer loan review.

j. Sales calls—existing commercial customers.

k. Advertising particular products.

l. Consumer deposit/withdrawal processing.

m. Commercial deposit/withdrawal processing.

activity based costing of customers rock solid bank and trust rsb amp t offers only 691149

Activity Based Costing of Customers

Rock Solid Bank and Trust (RSB&T) offers only checking accounts. Customers can write checks and use a network of automated teller machines. RSB&T earns revenue by investing the money deposited; currently, it averages 5.2 percent annually on its investments of those deposits. To compete with larger banks, RSB&T pays depositors 0.5 percent on all deposits. A recent study classified the bank’s annual operating costs into four activities:

Activity

Cost Driver

Cost

Driver Volume

Using ATM

Number of uses

$ 750,000

10,000,000 uses

Visiting branch

Number of visits

2,250,000

750,000 visits

Processing transaction

Number of transactions

1,500,000

40,000,000 transactions

Managing functions

Total deposits

3,000,000

$187,500,000 in deposits

Total overhead

$7,500,000

 

Data on two representative customers follow:

 

Customer A

Customer B

ATM uses

200

250

Branch visits

5

20

Number of transactions

40

1,500

Average deposit

$200

$6,000

Required

a. Compute RSB&T’s operating profits.

b. Compute the profit from Customer A and Customer B, assuming that customer costs are based only on deposits. Interest costs = 0.5 percent of deposits; operating costs are 4 percent (= $7,500,000/$187,500,000) of deposits.

c. Compute the profit from Customer A and Customer B, assuming that customer costs are computed using the information in the activity based costing analysis.

activity based costing of customers ethical issues red rsquo s lumber hired a consul 691150

Activity Based Costing of Customers: Ethical Issues

Red’s Lumber hired a consultant to update its system for reporting the cost of customers. The consultant showed Red an analysis that indicates that customer support costs are significantly higher for customers who order on weekends than for those who order during the week.

a. What pricing decisions might Red make based on the consultant’s information?

b. Would these be ethical? Why or why not?

c. The consultant comes back to Red’s office and explains that he made a mistake in the analysis. The relation is actually between cost of customer support and the customer’s gender. Does this affect your answers to requirements (a) and (b)?

activity based costing of customers ethical issues central state college csc is a st 691151

Activity Based Costing of Customers: Ethical Issues

Central State College (CSC) is a state supported college with a large business school. The business school offers an undergraduate degree and training programs for a local manufacturer. The state does not support the training programs, which are paid for by the manufacturer under a fixed price contract. The college president has asked the dean of the business school for a breakdown of costs by program. The president will be meeting with state legislators asking for an increase in support for the college’s programs. The dean has assigned you to lead the team that will develop the costs by program. The business school’s computer lab is a major cost item. The lab is used during the day for the undergraduate program and in the evening for the training program.

a. How will you recommend that the cost of the computer lab be allocated to the two programs? Be explicit in your description of the allocation base.

b. The dean tells you that the training program should not be allocated any costs other than their direct costs. She points out that the college was established for undergraduate education and the training program is an incremental activity. “After all, if we didn’t have the training program, we would still have the computer lab,” she says. Do you agree with the dean? Is the dean’s suggestion ethical?

activity based costing of suppliers davis fabricators buys metal for manufacturing f 691152

Activity Based Costing of Suppliers

Davis Fabricators buys metal for manufacturing from two suppliers, Alpha Metals and First Parts. If the metal is delivered late, the shipment to the customer is delayed. Delayed shipments lead to contractual penalties that call for Davis to reimburse a portion of the purchase price to the customer. During the past quarter, the purchasing and delivery data for the two suppliers showed the following:

 

Alpha

First

Total

Total purchases (tons)

10,000

6,000

16,000

Average purchase price

$ 10.00

$ 12.00

$ 10.75

Number of deliveries

80

20

100

Percentage of late deliveries

25%

5%

21%

The accounting department recorded $33,600 as the cost of late deliveries to customers.

Required

Assume that the average quality, measured by the percentage of late deliveries, and prices from the two companies will continue as in the past. What is the effective price for metal from the two companies when late deliveries are considered?

activity based costing of suppliers kinnear plastics manufactures various components 691153

Activity Based Costing of Suppliers

Kinnear Plastics manufactures various components for the aircraft and marine industry. Kinnear buys plastic from two vendors: Tappan Corporation and Hill Enterprises. Kinnear chooses the vendor based on price. Once the plastic is received, it is inspected to ensure that it is suitable for production. Plastic that is deemed unsuitable is disposed of. The controller at Kinnear collected the following information on purchases for the past year:

 

Tappan

Hill

Total purchases (tons)

2,200

4,100

Plastic discarded

110

410

The purchasing manager has just received bids on an order for 150 tons of plastic from both Tappan and Hill. Tappan bid $741 and Hill bid $720 per ton.

Required

Assume that the average quality, measured by the amounts discarded from the two companies, will continue as in the past. Which supplier would you recommend that the purchasing manager select? Explain.

resources used versus resources supplied tri state mill uses a special sander to fin 691154

Resources Used versus Resources Supplied

Tri State Mill uses a special sander to finish lumber. Data on the sander and its usage follow:

 

Cost Driver Rate

Cost Driver Volume

Resources used

 

 

Energy

$ 0.90 per machine hour

6,000 machine hours

Repairs

$16.00 per job

600 jobs

Resources supplied

 

 

Energy

$ 6,900

 

Repairs

12,000

 

Required

Compute unused resource capacity in energy and repairs for Tri State Mill.

resources used versus resources supplied gundy press reports the following informati 691155

Resources Used versus Resources Supplied

Gundy Press reports the following information about resources:

 

Cost Driver Rate

Cost Driver Volume

Resources used

 

 

Setups

$ 125 per run

175 runs

Clerical

15 per page

500 pages typed

Resources supplied

 

 

Setups

$22,500

 

Clerical

10,000

 

Required

Compute unused setup and clerical resource capacity for Gundy Press.

resources used versus resources supplied gunnison supply provides the following info 691156

Resources Used versus Resources Supplied

Gunnison Supply provides the following information about resources:

 

Cost Driver Rate

Cost Driver Volume

Resources used

 

 

Materials

$ 12 per pound

4,000 pounds

Energy

48 per machine hour

170 machine hours

Setups

300 per setup

40 setups

Purchasing

240 per purchase order

40 purchase orders

Customer service

160 per return

25 returns

Long term labor

80 per labor hour

160 labor hours

Administrative

60 per labor hour

210 labor hours

Resources supplied

 

 

Materials

$48,000

 

Energy

9,120

 

Setups

12,600

 

Purchasing

11,000

 

Customer service

8,800

 

Long term labor

13,250

 

Administrative

13,500

 

In addition, sales for the period totaled $150,000.

Required

a. Compute unused resource capacity for each resource.

b. Describe what the term unused resource capacity means.

assigning cost of capacity beth rsquo s supplies manufactures building tiles in one 691157

Assigning Cost of Capacity

Beth’s Supplies manufactures building tiles in one plant, which has a practical capacity of 25,000 tiles. The variable cost of the tile is $9.00 per unit, and the fixed costs of the plant are $300,000 annually. Current annual demand is 20,000 tiles. Beth bought the current plant because she expected that demand for the tiles would grow once her reputation was established.

Required

a. What cost per tile should the cost system report?

b. Given your answer to requirement (a), is there any cost of excess capacity? If yes, what is the cost of excess capacity and how should it be reported? If no, why not?

c. How would your answers to requirements (a) and (b) change if the smallest tile manufacturing plant that one could build (owing to technology) was able to produce 25,000 tiles?

costs of quality the following represents the financial information for trovatore co 691160

Costs of Quality

The following represents the financial information for Trovatore Corporation, a manufacturer of electronic components, for two months:

 

March

April

Sales

$245,000

$220,000

Costs

 

 

Process inspection

$ 1,650

$ 1,880

Scrap

1,850

1,930

Quality training

19,800

13,000

Warranty repairs

4,300

4,800

Testing equipment

7,000

7,000

Customer complaints

2,800

3,400

Rework

17,000

18,500

Preventive maintenance

13,500

9,500

Materials inspection

6,500

4,800

Field testing

9,400

12,400

Required

a. Classify these items into prevention (P), appraisal (A), internal failure (IF), or external failure (EF) costs.

b. Calculate the ratio of the prevention, appraisal, internal failure, and external failure costs to sales for March and April.

costs of quality nuke it now manufactures microwave ovens the following represents t 691161

Costs of Quality

Nuke It Now manufactures microwave ovens. The following represents the financial information from one of its manufacturing plants for two years.

 

Year 1

Year 2

Sales

$3,500,000

$3,800,000

Costs

 

 

Redesign process

$ 29,000

$ 37,000

Discard defective units

37,000

43,000

Training on equipment

250,000

210,000

Warranty claims

129,000

176,000

Contract cancellations

201,000

154,000

Rework

72,000

96,000

Preventive maintenance

114,000

152,000

Product liability claims

302,000

176,000

Final inspection

190,000

198,000

Required

a. Classify these items into prevention (P), appraisal (A), internal failure (IF), or external failure (EF) costs.

b. Calculate the ratio of the prevention, appraisal, internal failure, and external failure costs to sales for year 1 and year 2.

cost of quality environmental issues many companies have adapted the cost of quality 691162

Cost of Quality: Environmental Issues

Many companies have adapted the cost of quality framework to environmental issues. They assign costs to one of four categories: prevention (P), appraisal (A), internal failure (IF), and external failure (EF), where the categories refer to environmental activities and consequences of environmental failures. Classify the following costs incurred for environmental activities into the four categories.

a. Criminal penalties for illegal dumping.

b. Cleanup of leaks and spills on the plant floor.

c. Employee training: environmental policies.

d. Lost sales from bad publicity after toxic spill.

e. Fines for being out of compliance with environmental regulations.

f. Maintenance of machinery that handles hazardous material.

g. Monitoring costs of chemical processes.

h. Design of processes to minimize leakage and waste.

job costing process costing choosing a costing method bouwens corporation manufactur 691115

Job Costing, Process Costing, Choosing a Costing Method

Bouwens Corporation manufactures a solvent used in airplane maintenance shops. Bouwens sells the solvent to both U.S. military services and commercial airlines. The solvent is produced in a single plant in one of two buildings. Although the solvent sold to the military is chemically identical to that sold to the airlines, the company produces solvent for the two customer types in different buildings at the plant. The solvent sold to the military is manufactured in building 155 (B 155) and is labeled M Solv. The solvent sold to the commercial airlines is manufactured in building 159 (B 159) and is labeled C Solv. B 155 is much newer and is considered a model work environment with climate control and other amenities. Workers at Bouwens, who all have roughly equal skills, bid on their job locations (the buildings they will work in) and are assigned based on bids and seniority. As workers gain seniority, they also receive higher pay.

The solvent sold to the two customers is essentially identical, but the military requires Bouwens to use a base chemical with a brand name, MX. The solvent for the commercial airlines is called CX.

MX is required for military applications because it is sold by vendors on a preferred vendor list. The company sells solvent for the market price to the airlines. Solvent sold to the military is sold based on cost plus a fixed fee. That is, the government pays Bouwens for the recorded cost of the solvent plus a fixed amount of profit t. The cost can be computed according to “commonly used product cost methods, including job costing or process costing methods using either FIFO or weighted average methods.” Competition for the government business is very strong and Bouwens is always looking for ways to reduce the cost and the price it quotes the government.

Currently, Bouwens uses a job costing system in which each month’s production for each customer type is considered a “job.” Thus, every month, Bouwens starts and completes one job in B 155 and one job in B 159. (There is never any beginning or ending work in process at Bouwens.) Recently, a dispute arose between Jack, the product manager for themilitary solvent, and Jill, the product manager for the commercial solvent, over the proper costing system.

Jack: It is ridiculous to use job costing for this. We are producing solvent. Everyone knows that the chemicals are the same. The fact the B 155 has high cost labor is because all the senior employees want to work there. We could produce the same product with the employees in B 159. We should be using process costing and consider all the production, in both buildings for each month, as the batch.

Jill: Jack, the fact is that the military requires us to use a special chemical and their contracts require we keep track of the costs for their business. If we don’t separate the costing, we won’t know how profit table either business is.

The following is production and cost information for a typical month, July:

 

M Solv (B 155)

C Solv (B 159)

Total

Units started

2,000

10,000

12,000

Materials cost

$14,000

$ 40,000

$ 54,000

Conversion cost

30,000

120,000

 150,000

Total

$44,000

$160,000

$204,000

Required

a. Compute the unit costs of M Solv and C Solv for July using the current system (job costing) at Bouwens.

b. Compute the costs of M Solv and C Solv for July if Bouwens were to treat all production as the same (combining B 155 and B 159 production).

c. Recommend a costing method that best reflects the cost of producing M Solv and C Solv.

d. For your recommended costing system, compute the cost of both M Solv and C Solv for July.

mesa consultants has projects with both private and government clients its overhead 691116

Mesa Consultants has projects with both private and government clients. Its overhead consists of both negotiations costs (including proposal preparation) and general administrative costs. Mesa has two groups, private and government, each working solely with one type of client. The following estimated operating data are available for next year:

 

Government

Private

Direct costs

$4,000,0000

$1,000,000

Number of contracts

20

30

Costs in the two administrative departments are expected to be as follows:

Negotiations

$ 750,000

General administrative

850,000

Total

$1,600,00

Mesa management is considering two approaches to assigning overhead costs to projects. One is to use a plantwide rate based on direct costs. The second is to use separate rates for the negotiations and general administrative costs. Under this method, negotiations costs will be allocated based on the number of contracts, and general administrative costs will be allocated based on the direct costs.

Compute the overhead costs allocated to the two groups using (1) the plantwide rate and (2) the individual department rates. What are the advantages and disadvantages of the separate rates compared to the plantwide rate?

the production supervisor at the port arthur manufacturing facility has given more t 691117

The production supervisor at the Port Arthur Manufacturing facility has given more thought to the appropriate cost driver for setups and believes it should be production runs, not setup hours. Compute the activity based costs for the two camera models at Joplin assuming an activity based costing system where setup cost is allocated based on the number of production runs. Use data from Exhibits 9.13 and 9.14.

 

A

B

C

D

E

1

 

 

Cost Driver Volume

2

Activity

Cost Driver

J25P

J40X

Total

3

Assembly building

 

 

 

 

4

Assembling

Machine hours

6,000

30,000

36,000

5

Setting up machines

Setup hours

40

400

440

6

Handling material

Production runs

8

40

48

7

Packaging building

 

 

 

 

8

Inspecting and packing

Direct labor hours

60,000

22,800

82,800

9

Shipping

Number of shipments

100

200

300

10

 

 

 

 

 

 

Exhibit 9.13 Cost Drivers and Cost Driver Volumes—Port Arthur Manufacturing Facility

 

 

A

B

C

D

E

F

G

H

1

 

Overhead

÷

 

Cost Driver

=

 

Cost Driver

2

Building and Activity

Cost

 

 

Volume

 

 

Rate

3

Assembly building

 

 

 

 

 

 

 

4

Assembling

$ 1,080,000

÷

36,000

machine hours

=

$

30

per machine hour

5

Setting up machines

396,000

÷

440

setup hours

=

$

900

per setup hour

6

Handling material

144,000

÷

48

production runs

=

$ 3,000

per production run

7

Total Assembly building overhead

$ 1,620,000

 

 

 

 

 

 

8

 

 

 

 

 

 

 

 

9

Packaging building

 

 

 

 

 

 

 

10

Inspecting and packing

$

414,000

÷

82,800

direct labor hours

=

$

5

per direct labor hour

11

Shipping

396,000

÷

300

shipments

=

$ 1,320

per shipment

12

Total Packaging building overhead

$

810,000

 

 

 

 

 

 

13

Total overhead

$ 2,430,000

 

 

 

 

 

 

14

 

 

 

 

 

 

 

 

Exhibit 9.14 Third Quarter Overhead Cost Data—Port Arthur Manufacturing Facility

[[Exercises]]

plantwide versus department allocation munoz sporting equipment manufactures basebal 691118

Plantwide versus Department Allocation

Munoz Sporting Equipment manufactures baseball bats and tennis rackets. Department B produces the baseball bats, and Department T produces the tennis rackets. Munoz currently uses plant wide allocation to allocate its overhead to all products. Direct labor cost is the allocation base. The rate used is 200 percent of direct labor cost. Last year, revenue, materials, and direct labor were as follows:

 

Baseball Bats

Tennis Rackets

Revenue

$1,350,000

$900,000

Direct labor

250,000

125,000

Direct materials

550,000

275,000

Required

a. Compute the profit t for each product using plantwide allocation.

b. Maria, the manager of Department T, was convinced that tennis rackets were really more profitable than baseball bats. She asked her colleague in accounting to break down the overhead costs for the two departments. She discovered that had department rates been used, Department B would have had a rate of 150 percent of direct labor cost and Department T would have had a rate of 300 percent of direct labor cost. Recompute the profits for each product using each department’s allocation rate (based on direct labor cost).

c. Why are the results different in requirements (a) and ( b )?

activity based costing in a nonmanufacturing environment cathy the manager of cathy 691121

Activity Based Costing in a Nonmanufacturing Environment

Cathy, the manager of Cathy’s Catering, Inc., uses activity based costing to compute the costs of her catered parties. Each party is limited to 20 guests and requires four people to serve and clean up. Cathy offers two types of parties, an afternoon picnic and an evening formal dinner. The breakdown of the costs follows:

Activities (and cost drivers)

Afternoon Picnic

Formal Dinner

Advertising (parties)

$80 per party

$80 per party

Planning (parties)

$60 per party

$100 per party

Renting equipment

$40 per party plus

$60 per party plus

(parties, guests)

$8 per guest

$16 per guest

Obtaining insurance (parties)

$160 per party

$320 per party

Serving (parties, servers)

$40 per server per party

$60 per server per party

Preparing food (guests)

$16 per guest

$24 per guest

Per party costs do not vary with the number of guests.

Required

a. Compute the cost of a 20 guest afternoon picnic.

b. Compute the cost of a 20 guest evening formal dinner.

c. How much should Cathy charge for each guest for each type of party if she wants to cover her costs?

activity based versus traditional costing rodent corporation produces two types of c 691122

Activity Based versus Traditional Costing

Rodent Corporation produces two types of computer mice, wired and wireless. The wired mice are designed as low cost, reliable input devices. The company only recently began producing the higher quality wireless model. Since the introduction of the new product, profits have been steadily declining. Management believes that the accounting system is not accurately allocating costs to products, particularly because sales of the new product have been increasing.

Management has asked you to investigate the cost allocation problem. You find that manufacturing overhead is currently assigned to products based on their direct labor costs. For your investigation, you have data from last year. Manufacturing overhead was $330,000 based on production of 160,000 wired mice and 50,000 wireless mice. Direct labor and direct materials costs were as follows:

 

Wired

Wireless

Total

Direct labor

$261,000

$ 99,000

$360,000

Materials

187,500

171,000

358,500

Management has determined that overhead costs are caused by three cost drivers. These drivers and their costs for last year are as follows:

Cost Driver

Costs Assigned

Activity Level Wired Wireless

Total

Number of production runs

$ 150,000

20 5

25

Quality tests performed

135,000

6 9

15

Shipping orders processed

45,000

50 25

75

Total overhead

$ 330,000

 

 

Required

a. How much overhead will be assigned to each product if these three cost drivers are used to allocate overhead? What is the total cost per unit produced for each product?

b. How much overhead will be assigned to each product if direct labor cost is used to allocate overhead? What is the total cost per unit produced for each product?

c. How might the results from using activity based costing in requirement (a) help management understand Rodent’s declining profits?

activity based costing versus traditional costing doaktown products manufactures fis 691123

Activity Based Costing versus Traditional Costing

Doaktown Products manufactures fishing equipment for recreational uses. The Miramichi plant produces the company’s two versions of a special reel used for river fishing. The two models are the M 008, a basic reel, and the M 123, a new and improved version. Cost accountants at company headquarters have prepared costs for the two reels for the most recent period. The plant manager is concerned. The cost report does not coincide with her intuition about the relative costs of the two models. She has asked you to review the cost accounting and help her prepare a response to headquarters.

Manufacturing overhead is currently assigned to products based on their direct labor costs. For the most recent month, manufacturing overhead was $280,000. During that time, the company produced 12,000 units of the M 008 and 2,000 units of the M 123. The direct costs of production were as follows:

 

M 008

M 123

Total

Direct materials

$100,000

$80,000

$180,000

Direct labor

100,000

40,000

140,000

Management determined that overhead costs are caused by three cost drivers. These drivers and their costs for last year were as follows:

Cost Driver

Costs

 

Activity Level

 

M 008

M 123

Total

Number of machine hours

$120,000

5,000

3,000

8,000

Number of production runs

70,000

10

10

20

Number of inspections

90,000

20

40

60

Total overhead

$280,000

 

 

 

Required

a. How much overhead will be assigned to each product if these three cost drivers are used to allocate overhead? What is the total cost per unit produced for each product?

b. How much of the overhead will be assigned to each product if direct labor cost is used to allocate overhead? What is the total cost per unit produced for each product?

c. Draft a memo for the plant manager explaining why the two systems result in different costs along with your recommendation for which costing system to use.

activity based costing in a service environment we clean inc is a home cleaning serv 691124

Activity Based Costing in a Service Environment We Clean, Inc., is a home cleaning service. It originally specialized in serving small residential clients but recently started contracting for work in large apartment and office buildings. Julie Lodge, the owner, believes that the commercial sector has more growth opportunities and is considering dropping the residential service.

Twenty cleaning employees worked a total of 40,000 hours last year, 26,000 on residential jobs and 14,000 on commercial jobs. Wages were $15 per hour for all work done. Any materials used are included in overhead as supplies. All overhead is allocated on the basis of labor hours worked, which is also the basis for customer charges. Because of increased competition for commercial accounts, Julie can charge $35 per hour for residential work, but only $27 per hour for commercial work.

Required

a. If overhead for the year was $124,000, what were the profits of the commercial and the residential services using labor hours as the allocation base?

b. Overhead consists of costs of traveling, using equipment, and using supplies, which can be traced as follows:

     

Cost Driver Volume

Activity

Cost Driver

Cost

Commercial

Residential

Traveling

Number of clients served

$ 16,000

17

47

Using equipment

Equipment hours

36,000

3,750

2,250

Using supplies

Area serviced in square yards

72,000

130,000

70,000

Total overhead

$124,000

   

Recalculate profits for commercial and residential services based on these activity bases.

c. What recommendations do you have for management regarding the profitability of these two types of services?

activity based versus traditional costing isadore rsquo s implements inc manufacture 691125

Activity Based versus Traditional Costing

Isadore’s Implements, Inc., manufactures pens and mechanical pencils often used for gifts. Overhead costs are currently allocated using direct labor hours, but the controller has recommended an activity based costing system using the following data:

     

Cost Driver Volume

Activity

Cost Driver

Cost

Pencils

Pens

Setting up

Number of setups

$ 80,000

20

30

Inspecting

Number of parts

24,000

4

6

Packing and shipping

Number of boxes shipped

48,000

45,000

75,000

Total overhead

$152,000

   

Required

a. Compute the amount of overhead to be allocated to each product under activity based costing.

b. Compute the amount of overhead to be allocated to each product using labor hours as the allocation base. Assume that the number of labor hours required to assemble each box is 0.1 for pencils and 0.2 for pens and that 45,000 boxes of pencils and 75,000 boxes of pens were produced during the period.

c. Should the company follow the controller’s recommendations?

activity based versus traditional costing mdash ethical issues wendy chen establishe 691126

Activity Based versus Traditional Costing—Ethical Issues

Wendy Chen established Windy City Coaching (WCC) to provide teen counseling and executive coaching services to its clients. WCC charges a $300 fee per hour for each service. The revenues and costs for the year are shown in the following income statement:

WINDY CITY COACHING

Income Statement

Teen

Executive

 

Counseling

Coaching

Total

Revenue

$66,000

$135,000

$201,000

Expenses:

 

 

 

Administrative support

 

 

40,000

Transportation, etc

 

 

36,000

Equipment

 

 

20,000

Profit

 

 

$105,000

WCC has kept good records of the following data for cost allocation purposes:

   

Activity Level

Activity

Cost Driver

Teen

Executive

Providing administrative support

Number of clients

Counseling

Coaching

Traveling

Number of visits

6

4

Using equipment

Computer hours

100

150

   

900

700

Required

a. Complete the income statement using activity based costing and WCC’s three cost drivers.

b. Recompute the income statement using direct labor hours as the only allocation base (220 hours for teen counseling; 450 hours for executive coaching).

c. How might WCC’s decisions regarding pricing or dropping a service be altered if Wendy were to allocate all overhead costs using direct labor hours?

d. Under what circumstances would the labor based allocation and activity based costing (using Wendy’s three cost drivers) result in similar profit t results?

e. A local nonprofit t charity is looking for worthy causes to support through financial grants. A primary criterion for support is financial need. Wendy is thinking of applying for support for the teen counseling program. Which allocation method would give her the best chance of winning a grant? Would it be ethical for Wendy to report the income using this method in her application?

activity based costing cost flows through t accounts delta parts inc recently switch 691127

Activity Based Costing: Cost Flows through T Accounts

Delta Parts, Inc., recently switched to activity based costing from the department allocation method. The Fabrication Department manager has estimated the following cost drivers and rates:

 

 

Rate per

Activity Centers

Cost Drivers

Cost Driver Unit

Materials handling

Pounds of material handled

$18 per pound

Quality inspections

Number of inspections

$225 per inspection

Machine setups

Number of machine setups

$2,700 per setup

Running machines

Number of machine hours

$22.50 per hour

Direct materials costs were $300,000 and direct labor costs were $150,000 during July, when the Fabrication Department handled 3,750 pounds of materials, made 750 inspections, had 40 setups, and ran the machines for 15,000 hours.

Required

Use T accounts to show the flow of materials, labor, and overhead costs from the four overhead activity centers through Work in Process Inventory and out to Finished Goods Inventory. Use the accounts Materials Inventory, Wages Payable, Work in Process Inventory, Finished Goods Inventory, and four overhead applied accounts.

activity based costing cost flows through t accounts carolina fashions a shirt manuf 691128

Activity Based Costing: Cost Flows through T Accounts

Carolina Fashions, a shirt manufacturer, recently switched to activity based costing from the department product costing method. The manager of Building S, which manufactures the shirts, has identified the following cost drivers and rates for overhead:

 

 

Rate per

Activity Centers

Cost Drivers

Cost Driver Unit

Materials handling

Yards of material handled

$1 per yard

Quality inspections

Number of inspections

$100 per inspection

Machine setups

Number of machine setups

$800 per setup

Running machines

Number of machine hours

$10 per hour

Direct materials costs were $200,000 and direct labor costs were $100,000 during October, when Building S handled 40,000 yards of materials, made 800 inspections, had 100 setups, and ran the machines for 20,000 hours.

Required

Use T accounts to show the flow of materials, labor, and overhead costs from the four overhead activity centers through Work in Process Inventory and out to Finished Goods Inventory. Use the accounts Materials Inventory, Wages Payable, Work in Process Inventory, Finished Goods Inventory, and four overhead applied accounts.

activity based costing for an administrative service the personnel department at las 691129

Activity Based Costing for an Administrative Service

The personnel department at LastCall Enterprises handles many administrative tasks for the two divisions that make up LastCall: LaidBack and StressedOut. LaidBack division manages the company’s traditional business line. This business, although lucrative, is currently not growing. StressedOut, on the other hand, is the company’s new business, which has experienced double digit growth for each of the last three years.

The cost allocation system at LastCall allocates all corporate costs to the divisions based on a variety of cost allocation bases. Personnel costs are allocated based on the average number of employees in the two divisions.

There are two basic activities in the personnel department. The first, which is called employee maintenance, manages employee records. Virtually all of this activity occurs when employees are hired or leave the company. The other activity is payroll, which is an ongoing activity and requires the same amount of work for each employee regardless of the employee’s salary. Assorted data for LastCall for the last year follow:

 

LaidBack

StressedOut

Total

Number of employees (average)

100

25

125

Employees hired/leaving

5

15

20

The personnel department incurred the following costs during the year:

Employee maintenance

$ 120,000

Payroll

17,500

Total

$ 137,500

Required

a. Under the current allocation system, what are the costs that will be allocated from personnel to LaidBack? To StressedOut?

b. Suppose the company implements an activity based cost system for personnel with the two activities, employee maintenance and payroll. Use the number of employees hired/leaving as the cost driver for employee maintenance costs and the average number of employees for payroll costs. What are the costs that will be allocated from personnel to LaidBack? To StressedOut?

comparative income statements and management analysis ez seat inc manufactures two t 691131

Comparative Income Statements and Management Analysis

EZ Seat, Inc., manufactures two types of reclining chairs, Standard and Ergo. Ergo provides support for the body through a complex set of sensors and requires great care in manufacturing to avoid damage to the material and frame. Standard is a conventional recliner, uses standard materials, and is simpler to manufacture. EZ Seat’s results for the last fiscal year are shown in the following statement.

EZ SEAT, INC.

 

 

Income Statement

 

 

Ergo

Standard

Total

Revenue

$1,950,000

$1,840,000

$3,790,000

Direct materials

550,000

500,000

1,050,000

Direct labor

400,000

200,000

600,000

Overhead costs

 

 

 

Administration

 

 

234,000

Production setup

 

 

540,000

Quality control

 

 

360,000

Distribution

 

 

720,000

Operating profit

 

 

$ 286,000

EZ Seat currently uses labor costs to allocate all overhead, but management is considering implementing an activity based costing system. After interviewing the sales and production staff, management decides to allocate administrative costs on the basis of direct labor costs but to use the following bases to allocate the remaining costs:

 

 

Activity Level

Activity Base

Cost Driver

Ergo

Standard

Setting up

Number of production runs

50

100

Performing quality control

Number of inspections

200

200

Distribution

Number of units shipped

1,500

6,000

Required

a. Complete the income statement using the preceding activity bases.

b. Write a brief report indicating how management could use activity based costing to reduce costs.

c. Restate the income statement for EZ Seat using direct labor costs as the only overhead allocation base.

d. Write a report to management stating why product line profits differ using activity based costing compared to the traditional approach. Indicate whether activity based costing provides more accurate information and why (if you believe it does provide more accurate information). Indicate in your report how the use of labor based overhead allocation could cause EZ Seat management to make suboptimal decisions.

comparative income statements and management analysis bob rsquo s baskets inc manufa 691132

Comparative Income Statements and Management Analysis

Bob’s Baskets, Inc., manufactures and sells two types of baskets, deluxe and standard. Last year, Bob’s Baskets had the following costs and revenues:

BOB’S BASKETS, INC.

Income Statement

 

Deluxe

Standard

Total

Revenue

$216,000

$240,000

$ 456,000

Direct materials

20,000

20,000

40,000

Direct labor

48,000

72,000

120,000

Overhead costs Administration

 

 

30,000

Production setup

 

 

60,000

Quality control

 

 

30,000

Distribution

 

 

24,000

Operating profit

 

 

$ 152,000

Bob’s Baskets currently uses labor costs to allocate all overhead but is considering implementing an activity based costing system. After interviewing the sales and production staff, management decides to allocate administrative costs on the basis of direct labor costs but to use the following bases to allocate the remaining overhead:

 

 

Activity Level

Activity

Cost Driver

Deluxe

Standard

Setting up

Number of production runs

20

10

Performing quality control

Number of inspections

60

20

Distribution

Number of units shipped

80,000

120,000

Required

a. Complete the income statement using the preceding activity bases.

b. Write a report indicating how management might use activity based costing to reduce costs.

c. Restate the income statement for Bob’s Baskets using direct labor costs as the only overhead allocation base.

d. Write a report to management stating why product line profits differ using activity based costing compared to the traditional approach. Indicate whether activity based costing provides more accurate information and why (if you believe it does provide more accurate information). Indicate in your report how the use of labor based overhead allocation could cause management at Bob’s Baskets to make suboptimal decisions.

ethics and choice of accounting methods bob rsquo s baskets inc manufactures and sel 691133

Ethics and Choice of Accounting Methods

Bob’s Baskets, Inc., manufactures and sells two types of baskets, deluxe and standard. Last year, Bob’s Baskets had the following costs and revenues:

BOB’S BASKETS, INC.

Income Statement

 

Deluxe

Standard

Total

Revenue

$216,000

$240,000

$ 456,000

Direct materials

20,000

20,000

40,000

Direct labor

48,000

72,000

120,000

Overhead costs Administration

 

 

30,000

Production setup

 

 

60,000

Quality control

 

 

30,000

Distribution

 

 

24,000

Operating profit

 

 

$ 152,000

Bob’s Baskets currently uses labor costs to allocate all overhead but is considering implementing an activity based costing system. After interviewing the sales and production staff, management decides to allocate administrative costs on the basis of direct labor costs but to use the following bases to allocate the remaining overhead:

 

 

Activity Level

Activity

Cost Driver

Deluxe

Standard

Setting up

Number of production runs

20

10

Performing quality control

Number of inspections

60

20

Distribution

Number of units shipped

80,000

120,000

 Assume that you have prepared financial statements that show the operating profit for each of the two baskets manufactured by Bob’s Baskets. Further assume that under the activity based costing approach (requirement [ a ], the deluxe baskets are less profitable than when you used only direct labor costs to allocate overhead costs requirement [ c ] in You know that if management sees the activity based costing results, the company will probably quit producing deluxe baskets. You have friends who work on the production and marketing of deluxe baskets and you believe that they would lose their jobs if management dropped this line.

Required

Should you show the activity based costing results to management?

activity based costing and predetermined overhead allocation rates kitchen supply in 691134

Activity Based Costing and Predetermined Overhead Allocation Rates

Kitchen Supply, Inc. (KSI), manufactures three types of flatware: institutional, standard, and silver. It applies all indirect costs according to a predetermined rate based on direct labor hours. A consultant recently suggested that the company switch to an activity based costing system and prepared the following cost estimates for year 2 for the recommended cost drivers.

 

Recommended

Estimated

Estimated Cost

Activity

Cost Driver

Cost

Driver Activity

Processing orders

Number of orders

$ 54,000

200 orders

Setting up production

Number of production runs

216,000

100 runs

Handling materials

Pounds of materials used

360,000

120,000 pounds

Machine depreciation

 

 

 

and maintenance

Machine hours

288,000

12,000 hours

Performing quality control

Number of inspections

72,000

45 inspections

Packing

Number of units

144,000

480,000 units

Total estimated cost

 

$1,134,000

 

In addition, management estimated 7,500 direct labor hours for Year 2.

Assume that the following cost driver volumes occurred in January Year 2:

 

Institutional

Standard

Silver

Number of units produced

60,000

24,000

9,000

Direct materials costs

$39,000 450

 $24,000 450

 $15,000

Direct labor hours

 

 

 600

Number of orders

12

9

6

Number of production runs

3 15,000 580

3 6,000 140

6 3,000

Pounds of material

 

 

 80

Machine hours

 

 

 

Number of inspections

3 60,000

3 24,000

3 9,000

Units shipped

 

 

 

Actual labor costs were $15 per hour.

Required

a. Compute a predetermined overhead rate for year 2 for each cost driver using the estimated costs and estimated cost driver units prepared by the consultant. Also compute a predetermined rate for year 2 using direct labor hours as the allocation base.

b. Compute the production costs for each product for January using direct labor hours as the allocation base and the predetermined rate computed in requirement (a).

c. Compute the production costs for each product for January using the cost drivers recommended by the consultant and the predetermined rates computed in requirement (a).

d. Management has seen your numbers and wants an explanation for the discrepancy between the product costs using direct labor hours as the allocation base and the product costs using activity based costing. Write a brief response to management.

activity based costing and predetermined overhead rates college supply company csc m 691135

Activity Based Costing and Predetermined Overhead Rates

College Supply Company (CSC) makes three types of drinking glasses: short, medium, and tall. It presently applies overhead using a predetermined rate based on direct labor hours. A group of company employees recommended that CSC switch to activity based costing and identified the following activities, cost drivers, estimated costs, and estimated cost driver units for Year 5 for each activity center.

 

 

Recommended

Estimated

Estimated Cost

Activity

Cost Driver

Cost

Driver Units

Setting up production

Number of production runs

$ 24,000

100 runs

Processing orders

Number of orders

40,000

200 orders

Handling materials

Pounds of materials

16,000

8,000 pounds

Using machines

Machine hours

48,000

10,000 hours

Providing quality management

Number of inspections

40,000

40 inspections

Packing and shipping

Units shipped

32,000

20,000 units

 

 

$200,000

 

In addition, management estimated 2,000 direct labor hours for year 5.

Assume that the following cost driver volumes occurred in February Year 5:

 

Short

Medium

Tall

Number of units produced

1,000

500

400

Direct materials costs

$4,000

$2,500

$2,000

Direct labor hours

100

120

110

Number of orders

8

8

4

Number of production runs

2

4

8

Pounds of material

400

800

200

Machine hours

500

300

300

Number of inspections

2

2

2

Units shipped

1,000

500

300

Direct labor costs were $20 per hour.

Required

a. Compute a predetermined overhead rate for year 5 for each cost driver recommended by the employees. Also compute a predetermined rate using direct labor hours as the allocation base.

b. Compute the production costs for each product for February using direct labor hours as the allocation base and the predetermined rate computed in requirement (a).

c. Compute the production costs for each product for February using the cost drivers recommended by the employees and the predetermined rates computed in requirement (a). ( Note: Do not assume that total overhead applied to products in February will be the same for activity based costing as it was for the labor hour based allocation.)

d. Management has seen your numbers and wants an explanation for the discrepancy between the product costs using direct labor hours as the allocation base and the product costs using activity based costing. Write a brief response to management.

choosing an activity based costing system pickle motorcycles inc pmi manufactures th 691136

Choosing an Activity Based Costing System

Pickle Motorcycles, Inc. (PMI), manufactures three motorcycle models: a cruising bike (Route 66), a street bike (Main Street), and a starter model (Alley Cat). Because of the different materials used, production processes for each model differ significantly in terms of machine types and time requirements. Once parts are produced, however, assembly time per unit required for each type of bike is similar. For this reason, PMI allocates overhead on the basis of machine hours. Last year, the company shipped 1,000 Route 66s, 4,000 Main Streets, and 10,000 Alley Cats and had the following revenues and expenses:

PICKLE MOTORCYCLES, INC.

Income Statement

 

Route 66

Main Street

Alley Cat

Total

Sales

$7,600,000

$11,200,000

$9,500,000

$28,300,000

Direct costs

 

 

 

 

Direct materials

3,000,000

4,800,000

4,000,000

11,800,000

Direct labor

288,000

480,000

1,080,000

1,848,000

Variable overhead

 

 

 

 

Machine setup

 

 

 

468,000

Order processing

 

 

 

1,152,000

Warehousing costs

 

 

 

1,674,000

Energy to run machines

 

 

 

756,000

Shipping

 

 

 

648,000

Contribution margin

 

 

 

$ 9,954,000

Fixed overhead

 

 

 

 

Plant administration

 

 

 

1,760,000

Other fixed overhead

 

 

 

2,800,000

Gross profit

 

 

 

$ 5,394,000

PMI’s chief financial officer (CFO) hired a consultant to recommend cost allocation bases. The consultant recommended the following:

Activity

Cost Driver

Activity Level

 

Route 66 Main Street

Alley Cat

Setting up machines

Number of production runs

22 34

44

Processing orders

Number of sales orders received

400 600

600

Warehousing

Number of units held in inventory

200 200

400

Using energy

Machine hours

10,000 16,000

24,000

Shipping

Number of units shipped

1,000 4,000

10,000

The consultant found no basis for allocating the plant administration and other fixed overhead costs and recommended that these not be applied to products.

Required

a. Using machine hours to allocate production overhead, complete the income statement for Pickle Motorcycles. (See the “using energy” activity for machine hours.) Do not attempt to allocate plant administration or other fixed overhead.

b. Complete the income statement using the bases recommended by the consultant.

c. How might activity based costing result in better decisions by Pickle Motorcycles’s management?

d. After hearing the consultant’s recommendations, the CFO decides to adopt activity based costing but expresses concern about not allocating some of the overhead to the products (plant administration and other fixed overhead). In the CFO’s view, “Products have to bear a fair share of all overhead or we won’t be covering all of our costs.” How would you respond to this comment?

activity based costing cost flow diagram and predetermined overhead rates huron furn 691137

Activity Based Costing, Cost Flow Diagram, and Predetermined Overhead Rates

Huron Furniture is considering updating its cost system to an activity based costing system and is interested in understanding the effects. The company’s cost accountant has identified three overhead cost pools along with appropriate cost drivers for each pool.

Cost Pools

Costs

Activity Drivers

Utilities

$450,000

60,000 machine hours

Scheduling and setup

450,000

600 setups

Material handling

1,200,000

1,600,000 pounds of material

The company manufactures three models of tables (oval, round, and square). The plans for production for the next year and the budgeted direct costs and activity by product line are as follows:

 

 

Products

 

Oval

Round

Square

Total direct costs (material and labor)

$ 80,000

$ 80,000

$ 80,000

Total machine hours

30,000

10,000

20,000

Total number of setups

80

300

220

Total pounds of material

500,000

300,000

800,000

Total direct labor hours

3,200

1,800

5,000

Number of units produced

4,000

2,000

6,000

Required

a. The current cost accounting system charges overhead to products based on direct labor hours. What unit product costs will be reported for the three products if the current cost system continues to be used?

b. A consulting firm has recommended using an activity based costing system, with the activities based on the cost pools identified by the cost accountant. Prepare a cost flow diagram of the proposed ABC system.

c. What are the cost driver rates for the three cost pools identified by the cost accountant?

d. What unit product costs will be reported for the three products if the ABC system suggested by the cost accountant’s Classification of cost pools is used?

e. If management should decide to implement an activity based costing system, what benefits should it expect?

compute equivalent units fifo method materials are added at the beginning of the pro 691090

Compute Equivalent Units: FIFO Method

Materials are added at the beginning of the production process at Santiago Company, which uses a FIFO process costing system. The following information on the physical flow of units is available for the month of April: Cost data for April show the following:

Beginning WIP inventory

 

Direct materials costs

$ 24,300

Conversion costs

38,700

Current period costs

 

Direct materials costs

$ 280,950

Conversion costs

1,079,550

Required

a. Compute the cost equivalent units for the conversion cost calculation assuming Santiago uses the weighted average method.

b. Compute the cost per equivalent unit for materials and conversion costs for April.

compute costs per equivalent unit weighted average method the matsui lubricants plan 691091

Compute Costs per Equivalent Unit: Weighted Average Method

The Matsui Lubricants plant uses the weighted average method to account for its work in process inventories. The accounting records show the following information for a particular day:

Beginning WIP inventory Direct materials

$ 976

Conversion costs

272

Current period costs Direct materials

11,440

Conversion costs

6,644

Quantity information is obtained from the manufacturing records and includes the following:

Beginning inventory

600 units (60% complete as to materials,

 

53% complete as to conversion)

Current period units started

4,000 units

Ending inventory

1,200 units (40% complete as to materials,

 

20% complete as to conversion)

Required

Compute the cost per equivalent unit for direct materials and conversion costs.

compute costs per equivalent unit fifo method the matsui lubricants plant uses the w 691093

Compute Costs per Equivalent Unit: FIFO Method

The Matsui Lubricants plant uses the weighted average method to account for its work in process inventories. The accounting records show the following information for a particular day:

Beginning WIP inventory Direct materials

$ 976

Conversion costs

272

Current period costs Direct materials

11,440

Conversion costs

6,644

Quantity information is obtained from the manufacturing records and includes the following:

Beginning inventory

600 units (60% complete as to materials,

 

53% complete as to conversion)

Current period units started

4,000 units

Ending inventory

1,200 units (40% complete as to materials,

 

20% complete as to conversion)

Required

Compute the cost per equivalent unit for direct materials and for conversion costs using the FIFO method.

compute costs per equivalent unit weighted average method pacific ink had beginning 691094

Compute Costs per Equivalent Unit: Weighted Average Method

Pacific Ink had beginning work in process inventory of $372,480 on October 1. Of this amount, $152,460 was the cost of direct materials and $220,020 was the cost of conversion. The 48,000 units in the beginning inventory were 30 percent complete with respect to both direct materials and conversion costs.

During October, 102,000 units were transferred out and 30,000 remained in ending inventory. The units in ending inventory were 80 percent complete with respect to direct materials and 40 percent complete with respect to conversion costs. Costs incurred during the period amounted to $1,171,800 for direct materials and $1,513,920 for conversion.

Required

Compute the cost per equivalent unit for direct materials and for conversion costs using the weighted average method.

compute costs per equivalent unit weighted average method pacific ink had beginning 691095

Compute Costs per Equivalent Unit: Weighted Average Method

Pacific Ink had beginning work in process inventory of $372,480 on October 1. Of this amount, $152,460 was the cost of direct materials and $220,020 was the cost of conversion. The 48,000 units in the beginning inventory were 30 percent complete with respect to both direct materials and conversion costs.

During October, 102,000 units were transferred out and 30,000 remained in ending inventory. The units in ending inventory were 80 percent complete with respect to direct materials and 40 percent complete with respect to conversion costs. Costs incurred during the period amounted to $1,171,800 for direct materials and $1,513,920 for conversion.

Compute the costs of goods transferred out and the ending inventory using the weighted average method.

assign costs to goods transferred out and ending inventory fifo method pacific ink h 691097

Assign Costs to Goods Transferred Out and Ending Inventory: FIFO Method

Pacific Ink had beginning work in process inventory of $372,480 on October 1. Of this amount, $152,460 was the cost of direct materials and $220,020 was the cost of conversion. The 48,000 units in the beginning inventory were 30 percent complete with respect to both direct materials and conversion costs.

During October, 102,000 units were transferred out and 30,000 remained in ending inventory. The units in ending inventory were 80 percent complete with respect to direct materials and 40 percent complete with respect to conversion costs. Costs incurred during the period amounted to $1,171,800 for direct materials and $1,513,920 for conversion.

 Compute the cost of goods transferred out and the cost of ending inventory using the FIFO method. Is the ending inventory higher or lower under the weighted average method compared to FIFO? Why?

prepare a production cost report fifo method assume that el paso corporation provide 691098

Prepare a Production Cost Report: FIFO Method

Assume that El Paso Corporation provides you with the following information for one of its department’s operations for September (no new material is added in Department B):

WIP inventory—Department B

 

Beginning inventory (7,500 units, 20% complete with respect

 

to Department B costs)

 

Transferred in costs (from Department A)

$ 29,000

Department B conversion costs

7,675

Current work (17,500 units started)

 

Prior department costs

70,000

Department B costs

149,075

The ending inventory has 2,500 units, which are 50 percent complete with respect to Department B costs and 100 percent complete for prior department costs.

 

Required

Prepare a production cost report using FIFO.

prepare a production cost report weighted average method assume that el paso corpora 691099

Prepare a Production Cost Report: Weighted Average Method

Assume that El Paso Corporation provides you with the following information for one of its department’s operations for September (no new material is added in Department B):

WIP inventory—Department B

 

Beginning inventory (7,500 units, 20% complete with respect

 

to Department B costs)

 

Transferred in costs (from Department A)

$ 29,000

Department B conversion costs

7,675

Current work (17,500 units started)

 

Prior department costs

70,000

Department B costs

149,075

The ending inventory has 2,500 units, which are 50 percent complete with respect to Department B costs and 100 percent complete for prior department costs. 

 

Required

a. Prepare a production cost report using the weighted average method.

b. Is the ending inventory higher using FIFO or the weighted average method? Why?

c. Would you recommend that El Paso use the FIFO method or the weighted average method? Explain.

operation costing ethical issues brokia electronics manufactures three cell phone mo 691100

Operation Costing: Ethical Issues

Brokia Electronics manufactures three cell phone models, which differ only in the components included: Basic, Photo, and UrLife. Production takes place in two departments, Assembly and Special Packaging. The Basic and Photo models are complete after Assembly. The UrLife model goes from Assembly to Special Packaging and is completed there. Data for July are shown in the following table. Conversion costs are allocated based on the number of units produced. There are no work in process inventories.

 

Total

Basic

Photo

UrLife

Materials         

$2,240,000

(40,000 units)

(30,000 units)

(10,000 units)

Conversion costs:

 

$ 480,000

$1,200,000

$560,000

Assembly      

$1,400,000

     

Special Packaging

400000

     

Total conversion costs

$1,800,000

     

a. What is the cost per unit transferred to finished goods inventory for each of the three phones in July?

b. The UrLife model is sold only to the government on a cost plus basis. The marketing vice president suggests that conversion costs in Assembly could be allocated on the basis of material costs so he can offer a lower price for the Basic model.

1. What cost would be reported for the three models if the marketing vice president’s suggestion is adopted?

2. Would this be ethical?

operation costing ferdon watches inc makes four models of watches gag gift commuter 691101

Operation Costing

Ferdon Watches, Inc., makes four models of watches, Gag Gift, Commuter, Sport, and Retirement. Ferdon manufactures the watches in four departments: Assembly, Polishing, Special Finishing, and Packaging. All four models are started in Assembly where all material is assembled. The Gag Gift is transferred to Packaging, where it is packaged and transferred to finished goods inventory. The Commuter and Sport are assembled, then transferred to Polishing. Once the polishing process is completed, they are transferred to Packaging and then finished goods. The Retirement model is assembled and then transferred to Special Finishing, and then Packaging. When packaged, it is transferred to finished goods.

Data for October are shown in the following table. Conversion costs are allocated based on the number of units processed in each department.

   

Gag Gift

Commuter

Sport

Retirement

 

Total

(5,000 units)

(10,000 units)

(13,000 units)

(2,000 units)

Conversion costs:

         

Assembly

$120,000

$15,000

$90,000

$156,000

$60,000

Polishing

69,000

       

Special Finishing

20,000

       

Packaging

90000

       

Total conversion

         

costs

$299,000

       

a. Draw the flow of the different models through the production process.

b. What is the cost per unit transferred to finished goods inventory for each of the four watches in October?

compute equivalent units select the best answer for each of the following independen 691102

Compute Equivalent Units

Select the best answer for each of the following independent multiple choice questions.

a. Adams Company’s production cycle starts in Department A. The following information is available for July:

 

Units

Work in process, July 1 (60% complete)

150,000

Started in July

720,000

Work in process, July 31 (30% complete)

80,000

Materials are added at the beginning of the process in Department A. Using the weightedaverage method, what are the equivalent units of production for the month of July?

 

Materials

Conversion

(1)

720,000

744,000

(2)

870,000

814,000

(3)

734,000

720,000

(4)

795,000

734,000

(5)

None of the above

 

Department B is the second stage of Boswell Corporation’s production cycle. On November 1, beginning work in process contained 50,000 units, which were 30 percent complete as to conversion costs. During November, 320,000 units were transferred in from the first stage of the production cycle. On November 30, ending work in process contained 40,000 units, which were 65 percent complete as to conversion costs. Materials are added at the end of the process. Using the weighted average method, the equivalent units produced during November were as follows:

 

Prior

 

 

 

Department Costs

Materials

Conversion

(1)

370,000

330,000

304,000

(2)

370,000

330,000

356,000

(3)

370,000

330,000

345,000

(4)

320,000

330,000

356,000

(5)

None of the above

 

 

Department C is the first stage of Cohen Corporation’s production cycle. The following equivalent unit information is available for conversion costs for the month of September:

Beginning work in process inventory (30% complete)

20,000

Started in September

340,000

Completed in September and transferred to Department D

320,000

Ending work in process inventory (70% complete)

40,000

 

Using the FIFO method, the equivalent units for the conversion cost calculation are:

 

(1)

342,000

(2)

298,000

(3)

348,000

(4)

320,000

(5)

None of the above.

d. Draper Corporation computed the physical flow of units for Department D for the month of December as follows:

Units completed

 

From work in process on December 1

40,000

From December production

140,000

Total

180,000

Materials are added at the beginning of the process. Units of WIP at December 31 were 32,000. As to conversion costs, WIP at December 1 was 70 percent complete and WIP at December 31 was 50 percent complete. What are the equivalent units produced for the month of December using the FIFO method?

 

Materials

Conversion

(1)

172,000

172,000

(2)

212,000

200,000

(3)

212,000

204,000

(4)

172,000

168,000

(5)

None of the above

 

prepare a production cost report weighted average method douglas toys is a manufactu 691104

Prepare a Production Cost Report: Weighted Average Method

Douglas Toys is a manufacturer that uses the weighted average process costing method to account for costs of production. It produces a plastic toy in three separate departments: Molding, Assembling, and Finishing. The following information was obtained for the Assembling Department for the month of September.

Work in process on September 1 had 100,000 units made up of the following:

 

 

Degree of

 

Amount

Completion

Prior department costs transferred in from the Molding Department

$128,000

100%

Costs added by the Assembling Department

 

 

Direct materials

80,000

100%

Direct labor

28,800

60%

Manufacturing overhead

18,400

50%

 

$127,200

 

Work in process, September 1

$255,200

 

During September, 500,000 units were transferred in from the Molding Department at a cost of $640,000. The Assembling Department added the following costs:

Direct materials

$384,000

Direct labor

144,000

Manufacturing overhead

75,600

Total costs added

$603600

Assembling finished 400,000 units and transferred them to the Finishing Department. At September 30, 200,000 units were still in work in process inventory. The degree of completion of work in process inventory at September 30 was as follows:

Direct materials

90%

Direct labor

70

Manufacturing overhead

35

Required

a. Prepare a production cost report using the weighted average method.

b. Management would like to decrease the costs of manufacturing the toy. In particular, it has set the following per unit targets for this product in the Assembling Department: Materials, $0.80; labor, $0.40; and manufacturing overhead, $0.18. Has the product achieved management’s cost targets in the Assembling Department? Write a short report to management stating your answer(s).

prepare a production cost report fifo method douglas toys is a manufacturer that use 691105

Prepare a Production Cost Report: FIFO Method

Douglas Toys is a manufacturer that uses the weighted average process costing method to account for costs of production. It produces a plastic toy in three separate departments: Molding, Assembling, and Finishing. The following information was obtained for the Assembling Department for the month of September.

Work in process on September 1 had 100,000 units made up of the following:

 

 

Degree of

 

Amount

Completion

Prior department costs transferred in from the Molding Department

$128,000

100%

Costs added by the Assembling Department

 

 

Direct materials

80,000

100%

Direct labor

28,800

60%

Manufacturing overhead

18,400

50%

 

$127,200

 

Work in process, September 1

$255,200

 

During September, 500,000 units were transferred in from the Molding Department at a cost of $640,000. The Assembling Department added the following costs:

Direct materials

$384,000

Direct labor

144,000

Manufacturing overhead

75,600

Total costs added

$603600

Assembling finished 400,000 units and transferred them to the Finishing Department. At September 30, 200,000 units were still in work in process inventory. The degree of completion of work in process inventory at September 30 was as follows:

Direct materials

90%

Direct labor

70

Manufacturing overhead

35

a. Prepare a production cost report using FIFO.

b. Answer requirement ( b ) in Problem 8 39.

prepare a production cost report and show cost flows through accounts fifo method in 691107

Prepare a Production Cost Report and Show Cost Flows through Accounts: FIFO Method

In its Department R, Recyclers, Inc., processes donated scrap cloth into towels for sale in local thrift shops. It sells the products at cost. The direct materials costs are zero, but the operation requires the use of direct labor and overhead. The company uses a process costing system and tracks the processing volume and costs incurred in each period. At the start of the current period, 300 towels were in process and were 60 percent complete. The costs incurred were $168. During the month, costs of $10,800 were incurred, 2,700 towels were started, and 150 towels were still in process at the end of the month. At the end of the month, the towels were 20 percent complete.

Required

a. Prepare a production cost report; the company uses FIFO process costing.

b. Show the flow of costs through T accounts. Assume that current period conversion costs are credited to various payables.

c. Management is concerned that production costs are rising and would like to hold them to less than $4.25 per unit. Has the company achieved this target? Write a short report to management stating your answer.

process costing pantanal inc manufactures car seats in a local factory for costing p 691108

Process Costing

Pantanal, Inc., manufactures car seats in a local factory. For costing purposes, it uses a first in, first out (FIFO) process costing system. The factory has three departments: Molding, Assembling, and Finishing. Following is information on the beginning work in process inventory in the Assembling Department on August 1:

 

Costs

 

Degree of

Completion

Work in process beginning inventory (10,000 units)

 

 

Transferred in from Molding

$ 98,000

100%

Direct materials costs

164,400

60

Conversion costs

61,000

40

Work in process balance (August 1)

$323,400

100%

During August, 102,000 units were transferred in from the Molding Department at a cost of $2,142,000 and started in Assembling. The Assembling Department incurred other costs of $1,164,600 in August as follows:

 

August Costs

Direct materials costs

$ 939,600

Conversion costs

225,000

Total August costs

$1,164,600

At the end of August, 16,000 units remained in inventory that were 90 percent complete with respect to direct materials and 50 percent complete with respect to conversion.

Required

Compute the cost of goods transferred out in August and the cost of work in process ending inventory.

solving for unknowns weighted average method for each of the following independent c 691110

Solving for Unknowns: Weighted Average Method

For each of the following independent cases, determine the units or equivalent units requested (assuming weighted average costing).

a. The WIP Inventory account had a beginning balance of $11,400 for conversion costs on items in process and, during the period, $108,600 in conversion costs were charged to it. Also during the period, $115,200 in costs were transferred out. There were 2,400 units in the beginning inventory, and 28,800 units were transferred out during the period. How many equivalent units are in the ending inventory?

b. During the period, 8,400 units were transferred into the department. The 12,800 units transferred out were charged to the next department at an amount that included $26,880 for direct materials costs. The ending inventory was 25 percent complete with respect to direct materials and had a direct materials cost of $5,040 assigned to it. How many physical units are in the ending inventory?

c. Beginning inventory consisted of 16,000 units with a direct materials cost of $56,800. The equivalent work represented by all direct materials costs in the WIP Inventory account amounted to 72,000 units. Ending inventory had 24,000 units that were 20 percent complete with respect to materials. The ending inventory had a $18,000 direct materials cost assigned. What was the total materials cost incurred this period?

d. Beginning inventory had 12,300 units 40 percent complete with respect to conversion costs. During the period, 10,500 units were started. Ending inventory had 10,000 units 30 percent complete with respect to conversion costs. How many units were transferred out?

operation costing work in process inventory washington inc makes three models of mot 691111

Operation Costing: Work in Process Inventory

Washington, Inc., makes three models of motorized carts for vacation resorts, X 10, X 20, and X 40. Washington manufactures the carts in two assembly departments: Department A and Department B. All three models are processed initially in Department A, where all material is assembled. The X 10 model is then transferred to finished goods. After processing in Department A, the X 20 and X 40 models are transferred to Department B for final assembly, and then transferred to finished goods.

There were no beginning work in process inventories on April 1. Data for April are shown in the following table. Ending work in process is 25 percent complete in Department A and 60 percent complete in Department B. Conversion costs are allocated based on the number of equivalent units processed in each department.

 

Total

X 10

X 20

X 40

Units started

 

500

300

200

Units completed in Department A

 

400

260

180

Units completed in Department B

 

 

225

165

Materials

$450,000

$75,000

$135,000

$240,000

Conversion costs:

 

 

 

 

Department A

$264,000

 

 

 

Department B

42,000

 

 

 

Total conversion costs

$306,000

 

 

 

a. What is the unit cost of each model transferred to finished goods in April?

b. What is the balance of work in process inventory on April 30 for Department A? Department B?

operations costing work in process inventory miller outdoor equipment moe makes four 691112

Operations Costing: Work in Process Inventory

Miller Outdoor Equipment (MOE) makes four models of tents. The model names are Rookie, Novice, Hiker, and Expert. MOE manufactures the tents in two departments: Stitching and Customizing. All four models are processed initially in Stitching where all material is assembled and sewn into a basic tent. The Rookie model is then transferred to finished goods. After processing in Stitching, the other three models are transferred to Customizing for additional add ons, and then transferred to finished goods.

There were no beginning work in process inventories on August 1. Data for August are shown in the following table. Ending work in process is 40 percent complete in Stitching and 20 percent complete in Customizing. Conversion costs are allocated based on the number of equivalent units processed in each department.

 

Total

Rookie

Novice

Hiker

Expert

Units started

 

600

480

290

150

Units completed in Stitching Units competed in Customizing

 

540

450 440

270 250

120 100

Materials

$ 39,720

$12,000

$11,520

$8,700

$7,500

Conversion costs:

 

 

 

 

 

Stitching.

$ 50,260

 

 

 

 

Customizing

$24,000

 

 

 

 

Total conversion cost

$74,260

 

 

 

 

Required

a. What is the unit cost of each model transferred to finished goods in August?

b. What is the balance of the Work in Process Inventory on August 31 for Stitching? For Customizing?

process costing and ethics increasing production to boost profits pacific siding inc 691113

Process Costing and Ethics: Increasing Production to Boost Profits Pacific Siding Incorporated produces synthetic wood siding used in the construction of residential and commercial buildings. Pacific Siding’s fiscal year ends on March 31, and the weighted average method is used for the company’s process costing system.

Financial results for the first 11 months of the current fiscal year (through February 28) are well below the expectations of management, owners, and creditors. Halfway through the month of March, the chief executive officer (CEO) and the chief financial officer (CFO) ask the controller to estimate the production results for the month of March in the form of a production cost report (the company has only one production department). This report is shown below.

Armed with the preliminary production cost report for March, and knowing that the company’s production is well below capacity, the CEO and CFO decide to produce as many units as possible for the last half of March, even though sales are not expected to increase any time soon. The production manager is told to push his employees to get as far as possible with production, thereby  increasing the percentage of completion for ending WIP inventory. However, since the production process takes three weeks to complete, all of the units produced in the last half of March will be in WIP inventory at the end of March.

Required

a. Explain how the CEO and CFO expect to increase profit (net income) for the year by boosting production at the end of March. Assume that most overhead costs are fixed.

b. Using the following assumptions, prepare a revised estimate of production results in the form of a production cost report for the month of March. Assumptions based on the CEO and CFO request to boost production:

(1) Units started and partially completed during the period will increase to 225,000 (from the  initial estimate of 70,000). This is the projected ending WIP inventory at March 31.

(2) Percentage of completion estimates for units in ending WIP inventory will increase to 80 percent for direct materials, 85 percent for direct labor, and 90 percent for overhead.

(3) Costs incurred during the period will increase to $95,000 for direct materials, $102,000 for direct labor, and $150,000 for overhead (recall that most overhead costs are fixed).

(4) All units completed and transferred out during March are sold by March 31.

c. Compare your new production cost report with the one prepared by the controller. How much do you expect profit to increase as a result of increasing production during the last half of March?

d. Is the request made by the CEO and CFO ethical? Explain your answer.

show cost flows fifo method over or underapplied overhead vermont company uses conti 691114

Show Cost Flows: FIFO Method, Over or Underapplied Overhead

Vermont Company uses continuous processing to produce stuffed bears and FIFO process costing to account for its production costs. It uses FIFO because costs are quite unstable due to the volatile price of fi ne materials it uses in production. The bears are processed through one department. Overhead is applied on the basis of direct labor costs, and the application rate has not changed over the period covered by the problem. The Work in Process Inventory account showed the following balances at the start of the current period:

Direct materials

$131,000

Direct labor

260,000

Overhead applied

325,000

These costs were related to 52,000 units that were in process at the start of the period. During the period, 60,000 units were transferred to finished goods inventory. Of the units finished during this period, 80 percent were sold. After units have been transferred to finished goods inventory, no distinction is made between the costs to complete beginning work in process inventory and the costs of goods started and completed in work in process this period.

The equivalent units for materials this period were 50,000 (using FIFO). Of these, 10,000 were equivalent units with respect to materials in the ending work in process inventory. Materials costs incurred during the period totaled $300,400.

Conversion costs of $1,287,000 were charged this period for 62,500 equivalent units (using FIFO). The ending inventory consisted of 22,000 equivalent units of conversion costs. The actual manufacturing overhead for the period was $660,000.

Required

Prepare T accounts to show the flow of costs in the system. Any difference between actual and applied overhead for the period should be debited or credited to Cost of Goods Sold.

assigning costs missing data the following t accounts represent november activity ma 691056

Assigning Costs: Missing Data

The following T accounts represent November activity.

Materials Inventory

Work In Process Inventory

EB (11/30) 56,400

 

BB (11/1) 32,600

 

 

 

Dir. Materials 86,200

 

Finished Goods Inventory

Cost of Goods Sold

EB (11/30) 101,000

 

 

 

Manufacturing Overhead Control

Applied Manufacturing Overhead

Actual

 

 

264,000

Wages Payable

Sales Revenue

 

 

 

725,400

Additional Data

• Materials of $113,600 were purchased during the month, and the balance in the Materials Inventory account increased by $11,000.

• Overhead is applied at the rate of 150 percent of direct labor cost.

• Sales are billed at 180 percent of cost of goods sold before the over or underapplied overhead  is prorated.

• The balance in the Finished Goods Inventory account decreased by $28,600 during the month before any proration of under or overapplied overhead.

• Total credits to the Wages Payable account amounted to $202,000 for direct and indirect labor.

• Factory depreciation totaled $48,200.

• Overhead was underapplied by $25,080. Overhead other than indirect labor, indirect materials,  and depreciation was $198,480, which required payment in cash. Underapplied overhead is to be allocated.

• The company has decided to allocate 25 percent of underapplied overhead to Work in Process Inventory, 15 percent to Finished Goods Inventory, and the balance to Cost of Goods Sold.  Balances shown in T accounts are before any allocation.

Required

Complete the T accounts.

analysis of overhead using a predetermined rate kansas company uses a job costing ac 691057

Analysis of Overhead Using a Predetermined Rate

Kansas Company uses a job costing accounting system for its production costs. The company uses a predetermined overhead rate based on direct labor hours to apply overhead to individual jobs. The company prepared an estimate of overhead costs at different volumes for the current year as follows:

Direct labor hours

150,000

180,000

210,000

Variable overhead costs

$1,050,000

$1,260,000

$1,470,000

Fixed overhead costs

648,000

648,000

 648,000

Total overhead

$1,698,000

$1,908,000

$2,118,000

The expected volume is 180,000 direct labor hours for the entire year. The following information is for March, when Jobs 6023 and 6024 were completed.

Inventories, March 1

 

Materials and supplies

$ 31,500

Work in process (Job 6023)

$162,000

Finished goods

$337,500

Purchases of materials and supplies

 

Materials

$405,000

Supplies

$ 45,000

Materials and supplies requisitioned for production

 

Job 6023

$135,000

Job 6024

112,500

Job 6025

76,500

Supplies

18,000

 

$342,000

Factory direct labor hours (DLH)

 

Job 6023

10,500 DLH

Job 6024

9,000 DLH

Job 6025

6,000 DLH

Labor costs

 

Direct labor wages (all hours @ $8)

$204,000

Indirect labor wages (12,000 hours)

51,000

Supervisory salaries

108,000

Building occupancy costs (heat, light, depreciation, etc.)

 

Factory facilities

$ 19,500

Sales and administrative offices

7,500

Factory equipment costs

 

Power

12,000

Repairs and maintenance

4,500

Other

7,500

 

$ 24,000

Required

Answer the following questions.

a. Compute the predetermined overhead rate (combined fixed and variable) to be used to apply overhead to individual jobs during the year. ( Note: Regardless of your answer to requirement [ a ], assume that the predetermined overhead rate is $9 per direct labor hour. Use this amount in answering requirements [ b ] through [ e ].)

b. Compute the total cost of Job 6023 when it is finished.

c. How much of factory overhead cost was applied to Job 6025 during March?

d. What total amount of overhead was applied to jobs during March?

e. Compute actual factory overhead incurred during March.

f. At the end of the year, Kansas Company had the following account balances:

Overapplied Overhead

$ 3,000

Cost of Goods Sold

2,940,000

Work in Process Inventory

114,000

Finished Goods Inventory

246,000

How would you recommend treating the overapplied overhead, assuming that it is not material? Show the new account balances in the following table.

Overapplied Overhead

_________

Cost of Goods Sold

_________

Work in Process Inventory

_________

Finished Goods Inventory

_________

analysis of overhead using a predetermined rate ucd company uses a job costing accou 691058

Analysis of Overhead Using a Predetermined Rate

UCD Company uses a job costing accounting system for its production costs. A predetermined overhead rate based on direct labor hours is used to apply overhead to individual jobs. An estimate of overhead costs at different volumes was prepared for the current year as follows:

Direct labor hours

30,000

40,000

50,000

Variable overhead costs

$360,000

$480,000

$ 600,000

Fixed overhead costs

500,000

500,000

500,000

Total overhead

$860,000

$980,000

$1,100,000

The expected volume is 40,000 direct labor hours for the entire year. The following information is for October, when jobs 1011 and 1015 were completed.

Inventories, October 1

 

Raw materials and supplies

$ 42,000

Work in process (Job 1011)

91,300

Finished goods

227,900

Purchases of raw materials and supplies

 

Raw materials

629,000

Supplies

79,300

Materials and supplies requisitioned for production

 

Job 1011

281,000

Job 1015

234,500

Job 1017

47,200

Supplies

76,700

 

$639,400

Machine hours (MH)

 

Job 1011

6,200 MH

Job 1015

6,100 MH

Job 1017

3,700 MH

Direct labor hours (DLH)

 

Job 1011

14,000 DLH

Job 1015

6,100 DLH

Job 1017

3,700 DLH

Labor costs

 

Direct labor wages (all hours @ $12)

$285,600

Indirect labor wages (12,000 hours)

63,000

Supervisory salaries

128,000

Building occupancy costs (heat, light, depreciation, etc.)

 

Factory facilities

36,900

Sales and administrative offices

14,200

Factory equipment costs

 

Power

21,800

Repairs and maintenance

8,200

Other

9,900

 

$ 39,900

Required

Answer the following questions.

a. Compute the predetermined overhead rate (combined fixed and variable) to be used to apply overhead to individual jobs during the year.

b. Compute the total cost of Job 1011 when it is finished.

c. How much of factory overhead cost was applied to Job 1017 during October?

d. What total amount of overhead was applied to jobs during October?

e. Compute actual factory overhead incurred during October.

f. At the end of the year, UCD Company had the following account balances:

 

Balance

Underapplied Overhead

$ 3,000,000

Cost of Goods Sold

28,000,000

Work in Process Inventory

4,000,000

Finished Goods Inventory

8,000,000

How would you recommend treating the Underapplied Overhead? Show the effect on the account balances in the following table.

Overapplied Overhead

_________

Cost of Goods Sold

_________

Work in Process Inventory

_________

Finished Goods Inventory

_________

finding missing data a new computer virus acctbgone destroyed most of the company re 691059

Finding Missing Data

A new computer virus (AcctBGone) destroyed most of the company records at BackupsRntUs. The computer experts at the company could recover only a few fragments of the company’s factory ledger for February as follows:

Direct Materials Inventory

Work In Process Inventory

BB (2/1) 90,000

 

BB (2/1) 27,000

 

Finished Goods Inventory

Cost of Goods Sold

EB (2/29) 66,000

 

 

 

Manufacturing Overhead Control

Accounts Payable

 

 

 

54,000 EB (2/29)

Further investigation and reconstruction from other sources yielded the following additional information.

• The controller remembers clearly that actual manufacturing overhead costs are recorded at $18 per direct labor hour. (The company assigns actual overhead to Work in Process Inventory.)

• The production superintendent’s cost sheets showed only one job in Work in Process Inventory on February 29. Materials of $15,600 had been added to the job, and 300 direct laborhours had been expended at $36 per hour.

• The Accounts Payable are for direct materials purchases only, according to the accounts payable clerk. He clearly remembers that the balance in the account was $36,000 on February 1. An analysis of canceled checks (kept in the treasurer’s office) shows that payments of $252,000 were made to suppliers during the month.

• The payroll ledger shows that 5,200 direct labor hours were recorded for the month. The employment  department has verified that there are no variations in pay rates among employees (this infuriated Steve Fung, who believed that his services were underpaid).

• Records maintained in the finished goods warehouse indicate that the finished goods inventory totaled $108,000 on February 1.

• The cost of goods manufactured in February was $564,000.

Required

Determine the following amounts:

a. Work in process inventory, February 29.

b. Direct materials purchased during February.

c. Actual manufacturing overhead incurred during February.

d. Cost of goods sold for February.

cost accumulation service youth athletic services yas provides adult supervision for 691060

Cost Accumulation: Service

Youth Athletic Services (YAS) provides adult supervision for organized youth athletics. It has a president, William Mayes, and five employees. He and one of the other five employees manage all marketing and administrative duties. The remaining four employees work directly on operations. YAS has four service departments: managing, off ciating, training, and dispute resolution. A time card is marked, and records are kept to monitor the time each employee spends working in each department. When business is slow, there is idle time, which is marked on the time card. (It is necessary to have some idle time because some direct labor hours must be available to accommodate fluctuating peak demand periods throughout the day and the week.)

Some of the July operating data are as follows:

 

Idle time

 

 

 

Dispute

Idle Time

 

Managing

Officiating

Training

Resolution

Sales revenue

 

$6,950

$7,900

$3,000

$1,000

Direct labor (in hours)

25

320

80

125

90

Direct overhead traceable to departments

 

 

 

 

 

Equipment

 

$ 950

$ 875

$ 700

$ 10

Supplies

 

200

300

250

200

Transportation

 

375

1,000

150

50

Other Data

• The four employees working in the operating departments all make $15 per hour.

• The fifth employee, who helps manage marketing and administrative duties, earns $2,250 per month, and William earns $3,000 per month.

• Indirect overhead amounted to $768 and is assigned to departments based on the number of direct labor hours used. Because there are idle hours, some overhead will not be assigned to a department.

• In addition to salaries paid, marketing costs for items such as advertising and special promotions totaled $600.

• In addition to salaries paid, other administrative costs were $225.

• All revenue transactions are cash; all others are on account.

Required

Management wants to know whether each department is contributing to the company’s profit t. Prepare  an income statement for July that shows the revenue and cost of services for each department. Write a short report to management about departmental profitability. No inventories are kept.

job costs in a service company on september 1 two jobs were in process at pete rsquo 691062

Job Costs in a Service Company

On September 1, two jobs were in process at Pete’s Patios. Details of the jobs follow:

Job Number

Direct Materials

Direct Labor

PP 24

$2,038

$ 768

PP 30

1,280

3,360

Materials Inventory on September 1 totaled $11,040, and $1,392 worth of materials was purchased during the month. Indirect materials of $192 were withdrawn from materials inventory. On September 1, finished goods inventory consisted of two jobs, PP 12, costing $4,704, and PP 14, with a cost of $1,896. Costs for both jobs were transferred to Cost of Services Billed during the month.

Also during September, Jobs PP 24 and PP 30 were completed. Completing Job PP 24 required an additional $2,720 in direct labor. The completion costs for Job PP 30 included $1,296 in direct materials and $8,000 in direct labor.

Pete’s Patios used a total of $3,768 of direct materials (excluding the $192 indirect materials) during the period, and total direct labor costs during the month amounted to $16,320. Overhead has been estimated at 50 percent of direct labor costs, and this relation has been the same for the past few years.

Required

Compute the costs of Jobs PP 24 and PP 30 and the balances in the September 30 inventory accounts.

tracing costs in a job company the following transactions occurred in january at dun 691063

Tracing Costs in a Job Company

The following transactions occurred in January at Dungan Cabinetry, a furniture maker that uses job costing:

1. Purchased $53,700 in materials on account.

2. Issued $1,500 in supplies from the materials inventory to the production department.

3. Paid for the materials purchased in (1).

4. Issued $25,500 in direct materials to the production department.

5. Incurred wage costs of $42,000, which were debited to Payroll, a temporary account. Of this amount, $13,500 was withheld for payroll taxes and credited to Payroll Taxes Payable. The remaining

$28,500 was paid in cash to the employees. See transactions (6) and (7) for additional information about Payroll.

6. Recognized $21,000 in fringe benefit costs, incurred as a result of the wages paid in (5). This

$21,000 was debited to Payroll and credited to Fringe Benefits Payable.

7. Analyzed the Payroll account and determined that 60 percent represented direct labor;

30 percent, indirect manufacturing labor; and 10 percent, administrative and marketing costs.

8. Paid for utilities, power, equipment maintenance, and other miscellaneous items for the manufacturing plant totaling $32,400.

9. Applied overhead on the basis of 175 percent of direct labor costs.

10. Recognized depreciation of $17,250 on manufacturing property, plant, and equipment.

Required

a. Prepare journal entries to record these transactions.

b. The following balances appeared in the accounts of Dungan Cabinetry:

 

Beginning

Ending

Materials Inventory

$55,575

Work In Process Inventory

12,375

Finished Goods Inventory

62,250

$49,800

Cost of Goods Sold

98,775

Prepare T accounts to show the flow of costs during the period.

cost flows through accounts brighton services repairs locomotive engines it employs 691064

Cost Flows through Accounts

Brighton Services repairs locomotive engines. It employs 100 full time workers at $20 per hour. Despite operating at capacity, last year’s performance was a great disappointment to the managers. In total, 10 jobs were accepted and completed, incurring the following total costs:

 

Direct materials

$1,035,400

Direct labor

4,000,000

Manufacturing overhead

1,040,000

Of the $1,040,000 manufacturing overhead, 30 percent was variable overhead and 70 percent was fixed.

This year Brighton Services expects to operate at the same activity level as last year, and overhead costs and the wage rate are not expected to change. For the first quarter of this year, Brighton Services completed two jobs and was beginning the third ( job 103). The costs incurred follow:

Job

Direct Materials

Direct Labor

101

$137,200

$490,000

102

93,000

312,400

103

94,000

197,600

Total manufacturing overhead

 

 271,200

Total marketing and administrative costs

112,000

       

You are a consultant associated with Lodi Consultants, which Brighton Services has asked for help. Lodi’s senior partner has examined Brighton Services’s accounts and has decided to divide actual factory overhead by job into fixed and variable portions as follows:

Actual Manufacturing Overhead

 

Variable

Fixed

101

$29,900

$104,000

102

27,500

88,200

103

4,600

17,000

 

$62,000

$209,200

In the first quarter of this year, 40 percent of marketing and administrative cost was variable and 60 percent was fixed. You are told that Jobs 101 and 102 were sold for $850,000 and $550,000, respectively. All over or underapplied overhead for the quarter is written off to Cost of Goods Sold.

Required

a. Present in T accounts the actual manufacturing cost flows for the three jobs in the first quarter of this year.

b. Using last year’s overhead costs and direct labor hours as this year’s estimate, calculate predetermined overhead rates per direct labor hour for variable and fixed overhead.

c. Present in T accounts the normal manufacturing cost flows for the three jobs in the first quarter of this year. Use the overhead rates derived in requirement ( b ).

d. Prepare income statements for the first quarter of this year under the following costing systems:

(1) Actual.

(2) Normal.

show flow of costs to jobs kim rsquo s asphalt does driveway and parking lot resurfa 691065

Show Flow of Costs to Jobs

Kim’s Asphalt does driveway and parking lot resurfacing work for large commercial clients as well as small residential clients. An inventory of materials and equipment is on hand at all times so that work can start as quickly as possible. Special equipment is ordered as required. On May 1, the Materials and Equipment Inventory account had a balance of $24,000. The Work in Process Inventory account is maintained to record costs of work not yet complete. There were two such jobs on May 1 with the following costs:

 

Job 27

Job 33

 

Highlands Mall

Pine Ridge Estates

Materials and equipment

$10,250

$35,000

Direct labor

9,000

17,500

Overhead (applied)

2,700

5,250

Overhead has been applied at 30 percent of the costs of direct labor. During May, Kim’s Asphalt started two new jobs. Additional work was carried out on Jobs 27 and 33, with the latter completed and billed to Pine Ridge Estates. Details on the costs incurred on jobs during May follow:

Job

27

33

34

35

Materials and equipment

$2,000

$3,200

$4,600

$2,000

Direct labor (wages payable)

3,000

4,500

5,900

1,600

Other May Events

1. Received $12,500 payment on Job 24 delivered to customer in April.

2. Purchased materials and equipment for $9,400.

3. Billed Pine Ridge Estates $87,500 and received payment for $50,000 of that amount.

4. Determined that payroll for indirect labor personnel totaled $650.

5. Issued supplies and incidental materials for current jobs costing $155.

6. Recorded overhead and advertising costs for the operation as follows (all cash except equipment depreciation):

Property taxes

$550

Storage area rental

675

Truck and delivery cost

320

Advertising and promotion campaign

600

Inspections

200

Telephone and other miscellaneous

325

Equipment depreciation

450

Required

a. Prepare journal entries to record the flow of costs for operations during May.

b. Calculate the amount of over or underapplied overhead for the month. This amount is debited or credited to Cost of Goods Sold.

c. Determine inventory balances for Materials and Equipment Inventory and Work in Process Inventory.

job costing and ethics old port shipyards does work for both the u s navy and privat 691069

Job Costing and Ethics

Old Port Shipyards does work for both the U.S. Navy and private shipping companies. Old Port’s major business is renovating ships, which it does at one of two company dry docks referred to by the names of the local towns: Olde Town and Newton. Data on operations and costs for the two dry docks follow:

 

Olde Town

Newton

Overhead cost

$20,000,000

$80,000,000

Direct labor hours

200,000

200,000

Virtually all dry dock costs consist of depreciation. The Newton dry dock is much newer, so the depreciation on it is much higher. Dry dock overhead is charged to jobs based on direct labor hours for the specific dock.

Old Port is about to start two jobs, one for the Navy under a cost plus contract and one for a private shipping company for a fixed fee. Both jobs will require the same number of hours. You have been asked to prepare some costing information. Your supervisor tells you that she is sure the Navy job will be done at Newton and the private job will be done at Olde Town.

Required

a. Compute the overhead rate at the two shipyards.

b. Why do you think your supervisor says that the Navy job will be done at Newton?

c. Is the choice of the production location ethical? Why?

job costing and ethics chuck moore supervises two consulting jobs for the firm of pr 691070

Job Costing and Ethics

Chuck Moore supervises two consulting jobs for the firm of Price and Waters, LLP, which is a consulting firm that helps organizations become more efficient. One of the consulting jobs is for the U.S. Department of Defense and the other is for General Motors, Inc. Chuck received the monthly cost reports about three weeks after month end. The General Motors job contained bad news. After getting up his nerve, Chuck called his boss the following week to pass on the bad news. “The General Motors job is only half done, but we have already spent all of the $1 million that we expected to spend on that job,” he said. “However, we have spent only $500,000 of the $800,000 that we expected to spend on the U.S. Department of Defense job, even though we are 90 percent done with the work.”

His boss told Chuck, “Assign the rest of the costs needed to complete the General Motors job to your U.S. Department of Defense job. We’re under budget on that job and we get reimbursed for costs on government jobs.”

Required

a. What should Chuck do?

b. Does it matter that Chuck’s consulting firm is reimbursed for costs on the government jobs? Explain.

bart rsquo s beverages is a small local operation that makes only one flavor of soft 691072

Bart’s Beverages is a small, local operation that makes only one flavor of soft drink. The following data areavailable for operations in its Blending Department during October.

 

 

Percent

 

 

Barrels

Complete

Costs

Beginning work in process inventory, October 1

1,000

 

 

Materials costs

 

25%

$ 1,113

Conversion costs

 

10

194

Units started in October

5,000

 

 

Costs incurred in October

 

 

 

Materials costs

500

80

 

Conversion costs

 

 

22,487

 Ending work in process inventory, October 1

 

 

14,056

Materials costs

 

 

?

Conversion costs

 

40

?

Using weighted average process costing, prepare a cost of production report for October.

the bottling department at bart rsquo s beverages receives the blended soft drink in 691073

The Bottling Department at Bart’s Beverages receives the blended soft drink in barrels and pours it into bottles. The following data are available for operations in the Bottling Department during August. (Note that the units here are bottles, not barrels.)

 

Bottles

Percent Complete

Costs

Beginning work in process (WIP) inventory,

 

 

 

August 1

50,000

 

 

Transferred in costs

 

100%

$ 6,998

Materials costs

 

–0–

–0–

Conversion costs

 

5

69

Units started in August

240,000

 

 

Costs incurred in August

 

 

 

Transferred in costs

 

 

$33,602

Materials costs

 

 

58,000

Conversion costs

 

 

7,611

Ending WIP inventory, August 31

40,000

 

 

Transferred in costs

 

100

?

Materials costs

 

100

?

Conversion costs

 

15

?

Using weighted average process costing, prepare  a cost of production report for August for the Bottling Department.

rigney corporation manufactures two models of travel alarms the m 24 and the d 12 wh 691074

Rigney Corporation manufactures two models of travel alarms, the M 24 and the D 12, which differ only in the quality of plastic used for the cases. Production takes place in two departments, Manufacturing and Finishing. Data for the only two work orders for February are shown in the following table. Conversion costs are allocated based on the number of units produced. There are no work in process inventories.

Materials

Total

M 24 (5,000 units)

D 12 (9,000 units)

Manufacturing

$42,500

$20,000

$22,500

Conversion costs

 

 

 

Manufacturing

42,000

 

 

Finishing

21,000

 

 

Total conversion cost

$63,000

 

 

What is the cost per unit transferred to finished goods inventory for the two models, M 24 and D 12, in February?

when using the weighted average method of process costing total equivalent units pro 691080

When using the weighted average method of process costing, total equivalent units produced for a given period equal:

a. The number of units started and completed during the period plus the number of units in beginning work in process plus the number of units in ending work in process.

b. The number of units in beginning work in process plus the number of units started during the period plus the number of units remaining in ending work in process times the percentage of work necessary to complete the items.

c. The number of units in beginning work in process times the percentage of work necessary to complete the items plus the number of units started and completed during the period plus the number of units started this period and remaining in ending work in process times the percentage of work necessary to complete the items.

d. The number of units transferred out during the period plus the number of units remaining in ending work in process times the percentage of work necessary to complete the items.

e. None of these.

compute equivalent units bears inc adds materials at the beginning of the process in 691085

Compute Equivalent Units

Bears, Inc., adds materials at the beginning of the process in Department MO. The following information on physical units for Department MO for the month of July is available:

Work in process, July 1 (75% complete with respect to conversion)  .

27,000

Started in July

180,000

Completed

180,000

Work in process, July 30 (60% complete with respect to conversion) 

27,000

Required

a. Compute the equivalent units for materials costs and for conversion costs using the weighted average method.

b. Compute the equivalent units for materials costs and for conversion costs using the FIFO method.

compute equivalent units ethical issues aaron company has a process costing system a 691086

Compute Equivalent Units: Ethical Issues

Aaron Company has a process costing system. All materials are introduced when conversion costs reach 50 percent. The following information is available for physical units during March.

Work in process, March 1 (60% complete as to conversion costs)

150,000

Units started in March

600,000

Units transferred to Finishing Department in March

630,000

Work in process, March 31 (40% complete as to conversion costs)

120,000

Required

a. Compute the equivalent units for materials costs and for conversion costs using the weightedaverage method.

b. Compute the equivalent units for materials costs and for conversion costs using the FIFO method.

c. The company president has been under considerable pressure to increase income. He tells the controller to change the estimated completion for ending work in process to 60 percent (from 40 percent).

1. What effect will this change have on the unit costs of units transferred to finished goods in March?

2. Would this be ethical?

3. Is this likely to be a successful strategy for affecting income over a long period of time?

compute costs per equivalent unit weighted average method the following information 691087

Compute Costs per Equivalent Unit: Weighted Average Method

The following information pertains to the Davenport plant for the month of May (all materials are added at the beginning of the process):

 

Units

Material Costs

Beginning work in process

270,000

$ 99,000

Started in May

720,000

316,800

Units completed

765,000

 

Ending work in process

225,000

 

Required

Compute the cost per equivalent unit for materials using the weighted average method.

compute costs per equivalent unit weighted average method the following information 691088

Compute Costs per Equivalent Unit: Weighted Average Method

The following information pertains to the Davenport plant for the month of May (all materials are added at the beginning of the process):

 

Units

Material Costs

Beginning work in process

270,000

$ 99,000

Started in May

720,000

316,800

Units completed

765,000

 

Ending work in process

225,000

 

Required

Compute the cost per equivalent unit for materials using the FIFO method.

compute equivalent units fifo method materials are added at the beginning of the pro 691089

Compute Equivalent Units: FIFO Method

Materials are added at the beginning of the production process at Santiago Company, which uses a FIFO process costing system. The following information on the physical flow of units is available for the month of April:

Beginning work in process (40% complete)

45,000

Started in April

510,000

Completed in April and transferred out

480,000

Ending work in process (70% complete)

75,000

Required

Compute the equivalent units for the conversion cost calculation.

tiger furnishings produces two models of cabinets for home theater components the ba 691021

Tiger Furnishings produces two models of cabinets for home theater components, the Basic and the Dominator. Data on operations and costs for March follow:

 

Basic

Dominator

Total

Units produced

1,000

250

1,250

Machine hours

4,000

2,000

6,000

Direct labor hours

3,000

2,000

5,000

Direct materials costs

$10,000

$ 3,750

$ 13,750

Direct labor costs

64,500

35,500

100,000

Manufacturing overhead costs

 

 

174,100

Total costs

 

 

$287,850

Draw the cost flow diagram assuming that Tiger Furnishings uses direct labor costs to allocate overhead costs.

operations costing howrley david inc manufactures two models of motorcycles the fatb 691022

Operations Costing

Howrley David, Inc., manufactures two models of motorcycles: the Fatboy and the Screamer. Both models are assembled in the same plant and require the same assembling operations. The difference between the models is the cost of materials. The following data are available for August.

 

Fatboy

Screamer

Total

Number of units assembled

2,000

4,000

6,000

Materials cost per unit

$2,000

$3,000

 

Other costs:

 

 

 

Direct labor

 

 

$6,000,000

Indirect materials

 

 

1,800,000

Other overhead

 

 

4,200,000

Required

Howrley David uses operations costing and assigns conversion costs based on the number of units assembled. Compute the cost of each model assembled in August.

operations costing organic grounds produces two brands of coffee star and bucks the 691023

Operations Costing

Organic Grounds produces two brands of coffee: Star and Bucks. The two coffees are produced in one factory using the same production process. The only difference between the two coffees is the cost of the unroasted coffee beans. The following data are available for February

 

Star

Bucks

Total

Number of pounds of coffee produced

5,000

20,000

25,000

Cost of unroasted beans and packaging per pound

$4

$6

 

Other costs:

 

 

 

Direct labor

 

 

$ 50,000

Indirect materials

 

 

15,000

Other overhead

 

 

100,000

Required

Organic Grounds uses operations costing and assigns conversion costs based on the number of pounds of coffee produced. Compute the cost unit of each brand of coffee produced in February.

predetermined overhead rates tiger furnishings produces two models of cabinets for h 691024

Predetermined Overhead Rates

Tiger Furnishings produces two models of cabinets for home theater components, the Basic and the Dominator. Data on operations and costs for March follow:

 

Basic

Dominator

Total

Units produced

1,000

250

1,250

Machine hours

4,000

2,000

6,000

Direct labor hours

3,000

2,000

5,000

Direct materials costs

$10,000

$ 3,750

$ 13,750

Direct labor costs

64,500

35,500

100,000

Manufacturing overhead costs

 

 

174,100

Total costs

 

 

$287,850

Required

Compute the individual product costs per unit assuming that Tiger Furnishings uses direct labor costs to allocate overhead to the products.

predetermined overhead rates tiger furnishings produces two models of cabinets for h 691025

Predetermined Overhead Rates

Tiger Furnishings produces two models of cabinets for home theater components, the Basic and the Dominator. Data on operations and costs for March follow:

 

Basic

Dominator

Total

Units produced

1,000

250

1,250

Machine hours

4,000

2,000

6,000

Direct labor hours

3,000

2,000

5,000

Direct materials costs

$10,000

$ 3,750

$ 13,750

Direct labor costs

64,500

35,500

100,000

Manufacturing overhead costs

 

 

174,100

Total costs

 

 

$287,850

Required

Compute the individual product costs per unit assuming that Tiger Furnishings uses machine hours to allocate overhead to the products.

predetermined overhead rates tiger furnishings produces two models of cabinets for h 691026

Predetermined Overhead Rates

Tiger Furnishings produces two models of cabinets for home theater components, the Basic and the Dominator. Data on operations and costs for March follow:

 

Basic

Dominator

Total

Units produced

1,000

250

1,250

Machine hours

4,000

2,000

6,000

Direct labor hours

3,000

2,000

5,000

Direct materials costs

$10,000

$ 3,750

$ 13,750

Direct labor costs

64,500

35,500

100,000

Manufacturing overhead costs

 

 

174,100

Total costs

 

 

$287,850

 

The president of Tiger Furnishings is confused about the differences in costs that result from using direct labor costs and machine hours.

Required

a. Explain why the two product costs are different.

b. How would you respond to the president when asked to recommend one allocation base or the other?

c. The president says to choose the allocation base that results in the highest income. Is this an appropriate basis for choosing an allocation base?

 

two stage allocation and product costing owl eye radiologists or does various types 691028

Two Stage Allocation and Product Costing

Owl Eye Radiologists (OR) does various types of diagnostic imaging. Radiologists perform tests using sophisticated equipment. OR’s management wants to compute the costs of performing tests for two different types of patients: those who are hospitalized (including those in emergency rooms) and those who are not hospitalized but are referred by physicians. The data for June for the two categories of patients follow.

Hospital Patients Other Patients

Total

Units (i.e., procedures) produced . .

640

860

1,500

Equipment hours used

240

120

360

Direct labor hours

480

180

660

Direct labor costs

$38,400

$10,800

$49,200

Overhead costs

 

 

$49,560

         

The accountant first assigns overhead costs to two pools: overhead related to equipment hours and overhead related to labor hours. The analysis of overhead accounts by the cost accountant follows:

Account

Amount

Related to:

Utilities

$ 4,800

Equipment hours

Supplies

12,600

Labor hours

Indirect labor and supervision

20,400

Labor hours

Equipment depreciation and maintenance .

8,400

Equipment hours

Miscellaneous

3,360

Equipment hours

Required

a. Compute the predetermined overhead rates assuming that Owl Eye Radiologists uses equipment hours to allocate equipment related overhead costs and labor hours to allocate labor related overhead costs.

b. Compute the total costs of production and the cost per unit for each of the two types of patients undergoing tests in June.

account analysis two stage allocation and product costing tiger furnishings rsquo s 691030

Account Analysis, Two Stage Allocation, and Product Costing

Tiger Furnishings’s CFO believes that a two stage cost allocation system would give managers better cost information. She asks the company’s cost accountant to analyze the accounts and assign overhead costs to two pools: overhead related to direct labor cost and overhead related to machine hours.

The analysis of overhead accounts by the cost accountant follows:

Manufacturing Overhead

Overhead Estimate

Cost Pool Assignment

Utilities

$ 1,800

Machine hour related

Supplies

5,000

Direct labor cost related

Training

10,000

Direct labor cost related

Supervision

25,800

Direct labor cost related

Machine depreciation

32,000

Machine hour related

Plant depreciation

14,200

Machine hour related

Miscellaneous

85,300

Direct labor cost related

Required

a. Draw the cost flow diagram that illustrates the two stage cost allocation of overhead for Tiger Furnishings using the results of the cost accountant’s analysis of accounts.

b. Compute the product costs per unit assuming that Tiger Furnishings uses direct labor costs and machine hours to allocate overhead to the products.

product costing cost estimation and decision making i don rsquo t understand this la 691031

Product Costing, Cost Estimation, and Decision Making

I don’t understand this. Last year [year 1], we decided to drop our highest end Red model and only produce the Yellow and Green models, because the cost system indicated we were losing money on Red. Now, looking at the preliminary numbers, our profit t is actually lower than last year and it looks like Yellow has become a money loser, even though our prices, volumes, and direct costs are the same. Can someone please explain this to me and maybe help me decide what to do next year?

Robert Dolan

President & CEO

Dolan Products

Dolan Products is a small, family owned audio component manufacturer. Several years ago, the company decided to concentrate on only three models, which were sold under many brand names to electronic retailers and mass market discount stores. For internal purposes, the company uses the product names Red, Yellow, and Green to refer to the three components. Data on the three models and selected costs follow:

Year 1

Red

Yellow

Green

Total

Units produced and sold

5,000

10,000

20,000

35,000

Sales price per unit

$150

$100

$75

 

Direct materials cost per unit

$ 70

$ 50

$30

 

Direct labor hours per unit

2

1

0.5

 

Wage rate per hour

$ 20

$ 20

$20

 

Total manufacturing overhead.

 

 

 

$750,000

This year (year 2), the company only produced the Yellow and Green models. Total overhead was $650,000. All other volumes, unit prices, costs, and direct labor usage were the same as in year 1. The product cost system at Dolan Products allocates manufacturing overhead based on direct labor hours.

Required

a. Compute the product costs and gross margins (revenue less cost of goods sold) for the three products and total gross profit t for year 1.

b. Compute the product costs and gross margins (revenue less cost of goods sold) for the two remaining products and total gross profit t for year 2.

c. Should Dolan Products drop Yellow for year 3? Explain.

jennifer rsquo s home remodeling worked on three jobs during march job 13 was in pro 691032

Jennifer’s Home Remodeling worked on three jobs during March. Job 13 was in process on March 1 with total charges of $5,500. During the month, the following additional transactions occurred.

a. Purchased $10,000 worth of new materials on account.

b. Charged materials to jobs as follows: $1,000 to Job 13, $4,000 to Job 14, $3,000 to Job 15, and $2,000 as indirect materials.

c. Charged labor to jobs as follows: $1,000 to Job 13, $3,000 to Job 14, $2,000 to Job 15, and $1,000 as indirect labor.

d. Incurred indirect expenses totaling $13,000 including depreciation of $4,000. This also included credits of $9,000 to Accounts Payable.

e. Applied manufacturing overhead for March to Work in Process based on materials used in each job. The predetermined rate was based on expected materials of $80,000 and expected overhead of $120,000 for this year.

Show the journal entries to record these transactions.

jennifer rsquo s home remodeling worked on three jobs during march job 13 was in pro 691033

Jennifer’s Home Remodeling worked on three jobs during March. Job 13 was in process on March 1 with total charges of $5,500. During the month, the following additional transactions occurred.  Suppose that the following additional transactions occurred:

a. Completed and charged the following jobs to Finished Goods: Job 13 for $9,000 and Job 14 for $13,000.

b. Sold Job 13 for $12,000 and Job 14 for $15,000, both for cash.

c. Closed Manufacturing Overhead Control and Applied Manufacturing Overhead. Actual manufacturing overhead incurred for the month was $13,000. Any over or underapplied overhead is written off to Cost of Goods Sold. Show journal entries for these transactions. Include the entry to close the manufacturing overhead accounts to Cost of Goods Sold for the month of March.

assigning costs to jobs the following transactions occurred in january at tarnsdale 691038

Assigning Costs to Jobs

The following transactions occurred in January at Tarnsdale Fabricators, a manufacturer of custom tools:

1. Purchased $17,000 of materials.

2. Issued $16,800 in direct materials to the production department.

3. Issued $1,200 of supplies from the materials inventory.

4. Paid for the materials purchased in transaction (1).

5. Returned $2,200 of the materials issued to production in (2) to the material inventory.

6. Direct labor employees earned $31,000, which was paid in cash.

7. Paid $17,200 for miscellaneous items for the manufacturing plant. Accounts Payable was credited.

8. Recognized depreciation on manufacturing plant of $35,000.

9. Applied manufacturing overhead for the month.

Tarnsdale uses normal costing. It applies overhead on the basis of direct labor costs using an annual, predetermined rate. At the beginning of the year, management estimated that direct labor costs for the year would be $434,000. Estimated overhead for the year was $403,620. The following balances appeared in the inventory accounts of Tarnsdale Fabricators for January:

 

Beginning

Ending

Materials Inventory

?

$12,600

Work in Process Inventory

?

10,500

Finished Goods Inventory

$2,600

$ 7,100

Cost of Goods Sold

?

74,500

Required

a. Prepare journal entries to record these transactions.

b. Prepare T accounts to show the flow of costs during the period from Materials Inventory through Cost of Goods Sold.

assigning costs to jobs partially completed t accounts and additional information fo 691041

Assigning Costs to Jobs

Partially completed T accounts and additional information for Pine Ridge Corporation for the month of February follow.

Materials Inventory

 

BB (2/1)

 

56,000

 

 

227,000

190,000

Work In Process Inventory

 

BB (2/1)

100,000

 

Labor

187,500

 

Finished Goods Inventory

 

BB (2/1)

 

200,000

 

 

345,000

251,000

Manufacturing Overhead Control

155,000

 

 

Applied Manufacturing Overhead

 

 

150,000

Cost of Goods Sold

 

 

 

Additional information for February follows:

• Labor wage rate was $25 per hour.

• During the month, sales revenue was $600,000, and selling and administrative costs were $105,000.

• This company has no indirect materials or supplies.

• The company applies manufacturing overhead on the basis of direct labor costs.

Required

a. What was the cost of direct materials issued to production during February?

b. What was the over or underapplied manufacturing overhead for February?

c. What was the manufacturing overhead application rate in February?

d. What was the cost of products completed during February?

e. What was the balance of the Work in Process Inventory account at the end of February?

f. What was the operating profit t for February? Any over or underapplied overhead is written off to Cost of Goods Sold.

prorate under or overapplied overhead mark corporation estimates its manufacturing o 691044

Prorate Under or Overapplied Overhead

Mark Corporation estimates its manufacturing overhead to be $90,000 and its direct labor costs to be $200,000 for year 1. The actual direct labor costs were $50,000 for Job 301, $75,000 for Job 302, and $100,000 for Job 303 during year 1; the actual manufacturing overhead was $97,000. Manufacturing overhead is applied to jobs on the basis of direct labor costs using predetermined rates.

Prepare an entry to allocate the under or overapplied overhead. Overhead applied in each of the inventory accounts is as follows:

Work in process inventory

$10,125

Finished goods inventory

30,375

Cost of goods sold

60,750

predetermined overhead rates aspen company estimates its manufacturing overhead to b 691045

Predetermined Overhead Rates

Aspen Company estimates its manufacturing overhead to be $625,000 and its direct labor costs to be $500,000 for year 2. Aspen worked on three jobs for the year. Job 2 1, which was sold during year 2, had actual direct labor costs of $195,000. Job 2 2, which was completed, but not sold at the end of the year, had actual direct labor costs of $325,000. Job 2 3, which is still in work in process inventory, had actual direct labor costs of $130,000. Actual manufacturing overhead for year 2 was $825,000. Manufacturing overhead is applied on the basis of direct labor costs.

Required

a. How much overhead was applied to each job in year 2?

b. What was the over or underapplied manufacturing overhead for year 2?

prorate over or underapplied overhead aspen company estimates its manufacturing over 691046

Prorate Over or Underapplied Overhead

Aspen Company estimates its manufacturing overhead to be $625,000 and its direct labor costs to be $500,000 for year 2. Aspen worked on three jobs for the year. Job 2 1, which was sold during year 2, had actual direct labor costs of $195,000. Job 2 2, which was completed, but not sold at the end of the year, had actual direct labor costs of $325,000. Job 2 3, which is still in work in process inventory, had actual direct labor costs of $130,000. Actual manufacturing overhead for year 2 was $825,000. Manufacturing overhead is applied on the basis of direct labor costs.

Prepare an entry to allocate over or underapplied overhead to:

a. Work in Process.

b. Finished Goods.

c. Cost of Goods Sold.

applying overhead using a predetermined rate mary rsquo s landscaping uses a job ord 691047

Applying Overhead Using a Predetermined Rate

Mary’s Landscaping uses a job order cost system. The following debits (credits) appeared in Workin Process Inventory for August:

 

Description

Amount

August 1

Balance

$ 6,250

For the month

Direct materials

40,500

For the month

Direct labor

27,000

For the month

Factory overhead

21,600

For the month

To finished goods

(81,000)

Mary’s applies overhead to production at a predetermined rate of 80 percent based on direct labor cost. Job 3318, which was started during August and is the only job still in process at the end of August, has been charged direct labor of $3,375.

Required

What cost amount of direct materials was charged to Job 3318?

applying overhead using a predetermined rate turco products uses a job order cost sy 691048

Applying Overhead Using a Predetermined Rate

Turco Products uses a job order cost system. The following debits (credits) appeared in Work in Process Inventory for September:

 

Description

Amount

September 1

Balance

$ 70,200

For the month

Direct materials

421,200

For the month

Direct labor

262,600

For the month

Factory overhead

315,120

For the month

To finished goods

(832,000)

Turco applies overhead to production at a predetermined rate of 120 percent based on direct labor cost. Job 9 27, the only job still in process at the end of September, has been charged direct labor of $35,100.

Required

What cost amount of direct materials was charged to Job 9 27?

job costing in a service organization at the beginning of the month arthur rsquo s o 691050

Job Costing in a Service Organization

At the beginning of the month, Arthur’s Olde Consulting Corporation had two jobs in process that had the following costs assigned from previous months:

Job Number

Direct Labor

Applied Overhead

SY 400

$23,040

?

SY 403

15,120

?

During the month, Jobs SY 400 and SY 403 were completed but not billed to customers. The completion costs for SY 400 required $25,200 in direct labor. For SY 403, $72,000 in labor was used.

During the month, the only new job, SY 404, was started but not finished. Total direct labor costs for all jobs amounted to $148,320 for the month. Overhead in this company refers to the cost of work that is not directly traced to particular jobs, including copying, printing, and travel costs to meet with clients. Overhead is applied at a rate of 60 percent of direct labor costs for this and previous periods. Actual overhead for the month was $90,000.

Required

a. What are the costs of Jobs SY 400 and SY 403 at (1) the beginning of the month and (2) when completed?

b. What is the cost of Job SY 404 at the end of the month?

c. How much was under or overapplied service overhead for the month?

assigning costs mdash missing data the following t accounts represent september acti 691055

Assigning Costs—Missing Data

The following T accounts represent September activity.

Materials Inventory

BB (9/1)

8,000

 

 

(a)

4,300

EB (9/30)

9,700

(b)

Finished Goods Inventory

BB (9/1)

14,200

 

 

(e)

(f)

EB (9/30)

(g)

 

Applied Overhead Control

 

 

(d)

Wages Payable

 

 

124,300

 

162,000

(c)

 

 

36,200

 

 

119,500

EB (9/30)

Accounts Payable—Material Suppliers

 

 

100,000

Prepaid Expenses

BB (9/1)

24,300

(i)

EB (9/30)

21,100

 

Work In Process Inventory

BB (9/1)

22,300

 

 

180,500

 

 

121,000

 

 

94,000

 

EB (9/30)

17,700

 

Cost of Goods Sold

 

402,800

 

Manufacturing Overhead Control

121,000

 

 

4,300

 

 

36,200

 

 

31,600

 

 

3,200

 

 

Accumulated Depreciation— Plant & Equipment

 

 

204,100

BB (9/1)

 

 

 

(h)

 

 

 

235,700

EB (9/30)

 

Required

Compute the missing amounts indicated by the letters (a) through (i).

high low method scattergraph cubicle solutions sells productivity software such as w 690991

High Low Method, Scattergraph

Cubicle Solutions sells productivity software such as word processors, spreadsheets, and personal information managers. Cubicle prides itself on customer support and maintains a large call center where customers can call in with technical questions about the installation and use of Cubicle products. Monthly data on the number of support calls and call center costs for the last year have been collected and are shown below (all activities and costs are in thousands):

Month

Support Calls

Call Center Cost

1

37

$528

2

53

690

3

45

590

4

39

535

5

61

720

6

47

610

7

44

590

8

48

620

9

55

700

10

40

545

11

42

565

12

41

550

Required

a. Estimate the monthly fixed costs and the unit variable cost per support call using the high low estimation method.

b. Draw a scattergraph relating call center costs to the number of support calls.

c. Considering your scattergraph, how much confi dence do you have in your estimate from requirement (a)?

high low method scattergraph academy products manufactures a variety of custom compo 690992

High Low Method, Scattergraph

Academy Products manufactures a variety of custom components for use in aircraft navigation and communications systems. The controller has asked for your help in estimating fixed and variable overhead costs for Academy’s Rio Puerco plant. The controller tells you that the best cost driver for estimating overhead is machine hours. Monthly data on machine hours and overhead costs for the last year have been collected and are shown below:

Month

Machine Hours

Overhead Costs

1

630,000

$ 660,000

2

900,000

2,170,000

3

765,000

1,220,000

4

665,000

780,000

5

1,035,000

3,700,000

6

800,000

1,400,000

7

750,000

1,100,000

8

815,000

1,500,000

9

935,000

2,500,000

10

680,000

840,000

11

715,000

980,000

12

700,000

910,000

Required

a. Estimate the monthly fixed costs and the unit variable cost per machine hour using the highlow estimation method.

b. Draw a scattergraph relating overhead costs to the number of machine hours.

c. Considering your scattergraph, how much confi dence do you have in your estimate from requirement (a)?

interpretation of regression results simple regression using a spreadsheet hartman c 690993

Interpretation of Regression Results: Simple Regression Using a Spreadsheet

Hartman Company’s Lucas plant manufactures thermostatic controls. Plant management has experienced fluctuating monthly overhead costs and wants to estimate overhead costs accurately to plan its operations and its financial needs. Interviews with plant personnel and studies reported in trade publications suggest that overhead in this industry tends to vary with labor hours.

A member of the controller’s staff proposed that the behavior pattern of these overhead costs be determined to improve cost estimation. Another staff member suggested that a good starting place for determining cost behavior patterns is to analyze historical data. Following this suggestion, monthly data were gathered on labor hours and overhead costs for the past two years. No major changes in operations occurred over this period of time. The data are shown in the following table:

Month

Labor Hours

Overhead Costs

1

251,563

$2,741,204

2

238,438

2,166,231

3

192,500

1,902,236

4

271,250

2,590,765

5

323,750

3,071,812

6

290,938

2,618,161

7

271,250

2,480,231

8

251,563

2,745,558

9

231,875

2,211,799

10

343,438

3,437,704

11

185,938

2,314,436

12

231,875

2,550,630

13

382,813

3,603,709

14

376,250

3,404,786

15

290,938

3,016,493

16

395,938

3,638,331

17

356,563

3,553,886

18

323,750

3,191,617

19

389,375

3,481,714

20

317,188

3,219,519

21

343,438

3,495,424

22

336,875

3,207,258

23

382,813

3,600,622

24

376,250

3,736,658

Required

a. Use the high low estimation method to estimate the overhead cost behavior (fi xed and variable portions components of cost) for the Lucas plant.

b. Prepare a scattergraph showing the overhead costs plotted against the labor hours.

c. Use a spreadsheet program to compute regression coefficients to describe the overhead cost equation.

d. Use the results of your regression analysis to develop an estimate of overhead costs assuming  350,000 labor hours will be worked next month.

interpretation of regression results simple regression regression problems your comp 690994

Interpretation of Regression Results: Simple Regression, Regression Problems

Your company is preparing an estimate of its production costs for the coming period. The  controller estimates that direct materials costs are $45 per unit and that direct labor costs are $21 per hour. Estimating overhead, which is applied on the basis of direct labor costs, is difficult.

The controller’s office estimated overhead costs at $3,600 for fixed costs and $18 per unit for variable costs. Your colleague, Lance, who graduated from a rival school, has already done the analysis and reports the “correct” cost equation as follows:

Overhead = $10,600 + $16.05 per unit

Lance also reports that the correlation coefficient for the regression is .82 and says, “With 82 percent of the variation in overhead explained by the equation, it certainly should be adopted as the best basis for estimating costs.”

When asked for the data used to generate the regression, Lance produces the following:

Month

Overhead

Unit Production

1

$57,144

3,048

2

60,756

3,248

3

77,040

4,176

4

56,412

3,000

5

81,396

3,408

6

72,252

3,928

7

63,852

3,336

8

73,596

4,016

9

77,772

4,120

10

60,048

3,192

11

61,632

3,368

12

73,920

4,080

13

73,248

3,888

The company controller is somewhat surprised that the cost estimates are so different. You have therefore been assigned to check Lance’s equation. You accept the assignment with glee.

Required

Analyze Lance’s results and state your reasons for supporting or rejecting his cost equation.

interpretation of regression results multiple choice the business school at eastern 690995

Interpretation of Regression Results: Multiple Choice

The Business School at Eastern College is collecting data as a first step in the preparation of next year’s budget. One cost that is being looked at closely is administrative staff as a function of student credit hours. Data on administrative costs and credit hours for the most recent 13 months follow:

Month

Administrative Costs

Credit Hours

July

$ 543,064

525

August

346,975

242

September

960,036

2,923

October

908,855

2,100

November

1,084,705

2,749

December

774,686

2,335

January

920,375

2,812

February

1,029,000

2,883

March

880,496

2,234

April

806,085

2,358

May

1,049,908

2,856

June

715,756

882

July

538,301

662

Total

$10,558,242

25,561

Average

$ 812,172

1,966

The controller’s office has analyzed the data and given you the results from the regression analysis:

 width=

Required

a. In the standard regression equation y = a + bx , the letter b is best described as the

(1) Independent variable.

(2) Dependent variable.

(3) Constant coefficient.

(4) Correlation coefficient.

(5) Variable cost coefficient.

b. In the standard regression equation y = a + bx , the letter y is best described as the

(1) Independent variable.

(2) Correlation coefficient.

(3) Constant coefficient.

(4) Variable cost coefficient.

(5) Dependent variable.

c. In the standard regression equation y = a + bx , the letter x is best described as the

(1) Independent variable.

(2) Dependent variable.

(3) Constant coefficient.

(4) Variable cost coefficient.

(5) Correlation coefficient.

d. If the controller uses the high low method to estimate costs, the cost equation for administrative costs is (numbers are rounded to the nearest dollar)

(1) Cost = $291,637 + 229 × Credit hours.

(2) Cost = $233,571 + 101 × Credit hours.

(3) Cost = $229.50 × Credit hours.

(4) Cost = $404,874.

(5) Some other equation.

e. Based on the results of the controller’s regression analysis, the estimate of administrative costs in a month with 2,100 credit hours would be

(1) $834,993.

(2) $844,200.

(3) $404,917.

(4) $839,575.

(5) Some other amount.

f. The correlation coefficient (rounded) for the regression equation for administrative costs is

(1) 0.871.

 (2) 0.933.

(3) 0.859.

(4) √0.933.

(5) Some other amount.

g. The percent of the total variance (rounded) that can be explained by the regression is

(1) 85.9.

(2) 87.1.

(3) 93.3.

(4) 96.6.

(5) Some other amount.

interpretation of regression results simple regression your company provides a varie 690996

Interpretation of Regression Results: Simple Regression

Your company provides a variety of delivery services. Management wants to know the volume of a particular delivery that would generate $10,000 per month in operating profits before taxes. The company charges $20 per delivery. 0The controller’s office has estimated overhead costs at $9,000 per month for fixed costs and $12 per delivery for variable costs. You believe that the company should use regression analysis. Your analysis shows the results to be:

Monthly overhead = $26,501 + $10.70 per delivery

Your estimate was based on the following data:

 

 

Number of

Month

Overhead Costs

Deliveries

1

$142,860

11,430

2

151,890

12,180

3

192,600

15,660

4

141,030

11,250

5

203,490

12,780

6

180,630

14,730

7

159,630

12,510

8

183,990

15,060

9

194,430

15,450

10

150,120

11,970

11

154,080

12,630

12

184,800

15,300

13

183,120

14,580

The company controller is somewhat surprised that the cost estimates are so different. You have been asked to recheck your work and see if you can figure out the difference between your results and the controller’s results.

Required

a. Analyze the data and your results and state your reasons for supporting or rejecting your cost equation.

b. Write a report that informs management about the correct volume that will generate $10,000 per month in operating profits before taxes.

interpretation of regression results lerner inc is accumulating data to prepare its 690997

Interpretation of Regression Results

Lerner, Inc., is accumulating data to prepare its annual profit plan for the coming year. The behavior pattern of the maintenance costs must be determined. The accounting staff has suggested using regression to derive an equation in the form of y = a + bx for maintenance costs. Monthly data regarding maintenance hours and costs for the preceding year have been entered into the regression analysis.

Total hours of maintenance for the year

4,800 $

Total costs for the year

43,200

Regression results:

 

Intercept

$684.65 

b coefficient

$ 7.29

R2

0.79724

Required

a. In a regression equation expressed as y = a + bx , how is the letter b best described?

b. How is the letter y in the regression equation best described?

c. How is the letter x in the regression equation best described?

d. Based on the data derived from the regression analysis, what are the estimated costs for 360 maintenance hours in a month? 

e. What is the percent of the total variance that can be explained by the regression equation?

learning curves appendix b krylon company purchases eight special tools annually fro 691000

Learning Curves (Appendix B)

Krylon Company purchases eight special tools annually from CO., Inc. The price of these tools has increased each year, reaching $100,000 per unit last year. Because the purchase price has increased significantly, Krylon management has asked for a cost estimate to produce the tools in its own facilities.

A team of employees from the engineering, manufacturing, and accounting departments has prepared a report for management that includes the following estimate to produce the first unit. Additional production employees will be hired to manufacture the tools. However, no additional equipment or space will be needed.

The report states that total incremental costs for the first unit are estimated to be $120,000, as shown here.

Materials

$40,000

Direct labor, consisting entirely of hourly production workers (varies with production volume)

80,000

Overhead and administrative costs are not affected by producing this tool.

The current purchase price is $100,000 per unit, so the report recommends that Krylon continue to purchase the product from CO., Inc.

Required

Assume that Krylon could experience labor cost improvements on the tool production consistent with an 80 percent learning curve. Should Krylon produce or purchase its annual requirement of eight tools? Explain your answer. (Note that the 80 percent learning rate coefficient is −0.3219.)